Download as pdf or txt
Download as pdf or txt
You are on page 1of 942

Solutions to Irodov's Problems

in General Physics
CONTENTS

Preface vii

Part One: Physical Fundamentals of Mechanics 1


1.1 Kinematics 1
1.2 The Fundamental Equation of Dynamics 39
1.3 Law of Conservation of Energy, Momentum and Angular Momentum 76
1.4 Universal Gravitation 123
1.5 Dynamics of a Solid Body 140
1.6 Elastic Deformations of a Solid Body 170
1.7 Hydrodynamics 187
1.8 Relativistic Mechanics 202

Part Two: Thermodynamics and Molecular Physics 223


2.1 Equation of the Gas State. Processes 223
2.2 The First Law of Thermodynamics. Heat Capacity 236
2.3 Kinetic Theory of Gases. Boltzmann’s Law and Maxwell’s Distribution 257
2.4 The Second Law of Thermodynamics. Entropy 273
2.5 Liquids. Capillary Effects 289
2.6 Phase Transformations 297
2.7 Transport Phenomena 308

Part Three: Electrodynamics 321


3.1 Constant Electric Field in Vacuum 321
3.2 Conductors and Dielectrics in an Electric Field 349
3.3 Electric Capacitance. Energy of an Electric Field 369
3.4 Electric Current 392
3.5 Constant Magnetic Field. Magnetics 425
3.6 Electromagnetic Induction. Maxwell’s Equations 454
3.7 Motion of Charged Particles in Electric and Magnetic Fields 486
PHYSICAL FUNDAMENTALS
OF MECHANICS
PART 1
1.1 Kinematics

1.1 Let v0 be the stream velocity and v ¿ the velocity of motorboat with respect to water.
The motorboat reached point B while going downstream with velocity (v0 + v ¿) and
then returned with velocity (v ¿ – v0 ) and passed the raft at point C. Let t be the time
for the raft (which flows with stream with velocity v0 ) to move from point A to C, dur-
ing which the motorboat moves from A to B and then from B to C.
Therefore,
(v0 + v ¿)t - l (v0 + v ') τ
l
t = = t + A
v0
v0 (v ¿ – v0) B
l C
On solving, we get
l
v0 = = 3 km/h (on substituting values)
2t
Alternate:
In the frame of reference moving with the stream velocity the raft becomes stationary.
The boat travels for time t and turns and then travels back to reach the raft. As the
raft is still at its original position, hence it would take exactly further time t (the same
time interval) for the boat to reach the raft. The boat thus takes time 2t to reach the
raft in the frame of stream. In Newtonian mechanics time interval between two events
is frame independent, so 2t is also the time interval in the frame of river bank. In the
mean time the raft has travelled l. Hence the speed v0 of the river is
l
v0 = = 3 km/h
2t

1.2 Let s be the total distance traversed by the point and t1 the time taken to cover half the
distance. Further let 2t be the time to cover the rest half of the distance.
s s
Therefore, = v0t1 or t1 =
2 2v0
s s
and = (v1 + v2)t or 2t =
2 v1 + v2
2 PART ONE PHYSICAL FUNDAMENTALS OF MECHANICS

Hence the sought average velocity is given by

s s 2v0(v1 + v2 )
6v 7 = = =
t1 + 2t [s /2v0] + [s / [(v1 + v2 )] v1 + v2 + 2v0

1.3 As the car starts from rest and finally comes to a stop, and the rate of acceleration and
deceleration are equal, the distances covered as well as the times taken are same in
these phases of motion.
Let ¢t be the time for which the car moves uniformly. Then the acceleration/deceleration
time is (t - ¢t )/2 each. So,

1 (t - ¢t)2 (t - ¢t)
6v 7 t = 2e w f + w ¢t
2 4 2

4 6v7 t
or ¢t 2 = t2 -
w

46 v 7
Hence, ¢t = t 1 - = 15 s
A wt

1.4 (a) Sought average velocity s (m)


2 c
s 200 cm
6v7 = = = 10 cm/s
t 20 s
b
(b) For the maximum velocity, ds>dt should be 1
maximum. From the figure ds>dt is maximum
for all points on the line ab thus the sought max-
a
imum velocity becomes average velocity for the
line ab and is o
10 14 20 t (s)
100 cm
= = 25 cm/s
4s
(c) Time t0 should be such that corresponding to it the slope ds>dt should coincide
with the chord Oc, to satisfy the relationship ds>dt = s>t0. From figure the tangent
at point c passes through the origin and thus corresponding time t = t0 = 16 s.

1.5 Let the particles collide at the point A (see figure), whose position y A
v1 t
vector is r3 (say). If t be the time taken by each particle to reach
point A, then from triangle law of vector addition: r1
r3 v2 t
r3 = r1 + v1t = r2 + v2t
r2
So, r1 - r2 = (v2 - v1)t (1)
x
O
1.1 KINEMATICS 3

| r1 - r2 |
Therefore, t = (2)
| v2 - v1 |
From Eqs. (1) and (2)
| r1 - r2 |
r1 - r2 = (v2 - v1) =
| v2 - v1|
r1 - r2 v2 - v1
Thus, = , which is the sought relationship.
| r1 - r2 | | v2 - v1 |

1.6 We have
v ¿ = v - v0
In the vector diagram for above relation, let us drop a dotted perpendicular from the
tip of v onto the line of action of v0. Using the property of right angle triangle,
v ¿ = 2(v cos w + v0) 2 + (v sin w) 2 = 2v 20 + v 2 + 2v0 v cos w

= 39.7 km/h (on substituting values) v'


v sin w v
and tan w¿ =
v cos w + v0 ϕ ϕ′
On substitution w¿ = 19.1° v0
1.7 Let us suppose one of the swimmers (say 1) crosses the river along AB, which is
obviously the shortest path. Time taken to cross the river by the swimmer 1
d
t1 = (where AB = d is the width of the river) (1)
2v ¿ 2 - v 20
For the other swimmer (say 2), who follows the quickest path, the time taken to cross
the river,
d
t2 = (2)
v¿

B B x C

v0 v0
d d
v1 v' v2
v'

A A

In the time t2, drifting of the swimmer 2, becomes


v0
x = v0t2 = d (using Eq. 2) (3)
v¿
4 PART ONE PHYSICAL FUNDAMENTALS OF MECHANICS

If t3 be the time for swimmer 2 to walk the distance x to come C to B (see figure), then
x v0d
t3 = = (using Eq. 3) (4)
u v ¿u
According to the problem t1 = t2 + t3

d d v0d
or = +
2v ¿ 2 - v 20 v¿ v ¿u
v0 v¿
Q = - 1
u v ¿ 2 - v 20
v0
Thus, u = = 3 km/h
(1 - v 20 /v¿ 2)-1/ 2 - 1

1.8 Let l be the distance covered by the boat A along the river as well as by the boat B
across the river. Let v0 be the stream velocity and v  the velocity of each boat with
respect to water. Therefore time taken by the boat A in its journey
l l 2lv ¿
tA = + = 2
v ¿ + v0 v ¿ - v0 v ¿ - v 20
and for the boat B
l l 2l
tB = + =
2v ¿ 2 - v 20 2v ¿ 2 - v 20 2v ¿ 2 - v 20
tA v¿ v¿
awhere h = b
h
Hence, = =
tB 2v ¿ 2 - v 20 2h2 - 1 v0

tA
On substitution, = 1.8
tB

1.9 Let v0 be the stream velocity and v  the velocity of boat with respect to water. As
v0>v ¿ = n = 2 7 0, some drifting of the boat is inevitable.
Let v¿ make an angle u with flow direction (see figure), then the time taken to cross the
river
d
t = (where d is the width of the river)
v ¿sin u
In this time interval, the drifting of the boat

x = 1v ¿cos u + v02t

= 1v ¿cos u + v02 = 1cot u + h cosec u2d


d
v ¿sin u
1.1 KINEMATICS 5

For xmin (minimum drifting) A x B


y
v0
1cot u + h cosec u2 = 0, which yields
d
du d v' v
1 1
cos u = - = -
n 2 q
Hence, u = 120° O x

Alternate:
Let v0 be the stream velocity, v ¿ the velocity of boat with respect to water and v be
the resultant velocity of boat, i.e., v = v0 + v¿.
The angle u from the direction of stream at which
boat can be rowed lies in the interval 0 to y
p (0 … u … p). Now let us draw the vector dia- D
gram of velocity vectors. In the figure, a semicircle C
has been drawn whose radius is v ¿ . The tip of vec- B
tor v¿ lies on this semicircle. One can observe that v'
v a
for minimum drifting, angle a should be maximum A a q
x
and the line AB representing vector v becomes tan- v0 O
gent to the semicircle so that line OB representing
v¿ becomes perpendicular to it. The minimum drifting is CD.
OB v¿ 1
From the figure sin a = = = (1)
AO v0 n
p
u = + a (see figure)
2
Hence, u = sin-1 (1>n) + p>2 = 120° (on substituting values)
In the triangle OCD
OC
sin a = (2)
OD
From Eqs. (1) and (2)
OC 1
=
OD n

or OD = n (OC)
So, OC = 2(OD)2 - (OC)2 = OC 1n 2 - 1

Hence, xmin = d 2n 2 - 1
6 PART ONE PHYSICAL FUNDAMENTALS OF MECHANICS

1.10 Let us suppose one of the bodies, say 1, is thrown upwards and the other body
(say 2) at an angle u to the horizontal. Indicate y-axis vertically upward and x-axis
along horizontal. Let after the time interval t, the particles 1 and 2 have coordinates
(0, y1) and (x2, y2)
For body 1,
1 2 y
y1 = v0t - gt
2 (0, y1) l
and for body 2, (x2, y2)
v0 v0
1
x2 = v0 cos u t and y2 = v0 sin u t - gt 2
2 x
Thus the sought separation between the particles 1 and 2 after the time interval t

l = 2x 22 + ( y1 - y2 )2 = 2(v0 cos ut)2 + {v0t (1 - sin u)2}


= v0 t 22(1 - sin u)
= 22 m, on putting the values of v0, t and u

Alternate:
The solution of this problem becomes interesting in the frame attached v0(12)
with one of the bodies. Let the body thrown straight up be 1 and the
other body be 2, then for the body 1 in the frame of 2 from the kine- v01
matical equation for constant acceleration (since both are moving v02
under constant acceleration) is
0
1
r12 = r0(12) + v0(12) t + w12t 2
2
So, r12 = v0(12) t (because w12 = 0 and r0(12) = 0)
or | r12 | = | v0(12) | t
But, | v01 | = | v02 | = v0
So, from properties of triangle
| v0(12)| = 2v 20 + v 20 - 2v0v0 cos (p>2 - u0 )
Hence, the sought distance

| r12 | = v0t 22 (1 - sin u0 ) = 22 m

1.11 Let the velocities of the particles (say v¿1 and v¿2 ) become mutually perpendicular after
time t. Then their velocities become
v¿1 = v1 + gt ; v¿2 = v2 + g t
1.1 KINEMATICS 7

As v¿1 ⬜ v¿2 so, v¿1 # v¿2 = 0


or (v1 + gt) # (v2 + gt) = 0
or - v1v2 + g 2t 2 = 0
v2 v1
2v1v2
Hence, t =
g
v2′ v1′
In the frame attached with 2 for the particle 1
v2 t v1 t
1
r = r0 + v0t + wt2
2
Both the particles are initially at the same position and have same acceleration g, so
r0 = 0, w = 0, and v0 = | v1 - v2 |. Thus the sought distance is
| r | = | v0 |t = (v1 + v2)t
v1 + v2
= 1v1v2 (using value of t )
g
= 2.5 m, on putting the values of v1, v2 and g

1.12 From the symmetry of the problem all the three points are
always located at the vertices of equilateral triangles of varying
side length and finally meet at the centroid of the initial
equilateral triangle. Symmetry of the problem tells us that the ve- a 0 a
locity with which point 1 approaches 2 is equal to the velocity
by which 2 approaches 3 and 3 approaches 1.
a
Method 1:
The rate at which 1 approaches 2 or the separation between 1 and 2 decreases with
time is the projection of v12 towards r21. At an arbitrary instant when the equilateral
triangle has edge length l  a, the velocity with which 1 approaches 2 becomes
- dl
= v - v cos a b
2p
dt 3 1
v
0 t
3v
On integrating: - dl = dt (where t is the sought time) l
3 2 3 v v
a 0 2
2p/3 3
3 2a
or a = vt so, t =
2 3v
Method 2:
If we concentrate on the motion of any one point say 3, one can observe that at all
the time its velocity makes an angle 30 with 3O and is equal to v cos 30° = 13/2 v.
8 PART ONE PHYSICAL FUNDAMENTALS OF MECHANICS

Initially 3O equals a> 13, so the sought time 1


v
a> 13
2a
t = =
( 13>2)v 3v
(In the figure, velocity of point 3 has been resolved into a O
two rectangular components, one pointing towards the 3 v/2
centre of the triangle and other perpendicular to it.) v 2v
2 3
1.13 Let us locate the points A and B at an arbitrary instant of
time (see figure). B
u
If A and B are separated by the distance s at this moment,
then the points converge or point A approaches B with v
velocity - ds>dt = v - u cos a where angle a varies with l
time. On integrating, A

0 T

- ds = (v - u cos a) dt (where T is the sought time)


3 3
l 0
T

or l = (v - u cos a) dt
3
0
T

= vT - u cos a dt (1)
3
0
As both A and B cover the same distance in x-direction during the sought time inter-
val, so the other condition which is required, can be obtained by the equation,
T

uT = v cos a dt (2)
3
0
Solving Eqs. (1) and (2), we get
ul
T =
- u2 v2
One can see that if u = v, or u  v, point A cannot catch B.

1.14 In the reference frame fixed to the train, the distance between the two events is
obviously equal to l. Suppose the train starts moving at time t = 0 in the positive
direction and take the origin (x = 0) at the head-light of the train at t = 0. Then the
coordinate of first event in the earth’s frame is
1
x1 = wt 2
2
1.1 KINEMATICS 9

and similarly the coordinate of the second event is

1
x2 = w (t + t)2 - l
2
The distance between the two events is obviously

x1 - x2 = l - w t (t + t>2) = 0.242 km

in the reference frame fixed on the Earth.


For the two events to occur at the same point in the reference frame K, moving with
constant velocity V relative to the earth, the distance traveled by the frame in the time
interval t must be equal to the above distance.

Thus, Vt = l - w t (t + t>2)

l
So, V = - w (t + t>2) = 4.03 m/s
t
The frame K must clearly be moving in a direction opposite to the train so that if (for
example) the origin of the frame coincides with the point x1 on the earth at time t, it
coincides with the point x2 at time t + t.

1.15 (a) As time interval between two events is reference independent in Newtonian
mechanics, so it is better to solve the problem in the elevators frame having the
observer at its floor. (see figure). Let us denote separation between floor and ceil-
ing by h = 2.7 m and the acceleration of the elevator by w = 1.2 m/s2 .
From the kinematical formula y
1
y = y0 + v0y t + w t2
2 y
h w
Here, y = 0, y0 = + h, v0y = 0
O
and wy = wbolt(y) - wele(y)

= (–g) - (w) = - ( g + w)

1
So, 0 = h + [ - ( g + w)]t 2
2
2h
or t = = 0.7 s (on substituting values)
Ag + w
10 PART ONE PHYSICAL FUNDAMENTALS OF MECHANICS

(b) At the moment the bolt loses contact with the elevator, it has already acquired the
velocity equal to elevator, given by;
v0 = (1.2) (2) = 2.4 m/s
In the reference frame attached with the elevator shaft (ground) and pointing the
y-axis upward, we have for the displacement of the bolt,
1
¢y = v0y t + w t2
2 y
1 v0
= v0t + ( - g)t 2
2 i
1
or ¢y = (2.4)(0.7) + ( - 9.8)(0.7)2 y
2
= - 0.7 m f

Hence the bolt comes down or displaces downward relative to the point, when it
loses contact with the elevator by the amount 0.7 m (see figure).
Obviously the total distance covered by the bolt during its free fall time
v 20
s = | ¢y | + 2 a b = 0.7 m +
(2.4)2
m = 1.3 m
2g (9.8)
1.16 After time t 7 0, the separation between the particles
l = 2(l1 - v1t)2 + (l2 - v2t)2
For l and hence l 2 to be minimum
dl 2
= 0
dt
d
or {(l + v1t)2 + (l2 - v2t)2} = 0
dt 1
(v1l 1 - v2l2)
which gives t =
2v 21 + v 22
On using the value of t, in the expression of l
| v1l2 - v2l1 |
lmin = .
1v 21 + v 22
Alternate:
Let the particles 1 and 2 be at points B and A respectively at t  0, at the distances l1
and l2 from intersection point O.
Let us fix the inertial frame with the particle 2. Now the particle 1 moves relative to this
reference frame with a relative velocity v12 = v1 - v2, and its trajectory is the straight
line BP. Obviously, the minimum distance between the particles is equal to the length
of the perpendicular AP dropped from point A on to the straight line BP (Fig. (a)).
1.1 KINEMATICS 11

v2

q P
l2 M
q v1

v12
q v12
v1 q
B O v2
l1
(a) (b)
v1
From Fig. (b), v12 = 2v 21 + v 22, tan u = ,
v2
v2 v1
So, cos u = and sin u = (1)
2v 21 + v 22 2v 21 + v 22
From Fig. (a), the shortest distance

AP = AM sin u = (OA - OM ) sin u = (l2 - l1 cot u) sin u

v2 v1 | v1l2 - v2l1 |
or AP = a l2 - l1 b = (using Eq. 1)
v1 2v 21 + v 22 2v 21 + v 22
BP BM + MP l1 cosec u + (l2 - l1 cot u) cos u
The sought time t = = =
| v12 | | v12 | 2v 21 + v 22
l2 cos u + l1 sin u l2v2 + l1v1
= = (using Eq. 1)
2v 21 + v 22 v 21 + v 22

1.17 Let the car turns off the highway at a distance x from the point D. x
C D
So, CD  x, and if the speed of the car in the field is v, then the A
time taken by the car to cover the distance AC  (AD  x) on the
highway l
l2 + x 2
AD - x
t1 = (1) B
hv
and the time taken to travel the distance CB in the field
2l 2 + x 2
t2 = (2)
v
So, the total time elapsed to move the car from point A to B
AD - x 2l 2 + x 2
t = t1 + t2 = +
hv v
12 PART ONE PHYSICAL FUNDAMENTALS OF MECHANICS

For t to be minimum

c- + d = 0
dt 1 1 x
= 0 or
dx v h 2l + x 2
2

l
or h 2x 2 = l 2 + x 2 or x =
2h 2 - 1
1.18 From question vx (0) = 0, so the motion starts from vx
origin, thus from the plot of vx(t) we have 1 a b
c e
t; 0 … t … 1 0
1 2 3 4 5 6 7
1; 1 6 t … 3 –1
vx =
L4 - t; 3 6 t … 6 –2
d t
2t - 14 ; 6 6 t … 7

and vx = 0 for t 7 7.

As at t = 0, x = 0,
x(t)
t2
;0 … t … 1
2 3
t 1
t - ;1 6 t … 3 2
f 2
So, x (t) = vx dt = 1
3 t2
0 4t - + 5; 3 6 t … 6 t
2
0 1 2 3 4 5 6 7
t 2 - 14t + 49 ; 6 6 t … 7

s(t)
t2
;0 … t … 1 6
2
5
1
t - ;1 6 t … 3 4
t 2 7/2
h t2 3
Now, s (t) = | vx |dt = 4t - - 5; 3 6 t … 4 5/2
3 2 2
0 2
t
- 4t + 11; 4 6 t … 6 1
2 1/2
t
14t - t 2 - 43 ; 6 6 t … 7 0 1 2 3 4 5 6 7
ax
1; 0 … t 6 1
dvx 2
0; 1 6 t … 3
Now, ax = =
dt -
L 1; 3 6 t … 6 1
2; 6 6 t … 7
t
1 2 3 4 5 6 7
1.1 KINEMATICS 13

1.19 (a) Mean velocity


Total distance covered v0 R
<v > = f
Time elapsed i v
s pR
== = 50 cm/s (1)
t t
| ¢r | 2R
(b) Modulus of mean velocity vector | <v > |= = = 32 cm/s (2)
¢t t
(c) Let the point moves from i to f along the half circle (see figure) and v0 and v be
the speed at the points respectively.
dv
We have = wt
dt
or v = v0 + wt t (as wt is constant, according to the problem)
t

(v0 + wt t)dt
3 v0 + (v0 + wt t) v0 + v
0
So, <v > = t
= = (3)
2 2
dt
3
0

So, from Eqs. (1) and (3)


v0 + vpR
= (4)
2 t
Now, the modulus of the mean vector of total acceleration
| ¢v | | v - v0 | v0 + v
|<w>| = = = (see figure) (5)
¢t t t
Using Eq. (4) in Eq. (5), we get
2pR
|<w>| = = 10 cm/s2 (on substituting values)
t2

1.20 (a) We have r = at (1 - at)


dr
So, v = = a (1 - 2at)
dt
dv
and w = = - 2aa
dt
(b) From the equation
r = at (1 - at)
1
r = 0, at t = 0 and also at t = ¢t =
a
14 PART ONE PHYSICAL FUNDAMENTALS OF MECHANICS

1
So, the sought time ¢t =
a

As v = a (1 - 2at)

1
a (1 - 2at) for t …
2a
So, v = |v| = d
1
a (2at - 1) for t 7
2a
Hence, the sought distance
1>2a 1>a

s = v dt = a (1 - 2a t) dt + a (2at - 1) dt
3 3 3
0 1>2a

a
Simplifying, we get, s =
2a

1.21 (a) As the particle leaves the origin at t = 0

So, ¢x = x = vx dt (1)
3

v = v0 a 1 - b
t
As
t
(where v0 is directed towards the + ve x-axis).

vx = v0 a 1 - b
t
So, (2)
t
From Eqs. (1) and (2),
t

v0 a1 - b dt = v0t a 1 - b
t t
x = (3)
3 t 2t
0
Hence x coordinate of the particle at t = 6 s

x = 10 * 6 a 1 - b = 24 cm = 0.24 m
6
2 * 5
Similarly at t = 10 s
x = 10 * 10 a 1 - b = 0
10
2 * 5

and at t = 20 s

x = 10 * 20 a 1 - b = - 200 cm = - 2 m
20
2 * 5
1.1 KINEMATICS 15

(b) At the moment the particle is at a distance of 10 cm from the origin, x = 10 cm.
Putting x = + 10 in Eq. (3)

10 = 10t a 1 - b
t
or t 2 - 10t + 10 = 0
10
10  2100 - 40
So, t = = 5 215 s
2
Now putting x = - 10 in Eq. (3)

- 10 = 10 a1 - b
t
10

On solving, t = 5  235 s
As t cannot be negative, so,
t = (5 + 235) s
Hence the particle is at a distance of 10 cm from the origin at three moments of time:
t = 5  215 s, 5 + 235 s

v = v0 a 1 - b
t
(c) We have
t

v0 a 1 - b
t
for t … t
t
v = |v| =
t
So,
v0 a - 1b
t
for t 7 t
t
t

v0 a 1 - b dt for t … t = v0t (1 - t/2t)


t
So, s = (4)
3 t
0
t t

v0 a 1 - b dt + v0 a - 1 b dt for t 7 t
t t
and s =
3 t 3 t
0 t

= v0t[1 + (1 - t>t)2]>2 for t 7 t (5)


4 4

v0 a 1 - b dt = 10 a 1 - b dt = 24 cm
t t
s =
3 t 3 5
0 0

For t = 8 s
5 8

10 a1 - b dt + 10 a - 1 b dt
t t
s =
3 5 3 5
0 5
16 PART ONE PHYSICAL FUNDAMENTALS OF MECHANICS

On integrating and simplifying, we get


s = 34 cm
On the basis of Eqs. (3) and (4), x (t) and s (t) plots can be drawn as shown in the
answer sheet.
1.22 (a) As particle is in unidirectional motion, it is directed along the x-axis all the time. At
t = 0, x = 0 .
dv v dv 1 d 2
So, ¢x = x = s and w = = = (v )
dt ds 2 ds
According to the problem v = a 1x = a 1s

So, v 2 = a 2s
1 d 1 d a2
and w = (v 2) = (a 2s) = (1)
2 ds 2 ds 2
dv a2 a2
As, w = = so, dv = dt
dt 2 2
a2
On integrating, v = t = v (t) (2)
2
(b) Let t be the time to cover first s m of the path. From Eq. (2)
t
a2 a2 t 2
s = v dt = t dt =
3 32 2 2
0
2
Hence, t = 1s (3)
a
The mean velocity of particle
21s >a
a2
t dt
3 2
1 v (t) dt 0 a 1s
<v > = = =
1 dt 2 1s>a 2
1.23 According to the problem
v dv
- = a 1v (as v decreases with time)
ds
0 s

or - 1v dv = a ds
3 3
v0 0

2 3>2
On integrating we get, s = v
3a 0
1.1 KINEMATICS 17

Again according to the problem


dv dv
- = a1v or - = a dt
dt 2v
0 t
dv
or - = a dt
3 1v 3
v0 0

2 1v0
Thus, t =
a
1.24 (a) As r = at i - bt 2 j y
So, x = at, y = - bt 2
–bx 2 O
and therefore y = x
a2
which is equation of a parabola, whose graph is shown
in the figure.
(b) As r = at i - bt 2 j
dr
v = = a i - 2b j (1)
dt
So, v = 2a2 + ( - 2bt )2 = 2a2 + 4b 2t 2

Differentiating Eq. (1) with respect to time, we get


dv
w = = - 2b j
dt
So, | w | = w = 2b
v#w (a i - 2bt j) # ( - 2b j)
(c) cos a = =
vw ( 2a 2 + 4b 2 t 2) 2b
2bt
or cos a =
2a2 + 4b 2t 2
a = tan-1 a b
a a
So, tan a = or
2 bt 2bt

(d) The mean velocity vector


t
1 vdt 1 (a i - 2bt j)dt
0
6v 7 = = = a i - bt j
1 dt t

Hence, | 6v 7 | = 2a2 + ( - bt)2 = 2a 2 + b 2t 2


18 PART ONE PHYSICAL FUNDAMENTALS OF MECHANICS

1.25 (a) We have


x = at and y = at (1 - at) (1)
Hence, y (x) becomes

a1 - b = x - x 2 (parabola)
ax ax a
y =
a a a

(b) Differentiating Eq. (1) we get


vx = a and vy = a (1 - 2 at) (2)

So, v = 2v 2x + v 2y = a 21 + (1 - 2at)2

Differentiating Eq. (2) with respect to time


wx = 0 and wy = - 2a a
So, w = 2w 2x + w 2y = 2a a (3)

(c) From Eqs. (2) and (3) we have v = a i + a (1 - 2at) j and w = 2a a j.


p 1 v#w - a (1 - 2a t0)2aa
So at t - t0, cos = = =
4 22 vw a 21 + (1 - 2 at0)2 2a a

On simplifying, 1 - 2at0 = 1
1
As t0 Z 0, t0 =
a

1.26 Differentiating motion law: x = a sin vt, y = a (1 - cos vt), with respect to time,
vx = a v cos vt and vy = a v sin vt .

So, v = a v cos vt i + a v sin vt j (1)


and v = a v = constant (2)
Differentiating Eq. (1) with respect to time
dv
w = = - a v2 sin vt i + a v2 cos vt j (3)
dt
(a) The distance s traversed by the point during the time t is given by
t t

s = v dt = a v dt = avt (using Eq. 2)


3 3
0 0

(b) Taking scalar product of v and w


We get, v # w = (av cos vt i + a v sin vt j) # (av 2 sin vt ( - i) + a v 2 cos vt j)
1.1 KINEMATICS 19

So, v # w = - a 2v 2 sin vt cos vt + a 2v 3 sin vt cos vt = 0


Thus, v⬜ w, i.e., the angle between velocity vector and acceleration vector equals p/ 2.

1.27 According to the problem


w = w ( - j)
dvx dvy
So, wx = = 0 and wy = = -w (1)
dt dt
Differentiating equation of trajectory, y = ax - bx 2, with respect to time
dy adx dx
= - 2bx (2)
dt dt dt

` `
dy a dx
So, =
dt x = 0 dt x = 0

Again differentiating with respect to time


d 2y
a b
ad 2x dx 2 d 2x
= - 2b - 2b x
dt 2 dt 2 dt dt 2

- w = a (0) - 2b a b - 2bx (0) (using Eq. 1)


dx 2
or
dt
dx w
or = (using Eq. 1) (3)
dt A 2b

`
dy w
Using Eq. (3) in Eq. (2) = a (4)
dt x = 0 A 2b
Hence, the velocity of the particle at the origin

a b + a b
dx 2 dy 2 w w
v = = + a2 (using Eqs. 3 and 4)
A dt x = 0 dt x = 0 A 2b 2b
w
Hence, v = (1 + a 2)
A 2b

1.28 As the body is under gravity of constant acceleration g, its velocity vector and
displacement vectors are:
v = v0 + gt (1)

1 2
and ¢r = r = v0t + gt (as r = 0 at t = 0) (2)
2
20 PART ONE PHYSICAL FUNDAMENTALS OF MECHANICS

So, v  over the first t seconds


¢r r 1
6v 7 = = = v0 + gt (3)
¢t t 2
Hence from Eq. (3), v over the first t seconds
g
6v 7 = v0 + t (4)
2
For evaluating t, take
v # v = (v0 + gt) # (v0 + gt) = v 20 + 2(v0 # g)t + g 2t 2
or v 2 = v 20 + 2(v0 # g) t + g 2t 2
But we have v = v0 at t = 0 and also at t = t (also from energy conservation).
Hence using this property in Eq. (5),
v 20 = v 20 + 2(v0 # g)t + g 2 t2
2(v0 # g)
As t Z 0, so, t = -
g2
Putting this value of t in Eq. (4), the average velocity over the time of flight
(v0 # g)
6 v 7 = v0 - g
g2

1.29 The body thrown in air with velocity v0 at an angle a from the horizontal lands at
point P on the Earth’s surface at same horizontal level (see figure). The point of pro-
jection is taken as origin, so ¢x = x and ¢y = y .
1 v0
(a) From the equation ¢y = v0y t + wyt 2 y
2
1
or 0 = v0 sin at - g t2
2
H
2v0 sin a α P
As t Z 0, so, time of motion t = O x
R/2
g α
(b) At the maximum height of ascent, vy = 0 w=g wn
so, from the equation v 2y = v 20y + 2wy ¢y
0 = (v0 sin a)2 - 2gH
v 20 sin2 a
Hence maximum height H =
2g
During the time of motion the net horizontal displacement or horizontal range,
will be obtained by the equation
1
¢x = v0xt + wxt2
2
1.1 KINEMATICS 21

1 v 02 sin 2 a
or R = v0cos at - (0) t = v0 cos a t =
2
2 g
when R = H
v 02 sin 2 a v 20 sin2 a
= or tan a = 4, so, a = tan-1 4
g 2g
(c) For the body, x (t) and y (t) are
x = v 0 cos at (1)
1 2
and y = v0 sin at -
gt (2)
2
Hence putting the value of t from Eq. (1) into Eq. (2) we get

gx 2
y = v 0 sin a a b - g a b = x tan a -
x 1 x 2

v0 cos a 2 v0 cos a 2 v 20 cos2 a

Note: One can use the formula of curvature radius of a trajectory y (x), to solve part (d),
[1 + (dy/dx)2]3>2
R =
|d 2y /dx 2|

1.30 We have, vx = v0 cos a, vy = v0 sin a - gt

Thus, v 2 = v 2x + v 2y = v 02 - 2gt v0 sin a + g 2t 2

dvt 1 d 1
Now, wt = = (vt )2 = (g2t - gv0 sin a)
dt 2vt dt v
g vy
= - (v 0 sin a - gt) = - g
v vt
| vy |
Hence, | wt | = g
v
v y2
Now, wn = 2w 2 - w t2 = A g 2 - g 2 2
vt

C where vx = 2vt2 - v y2 = 2v 2 - v 2y D
vx
or wn = g
vt
vy
wv = wt = - g
v
On the basis of obtained expressions or facts the sought plots can be drawn as shown
in the figure of answer sheet.
22 PART ONE PHYSICAL FUNDAMENTALS OF MECHANICS

1.31 The ball strikes the inclined plane (Ox) at point O (origin) with velocity
v0 = 22gh (1)
As the ball classically rebounds, it recalls with same velocity v0, at the same angle a
from the normal on y axis (see figure). Let the ball strike the incline second time at P,
which is at a distance l (say) from the point O, along the incline. From the equation
1 y
y = v0y t + w t2
2 y v0
h
1
0 = v0 cos at - g cos at 2
2 90°–
O
where t is the time of motion of ball in air while moving
from O to P.
2v0 g P
As t Z 0, so, t = g (2) α
x
Now from the equation.
1
x = v0x t + w t2
2 x
1
l = v0 sin at + g sin a t2
2
2v0 2v0 4v 02 sin a
l = v0 sin a a b + g sin a a b =
1
So, (using Eq. 2)
g 2 g g
Hence the sought distance,
4(2gh) sin a
l = = 8h sin a (using Eq. 1)
g
1.32 Total time of motion
2v0 sin a tg 9.8 t
t = or sin a = = (1)
g 2v0 2 * 240
and horizontal range
R 5100 85
R = v0 cos at or cos a = = = (2)
v0t 240 t 4t
From Eqs. (1) and (2)
(9.8)2t 2 (85)2
+ = 1
(480)2 (4t2)2

On simplifying t 4 - 2400t2 + 1083750 = 1


Solving for t2, we get
2400  21425000 2400  1194
t2 = =
2 2
1.1 KINEMATICS 23

Thus, t = 42.39 s = 0.71 min


and t = 24.55 s = 0.41 min (depending on the angle a)

1.33 Let the shells collide at the point P (x, y). If the first shell takes t s to collide with sec-
ond and ¢t be the time interval between the firings, then
x = v0 cos u1 t = v0 cos u2 (t – ¢t) (1)
1 y
and y = v0 sin u2 (t - ¢t) - g (t - ¢t)2 (2)
2
v0
¢t cos u2 v0 (x,y)
From Eq. (1) t = (3) P
cos u2 - cos u1 q1
q2
From Eqs. (2) and (3) O x
2v0 sin (u1 - u2)
¢t = as ¢t Z 0
g (cos u2 + cos u1)
= 11 s (on substituting values)

1.34 According to the problem


dy
(a) = v0 or dy = v0 dt
dt
y t

Integrating dy = v0 dt or y = v0t (1)


3 3
0 0

dx
Also, we have = ay or dx = ay dt = av0t dt (using Eq. 1)
dt

x t
1 1 ay 2
So, dx = av0 t dt, or x = av0t 2 = (using Eq. 1)
3 3 2 2 v0
0 0

(b) According to the problem


vy = v0 and vx = ay (2)

So, v = 2v 2x + v 2y = 2v 20 + a2y 2

dv a 2y dy a2y
Therefore, wt = = =
dt 2v 20 + ay 2 dt 21 + (ay/v0)2
24 PART ONE PHYSICAL FUNDAMENTALS OF MECHANICS

Differentiating Eq. (2) with respect to time.


dvy dvx dy
= wy = 0 and = wx = a = av0
dt dt dt

So, w = | wx | = av0

a 4y 2 av0
Hence, wn = 2w 2 - w t2 = a2v 20 - =
A 1 + (ay/v0 )2 21 + (ay/v0 )2

1.35 (a) The velocity vector of the particle


v = a i + bx j

dx dy
So, = a and = bx (1)
dt dt

x t

From Eq. (1) dx = a dt or x = at (2)


3 3
0 0

and dy = bx dt = bat dt
y t
1
Integrating dy = ab t dt or y = ab t 2 (3)
3 3 2
0 0

From Eqs. (2) and (3), we get,


b
y = (x2) (4)
2a

(b) The curvature radius of trajectory y (x) is


[1 + (dy >dx)2]3>2
R =
| d 2y > dx 2 |
(5)

b 2
Let us differentiate the path equation y = x with respect to x,
2a
dy b d 2y b
= x and 2
= (6)
dx a dx a
From Eqs. (5) and (6), the sought curvature radius is

c 1 + a xb d
a b 2 3>2
R =
b a
1.1 KINEMATICS 25

1.36 In accordance with the problem


wt = a # T
vd v
But, wt = or v dv = wt ds
ds
So, v dv = (a # T) ds = a # ds T = a # d r
or v dv = a i # dr = a dx (because a is directed towards the x-axis)
v x

So, v dv = a dx
3 3
0 0

Hence, v2 = 2ax or v = 22ax

1.37 The velocity of the particle v = at .


dv
So, = wt = a (1)
dt
v2 a 2t 2
and wn = = (using v = at ) (2)
R R
From s = v dt
L
t t

2pRh = v dt = at dt
3 3
0 0
4ph t2
So, = (3)
a R
From Eqs. (2) and (3), wn = 4pah

Hence, w = 2w 2t + w n2

= 2a2 + (4pa h )2 = a21 + 16p2h2 = 0.8 m/s2

1.38 (a) According to the problem


| wt | = | wn |
- dv v2
For v (t), =
dt R
Integrating this equation from v0 … v … v and 0 … t … t , we get
v t
dv 1 v0
- = dt or v = (1)
3v
2 R3 v0t
v0 0 a1 + b
R
26 PART ONE PHYSICAL FUNDAMENTALS OF MECHANICS

v dv v2
Now for v (s), - =
ds R

Integrating this equation from v0 … v … v and 0 … s … s , we get


v s
dv 1 v s
= - ds or ln = -
3 v R3 v0 R
v0 0

Hence, v = v0e-s/R (2)


(b) The normal acceleration of the point
v 0e 2 -2s/R
v2
wn = = (using Eq. 2)
R R
In accordance with the problem

| wt | = | wn | and wt t ⬜ wnn
v 20 v2
So, w = 22wn = 22 e -2s/R = 22
R R

1.39 From the equation v = a1s


dv a ds a a2
wt = = = a 1s =
dt 2 1s dt 2 1s 2
v2 a 2s
and wn = =
R R
As wt is a positive constant, the speed of the particle increases with time, and the tan-
gential acceleration vector and velocity vector coincides in direction.
Hence the angle between v and w is equal to angle between wt t and w and a can
be found by the formula
| wn | a2s/R 2s
tan a = = 2 =
| wt | a /2 R

1.40 Differentiating the equation l = a sin v t


dl
= v = av cos vt
dt
dv
So, wt = = - av2 sin vt (1)
dt
v2 a2v2 cos2 vt
and wn = = (2)
R R
1.1 KINEMATICS 27

(a) At the point l = 0, sin vt = 0 and cos vt = 1 so, vt = 0, p, etc.


a2v2
Hence, w = wn = = 2.6 m/s2 (on substituting values)
R
Similarly at l =  a, sin vt = 1 and cos vt = 0, so, wn = 0.

Hence, w = | wt | = av 2 = 3.2 m/s2 (on substituting values)

(b) Using sin vt = l /a in Eqs. (1) and (2) we get


wt = - v2l
v2 2
and wn = (a - l 2)
R
v4 2
Therefore, wt = v4l 2 + (a - l 2)2 = f (l )
A R2
On differentiating with respect to l and putting
dwt
= 0
dl

= a2 a1 – 2 b
R2 R2
we get, l 2 = lm2 = a2 –
2 2a
R2
So, lm = a 1 - = 0.37 m (on substituting values)
A 2a2

The corresponding wmin at l = lm is

v4 a a2 - b + 2a b
R2 v4 R 2 2
wmin =
A 2 R 2
R2 R2
= av2 1 - +
A 2a2 4a 2
R 2
= av2 1 - a b = 2.5 m/s2 (on substituting values)
A 2a
1.41 As wt = a and at t = 0, the point is at rest.
1 2
So, v(t) and s (t) are v = at and s = at (1)
2
Let R be the curvature radius, then
v2 a2t 2 2as
wn == = (using Eq. 1)
R R R
But according to the problem
wn = bt 4
28 PART ONE PHYSICAL FUNDAMENTALS OF MECHANICS

a 2t 2 a2 a2
So, bt 4 = or R = 2 = (using Eq. 1) (2)
R bt 2bs
Therefore, w = 2wt2 + w 2n = 2a2 + (2as /R)2 = 2a2 + (4bs2/a2 )2 (using Eq. 2)

Hence, w = a21 + (4bs2 /a 3)2

1.42 (a) Let us differentiate twice the path equation y (x) with respect to time,
dx d 2y
= 2a c a b + x 2 d
dy dx 2 d 2x
= 2ax ; 2
dt dt dt dt dt
Since the particle moves uniformly, its acceleration at all points of the path is normal
and at the point x = 0 it coincides with the direction of derivative d 2y/dt 2.
Keeping in mind that at the point x = 0, ` ` = v,
dx
dt
d 2y
we get w = ` ` = 2av 2 = wn
dt 2 x = 0

v2 1
So, wn = 2av 2 = or R =
R 2a
Note: We can also calculate it from the formula of problem 1.35(b).
(b) Differentiating the equation of the trajectory with respect to time we see that
dx dy
b2x + a 2y = 0 (1)
dt dt

which implies that the vector (b2xi + a2y j) is normal to the velocity vector
dx dy
v = i + j which, of course, is along the tangent. Thus the former vector is
dt dt
along the normal and the normal component of acceleration is clearly
d 2x d 2y
b2x + 2y
w#n
a
a on using wn = b
dt 2 dt 2
wn =
(b 4x 2 + a 4y 2)1/2 |n|
At x = 0, y =  b, so at x = 0
d 2y
wn =  `
dt 2 x = 0

Differentiating Eq. (1)


d 2y
b2 a b + b 2x a 2 b + a2 a b + a 2y a b = 0
dx 2 d 2x dy 2
dt dt dt dt
1.1 KINEMATICS 29

dy
Also from Eq. (1) = 0 at x = 0
dt

a b = v (since tangential velocity is constant = v)


dx
So,
dt
d 2y
a b = 
b 2
Thus, v
dt 2 a2
bv 2 v2
and | wn | = =
a2 R
a2
This gives R =
b

1.43 Angular velocity of point A, with respect to centre C of the circle or turning rate of
line CA taking the line OCX as reference line becomes
-d u
b = 2v
d (2u)
vC = - = 2a A
dt dt
x
because angular speed of line OA is v = - du/dt. q
The turning rate of line CA is also the turning rate of velocity r 2q
vector of point A, which is given by vA /R . C
q
So, vA = vC R = (2v) R = 0.4 m/s (on substituting values) O

Alternate:
transverse velocity
Angular speed of a point relative to origin =
magnitude of position vector
So, angular speed of point A relative to origin O
component velocity of point A perpendicular to OA
=
Length of line OA
A vA
vA cos u q
v = x
r q R
r 2q
But from sine property of triangle OAC, r = 2R cos u.
vA q C
So, v = or vA = 2vR R
2R O
From both the methods the obtained speed vA is constant, so
the tangential acceleration of particle A is zero and
vA2 (2vR)2
w = wn = R
= = 4v2R = 0.32 m/s2 (on substituting values)
R
30 PART ONE PHYSICAL FUNDAMENTALS OF MECHANICS

1.44 Differentiating w (t) with respect to time


dw
= vz = 2at (1)
dt
For fixed axis rotation, the speed of the point A
v
v = vR = 2 at R or R = (2)
2at
Differentiating with respect to time
dv v
wt = = 2aR = (using Eq. 1)
dt t
v2 v2
But, wn = = = 2atv (using Eq. 2)
R v/2at
v
So, w = 2wt2 + wn2 = 2(v/t)2 + (2atv)2 = 21 + 4a2t 4
t
= 0.7 m/s2 (on substituting values)

1.45 The shell acquires a constant angular acceleration at the same time as it accelerates
linearly.
1 1
l = wt 2 and 2pn = bt 2
2 2
w b
So, =
l 2pn
(where w  linear acceleration and b = angular acceleration).
w
Then, v = 22b 2pn = 2 (2pn)2
A l
But, v 2 = 2wl
2pnv
Hence, v = = 2.0 * 103 rad/s (on substituting values)
l
1.46 Let us take the rotation axis as z-axis whose positive direction is associated with the pos-
itive direction of the co-ordinate w, the rotation angle, in accordance with the right-hand
screw rule (see figure).
(a) Differentiating w(t) with respect to time twice, we get
dw z
= a - 3bt 2 = vz (1)
dt
d 2w dvz
2
= = b z = - 6bt (2)
dt dt
From Eq. (1) the solid body comes to stop at
a ϕ
¢t = t =
A 3b
1.1 KINEMATICS 31

The angular velocity v = a - 3 bt 2, for 0 … t … 2a/3b


2a/3b

vdt (a - 3bt 2) dt
3
3 0
So, 6 v7 = =
dt 2a/3b
3
dt
3
0

= [at - bt 3]02a/3b/ 2a/3b = 2a/3 = 4 rad/s

Similarly, b = | b z | = 6bt for all values of t.

2a/3b

bdt 6 bt dt
3
L
So, 6b7 = = 0
= 23 ab = 6 rad/s2
2a/3b
dt
L
dt
3
0

(b) From Eq. (2), b z = - 6bt

So, (bz )t = 2a/3b = - 6b a = - 2 23ab


A 3b
Hence, b = | (bz )t = 1a/3b | = 2 13ab = 12 rad/s2

1.47 Angle a is related with | wt | and wn by means of the formula


wn
tan a = where wn = v2R and | wt | = bR (1)
| wt |

where R is the radius of the circle which an arbitrary point of the body circumscribes.
From the given equation b = dv/dt = at (here b = dv/dt as b is positive for all val-
ues of t)
v t
1 2
Integrating dv = a tdt or v = at
3 3 2
0 0

wn = v2R = a b R =
at 2 2 a2t 4
So, R
2 4
32 PART ONE PHYSICAL FUNDAMENTALS OF MECHANICS

and | wt | = bR = at R
Putting the values of | wt | and wn in Eq. (1), we get

t = c a b tan a d
a2t 4R/4 at 3 4 1/3
tan a = = or
atR 4 a
t = 7s (on substituting values)

1.48 In accordance with the problem, b x 6 0.


dv
Thus - = k 1v (where k is proportionality constant)
dt
v t
dv kt
or - = k dt or 1v = 1v0 - (1)
3 1v 3 2
v0 0

21v0
When v = 0, total time of rotation t = t =
k
22v0/k

a v0 + - kt2v0 b dt
k 2t 2
v dt 3 4
3
Average angular velocity 6 v 7 = = 0

dt 2 2v0 /k
3

c v0t + - 2v0t 2 d
k 2t 3 k 22v0/k

12 2 0 v0
Hence, 6v7 = =
2 2v0>k 3

dw
1.49 (a) We have v = v0 - a w =
dt

Integrating this Eq. within its limit for (w) t


w t
dw v0 - kw
= dt or ln = - kt
3 v0 - a w 3 v0
0 0
v0
Hence, w = (1 - e-kt ) (1)
k

(b) From the equation, v = v0 - k w and Eq. (1) or by differentiating Eq. (1)
v = v0e-kt
1.1 KINEMATICS 33

1.50 Let us choose the positive direction of z-axis (stationary rotation axis) along the vec-
tor b 0. In accordance with the equation
dvz dvz z
= b z or vz = bz
dt dw z
z
or vzdvz = b zdw = b cos w dw O
/2

Integrating this equation within its limit for vz (w)


vt

or vz dvz = b 0 sin w
3
0

Hence, vz =  22b 0 sin w

The plot vz (w) is shown in the figure. It can be seen that as the angle w grows, the vec-
tor ␻ first increases, coinciding with the direction of the vector ␤ 0(vz 7 0), reaches
the maximum at w = w/2, then starts decreasing and finally turns into zero at w = p.
After that the body starts rotating in the opposite direction in a similar fashion
(vz 6 0). As a result, the body will oscillate about the position w = w/2 with an am-
plitude equal to p/2.

1.51 A rotating disk moves along the x-axis, in plane motion in x -y plane. For the calcula-
tion of velocity only, plane motion of a solid can be imagined to be in pure rotation
about a point (say I) at a certain instant known as instantaneous centre of rotation. The
axis is directed along ␻ of the solid which passes through the point I at that instant
and is known as instantaneous axis of rotation.
Therefore the velocity vector of an arbitrary point (P ) of the solid can be represented as:
vP = ␻ * rPI = v * ␳ PI (1)

(where ␳PI is normal location of point P relative to instantaneous rotation axis pass-
ing through point I ).
So instantaneous rotation axis I is at the perpendicular distance rPI = vP /v from point P.
On the basis of Eq. (1) for the centre of mass (C.M.) of the disk, velocity is
y
vC = ␻ * ␳ CI (2)
According to the problem vC c c i and ␻ c c k so to satis- I
fy the Eq.(2), ␳CI is directed towards ( - j). Hence point I is y rCI
at a distance ␳CI = y, above the centre of the disk along y- O
C v x
axis. Using all these facts in Eq. (2), we get
vC
vC = vy or y = (3)
v
34 PART ONE PHYSICAL FUNDAMENTALS OF MECHANICS

(a) From the angular kinematical equation


vz = v0z + bz t (4)
v = bt
On the other hand
x = vt, t = x/v (where x is the x coordinate of the C.M.) (5)
From Eqs. (4) and (5),
bx
v =
v
Using this value of v in Eq. (3) we get
vC v v2
y = = = (hyperbola)
v bx/v bx
(b) As centre C moves with constant acceleration w, with zero initial velocity
1
So, x = wt 2 and vc = wt
2
2x
Therefore, vC = w = 12xw
Aw
vC 12wx
Hence, y = = (parabola)
v v
1.52 (a) The general plane motion of a solid can be imagined as the combination of trans-
lation with C.M. and rotation about C.M.
So, vA = vC + vAC = vC + ␻ * rAC = vC + ␻ * ␳AC (1)

and wA = wC + wAC = wC + ␻ * (␻ * rAC ) + (␤ * rAC )


= wC + v2 ( - ␳ AC ) + (␤ * ␳ AC ) (2)
r
where ␳ is the component of r normal to the axis of rota- axis of
tion and directed away from it. In this problem rotation
rAC = rAC = R and vC = v .

Let the point A touch the horizontal surface at t = 0, further let us locate the point
A at time t, when it makes an angle u from vertical (see figure).
On the basis of Eqs. (1) and (2) the pictorial diagrams for velocity and accelera-
tion are as follows:
w

wR C C w
v R
θ w 2R θ
v w
A A
1.1 KINEMATICS 35

As the rolling is without slipping along a line, so, vc = vR and wC = bR.


According to the problem vC = v (constant), so v = v/R, wC = 0 and b = 0. Using
these facts, wA = v 2/R =2.0 m/s 2 and the vector w, is directed toward centre C of the
wheel:
vA = 2v 2 + (vR) 2 + 2v (vR) cos(p- u)
= 2v 2 + v 2 + 2v 2 cos (p - u) = v 22(1 - cos u)
= 2v sin (u/2) = 2v sin (vt /2)

Hence, distance covered by the point A during time interval 2p/v


2p/v

s = vA dt = 2 v sin (vt/2)dt
3 3
0
8v
= = 8R = 4.0 m (on substituting values)
v
Note: One can easily find vA, assuming the body to rotate about the instantaneous cen-
tre of rotation of zero velocity (not of zero acceleration), which is the contact point of
the rolling body in this case.

1.53 As the ball rolls without slipping on the rigid surface along a line so, contact point O
has zero velocity, which is possible when the ball rotates in clock wise sense if it’s
centre moves towards right and such that vC = vR, and wC = w = bR. As motion
starts at t = 0 with constant acceleration w so at time t, the velocity of mass centre C
becomes vC = wt and v = vC /R = wt /R.
(a) The contact point O becomes instantaneous centre of rotation, thus, the velocity
of any arbitrary point P (say) of the ball can be obtained by the relation
vP = ␻ * ␳ PO (1)
Using Eq. (1), the pictorial diagram to find the velocities of the points A and B
of the ball is shown below
A w(2R) A wc+bR
w2R

B w2R B
C w
w
2R
ω 2R wR

O R O w

Hence, vA = 2vC = 2wt = 10 cm/s


and vB = 22 vC = 22 wt = 7.1 cm/s
36 PART ONE PHYSICAL FUNDAMENTALS OF MECHANICS

(b) One can write for the any arbitrary point P


wP = wC + wPC = wC + ␻ 2 ( - ␳ PC) + (␤ * ␳ PC ) (2)
Taking into account Eq. (2), one can easily get the acceleration of the points O,
A and B on using the pictorial diagram.
So, w0 = w2t2/R = 2.5 cm/s2

= 2w 1 + a b = 5.6 cm/s2
w 4t 4 wt 2 2
wA = 4w 2 +
C R 2 C 2R

aw - b + w 2 = 2.5 m/s2
w 2t 2 2
and wB =
A R

1.54 In the frame of horizontal plane the direction of velocity at a point is along the tan-
gent of the curve at that point, so normal acceleration at that point is perpendicular
to the direction of velocity vector and directed inwards. We make the pictorial dia-
gram as in solution of problem 1.53.

A 2vC A w+ r
w2R n
C B w2R B
vC Cw w
45° 45°
n 2vC n r

O O

As an arbitrary point of the cylinder follows a curve, its normal acceleration and ra-
dius of curvature are related by the well-known equation
v2
wn =
R
v2A
So, for point A, wA(n ) =
RA
(2vC)2
or v2r =
RA
which gives RA = 4r (on using vC = vr)

vB2
Similarly for point B, wB (n) =
RB
1.1 KINEMATICS 37

(22vC )2
or v2r cos 45° =
RB
v2C
RB = 2 22 = 222r (again on using vC = vr)
v2r

1.55 The angular velocity is a vector as infinitesimal rotation commute. As for relative lin-
ear velocity, the relative angular velocity of the body 1 with respect to the body 2 is
clearly
␻ 12 = ␻ 1 - ␻ 2

As ␻ 1 ⬜ ␻ 2, so, | ␻ 12 | = 2v21 + v22 = 5 rad/s (on substituting values)


If frame K ¿ rotates with angular velocity ␻ with respect to frame K, the relation be-
tween the time derivatives of any vector a, seen from different frames is:
da
2 = da 2 + ␻ * a
dt K dt K ¿
If frame attached with the intersection point of two axes (about which solids are ro-
tating) is K and the frame attached with rotating solid with angular velocity ␻ 2 is K ¿
d␻ 1 d␻ 1
Then, a b = a b + ␻2 * ␻1
dt K dt K¿

d␻ 1
However, a b = 0 (as the first body rotates with constant angular velocity in space)
dt K
d␻ 1
and a b = ␤ 12 (the sought angular velocity)
dt K¿

Hence, ␤ 12 = ␻ 1 * ␻ 2

So, | ␤ 12 | = v1v2 = 12 rad/s2 (on substituting values)

1.56 (a) We have ␻ = at i + bt 2 j (1)

So, v = 2(at)2 + (bt 2)2, thus, v | t = 10s = 7.81 rad/s

Differentiating Eq. (1) with respect to time


d␻
␤ = = a i + 2bt j (2)
dt

So, ␤ = 2a2 + (2bt)2


38 PART ONE PHYSICAL FUNDAMENTALS OF MECHANICS

and ␤ |t = 10 s = 1.3 rad/s2

␻#␤ (at i + bt2 j) # (ai + 2bt j)


(b) cos a = =
vb 2(at)2 + (bt 2)2 2a2 + (2bt)2

Putting the values of (a) and (b) and taking t  10 s, we get


a = 17°

1.57 (a) Let the axis of the cone (OC ) rotate in anticlockwise sense with constant angular
velocity ␻ ¿ and the cone itself about it’s own axis (OC ) in clockwise sense with
angular velocity ␻ 0 . Then the resultant angular velocity of the cone

␻ = ␻ + ␻0

As the rolling is pure the magnitudes of the vectors O'


w
␻¿ and ␻ 0 can be easily found from the figure as
v v w0 w'
v¿ = and v0 =
R cot a R
O v
As ␻ ⬜ ␻ 0 , hence
C
R
a b + a b
v 2 v 2
v = 2v¿ 2 + v20
A R cot a R A
v
= = 2.32 rad/s
R cos a

(b) Vector of angular acceleration

d␻ d (␻ ¿ + ␻ 0) d␻ 0
␤ = = = (as ␻ ¿ = constant)
dt dt dt

If any vector a (say) rotates with angular velocity ␻ keeping its values constant
then da /dt = ␻ * a. The vector ␻ 0 keeping its magnitude constant rotates about
the OO¿ axis with the angular velocity ␻ ¿ . So d␻ 0 /dt = (␻ ¿ * ␻ 0). Hence,
␤ = ␻ ¿ * ␻ 0. The magnitude of the vector ␤ is equal to b = v¿ v0 (as ␻ ¿ ⬜ ␻ 0).

a b
v v
So, b =
R cot a R

v2
= tan a = 2.3 rad/s2, on putting the values of v, R and a
R2
1.2 THE FUNDAMENTAL EQUATION OF DYNAMICS 39

Alternate:
Vector ␻ 0 is turning keeping the magnitude constant. To find d␻ 0 , let us make a vec-
tor diagram as shown in the figure. The circle shown in the figure has the radius v0
and the tail of ␻ 0 and of ␻ 0 + d␻ 0 coincides at the centre of the circle.

Let ␻ 0 turn by the small angle dw in the differential time interval


dt, so dw = v¿dt. Thus | d␻ 0 | L arc length = v0 dw = v0 v¿dt.
As ␻ 0 is along radial line and d␻ 0 is along the tangent in the w0+dw0
dw0
turning sense of ␻ 0 so d␻ 0 ⬜ ␻ 0 . In vector form w0
df=w'dt

d␻ 0 = (␻ ¿ * ␻ 0 ) dt
d␻ 0
Hence, ␤ = = (␻ ¿ * ␻ 0 )
dt

1.58 The axis AB acquired the angular velocity


␻ ¿ = ␤ 0t
Using the facts of the solution of problem 1.57, the angular velocity of the body is

v = 2v20 + v¿ 2
B
= 2v20 + b 20 t 2 = 0.6 rad/s '

The angular acceleration,


0 A
d␻¿ d(␻ ¿ + ␻ 0 ) d␻ ¿ d␻ 0
␤ = = = +
dt dt dt dt
d␻ 0 d␻ ¿
But, = ␻¿ * ␻ 0 and = ␤0
dt dt
So, ␤ = (␤ 0 + ␻ ¿ * ␻ 0) = ␤ 0 + (␤ 0t * ␻ 0) (because ␻ ¿ = ␤ 0 t )

As, ␤ 0 ⬜ ␻ 0 so, ␤ = 2(v0 b 0t)2 + b 20 = b 0 21 + (v0t)2 = 0.2 rad/s2

1.2 The Fundamental Equation of Dynamics

1.59 Let R be the constant upward thrust on the aerostat of mass m, coming down with a
constant acceleration w. Applying Newton’s second law of motion for the aerostat in
projection form
Fy = mwy
mg - R = mw (1)
40 PART ONE PHYSICAL FUNDAMENTALS OF MECHANICS

Now, if ¢m be the mass to be dumped, then using the equation Fy = mwy


R - (m - ¢m) g = (m - ¢m)w (2)

From Eqs. (1) and (2), we get


2mw
¢m =
g + w
1.60 Let us write the fundamental equation of dy- N1 N2
x T2
namics for all the three blocks in terms of pro- T1 T2
jections, having taken the positive direction of
x and y axes as shown in the figure and using kN 1 kN 2
T1 m1 g m 2g
the fact that kinematical relation between the
accelerations is such that the blocks move y
with same value of acceleration (say w)
m0g - T1 = m0w (1) m 0g
T1 - T2 - km1g = m1w (2)
and T2 - km2g = m2w (3)

The simultaneous solution of Eqs. (1), (2) and (3) yields,


[m0 - k (m1 + m2)]
w = g
m0 + m1 + m2
(1 + k) m0
and T2 = m2g
m0 + m1 + m2

As the block m0 moves down with acceleration w, so in vector form


[m0 - k (m1 + m2)] g
w =
m0 + m1 + m2

1.61 (a) Let us indicate the positive direction of x-


axis along the incline (see figure). The fig- N1 R N2
ures show the force diagram for the blocks. fr1 fr2
Let R be the force of interaction between the R
bars and they are obviously sliding down m1g m2g
with the same constant acceleration w.
Newton’s second law of motion in projec- x x
tion form along x-axis for the blocks gives
m1g sin a - k1m1 g cos a + R = m1w (1)

m2g sin a - R - k2m2 g cos a = m2w (2)


1.2 THE FUNDAMENTAL EQUATION OF DYNAMICS 41

Solving Eqs. (1) and (2) simultaneously, we get


k1m1 + k2m2
w = g sin a - g cos a (3)
m1 + m2
m1m2 (k1 - k2) g cos a
and R =
m1 + m2
(b) When the blocks just slide down the plane, w = 0, then from Eq. (3)
k1m1 + k2m2
g sin a - g cos a = 0
m1 + m2
or (m1 + m2) sin a = (k1m1 + k2m2) cos a
(k1m1 + k2m2)
Hence, tan a =
m1 + m2

1.62 Case 1: When the body is launched up.


Let k be the coefficient of friction, u the velocity of projection and l the distance tra-
versed along the incline. Retarding force on the block = mg sin a + kmg cos a and
hence the retardation = g sin a + kg cos a.
Using the equation of particle kinematics along the incline,
0 = u2 - 2(g sin a + k g cos a) l
u2
or l = (1)
2(g sin a + k g cos a)
and 0 = u - (g sin a + k g cos a) t
or u = (g sin a + k g cos a) t (2)
1
Using Eq. (2) in Eq. (1) we get l = (g sin a + k g cos a) t 2 (3)
2
Case 2: When the block comes downward.
The net force on the body  mg sin ␣  kmg cos ␣ and hence its acceleration
 g sin ␣  kg cos ␣.
Let, t ¿ be the time required, then
1
l = (g sin a - k g cos a)t ¿ 2 (4)
2
From Eqs. (3) and (4)
t2 sin a - k cos a
=
t ¿2 sin a + k cos a
t 1
But, = (according to the question)
t¿ h
42 PART ONE PHYSICAL FUNDAMENTALS OF MECHANICS

Hence, on solving we get


(h2 - 1)
k = tan a = 0.16
(h2 + 1)

1.63 At the initial moment, obviously the tension in the thread


connecting m1 and m2 equals the weight of m2.
N T T
(a) For the block m2 to come down or the block m1 to go
up, the conditions is
m 2 g - T Ú 0 and T - m1g sin a - fr Ú 0
fr m1g m2g
where T is tension and ƒr is friction which in the lim-
iting case equals km 1 g cos a.
Then m2g - m1g sin a 7 km1 g cos a
m2
or 7 (k cos a + sin a)
m1
(b) Similarly in the case
m1g sin a - m 2 g 7 frlim
or m1g sin a - m 2 g 7 km 1 g cos a
m2
or 6 (sin a - k cos a)
m1

(c) For this case, neither kind of motion is possible, and ƒr need not be limiting.
m2
Hence, (k cos a + sin a) 7 7 (sin a - k cos a)
m1

1.64 From the conditions obtained in the previous problem, first we will check whether the
mass m2 goes up or down.
Here, m 2 /m1 = h 7 sin a + k cos a, (substituting the values). Hence the mass m2 will
come down with an acceleration (say w). From the free body diagram of previous
problem,
m2g - T = m2w (1)

and T - m1g sin a - km1g cos a = m1w (2)


Adding Eqs. (1) and (2), we get,
m2g - m1g sin a - km1g cos a = (m1 + m2) w
(m2/m1 - sin a - k cos a) g (h - sin a - k cos a)g
w = =
(1 + m2/m1) 1 + h
1.2 THE FUNDAMENTAL EQUATION OF DYNAMICS 43

Substituting all the values, w = 0.048 g L 0.05 g


As m2 moves down with acceleration of magnitude w = 0.05 g 7 0, thus in vector
form, acceleration of m2 is given by
(h - sin a - k cos a)g
w2 = = 0.05 g
1 + h =

1.65 Let us write Newton’s second law in projection form along positive x-axis for the
plank and the bar
fr = m1w1, F - fr = m2w2 (1)

At the initial moment, fr represents the static friction, and as F


the force F grows so does the friction force fr, but up to its m21
fr fr
limiting value, i.e., fr = frs (max) = kN = km2g . m12
Unless this value is reached, both bodies move as a single
body with equal acceleration. But as soon as the force fr reaches the limit, the bar
starts sliding over the plank, i.e., w2 Ú w1.
Substituting here the values of w1 and w2 taken from Eq. (1) and taking into account that
ƒr = km2g, we obtain,
km2
(at - m2g)/m2 Ú g
m1
(where the sign “=” corresponds to the moment t = t0).

kgm2(m1 + m2)
Hence, t0 =
am1
at
If t … t0 , then w1 = w2 =
m1 + m2
km2g (at - k m2 g)
and if t 7 t0 , then w1 = = constant, w2 =
m1 m2
On this basis w1(t) and w2(t) plots are as shown in the figure of answer sheet.

1.66 Let us designate the x-axis (see figure) and apply Fx = mwx for body A
mg sin a - kmg cos a = mw N
fr
or w = g sin a - kg cos a
Now, from kinematical equation: mg

l sec a = 0 + (1/2) wt 2
l
or t = 22l sec a/(sin a - k cos a) g x
= 22l/(sin 2a/2 - k cos2 a)g (using Eq. 1)
44 PART ONE PHYSICAL FUNDAMENTALS OF MECHANICS

d a - k cos2 a b
sin 2a
2
For tmin, = 0
da

2 cos 2a
i.e., + 2k cos a sin a = 0
2
1
or tan 2a = - Q a = 49°
k
and putting the values of a, k and l in Eq. (2) we get tmin = 1 s.

1.67 Let us fix the x -y co-ordinate system to the wedge, taking the x-axis up along the
incline and the y-axis perpendicular to it (see figure).
T
Now, we draw the free body diagram for the bar.
Let us apply Newton’s second law in projection form along x- and
y-axes for the bar N

T cos b - mg sin a - fr = 0 (1)


y fr
mg
T sin b + N - mg cos a = 0

or N = mg cos a - T sin b (2)


x
But, ƒr = kN and using Eq. (2) in Eq. (1), we get

T = mg sin a + kmg cos a/(cos b + k sin b) (3)


For Tmin the value of (cos b + k sin b) should be maximum

d (cos b + k sin b)
So, = 0 or tan b = k
db
Putting this value of b in Eq. (3), we get
mg (sin a + k cos a)
Tmin =
21 + k 2
1.68 First of all let us draw the free body diagram for the small body of mass m and indi-
cate x-axis along the horizontal plane and y-axis, perpendicular to it, as shown in the
figure. Let the block break off the plane at t = t0 , i.e., N = 0.
So, N = mg - at0 sin a = 0 y N F
mg α
or t0 = (1) x
a sin a
From Fx = mwx, for the body under investigation
mg
mdvx
= at cos a
dt
1.2 THE FUNDAMENTAL EQUATION OF DYNAMICS 45

Integrating within the limits for v (t)


v t

m dvx = a cos a tdt (using Eq. 1)


3 3
0 0
ds a cos a 2
So, v = = t (2)
dt 2m
Integrating, Eq. (2) for s (t) we get
a cos a t 3
s = (3)
2m 3
Using the value of t = t0 from Eq. (1), into Eqs. (2) and (3)
mg2 cos a m2g 3cos a
v = and s =
2 a sin2 a 6 a2sin3 a

1.69 Newton’s second law of motion in projection form, along horizontal or x-axis, i.e.,
Fx = mwx gives
dv N
F cos (as) = mv (as a = as) F
ds
α
or F cos (as) ds = mvdv x

Integrating, over the limits for v (s)


x mg
F v2
cos (ax) dx =
m 3 2
0

2F sin a
or v = (as a = as)
A ma
= 22g sin a/3a (using F = mg /3)

This is the sought relationship.

1.70 From the Newton’s second law in projection form


For the bar,
T - 2 kmg = (2m) w (1)
For the motor,
T - kmg = mw ¿ (2)

Now, from the equation of kinematics in the frame of


bar or motor T T
2m
1 m
l = (w + w¿) t 2 (3) fr fr
2
46 PART ONE PHYSICAL FUNDAMENTALS OF MECHANICS

From Eqs. (1), (2) and (3) we get on eliminating T and w¿

t = 22l/(kg + 3w)

1.71 (a) Let us write Newton’s second law in vector form F = m w, for both the blocks (in
the frame of ground)
T + m1g = m1w1 (1)
T + m2g = m2w2 (2)

These two equations contain three unknown quantities w1, w2


and T. The third equation is provided by the kinematic rela-
tionship between the accelerations
T T
w1 = w0 + w¿, w2 = w0 - w¿ (3) w0
m2
where w is the acceleration of the mass m1 with respect to the m1
pulley or elevator car. m2 g

Summing up term wise the left hand and the right-hand sides m1 g
of these kinematical equations, we get
w1 + w2 = 2w0 (4)
The simultaneous solution of Eqs. (1), (2) and (4) yields
(m1 - m2) g + 2m2w0
w1 =
m1 + m2
Using this result in Eq. (3), we get,
m1 - m2 2m1m2
w1 ¿ = (g - w0) and T = (w0 - g)
m1 + m2 m1 + m2
m1 - m2
Using the results in Eq. (3) we get, w = (g - w0)
m1 + m2
(b) Obviously the force exerted by the pulley on the ceiling of the car

4m1m2
F = - 2T = (g - w0)
m1 + m2
Note: One could also solve this problem in the frame of elevator car.
1.72 Let us write Newton’s second law for both, bar 1 and body 2 in terms of projection
having taken the positive direction of x1 and x2 as shown in the figure and assuming
that body 2 starts sliding, say, upward along the incline.
T1 - m1g sin a = m1w1 (1)
m2g - T2 = m2w (2)
1.2 THE FUNDAMENTAL EQUATION OF DYNAMICS 47

For the pulley, moving in vertical direction from the x1


equation Fx = mwx
N T1
2T2 - T1 = (mp) w1 = 0 T1

(as mass of the pulley mp = 0) T2 x2


T2
T2
or T1 = 2T2 (3) m1g
As the length of the threads are constant, the kinemat- m2g
ic relationship of accelerations becomes
w = 2w1 (4)

Simultaneous solutions of all these equations yields

2g (2 m2 /m1 - sin a) 2g (2h - sin a)


w = =
(4 m2 /m1 + 1) 4h + 1

As h 7 1, w is directed vertically downward, and hence in vector form


2 (2h - sin a)
w = g
4h + 1

1.73 Let us write Newton’s second law for masses m1 and m2 and moving pulley in verti-
cal direction along positive x-axis (see figure):
m1g - T = m1w1x w0 x1
T1
m2g - T = m2w2x m0

T1 - 2T = 0 (as m = 0) T1

or T1 = 2T
T w0
Again using Newton’s second law in projection form for T
mass m0 along positive x1 direction (see figure), we get x
m1g
T1 = m0w0 m2g

The kinematic relationship between the accelerations of masses given in terms of pro-
jection on the x-axis
w1x + w2x = 2w0
Simultaneous solution of the obtained five equations yields, in vector form
[4m1m2 + m0 (m1 - m2)]g
w1 =
4m1m2 + m0(m1 + m2 )
48 PART ONE PHYSICAL FUNDAMENTALS OF MECHANICS

1.74 As the thread is not tied with m, so if there were no friction between the thread
and the ball m, the tension in the thread would be zero and as a result both bod-
ies will have free fall motion. Obviously in the given problem it is the friction force
exerted by the ball on the thread, which becomes the tension in the thread. From
the condition or language of the problem wM 7 wm and as both are directed down-
ward so, relative acceleration of M = wM - wm and is directed downward.
Kinematical equation for the ball in the frame of rod in projection form along
upward direction gives
1
l = (w - wm ) t 2 (1)
2 M
Newton’s second law in projection form along vertically down di-
rection for both, rod and ball gives, T =fr
wm
Mg - ƒr = MwM (2) wM
fr
mg - ƒr = mwm (3) mg

Multiplying Eq. (2) by m and Eq. (3) by M and then subtracting Eq. Mg
(3) from Eq. (2) and after using Eq. (1), we get

2l Mm
ƒr =
(M - m) t 2

1.75 Suppose, the ball goes up with acceleration w1 and the rod comes down with the
acceleration w2.
As the length of the thread is constant, 2w1 = w2 (1)
Newton’s second law in projection form, vertically upwards for the ball
and vertically downwards for the rod respectively gives, T
T - mg = mw1 (2) T'
T'
T'
and Mg - T ¿ = Mw2 (3)

but, T = 2T ¿ (because pulley is massless) (4) T

From Eqs. (1), (2), (3) and (4)


mg Mg
(2M - m)g (2 - h)g
w1 = = (upwards)
m + 4M h + 4

2(2 - h)g
and w2 = (downwards)
(h + 4)
1.2 THE FUNDAMENTAL EQUATION OF DYNAMICS 49

From kinematical equation in projection form, we get


1
l = (w + w2) t 2
2 1
as w1 and w2 are in the opposite direction.
Putting the values of w1 and w2, the sought time becomes

t = 22l (h + 4)/3 (2 - h)g = 1.4 s

1.76 Using Newton’s second law in projection form along x-axis for the body 1 and along
negative x-axis for the body 2, respectively, we get
m1g - T1 = m1w1 (1)
T2 - m2g = m2w2 (2) T2
For the pulley lowering in downward direction from along x -axis, T2
Newton’s law gives
T1
T1 - 2T2 = 0 (as pulley is massless) T1 T2
or T1 = 2T2 (3) h m1g
As the length of the thread is constant, so m2g
w2 = 2w1 (4)

The simultaneous solution of above equations yields


2(m1 - 2m2) g 2(h - 2) m1
w2 = = g a as = hb (5)
4m2 + m1 h + 4 m2

Obviously during the time interval in which the body 1 comes to the horizontal floor cov-
ering the distance h, the body 2 moves upward the distance 2h. At the moment when
the body 2 is at the height 2h from the floor its velocity is given by the expression

2(h - 2) g 8h (h - 2)g
v 22 = 2w2(2h) = 2 c d 2h =
h + 4 h + 4

After the body m1 touches the floor, the thread becomes slack or the tension in the
thread zero, thus as a result body 2 is only under gravity for it’s subsequent motion.
Owing to the velocity v2 at that moment or at the height 2h from the floor, the body
2 further goes up under gravity by the distance,
v22 4h (h - 2)
h¿ = =
2g h + 4
Thus the sought maximum height attained by body 2 is
50 PART ONE PHYSICAL FUNDAMENTALS OF MECHANICS

4h (h - 2)
H = 2h + h¿ = 2h +
(h + 4)

6hh
= = 0.6 m (on substituting values)
h + 4

1.77 Let us draw free body diagram of each body, i.e., of rod A and of wedge B and also
draw the kinematical diagram for accelerations, after analysing the directions of
motion of A and B. The kinematic relationship of accelerations is
wA
tan a = (1)
wB
Let us write Newton’s second law for both bodies in terms of projections having taken
positive directions of y - and x -axes as shown in the figure:
mA g - N cos a = mAwA (2)
and N sin a = mBwB (3)

Simultaneous solution of Eqs. (1), (2) and (3) yields


mA g sin a g
wA = =
mA sin a + mB cot a cos a (1 + h cot2 a)
wA g
and wB = =
tan a (tan a + h cot a)

N" wAB
mA g B
wA
N
mA g
y N
wB
x
N'

Note: We may also solve this problem using conservation of mechanical energy instead
of Newton’s second law.

1.78 Let us draw free body diagram of each body and fix the coordinate system, as shown
in the figure. After analysing the motion of M and m on the basis of force diagrams,
let us draw the kinematic diagram for accelerations (see figure).
As the length of thread is constant so,
1.2 THE FUNDAMENTAL EQUATION OF DYNAMICS 51

T
T
T kN y
m wmM wm
N
N
kN T wmM
M wM
T mg
x
wM

dsmM = dsM and as vmM and vM do not change their directions that’s why

| wmM | = | wM | = w (say)

As wm = wmM + wM (1)
So, from the triangle law of vector addition
wm = 22 w
From equation Fx = mwx, for the wedge and the block,
T - N = Mw (2)

and N = mw (3)

Now, from equation Fy = mwy, for the block,


mg - T - kN = mw (4)
Simultaneous solution of Eqs. (2), (3) and (4) yields
mg g
w = =
(km + 2m + M) (k + 2 + M/m)
Hence, using Eq. (1)
g 22
wm =
(2 + k + M/m)

1.79 Bodies 1 and 2 will remain at rest with respect to bar A for wmin … w … wmax where
wmin is the sought minimum acceleration of the bar. Beyond these limits there will be
a relative motion between bar and the bodies. For 0 … w … wmin, the tendency of
body 1 in relation to the bar A is to move towards right and is in the opposite sense
for w Ú wmax. On the basis of above argument the static friction on 2 by A is directed
upward and on 1 by A is directed towards left for the purpose of calculating wmin.
Let us write Newton’s second law for bodies 1 and 2 in terms of projection along
positive x-axis (see figure).
T - fr1 = mw or fr1 = T - mw (1)
52 PART ONE PHYSICAL FUNDAMENTALS OF MECHANICS

N2 = mw (2) N1 w
T
As body 2 has no acceleration in vertical direction, so fr1
fr2 = mg - T (3)
mg T
From Eqs. (1) and (3) fr2 N2
(fr1 + fr2) = m ( g - w) (4) mg
But, fr1 + fr2 … k (N1 + N2)
or fr1 + fr2 … k (mg + mw) (5)
From Eqs. (4) and (5)
g (1 - k)
m (g - w) … mk (g + w), or wÚ
(1 + k)
g (1 - k)
Hence, wmin =
(1 + k)

1.80 On the basis of the initial argument of the solution of problem 1.79, the tendency of
bar 2 with respect to 1 will be to move up along the plane.
Let us fix x -y coordinate system in the frame of ground as shown in the figure.
From second law of motion in projection form along y - and x - axes
mg cos a - N = mw sin a

or N = m (g cos a - w sin a) (1) N


w y
mg sin a + ƒr = mw cos a x
fr mg
or ƒr = m (w cos a - g sin a) (2)

but ƒr … kN

So from Eqs. (1) and (2), (w cos a - g sin a) … k (g cos a + w sin a)

or w (cos a - k sin a) … g (k cos a + sin a)

(k cos a + sin a)
or w … g
cos a - k sin a
So, the sought maximum acceleration of the wedge is

(k cos a + sin a) g (k cot a + 1)g


wmax = = where cot a 7 k
cos a - k sin a cot a - k
1.2 THE FUNDAMENTAL EQUATION OF DYNAMICS 53

1.81 Let us draw the force diagram of each body, and on this basis we observe that the
prism moves towards right (say) with an acceleration w1 and the bar 2 of mass m2
moves down the plane with respect to 1, say with acceleration w21, then
w2 = w21 + w1 (see figure).
Let us write Newton’s second law of both bodies in projection form along positive y2
and x1 axes as shown in the figure.
m2g cos a - N = m2w2(y ) = m2 [w21(y ) + w1(y )] = m2 [0 + w1 sin a]
2 2 2

or m2g cos a - N = m2w1 sin a (1)

and N sin a = m1w1 (2)


Solving Eqs. (1) and (2), we get N
m2g sin a cos a g sin a cos a w21
w2
w1 = = y2 N
m1 + m2 sin a2 (m1/m2) + sin2 a
x2 mg
x1 w1

1.82 To analyse the kinematical relations between the bodies, sketch the force diagram of each
body as shown in the figure.
T T
On the basis of force diagram, it N T
is obvious that the wedge M
will move towards right and the wmM
block will move down along x y wm
N Mg
the wedge. As the length of the mg
x1
thread is constant, the distance
wM
travelled by the block on the
N'
wedge must be equal to the dis-
tance travelled by the wedge on the floor. Hence dsmM = dsM . As vmM and vM do not
change their directions and acceleration that’s why wmM c c vmM and wM c c vM and
wmM = w (say) and accordingly the diagram of kinematical dependence is shown in
figure.
As wm = wmM + wM, so from triangle law of vector addition

wm = 2w2M + w2M - 2 wmM wM cos a = w 22(1 - cos a) (1)

From Fx = mwx, (for the wedge),


T - T cos a + N sin a = M w (2)
For the bar m let us fix x - y coordinate system in the frame of ground. Newton’s law in
in projection form along x - and y -axes (see figure) gives
54 PART ONE PHYSICAL FUNDAMENTALS OF MECHANICS

mg sin a - T = mwm (x) = m 3wmM (x) + wM (x)4


= m [wmM + wM cos (p - a)] = mw (1 - cos a) (3)

mg cos a - N = mwm(y) = m [wmM (y) + wm (y)] = m [0 + w sin a] (4)


Solving the above equations simultaneously, we get
mg sin a
w =
M + 2m (1 - cos a)
Note: We can study the motion of the block m in the frame of wedge also, alternately
we may solve this problem using conservation of mechanical energy.

1.83 Let us sketch the diagram for the motion of the particle of mass m along the circle of
radius R as shown in the figure.
For a particle, modulus of change in linear momentum | ¢p | = m | ¢v | .
(a) In this case | ¢v | =22v (see figure), and the time taken in describing quarter of the circle,
pR
¢t =
2v vf
vi
| ¢ p| m | ¢v |
Hence, | 6F7| = = R
¢t ¢t
O
22 mv 2 22 mv 2
= =
pR/2v pR

(b) In this case vi = 0 so, vf = v(t). Thus | ¢v | = | v(t) | = v (t) = wt t .

| ¢p | m| ¢v | m| v(t) |
Hence, | 6 F7 | = = = = mwt
¢t ¢t t
1.84 While moving in a loop, normal reaction exerted by the flyer on the loop at different
points and uncompensated weight if any contribute to the weight of flyer at those points.
(a) When the aircraft is at the lowermost point, Newton’s second law of motion in
projection form, Fn = mwn gives
mv 2 mg
N - mg = N"
R
O N'
mv 2
or N = mg + = 2.09 kN N
R
(b) When it is at the upper most point, again from Fn = mwn , we get
mg
mv 2 mg
N – + mg =
R
mv2
N– = - mg = 0.7 kN
R
1.2 THE FUNDAMENTAL EQUATION OF DYNAMICS 55

(c) When the aircraft is at the middle point of the loop, again from Fn = mwn
mv 2
N¿ = = 1.4 kN
R
The uncompensated weight is mg. Thus effective weight = 2N ¿ 2 + m2g 2 = 1.56 kN
acts obliquely.

1.85 (a) Let us depict the forces acting on the small sphere m, (at an arbitrary position
when the thread makes an angle u from the vertical) and write equation F = mw
via projection on the unit vectors t and n. From Ft = mwt , we have

dv
mg sin u = m
dt
vdv vdv
= m = m
ds l ( - d u)
T
(as vertical is reference line of angular position)
l n
or vdv = - gl sin ud u
Integrating both the sides we get t
v u mg

vdv = - g l sin u d u
3 3
0 p/2

v2
or = g l cos u
2
v2
Hence, = 2 g cos u = wn (1)
l
Note: Eq. (1) can be easily obtained by the conservation of mechanical energy.

From Fn = mwn

mv2
T - mg cos u =
l

Using Eq. (1) we have


T = 3mg cos u (2)

Again from equation Ft = mwt ,


mg sin u = mwt or wt = g sin u (3)
56 PART ONE PHYSICAL FUNDAMENTALS OF MECHANICS

Hence, w = 2w2t + w2n = 2(g sin u)2 + (2g cos u)2 (using Eqs. 1 and 3)
= g 21 + 3 cos2u .

(b) Vertical component of velocity, vy = v sin u

So, vy2 = v 2 sin2 u = 2 gl cos u sin2 u (using Eq. 1)

d (cos u sin2 u)
For maximum vy or vy2 , = 0
du
1
which yields cos u =
23
1
Therefore from Eq. (2), T = 3mg = 23 mg
23
(c) We have w = wt t + wn n thus wy = wt (y) + wn (y)
But in accordance with the problem wy = 0
So, wt (y) + wn (y) = 0

or - g sin u sin u + 2g cos2 u = 0


1
or cos u = or, u = 54.7°
23
1.86 The ball has only normal acceleration at the lowest position and only tangential
acceleration at either of the extreme positions, Let v be the speed of the ball at its
lowest position and l be length of the thread, then according to the problem
v2
= g sin a (1)
l
where a is the maximum deflection angle.
From Newton’s law in projection form:
α
Ft = mwt θ
l T
dv
- mg sin u = mv n
ldu t
v
or - gl sin ud u = vdv
mg
On integrating both the sides within their limits.
a 0

- gl sin ud u = vdv
3 3
0 v
1.2 THE FUNDAMENTAL EQUATION OF DYNAMICS 57

or v 2 = 2gl (1 - cos a) (2)


Note: Eq. (2) can easily be obtained by the conservation of mechanical energy of the
ball in the uniform field of gravity.
From Eqs. (1) and (2) with u = a
sin a = 2(1 - cos a)
or sin a + 2 cos a = 2
On solving we get,
a ⬵ 53°

1.87 Let us depict the forces acting on the body A (which are the force of gravity mg and
the normal reaction N) and write equation F = m w via projection on the unit vectors
t and n (see figure).
From Ft = mwt
N
dv
mg sin u = m
dt t
R n
vdv vdv
= m = m mg
ds Rdu
or gR sin ud u = vdv
Integrating both sides for obtaining v (u), we get
u v

gR sin u du¿ = vdv


3 3
0 0

or v 2 = 2gR (1 - cos u) (1)

From Fn = mwn
v2
mg cos u - N = m (2)
R
At the moment the body loses contact with the surface, N = 0, and therefore Eq. (2)
becomes
v 2 = gR cos u (3)
where v and u correspond to the moment when the body loses contact with the surface.
Solving Eqs. (1) and (3), we obtain
2
cos u = or u = cos-1 (2/3) ⬵48° and v = 22gR/3
3
1.88 At first draw the free body diagram of the device as, shown. The forces, acting on the
sleeve are its weight, acting vertically downward, spring force, along the length of the
spring and normal reaction by the rod, perpendicular to its length.
Let F be the spring force, and ¢l be the elongation.
58 PART ONE PHYSICAL FUNDAMENTALS OF MECHANICS

From Fn = mwn,

N sin u + F cos u = m v2r (1)

where r cos u = (l0 + ¢l ).


Similarly, from Ft = mwt

N cos u - F sin u = 0 or N = F sin u/cos u (2)

From Eqs. (1) and (2)

F (sin u/cos u ) # sin u + F cos u = m v2 r


= m v2 (l0 + ¢l )/cos u w r
q F
On putting F = k ¢ l, N

k¢l sin2 u + k ¢ l cos2 u = m v2 (l0 + ¢ l ) mg

On solving, we get
l0 l0
¢l = m v2 =
k - m v2 (k/m v2 - 1)

It is independent of the direction of rotation.

1.89 According to the question, the cyclist moves along the circular path and the centripetal
force is provided by the frictional force. Thus from the equation

Fn = mwn
mv 2 mv 2
ƒr = or kmg =
r r

k0 a 1 - b g =
r v2
or or v 2 = k0 (r - r 2/R) g (1)
R r

d ar - b
r2
R
For vmax, we should have = 0
dr

2r R
or 1 - = 0, so r =
R 2

1
Hence, vmax = 2k0gR
2
1.2 THE FUNDAMENTAL EQUATION OF DYNAMICS 59

1.90 As initial velocity is zero thus,

v 2 = 2w4 s = f (s) (1)


As wt 7 0 the speed of the car increases with time or distance. Till the moment, slid-
ing starts, the static friction provides the required centripetal accelaration the car.

Thus, fr = mw, but ƒr … kmg

wt2 + a b … k 2g 2
v2 2
So, w 2 … k2g2 or
R

or v 2 … R (k 2g 2 - w t2)

Hence, 2
vmax = R 2(k 2g 2 - w t2)
2
vmax
a b - 1 = 60 m
R kg
So, from Eq. (1), the sought distance s = =
2wt 2 B wt

1.91 Since the car follows a sinusoidal curve, so the maximum velocity at which it can ride
without sliding is the point of minimum radius of curvature obviously in this case the
static friction between the car and the road is limiting. Hence, from the equation
Fn = mwn
mv 2
… kmg
R
or v … 2 kRg

So, vmax = 2kR min g (1)

We know that the radius of curvature for a curve at any point (x, y) is given as

[1 + (dy>dx)2]3/2
R = 2 2 (2)
(d 2y)>dx 2

For the given curve,


d 2y -a
cos a b
dy a x x
= and = sin
dx a a dx 2 a2 a

Substituting this value in Eq. (2), we get

[1 + (a 2/a2) cos2 (x/a)]3/2


R =
(a/a2) sin (x/a)
60 PART ONE PHYSICAL FUNDAMENTALS OF MECHANICS

x p
For the minimum R, =
a 2

and therefore, corresponding radius of curvature


a2
Rmin = (3)
a
Hence, from Eqs. (1) and (3)
vmax = a 2kg /a

1.92 The sought tensile stress acts on each element of the chain. Hence divide the chain
into small, similar elements so that each element may be assumed as a particle. We
consider one such element of mass dm, which subtends angle d a at the centre. The
chain moves along a circle of known radius R with known angular speed v and
certain forces act on it. We have to find one of these forces.

O y
T T
z dm
dN
dm y d
R x
O

dmg x

From Newton’s second law in projection form, Fx = mwx , we get


2T sin (da/2) - dN cos u = dm v2R
and from FZ = mwz , we get
dN sin u = (dm) g
Then putting dm = mda/2p and sin (da/2) = da/2 and solving, we get
m (v2 R + g cot u)
T =
2p

1.93 Let us consider a small element of the thread and draw free body diagram for this
element.

(a) Applying Newton’s second law of motion in projection form, Fn = mwn for this
element,
1.2 THE FUNDAMENTAL EQUATION OF DYNAMICS 61

(T + dT ) sin (du/2) + T sin (du/2) - dN = dmv2R = 0


or 2T sin (du/2) = dN [neglecting the term (dT sin du/2)]

T du = dN a as sin b
du du
or L (1)
2 2
Also, dfr = kdN = (T + dT) - T = dT (2)

From Eqs. (1) and (2) T


kdN
dT dN
kT du = dT or = kd u
T
d T+dT
Integrating u from u = 0 to u = p, we get T2
T2 T1
ln = kp (3) m2
T1
m1
1 T2 1
So, k = ln = ln h0
p T1 p
T2 mg g m2
¢as = = = h0≤
T1 m1g m1

(b) When m2 /m1 = h 7 h0 , the blocks will move with same value of acceleration
(say w) and clearly m2 moves downward. From Newton’s second law in projec-
tion form (downward for m2 and upward for m1), we get

m2 g - T2 = m2w (4)
and T2 - m 1 g = m1w (5)
T2
Also = h0 (6)
T1

Simultaneous solution of Eqs. (4), (5) and (6) yields


(m2 - h0m1)g (h - h0) m2
w = = g a as = hb
(m2 + h0m1) (h + h0) m1

1.94 The force with which the cylinder wall acts on the particle will provide centripetal
force necessary for the motion of the particle, and since there is no acceleration act-
ing in the horizontal direction, horizontal component of the velocity will remain con-
stant throughout the motion.

So, vx = v0 cos a
62 PART ONE PHYSICAL FUNDAMENTALS OF MECHANICS

Using, Fn = mwn, for the particle of mass m


mvx2 mv 02 cos2 a
N = = v0
R R

which is the required normal force.


x

1.95 Obviously the radius vector describing the position of the particle rela-
tive to the origin of the coordinate is,
r = xi + y j = a sin v t i + b cos vt j
Differentiating twice with respect to time
d 2r
w = = - ␻ 2 (a sin v t i + b cos v t j) = - v2r (1)
dt 2
Thus, F = m w = - m v2r (using Eq. 1)

1.96 (a) We have ¢p = Fdt = m g dt = m gt (1)


3 3
0

(b) Using the solution of problem 1.28 (b), the total time of motion,
2(v0 # g)
t = -
g2
Hence, using t = t in Eq. (1)
| ¢p | = mg t
- 2m (v0 # g)
=
g
1.97 From the equation of the given time dependence force F = a t (t - t) at t = t, the
force vanishes.
t

(a) Thus, ¢p = p = Fdt


3
0
t
at3
or p = a t (t - t) dt =
3 6
0
at3
But, p = mv so, v=
6m
1.2 THE FUNDAMENTAL EQUATION OF DYNAMICS 63

(b) Again from the equation F = mw


dv
at (t - t) = m
dt
or a (t t - t 2) dt = md v
Integrating within the limits for v(t)
t v

a (t t - t 2) dt = m dv
3 3
0 0

a - b = a - b
a tt 2 t3 at 2 t t
or v =
m 2 3 m 2 3

a - b
at 2 t t
Thus, v = for t … t
m 2 3
Hence, distance covered during the time interval t = t is given by
t t

a - b dt =
at 2 t t a t4
s = vdt =
3 3 m 2 3 m 12
0 0

1.98 We have F = F0 sin vt


dv
or m = F0 sin vt or md v = F0 sin vt dt
dt
On integrating,
- F0
mv = cos vt + C (where C is integration constant)
v
F0
When t = 0, v = 0, so C =
mv
- F0 F0
Hence, v = cos v t +
mv mv

As cos vt … 1 so,
F0
v = (1 - cos vt)
mv
t
F0t F0 sin vt F0
Thus, s = vdt = - = (vt - sin vt) .
3 mv mv2 mv2
0
(see figure in the answer sheet)
64 PART ONE PHYSICAL FUNDAMENTALS OF MECHANICS

1.99 According to the problem, the force acting on the particle of mass m is, F = F0 cos vt .

dv F0
So, m = F0 cos vt or dv= cos vt dt
dt m

Integrating, within the limits


v t
F0 F0
dv = cos vt dt or v = sin vt (1)
3 m 3 mv
0 0

It is clear from Eq. (1), that after starting at t = 0, the particle comes to rest for the
first time at t = p>v.
From Eq. (1)
F0 p
v = |v| = sin vt for t … (2)
mv v
Thus during the time interval t = p/v, the sought distance
p/v
F0 2F
s = sin vt dt =
mw 3 mv2
0
From Eq. (1)
F0
vmax = as | sin vt | … 1
mv

dv
1.100 (a) From the problem F = - r v so, m = -r v
dt
dv
Thus, m = - rv [as dv c T v]
dt
dv r
or = - dt
v m
r
On integrating ln v = - t + C
m
But at t = 0, v = v0 so, C = ln v0.
v r
So, ln = - t or v =v0e- (r/m)t
v0 m

Thus, for t : q, v = 0 .
dv -r
(b) We have m = - rv so, dv = ds
dt m
Integrating within the given limits to obtain v (s) , we get
1.2 THE FUNDAMENTAL EQUATION OF DYNAMICS 65

v s
r rs
dv = - ds or, v = v0 - (1)
3 m 3 m
v0 0

mv0
Thus for v = 0, s = stotal =
r
mdv dv -r
(c) We have = - rv or, = dt
dt v m
v0/h t
dv -r v0 r
or = dt or, ln = - t
3 v m 3 hv0 m
0 0

- m ln (1/h) m ln h
So, t = =
r r
Now, average velocity over this time interval
m
r ln h
rt
v0e- m dt
vdt 3 v0(h - 1)
3 0
6v7 =  =
m / r ln h h ln h
dt
3

1.101 According to the problem


dv dv
m = - kv 2 or, m = - kdt
dt v2

Integrating, within the limits


v t
dv k m (v0 - v)
2
= - dt or t = (1)
3v m 3 k v0v
v0 0

To find the value of k, rewrite


dv dv k
mv = - kv 2 or = - ds
ds v m

On integrating
v h
dv k
= - ds
3 v m 3
v0 0
66 PART ONE PHYSICAL FUNDAMENTALS OF MECHANICS

m v0
So, k = ln (2)
h v

Putting the value of k from Eq. (2) in Eq. (1), we get


h (v0 - v)
t = v0
v0v ln
v

1.102 From Newton’s second law for the bar in projection form, Fx = mwx , along x direc-
tion, we get
mg sin a - kmg cos a = mw

dv
or v = g sin a - ax g cos a (as k = ax) N fr
dx
x
or vdv = (g sin a - ax g cos a) dx mg
v x

or vdv = g (sin a - x cos a) dx


3 3
0 0
v 2 x2
So, = g (x sin a - a cos a) (1)
2 2
From Eq. (1),
2
v = 0 at either x = 0, or x =
tan a
a
As the motion of the bar is unidirectional it stops after going through a distance of
2 /a tan a.
From Eq. (1), for vmax,

d x2 1
(x sin a - a cos a) = 0 or x = tan a
dx 2 a
Hence, the maximum velocity will be at the distance, x = tan a/a .
Putting this value of x in Eq. (1), the maximum velocity,

g sin a tan a
vmax =
A a
1.103 Since the applied force is proportional to the time and the frictional force also ex-
ists, the motion does not start just after applying the force. The body starts its mo-
tion when F equals the limiting friction. Let the motion start after time t0, then
1.2 THE FUNDAMENTAL EQUATION OF DYNAMICS 67

kmg
F = at0 = kmg or t0 =
a
So, for t … t0, the body remains at rest and for t 7 t0 obviously
mdv
= a (t - t0) or, mdv = a (t - t0) dt
dt
Integrating, and noting v = 0 at t = t0, we have for t 7 t0
v t
a
mdv = a (t - t0 ) dt or v = (t - t 0)2
3 3 2m
0 0
t
a a
Thus, s = vdt = (t - t0)2 dt = (t - t0)3
3 2m L 6m
t0

1.104 While going upward, from Newton’s second law, in vertical direction
vdv vdv
m = - (mg + kv 2) or = - ds
ds ( + kv 2 /m )
g
At the maximum height h, the speed v = 0, so
0 h
vdv
= - ds
3 g + (kv /m)
2
3
v0 0

Integrating and solving, we get

k v 20
ln a 1 + + b
m
h = (1)
2k mg
When the body falls downward, the net force acting on the body in downward di-
rection equals (mg - kv 2) .
Hence net acceleration, in downward direction, according to second law of motion
vdv kv 2 vdv
= g- or, = ds
ds m g - kv 2/m
v¿ h
vdv
Thus, = ds
3 g - kv2/m 3
0 0

Integrating and putting the value of h from Eq. (1), we get


v0
v¿ =
21 + k v02/mg
68 PART ONE PHYSICAL FUNDAMENTALS OF MECHANICS

1.105 Let us fix x -y co-ordinate system to the given plane, taking x-axis in the direction
along which the force vector was oriented at the moment t = 0, then the fundamen-
tal equation of dynamics expressed via the projection on x and y axes gives,
dvx
F cos vt = m
dt
dvy
and F sin vt = m
dt
F
(a) Using the condition v (0) = 0, we obtain vx = sin v t
mv
F
and vy = (1 - cos vt)
mv

v = 2vx2 + v y2 = a b sin a b 2
2F 2 vt
Hence,
mv 2
(b) It is seen from this that the velocity v turns into zero after the time interval ¢t,
which can be found from therelation, v ¢t 2 = p. Consequently, the sought dis-
tance is
¢t y
8F
s = v dt = 2
F
3 mv θ=ωt
0

v dt
3
Average velocity, 6v7 =
O x
dt
3
2p/v

sin a b dt/(2pv) =
2F vt 4F
So, 6 v7 =
3 mv 2 pm v
0

1.106 The acceleration of the disk along the plane is determined by the projection of the
force of gravity on this plane Fx = mg sin a and the friction force fr = kmg cos a. In
our case, k = tan a and therefore
fr
fr = Fx = mg sin a
v
Let us find the projection of acceleration on the direction of
the tangent to the trajectory and on the x-axis
mwt = Fx cos w - fr = mg sin a (cos w - 1)
x
mwx = Fx - fr cos w = mg sin a (1 - cos w)
1.2 THE FUNDAMENTAL EQUATION OF DYNAMICS 69

It is seen from this that wt = - wx, which means that the velocity v and its projec-
tion vx differ only by a constant value C which does not change with time, i.e.,
v = - vx + C, where vx = v cos w. The constant C is found from the initial condi-
tion v = v0, Ux = 0 since w = p / 2 initially. Finally, we obtain

C = v0
v0
v =
(1 + cos w)

In the course of time w : 0 and v : v0/2. (Motion then is unaccelerated.)

1.107 Let us consider an element of length ds at an angle w T


from the vertical diameter. As the speed of this element
is zero at initial instant of time, its centripetal accelera- dN
tion is zero, and hence, dN - lds g cos w = 0, where l d
is the linear mass density of the chain. Let T and T + dT
be the tension at the upper and the lower ends of ds, T+dT
dmg
then we have from Ft = mwt ,
(T + dT) + l ds g sin w - T = ldswt

or dT + l Rd wg sin w = ldswt

If we sum the above equation for all elements, the term dT = 0, because there is
no tension at the free ends, so 3

l /R

lgR sin w d w = lwt ds = llwt


3 3
0

a 1 - cos b
gR l
Hence, wt =
l R
As wn = a at initial moment, so

a1 - cos b
gR l
w = | wt | =
l R
1.108 In the problem, we require the velocity of the body, relative to the sphere, which
itself moves with an acceleration w0 in horizontal direction (say towards left).
Hence it is advisable to solve the problem in the frame of sphere (a translating
frame here).
70 PART ONE PHYSICAL FUNDAMENTALS OF MECHANICS

At an arbitrary moment, when the body is at an angle u N


with the vertical, we sketch the force diagram for the body mw0
n
and write the second law of motion in projection form
t
Fn = mwn as,
mg
mv 2 w0
as, mg cos u - N - mw0 sin u = (1)
R

At the break off point, N = 0, u = u0 and let v = v0 ,


so Eq. (1) becomes,
v20
= g cos u0 - w0 sin u0 (2)
R

From Ft = mwt

vdv v dv
mg sin u - mw0 cos u = m = m
ds Rdu
or vdv = R (gsin u + w0 cos u) d u

v0 u0

Integrating v dv = R (g sin u + w0 cos u) du


3 3
0 0

v 20
= g 11 - cos u02 + w0 sin u0 (3)
2R
Note that Eq. (3) can also be obtained by the work-energy theorem A = ¢T (in the
frame of sphere). Therefore,
1
mgR (1 - cos u0) + mw0 R sin u0 = mv 02
2

[here mw0 R sin u0 is the work done by the pseudoforce ( - m w0)]


v 02
or = g(1 - cos u0) + w0 sin u0
2R

Solving Eqs. (2) and (3), we get


2 + k 25 + 9k 2 w0
and u0 = cos-1 c d a where k = b
2gR
v0 =
A 3 3 (1 + k )
2 g

Hence, u0 2 = 17°
w0 = g
1.2 THE FUNDAMENTAL EQUATION OF DYNAMICS 71

1.109 This is not central force problem unless the path is a circle about the said point.
Rather here Ft (tangential force) vanishes. Thus, the equation of motion becomes

vt = v0 = constant

m v20
1for r = r02
A
and =
r rn

We can consider the latter equation as the equilibrium under two forces. When the
motion is perturbed, we write r = r0 + x and the net force acting on the particle is

m v 02 -A
= n a1 - b
A nx
- +
(r0 + x)n r0 + x r0 r0

m v 02
a1 - b
x
+
r0 r0
m v 20
= - (1 - n) x
r 20
This opposes the displacement x, if n 6 1. (m v 20 /r is an outward directed centrifugal
force while - A/r n is the inward directed external force.)

1.110 Let us observe the behaviour of the sleeve in the fra-


me fixed to the rotating rod bent into the shape of
O
half circle. We resolve all the forces into tangential
and normal components, then the net downward tange-
ntial force on the sleeve is
N
mg sin u a 1 - cos u b
v2R
mw2Rsinq
g
O'
This vanishes for u = 0 and for u = u0 = cos-1( g/v2R) , mg
which is real if v2R 7 g .

If v2 R 6 g, then (1
[ - ( v2R/g ) cos u )] is always positive for small values of u and
hence the net tangential force near u = 0 opposes any displacement away from it,
and u = 0 is then stable.
If v2 R 7 g, then [(1 - ( v 2R/g) cosu ] is negative for small u near u = 0 , and u = 0 is then
unstable.
However u = u0 is stable because the force tends to bring the sleeve near the equi-
librium position u = u0.
If v2 R = g, the two positions coincide and becomes a stable equilibrium point.
72 PART ONE PHYSICAL FUNDAMENTALS OF MECHANICS

1.111 Define the axes as shown with z along the local vertical, x due east and y due no-
rth. (We assume we are in the northern hemisphere.) Then the Coriolis force has
the components as
Fcor = - 2m (v * v) y-North
z-Vertical
= 2mv [vy cos u - vz sin u) i - vx cos u j + vx cos u k]
x-East
= 2m v (vy cos u - vz sin u)i

Since vx is small when the direction in which the gun is =90°–


fired is due north. Thus the equations of motion (ne-
glecting centrifugal forces) are
$ $
x = 2m v(vy sin w - vz cos w), y = 0 and z = - g

Integrating we get y = v (constant)


z = - gt and x = 2vv sin wt + vgt 2 cos w

Finally,
1
x = vvt 2 sin w + vgt 3 cos w
3
Now, v 77 gt in the present case. So,

x = vv sin w a b = v sin u
s 2 s2
v v

= 7 cm (to the east)

1.112 For the observer attached in the frame of disks


N + Fcor + mg + Fcf = 0
So, N = - (Fcor + mg + Fcf)

So, | N | = 2(2m v¿v)2 + (mg)2 + (mv2r)2

1.113 The sleeve is free to slide along the rod AB. Thus the centrifugal force acts on it in
the outward direction along the river.
So, we have mv¿ = mv2r
dr
where v¿ =
dt

a v b
vdv d 1 2
But, w¿ = =
dr dr 2
1.2 THE FUNDAMENTAL EQUATION OF DYNAMICS 73

1 2 1
So, v = v2r2 + constant
2 2
or v 2 = v 02 + v2r 2

v0 being the initial velocity when r = 0. The Coriolis force is then

2mv 2v02 + v2r2 = 2mv2r 21 + v02>v2r2) = 2 # 83 N (on substituting values)

1.114 The disk OBAC is rotating with angular velocity v about the axis OO¿ passing
through the edge point O. The equation of motion in rotating frame is

mw¿ = F + mv2 R + 2m(v¿ * ␻) = F + Fin O'


where Fin is the resultant inertial force (pseudo force) which is the B
vector sum of centrifugal and Coriolis forces.
O A
(a) At A, Fin vanishes.
C
Thus, 0 = - 2m v2R n + 2mv¿vn
where n is the inward drawn unit vector to the centre, from the point in question, here A.
Thus, v¿ = vR
v¿ 2 v¿ 2
So, w = = = v2R
r R
O'
(b) At B, Fin = mv2r + 2mv¿v2 ( - R) 2r
m
B v'
v ¿ = vr = constant (from question)
r 2mv'
q
Fin = mv2r + 2mv2R O
R C n
A

From geometry,
r = 2R cos u (see figure)

So, | Fin | = mv2 24R2 - r2

1.115 The equation of motion in the rotating coordinate system is N

m w¿ = F + m v2 R + 2m (v¿ * v)
Fcf
. .
Now, v¿ = R u eu + R sin u w ew mg

and v = v cos u er - v sin u eu


74 PART ONE PHYSICAL FUNDAMENTALS OF MECHANICS

er eu ew
1 . .
F = 3 0 Ru (R sin u)w 3
2m cor
v cos u - v sin u 0
# # #
= er (vR sin2 u w) + [vR sin u cos u w]eu - (vR u cos u)ew

Now, on the sphere


# # .. # #
w¿ = (- R u2 - R sin u2 w2)er + (R u - R sin u cos u w2) eu
.. . #
+ (R sin uw + 2R cos u u w) e
w

Thus the equations of motion are


# ## #
m ( - R u2 - R sin2u w2 ) = N - mg cos u + mv2 R sin2 u + 2mvR sin2 uw
## # #
m (R u - R sin u cos u w2 ) = mg sin u + mv2R sin u cos u + 2mvR sin u cos u w
.. . # ##
m (R sin u w + 2R cos u u w) = - 2 mvR u cos u
#
From the third equation, we get, w = - v.
A result that is easy to understand by considering the motion in non-rotating frame.
# #
Eliminating w we get, mR u2 = mg cos - N
..
mR u = mg sin u

Integrating the last equation


1 #
mR u2 = mg (1 - cos u) (1)
2

Hence, N = (3 cos u - 2) mg
2
So the body must fly off for u = u0 = cos-1 , exactly as if the sphere were non-
3
rotating.
5
Now, at this point F cf = centrifugal force = m v2R sin u0 = mv2R
A9
#
Fcor = 2m 2(v2R)2 sin2 u cos2u + (v2R 2)2 sin4 u + (vR)2cos2 uu2
Putting the value of u from Eq. (1)
u = u0

2mv2R 8g
we get, Fcor = 5 + = 17 N. (on substituting values)
3 A 3v2R
1.2 THE FUNDAMENTAL EQUATION OF DYNAMICS 75

1.116 (a) When the train is moving along a meridian only the Coriolis force has a lateral
component and its magnitude (see the previous problem) is,
#
2mvv cos u = 2mv sin l (here we have put R u : v)

2p 54000 23
So, Flateral = 2 * 2000 * 103 * * *
86400 3600 2
= 3.77 kN (we write l for the latitude)

(b) The resultant of the inertial forces acting on the train is,
# #
Fin = - 2mvR u cos u ew + (mv2 R sin u cos u + 2m vR sin u cos u w) eu
#
+ (mv2 R sin2u + 2m vR sin2 u w) er
# # 1 v
This vanishes if u = 0, w = - v Fcor
2
1 1
Thus, v = vwew,vw = - vR sin u = - vR cos l O
2 2
(we write l for the latitude here)

Thus the train must move from the east to west along
the 60th parallel with a speed,
1 1 2p
vR cos l = * 10-4 * 6.37 * 106 = 115.8 m/s L 420 km/h .
2 4 8.64

1.117 We go to the equation given in problem 1.111. Here vy = 0 so we can take y = 0;


#
thus, we get for the motion in the x z plane.
$
x = - 2vvz cos w
#
and z = -g
1 2
Integrating, z = - gt
2
#
x = vg cos wt2

vg cos wt 3 = vg cos w a b
1 1 2h 3/2 2vh 2h
So x = = cos w
3 3 g 3 A g

There is thus a displacement to the east of


2 2p 2 * 500
* 500 * * 10-4 * 1 * = 26 cm
3 8.64 A 9.8
76 PART ONE PHYSICAL FUNDAMENTALS OF MECHANICS

1.3 Law of Conservation of Energy, Momentum


and Angular Momentum

1.118 As F is constant so the sought work done


A = F # ¢r = F # (r2 - r1)
or A = (3i + 4j ) # [(2i - 3j ) - (i + 2j)] = (3i + 4 j ) # (i - 5j) = 17 J

1.119 As locomotive is in unidirectional motion, its acceleration


dv 1 dv 2 a2
w = = =
dt 2 ds 2
ma 2
Hence, force acting on the locomotive F = mw =
2
Let, v = 0 at t = 0, then the distance covered during the first t seconds
1 2 1 a2 2 a2 2
s = wt = t = t
2 2 2 4
Hence the sought work,
ma 2 (a 2t 2) ma 4t 2
A = Fs = =
2 4 8

1 2 as 2
1.120 We have T = mv 2 = as 2 or v 2 = (1)
2 m
Differentiating Eq. (1) with respect to time
4as 2 as
2vwt = v or wt = (2)
m m
Hence net acceleration of the particle

a b + a b =
2as 2 2as 2 2 2as
w = 2wt2 + wn2 = 11 + (s>R) 2
C m mR m
F
Hence the sought, force F = mw = 2as 11 + (s/R)2
N
1.121 Let F, make an angle u with the horizontal at an arbi-
trary instant of time (see figure). Newton’s second law h
in projection form along the direction of the force fr
gives y mg
F = kmg cos u + mg sin u (because there is no acceler- x
ation of the body) l
1.3 LAW OF CONSERVATION OF ENERGY, MOMENTUM AND ANGULAR MOMENTUM 77

As F c c dr, the differential work done by the force F


dA = F # dr = Fds (where ds = | dr | )
= kmg ds (cos u) + mg ds sin u
= kmg dx + mg dy
l h

Hence, A = kmg dx + mg dy
3 3
0 0

= kmgl + mgh = mg (kl + h)

1.122 Let s be the distance covered by the disk along the incline, from the equation of in-
crement of mechanical energy of the disk in the field of gravity: ¢T + ¢U - Aƒr
0 + ( - mgs sin a) = - kmg cos a s - kmgl
N'1 1
kl
or s = (1) fr1
sin a - k cos a N2
mg s
Hence the sought work 2 fr2
l
Aƒr = - kmg [s cos a + l ]
klmg mg
Aƒr = - (using Eq. 1)
1 - k cot a
On putting the values
Aƒr = - 0.05 J

1.123 Let x be the compression in the spring when the bar m2 is about to shift. Therefore
at this moment spring force on m2 is equal to the limiting friction between the bar
m2 and horizontal floor. Hence
kx = km2g [where k is the spring constant (say)] (1)
For the block m1 from work-energy theorem:
A = ¢T = 0 for minimum force. (A here includes the work done in stretching the
spring.)
1 2 x
So, Fx - kx - kmgx = 0 or k = F - km1g (2)
2 2
m2
From Eqs. (1) and (2), F = kg a m1 + b
2
78 PART ONE PHYSICAL FUNDAMENTALS OF MECHANICS

1.124 From the initial condition of the problem, the limiting friction between the chain
lying on the horizontal table equals the weight of the over hanging part of the chain,
i.e., lhlg = kl(1 - h)lg (where l is the linear mass density of the chain).
h
So, k = (1)
1 - h
Let (at an arbitrary moment of time) the length of the chain on the table be x. So the
net friction force between the chain and the table, at this moment
ƒr = kN = klxg (2) N
x
The differential work done by the friction forces fr

dA = fr # d r = - fr ds
xg
= - klxg ( - dx) = lg a b xdx
h
(3) (l−x)g
1 - h

(Note that here we have written ds = - dx, because ds is essentially a positive term
and as the length of the chain decreases with time, dx is negative.)
Hence, the sought work done
0
h mgl
A = lg xdx = - (1 - h)h = - 1.3 J
3 1 - h 2
(1 - h)l

1.125 The velocity of the body, t seconds after the beginning of the motion becomes
v = v0 + gt. The power developed by the gravity (mg) at that moment, is
P = mg # v = m (g # v0 + g 2t) = mg ( gt - v0 sin a)
As mg is a constant force, so the average power
A mg # ¢r
6P 7 = =
t t
where ¢r is the net displacement of the body during time of flight.
As, m g ⬜ ¢r, so 6P 7 = 0

1.126 We have wn = v 2 /R = at 2 or v = 1aR t

where t is defined to start from the beginning of motion from rest.


dv
So, wt = = 1aR
dt
1.3 LAW OF CONSERVATION OF ENERGY, MOMENTUM AND ANGULAR MOMENTUM 79

Instantaneous power, P = F # v = m (wt t + wnn) # ( 1aR t t), where t and n are unit
vectors along the direction of tangent (velocity) and normal, respectively.
So, P = mwt 1aR t = maRt

Hence the sought average power


t t

P dt ma Rt dt
3 3
0 0
6P 7 = t
=
t
dt
3
0

maRt 2 maRt
Hence, 6P 7 = =
2t 2

1.127 Let the body m acquire the horizontal velocity v0 along positive x-axis at the point O.
(a) Velocity of the body t seconds after the beginning of the motion,

v = v0 + wt = (v0 - kg t) i (1)

Instantaneous power P = F # v = ( - kmg i) # (v0 - kgt) i = - kmg (v0 - kgt)


From Eq. (1), the time of motion t = v0 >kg.
Hence sought average power during the time of motion
t

- kmg (v0 - kgt)dt


3 kmg v0
0
6P 7 = = - = - 2 W (on substituting values)
t 2

(b) From Fx = mwx


dvx
- kmg = mwx = mvx
dx
or vxdvx = - kgdx = - agxdx

To find v (x), let us integrate the above equation


v x

vxdvx = - ag xdx or, v 2 = v02 - agx 2


3 3
v0 0
80 PART ONE PHYSICAL FUNDAMENTALS OF MECHANICS

Now, P = F # v = - ma xg 2v 02 - a gx 2 (2)

For maximum power,

A 2v02x 2 - lgx 4 B = 0 which yields x =


d v0
dt 12ag
Putting this value of x in Eq. (2), we get
1
Pmax = - mv02 1ag
2

1.128 Centrifugal force of inertia is directed outward along radial line, thus the sought work
r2

mv 2 1r 22 - r 21 ) = 0.20 J (on substituting values)


1
A = mv 2 rdr =
3 2
r1

1.129 Since the springs are connected in series, the combination may be treated as a single
spring of spring constant.
k1k2
So, k =
k1 + k2

From the equation of increment of mechanical energy,


¢T + ¢Uspring = Aext
k1k2
k¢l 2 = Aext or Aext = a b ¢l 2
1 1
0 +
2 2 k1 + k2

1.130 First, let us find the total height of ascent. At the beginning and at the end of the path,
velocity of the body is equal to zero, and therefore the increment of the kinetic energy
of the body is also equal to zero. On the other hand, according to work-energy theorem,
¢T is equal to the algebraic sum of the work A performed by all the forces, i.e. by the
force F and gravity, over this path. However, since ¢T = 0 then A = 0. Taking into ac-
count that the upward direction is assumed to coincide with the positive direction of
the y-axis, we can write
h h

A = (F + mg) # dr = (Fy - mg) dy


3 3
0 0
h

= mg (1 - 2 ay) dy = mgh (1 - ah) = 0 (when h = 1>a)


3
0
1.3 LAW OF CONSERVATION OF ENERGY, MOMENTUM AND ANGULAR MOMENTUM 81

The work performed by the force F over the first half of the ascent is
h>2 h>2
3 mg
AF = Fydy = 2mg (1 - ay) dy =
3 3 4a
0 0

The corresponding increment of the potential energy is


mgh mg
¢U = =
2 2a

, we get Fr = c - 3 + 2 d
dU 2a b
1.131 From the equation Fr = -
dr r r
(a) We have at r = r0, the particle is in equilibrium position, i.e, Fr = 0.
2a
So, r0 =
b
To check whether the position is steady (the position of stable equilibrium), we
have to satisfy
d 2U
7 0
dr 2

= c - 3d
d 2U 6a 2b
We have 2
dr r4 r

Putting the value of r = r0 = 2a /b, we get

d 2U b4
= (as a and b are positive constants)
dr 2 8a 3
d 2U b2
So, = 7 0
dr 2 8a 3
which indicates that the potential energy of the system is minimum, hence this
position is steady.

= c- 3 + 2d
dU 2a b
(b) We have Fr = -
dr r r

dFr
For F to be maximum, = 0
dr
3a -b 3
So, r = and Fr (max) =
b 27a 2
As Fr is negative, the force is attractive.
82 PART ONE PHYSICAL FUNDAMENTALS OF MECHANICS

1.132 (a) We have


0U - 0U
Fx = - = - 2ax and Fy = = - 2by
0x 0y
So, F = 2ax i - 2by i and F = 2 1a2x 2 + b 2y 2
For a central force, r * F = 0.
Here, r * F = (x i + y j) * ( - 2a x i - 2by j)
= - 2 bxy k - 2axy (k) Z 0
Hence the force is not a central force.
(b) As U = ax 2 + by 2
0U - 0U
So, Fx = = - 2 ax and Fy = = - 2by
0x 0y
So, F = 2Fx2 + Fy2 = 24a2x 2 + 4b 2y 2
According to the problem
F = 2 2a 2x 2 + b 2y 2 = C (constant)
C2
or a 2x 2 + b 2y 2 =
2
x2 y2 C2
or 2
+ 2
= = k (say)
b a 2a 2 b 2
Therefore, the surface for which F is constant is an ellipse.
For an equipotential surface U is constant.
So, ax 2 + by 2 = C0 (constant)
x2 y2 C0
or + = = K0 (constant)
2b 2 2a 2 ab
Hence, the equipotential surface is also an ellipse.

1.133 Let us calculate the work performed by the forces of each field over the path from
a certain point 1 (x1, y1) to another point 2 (x2, y2)
x2

(i) dA = F # d r = ay i # d r = ay dx or, A = a ydx


3
x1

(ii) dA = F # dr = (ax i + by j) # d r = ax dx + bydy


1.3 LAW OF CONSERVATION OF ENERGY, MOMENTUM AND ANGULAR MOMENTUM 83

x2 y2

Hence, A = axdx + bydy


3 3
x1 y1

In the first case, the integral depends on the function of type y (x), i.e., on the shape
of the path. Consequently, the first field of force is not potential. In the second case,
both the integrals do not depend on the shape of the path. They are defined only by
the coordinate, of the initial and final points of the path, therefore the second field of
force is potential.

1.134 Let s be the sought distance, then from the equation of increment of mechanical energy.
¢T + ¢Ugr = Afr

a0 - mv02 b + (mg s sin a) = - kmg cos as


1
2
v02>2g
s =
1sin a + k cos a2
or

- km v02
Hence, Aƒr = - kmg cos as =
2(k + tan a)

1.135 Velocity of the body at height h, vh = 22g (H - h), horizontally (from the figure
given in the problem book). Time taken in falling through the distance h is given by
2h
t = (as initial vertical component of the velocity is zero)
A g
2h
Now, s = vh t = 22g (H + h) * = 24(Hh - h 2)
A g
d H
For smax, (Hh - h 2) = 0, which yields h =
ds 2
Putting this value of h in the expression obtained for s, we get
smax = H

1.136 To complete a smooth vertical track of radius R,


the minimum height at which a particle starts, v N
must be equal to 52 R. Let us first prove this. mg
In the figure there is a smooth track and a small h
body is released at height h. We have to find
R
minimum value for h so that the body can com-
plete smooth vertical track of radius R. As the
84 PART ONE PHYSICAL FUNDAMENTALS OF MECHANICS

track is smooth and the body is under the action of only two forces, normal reac-
tion and gravitational pull. The mechanical energy of the block in the gravitational
field is conversed, because normal reaction does zero work where as the gravitation-
al force is conservative. From the conservation of mechanical energy between the
point of release and the upper most point of the circular track

¢T + ¢Ugr = 0

a mv 2 - 0 b - mg (h - 2R) = 0
1
2
which yields v 2 = 2g (h - 2R) (1)
To complete the vertical circular track the condition is that the normal reaction at the
upper most point of the circular track N Ú 0.
From Newtons second law in projection form towards the centre of the track
Fn = mwn (where wn is the normal acceleration)
v2
N + mg = m
R

N = m a - gb
v2
R
As N Ú 0 at upper most point of the circular track

ma - gb Ú 0
v2
So,
R
or v 2 Ú gR (2)
From Eqs. (1) and (2)
2g (h - 2R) Ú gR
5
Hence, h Ú R
2
Thus in our problem, the body could not reach the upper most point of the vertical
track of radius R/2. Let the particle A
leave the track at some point with v
speed v (see figure). Now from ene-
rgy conservation for the body A in the
field of gravity mg
h

mg ch - (1 + sin u) d = mv 2
h 1
2 2 h/2

or v = gh (1 - sin u)
2
(3)
1.3 LAW OF CONSERVATION OF ENERGY, MOMENTUM AND ANGULAR MOMENTUM 85

At the break off point the normal reaction N = 0, so from Newton’s second law at
this point
Fn = mwn

mv 2
or mg sin u =
(h>2)
gh
So, v2 = sin u (4)
2
2 gh
From Eqs. (3) and (4), sin u = and v =
3 A 3
After leaving the track, the body A comes in air and further goes up v' v
and at maximum height of it’s trajectory in air and it’s velocity (say v ¿) π/2−θ
becomes horizontal (Fig.). Hence, the sought velocity of A at this point.

v ¿ = v cos a - u b = v sin u =
p 2 gh θ
2 3A 3

1.137 Let the point of suspension be shifted with velocity vA in the horizontal direction
towards left, then in the rest frame of point of suspension the ball starts with same
velocity horizontally towards right. Let us work in this frame. From Newton’s sec-
ond law in projection form towards the point of suspension at the upper most point
(say B):
mvB2 mv B2
mg + T = or T = - mg (1)
l l
Condition required to complete the vertical circle is that T Ú 0. (2)

mv 2A = mg 12l2 + mvB2
1 1
But,
2 2
So, v 2B = v 2A - 4gl (3)
From Eqs. (1), (2) and (3) vB B

m 1v 2A - 4gl 2 T vC
T = - mg Ú 0 or vA Ú 25gl
l mg
C
Thus vA (min) = 15gl T'
l
From the equation Fn = mwn at point C
vA
m vC2 A
T¿ = (4)
l
86 PART ONE PHYSICAL FUNDAMENTALS OF MECHANICS

Again from energy conservation


1 1
mvA2 = mvC2 + mgl (5)
2 2
From Eqs. (4) and (5)
T = 3mg

1.138 Since the tension is always perpendicular to the velocity vector, the work done by the
tension force will be zero. Hence, according to the work energy theorem, the kinetic
energy or velocity of the disk will remain constant during it’s motion. Hence, the
sought time t = s /v0, where s is the total distance traversed by the small disk during
it’s motion.
Now, at an arbitrary position (see figure)
ds = (l0 - R u) d u
ds
l>R dθ

Rθ) v0
So, s = (l0 - R u) d u θ (l 0–
3 B l0 A
0

l 02 Rl 02 l 02
or s = - =
R 2R 2 2R
l 02
Hence, the required time t =
2Rv 0

Note: It should be clearly understood that the only uncompensated force acting on
the disk A in this case is the tension T , of the thread. It is easy to see that there is no
point here, relative to which the moment of force T is invariable in the process of
motion. Hence conservation of angular momentum is not applicable here.

1.139 Suppose that ¢l is the elongation of the rubber cord. Then, from energy conservation
¢Ugr + ¢Uel = 0 (as ¢T = 0)
1
or - mg (l + ¢l) + k ¢l 2 = 0
2
1
or k¢l 2 - mg ¢l - mgl = 0
2

k
mg  (mg)2 + 4 * mgl
A 2 mg 2kl
or ¢l = = B1 + 1  R
k k A mg
2 *
2
1.3 LAW OF CONSERVATION OF ENERGY, MOMENTUM AND ANGULAR MOMENTUM 87

2kl
Since the value of 1 + is certainly greater than 1, hence negative sign is avoided.
A mg

a1 + 1 + b
mg 2kl
So, ¢l =
k A mg

1.140 When the thread PA is burnt, obviously the speed of the bars will be equal at any
instant of time until it breaks off. Let v be the speed of each block and u be the angle,
which the elongated spring makes with the vertical at the moment when the bar A
breaks off the plane. At this stage the elongation in the spring is
¢l = l0 sec u - l0 = l0 (sec u - 1) (1)
Since the problem is concerned with position and there are no forces other than con-
servative forces, the mechanical energy of the system (both bars + spring) in the
field of gravity is conserved, i.e., ¢T + ¢U = 0.

2a mv 2 b + kl 02 (sec u - 1) 2 - mgl0 tan u = 0


1 1
So, (2)
2 2
From Newton’s second law in projection form along vertical direction:
mg = N + k l0 (sec u - 1) cos u kl0 (secq–1)
N
But, at the moment of break off, N = 0.
Hence, kl0 (sec u - 1) cos u = mg T
kl0 - mg
or cos u = (3)
kl0 mg
5mg
Taking k = , simultaneous solution of Eqs. (2) and (3) yields
l0
19gl0
v = = 1.7 m/s
C 32

1.141 Obviously the elongation in the cord, ¢l = l0 (sec u - 1), at the moment the sliding
first starts and at the moment horizontal projection of spring force equals the limit-
ing friction.
So, k¢l sin u = kN (1)
N
(where k is the elastic constant) l

From Newton’s law in projection form along vertical fr


direction:
k¢l cos u + N = mg
or N = mg - k¢l cos u (2) mg
88 PART ONE PHYSICAL FUNDAMENTALS OF MECHANICS

From Eqs. (1) and (2),


k¢l sin u = k (mg - k¢l cos u)
kmg
or k =
¢l sin u + k¢l cos u
From the equation of the increment of mechanical energy: ¢U + ¢T = Aƒr

a k¢l 2 b = Aƒr
1
or
2
kmg ¢l 2
or = Aƒr
2¢l (sin u + k cos u)
(kmgl0 (sec u - 1)
Thus, Afr = = 0.09 J (on substituting values)
2 (sin u - k cos u)

1.142 Let the deformation in the spring be ¢l, when the rod AB has attained the angular
velocity v. From the second law of motion in projection form Fn = mwn.
mv2l0
k¢l = mv2 (l0 + ¢l ) or ¢l =
k - mv2
1 1
From the energy equation, Aext = mv 2 + k¢l 2
2 2
1 1
= mv2 (l0 + ¢l ) 2 + k¢l 2
2 2
2
mv l0 2 mv2l 20 2
= mv2 al0 + b a b
1 1
+ k
2 k - mv2 2 k - mv2
k l0 h (1 + h)
2
mv 2
On solving Aext = , where h =
2 (1 - h) 2 k

1.143 We know that acceleration of centre of mass of the system is given by the expression.
m1w1 + m2w2
wC =
m1 + m2
Since w1 = - w2
T
(m1 - m2) w1 T
wC = (1) m2 w
m1 + m2
w m1
Now from Newton’s second law F = m w, for bodies m1 and m2, m2g
respectively, m1g
T + m1g = m1w1 (2)
1.3 LAW OF CONSERVATION OF ENERGY, MOMENTUM AND ANGULAR MOMENTUM 89

and T + m2g = m2w2 = - m2w1 (3)


(m1 - m2) g
Solving Eqs. (2) and (3) w1 = (4)
m1 + m2
(m1 - m2) 2 g
Thus, from Eqs. (1) and (4), wC =
( m1 + m2) 2

1.144 As the closed system consisting of two particles m1 and m2 is initial- m2


ly at rest, the centre of mass of the system will remain at rest. Further
as m2 = m1>2, the centre of mass of the system divides the line join-
ing m1 and m2 at all the moments of time in the ratio 1:2. In addi- C
tion to it, the total linear momentum of the system at all the times is
zero. So, p1 = - p2 and therefore the velocities of m1 and m2 are also
directed in opposite sense. Bearing in mind all these things, the m1
sought trajectory is as shown in the figure.

1.145 First of all, it is clear that the chain does not move in the ver- T
tical direction during the uniform rotation. This means that
the vertical component of the tension T balances gravity. As
for the horizontal component of the tension T, it is constant
in magnitude and permanently directed toward the rotation C m ω2ρ
axis. It follows from this that the centre of mass of the chain,
the point C, travels along horizontal circle of radius r (say).
Therefore we have
T cos u = mg and T sin u = mv2 r mg

g tan u
Thus r = = 0.8 cm
v2
mg
and T = = 5 N (on substituting values)
cos u

1.146 (a) Let us draw free body diagram and write Newton’s N
second law in terms of projection along vertical and
horizontal directions, respectively,
l
N cos a - mg + fr sin a = 0 (1)
ƒr cos a - N sin a = mv 2l (2) fr
From Eqs. (1) and (2)
sin a
ƒr cos a - ( - ƒr sin a + mg) = mv2l mg
cos a
90 PART ONE PHYSICAL FUNDAMENTALS OF MECHANICS

ƒr = mg a sin a + cos a b
v2l
So, (3)
g
= 6 N (on substituting values)

(b) For rolling, without sliding,


ƒr … k N
but, N = mg cos a - mv2l sin a

mg a sin a + cos a b … k (mg cos a - m v2 l sin a)


v2l
(using Eq. 3)
g

Rearranging, we get
mv2 l (cos a + k sin a) … (k mg cos a - mg sin a )

Thus, v … 2g (k - tan a)>(1 + k tan a)l = 2 rad>s

1.147 (a) Total kinetic energy in frame K ¿ is


1 1 2
T = m (v - V ) 2 + m2 (v2 - V)
2 1 1 2

This is minimum with respect to variation in V, when

= 0, i. e., m11v1 - V2 + m 1v - V2 = 0
dT
dV 2 2

m1v1 + m2v2
or V = = vC
m1 + m2

Hence, it is the frame of centre of mass in which kinetic energy of a system is


minimum.
(b) Linear momentum of the particle 1 in the K ¿ or C frame
m1m2
p1 = m11v1 - vC2 = 1v1 - v22
'
m1 + m2
m1m2
p1 = m1v1 - v22, where
'
or m= = reduced mass
m1 + m2
p2 = m1v2 - v12
'
Similarly,
So, | p~1 | = | p
~ | = ~p = m v , where v = | v - v |
2 rel rel 1 2
1.3 LAW OF CONSERVATION OF ENERGY, MOMENTUM AND ANGULAR MOMENTUM 91

Now the total kinetic energy of the system in the C frame is

~ p~ 2 p~2 p~ 2
T = T~1 + T~2 = + =
2m1 2m2 2m

~ 1 1 2
Hence, T = m v 2rel = m | v1 - v2 |
2 2

1.148 To find the relationship between the values of the mechanical energy of a system in
the K and C reference frames, let us begin with the kinetic energy T of the system.
'
The velocity of the i-th particle in the K frame may be represented as vi = v i + vC.
Now we can write

T = a mi v 2i = a mi 1vi + vC2 # 1vi + vC2


1 1 ' '
2 2
1 '2 ' 1
= a mi v 1 + vC a mi vi + a mi vC2
2 2
'
Since in the C frame a mi vi = 0, the previous expression takes the form

' 1 '2 ' 1


T = T + m v C = T + mV 2 (since according to the problem vC = V ) (1)
2 2

Since the internal potential energy U of a system depends only on its configuration,
the magnitude U is the same in all reference frames. Adding U to the left and right
hand sides of Eq. (1), we obtain the sought relationship,
' 1
E = E + mV 2
2

' ' '


1.149 As initially U = U = 0, so, E = T. From the solution of problem 1.147(b)
' 1
T = m | v1 - v2 |
2

As v1 ⬜ v2

' 1 m1m2
Thus, T = (v 2 + v 22)
2 m1 + m2 1

1.150 Velocities of masses m1 and m2 after t seconds are, respectively,


v¿1 = v1 + gt and v¿2 = v2 + gt
92 PART ONE PHYSICAL FUNDAMENTALS OF MECHANICS

Hence the final momentum of the system


p = m1v1 + m2v2 = m1v1 + m1v2 + (m1 + m2) gt
= p0 + mgt (where, p0 = m2v1 + m2v2 and m = m1 + m2)
Radius vector, is given by
1
rC = vC t + w t2
2 C
(m1v1 + m2v2)t 1 2 1 m1v1 + m2v2
= + gt = v0t + gt 2, where v0 =
(m1 + m2) 2 2 m1 + m2

1.151 After releasing, bar 2 acquires velocity v2, obtained by the energy conservation:
1 1
m2v 22 = kx 2 or v2 = x 1k>m2
2 2
Thus, the sought velocity of C.M.

0 + m2x 1k>m2 x 2m2k


vC = =
m1 + m2 m1 + m2

1.152 Let us consider both blocks and spring as the physical system. The centre of mass
of the system moves with acceleration a = F /(m1 + m2) towards right. Let us work in
the frame of centre of mass. As this frame is a non-inertial frame (accelerated with re-
spect to the ground) we have to apply a pseudo force m1a towards left on the block
m1 and m2 a towards left on the block m2 .
As the centre of mass is at rest in this
frame, the blocks move in opposite direc- m2a
m1a m1 m2 F
tions and come to instantaneous rest at
some instant. The elongation of the
spring will be maximum or minimum at this instant. Assume that the block m1 is
displaced by the distance x1 and the block m2 through a distance x2 from the ini-
tial positions.
From the equation of increment of mechanical energy in C.M. frame
'
¢T + ¢U = Aext
where Aext also includes the work done by the pseudo forces.
' 1
Here, ¢T = 0, ¢U = k (x1 + x2)2 and
2
m2F m1F m1 F (x1 + x2)
Wext = aF - b x2 + x1 =
m1 + m2 m1 + m2 m1 + m2
1.3 LAW OF CONSERVATION OF ENERGY, MOMENTUM AND ANGULAR MOMENTUM 93

1 m1(x1 + x2)F
or k (x1 + x2)2 =
2 m1 + m2
2m1F
x1 + x2 = 0 or x1 + x2 =
k 1m1 + m22
So,

Hence, the maximum and minimum separations between the blocks are
2m1F
l0 + and l0, respectively.
k (m1 + m2)

Alternate:
Let us link the inertial frame with m1 m2 F
horizontal floor and point the coordi- O x
nate axis as shown in figure. x1 l

At an arbitrary instant of time the separation between the blocks is l and the x
coordinate of the block m1 is x1. From Newton’s second law for the block m1,
d 2x1
m1 = k (l - l0 ) (1)
dt 2
Similarly for block m2,
d 2 (x1 + l )
m2 = F - k (l - l0) (2)
dt 2
Multiplying Eq. (2) by m1 and Eq. (1) by m2 and further subtracting Eq. (1) from
Eq. (2), we get
d 2 (x + l ) d 2 x1
m1m2 - m m
1 2 = m1F = (m1 + m2) k (l - l0 )
dt 2 dt 2
d 2l
or m1m2 = m1F - (m1 + m2) k (l - l0 ) (3)
dt 2
d 2 (l - l0 )
or m1m2 = m1F - (m1 + m2) k (l - l0 )
dt 2
d 2 (l - l0) k (m1 + m2) (l - l0 ) F
or 2
+ = (4)
dt m m
1 2 m2
k (m1 + m2) F
Putting = v2 and = A
m1m2 m2
d 2x
and comparing with differential equation + v2x = A, we get
dt 2
A
l - l0 = + B sin (vt + d) (5)
v2
94 PART ONE PHYSICAL FUNDAMENTALS OF MECHANICS

d (l - l0 )
But at time t = 0, l - l0 = 0 and = 0
dt
So, B v cos (vt + d) |t = 0 = 0 and hence d = p>2.
Therefore, Eq. (5) becomes
A
l - l0 = + B cos vt (6)
vu2
-A
But at t = 0, l - l0 = 0, so, B =
v2
Hence, Eq. (6) becomes
A
l - l0 = (1 - cos vt) (7)
v2

But (l - l0) will be maximum, when cos v t = - 1


Therefore, from Eq. (7)
2A 2Fm1m2
l - l0 = =
v2 km2 (m1 + m2)
2Fm1
or l = l0 +
k (m1 + m2)
Similarly (l - l0) will be minimum when cos v t = 1
Therefore, from Eq. (7)
l - l0 = 0 or l = l0

1.153 (a) The initial compression in the spring ¢l must be such that after burning of the
thread, the upper cube rises to a height that produces a tension in the spring
that is atleast equal to the weight of the lower cube. Actually, the spring will first
go from its compressed state to its natural length and then get elongated beyond
this natural length. Let l be the maximum elongation produced under these cir-
cumstances.
Then kl = mg (1)
Now, from energy conservation,
1 1
k¢l 2 = mg (¢l + l ) + kl 2 (2)
2 2
(because at maximum elongation of the spring, the speed of upper cube becomes
zero).
1.3 LAW OF CONSERVATION OF ENERGY, MOMENTUM AND ANGULAR MOMENTUM 95

From Eqs. (1) and (2), C v


l
2mg¢l 3m 2g 2 3mg - mg B
¢l 2 - - = 0 or ¢l = ,
k k2 k k
l
Therefore, acceptable solution of ¢l equals 3mg>k.
(b) Let v be the velocity of upper cube at the position (say, at 2)
when the lower block breaks off the floor, then from energy
conservation,
1 1
mv 2 = k (¢l 2 - l 2) - mg (l + ¢l )
2 2

a where l = b
mg mg
and ¢l = 7
k k
mg 2
or v 2 = 32 (3)
k
The displacement of C.M. (of two blocks systems) in the interval in which
upper block reaches position 2 is

m . 0 + m (7mg /k + mg/k) 4 mg
¢yC = =
1 2m k
When the upper block reaches position 2, the lower block just leaves the
floor with zero initial velocity. Now treat two blocks system like a single
particle of mass 2m projected vertically upward with the velocity of C.M.
mv + 0 v
vC = =
2m 2
So, the further vertical displacement of C.M. of two block system
vC2 (v>2) 2 4mg
¢yC = = = (using Eq. 3)
2 2g 2g k
Hence, the net displacement of the C.M. of the system, in upward direction
8mg
¢yC = ¢yC + ¢yC =
1 2 k

1.154 Due to ejection of mass from a moving system (which moves due to inertia) in a di-
rection perpendicular to it, the velocity of moving system does not change. The mo-
mentum change being adjusted by the forces on the rails. Hence in our problem ve-
locities of buggies change only due to the entrance of the man coming from the
other buggy. From conservation of linear momentum for the system (buggy 1 + man
coming from buggy 2) in the direction of motion of buggy 1,
Mv1 - mv2 = 0 (1)
96 PART ONE PHYSICAL FUNDAMENTALS OF MECHANICS

Similarly from conservation of linear momentum for the system (buggy 2 + man
coming from buggy 1) in the direction of motion of buggy 2,
Mv2 - mv2 = (M + m)v (2)
Solving Eqs. (1) and (2), we get
mv Mv
v1 = and v2 =
M - m M - m

As v1 c T v and v2 c c v
-mv Mv
So, v1 = and v2 =
(M - m) (M - m)

1.155 From momentum conservation, for the system (rear buggy + man)

(M + m) v0 = m (u + vR) + M vR (1)

From momentum conservation, for the system (front buggy + man coming from rear
buggy)
M v0 + m (u + vR ) = (M + m) vF
M v0 m
So, vF = + (u + vR)
M + m M + m
Putting the value of vR from Eq. (1), we get
mM
vF = v0 + u
(M + m) 2

1.156 (i) Let v1 be the velocity of the buggy after both men jump off simultaneously. For the
closed system (two men + buggy), from the conservation of linear momentum,
M v1 + 2m (u + v1) = 0
- 2m u
or v1 = (1)
M + 2m
(ii) Let v be the velocity of buggy with man, when one man jumps off the buggy. For
the closed system (buggy with one man + other man) from the conservation of
linear momentum:
0 = (M + m) v + m (u + v) (2)
Let v2 be the sought velocity of the buggy when the second man jumps off the
buggy; then from conservation of linear momentum of the system (buggy +
one man):
(M + m)v = Mv2 + m (u + v2) (3)
1.3 LAW OF CONSERVATION OF ENERGY, MOMENTUM AND ANGULAR MOMENTUM 97

Solving Eqs. (2) and (3) we get


m (2M + 3m)u
v2 = (4)
(M + m)(M + 2m)

From Eqs. (1) and (4)


v2 m
= 1 + 7 1
v1 2(M + m)
Hence, v2 7 v1

1.157 The descending part of the chain is in free fall, it has speed v = 12gy }y
at the instant when the chain descended a distance y. The length of the
chain which lands on the floor, during the differential time interval dt
following this instant is vdt. dy v
For the incoming chain element on the floor, from dpy = Fy dt (where
y-axis is directed down)

0 - (l vdt) v = Fydt
or Fy = - lv 2 = - 2lgy

Hence, the force exerted on the falling chain equals lv 2 and is directed upward.
Therefore from third law the force exerted by the falling chain on the table at the
same instant of time becomes lv 2 and is directed downward.
Since a length of chain of weight (lyg) already lies on the table and the table is at rest,
the total force on the floor is (2lyg) + (lyg) = (3lyg) or the weight of a length 3y of
chain.

1.158 Velocity of the ball, with which it hits the slab, v = 12gh.
After first impact v ¿ = ev (upward) but according to the problem
v 1
v¿ =
, so e = (1)
h h
and momentum imparted to this slab equals

mv - ( - mv ¿) = mv (1 + e)
Similarly, velocity of the ball after second impact is
v – = ev ¿ = e 2v
and momentum imparted is
m (v ¿ + v –) = m (1 + e)ev
Again, momentum imparted during third impact is
m (1 + e)e 2v, and so on
98 PART ONE PHYSICAL FUNDAMENTALS OF MECHANICS

Hence, net momentum imparted = mv (1 + e) + mve ¿ (1 + e) + Á


= mv (1 + e) (1 + e + e 2 + Á )
(1 + e)
= mv (from summation of G.P.)
1 - e
= m12ghn(h + 1)>(h - 1)
(1+ 1/h)
= 12gh (using Eq. 1)
(1 - 1/ h)
= 0.2 kg m /s (on substituting values)

1.159 (a) Since the resistance of water is negligibly small the resultant of all external forces
acting on the system “a man and a raft” is equal to zero. This means that the po-
sition of the C.M. of the given system does not change in the process of motion.

i.e. , rC = constant or ¢rC = 0, a mi ¢ri = 0

or m (¢rmM + ¢rM) + M¢rM = 0

m l¿
Thus, m( l ¿ + l ) + M l = 0, or l = -
m + M

(b) As net external force on “man-raft” system is equal to zero, therefore the mo-
mentum of this system does not change.
So, 0 = m [v¿ (t) + v2 (t)] + M v2(t)
m v ¿(t)
or, v2(t) = - (1)
m + M
Thus, the sought force on the raft
d v2(t) Mm d v¿(t)
M = -
dt m + M dt
Note: We may get the result of part (a), if we integrate Eq. (1) over the time of motion
of man or raft.

1.160 The displacement of the C.M. of the system, man (m), ladder (M - m) and the coun-
terweight (M ), is described by radius vector
©mi ¢ri M¢rM + (M –m) ¢r(M - m) + m ¢rm
¢rC = = (1)
©mi 2M
But, ¢rm = - ¢r(M - m)
and, ¢rm = ¢rm (M - m) + ¢r(M - m) (2) m (M – m) + m
1.3 LAW OF CONSERVATION OF ENERGY, MOMENTUM AND ANGULAR MOMENTUM 99

Using Eq. (2) in Eq. (1) we get


ml
¢rC =
2M
Alternate:
Initially all the bodies of the system are at rest, and therefore the increments of lin-
ear momentum of the bodies in their motion are equal to the momentum themselves.
The rope tension is the same both on the left and on the right-hand side, and con-
sequently the momentum of the counter-balancing mass ( p1) and the ladder with the
man ( p2) are equal at any instant of time, i.e., p1 = p2,
or M v1 = m v + (M - m)v2
where v1, v, and v2 are the velocities of the mass, the man, and the ladder, respec-
tively. Taking into account that v2 = - v1 and v = v2 + v ¿ , where v ¿ is the man’s
velocity relative to the ladder, we obtain
v1 = (m>2M)v ¿
On the other hand, the momentum of the whole system

p = p1 + p2 = 2p1 or 2M VC = 2M v1 (1)
where VC is the velocity of the centre of inertia of the system. With allowance made
for Eq. (1) we get
VC = v1 = (m>2M)v ¿
Finally, the sought displacement is

¢rC = VC dt = (m>2M ) v¿dt = (m>2M ) ¢r¿


3 3
1.161 Velocity of cannon as well as that of shell equals 22gl sin a down the inclined plane
taken as the positive x-axis. From the linear impulse momentum theorem in projec-
tion form along x-axis for the system (cannon + shell),
¢px = Fx ¢t
i.e., p cos a - M 12gl sin a = Mg sin a¢ t (as mass of the shell is negligible)

p cos a - M 12gl sin a


or ¢t =
Mg sin a

1.162 From conservation of momentum, for the system “ bullet + body ” along the initial
direction of bullet
mv0
mv0 = (m + M) v or v = (1)
m + M
where v0 is the initial velocity of the bullet and v is combined velocity, after the
collision.
100 PART ONE PHYSICAL FUNDAMENTALS OF MECHANICS

From conservation of mechanical energy of the system (bullet + body) in the field
of gravity:
1
(m + M )v 2 = (M + m)gl (1 - cos u), or, v 2 = 2gl (1 - cos u) (2)
2
(a) From Eqs. (1) and (2),
(m + M ) 2(M + m)
v0 = 12gl (1 - cos u) = 1gl sin (u>2)
m m
2M
v0 ⬵ 2gl sin (u>2)
W
As m M,
m
(b) Fraction of initial kinetic energy turned into heat
1
Ti - Tƒ Tƒ (m + M)v 2
2
= = 1 - = 1 -
Ti Ti 1
mv02
2

⬵ a 1 - b (using Eq. 1)
m m
= 1 -
(m + M) M

1.163 When the disk breaks off the body M, its velocity towards right (along x-axis)
equals the velocity of the body M, and let the disk’s velocity in upward direction
(along y-axis) at that moment be v y¿ .
From conservation of momentum, along x-axis for the system (disk + body)
mv
mv = (m + M) v x¿ or vx¿ = (1)
m + M
And from energy conservation, for the same system in the field of gravity:
1 1 1
mv 2 = (m + M )v x¿ 2 + mv y¿ 2 + mgh¿
2 2 2
where h¿ is the height of break-off point from initial level.
1 1 m 2v 2 1
So, mv 2 = (m + M) + mv y¿ 2 + mgh¿ (using Eq. 1)
2 2 (M + m) 2
mv 2
or v y¿2 = v 2 - - 2gh¿
(m + m)
Also, if h– is the height of the disk from the break-off point, then,
v y¿ 2 = 2gh–
mv 2
So, 2g (h– + h¿) = v 2 -
(M + m)
1.3 LAW OF CONSERVATION OF ENERGY, MOMENTUM AND ANGULAR MOMENTUM 101

Hence, the total height, raised from the initial level is


Mv 2
h¿ + h– =
2g (M + m)

1.164 (a) When the disk slides and comes to the plank, it has a velocity equal to
v = 12gh. Due to friction between the disk and the plank the disk slows down
and after some time the disk moves with the plank with velocity v ¿ (say).
From the momentum conservation for the system (disk + plank) along horizon-
tal towards right:
mv
mv = (m + M) v ¿ or v ¿ =
m + M
Now from the equation of the increment of total mechanical energy of a system:
1 1
(M + m)v ¿ 2 - mv 2 = Afr
2 2
1 m2 v 2 1
or (M + m) - mv 2 = Afr
2 (m + M) 2 2

v c - m d = Afr
1 2 m2
so,
2 M + m

Afr = - a b gh = - mgh
mM
Hence,
m + M

a where m = = reduced mass b .


mM
m + M
(b) We look at the problem from a frame in which the hill is moving (together with
the disk on it) to the right with speed u. Then in this frame the speed of the disk
when it just gets onto the plank is, by the law of addition of velocities,
v = u + 12gh. Similarly the common speed of the plank and the disk when
they move together is
m
v = u + 12gh
m + M

1 1 1
Then as above Afr = (m + M) v 2 - mv 2 - Mu 2
2 2 2

(m + M) e u2 + 2gh f
1 2m m2
= u 12gh +
2 m + M (m + M)2
1 1
- (m + M )u2 - m2u 12gh - mgh
2 2
102 PART ONE PHYSICAL FUNDAMENTALS OF MECHANICS

We see that Afr is independent of u and is in fact just - mgh as in (a). Thus the
result obtained does not depend on the choice of reference frame.
Do note however that it will be incorrect to apply “conservation of energy” for-
mula in the frame in which the hill is moving. The energy carried by the hill is
not negligible in this frame. See also the next problem.
1.165 In a frame moving relative to the Earth, one has to include the kinetic energy of the
Earth as well as Earth’s acceleration to be able to apply conservation of energy to the
problem. In a reference frame falling to the Earth with velocity v0, the stone is initial-
ly going up with velocity v0 and so is the Earth. The final velocity of the stone is
0 = v0 - gt and that of the Earth is v0 + (m/M )gt (M is the mass of the Earth), from
Newton’s third law, where t = time of fall. From conservation of energy

mv 02 + Mv 02 + mgh = M a v0 + v0 b
1 1 1 m 2

2 2 2 M

v0 a m + b = mgh
1 2 m2
Hence,
2 M

Neglecting m /M in comparison with 1, we get


v02 = 2gh or v0 = 12gh

The point in this is that in the Earth’s rest frame the effect of Earth’s acceleration is
of order m>M and can be neglected but in a frame moving with respect to the Earth
the effect of Earth’s acceleration must be kept because it is of order one (i.e., large).

1.166 From conservation of momentum for the closed system “both colliding particles”
m1v1 + m2v2 = (m1 + m2) v

m1v1 + m2v2 1(3i - 2 j ) + 2(4 j - 6k)


or v = = = i + 2j - 4k
m1 + m2 3

Hence, | v | = 11 + 4 + 16 m/s = 4.6 m/s

1.167 For perfectly inelastic collision, in the C.M. frame, final kinetic energy of the colliding
system (both spheres) becomes zero. Hence initial kinetic energy of the system in
C.M. frame completely turns into the internal energy (Q) of the formed body.
1
Hence, Q = Ti = m| v1 - v2 | 2
2
1
Now from energy conservation ¢T = - Q = - m| v1 - v2 |2
2
1.3 LAW OF CONSERVATION OF ENERGY, MOMENTUM AND ANGULAR MOMENTUM 103

In laboratory frame the same result is obtained as


1 (m1v1 + m2v2)
2
' 1 1
¢T = - m1| v1 |2 + m2| v2 |2
2 m1 + m2 2 2
1
= - m| v1 - v2 |2
2

1.168 (a) From conservation of linear momentum


p1 = p1¿ + p2¿ or, p1 - p1¿ = p2¿
As p¿1 ⬜ p1 so, p1 2 + p¿1 2 = p¿2 2 (1)
p12 p¿1 2 p¿2 2
From conservation of kinetic energy = + (2)
2m1 2m1 2m2
p12 p¿1 2 (p12 + p¿12)
Using Eq. (1) in Eq. (2) we get = +
2m1 2m1 2m2
p¿1 2 m2 - m1
which yields = a b (3)
p1 2 m2 + m1
T1 ¿ p¿ 1 2>2m1 p¿ 1 2
So, sought fraction of kinetic energy h = 1 - = 1 - = 1 -
T1 p1 2>2m1 p1 2

m2 - m1 2m1
Hence, h = 1 - a b =
m2 + m1 m1 + m2

(b) For two particles closed system momentum of each particle in their C.M. frame are
always equal and opposite. In C.M. frame the kinetic energy of the two particle
~
system T = p~ 2/2m where m is the reduced mass. In perfectly elastic collision, the
kinetic energy of the system is conserved.
' '
p ¿2 p2 ' '
So, = , which gives p ¿ = p
2m 2m

Being head on collision, both the particles have to keep their motion along the
same straight line before and after the collision. On collision, momentum vector of
each particle has to change due to the reaction force
by the other particle. So only choice left to each of the ∼
p1 ∼
p2
particle is to reverse the direction of its momentum before
after the collision keeping the magnitude constant, i.e., ∼ ∼ after
' ' p′1 p′2
pi¿ = - pi, where i = 1, 2.
104 PART ONE PHYSICAL FUNDAMENTALS OF MECHANICS

The same can be said about the velocity of each particle in the C.M. frame
' '
v ¿i = - vi
' '
but v¿i = VC + v i¿ = VC - vi = VC - (vi - VC ) = 2VC - vi
Hence the velocity of i-th particle after collision.
v¿i = 2 VC - vi (where i = 1, 2)
So velocity of particle 1 just after the collision from above relation is
m1v1 (m1 - m2) v1
v¿1 = 2 VC - v1 = 2 a b - v1 =
m1 + m2 m1 + m2

v ¿1 2 (m1 - m2) 2
Therefore, =
v1 2 (m1 + m2) 2
T1 ¿ v ¿1 2 (m1 - m2)2 4m1m2
Thus sought fraction = 1 - = 1 - = 1 - =
T1 v1 2 (m1 + m2 )2 (m1 + m2) 2

1.169 (a) Being perfectly elastic head on collision, the velocity of i-th particle after collision
v i¿ = 2VC - vi (where i = 1, 2) (see previous problem)
m1v1 (m1 - m2) v1
So, v¿1 = 2 a b - v1 = and (1)
m1 + m2 m1 + m2
m1v1
v¿2 = 2 a b (2)
m1 + m2
According to the problem
v¿2 = - v¿1
m1v1 (m2 - m1) v1
so, 2a b = which gives 2m1 = (m2 - m1)
m1 + m2 m1 + m2
m1 1
Hence, =
m2 3
(b) From conservation of linear momentum in the direction perpendicular to initial
motion direction of striking particle 1 gives
p¿1 sin 30° = p¿2 sin 30°
So, p¿1 = p¿2 (1)
From conservation of linear momentum
p1 = p1¿ + p2¿ or, p1 - p1¿ = p2¿
so, p 21 + p¿1 2 - 2p1 p¿1 cos 30° = p¿2 2 (2)
1.3 LAW OF CONSERVATION OF ENERGY, MOMENTUM AND ANGULAR MOMENTUM 105

On using Eq. (1) in Eq. (2) we get on manipulation


p1 p1
p¿1 = = (3)
2 cos 30° 13
From conservation of kinetic energy
p 21 p 1¿ 2 p 2¿ 2
= +
2m1 2m1 2m2
On using Eq. (1) and Eq. (3), we get on solving
m1
= 2
m2

1.170 At the moment of maximum deformation the velocity of colliding bodies along
their common normal n (here the line joining the mass centres of the balls) must be
equal so
v¿1n = v¿2n = v¿n (say) (1)
t
As the balls are smooth so, velocity of each ball along
v1
their common tangent t remains constant.
45° n

So, v¿ 1t = v1t = v1 sin 45° and v¿ 2t = v2t = 0 (2)

Now, from conservation of linear momentum


v1
mv1 cos 45° = 2m v¿n so, v¿n = (3)
2 12
From energy conservation, gain in internal potential energy is due to loss of kinetic
energy of the system. Initial kinetic energy of the system is the kinetic energy T1 of
striking ball only so, the sought fraction h = 1 - T system
¿ / T1.

1 1
But, T ¿ system = m [v¿1n 2 + v¿1t 2] + m [v 2n
¿ 2 + v¿2t2]
2 2

On using Eqs. (1), (2) and (3), we get

1 3v1 2
T ¿ system = m
2 4

T ¿ system 3v1 2 1
n mv1 2 =
1 3
So, = m
T1 2 4 2 4
3 1
Hence, sought fraction h = 1 - =
4 4
106 PART ONE PHYSICAL FUNDAMENTALS OF MECHANICS

1.171 From the conservation of linear momentum of the shell just before and after its frag-
mentation

3v = v1 + v2 + v3 (1)

where v1, v2 and v3 are the velocities of its fragments.


From the energy conservation

3hv 2 = v12 + v22 + v32 (2)


'
Now v or viC = vi - vC = vi - v (3)

where vC = v = velocity of the C.M. of the fragments the velocity of the shell.
Obviously in the C.M. frame the linear momentum of a system is equal to zero so
' ' '
v1 + v2 + v3 = 0 (4)
Using Eqs. (3) and (4) in Eq. (2), we get

3hv2 = (v + v1)2 + (v + v2)2 + (v - v1 - v2)2 = 3v2 + 2v 12 + 2v 12 + 2 v1 # v2


' ' ' ' ' ' ' '

' ' ' '


or 2 v 12 + 2 v1v 2 cos u + 2 v 22 + 3 (1 - h)v 2 = 0 (5)
' ' ' ' '
If we have had used v2 = - v1 - v3, then Eq. 5 would contain v 3 instead of v2 and
so on. The problem being symmetrical we can look for the maximum of any one.
Obviously it will be the same for each.

' ' '


For v 22 cos 2 u Ú 8 (2 v 22 + 3 (1 - h) v 2) or 6 (h - 1)v 2 Ú (4 - cos 2 u) v 22
' 6(h - 1) '
So, v2 … v or v2(max) = 12(h - 1) v
A 4 - cos2u

Hence, v2(max) = | v + v2 |max = v + 12(h - 1)v = v 11 + 12(h - 1)2 = 1 km/s


'

Thus, owing to the symmetry

v1(max) = v2(max) = v3(max) = v (1 + 12(h - 1)2 = 1 km/s

Alternate:
The maximum velocity can be attained by that fragment (say 1) which has experienced
the reaction force by the rest of two fragment (2 and 3) in the initial motion direction
of the shell. Symmetry of the problem tells us that the velocities of fragments 2 and 3
are same and so is the reaction force exerted by the fragment 2 and 3 on fragment 1.
Let v¿ is the velocity of each of the fragments 2 and 3 and v1 ¿ of fragment 1 after the
collision.
1.3 LAW OF CONSERVATION OF ENERGY, MOMENTUM AND ANGULAR MOMENTUM 107

From conservation of linear momentum


3mv = mv1 ¿ - 2 (mv¿)
3v = v1 ¿ - 2v ¿ (1)
From conservation of energy

(3m) v 2 = mv1¿ 2 + a mv ¿ 2 b
h 1 1
2 2 2
So, h 3v 2 = v1¿ 2 + v 2 (2)
Solving Eqs. (1) and (2), we get
v1 ¿ = v 11 + 12(h - 1)2 = 1 km/s (on substituting values)

1.172 From the conservation of momentum


mv1 = mv 1¿ + mv 2¿ or, v1 = v 1¿ + v 2¿ (1)
From the given condition,
T - T¿ T¿ v ¿ 21 + v ¿ 22
h = = 1 - = 1 -
T T v1 2
or v ¿ 21 + v ¿ 22 = (1 - h) v 12 (2)
From Eqs. (1) and (2)
v ¿ 12 + (v1 - v 1¿ ) 2 = (1 - h) v 12
or 2v ¿ 12 - 2v 1¿ v1 + hv 12 = 0

2v1  14v 12 - 8h v 12
So, v¿1 =
4
= C v1  1v1 2 - 2h v 12 D = v1 A 1  11 - 2h B
1 1
2 2
Positive sign gives the velocity of the second particle which lies ahead. The negative
sign is correct for v1. So, v 1¿ = 1 /2 v1 11 - 11 - 2h2 = 5 m/s will continue moving
in the same direction.
Note that v 1¿ = 0 if h = 0.

Alternate:
As internal energy is independent of reference frame, so loss in kinetic energy of the
system in the C.M. frame is the same.
' '
So, T - T¿ = T - T¿
'
T¿ T¿
a1 - b = a1 - b
T
or ' '
T T T
108 PART ONE PHYSICAL FUNDAMENTALS OF MECHANICS

T¿
Using a1 - b = h, T = mv 2, and T = mv 2rel = a b v 2, where m is mass of
1 ' 1 1 m
'
T 2 2 2 2
each particle, we get
T¿
a1 - ' b = 2h
T
Now, according to the problem
' ' '
T - T ¿ = 2hT
' ' '
p2 p ¿2 p2
- = 2h
2m 2m 2m
' '
which yields p ¿ = p 1(1 - 2h)

Being head-on collision, final momentum in C.M. frame of any particle should be re-
versed, so
' '
pi¿ = pi 1(1 - 2h) (where i = 1, 2)
' '
or v i¿ = - vi 1(1 - 2h),
' '
But, vi¿ = vi ¿ + vC = - vi 1(1 - 2h) + vC = - (vi - vC) 1(1 - 2h) + vC
Thus, vi¿ = vc [1 + 11 - 2h] - vi 11 - 2h
mv v
Using vC = = (where v is the velocity of striking particle)
2m 2
v
vi = (1 - 11 - 2h)
2
1.173 From conservation of linear momentum in the initial motion direction of striking par-
ticle 1 and perpendicular to it gives
mv1 = Mv 2¿ cos u and mv 1¿ = Mv 2¿ sin u , respectively
Thus, v1 ¿ = v1 tan u
T ¿system - Tsystem
Percentage change in the kinetic energy of the system = * 100
Tsystem
T ¿system
= a - 1 b * 100
Tsystem

mv1 2
mv12 tan 2 u + M a b
1 1

- 1 T * 100 = a tan2 u + sec 2 u - 1 b * 100


2 2 M cos u m
= D
1 M
mv 2
2 1
Putting the values of u and m /M , we get % of change in kinetic energy = - 40%.
1.3 LAW OF CONSERVATION OF ENERGY, MOMENTUM AND ANGULAR MOMENTUM 109

1.174 (a) Let the particles m1 and m2 move with velocities v1 and v2, respectively. On the
basis of solution of problem 1.147(b):
'
p = mvrel = m| v1 - v2 |
As v1 ⬜ v2
' m1m2
so, p = m 2v12 + v 22 where m =
m1 + m2
(b) Again from solution of problem 1.147(b):
' 1 2 1
T = mv = m| v1 - v2 | 2
2 rel 2
T = m 1v12 + v222
' 1
So,
2
1.175 From conservation of momentum
p1 = p¿1 + p2 or, p1 - p¿1 = p2
So, p12 - 2 p1p1¿ cos u1 + p1¿ 2 = p2¿ 2, where u is the angle between p1 and p1¿.
From conservation of energy
p12 p1¿ 2 p2¿ 2
= +
2m1 2m1 2m2
Eliminating p2 ¿ we get
m2 m2
0 = p1¿ 2 a 1 + b - 2p1¿p1 cos u1 + p1 2 a1 - b
m1 m1

This quadratic equation for p1¿ has a real solution in terms of p1 and cos u1 only if
m22
4 cos 2 u1 Ú 4 a 1 - b
m12
m 22 m2
or sin 2 u1 … or, sin u1 Ú -
m12 m1
This clearly implies (since only + sign makes sense) that
m2
sin u1 max =
m1
Alternate:
The solution of the problem becomes standard in the frame of C.M., which is moving
with velocity vC = m1v1 /(m1 + m2) in the frame of laboratory. In the frame of C.M., the
momenta of the particles must always be equal and opposite.
' '
So, p1 = p2,
110 PART ONE PHYSICAL FUNDAMENTALS OF MECHANICS

' ' '


but | p1 | = | p2 | = p = m vrel = m v1 (where m = m1m2 /(m1 + m2) is the reduced
mass of the system).
'
The initial kinetic energy of the system in the frame of C.M. Ti = p' 2 / 2m.
As the collision is perfectly elastic and the reduced mass the system is constant.
' '
' ' p2 p2
Ti = Tƒ or = C' C
2m 2m q
' '
Hence, p = p¿ D
q1 q~
A m1vc B
In the frame of laboratyry from the conservation q (1max) O m2vc
of linear momentum m1 > m2

p1¿ + p2¿ = p1 = (m1 + m2 ) vC

Now let us draw the so-called vector diagram (see figure) of momenta. First we de-
pict the vector p1 as the section AB and the p1 and p2 each of which represents ac-
cording to
'
p1¿ = p1¿ + m1vC

'
and p2¿ = p2¿ + m2vC
'
Note that this diagram is valid regardless of the angle u . The point C (centre of
'
mass) therefore can be located only on the circle of radius p having it’s centre at
the point O, which divides the section AB into two parts in the ratio AO : OB =
m1: m2. In our case, this circle passes through the point B, the end point of vector
' '
p1 since the section OB = m2vC = mv1 = p. This circle is the locus of all possible
locations of the apex C of the momenta triangle ABC, whose side AC and CB rep-
resent the possible momentum of particles p1¿ and p2¿ after the collision in the
frame of laboratory.
Depending on the particle mass ratio (m1 = m2) the point A, the beginning of the
vector p1 can be located inside the given circle, on it, or outside it. In our case point
A will lie outside the circle and it is also another intersting fact that the particle m1
can be scattered by the same angle u1, where it possesses the momentum AC or AD
(figure) The same is true for particle m2.The maximum scattering in the laboratory
u1 max corresponds to the case when the velocity vector in the frame of laboratory
become a tangent (AC ¿ ) to the circle (see figure).
It then follows that
(m1m2>m1 + m2) v1
'
OC ¿ p m2
sin u1 max = = = =
AO m1vC (m 1 >m1 + m2) v1
2 m1

Note: It is clear that um is limited to p>2 for m1 = m2, for the case m2 7 m1, point A
will be inside the sphere, and all angles of scattering from 0 to p>2 are permitted.
1.3 LAW OF CONSERVATION OF ENERGY, MOMENTUM AND ANGULAR MOMENTUM 111

1.176 From the symmetry of the problem, the velocity of the disk A will be directed either
in the initial direction or opposite to it just after the impact. Let the velocity of the
disk A after the collision be v ¿ and be directed towards right after the collision. It is
also clear from the symmetry of the problem that the disks B and C have equal speed
(say v – ) in the directions shown. From the condition of the problem,
d
h
so, sin u = 24 - h2>2
2 h
cos u = = (1)
d 2

For the three disk system, from the conservation of linear momentum in the symme-
try direction (towards right)
mv = 2mv – sin u + mv ¿ or v = 2v – sin u + v ¿ (2)

From the definition of the coefficient of restitution, we have for disks A , B and C
v – - v ¿ sin u
e =
v sin u - 0
B v"
But e = 1, for perfectly elastic collision.
A
So, v sin u = v– - v ¿ sin u (3)
v
From Eqs. (2) and (3) v' d

v (1 - 2 sin2 u) d
v¿ =
(1 + 2 sin2 u) v"
C
- 2)
v (h2
= (using Eq. 1)
6 - h2
Hence, we have,
v (h 2 - 2)
v¿ =
6 - h2
Therefore, the disk A will recoil if h 6 12 and stop if h = 12.
Note:One can write the equations of momentum conservation along the direction
perpendicular to the initial direction of disk A and the conservation of kinetic ener-
gy instead of the equation of restitution.

1.177 (a) Let a molecule come with velocity v1 to strike another stationary molecule an just
after collision their velocities become v1¿ and v ¿2, respectively. As the mass of each
molecule is same, conservation of linear momentum and conservation of kinetic
energy for the system (both molecules), respectively, gives

v1 = v 1¿ + v 2¿
and v12 = v 1¿ 2 + v 2¿ 2
112 PART ONE PHYSICAL FUNDAMENTALS OF MECHANICS

From the property of vector addition it is obvious from the obtained equations that

v1 ⬜ v2 or v1 # v2 = 0

(b) Due to the loss of kinetic energy in inelastic collision v12 7 v 1¿ 2 + v 2¿2 so,
v1 # v2 7 0 and therefore angle of divergence 6 90°.

1.178 Suppose that at time t, the rocket has the mass m and the velocity v, relative to the
reference frame, employed. Now consider the inertial frame moving with the veloc-
ity that the rocket has at the given moment. In this reference frame, the momentum
increment that the rocket and ejected gas system acquires during time dt is,
d p = md v + mdt u = F dt
dv
or m = F - mu
dt
or m w = F - mu

1.179 According to the question, F = 0 and m = - dm>dt so the equation for this system
becomes,

dv dm
m = u
dt dt

As d v c T u, so, m dv = - udm.

Integrating within the limits


v 0
1 dm v m0
dv = - or = ln
u 3 3 m u m
0 m0
m0
Thus, v = u ln
m

As d v c T u, so in vector form v = - u ln m0 /m .

1.180 According to the question, F (external force) = 0

dv dm
So, m = u
dt dt
As, dvc T u
So, in scalar form mdv = - udm
wdt dm
or = -
u m
1.3 LAW OF CONSERVATION OF ENERGY, MOMENTUM AND ANGULAR MOMENTUM 113

Integrating within the limit for m (t)


m
wt dm v m
= - or, = - ln
u 3 m u m0
m0

Hence, m = m0 e – (wt>u)

1.181 As F = 0, from the equation of dynamics of a body with variable mass,


dv dm dm
m = u or d v = u (1)
dt dt m
Now dv c T u and since u ⬜ v, we must have | d v | = v0d a (because v0 is constant),
where da is the angle by which the spaceship turns in time dt.

dm u dm
So, - u = v0da or da = -
m v0 m
m
m0
ln a b
u dm u
or a = - =
v0 3 m v0 m
m0

dm
1.182 We have = - m or dm = - mdt
dt
m t

Integrating dm = - m dt or, m = m0 - mt
3 3
m0 0

As u = 0 so, from the equation of variable mass system


dv dv F
(m0 - mt) = F or = w =
dt dt (m0 - mt)
v t
dt
or dv = F
3 3 (m0 - mt)
0 0
m0
b
F
Hence, v = lna
m m0 - mt

1.183 Let the car be moving in a reference frame to which the hopper is fixed and at any
instant of time, let its mass be m and velocity v.
Then from the general equation, for variable mass system
dv dm
m = F + u
dt dt
114 PART ONE PHYSICAL FUNDAMENTALS OF MECHANICS

We write the equation, for our system as,


dv dm
m = F - v as, u = - v
dt dt
d
So, (m v) = F
dt
Ft
and v = on integration
m
But m = m0 + mt
Ft
so, v =
m0 a 1 + b
mt
m0
Thus, the sought acceleration is
dv F
w = =
m0 a 1 + b
dt mt 2
m0

1.184 Let the length of the chain inside the smooth horizontal tube at an arbitrary instant
be x. From the equation,
dm
mw = F + u
dt
as u = 0, F c c w, for the chain inside the tube
m
lxw = T , where l = (1)
l
Similarly for the overhanging part, A T
u = 0 x
T
Thus, mw = F
h hg
or l hw = lhg - T (2)
From Eqs. (1) and (2)
dv
l(x + h) w = l hg or, (x + h) v = hg
ds
dv
or (x + h) v = gh
( - dx)
(As the length of the chain inside the tube decreases with time, ds = - dx.)
1.3 LAW OF CONSERVATION OF ENERGY, MOMENTUM AND ANGULAR MOMENTUM 115

dx
or vdv = - gh
x + h
v 0
dx
Integrating, vdv = - gh
3 3 x + h
0 (l - h)

= gh ln a b 2gh ln a b
v2 l l
or or v =
2 h A h

1.185 The angular momentum of the particle relative to point O is given as function of a
time as M = a + bt 2.
So, force moment relative to point O, is given by

dM
= 2bt
N =
dt
As force moment N = 2bt , so vector N coincides with vector b.
α
Let us depict the vectors N and M at an arbitrary instant of time N
t, when angle between M and N is a (see figure). It is obvious
from the figure that tan a = a /bt 2, therefore a = 45°, the time a M
t = t0 = 1a /b .
bt 2
Thus, N = 2b 1a /b , when vector N forms angle a = 45° with vector M.

Alternate:
Let the angle between M and N, a = 45° at t = t0.

1 M#N (a + bt02) # (2bt0)


Then = =
12 |M| |N| 2a 2 + b 2t 40 2bt0
2b 2t03 bt 20
= =
2a 2 + b 2t0 4 2bt0 2a 2 + b 2t0 4

a
So, 2b 2t0 4 = a 2 + b 2t04 or t0 = (as t0 cannot be negative)
Ab

a
Hence, N = 2b
Ab
116 PART ONE PHYSICAL FUNDAMENTALS OF MECHANICS

M(t) = r * p = a v0t + gt b * m (v0 + gt)


1 2
1.186
2

= mv0 gt 2 sin a + a b ( - k) + mv0gt 2 sin a + ab (k)


p 1 p
2 2 2
1
= mv0 gt 2 cos a(–k)
2
mv0 gt 2 cos a
Thus M(t) =
2
Thus, angular momentum at maximum height, i.e., at t
t v0 sin a v0
= mg
2 g r
y
mv 3
M a b = a 2g0 b sin 2 a cos a
t
2 α
O x
= 37 kg m 2/s
Alternate:
t t

M(0) = 0 so , M(t) = N dt = (r * mg)


3 3
0 0
t

c a v0t + gt b * mg d dt = (v0 * mg)


1 2 t2
=
3 2 2
0
1.187 (a) The disk experiences gravity, the force of reaction of the horizontal surface, and
the force R of reaction of the wall at the moment of the impact against it. The
first two forces counter-balance each y
other, leaving only the force R. It’s A
directed normally
moment relative to any point of the upward
line along which the vector R acts or
O n
along normal to the wall is equal to R
zero and therefore the angular mo-
mentum of the disk relative to any of l
these points does not change in the
given process. O'
(b) During the course of collision with wall the position of disk is same and is
equal to r00¿ . Obviously the increment in momentum
¢M = r00¿ * ¢p = 2mv l cos a n
Here, | ¢M | = 2mv l cos a
1.3 LAW OF CONSERVATION OF ENERGY, MOMENTUM AND ANGULAR MOMENTUM 117

1.188 (a) The ball is under the influence of forces T and mg at all the moments of time,
while moving along a horizontal circle. Obviously the vertical component of T
balances mg and so the net moment of these two about any point becomes
zero. The horizontal component of T, which provides the centripetal accelera-
tion to ball is already directed toward the centre (C ) of the horizontal circle,
thus its moment about the point C equals zero at all the moments of time.
Hence the net moment of the force acting on the ball about point C equals
zero and that’s why the angular momentum of the ball is conserved about the
horizontal circle.
(b) Let a be the angle which the thread forms with the vertical . M
Now from equation of particle dynamics Mi
T cos a = mg and T sin a = mv 2l sin a Mf
O
Hence on solving, T
g
cos a = (1) C
v 2l
mg
As | M | is constant in magnitude, so from the figure

| ¢M | = 2M cos a
where M = | Mi | = | Mƒ |
= | rbo * m v | = mvl (as rbo ⬜ v)
Thus, | ¢M | = 2mvl cos a = 2 mvl 2 sin a cos a

1 - a b
2mgl g 2
= (using Eq. 1)
v C v 2t

1.189 During the free fall, time t = 22h /g, the reference point O moves in horizontal di-
rection by the distance Vt. In the translating frame as Mi(O) = 0, so

¢M = Mƒ = rf * m vƒ

= a gt - Vt b * m (gt - V)
1 2
2
1
= m (g * V) t 2
2

mgV a b (using the value of t and taking into account V⬜ g)


1 2h
So, | Mƒ | =
2 g

Hence, | ¢M | = mVh
118 PART ONE PHYSICAL FUNDAMENTALS OF MECHANICS

1.190 The Coriolis force is 2m (v ¿ * V). Here V is along the z-axis (vertical). The mov-
ing disk is moving with velocity v0 which is constant, the motion is along the x-axis,
then the Coriolis force is along y-axis and has the magnitude 2mv0v. At time t, the
distance of the centre of moving disk from O is v0t (along x-axis). Thus the torque
N due to the Coriolis force is N = 2m v0v # v0t along the z-axis. Hence, equating this
to dM /dt, we get

dM
= 2mv 02 vt
dt
or
M = mv 02 vt 2 + constant

The constant is irrelevant and may be put equal to zero if the disk is originally set
in motion from the point O.
This discussion is approximate. The Coriolis force which causes the disk to swerve
from straight line motion and thus causes deviation from the above formula will be
substantial for large t.
#
1.191 If r = radial velocity of the particle then the total energy of the particle at any in-
stant is
1 #2 M2
mr + + kr 2 = E (1)
2 2mr 2
where the second term is the kinetic energy of angular motion about the centre O.
#
Then the extreme values of r are determined by r = 0 and solving the resulting
quadratic equation

M2
k (r 2) 2 - Er 2 + = 0
2m

E  2E 2 - 2M 2k /m
we get r2 =
2k

From this we see that

E = k1r 12 + r 122

mv 22 + kr22 = k1r12 + r222


1
So,
2
2kr1 2
Hence, m =
v22
1.3 LAW OF CONSERVATION OF ENERGY, MOMENTUM AND ANGULAR MOMENTUM 119

Note: Eq. (1) can be derived from the standard expression for kinetic energy and
angular momentum in plane polar coordinates:
1 #2 1 # #
T = mr + mr 2 u2 and M = mr 2u, respectively
2 2

1.192 The swinging sphere experiences two forces: the gravitational force and the tension
of the thread. Now, it is clear from the condition, given in the problem, that the mo-
ment of these forces about the vertical axis, passing through the point of suspension
NZ = 0. Consequently, the angular momentum MZ of the sphere relative to the given
axis (z) is constant.

Thus, mv0 (l sin u) = mvl (1)

where m is the mass of sphere and v is its velocity in the position, when the thread
forms an angle p>2 with the vertical. Mechanical energy is also conserved, as the
sphere is under the influence of only one other force, i.e. tension, which does not
perform any work, as it is always perpendicular to the velocity.
1 1
So, mv 02 + mgl cos u = mv 2 (2)
2 2
From Eqs. (1) and (2), we get
v0 = 22gl>cos u

1.193 Forces, acting on the mass m are shown in the figure. As N = m g, the net torque
of these two forces about any fixed point must be equal to zero. Tension T, acting
on the mass m is a central force, which is always directed towards the centre O.
Hence the moment of force T is also zero about the point O and therefore the an-
gular momentum of the particle m is conserved about O.
#
So, mr 2u = mr02 v0
# v0r 02 N
or v = u=
r2
T
O
From Fr = mar
$ #
- T = m ( r - r u 2) mg

As the
# thread is pulled$ with constant speed F
so, r = constant and r = 0 (where dot stands
for time derivative).
#
Thus T = F = mr u2
120 PART ONE PHYSICAL FUNDAMENTALS OF MECHANICS

Hence, the sought tension is


# v0r 02 2 m v 02r04
F = mr u 2 = mr a 2 b =
r r3

1.194 On the given system the weight of the body m is the only force whose moment is
effective about the axis of pulley. Let us take the sense of V of the pulley at an ar-
bitrary instant as the positive sense of axis of rotation (z-axis).

As Mz(0) = 0 so ¢Mz = Mz(t) = Nzdt


3
t

So, Mz(t) = mgR dt = mgRt


3
0

1.195 Let the point of contact of sphere at initial moment N


(t = 0) be at O. At an arbitrary moment, the forces r O
acting on the sphere are shown in the figure. fr
As N = mg cos a and both (N and mg cos a) pass mg cos a
mg sin a
through the same line, their net torque about any a
point is zero. The force of static friction passes
through the point O, thus its moment about point x
O becomes zero. Hence mg sin a is the only force
which has effective torque about point O, and is given by | N | = mgR sin a normal-
ly emerging from the plane of figure.

As M(t = 0) = 0 so ¢M = M(t) = N dt
3
Hence, M (t) = Nt = mg R sin at

1.196 Let position vectors of the particles of the system be ri and ri¿ with respect to the
points O and O¿ respectively, then we have

ri = ri¿ + r0 (1)
(where r0 is the radius vector of O¿ with respect to O ).
Now, the angular momentum of the system relative to the point O can be written
as follows:

M = ©ri * pi = © ( ri¿ * pi ) + © (r0 * pi ) (using Eq. 1)


3

or M = M¿ + (r0 * p) (where p = ©pi ) (2)


1.3 LAW OF CONSERVATION OF ENERGY, MOMENTUM AND ANGULAR MOMENTUM 121

From Eq. (2), if the total linear momentum of the system, p = 0, then its an-
gular momentum does not depend on the choice of the point O .
Note In the C.M. frame, the system of particles, as whole is at rest.

1.197 On the basis of solution of problem 1.196. we have concluded that; “in the C.M.
frame, the angular momentum of system of particles is independent of the choice of
the point, relative to which it is determined” and in accordance with the problem
this is denoted by M.
We denote the angular momentum of the system of particles, '
relative to the point O,
by M. Since the internal and proper angular momentum M, in the C.M. frame, does
not depend on the choice of the point O¿ , this point may be taken coincident with
the point O of the K-frame, at a given moment of time. Then at that moment, the ra-
dius vectors of all the particles, in both reference frames, are equal (ri¿ = ri ) and the
velocities are related by the equation
'
vi = vi + vC

where vC is the velocity of C.M. frame, relative to the K-frame. Consequently, we may
write
'
M = ©mi(ri * vi ) = ©mi (ri * vi) + ©mi(ri * vC )

'
or M = M + m(rC * vC ), as ©mi ri = m rC (where m = ©mi )
' '
or M = M + (rC * m vC ) = M + (rC * p)

1.198 Taking striking ball as particle 1 and upper ball or the dumbbell as particle 2 the
velovity acquired by particle 2 on collsion, from the formula
v0 1
vi¿ = 2vC - vi (see solution of problem 1.168) m m /2

m v0
b - 0 =
4 y C
v2¿ = 2a v
m + m /2 3 0
x
m /2
To find the proper angular momentum of the dumbbell, let 2
us denote upper ball of the dumbbell as particle 1 and
lower ball as particle 2.
Then proper angular momentum vector is
' ' '
M = r1C * m1v1C + r2C * m2v2C = r1C * p1 + r2C * p2
122 PART ONE PHYSICAL FUNDAMENTALS OF MECHANICS

' '
In C.M. frame net linear momentum of any system is zero, so, p1 = - p2.
' ' '
So, M = (r1C - r2C) * p1 = r12 * p1
'
We have p1 = m (v1 - v2) (see solution of problem 1.147)
m 4 v0 mv0
= a i - 0b = i
4 3 3
mv0 mv0 l
M = [r12 * p1] = c l j * id =
' '
Hence, ( - k)
3 3
' mv0 l
Hence, M =
3

1.199 In the C.M. frame of the system (both the disks + spring), the linear momentum of
' '
the disks are related by the relation, p1 = - p2, at all the moments of time.
' ' '
Where, p1 = p2 = p = mvrel
And the total kinetic energy of the system,
1
T = 2
mv rel [see solution of problem 1.147(b)]
2
Bearing in mind that at the moment of maximum deformation of the spring, the pr-
ojection of vrel along the length of the spring becomes zero, i.e., vrel (x) = 0 .
The conservation of mechanical energy of the considered system in the C.M. frame gives

a b v 02 = kx 2 + a b vrel2 (y)
1 m 1 1 m
(1)
2 2 2 2 2
Now from the conservation of angular momentum of the system about the C.M.,
1 l0 l0 + x m
a b a v0 b = 2 a b
m
v
2 2 2 2 2 rel (y)
v0l0 x -1
= v0 a1 + b = v0 a 1 - b ( as x V l0) (2)
x
or vrel (y) =
(l0 + x) l0 l0

mv 20 c 1 - a 1 - b d = kx 2
1 x 2
Using Eq. (2) in Eq. (1),
2 l0

mv 02 c 1 - a1 - + 2 b d = kx 2
1 2x x2
or
2 l0 l0
2
mv 0 x
or = kx 2 [neglecting x 2>l0 2 ]
l0
mv 02
As x Z 0, thus x =
kl0
1.4 UNIVERSAL GRAVITATION 123

1.4 Universal Gravitation

Mv 2 gMMS gMS
1.200 We have = 2
or r = 2 (here MS is the mass of the Sun)
r r v
v v v3
v = = =
gMS >v 2
Thus
r gMS
2pgMS 2p * 6.67 * 10-11 * 1.97 * 1030
So, T = = = 1.94 * 107 s = 225 days
v3 (34.9 * 103)3
(The answer is incorrectly written in terms of the planetary mass M in answer sheet.)

1.201 For any planet


gMMS gMS
MR v2 = or v =
R2 A R3

= 2pR 3>2> 2gMS


2p
So, T =
v
TJ RJ 3>2
(a) Thus, = a b
TE RE
RJ TJ
= a b
2>3
So, = (12)2>3 = 5.24
RE TE

gMS 1gMS
RJ = aT b
2>3
(b) V 2J = and
RJ 2p

1gMS22>3 12p22>3 2pgMS 2>3


So, V 2J = or VJ = a b
T 2>3 T

where T = 12 years, MS = mass of the Sun.

Putting the values, we get VJ = 12.97 km/s


VJ2 2pgMS
= a b * a b
2>3 2p 2>3
Acceleration =
RJ T T 1gMS

= a b (gMs )1/3
2p 4>3
T
= 2.15 * 10-4 km/s2
124 PART ONE PHYSICAL FUNDAMENTALS OF MECHANICS

1.202 Semi-major axis = (r + R)>2. It is sufficient to consider the motion be along a circle
of semi-major axis (r + R)/2 for T does not depend on eccentricity.
3>2
[2 p (r + R )/2 ]
Hence, T =
1gMS

= p 21r + R23>2gMS (again MS is the mass of the Sun)

1.203 We can think of the body as moving in a very elongated orbit of maximum distance
R and minimum distance 0 so semi major axis = R>2. Hence if t is the time of fall
then

a b = a b or t2 =
2t 2 R/2 3 T2
T R 32

or t = T>4 22 = 365>412 = 64.5 days

1.204 T = 2p R 3>2> 1r Ms . If the distances are scaled down, R 3>2 decreases by a factor
h3>2 and so does Ms . Hence T does not change.

1.205 The double star can be replaced by a single star of mass m1m2/(m1 + m2) moving
about the centre of mass subjected to the force gm1m2>r 2. Then

2pr 3>2
T=
m1m2
gm1m2n
A m1 + m2

2pr 3>2
=
1gM

T
So, r 3>2 = 1gM
2p

r = a b 1gM21>3 = 32gM (T>2 p) 2


T 2>3
or
2p

1.206 (a) The gravitational potential due to m1 at the point of location of m2

q q
gm1 gm1
V2 = G .dr = - dx = -
3 3 x2 r
r r
1.4 UNIVERSAL GRAVITATION 125

gm1m2
So, U21 = m2V2 = -
r
gm1m2 r m2
Similarly, U12 = - m1 x
r x
dx
gm1m2
Hence, U12 = U21 = U = -
r

(b) Choose the location of the point mass as the origin. Then the potential energy dU
of an element of mass dM = M /l dx of the rod in the field of the point mass is
M 1
dU = - gm
dx
l x
where x is the distance between the element and the point. (Note that the rod
and the point mass are on a straight l m
line.) Then if a is the distance of the x
x
nearer end of the rod from the point
dx
mass,
a+l

ln a 1 + b
mM dx M l
U = -g = - gm
l 3 x l a
a
The force of interaction is
0U
a- b = -
mM 1 l gmM
F = - = g *
0a l l a 2
a 1a + l 2
1 +
a
Minus sign means attraction.
1.207 As the planet is under central force (gravitational interaction), its angular momentum
is conserved about the Sun (which is situated at one of the focii of the ellipse).
v 22 r 21
So, mv1r1 = mv2r2 or v 12 = (1)
r 21
From the conservation of mechanical energy of the system (Sun planet),
gMSm 1 gMSm 1
- + mv12 = - + mv22
r1 2 r2 2
gMS 2 gMS
1 2 r2
v2 2 = - a b + v 22
1
or - + (using Eq. 1)
r1 2 r2 r2 2

Thus, v 2 = 22g MS r 1>r 2 (r 1 + r 2) (2)

Hence, M = mv 2r 2 = m 22gMSr 1r 2>(r 1 + r 2)


126 PART ONE PHYSICAL FUNDAMENTALS OF MECHANICS

1.208 From the previous problem, if r 1, r 2 are the maximum and minimum distances from
the Sun to the planet and v1, v2 are the corresponding velocities, then, say,
1 gmMS
E = mv 22 -
2 r2
gmMS
# r1 gmMS - gmMS gmMS
= - = = (using Eq. 2 of problem 1.207)
r 1 + r2 r 2 r2 r1 + r2 2a

where 2a = major axis = r 1 + r 2.


The same result can also be obtained directly by writing an equation analogous to
Eq. (1) of problem 1.191
1 M2 gmMS
E = mr 2 - 2
-
2 2mr r
(Here M is angular momentum of the planet and m is its mass.) For extreme posi-
.
tion r = 0 and we get the quadratic equation
M2
Er 2 + gm MS r - = 0
2m
The sum of the two roots of this equation is
gmMS
r1 + r2 = - = 2
E
gmMS
Thus, E = - = constant
2a

1.209 From the conservation of angular momentum about the Sun.


mv0r0 sin a = mv 1r 1 = mv 2 or v 1r 1 = v 2r 2 = v0r 2 sin a (1)
From conservation of mechanical energy,
1 gMSm 1 gMSm
mv02 - = mv 21 -
2 r0 2 r1

v 20 gMS v 20 r 20 sin 2 a gMS


or - = - (using Eq. 1)
2 r0 2r 21 r1

2gMS
or av 20 - b r 21 + 2gMS r1 - v02 r02 sin a = 0
r0

2gMS
- 2gMS  4g 2MS2 + 4 1v02r 20 sin2 a2 a v 20 - b
C r0
So, r1 =
2gMS
2 a v 20 - b
r0
1.4 UNIVERSAL GRAVITATION 127

v 20 r 20 sin2 a v 20
a b
2
r0 C 1  21 - (2 - h) sin2 a D
1  1 - -
B gMS r0 rMS
= =
v0 (2 - h)
a b
2
-
r0 gMS

where h = v 20 r0 >gMS .

1.210 At the minimum separation with the Sun, the cosmic body’s velocity is perpendicu-
lar to its position vector relative to the Sun. If rmin be the sought minimum distance,
from conservation of angular momentum about the Sun (C),
v0l
mv0l = mvrmin or v = (1)
rmin
From conservation of mechanical energy of the system (Sun 1 cosmic body),
1 gMSm 1
mv 20 = - + mv 2
2 rmin 2

v 20 gMS v 20
So, = - + (using Eq. 1)
2 rmin 2r 2min

or v 20 r 2min + 2gMSrmin - v 20 l 2 = 0

- 2gMS  24g2m2 + 4v 20 v 20 l 2 - gMS  2g 2MS2 + v 40 l 2


So, rmin = =
2v 20 v 20
Hence, taking positive root
rmin = 1gMS >v 202 3 21 + 1lv 20 >gMS22 - 14

1.211 (a) Suppose that the sphere has a radius equal to a. We may imagine that the sphere
is made up of concentric thin spherical shells (layers) with radii ranging from 0
to a, and each spherical layer is made up of elementary bands (rings). Let us first
calculate potential due to an elementary band of a spherical layer at the point of
location of the point mass m (say point P) (see figure). As all the points of the
band are located at the distance l from the point P, so,
g0M
0w = - (where 0M is mass of the band) (1)
l

0M = a b (2pa sin u) (adu)


dM
4pa2

= a b sin udu
dM
(2)
2
128 PART ONE PHYSICAL FUNDAMENTALS OF MECHANICS

and l 2 = a 2 + r 2 - 2 ar cos u (3)


Differentiating Eq. (3), we get
ldl = ar sin ud u (4)
Hence using above equations
a
0w = - a b dl
gdM l
(5) d
2ar
O r m
Now integrating Eq. (5) over the whole P
spherical layer
r +a
gdM
dw = 0w = - dl
L 2ar 3
r -a

gdM
So, dw = - (6)
r
Eq. (6) demonstrates that the potential produced by a thin uniform spherical
layer outside the layer is such as if the whole mass of the layer were concentrat-
ed at its centre. Hence the potential due to the sphere at point P is given by

g gM
w = dw - dM = - (7)
L r L r

This expression is similar to that of Eq. (6).


Hence the sought potential energy of gravitational interaction of the particle m
and the sphere,
gMm
U = mw = -
r
0w
(b) Using the equation Gr = -
0r
gM
Gr = - 2 (using Eq. 7)
r
gM gmM
So, G = - 3 r and F = m G = - 3 r (8)
r r

1.212 (The problem has already a clear hint in the answer sheet of the problem book. Here
we adopt a different method.)
Let m be the mass of the spherical layer, which is imagined to be made up of rings.
At a point inside the spherical layer at distance r from the centre, the gravitational
potential due to a ring element of radius a equals
gm
dw = - dl (see Eq. (5) of solution of problem 1.211)
2ar
1.4 UNIVERSAL GRAVITATION 129

a+r
gm gm a
So, w = dw = - dl = -
L 2ar 3 a
a-r dq l
0w O r
Hence, Gr = - = 0
0r
Hence, gravitational field strength as well as field force becomes
zero inside a thin spherical layer.

1.213 One can imagine that the uniform hemisphere is made up of thin hemispherical lay-
ers of radii ranging from 0 to R. Let us consider such a layer (see figure). Potential at
point O due to this layer is

a b dr
gdm 3gM M 4pr 2
dw = - = - 3 rdr, where dm =
r r (2>3) pR 3 2
(This is because all points of each hemispherical shell are equidistance from O.)
R
3gM 3gM
Hence, w = dw = - 3 rdr = -
L R 3 2R
0

Hence, the work done by the gravitational field


force on the particle of mass m, to remove it to in- M
finity is given by the formula
dr
A = mw, since w = 0 at infinity r
r
Hence the sought work, m
O
3gmM
A 0: q = -
2R
(The work done by the external agent is – A .)

1.214 In the solution of problem 1.211, we have obtained w and G due to a uniform sphere,
at a distance r from it’s centre outside it. We have from Eqs. (7) and (8) of problem 1.211,
gM gM
w = - and G = - 3 r (A)
r r
In accordance with the Eq. (1) of the solution of 1.212, potential due to a spherical
shell of radius a, at any point, inside it becomes
gM 0w
w = = constant and Gr = - = 0 (B)
a 0r
For a point (say P) which lies inside the uniform solid sphere, the potential w at that
point may be represented as a sum
winside = w1 + w2
130 PART ONE PHYSICAL FUNDAMENTALS OF MECHANICS

where w1 is the potential of a solid sphere having radius r and w2 is the potential of
the layer of radii r and R. In accordance with Eq. (A)

a pr 3 b = 3 r 2
g M 4 gM
w1 = -
r (4>3) pR 3 3 R
The potential w2 produced by the layer (thick shell) is the same at all points inside
it. The potential w2 is easiest to calculate, for the point positioned at the layer’s cen-
tre. Using Eq. (B)
R
dM 3 gM 2
w2 = - g = - (R - r 2)
3 r 2 R3
r

4pr 2dr = a 3 b r 2dr


M 3M
where dM = 3
(4>3) pR R
is the mass of a thin layer between the radii r and r + dr.

winside = w1 + w2 = a b a3 - 2 b
gM r2
Thus, (C)
2R R

- 0w
From Eq. (B) Gr =
0r
gMr
Gr =
R3
gM 4
or G = - r = - g prr
R3 3
M
where r = is the density of the sphere.
4 /3 pR 3
The plots w (r) and G (r) for a uniform sphere of radius R are shown in the figure
of answer sheet.

Alternate:
Like Gauss’s theorem of electrostatics, one can derive Gauss’s theorem for gravitation
in the form G.dS = - 4p gmenclosed. For calculation of G at a point inside the sphere
at a distance r from its centre, let us consider a Gaussian surface of radius r. Then

Gr4pr 2 = - 4pg a br
M 3 gM
or Gr = - r
r3 R3

r = - g pr r a as r = b
gM 4 M
Hence, G = -
R3 3 (4>3)pR 3
1.4 UNIVERSAL GRAVITATION 131

q R q
gM gM
So, w = Grdr = - 3 rdr + - 2 dr
3 3 R 3 r
r r R
Integrating and summing up, we get

a3 - 2 b
gM r2
w = -
2R R
From Gauss’ theorem for a point outside it,
gM
Gr4pr 2 = - 4pg M or Gr = -
r2
q
gM
Thus, w (r) = Grdr = -
3 r
r

1.215 Treating the cavity as negative mass of density - r in a uniform sphere density + r
and using the superposition principle, the sought field strength is

G = G1 + G2

4 4
or G = - pgrr + + - gp ( - r) r -
3 3
where r + and r - are the position vectors of an arbitrary r+ P
r_
point P inside the cavity with respect to centre of sphere l
and cavity, respectively.
4 4
Thus, G = - pgr (r + - r–) = - pgrl
3 3

1.216 We partition the solid sphere into thin spherical layers and consider a layer of thick-
ness dr lying at a distance r from the centre of the ball. Each spherical layer presses
on the layers within it. The considered layer is attracted to the part of the sphere lying
within it (the outer part does not act on the layer). Hence for the considered layer
dp 4pr 2 = dF

g (4 /3 p r 3 r) (4pr 2drr)
or dp 4 pr 2 = dr
r2
r
(where r is the mean density of sphere)

4
or dp = pgr2 rdr R
3
132 PART ONE PHYSICAL FUNDAMENTALS OF MECHANICS

R
2p 2 2
Thus, p = dp = gr (R - r 2)
3 3
r
(the pressure must vanish at r = R )

(1 - (r 2>R 2)) gM 2>pR 4, putting r = M>(4>3) pR 3


3
or p =
8
Putting r = 0, we have the pressure at sphere’s centre, and treating it as the Earth
where mean density is equal to r = 5.5 * 103 kg/m3 and R = 64 * 102 km we
have, p = 1.73 * 1011 Pa or 1.72 * 106 atm.

1.217 (a) Since the potential at each point of a spherical surface (shell) is constant and is
equal to w = - gm /R , [as we have in Eq. (1) of solution of problem 1.212].
We obtain in accordance with the equation
1 1
U = dm w = w dm
23 2 3

a- b m = -
1 gm gm2
=
2 R 2R

(The factor 1/2 is needed otherwise contribution of different mass elements is


counted twice.)

(b) In this case the potential inside the sphere depends only on r (see Eq. (C) of the
solution of problem 1.214)

a1 - b
3gM r2
w = -
2R 3R 2

Here dm is the mass of an elementary spherical layer confined between the radii
r and r + dr given by

dm = (4pr 2 drr) = a b r 2dr


3m
R3

1
U = dm w
2 3
R

a 3 b r 2 dr - a1 - b
1 3m 3gm r2
=
23 R 2R 3R 2
0
1.4 UNIVERSAL GRAVITATION 133

After integrating, we get


3 gm2
U = -
5 R

gME
1.218 Let v = = circular frequency of the satellite in the outer orbit,
A r3
gME
v0 = = circular frequency of the satellite in the inner orbit.
A (r - ¢r)3
So, relative angular velocity = v0  v , where - ve sign is to be taken when the satel-
lites are moving in the same sense and + ve sign if they are moving in opposite sense.
Hence, time between closest approaches
2p 2p 1 4.5 days (d = 0)
= = b
v0  v 2gME >r 3>2 3¢r /2r + d 0.80 hour (d = 2)

where d is 0 in the first case and 2 in the second case.

gM 6.67 * 10-11 * 5.96 * 1024


1.219 v1 = = = 9.8 m/s2
R2 (6.37 * 106)2
2 * 22
v2 = v2R = a b R = a b 6.37 * 106 = 0.034 m/s2
2p 2 2

T 24 * 3600 * 7

gMS 6.67 * 10-11 * 1.97 * 1030


and v3 = = = 5.9 * 10-3 m/s2
R 2mean (149.50 * 106 * 103)2

Then, v1 : v2 : v3 = 1 : 0.0034 : 0.0006

1.220 Let h be the sought height in the first case, so


99 gM
g =
100 (R + h)2

gM g
= =
R2 (1 + h/R )2 (1 + h /R )2

h -2
= a1 + b
99
or
100 R
134 PART ONE PHYSICAL FUNDAMENTALS OF MECHANICS

From the statement of the problem, it is obvious that in this case h V R

= a1 - b = a b km = 32 km
99 2h R 6400
Thus, or h =
100 R 200 200
In the other case, if h¿ be the sought height, then
h¿ -2 h¿ -2
= g a1 + b = a1 + b
g 1
or
2 R 2 R
From the language of the problem, in this case h¿ is not very small in comparison with
R. Therefore in this case we cannot use the approximation adopted in the previous case.

a1 + b = 2
h¿ 2
Here,
R
h¿
So, =  12 - 1
R
As ve sign is not acceptable
h¿ = ( 12 - 1) R = ( 12 - 1) 6400 km = 2650 km

1.221 Let the mass of the body be m and let it go upto a height h.
From conservation of mechanical energy of the system
gMm 1 - gMm
- + mv 20 = + 0
R 2 (R + h)
Using gM /R 2 = g, in above equation and on solving we get
Rv 20
h =
2gR - v 20

1.222 Gravitational pull provides the required centripetal acceleration to the satelite. Thus
if h be the sought distance, we have
mv 2 gmM
= or (R + h) v 2 = gM
(R + h) (R + h)2
gM
or Rv 2 + hv 2 = gR 2 as g =
R2
gR - Rv
2 2
= R c 2 - 1d
gR
Hence, h = 2
v v

1.223 A satellite that hovers above the Earth’s equator and co-rotates with it moving from
west to east with the diurnal angular velocity of the Earth appears stationary to an ob-
server on the Earth. It is called geostationary. For this calculation we may neglect the
annual motion of the Earth as well as all other influences. Then, by Newton’s law,

a b r
gMm 2p 2
= m
r2 T
1.4 UNIVERSAL GRAVITATION 135

where M = mass of the Earth, T = 86400 seconds = period of daily rotation of the
Earth and r = distance of the satellite from the centre of the Earth.
T 2
Then, r = 3 gM a b
A 2p

Substitution of M = 5.96 * 1024 kg, gives r = 4.220 * 104 km .


Thus instantaneous velocity with respect to an inertial frame fixed to the centre of
the Earth at that moment will be

a b r = 3.07 km/s
2p
T
and the acceleration will be the centripetal acceleration

a b r = 0.223 m/s 2
2p 2
T

1.224 We know from the previous problem that a satellite moving west to east at distance
R = 2.00 * 104 km from the centre of the Earth will be revolving round the Earth
with an angular velocity faster than the Earth’s diurnal angular velocity. Let
v = angular velocity of the satellite,
v0 = 2p /T = angular velocity of the Earth.
Then v - v0 = 2p /t
is the relative angular velocity with respect to Earth. Now by Newton’s law
gM
= v2R
R2

a b
R 3 2p 2p 2
So, M = +
g t T

a b
4 p2R 3 T 2
= 1 +
gT 2 t
Substitution gives
m = 6.27 * 1024 kg

1.225 The velocity of the satellite in the inertial space fixed frame is 2gM /R east to west.
With respect to the Earth fixed frame, from the v œ = v - (␻ * r), the velocity is

2pR gM
v¿ = + = 7.03 km/s
T A R
136 PART ONE PHYSICAL FUNDAMENTALS OF MECHANICS

Here M is the mass of the Earth and T is its period of rotation about its own axis. If
the satellite were moving from west to east, velocity would be
2pR gm
- +
T A R
To find the acceleration we note the formula

m w¿ = F + 2 m (v¿ * ␻) + mv2 R

Here F = gMm /R 3 R and v ⬜ ␻ and v¿ * ␻ is directed towards the centre of the Earth.

+ 2 a b - a b R (toward the Earth’s rotation axis)


gM 2pR gM 2p 2p 2
Thus, w¿ = +
R 2 T A R T T

c d = 4.94 m/s2 on substitution


gM 2p 2pR gM
= + + 2
R 2 T T A R

1.226 From the well known relationship between the velocities of a particle with respect
to a space fixed frame (K) and rotating frame (K ¿) ,

v = v œ + (␻ * r)

v 1¿ = v - a b R
2p
we get,
T
Thus kinetic energy of the satellite in the Earth’s frame

mv 1¿ 2 = m a v - b
1 1 2pR 2
T 1¿ = (1)
2 2 T

Obviously when the satellite moves in opposite sense compared to the rotation of
the Earth, its velocity relative to the same frame would be

v2¿ = v + a b R
2p
T
and kinetic energy

mv 2¿ 2 = m a v + b
1 1 2pR 2
T 2¿ = (2)
2 2 T

From Eqs. (1) and (2)


1v + 2pR /T 22
T¿ =
1v - 2pR /T 22
(3)
1.4 UNIVERSAL GRAVITATION 137

Now from Newton’s second law


gMm mv 2 gM
= or v = = 1gR (4)
R2 R A R
Using Eqs. (3) and (4)
T2 ¿ 11gR + 2pR /T 22
= = 1.27 nearly (using Appendices)
T1 ¿ 1 1gR - 2pR /T 22

1.227 For a satellite in a circular orbit about any massive body, the following relation holds
between kinetic, potential and total energy
T = - E, U = 2E (1)
Thus, since total mechanical energy must decrease due to resistance of the cosmic
dust, the kinetic energy will increase and the satellite will ‘‘ fall’’. We see then, by work
energy theorem
dT = - dE = - dAfr

adt dv
So, mvdv = av 2vdt or = 2
m v
Now from Newton’s law at an arbitrary radius r from the Moon’s centre,
v2 gM gM
= 2 or v = (where M is the mass of the moon)
r r A r

gM gM
Then, vi = and vf = (where R = Moon’s radius)
A hR A R
vf t
dv am at
So, 2
= dt =
3 v m 3 m
vi 0

a - b =
m 1 1 m
or t = ( 1h - 1)
a vi vf a 2gM /R
m
= ( 1h - 1) (where g is Moon’s gravity)
a 1gR

The averaging implied by Eq. (1) (for non-circular orbits)makes the result approxi-
mate.

1.228 From Newton’s second law

gMm mv 20 gM
= or v0 = = 1.67 km/s (1)
R 2 R A R
138 PART ONE PHYSICAL FUNDAMENTALS OF MECHANICS

From conservation of mechanical energy

1 gMm 2gM
mve2 - = 0 or ve = = 2.37 km/s (2)
2 R A R

In Eqs. (1) and (2), M and R are the mass of the Moon and its radius. In Eq. (1) if
M and R represent the mass of the Earth and its radius, then, using appendices, we
can easily get orbital and escape velocities for Earth as
v0 = 7.9 km/s and ve = 11.2 km/s

1.229 In a parabolic orbit, E = 0


1 gMm gM
So, mv 2i - = 0 or vi = 12
2 R A R
where M = mass of the Moon, R  its radius and this is just the escape velocity. On
the other hand in orbit,
gMm gM
mvf 2 R = or vf =
R 2 A R
gM
Thus, ¢v = (1 - 12) = - 0.70 km/s
A R

1.230 From solution of problem 1.228 for the Earth surface

gM 2gM
v0 = and ve =
A R A R

Thus the sought additional velocity


gM
¢v = ve - v0 = ( 12 - 1) = 1gR ( 12 - 1)
A R
This ‘‘kick’’ in velocity must be given along the direction of motion of the satellite
in its orbit.
1.231 Let r be the sought distance then
ghM gM
= 2 or hr 2 = (nR - r)2
(nR - r)2 r
nR
or 1hr = (nR - r) or r = = 3.8 * 104 km
1h + 1

1.232 Between the Earth and the Moon, the potential energy (P.E.) of the spaceship will have
a maximum at the point where the attractions of the Earth and the moon balance each
other. This maximum P.E. is approximately zero. We can also neglect the contribution
1.4 UNIVERSAL GRAVITATION 139

of either body to the P.E. of the spaceship sufficiently near the other body. Then the
minimum energy that must be imparted to the spaceship to cross the maximum of the
P.E. is clearly (using E to denote the Earth)
gMEm
(1)
RE
With this energy the spaceship will cross over the hump in the P.E. and coast down
the hill of P.E. towards the moon and crash land on it. What the problem seeks is the
minimum energy required for soft-landing. That requires the use of rockets to lower-
ing and braking of the spaceship, and since the kinetic energy of the gases ejected
from the rocket will always be positive, the total energy required for soft-landing is
greater than that required for crash-landing. To calculate this energy we assume the
rockets are used fairly close to the moon when the spaceship has nearly attained its
terminal velocity on the moon 22gM0/R0 where M0 is the mass of the moon and R0
is its radius. In general dE = vdp and since the speed of the ejected gases is not less
than the speed of the rocket, and momentum transferred to the gases must equal the
momentum of the spaceship the energy E of the gas ejected is not less than the
kinetic energy of spaceship
g M0m
(2)
R0
Adding Eqs. (1) and (2) we get the minimum work done on the ejected gases to
bring about the soft landing.
ME M0
Amin = gm a + b
RE R0
On substitution we get 1.3 * 108 kJ.
1.233 Assume first that the attraction of the Earth can be neglected. Then the minimum ve-
locity, that must be imparted the body to escape from the Sun’s pull, is, as in solu-
tio of problem 1.230, equal to
( 12 - 1) v1
where v 21 = gMS >r, r = radius of the Earth’s orbit, MS = mass of the Sun.
In the actual case near the Earth, the pull of the Sun is small and does not change
much over distances, which are several times the radius of the Earth. The velocity v3
in question is that which overcomes the Earth’s pull with sufficient velocity to es-
cape the Sun’s pull.
1 gME 1
Thus, mv 23 - = m ( 12 - 1)2v 21
2 R 2
where R = radius of the Earth and ME = mass of the Earth.
Writing v1 2 = gME >R, we get
v3 = 22 v 22 + (12 - 1)2v 21 .= 16.6 km/s
140 PART ONE PHYSICAL FUNDAMENTALS OF MECHANICS

1.5 Dynamics of a Solid Body

1.234 Since, motion of the rod is purely translational, net torque about the C.M. of the rod
should be equal to zero.
F1
= F2 a - a b
l l a
Thus, F1 or = 1 - (1)
2 2 F2 l>2
For the translational motion of rod,
F1 mwC
F2 - F1 = mwC or 1 - = (2)
F2 F2
From Eqs. (1) and (2)
a mwC 2 aF2
= or l = = 1m
l>2 F2 mwC

1.235 Sought moment N = r * F = (a i + b j) * (A i + B j)


= aB k + Ab ( - k) = (aB - Ab) k
N aB - Ab
and arm of the force l = =
F 2A2 + B 2

1.236 Relative to point O, the net moment of force is given by


N = r1 * F1 + r2 * F2 = (a i * A j) + (b j * B i)
= ab k + AB ( - k) = (ab - AB)k (1)
Resultant of the external force
F = F1 + F2 = Aj + B i (2)

N # F = 0 (as N ⬜ F), so the sought arm l of the force F is given by

N ab - AB
l = = .
F 2A2 + B 2

1.237 For coplanar forces, about any point in the same plane,

©ri * Fi = r * Fnet (where Fnet = ©Fi = resultant force)

or Nnet = r * Fnet
Nnet
Thus length of the arm, l =
Fnet
1.5 DYNAMICS OF A SOLID BODY 141

Here obviously | Fnet | = 2F and it is directed toward right along AC. Take the origin
at C. Then about C,

N = a22 aF + F - 22 aF b , directed normally into the plane of figure


aF
22
(Here a = side of the square.)
a
Thus, N = F , directed into the plane of the figure.
12
F (a> 12) a a
Hence, l = = = sin 45°
2F 2 12 2
Thus the point of application of force is at the mid point of the side BC.

1.238 (a) Consider a strip of length dx at a perpendicular distance x from the axis about
which we have to find the moment of inertia of the rod. The elemental mass of
the rod equals
m
dm = dx
l
Moment of inertia of this element about the axis O
x
m
dI = dmx 2 = dx # x2 dx
l
Thus, moment of inertia of the rod, as a whole about the given axis
l
m 2 ml 2
I = x dx =
3 l 3
0

(b) Let us imagine the plane of plate as x–y plane taking the origin at the intersec-
tion point of the sides of the plate (see figure).
y
a
Obviously, Ix = dm y2
3
x b
a dx
b dy b y 2
a
m
= O x
3 ab y
0 a
ma2
= dx
3 b
mb 2 y
Similarly, Iy = O x
3
142 PART ONE PHYSICAL FUNDAMENTALS OF MECHANICS

Hence from perpendicular axis theorem


m 2
Iz = Ix + Iy = (a + b 2)
3
which is the sought moment of inertia.

1.239 (a) Consider an elementary disk of thickness dx. Moment of inertia of this element
about the z-axis, passing through its C.M. is
(dm)R 2 R2
dIz = = r Sdx
2 2
where r = density of the material of the plate and S = area of cross section of
the plate. Thus, the sought moment of inertia
x
b
rSR 2 R2
Iz = dx = rSb
2 3 2
0 O R
p b dx
= rbR 4 (as S = pR 2)
2
Putting all the values we get Iz = 2.8 g>m2 .
(b) Consider an element disk of radius r and thickness dx at a distance x from the
point O. Then r = x tan a and volume of the disk
= px 2 tan2 a dx
Hence, its mass dm = px 2 tan a dx # r

where r = density of the cone = mn pR 2h.


1
3
Moment of inertia of this element, about the axis OA,
r2
dI = dm
2
x 2 tan2 a r
= (px 2 tan2 a dx) dx h
2
pr 4 x
= x tan4 a dx
2 A R
Thus the sought moment of inertia
h
pr
I = tan4 a x 4 dx
2 3
0
prR 4 # h 5
a as tan a = b
R
=
10h 4 h
1.5 DYNAMICS OF A SOLID BODY 143

aputting r = b
3mR 2 3m
Hence, I =
10 pR 2h

1.240 (a) Let us consider a lamina of an arbitrary shape and indicate by 1,2 and 3, three
axes coinciding with x, y and z-axes and the plane of lamina as x -y plane.
Now, moment of inertia of a point mass about x-axis,
dIx = dm y 2 y z(3)
y(2)
Thus moment of inertia of the lamina about this axis,

Ix = dmy 2 r (x,y)
3 O x(1)

Similarly, Iy = dmx 2
3

and Iz = dmr 2
3

= dm (x 2 + y 2) as r = 2x 2 + y 2
3
Thus, Iz = Ix + Iy or I3 = I1 + I2

(b) Let us take the plane of the disk as x -y plane and z


origin to the centre of the disk (see figure). From the y
symmetry Ix = Iy . Let us consider a ring element of
radius r and thickness dr, then the moment of inertia
O x
or the ring element about the y-axis is given by
m
dIz = dmr 2 = (2prdr)r 2 dr
pR 2
Thus the moment of inertia of the disk about z-axis
R
2m mR 2
Iz = 2 r 3dr =
R 3 2
0

But we have Ix = Ix + Iy = 2Ix


Iz mR 2
Thus, Iz = =
2 4

1.241 For simplicity let us use a mathematical trick. We consider the portion of the given disk
as the superposition of two complete disks (without holes), one of positive density and
radius R and other of negative density but of same magnitude and radius R>2.
144 PART ONE PHYSICAL FUNDAMENTALS OF MECHANICS

As (area) r (mass), the respective masses of the consid-


ered disks are (4m/3) and ( - m/3), respectively, and 1
these masses can be imagined to be situated at their re-
spective centres (C.M.). Let us take point O as origin 2 O x
and point x-axis towards right. Obviously the C.M. of C
the shaded position of given shape lies on the x-axis.
Hence the C.M. (C) of the shaded portion is given by

( -m>3)( -R>2) + (4m>3)0 R


xC = =
a- b +
m 4m 6
3 3

Thus C.M. of the shape is at a distance R/6 from point O toward x-axis.
Using parallel axis theorem and bearing in mind that the moment of inertia of a
complete homogeneous disk of radius m0 and radius r0 equals 1>2m0r 20; the moment
of inertia of the small disk of mass (- m>3) and radius R>2 about the axis passing
through point C and perpendicular to the plane of the disk is given by

a- ba b + a- ba + b
1 m R 2 m R R 2
I2C =
2 3 2 3 2 6
mR 2 4
= - - mR 2
24 27

a bR + a ba b
1 4m 2 4m R 2
Similarly, I1C =
2 3 3 6
2 mR 2
= mR 2 +
3 27
Thus, the sought moment of inertia,
37
IC = I1C + I2C = mR 2
72
= 0.15 kg # m2

1.242 Moment of inertia of the shaded portion, about the axis pass-
ing through its centre,
r
I = a p R 3r bR 2 - a pr 3rb r 2
2 4 2 4 R
5 3 5 3
2#4
= pr(R 5 - r 5)
5 3
1.5 DYNAMICS OF A SOLID BODY 145

Now, if R = r + dr, the shaded portion becomes a shell, which is the required shape
to calculate the moment of inertia.
2#4
Now, I = pr {(r + dr)5 - r 5}
5 3
2#4
= pr (r 5 + 5r 4 dr + ... - r 5)
5 3
Neglecting higher terms, we get
2 2
= (4pr 2 drr) r 2 = mr 2
3 3

1.243 (a) Net force which is effective on the system (cylinder M + body m) is the weight
of the body m in a uniform gravitational field, which is a constant. Thus the ini-
tial acceleration of the body m is also constant.
From the conservation of mechanical energy of the said system in the uniform
field of gravity at time t = ¢t : ¢T + ¢Ugr = 0

1 1 MR 2 2
or mv 2 + v - mg ¢h = 0
2 2 2
1
or (2m + M)v 2 - mg ¢h = 0 (as v = vR at all times) (1)
4
But v 2 = 2w¢h
Hence using it in Eq. (1), we get
1 2mg
(2m + M) 2w ¢h - mg ¢h = 0 or w =
4 (2m + M)
w 2mg
From the kinematic relationship, b = =
R (2m + M) R
Thus the sought angular velocity of the cylinder
2mg gt
v(t) = bt = t =
(2m + M ) R (1 + M>2m) R
(b) Sought kinetic energy
1 1 ml 2 2
T (t) = mv 2 + v
2 2 2
1
= (2m + M ) R 2v2
4

mg 2 a 1 + b
1 M
= (on substituting value of v )
2 2m
146 PART ONE PHYSICAL FUNDAMENTALS OF MECHANICS

1.244 For equilibrium of the disk and axle


2T = mg or T = mg>2
As the disk unwinds, it has an angular acceleration b
given by T T

2Tr mgr
2Tr = Ib or b = =
I I
The corresponding linear acceleration is mg

mgr 2
rb = w =
I
Since the disk remains stationary under the combined action of this acceleration and
the acceleration (–w) of the bar which is transmitted to the axle, we must have
w = mgr 2/I .
1.245 Let the rod be deviated through an angle w from its initial position at an arbitrary in-
stant of time, measured relative to the initial position in the positive direction. From
the equation of the increment of the mechanical energy of the system

¢T = Aext
1 2
or Iv = Nz d w
2 3
w
1 Ml 2 2
or v = Fl cos w dw = Fl sin w
2 3 3
0
6F sin w
Thus, v =
A Ml

1.246 First of all, let us sketch a free diagram of each body. Since the cylinder is rotating
and massive, the tension will be different in both the sections of threads. From
Newton’s law in projection form for the bodies m1 and m2 and noting that
w1 = w2 = w = bR (as the thread is not slipping), we have (m1 7 m2):

m1g - T1 = m1w = m1 bR
and T2 - m2g = m2w (1)
T1 T2
Now from the equation of rotational dynamics of a solid about sta-
tionary axis of rotation, i.e.,
Nz = Ib z (for the cylinder)
w m1 m2 w
mR 2 b
or (T1 - T2)R = Ib = (2) m1g m2g
2
1.5 DYNAMICS OF A SOLID BODY 147

Simultaneous solution of the above equations yields:


(m1 - m2)g T1 m1(m + 4m2)
b = and =
m2(m + 4m1)
R a m1 + m2 + m/2 b
T2

1.247 As the system (m + m1 + m2) is under constant forces, the acceleration of body m1
and m2 is constant. In addition to it the velocities and acceleration of bodies m1 and
m2 are equal in magnitude (say v and w) because the length of the thread is constant.
From the equation of increment of mechanical energy, i.e., ¢T + ¢U = Afr , at time t
when block m1 is distance h below from initial position corresponding to t = 0,

(m1 + m2)v 2 + a b
1 1 mR 2 v 2
- m2gh = - km1gh (1)
2 2 2 R2

(as angular velocity v = V/R for no slipping of thread).


But v 2 = 2wh
So using it in Eq. (1), we get
2(m2 - km1)g
w = (2)
m + 2(m1 + m2)
Thus the work done by the friction force on m1

Afr = - km1gh = - km1g a wt 2 b


1
2
km1(m2 - km1)g 2t 2
= - (using Eq. 2)
m + 2(m1 + m2)

1.248 In the problem, the rigid body is in translation equilibrium but there is an angular
retardation. We first sketch the free body diagram of the cylinder. Obviously the fric-
tion forces, acting on the cylinder, are kinetic. From the condition of translational
equilibrium for the cylinder,
mg = N1 + kN2; N2 = kN1
mg mg
Hence, N1 = ; N2 = k kN 2 N2
1 + k2 1 + k2 C
For pure rotation of the cylinder about its rotation
mg
axis, Nz = Ib z N1
mR 2
or - kN1R - kN2R = bz
2 kN1
148 PART ONE PHYSICAL FUNDAMENTALS OF MECHANICS

kmgR (1 + k) mR 2
or - = bz
1 + k2 2
2k (1 + k)g
or bz = -
(1 + k 2)R
Now, from the kinematical equation,
v2 = v20 + 2b z ¢w
v20 (1 + k 2)R
we have, ¢w = (because v = 0)
4k (1 + k)g
Hence, the sought number of turns,
¢w v20 (1 + k 2 )R
n = =
2p 8pk (1 + k)g

1.249 It is the moment of friction force which brings the disk to rest. The force of friction
is applied to each section of the disk, and since these sections lie at different distances
from the axis, the moments of the forces of friction differ from section to section.
To find Nz, where z is the axis of rotation of the disk let us z
partition the disk into thin rings (see figure). The force of dr
friction acting on the considered element dfr = k (2prdrs)g,
(where s is the density of the disk).
O r
The moment of this force of friction is
R
dNz = - rdfr = - 2pk sgr 2dr
Integrating with respect to r from zero to R, we get
R
2
Nz = - 2p k sg r 2dr = - pk sgR 3
3 3
0
For the rotation of the disk about the stationary axis z, from the equation Nz = Ib z
2 (pR 2s)R 2 4kg
- pk sgR 3 = b z or b z = -
3 2 3R
Thus, from the angular kinematical equation vz = v0z + b z t
3R v0
0 = v0 + a - bt or t =
4kg
3R 4kg

1.250 According to the question,


dv dv
I = - k 1v or I = - kdt
dt 1v
1.5 DYNAMICS OF A SOLID BODY 149

kt
Integrating, 1v = - + 1v0
2I

k 2t 2 1v0 kt
or v = - + v0 (noting that at t = 0, v = v0 )
4I 2 I

Let the flywheel stop at t = t0, then from Eq. (1), t0 = 2I2v0 /k
Hence sought average angular velocity

2I 2v0>k

1k 2t 2/4I 2 - 2v0 kt/I + v02dt


3
<v> =
0 v0
=
2I2v0>k 3

dt
3
0

1.251 Let us use the equation dMz /dt = Nz, relative to the axis through O. (1)
For this purpose, let us find the angular momentum of the system Mz about the given
rotation axis and the corresponding torque Nz . The angular momentum is

m0
Mz = I v + mvR = a + mb R 2 v
2
m0
where I = R 2 and v = vR (no cord slipping).
2

dMz
= a + mR 2 b b z
MR 2
So, (2)
dt 2

The downward pull of gravity on the overhanging part is the only external force,
which exerts a torque about the z-axis, passing through O and is given by,

Nz = a b xgR
m
l

dMz
Hence, from the equation = Nz
dt

a + mR 2 b b z =
MR 2 m
x gR
2 l
150 PART ONE PHYSICAL FUNDAMENTALS OF MECHANICS

2mgx
Thus, bz = 7 0
lR (M + 2m)
Note: We may solve this problem using conservation of mechanical energy of the sys-
tem (cylinder thread) in the uniform field of gravity.

1.252 (a) Let us indicate the forces acting on the sphere and their points of application.
Choose positive direction of x and w (rotation angle) along the incline in down-
ward direction and in the sense of v (for unidirectional rotation) respectively.
Now from equations of dynamics of rigid body, i.e., Fx = mwC x and NC z = IC b z
we get
mg sin a - fr = mw (1) N fr
2 j
and frR = mR 2 b (2)
5
x a mg
But, fr … kmg cos a (3)
In addition, the absence of slipping provides the kinematical relationship between
the accelerations as.
w = bR (4)
The simultaneous solution of all the four equations yields
2 2
k cos a Ú sin a or k Ú tan a.
7 7
(b) Solving Eqs. (1) and (2) [of part (a)], we get
5
wC = g sin a
7
As the sphere starts at t = 0 along positive x-axis, for pure rolling
5
vC (t) = wC t = g sin at (5)
7
Hence, the sought kinetic energy is

mv 2C (as v = vC >R)
1 1 2 7
T = mv 2C + mR 2v2 =
2 2 5 10
7 5 2 5
= ma g sin atb = mg 2 sin2at 2
10 7 14

1.253 (a) Let us indicate the forces and their points of application for the cylinder.
Choosing the positive direction for x and w as shown in the figure, we write the
equation of motion of the cylinder axis and the equation of moments in the C.M.
frame relative to that axis, i.e., from equation Fx = mwC and Nz = IC b z we get
mR 2
mg - 2T = mwC and 2RT = b
2
1.5 DYNAMICS OF A SOLID BODY 151

As there is no slipping of thread on the cylinder j


wC = bR
T x
From these three equations T
mg 2 g
T = = 13 N, b =
6 3 R
mg
= 5 * 102 rad /s2
2g
(b) We have b =
3R
2 2
So, wC = g 7 0 or in vector from wC = g
3 3
P = F # v = F # (wC t)

= m g # a gtb = mg 2t
2 2
3 3

1.254 Let us depict the forces and their points of application corresponding to the cylinder
attached with the elevator. Newton’s second law for solid in vector form in the frame
of elevator, gives
2T + m g + m ( - w0) = m w œ (1)
The equation of moment in the C.M. frame relative to the cylinder axis, i.e., from
Nz = IC b z, is
mR 2 mR 2 w ¿
2TR = b =
2 2 R T T
(as thread does not slip on the cylinder, w¿ = bR ) j
mw¿ w0
or T = mg
4
From Eq. (1), T c T w , and so in vector form, mw0
mw
T = - (2)
4
Solving Eqs. (1) and (2), we get
2
w = (g - w0)
3
1
and sought force F = 2T = m (g - w0)
3

1.255 Let us depict the forces and their points of application for the spool. Choosing the
positive direction for x and w as shown in the figure, we apply Fx = mwx and
NC z = IC bz and get mg sin a - T = mw, Tr = Ib .
152 PART ONE PHYSICAL FUNDAMENTALS OF MECHANICS

“Notice that if a point of a solid in plane motion is O


connected with a thread, the projection of velocity
vector of the solid’s point of contact along the length T
of the thread equals the velocity of the other end of N
the thread (if it is not slacked)”. P φ
Thus in our problem, vP = v0 but v0 = 0, hence point r
P is the instantaneous centre of rotation
of zero velocity for the spool. Therefore vC = vr and
w
subsequently wC = br.
Solving the equations simultaneously, we get
mg
g sin a
w = = 1.6 m /s2
1 + I/mr 2

1.256 Let us sketch the force diagram for solid cylinder and apply Newton’s second law in
projection form along x- and y-axes (see figure). Then,
fr1 + fr2 = mwC (1)
N2
and N1 + N2 - mg - F = 0 j
or N1 + N2 = mg + F (2) N1
fr2
Now choosing positive direction of w as shown in
the figure and using NCz = IC b z, we get
fr1
FR - ( fr1 + fr2) R
mR 2 mR 2 wC
= b = (3)
2 2 R mg F
(as for pure rolling wC = bR ).
In addition,
fr1 + fr2 … k (N1 + N2) (4)
Solving the equations, we get
3k mg
F …
2 - 3k
3k mg
or Fmax =
2 - 3k
k (N1 + N2)
and wC (max) =
m

c mg + d =
k k 3kmg 2kg
= [mg + Fmax] =
m m 2 - 3k 2 - 3k
1.5 DYNAMICS OF A SOLID BODY 153

1.257 (a) Let us choose the positive direction of the rotation angle w, such that wCx and b z
have identical signs (see figure). Equation of motion, Fx = mwCx and NCz = IC b z
gives
F cos a - fr = mwCx and frR - Fr = IC b z = gmR 2 b z
In the absence of the slipping of the N
spool, wC x = b z R.
From the three equations we get F
F [cos a - (r>R)] R a
wCx = wC =
m (1 + g) r
C
(where cos a 7 r/R).
(b) As static friction ( fr) does not work on
x
the spool, from the equation of the incre- fr
ment of mechanical energy Aext = ¢T.
mg

1 1 v 2C 1
Aext = mv 2C + g mR 2 2 = m (1 + g)v 2C
2 2 R 2

m (1 + g)2wC x = m (1 + g) 2wC a wC t 2 b
1 1 1
=
2 2 2

F 2 a cos a - r/R b t 2
2

=
2m (1 + g)

Note: At cos a = r>R, there is no rolling and for cos a 6 r>R, wcx 6 0, i.e., the sp-
ool will move towards negative x-axis and rotate in anticlockwise sense.
1.258 For the cylinder, from equation Nz = I z about its stationary axis of rotation,
mr 2 4T
2Tr = b or b = (1)
2 mr
For the rotation of the lower cylinder, from equation NCz = IC b z ,
mr 2
2Tr = b¿
2 j
4T
or b¿ = = b
mr
T
Now for the translational motion of lower cylinder, from T
equation Fx = mwC x ,
mg - 2T = mwC (2) j'
mg
154 PART ONE PHYSICAL FUNDAMENTALS OF MECHANICS

As there is no slipping of threads on the cylinders,


wC = b ¿r + br = 2br (3)
Simultaneous solution of Eqs. (1), (2) and (3) yields
mg
T =
10

1.259 Let us depict the forces acting on the pulley and weight A, and indicate positive
direction for x and w as shown in the figure. For the cylinder from the equation,
Fx = m wx and NC z = IC b z, we get
Mg + TA - 2T = MwC (1)
IwC
and 2TR + TA(2R) = I b = (2)
R
T
For the weight A from the equation
T
Fx = mwx
We get, mg - TA = mwA (3)
As there is no slipping of the threads on the pulleys, R
2R
wA = wC + 2 bR = wC + 2wC = 3wC (4)
Simultaneous solution of above four equations gives TA
3 (M + 3m) g Mg
wA = TA
M + 9m + 1>R 2
mg
1.260 (a) For the translational motion of the system
(m1 m2), from the equation: FxmwCx , we get
F = (m1 + m2) wC or wC = F>(m1 + m2) (1)
Now for the rotational motion of cylinder from A
the equation: NCx = IC b z , we get
x
m1r 2 2F
Fr = b or br = (2) K
2 m1 F
B
But, wK = wC + br
F 2F F (3m1 + 2m2)
So, wK = + = (3)
m1 + m2 m1 m1(m1 + m2)

(b) From the equation of increment of mechanical energy: ¢T = Aext


Here, ¢T = T (t) so, T (t) = Aext
1.5 DYNAMICS OF A SOLID BODY 155

As force F is constant and is directed along x-axis the sought work done is given by
Aext = Fx
(where x is the displacement of the point of application of the force F during time
interval t) or
F 2t 2(3m1 + 2m2)
F a wKt 2 b =
1
= T (t) (using Eq. 3)
2 2m1(m1 + m2)
Alternate:
T (t) = Ttranslation (t) + Trotation (t)

1 m1r
2 F 2t 2(3m1 + 2m2)
(m1 + m2) a b + a b =
1 Ft 2 2Ft 2
=
2 (m1 + m2) 2 2 m1r 2m1 (m1 + m2)

1.261 Choosing the positive direction for x and w as shown in


figure, let us we write the equation of motion for the sphere C x
m2
Fx = mwC x and NC z = IC b z
fr
fr = m2w2; fr m1 F
2
(fr) r = m2r 2 b
5
(where w2 is the acceleration of the C.M. of sphere).
For the plank, from equation Fx = mwx , we have
F - fr = m1w1
In addition, the condition for the absence of slipping of the sphere yields the kine-
matic relation between the accelerations
w1 = w2 + br
Simultaneous solution of the four equations yields
F 2
w1 = and w2 = w
m1 + 2>7m2 7 1

1.262 (a) Let us depict the forces acting on the cylinder and their point of applications for
the cylinder and indicate positive direction of x and w as shown in the figure.
From the equations for the plane motion of a solid Fx = mwC x and NC z = IC b z ,
kmg = mwC x or wC x = kg (1)
kg x
mR 2
- kmg R = b z or b z = - 2 (2)
2 R 0 C mg
Let the cylinder start pure rolling at t = t0 after releasing
R fr
on the horizontal floor at t = 0.
156 PART ONE PHYSICAL FUNDAMENTALS OF MECHANICS

From the angular kinematical equation,


vz = voz + b z t,
kg
or v = v0 - 2 t (3)
R
From the equation of the linear kinematics,
vC x = voC x + wC xt
or vC = 0 + kgt 0 (4)
But at the moment t = t0, when pure rolling starts vC = vR.

kgt0 = a v0 - 2 t0 b R
kg
So,
R
v0R
Thus, t0 =
3 kg

(b) As the cylinder picks up speed till it starts rolling, the point of contact has purely
translatory movement equal to 1>2 wC t 20 in the forward direction but there is also a
backward movement of the point of contact of magnitude (v0t0 - 1>2 bt 20) R.
Because of slipping the net displacement is backwards. The total work done is
then,

Afr = kmg c wC t 20 - (v0t0 + bt 20 )R d


1 1
2 2

= kmg c kg t 20 - a - b t 0 R - v0t0R d
1 1 2kg 2
2 2 R
v0 R v0R v0R mv20R 2
= kmg c + - v0R d = -
3kg 6 3 6

The same result can also be obtained by the work-energy theorem, Afr = ¢T.

1.263 Let us write the equation of motion for the centre of the sphere at the moment of
breaking-off
mv 2
= mg cos u
R + r
where v is the velocity of the centre of the sphere at that mo-
ment, and u is the corresponding angle (see figure). The veloc- h
ity v can be found from the energy conservation law r C
R
1 1
mgh = mv 2 + I v2
2 2 O mg
1.5 DYNAMICS OF A SOLID BODY 157

where I is the moment of inertia of the sphere relative to the axis passing through
the sphere’s centre. i.e. I = 2>5 mr 2. In addition,
v = vr and h = (R + r) (1 + cos u)
From these four equations we obtain
v = 210g (R + r)>17r 2
1.264 Since the cylinder moves without sliding, the centre of the cylinder rotates about the
point O, while passing through the common edge of the planes. In other words, the
point O becomes the foot of the instantaneous axis of rotation of the cylinder.
At any instant during this motion the velocity of the C.M. is v1.
When the angle (as shown in the figure) is b , we have N
mv 21
= mg cos b - N
R
(where N is the normal reaction of the edge)
mg
O
NR
or v 21 = gR cos b - (1)
m
From the energy conservation law,
2
1 v 21 1 v0
I0 - I = mg R (1 - cos b)
2 R2 2 0 R2
mR 2 3
But, I0 = + mR 2 = mR 2
2 2
(from the parallel axis theorem).
4
Thus, v 21 = v 20 + g R (1 - cos b) (2)
3
From Eqs. (1) and (2)
gR NR
v 20 = (7 cos b - 4) -
3 m
The angle b in this equation is clearly smaller than or equal to a, so putting b = a, we
get
gR N0R
v 20 = (7 cos a - 4) - (where N0 is the corresponding reaction.)
3 M
Note that N Ú N0. No jumping occurs during this turning if N0 7 0. Hence, v0 must
be less than

gR
vmax = (7 cos a - 4)
A 3
 1.0 m/s (on substituting values)
158 PART ONE PHYSICAL FUNDAMENTALS OF MECHANICS

1.265 Clearly the tendency of bouncing of the hoop will be maximum when the small body
A will be at the highest point of the hoop during its rolling motion. Let the velocity
of C.M. of the hoop equal v at this position. The static friction does no work on the
hoop, so from conservation of mechanical energy: E1 = E2
v0 2
mv 20 + mR 2 a b - mgR = m (2v)2 + mv 2 + mR 2 a b + mgR
1 1 1 1 1 v 2
or 0 +
2 2 R 2 2 2 R

or 3v 2 = v 20 - 2gR (1)
From the equation Fn  mwn for body A at final position, as shown in Fig. (b):

mg + N ¿ = mv2R = m a b R
v 2
(2)
R

As the hoop has no acceleration in vertical N'


direction, so for the hoop,
N + N ¿ = mg (3)
N' mg
From Eqs. (2) and (3), v0
v
R mg
mv 2 A
N = 2mg - (4) (a) (b) N
R
As the hoop does not bounce,
N Ú 0 (5)
So from Eqs. (1), (4) and (5),
8 gR - v 20
Ú 0 or 8gR Ú v 20
3R
Hence, v0 … 28 gR

1.266 Since the lower part of the belt is in contact with the rigid floor, velocity of this part
becomes zero. The crawler moves with velocity v, hence the velocity of upper part of
the belt becomes 2v by the rolling condition and kinetic energy of upper part is
given by

a b(2v)2 = mv 2
1 m
2 2
which is also the sought kinetic energy, assuming that the length of the belt is much
larger than the radius of the wheels.

1.267 The sphere has two types of motion, one is the rotation about its own axis and the
other is motion in a circle of radius R. Hence the sought kinetic energy is,
1 1
T = I1 v21 + I2 v22 (1)
2 2
1.5 DYNAMICS OF A SOLID BODY 159

where I1 is the moment of inertia about its own axis, and I2 is the moment of iner-
tia about the vertical axis, passing through O.
2 2
But, I1 = mr 2 and I2 = mr 2 + mR 2 (using parallel axis theorem) (2)
5 5
In addition
v v
v1 = and v2 = (3)
r R
Using Eqs. (2) and (3) in Eq. (1), we get

mv 2 a1 + b
7 2r 2
T¿ =
10 7R 2

1.268 For a point mass of mass dm, looked at from C rotating frame, the equation is
dm w œ = f + dm v2r œ + 2dm (v œ * ␻)
where r  radius vector in the rotating frame with respect to rotation axis and
v  velocity in the same frame. The total centrifugal force is clearly
Fcf = a dm v2r œ = mv2 RC
where RC is the radius vector of the C.M. of the body with respect to rotation axis. Also
Fcor = 2m vCœ * v
where we have used the definitions,
m RC = a dm r œ and m v¿C = a dm v¿ .

1.269 Consider a small element of length dx at a distance x from the


point C, which is rotating in a circle of radius r = x sin u. O
Now, mass of the element = a b dx
m
B
l r
So, centrifugal force acting on this element dx
x
= a b dx v2 x sin u
m C
l
and moment of this force about C is
A

| dN | = a b dx v2x sin u # x cos u


m
O
l
mv2
= sin 2ux 2dx
2l
and hence, total moment
l>2
mv2 1
N = 2 sin 2ux 2dx = mv2l 2 sin 2u
3 2l 24
0
160 PART ONE PHYSICAL FUNDAMENTALS OF MECHANICS

1.270 Let us consider the system in a frame rotating with the rod. In this frame, the rod is
at rest and experiences not only the gravitational force mg and the reaction force R,
but also the centrifugal Fcf .
In the considered frame, from the condition of equilibrium, i.e., N 0z 0

l
or Ncf = mg sin u (1)
2

where Ncf is the moment of centrifugal force about O. To calculate Ncf, let us consider
an element of length dx, situated at a distance x from the point O. This element
is subjected to a horizontal pseudo force (m /l )dx v2x sin u. The moment of this
pseudo force about the axis of rotation through the point O is

dNcf = a b dx v2 x sin ux cos u


m 0
l

mv2 C
= sin u cos ux 2dx Fcf
l

l mg
mv2
So, Ncf = sin u cos u x 2dx
3 l
0

mv2l 2
= sin u cos u (2)
3
It follows from Eqs. (1) and (2) that,

cos u = a b or u = cos-1 a b
3g 3g
(3)
2v2l 2v2l

1.271 When the cube is given an initial velocity on


the table in some direction (as shown in
figure) it acquires an angular momentum
about an axis on the table perpendicular to
the initial velocity and (say) just below the G N
centre of gravity (C.G.) This angular momen- x
tum will disappear when the cube stops and Initial
this can only be due to a torque. Frictional velocity
forces cannot do this by themselves because Axis to the
they act in the plane containing the axis. initial velocity on
But if the forces of normal reaction act ec- Initial the table
centrically (as shown in figure), their torque angular momentum
1.5 DYNAMICS OF A SOLID BODY 161

can bring about the vanishing of the angular momentum. We can calculate the dis-
tance ¢ x between the point of application of the normal reaction and the C.G. of
the cube as follows. Take the moment about C.G. of all the forces. This must van-
ish because the cube does not turn on the table.
Then if the force of friction is fr
a
fr = N¢x
2
But N = mg and fr = kmg, so
ka
¢x =
2

1.272 As the rod is smooth, in the process of motion of the given system, the kinetic en-
ergy and the angular momentum relative to rotation axis do not vary. Hence, it fol-
lows that
1 Ml 2 2 1 1 Ml 2 2
v0 = m (v2l 2 + v ¿ 2) + v
2 3 2 2 3
(where v is the final angular velocity of the rod)
Ml 2 Ml 2
and v0 = v + ml 2v
3 3
From these equations we obtain
v0
v =
(1 + 3m /M )

and v 0l
v¿ =
21 + 3m/M

1.273 As surface is smooth, for further motion the frame attached with the C.M. is inertial.
Due to hitting of the ball, the angular impulse received by the rod about the C.M. is
equal to p l /2. If v is the angular velocity acquired by the rod, we have

ml 2 pl 6p
v = or v = (1)
12 2 ml

In the frame of C.M., the rod is rotating about an axis passing through its mid point
with the angular velocity v. Hence the force exerted by one half on the other =
mass of one half * acceleration of C.M. of that part, in the frame of C.M.

9p2
a v2 b = m
m l v2l
= = = 9N
2 4 8 2ml
162 PART ONE PHYSICAL FUNDAMENTALS OF MECHANICS

1.274 (a) In the process of motion of the given system the kinetic energy and the angular
momentum relative to rotation axis do not vary. Hence it follows that

mv 2 = mv ¿ 2 + a bv
1 1 1 Ml 2 3
2 2 2 3

l l Ml 2
and mv = mv ¿ + v
2 2 3
From these equations we obtain
3m - 4m
v¿ = a b v and v =
4v
3m + 4M l (1 + 4m>3M)

As v¿ c c v, so in vector form

3m - 4M
v¿ = a b v
3m + 4M
(b) Obviously the sought force provides the centripetal acceleration to the C.M. of the
rod and is
Fn = mwcn
l 8Mv 2
= Mv2 =
2 l (1 + 4M>3m)2

1.275 (a) About the axis of rotation of the rod, the angular momentum of the system is
conserved. Thus if the velocity of the flying bullet is v,

mvl = a ml 2 + b v
Ml 2
3

mv 3mv
v = = (as m V M ) (1)
m + (M>3)l Ml

Now from the conservation of mechanical energy of the system (rod with bullet)
in the uniform field of gravity,

a ml 2 + b v2 = (M + m) g (1 - cos a)
1 Ml 2 l
(2)
2 3 2
l
[because C.M. of rod raises by the height (1 - cos a)].
2
Solving Eqs. (1) and (2), we get

v = a b
M 2 a 6g a
gl sin and v = sin
m A3 2 A l 2
1.5 DYNAMICS OF A SOLID BODY 163

¢p = c m (vl ) + M a v b d - mv
l
(b) Sought
2

where vl is the velocity of the bullet and v (l>2) equals the velocity of C.M. of
the rod after the impact. Putting the value of v and v , we get

1 gl a
¢p = mv = M sin
2 A6 2
This is caused by the reaction at the hinge on the upper end.
(c) Let the rod start swinging with angular velocity v¿ in this case. Then, like in
part (a),

mvx = a + mx 2 b v¿ or v¿ L
Ml 2 3mvx
3 Ml 2
Final momentum is
t
M M 3 x
pf = mx v¿ + y v¿ dy L v¿l L mv
3 l 2 2 l
0

¢p = pf - pi L mv a - 1b
3x
So,
2l
This vanishes for x L 2>3l .

1.276 (a) As force F on the body is radial so its angular momentum about the axis becomes
zero and the angular momentum of system about the given axis is conserved.
Thus

v or v = v0 a 1 + b
MR 2 MR 2 2m
v0 + m v0R 2 =
2 2 M
(b) From the equation of the increment of the mechanical energy of the system
¢T = Aext

v - a + mR 2 b v20 = Aext
1 MR 2 2 1 MR 2
2 2 2 2
Putting the value of v from part (a) and solving we get
mv20R 2
a1 + b
2m
Aext =
2 M

1.277 (a) Among the external forces are, the reaction force by the support at the centre of
the disk, weight of disk, and weight of the man, acting on the system (disk man).
164 PART ONE PHYSICAL FUNDAMENTALS OF MECHANICS

First two forces pass through the centre O of the disk, so no torque about O,
that’s why about the vertical axis of rotations OZ is also zero. The weight of the
man is collinear to axis OZ, so no torque about this axis.
Let the man m1 stand at point A (say) at the edge of the disk. Let Ox be a fixed
line and such that at t  0 the line joining O to A coincides with it. At time t, let
⬔AOx  ␪ and ⬔AOP  w¿ where P denotes the position of the . man on the disk
.
(relative to disk). The angle xOP increases at .the rate
. w¿ - u, therefore the ve-
locity of the man at right angles to OP is R ( w .- u) and
. its angular momentum
about the rotation axis Oz of the disk is mR 2 ( w¿ - u). .
The disk has angular momentum about the Oz axis which becomes (m2R 2/2) u in
the opposite sense to conserve the angular momentum of the (disk + man) sys-
tem, which is zero.

m2R 2 . . .
Therefore, u - m1R 2(w ¿ - u ) = 0
2
m2R 2
m1R 2w¿ = a + m1R 2 b u
.
2 z

m1
or du = dw¿ A
m2
J m1 + a bK
2 O x
j'
u w¿
P
m1
On integrating du = - d w¿
3 3 £ m + m2 ≥
0 0 1
2

m1 w¿
u = a b
or m1 + m2>2 (1)

This gives the total angle of rotation of the disk.


(b) From Eq. (1)

m1 m1
= - a b = - a b
du dw¿ v¿(t)
dt m1 + m2>2 dt m1 + m2>2 R

Differentiating with respect to time

m1
= -a b
d2u 1 dv¿(t)
dt 2 m1 + m2>2 R dt
1.5 DYNAMICS OF A SOLID BODY 165

Thus, the sought force moment from the equation Nz = Ib z is

m2R 2 d 2 u m2R 2 m1
a b
1 dv¿(t)
Nz = = -
2 dt 2 2 m1 + m2>2 R dt
m1m2R dv ¿(t)
Hence, Nz = -
2m1 + m2 dt
1.278 (a) From the law of conservation of angular momentum of the system relative to ver-
tical axis z, it follows that
I1 v1z + I2v2z = (I1 + I2)vz
Hence, vz = (I1v1z + I2v2z )>(I1 + I2) (1)
Note that for vz 7 0, the corresponding vector ␻ coincides with the positive di-
rection to the z-axis, and vice versa. As both disks rotate about the same verti-
cal axis z, thus in vector form
␻ = (I1 ␻ 1 + I2 ␻ 2)>(I1 + I2)
However, the problem makes sense only if ␻ 1 cc ␻ 2 or ␻ 1 c T ␻ 2.
(b) From the equation of increment of mechanical energy of a system: Afr = ¢T
1 1 1
Afr = (I1 + I2) v2z - I1 v21 + I2 v22z
2 2 2
Using Eq. (1)
I1I2
Afr = (v1z - v2z )2
2(I1 + I2)

1.279 For the closed system (disk rod), the angular momentum is conserved about any
axis. Thus from the conservation of angular momentum of the system about the ro-
tation axis of rod passing through its C.M. gives:
l l hml 2
mv = mv¿ + v (1)
2 2 12
(where v ¿ is the final velocity of the disk and v angular velocity of the rod).
For the closed system linear momentum is also conserved. Hence,
mv = mv ¿ + h mvC (2)
(where vC is the velocity of C.M. of the rod).
From Eqs. (1) and (2) we get
lv
vC = and v - v ¿ = hvC
3
166 PART ONE PHYSICAL FUNDAMENTALS OF MECHANICS

Applying conservation of kinetic energy, as the collision is elastic


1 1 1 1 hml 2 2
mv 2 = mv ¿2 + hmv 2C + v (3)
2 2 2 2 12
or v 2 - v ¿2 = 4hv 2C and hence v + v ¿ = 4vC
4 - h 12v
Then v¿ = v and v = l/2
4 + h (4 + h)l
Vectorially, noting that we have taken v¿ parallel to v C
4 - h
v¿ = a bv
4 + h
So, v¿ = 0 for h = 4 and v¿ Tc v for h 7 4
Note: Instead of Eq. 3 one can also write the equation of coefficient of restitution.

1.280 See the diagram in the book (Fig. 1.72).


(a) When the shaft BB¿ is turned through 90, the platform must start turning with
angular velocity æ so that the angular momentum remains constant. Here,
I0v0
(I + I0)æ = I0 v0 or æ =
I + I0
The work performed by the motor is therefore,
2 2
1 1 I 0 v0
(I + I0)æ 2 =
2 2 I + I0
If the shaft is turned through 180, angular velocity of the sphere changes sign.
Thus from conservation of angular momentum,
I æ - I0v0 = I0v0
Here I0 v0 is the complete angular momentum of the sphere, i.e., we assume
that the angular velocity of the sphere is just - v0. Then
v0
æ = 2I0
I
and the work done must be
1 1 1 2I 20 v20
I æ 2 + I0 v20 - I0 v20 =
2 2 2 I
(b) In the first part of (a), the angular momentum vector of the sphere is processing
with angular velocity æ . Thus torque needed is
I 20 v20
I0v0 æ =
I + I0
1.5 DYNAMICS OF A SOLID BODY 167

1.281 The total centrifugal force can be calculated by,


O'
l0 T2
m 2 1
v xdx = ml0 v2
3 l0 2 A B
0
Then for equilibrium, mg mg
l l0 T1
O
(T2 - T1) = mg
2 2
1
and T2 + T1 = ml0v2
2
Thus T1 vanishes, when
2g 2g
v2 = ,v = = 6 rad>s
l A t
l0
Then T2 = mg = 25 N
l

1.282 See the diagram in the book (Fig. 1.72).


(a) The angular velocity v about OO can be resolved into a com- 90−

ponent parallel to the rod and a component v sin u perpendi- 90−

cular to the rod through C. Then component parallel to the rod


does not contribute to the angular momentum.
1 90−
M = I v sin u = ml 2 v sin u
12 90−
1
Also, Mz = M sin u = ml 2 v sin2 u
12
This can be obtained directly also.

(b) The modulus of M does not change but the modulus of the change of M is
1
| ¢M |  2M sin (90° - u) = ml 2 v sin 2 u
12

(c) Here, M ⬜ = M cos u = I v sin u cos u MI


t
wd MI
Now, ` ` = I v sin u cos u
dM vdt 1 Mz
= ml 2v2 sin2 u
dt dt 24 Mz

as M processes with angular velocity v.

1.283 (a) Here M = Iv is along the symmetry axis. It has two


components, the part I v cos u is constant and the part M ⬜ = I v sin u precesses.
Then,
168 PART ONE PHYSICAL FUNDAMENTALS OF MECHANICS

M^
M^

mg

` ` = I v sin u v¿ = mgl sin u


dM
dt
mgl
or v¿ = precession frequency = = 0.7 rad/s
Iv
(b) This force is the centripetal force due to precession. It acts inward and has the
magnitude

| F | = ` a miv¿ 2ri ` = m v¿ 2 l sin u = 12 mN

ri is the distance of the i th element from the axis. This is the force that the table
will exert on the top. See the diagram in the answer sheet.

1.284 See the diagram in the book (Fig. 1.73).


The moment of inertia of the disk about its symmetry axis is 1>2 mR 2 . If the angular
velocity of the disk is v , then the angular moment is 1>2 m R 2v . The precession fre-
quency being 2pn, we have

` ` = mR 2 v * 2pn
dM 1
dt 2

This must equal m (g + w) l, the effective gravitational torques (g being replaced by


g + w in the elevator). Thus,
(g + w)l
v = = 300 rad/s
pR 2n

1.285 The effective g is 2g2 + w 2 inclined at angle tan–1 w/g with the vertical. Then
with reference to the new “vertical” we proceed as in solution of problem 1.283.
Thus,

ml 2g 2 + w 2
v¿ = = 0.8 rad/s
Iv
1.5 DYNAMICS OF A SOLID BODY 169

v
The vector ␻ ¿ forms an angle u = tan -1 = 6° with the normal vertical.
g

1.286 The moment of inertia of the sphere is 2>5 mR 2 and hence the value of angular
momentum is 2>5 mR 2 v. Since it processes at speed v¿ the torque required is
2
mR 2vv¿ = F ¿l
5

2
So, F¿ = mR 2vv¿>l = 300 N (the force F  must be vertical)
5

1.287 The moment of inertia is 1>2 mr 2 and angular momentum is 1>2 mr 2v. The axle
oscillates about a horizontal axis making an instantaneous angle
2pt
w = wm sin
T

This means that there is a variable precession with a rate of precession dw/dt. The
maximum value of this is 2pwm /T. When the angle between the axle and the axis is
at its maximum value, a torque I vÆ acts on it which is equal to

1 2pw pmr 2v w
mr 2v =
2 T T

The corresponding gyroscopic force will be

pmr 2v w
= 90 N
lT

1.288 The revolutions per minute of the flywheel being n, the angular momentum of the
flywheel is l * 2pn. The rate of precession is v/R.
Thus, N = 2pINv>R = 5.97 kNm

1.289 As in the previous problem, a couple 2pInv>R must come in play. This can be done
if a force, 2pInv/Rl acts on the rails in opposite directions in addition to the centrifu-
gal and other forces. The force on the outer rail is increased and that on the inner
rail decreased. The additional force in this case has the magnitude 1.4 kN.
1.6 ELASTIC DEFORMATIONS OF A SOLID BODY 170

1.6 Elastic Deformations of a Solid Body

1.290 Variation of length with temperature is give by

¢l
lt = l0 (1 + a¢t) or = a¢t = e (1)
l0

s
But, e =
E

Thus, s = a ¢tE , which is the sought stress of pressure.


Putting the value of a and E from Appendix and taking ¢t = 100°C, we get
s = 2.2 * 103 atm

1.291 (a) Consider a transverse section of the tube and concentrate on an element which
subtends angle ¢w at the centre. The forces acting on a portion of length ¢l on
the element are:

(1) Tensile forces side ways of magnitude s¢r¢l.


The resultant of these is 2 s¢r ¢l sin ¢
>2 L s¢rl ¢
radically towards
the centre.
(2) The force due to fluid pressure = pr ¢
¢l .
Since these balance, we get pmax L sm ¢r>r
where sm is the maximum tensile force.
Putting the values, we get pmax = 19.7 atm.

(b) Consider an element of area dS = p (r ¢u>r)2 about z-axis


chosen arbitrarily. There are tangential tensile forces all around the ring of the
cap. Their resultant is

¢u ¢u
s c 2p ar b ¢r d sin
2 2

Hence, in the limit,

pmp a b = smp a b ¢r ¢u
r¢u 2 r¢u 2
2 2

2 sm ¢r
or pm = = 39.5 atm
r
1.6 ELASTIC DEFORMATIONS OF A SOLID BODY 171

1.292 Let us consider an element of rod at a distance x from its rotation axis (see figure).
From Newton’s second law in projection form directed towards the rotation axis,
m 2
- dT = (dm) 2x = x dx
l
m 2 x2
On integrating -T = + C (constant)
l 2
l
But at x =  or free end, T = 0
2
mv2 l 2 mv2l x dx
Thus, 0 = + C or C = -
2 4 8

a - b
m 2 l x2
Hence T =
2 4 l
m 2l
Thus Tmax = (at mid point)
8
Condition required for the problem is
Tmax = S sm

mv2l 2 2 sm
So, = S sm or v =
8 l B r
Hence the sought number of rps
1 2 sm
n = = (using the table n = 0.8 * 103 rps)
2p pl A r

1.293 Let us consider an element of the ring (see figure). From Newton’s law Fn = mwn
for this element, we get

Td u = a du b 2r (see solution of problem 1.93 or 1.92)


m
2p
m 2
So, T = r
2p T
Condition for the problem is
T m 2r dq
… sm or … sm
pr 2 2p2r 2 r
2p2 smr sm T
or 2max = =
pr 2(2prr) rr 2
172 PART ONE PHYSICAL FUNDAMENTALS OF MECHANICS

Thus, sought number of rps is


max 1 sm
n = =
2p 2pr A r

Using the table of appendices n = 23 rps.

1.294 Let the point O descend by the distance x (see figure). From the condition of equi-
librium of point O,
mg mg
2T sinu = mg or T = = 2(l>2)2 + x 2 (1)
2sin u 2x
T d2
Now, 2
= s = eE or T = eEp (2)
p(d>2) 4

(Here s is stress and e is strain.)


In addition to this,

2(l>2)2 + x 2 - l>2 l/2 l/2


1 + a b - 1
2x 2
e = = (3) x
l>2 B l T T
mg
From Eqs. (1), (2) and (3)
x mgl
x - =
21 + 12x>l22 pEd 2

x = l a b
mg 1>3
or 2
= 2.5 cm
2pEd

1.295 Let us consider an element of the rod at a distance x from the free end (see figure).
For the considered element T - T ¿ are internal restoring forces which produce
elongation and dT provide the acceleration to the element. For the element, from
Newton’s law
F0 F0
dT = (dm) w = a dx b
m
= dx
l m l

As free end has zero tension, on integrating the above expression,


T x
F0 F0
dT = dx or T = x
3 l 3 l
0 0
1.6 ELASTIC DEFORMATIONS OF A SOLID BODY 173

Elongation in the considered element of length dx is given by


s T Fxo xdx
0j = (x) dx = dx =
E SE SEl
l T T+dT
F0 F0l
Thus, total elongation j = xdx = x
SEl 3 2SE dx
x

Hence, the sought strain is


j F0
s = =
l 2SE

1.296 Let us consider an element of the rod at a distance r from its rotation axis. As the
element rotates in a horizontal circle of radius r, we have from Newton’s second law
in projection form directed toward the axis of rotation
T - (T + dT ) = (dm) 2r

- dT = a dr b 2r =
m m 2
or rdr
l l

At the free end tension becomes zero. Integrating the above expression we get
0 l
m
- dT = 2 rdr
3 l 3
T r T dT
r
-
a b = a1 - b
m 2 l2 r2 m 2l r2 dr
Thus, T =
l 2 2 l2
Elongation in elemental length dr is given by
s(r) T
0j = dr = dr
E SE
(where S is the cross sectional area of the rod and T is the tension in the rod at the
considered element)

a 1 - 2 b dr
m 2l r2
or 0j =
2SE l

Thus, the sought elongation


l

a 1 - 2 b dr
m 2l r2
j = dj =
3 2SE 3 l
0
174 PART ONE PHYSICAL FUNDAMENTALS OF MECHANICS

m 2l 2l (Sl r) 2 3
or j = = l
2SE 3 3SE
1 r 2l 3
= (where r is the density of the copper)
3 E

1.297 Volume of a solid cylinder


V = pr 2l
¢V p2r¢rl pr 2 ¢l 2¢r ¢l
So, = 2
+ 2
= + (1)
V pr l pr l r l
But longitudinal strain ¢l/l and accompanying lateral strain ¢r/r are related as
¢r ¢l
= -m (2)
r l
Using Eq. (2) in Eq. (1), we get
¢V ¢l
= (1 - 2m) (3)
V l

¢l - F>pr 2
But =
l E
(Because the increment in the length of cylinder ¢l is negative.)
¢V -F
So, = (1 - 2m) (where m is the Poisson’s ratio for copper)
V pr 2E
- Fl
Thus, ¢V = (1 - 2m)
E
= - 1.6 mm3
(Negative sign means that the volume of the cylinder has decreased.)

1.298 (a) As free end has zero tension, thus the tension in the rod at a vertical distance y
from its lower end
m
T = gy (1)
l
Let 0l be the elongation of the element of length dy, then
s(y)
0l = dy
E
T mgydy rgydy
= dy = = (where r is the density of copper)
SE SlE E
1.6 ELASTIC DEFORMATIONS OF A SOLID BODY 175

Thus, the sought elongation is


l

rgl 2>E
ydy 1
¢l = 0l = rg = (2)
3 3 E 2
0

(b) If the longitudinal (tensile) strain is e = ¢l>l, the accompanying lateral (compres-
sive) strain is given by
¢r
e¿ = = - me (3)
r
Then, since V = pr 2l, we have
¢V 2¢r ¢l
= +
V r l

¢l
= (1 - 2m) (using Eq. 3)
l
(where ¢l/l is given in part (a) and m is the Poisson ratio for copper).

1.299 (a) Since the hydrostatic pressure is uniform, the stress on each face of the bar is the
same (see figure (a)). The change in length of the bar can be thought of as the
sum of changes in length that would occur in the three independent cases sh-
own in the figures (b), (c) and (d).

p p p p

p
(a) (b)

p
p
(c) (d)

If we push on the ends of the block with a pressure p, the compressional strain
is p >E, and it is negative,

¢l1 p
= -
l E
176 PART ONE PHYSICAL FUNDAMENTALS OF MECHANICS

If we push on the two sides of the block with pressure p, the compressional st-
rain is again p>Y, but now we want the lengthwise strain. We can get that from
the sideways strain multiplied by - m . The sideways strain is
¢w p
= -
w E
¢l2 p
So, = + m
l E
If the we push on the top of the block, the compressional strain in once more
p>E , and the corresponding strain in the sideways direction is again - mp>E. We
get

¢l3 p
= + m
l E
Combining the results of the three problems i.e., taking ¢l = ¢l1 + ¢l2 + ¢l3,
we get
¢l p
= - (1 - 2m) (1)
l E
The problem is, of course, symmetrical in all three directions; it follows that
¢w ¢h p
= = - (1 - 2m) (2)
w h E
The change in the volume under hydrostatic pressure is also of some interest.
Since V = lwh, we can write, for small displacements.
¢V ¢l ¢w ¢h
= + +
V l w h
Using Eqs. (1) and (2), we have
¢V p
= - 3 (1 - 2m) (3)
V E
(b) But volume strain
¢V p
= - (where K is called the bulk modulus) (4)
V K
From Eqs. (3) and (4), we have
E
K =
3 (1 - 2m)
Hence the compressibility
1 3 (1 - 2m)
b = =
K E
1.6 ELASTIC DEFORMATIONS OF A SOLID BODY 177

1.300 A beam clamped at one end and supporting an applied load at the free end is called
a cantilever. The theory of cantilevers is discussed in the advanced text book on me-
chanics. The key result is that elastic forces in the beam generate a couple, whose
moment, called the moment of resistances, balances the external bending moment
due to weight of the beam, load etc. The moment of resistance, also called internal
bending moment (I.B.M) is given by
I.B.M. = EI/R
Here R is the radius of curvature of the beam at the
representative point (x, y ). I is called the geometrical ds
moment of inertia of the cross section relative to the axis
z
passing through the natural layer which remains un-
scratched Fig. (a).
(a)
I = z 2 dS (1)
3
The section of the beam beyond P exerts the bending mo- h
x
ment N (x) and we have,
EI p(x,y)
= N (x) (2) y
R (b)

If there is no load other than that due to the weight of


the beam, then
1
N (x) = rg(l - x)2 bh
2
(where r = density of steel).
Hence, at x = 0
rgl 2bh
a b =
I
R 0 2EI

Here, b = width of the beam perpendicular to paper.


h/2
bh3
Also, I = z 2 bdz =
3 12
-h/2

6rgl 2
a b =
I
Hence,
R 0 Eh2

1 Eh 2
R = = 0.121 km
6 rgl 2
178 PART ONE PHYSICAL FUNDAMENTALS OF MECHANICS

1.301 We use the equation given above and use the result that when y is small
1 d 2y d 2y N (x)
= ; thus, =
R dx 2 dx 2 EI
(a) Here N (x) = N0 is a constant. Then integration gives
dy N0x
= + C1
dx EI

a b = 0,
dy
But, for x = 0, so C1 = 0.
dx

N0x 2
Integrating again, y =
2EI
where we have used y = 0 for x = 0 to set the constant of integration at zero.
This is the equation of a parabola. The sag of the free end is
N0l 2
l = y (x = l ) =
2EI

(b) In this case N (x) = F (l - x) because the load F at the extremity is balanced by
a similar force at F directed upward and they constitute a couple. Then

d 2y F (l - x)
=
dx 2 EI
dy F (lx - x 2>2)
Integrating, = + C1
dx El
As before C1 = 0. Integrating again, using y = 0 for x = 0,
F 1lx 2>2 - x 3>62
a here l = b
Fl 3
y =
EI 3EI
Here for a square cross section
a>2
a4
I = z 2 adz =
3 12
-a>2

1.302 One can think of it as analogous to the previous F/2 F/2


case but with a beam of length l> 2 loaded upward
by a force F/2.
Fl 3
Thus, l =
48 EI
on using the last result of the previous problem.
1.6 ELASTIC DEFORMATIONS OF A SOLID BODY 179

1
1.303 (a) In this case N (x) = rgbh (l - x)2 (where b = width of the girder).
2
bh3
Also I =
12
Ebh2 d 2y r gbh 2
Then, = (l - 2lx + x 2)
12 dx 2 2

al 2x - lx 2 + b
dy 6rg x3
Integrating, =
dx Eh2 3
dy
Using = 0 for x = 0
dx

a b
6rg l 2x 2 lx 3 x4
Integrating again, y = - +
Eh2 2 3 12
6rgl 4 6rgl 4 3 3rgl 4
a b =
1 1 1
Thus, l = - + =
Eh2 2 3 12 2
Eh 12 2Eh2

d 2y
(b) As before, EI = N (x) where N (x) is the bending moment due to section PB.
dx 2
Thus, bending moment is clearly
2l
2l x
A y B
N = w dj(j - x) - wl (2l - x)
3 P
x

= w a 2l 2 - 2xl + b –wl (2l - x)


x2
2

= w a - xl b (here w = r gb h is weight of the beam per unit length)


x2
2

= w a b + c0
dy x3 x 2l
Now integrating, El -
dx 6 2

dy wl 3
or since = 0 for x = l, c0 =
dx 3

Integrating again, EIy = w a b +


x4 x 3l wl 3x
- + c1
24 6 3
As y = 0 for x = 0, c1 = 0. From this we find
5wl 4>24 5rgl 4
l = y (x = l ) = =
EI 2Eh2
180 PART ONE PHYSICAL FUNDAMENTALS OF MECHANICS

1.304 The deflection of the plate can be noticed by going to a co-rotating frame. In this
frame each element of the plate experiences a pseudo force proportional to its mass.
These forces have a moment which constitutes the bending moment of the problem.
To calculate this moment we note that the acceleration of an element at a distance
j from the axis is a = jb and the moment of the forces exerted by the section
between x and l is
x
1
N = rlhb j2dj = rlh b (l 3 - x 3)
3 3
l

From the fundamental equation


d 2y 1
EI = rlh b(l 3 - x 3)
dx 2 3
+h>2
lh3
The moment of inertia I = z 2Idz =
3 12
-h>2

Note that the neutral surface (i.e. the surface which contains lines which are neither
stretched nor compressed) is a vertical plane here and z is perpendicular to it.
d 2y 4rb 3
= (l - x 3)
dx 2 Eh2

al x - b + c1
dy 4rb 3 x4
Integrating =
dx Eh2 4

dy
Since = 0, for x = 0, c1 = 0. Integrating again,
dx

a b + c2
4rb l 3x 2 x5
y = -
Eh2 2 20
c2 = 0 (because y = 0 for x = 0)

9rbl 5
Thus, l = y (x = l ) =
5Eh2

1.305 (a) Consider a hollow cylinder of length l, outer radius r + ¢r, inner radius r, fixed
at one end and twisted at the other by means of a couple of moment N. The an-
gular displacement w, at a distance l from the fixed end, is proportional to both
l and N. Consider an element of length dx at the twisted end. It is moved by an
angle w as shown. A vertical section is also shown and the twisting of the par-
allelopipe of length l and area ¢r dx under the action of the twisting couple can
1.6 ELASTIC DEFORMATIONS OF A SOLID BODY 181

be discussed by elementary means. If f is the tangential force generated, then


shearing stress is ƒ/¢r dx and this must equal
r
r

Gu = G , since u =
l l
r

Hence, f = G¢r dx
l
The force f has moment fr about the axis and so the total moment is

2 2pr 3 ¢r

N = G¢r r dx = G
l 3 l

(b) For a solid cylinder we must integrate over r. Thus


r
2pr 3 dr
G pr 4 G

N = =
3 l 2l
0

dx

q
F

f
dx
dr

d2>2
2pr 3 dr
G p
1.306 Clearly N = = G
(d 42 - d 41)
3 l 32l
d1>2

Using G = 81 GPa = 8.1 * 1010 Nm-2

d2 = 5 * 10-2 m, d1 = 3 * 10-2 m

p

= 2.0° = radians, l = 3 m
90

p * 8.1 * p
N = (625 - 81) * 102 Nm
32 * 3 * 90

= 0.5033 * 103 N # m L 0.5 kNm


182 PART ONE PHYSICAL FUNDAMENTALS OF MECHANICS

1.307 The maximum power that can be transmitted by means of a shaft rotating about its
axis is clearly N where N is moment of the couple producing the maximum per-
missible torsion,
. Thus,
pr 4G
#
P = = 16.9 kW
2l

1.308 Consider an elementary ring of width dr at a distant r from the axis. The part outside
dN
exerts couple N + dr on this ring while the part inside exerts a couple N in the
dr
opposite direction. We have for equilibrium
dN
dr = - dIb
dr
where dI is the moment of inertia of the elementary ring, b is the angular accelera-
tion and minus sign is needed because the couple N (r) decreases, with distance,
vanishing at the outer radius, N (r2) = 0. Now
m
dI = 2pr dr r 2
p (r 22 - r 21 )

2mb
Thus, dN = r 3 dr
r 22 - r 21

1 mb
On integration, N = (r 4 - r 41 )
2 (r 2 - r 21 ) 2
2

dN
N + dr dr

r
dr

1.309 We assume that the deformation is wholly due to external load, neglecting the effect
of the weight of the rod (see next problem). Then a well known formula says, elas-
tic energy per unit volume
1 1
= stress * strain = se
2 2

1 m 2
This gives Ee L 0.04 kJ for the total deformation energy.
2 r
1.6 ELASTIC DEFORMATIONS OF A SOLID BODY 183

1.310 When a rod is deformed by its own weight the stress increases as one moves up, the
stretching force being the weight of the portion below the element considered.
The stress on the element dx is

rpr 2 (l - x) g>pr 2 = rg (l - x) x

The extension of the element is dx


¢dx = d¢x = rg(l - x) dx>E
Integrating we get the extension of the whole rod as l–x
1 rgl 2
¢l =
2 E
The elastic energy of the element is
1 rg ( l - x)
rg (l - x) pr 2dx
2 E
Integrating
1 pr 2r2g2l 3 ¢l 2
= pr 2lE a b
2
¢U =
6 E 3 l

1.311 The work done to make a loop out of a steel band appears as the elastic energy of
the loop and may be calculated from the same.
If the length of the band is l, the radius of the loop R = l>2p. Now consider an element
ABCD of the loop. The elastic energy of this element can be calculated by the same
sort of arguments as used to derive the formula for internal bending moment.
Consider a fiber at a distance z from the neutral surface PQ. This fiber experiences
a force p and undergoes an extension ds where d = Zd
, while PQ = s = Rd
.
Thus strain ds /s = Z /R #
If a is the cross sectional area of the fiber, the elastic energy associated with it is
ds
E a b Rd
a
1 Z 2
2 R B C

Summing over all the fibres we get p P Q p


A D
EI
E ld
R
a aZ 2 = R
2R 2R d

For the whole loop this gives, using d w = 2p


3

El p 2EI p2
=
R l
184 PART ONE PHYSICAL FUNDAMENTALS OF MECHANICS

d>2
hd3
Now, I = z 2h dZ =
3 12
-d>2

So the energy is
1 p2Ehd3
= 0.08 kJ
6 l
Alternate:
Suppose that the steel band was made into a hoop of radius R, then length of the
hoop l = 2pR.
Consider an infinitesimally thin section of radius r and thickness dr in the hoop. The
length of this section of the hoop is 2pr. Hence the longitudinal strain correspon-
ding to this section is
2pr - 2rR r
e = = - 1
2pR R
So elastic energy density is

E e = E a - 1b
1 2 1 r 2
u =
2 2 R
Thus the sought work done is nothing but elastic energy stored.
R + d2

E a - 1 b 2prdr h =
1 x 2 1 p2 Ehd3
U = 1udV = (on integrating)
Ld 2 R l
R-2

1.312 A rod should be treated as a solid cylinder. Let the upper end of the cylindrical rod be
fixed and let a couple be applied to the lower end in a plane perpendicular to its length
(with its axis coinciding with that of the cylinder) twisting it through an angle u (radi-
ans). This is an example of “pure shear”. In equilibrium position the twisting couple is
equal and opposite to the restoring couple. Let us calculate the value of this couple.
Imagine the cylinder to consist of a large number of co-axial, hollow cylinders, and con-
sider one such hollow cylinder of radius v, and radial thickness dr Fig. (a). Each ra-
dius of the lower end is turned through the same angle u, but the displacement is the
greatest at the rim, decreasing as the centre is approached, where it is reduced to zero.

O'
A

r O dr l
r f
B'
B O
r q B'
B
(a) (b)
1.6 ELASTIC DEFORMATIONS OF A SOLID BODY 185

Let AB Fig. (b) be a line, parallel to the axis, before the cylinder is twisted. On twist-
ing, since the point B shifts to B ¿, the line AB takes up the position AB ¿. The angle
through which this hollow cylinder is sheared is, therefore, BAB ¿ = f, say. Then
clearly,

BB¿ = lf (see Figs. a and b)

Also BB¿ = ru, so, angle of shear or shear strain

ru
f =
l

Obviously, f will have the maximum value where r is the greatest, i.e., the maxi-
mum strain is on the outermost part of the cylinder, and the least, on the innermost.
In other words, the shearing stress is not uniform all through.
Thus, although the angle of shear is the same for any one hollow cylinder, it is dif-
ferent for different cylinders, being the greatest for the outermost and the least for
the innermost one.

shearing stress F
Since G = =
strain or angle of shear f
Gru
We have F = Gf =
l

Now, face area of this hollow cylinder = 2pr dr .

Gru u
Therefore total shearing force this on area = 2pr dr * = 2pG r2dr
t l
Therefore, moment of this force about the axis OO ¿ Fig. (b) of the cylinder is
equal to
2pG ur2dr r 2pG ur3dr
=
l l

Integrating this expression between the limits, r = 0 and r = r, we have total twi-
sting couple on the cylinder,
r
u
N (u) = 1 2pG 2p G r3dr
0 l
r

c d =
2pG u 2pG u r4 r 2pG ur 4 pGur 4
= r3dr = = = Cu
l L l 4 0 2l 2l
0
186 PART ONE PHYSICAL FUNDAMENTALS OF MECHANICS

where C = pGr 4>2l is twisting couple per unit twist of the cylinder or torsional rigid-
ity of the cylinder.
Thus, when the rod is twisted through an angle u, a couple

pr 4G
N (u) = u
2l
appears to resist this. Work done in twisting the rod by an angle
or elastic ener-
gy stored is then

pr4G 2
U = N (u) d(u) =
= 7 J (on substituting values)
L 4l
0

1.313 The energy between radii r and r + dr is given by differentiation of

pr 4G 2
U =
(see solution of problem 1.312)
4l

pr 3dr
So, dU = G
2
l

dU pr 3dr G
2 1 G
2r 2
Its density is u = = =
dV 2prdr l l 2 l2

Alternate:
The sought energy density
1
u = Gf2 (where f is angle of shear or shear strain)
2

But, f = ar b (see solution of problem 1.312)


l

1 G
2r 2
So, u =
2 l2

1.314 The energy density is as usual 1>2 stress * stress. Stress is the pressure rgh. Strain
is b * rgh by definition of b . Thus

1
u = b (rgh)2 = 23.5 kJ/m3 (on substituting values)
2
1.7 HYDRODYNAMICS 187

1.7 Hydrodynamics
1.315 Between points 1 and 2, fluid particles are in nearly circular motion and therefore
have centripetal acceleration. The force for this acceleration, like for any other
situation in an ideal fluid, can only come from the pressure variation along the line
joining 1 and 2. This requires that pressure at 1 should be greater than the pressure
at 2 so that the fluid particles can have required acceleration. If there is no turbu-
lence, the motion can be taken as irrotational. Then by considering

v # dl = 0
C
2
1
along the circuit shown, we infer that v2 7 v1 .
The portion of the circuit near 1 and 2 are streamlines while the other two arms are
at right angle to streamlines.
(By electrostatic analogy, the density of streamlines is proportional to the velocity.)

1.316 From the conservation of mass


v1S1 = v2S2 (1)

But S1 6 S2 as shown in the figure of the problem book, therefore


v1 7 v2

As every streamline is horizontal between 1 and 2, Bernoulli’s theorem becomes

1 2
p + rv = constant
2
which gives p1 6 p2 as v1 7 v2

As the difference in height of the water column is ¢h, therefore

p2 - p1 = rg¢h (2)

From Bernoulli’s theorem between points 1 and 2 of a streamline

1 2 1
p1 + rv 1 = p2 + rv 22
2 2
1
or p2 - p1 = r (v 21 - v 22)
2
188 PART ONE PHYSICAL FUNDAMENTALS OF MECHANICS

1
or rg¢h = r (v 21 - v 22 ) (using Eq. 2) (3)
2
Using Eqs. (1) in (3), we get
2g¢h
v1 = S2
A S 22 - S 21
Hence the sought volume of water flowing per second

2g¢h
Q = v1S1 = S1S2
A S 22 - S 21
1.317 Applying Bernoulli’s theorem for points A and B,
1 2
pA = pB + rv as vA = 0
2
1 2
or rv = pA - pB = ¢hr0 g A
2
B
2¢hr0g
So, v = h
C r
Thus, rate of flow of gas,
2¢hr0g
Q = Sv = S
C r
The gas flows over the tube past it at B. But at A the gas becomes stationary as the
gas will move into the tube which already contains gas.

In applying Bernoulli’s theorem we should remember that


p 1
+ v 2 + gz
r 2
is constant along a streamline. In the present case, we are really applying Bernoulli’s
theorem somewhat indirectly. The streamline at A is not the streamline at B.
Nevertheless the result is correct. To be convinced of this, we need only apply
Bernoulli’s theorem to the streamline that goes through A by comparing the situa-
tion at A with that above B on the same level. In steady conditions, this agrees with
the result derived because there cannot be a transverse pressure differential.

1.318 Since, the density of water is greater than that of kerosene oil,
Kerosene oil h2
it will collect at the bottom. Now, pressure due to water level
equals h1r1g and pressure due to kerosene oil level equals
h2r2g. So, net pressure becomes h1r1g + h2r2g. Water h1
From Bernoulli’s theorem, this pressure energy will be con- A
verted into kinetic energy while flowing through the hole A.
1.7 HYDRODYNAMICS 189

1
i.e., h1r1g + h2r2g = r v2
2 1
r2
Hence, v = 2 a h1 + h2 b g = 3 m/s
C r1

1.319 Let H be the total height of the water column and the hole is made at a height h
from the bottom.
Then from Bernoulli’s theorem
1 2 v
rv = (H - h)rg
2
H h
or v = 1(H - h) 2g
which is directed horizontally.
l
For the horizontal range, l = vt

= 22g (H - h) # 2h
A g
= 2 2(Hh - h2)

Now, for maximum l


d (Hh - h2)
= 0
dh
H
which yields h = = 25 cm
2

1.320 Let the velocity of the water jet near the orifice be v ¿. Then applying Bernoulli’s
theorem,
1 2 1
rv = h0 rg + rv 2
2 2
or v¿ = 2v 2 - 2gh0 (1)
Here the pressure term on both sides is the same and is equal to the atmospheric
pressure.
Now, if water rises up to a height h, then at this height, whole of its kinetic energy
will be converted into potential energy. So,

1 v¿ 2
rv¿ 2 = rgh or h =
2 2g
v2
= - h0 = 20 cm (using Eq. 1)
2g
190 PART ONE PHYSICAL FUNDAMENTALS OF MECHANICS

1.321 Water flows through the small clearance into the orifice. Let d be the clearance, then
from the equation of continuity
(2pR1d)v1 = (2prd) v = (2pR2d) v2
or v1R1 = vr = v2R2 (1)
where v1,v2 and v are respectively the inward radial velocities of the fluid at 1, 2 and
3. Now by Bernoulli’s theorem just before 2 and just after it in the clearance
1 2
p0 + hrg = p2 + rv (2)
2 2
Applying the same theorem at 3 and 1, we find that this also equals
1 2 1
p + rv = p0 + rv 21 (3)
2 2
(since the pressure in the orifice is p0).
From Eqs. (2) and (3) we also have
v1 = 12gh (4)

rv a1 - a b b
1 2 v 2
and p = p0 +
2 1 v1
r
R2
R1
R1
= p0 + hrg a1 - a b b
2
ho
r
(using Eqs. 1 and 4) 23 1

1.322 For diagram of piston confirm that


ƒ = (p - p0 ) S (1)
as f is constant and p0 is constant, p is also constant.
From Bernouli’s equation from just inside and outside points of the orifice we get
1 2
p = p0 + rv
2 efflux V
S
1 B
or (p - p0) = rv 2efflux (2) A
2
If l is the length of liquid column at an arbitrary instant l
of time. Then from mass conservation

a- b S = vefflux
dl
(3)
dt
(where S is the piston area).
1.7 HYDRODYNAMICS 191

From Eq. (2) for constant p, we have vefflux = constant also.


So, V = vefflux St

(where V and + ve have meaning in accordance with the piston).

V
So, vefflux = (4)
St

Now the differential work done by the constant for F = (p - p0) S is given by

- dl
dA = F a - b dt = (p - p0) S a b dt
dl
dt dt

As both p and (- dl /dt) are constants, so, total work

dA = (p - p0) S a - b t
dl
W =
3 dt
veff s
= a rv efflux b S a b t
1 2
(using Eqs. 2 and 3)
2 S

1 3
= rv St
2 efflux

r a b St
1 V 3
= (using Eq. 4)
2 St

1 V3
= r
2 S2t 2

1.323 Water jet coming out from the orifice of area s at an arbitrary instant of time when
the height of water column in the cylindrical vessel is H.
vefflux = 22gH
As the rate of out-flux should be equal to the rate with which volume of the water
in the vessel decreases

dH
-S = s 22gH
dt
0
S dh S 2h
Hence, t= - =
s 12h 3 2H sA g
h
192 PART ONE PHYSICAL FUNDAMENTALS OF MECHANICS

1.324 In a rotating frame (with constant angular velocity), the Eulerian equation is

d v¿
- §p + rg + 2r(v¿ * ␻) + rv2 r = r
dt

In the frame of rotating tube the liquid in the “column” is practically static because
the orifice is sufficiently small. Thus the Eulerian equation in projection form along
r (which is the position vector of an arbitrary liquid element of length dr relative to
the rotation axis) reduces to
- dp O
+ rv2r = 0
dr
h B
A
or dp = rv2rdr
r dr
p r l
So, dp = rv2 rdr O
3 3
p0 (l - h)

rv2 2
Thus, p (r) = p0 + [r - ( l - h) 2] (1)
2

Hence the pressure at the end B just before the orifice is


rv2
p (l ) = p0 + (2lh - h 2) (2)
2
Then applying Bernoulli’s theorem at the orifice for the points just inside and out-
side of the end B
1 2 1
p0 + rv (2lh - h2) = p0 + rv 2 (where v is the sought velocity)
2 2

2l
So, v = vh - 1
Ah

1.325 The Euler’s equation is

dv
r = f - ¥p = - ¥(p + rgz) (where z is vertically upwards)
dt
0v
+ (v # ¥) v
dv
Now, = (1)
dt 0t

(v # ¥) v = ¥a v b - v * Curl v
1 2
But (2)
2
1.7 HYDRODYNAMICS 193

If we consider the steady (i.e. 0 v>0t = 0) flow of an incompressible fluid then


r = constant and as the motion in irrotational Curl v = 0
So from Eqs. (1) and (2)

r§a v2 b = - § (p + rgz)
1
2

rv 2 + rgz b = 0
1
§ap +
2

1 2
or p + rv + rgz = constant
2

1.326 Let the velocity of water flowing through A be vA and that through B be vB, then dis-
charging rate through A = QA = SvA and similarly through B = SvB.
Now, force of reaction at A is
h
A
FA = rQAvA = rSv 2B
S
Hence, the net force is h

F = rS 1v 2A - v 2B2 as FA c T FB
S
(1)
B
Applying Bernoulli’s theorem to the liquid flowing out of A we get

1 2
r0 + rgh = r0 + rv
2 A

And similarly at B
1 2
r0 + rg (h + ¢h) = r0 + rv
2 B

1v 2B - v 2A2
r
Hence, = ¢hrg
2

Thus, F = 2rgS ¢h = 0.50 N

1.327 Consider an element of height dy at a distance y from the top. The velocity of the
fluid coming out of the element is

v = 22gy

The force of reaction dF due to this is dF = rdAv 2 = r(bdy) 2gy, as in the previous
problem.
194 PART ONE PHYSICAL FUNDAMENTALS OF MECHANICS

Integrating F = r gb 2ydy
3
h-1

= rgb [h2 - (h - l)2] = r gbl (2h - l )


= 5 N (on substituting values)
(The slit runs from a depth h - l to a depth h from the top.)

1.328 Let the velocity of water flowing through the tube at a certain instant of time be u,
then u = Q >pr 2, where Q is the rate of flow of water and pr 2 is the cross section
area of the tube.
From impulse momentum theorem, for the stream of
water striking the tube corner, in x-direction in the time
interval dt, l x
Fxdt = - rQ udt or Fx = - rQu O y
Similarly, Fy = rQu
Therefore, the force exerted on the water stream by the
tube,
F = - rQ u i + rQu j
According to third law, the reaction force on the tube’s wall by the stream equals ( - F)
rQu i - rQu j
Hence, the sought moment of force about O becomes
rQ 2
N = - li * (rQui - Qu j) = rQul k = lk
pr2
rQ 2l
and |N| = = 0.70 Nm
pr 2

1.329 Let us take an arbitrary cross section of radius r of the narrowing tube. If p is the in-
side pressure at the location of taken ring element of radius r and width dl, and p0 is
outside atmosphere pressure, from symmetry of the problem, net force exerted on the
taken ring element due to the inside and outside pressure difference.
= dF sin u = (p - p0) 2prdl sin u
= (p - p0) 2p rdr (because dl sin u = dr)
Hence the sought net force on the tube

F = Fx = dF sin u = (p - p0) 2prdr (1)


L L
1.7 HYDRODYNAMICS 195

From Bernoulli’s equation dF

1 r
p - p0 = r (v 2efflux - v 2),
2 s
where vefflux = 22gh (2) S dF
and v is the speed of water at the location of taken ring element.
From conservation of mass
rvpr2 = rvefflux pr1 2
r1 2
or v = vefflux (3)
r2
(where r1 is the radius at open end).
Using Eq. (3) in Eq. (2), we get
r1 4
r a v 2efflux - v 2efflux b
1
p - p0 =
2 r4
r1 2 r1 2
rv efflux a 1 - b = rgh a 1 - b
1 2
= (4)
2 r4 r4
Using Eq. (4) in Eq. (1), we get
r2
r1 2
F = rgh a 1 - b 2prdr (5)
3 r4
r1

After integration and using pr2 2 = S and pr1 2 = s, we get


rgh (S - s)2
F = = 6N (on substituting the values)
S
Note: If we try to calculate F from the momentum change of the liquid flowing out we will
be wrong even as regards the sign of the force. There is of course the effect of pressure
at S and s but quantitative derivation of F from Newton’s law is difficult.

1.330 The Euler’s equation is


dv
= f - §p
r
dt
in the space fixed frame where f = - rgk downward. We assume incompressible
fluid so r is constant.
Then f = - § (rgz) where z is the height vertically upwards from some fixed origin.
We go to the rotating frame where the equation becomes
dv
r = - § (p + rgz) + rv2r + 2r (v * ␻)
dt
196 PART ONE PHYSICAL FUNDAMENTALS OF MECHANICS

The additional terms on the right are the well known coriolis and centrifugal forces.
In the frame rotating with the liquid v = 0, so

rv 2r 2 b = 0
1
§a p + rgz -
2
1
or p + rgz - rv2r 2 = constant
2
On the free surface p = constant; thus
v2 2
z = r + constant
2g
If we choose the origin at point r = 0 (i.e., the axis) of the free surface then
“constant” = 0 and
v2 2
z = r (the parabolic of revolution)
2g
At the bottom z = constant.
1
So, p = rv2r 2 + constant
2
If p = p0 on the axis at the bottom, then
1
p = p0 + rv2r 2
2
1.331 When the disk rotates the fluid in contact with it co-rotates, but the fluid in contact
with the walls of the cavity does not rotate. A velocity gradient is then set up lead-
ing to viscous forces.
At a distance r from the axis, the linear velocity is vr so there is a velocity gradient
vr>h both in the upper and lower clearance. The corresponding force on the element
whose radial width is dr is

a from the formula F = h A b


vr dv
h2prdr
h dx
The torque due to this force is
vr
h2prdr r
h
and the net torque considering both the upper and lower clearance is
R
v v
2 h 2pr 3 dr = pR 4h
3 h h
0
1.7 HYDRODYNAMICS 197

So power developed is
pR 4v2h
P = = 9.05 W (on substituting values)
h
(As instructed, end effects, i.e., rotation of fluid in the clearance r Ú R , has been
neglected.)
1.332 Let us consider a coaxial cylinder of radius r and thickness dr, then force of friction
or viscous force on this elemental layer is given by
dv
F = 2prlh
dr
This force must be constant from layer so that steady motion may be possible.
v
Fdr dr
So, = 2pl h dv (1)
r 3 R2 r R1
0
r v
dr
Integrating, F = 2pl h dv
3 r 3
R2 0

b = 2pl hv
r
or F lna (2)
R2
Putting r = R1, we get
R1
F ln = 2pl hv0 (3)
R2

Dividing Eqs. (2) by (3), we get


ln r /R2
v = v0
ln R1/R2
Note: The force F is supplied by the agency which tries to carry the inner cylinder
with velocity v0.

1.333 (a) Let us consider an elemental cylinder of radius r and thickness dr then from
Newton’s formula
dv dv
F = 2prl hr = 2pl hr 2 (1)
dr dr 2

and moment of this force acting on the element,


R1 dr
dv dv
N = 2pr 2l h r = 2pr 3l h r
dr dr
R2
dr
or 2pl hdv = N 3 (2)
r
198 PART ONE PHYSICAL FUNDAMENTALS OF MECHANICS

As in the previous problem N is constant when conditions are steady


v r
dr
Integrating, 2pl h dv = N 3
3 3 r
0 R1

c - 2d
N 1 1
or 2pl hv = (3)
2 R 21 r
Putting r = R2, v = v2, we get

c - 2d
N 1 1
2pl hv2 = (4)
2 R 21 R2
From Eqs. (3) and (4)
R 21 R 22
c - 2d
1 1
v = v2
R 22 - R 21 R 21 r
(b) From Eq. (4)
N R 21 R 22
N1 = = 4 phv2 2
l R 2 - R 21

1.334 (a) Let dV be the volume flowing per second through the cylindrical shell of thick-
ness dr then,

2
v
r r
R

dr

dV = - (2prdr) v0 a1 - b a b dr
r2 r3
dr = 2pv0 r -
R2 R2
The total volume is
R

ar - b dr = 2pv0
r3 R2 p 2
V = 2pv0 2
= R v0
3 R 4 2
0

(b) Let dE be the kinetic energy within the above cylindrical shell. Then,
1 1
dT = (dm) v 2 = (2prldrr) v 2
2 2

(2plr)r dr v 20 a1 - 2 b = plrv 20 c r - 2 + d dr
1 r2 2 2r 3 r5
=
2 R R R4
1.7 HYDRODYNAMICS 199

Hence, total energy of the fluid,


R
pR 2rl v 20
ar - b
2r 3 r5
T = plrv 20 + dr =
3 R2 R4 6
0

(c) Here frictional force is the shearing force on the tube, exerted by the fluid, which
equals - hS dv>dt.

v = v0 a 1 - b
r2
Given,
R2
dv r
So, = - 2v0 2
dr R
dv 2v0
At r = R, = -
dr R

Then, viscous force is given by F = - h (2pRl ) a b


dv
dr r = R
2v0
= - 2pR hl a - b = 4phv0l
R
(d) Taking a cylindrical shell of thickness dr and radius r, viscous force,
dv
F = - h(2prl )
dr
Let ¢p be the pressure difference, then net force on the element is
dv
¢ppr 2 + 2phlr
dr
But, since the flow is steady, Fnet = 0

- 2plhr dv>dr - 2plhr 1 - 2v0 r>R 22 4hv0l


or ¢p = = =
pr 2 pr 2 R2

1.335 The loss of pressure head in travelling a distance l is seen from the middle section to
be h2 - h1 = 10 cm. Since h2 - h1 = h1 in our problem and h3 - h2 = 15 cm =
5 + h2 - h1, we see that pressure head of 5 cm remains in compensated and must
be converted into kinetic energy of the liquid flowing out. Thus,
rv 2
= rg¢h (where ¢h = 3 h - 2 h )
2
Thus, v = 1 2g¢h ⬵ 1 m/s
200 PART ONE PHYSICAL FUNDAMENTALS OF MECHANICS

1.336 We know that, Reynold’s number (Re) is defined as, Re = rvl/h, where v is the ve-
locity, l is the characteristic length, and h the coefficient of viscosity. In the case of
circular cross-section the characteristic length is the diameter of cross-section d, and
v is taken as average velocity of flow of liquid.
rd1v1
Now, Re (Reynold’s number at x1 from the pipe end) = , where v1 is the
1 mh
velocity at distance x1.
rd2v2
Similarly, Re =
2 h
Re d1v1
=
1
So,
Re d2v2
2

From equation of continuity,


A1v1 = A2v2
or pr 21v1 = pr 22 v2 or d1v1r1 = d2v2r2
d1v1 r2 r0e-ax2
= = = e-a¢x (as x2 - x1 = ¢x)
d2v2 r1 r0e-ax1
Re
= ea¢x = 5
2
Thus,
Re
1

1.337 We know that Reynold’s number for turbulent flow is greater than that for laminar
flow.
rvd 2r1v1r1 2r2v2r2
Now, (Re )l = = and (Re)t =
h h1 h
But, (Re )t Ú (Re )l
r1v1r1 h2
So v2 = = 5 mm/s (on substituting values)
min r2r2h1

vr0d
1.338 We have R =
h
4p 2
and v is given by 6phrv = r (r - r0) g
3

(where r = density of lead, r 0= density of glycerine)


2 1
v = (r - r0)gr 2 = (r - r0) gd 2
9h 18h
1.7 HYDRODYNAMICS 201

1 1
Thus, = (r - r0) gr0d 3
2 18h2
and d = [9h2/r0(r - r0)g]1/3 = 5.2 mm (on substituting values)

dv
1.339 m = mg - 6phrv
dt
dv 6phr
or + v = g
dt m

+ kv = g awhere k =
dv 6phr a
or
dt m

dv d kt
or e kt + ke ktv = ge kt or e v = ge kt
dt dt
g g
or ve kt = e kt + C or v = + Ce-kt (where C is constant)
k k
g
Since v = 0 for t = 0, so 0 = + C
k
g
or, C = -
k
g
Thus, v = (1 - e-kt)
k

The steady state velocity is g /k .


v differs from g /k by n, where

e-kt = n
1
or t = ln n
k
1 (4p>3 )r 3P 4r 2r d 2r
Thus, = - = - = -
k 6phr 18h 18h

(We have neglected buoyancy in olive oil.)

- rd 2
t = lnn
18 h

= 0.20 s (on substituting values)


202 PART ONE PHYSICAL FUNDAMENTALS OF MECHANICS

1.8 Relativistic Mechanics

1.340 From the formula for length contraction

a l0 - l0 b = hl0
v2
1 -
A c2

v2
So, 1 - = (1 - h)2 or v = c 2h(2 - h)
c2

1.341 (a) In the frame in which the triangle is at rest, the space coordinates of the vertices
23 a 23 a
are (000), aa , + , 0b a a , - , 0 b , all measured at the same time t. In
2 2 2 2
the moving frame the corresponding coordinates at time t ¿ are

A : (vt¿, 0, 0), B : a 23 21 - b 2 + vt¿, , 0 b and


a a
2 2

C : a 3311 - b 2 + vt¿, - , 0 b
a a
2 2
The perimeter P is then

P = a + 2a a (1 - b 2) + b = a C 1 + 24 - 3b 2 D
3 1 1>2
4 4
(b) The coordinates in the first frame are shown at time t. The coordinates in the
moving frame are

B a 3
‚a ‚ 0
B 2 2
A

C A
(0,0,0) C (a,0,0)

13
A : (vt¿, 0, 0), B : a 11 - b 2 + vt¿, a , 0 b , C : (a11 - b 2 + vt¿, 0, 0)
a
2 2
The perimeter P is then
a
P = a11 - b 2 + [1 - b 2 + 3]1>2 * 2
2

= a111 - b 2 + 14 - b 2 2 a where b = b
v
c
1.8 RELATIVISTIC MECHANICS 203

1.342 In the rest frame, the coordinates of the ends of the rod in terms of proper length l0
A : (0, 0, 0), B : (l0 cos u0, l0 sin u0, 0)
at time t. In the laboratory frame the coordinates at time t¿ are

A : (vt¿, 0, 0), B : (l0 cos u0 11 - b 2 + vt¿, l0sin u0, 0)

Therefore, we can write


l cos u0 = l0 cos u0 11 - b 2 and l sin u = l0 sin u0 B

cos2 u + (1 - b 2) sin2 u
Hence, l 20 = (l 2) a b A
1 - b2
1 - b 2 sin2 u
a where b = b
v
or l0 =
A 1 - b2 c
= 1.08 m (on substituting values)

1.343 In the frame K in which the cone is at rest, the coordinates of A are (0, 0, 0) and of
B are (h, h tan u, 0). In the frame K¿, which is moving with velocity v along the axis
of the cone, the coordinates of A and B at time t¿ are

A : (- vt¿, 0, 0), B : (h - 11 - b 2 - vt¿, h tan u, 0)


Thus the taper angle in the frame K¿ is
y¿B - y ¿A
= a b or u¿ = 59° (on substituting values)
tan u
tan u¿ =
11 - b 2 x ¿B - xA
The lateral surface area is
S = ph¿ 2 sec u¿ tan u¿
tan u tan2 u
= ph2 (1 - b 2) 1 +
11 - b 2 A 1 - b2
= S0 11 - b 2cos2 u
= 3.3 m2 (on substituting values)
Here, S0 = ph2 sec u tan u is the lateral surface area in the rest frame and

h¿ = h 11 - b 2 awhere b =
b
v
c
1.344 Because of time dilation, a moving clock reads less time. We write,

t - ¢t = t 11 - b 2 awhere b = b
v
c
2 ¢t ¢t
+ a b = 1 - b2
2
Thus, 1 -
t t
204 PART ONE PHYSICAL FUNDAMENTALS OF MECHANICS

¢t ¢t
or v = c a2 - b
A t t
= 0.6 * 108 m/s (on substituting values)

1.345 In the frame K, the length l of the rod is related to the time of flight ¢t by
l = v¢t
In the reference frame fixed to the rod (frame k¿ ) the proper length l0 of the rod is
given by

l0 = v¢t¿
v ¢t
awhere b = b
l v
But, l0 = =
11 - b 2 11 - b 2 c
v ¢t
Thus, v ¢t¿ =
11 - b 2
¢t 2 ¢t 2
So, 1 - b 2 = a b or v = c 1 - a b
¢t¿ A ¢t¿

¢t
and l0 = c 1(¢t¿)2 - (¢t)2 = c ¢t¿ 1 - a b = 4.5 m (on substituting values)
A ¢t¿
1.346 The distance traveled in the laboratory frame of reference is v¢t where v is the
velocity of the particle. But by time dilation
¢t0
¢t =
11 - v 2>c 2
So, v = c 11 - (¢t0> ¢t)2

Thus the distance traversed is


s = c¢t 11 - (¢t0> ¢t)2
= 5 m (on substituting values)

1.347 (a) If t0 is the proper life time of the mount, the life time in the moving frame is
t0 vt0
=
11 - v 2>c 2 11 - v 2>c 2
and hence l

11 - v2>c2
l
Thus, t0 =
v
= 1.4 ms (on substituting values)
(b) The words “from the muon’s stand point” are not part of any standard
terminology.
1.8 RELATIVISTIC MECHANICS 205

1.348 In the frame K in which the particles are at rest, their positions are A and B whose
coordinates may be taken as,
K
A : (0, 0, 0), B = (l0, 0, 0)

In the frame K¿ with respect to which K is moving with a A B


velocity v, the coordinates of A and B at time t¿ in
the moving frame are

A = (vt¿, 0, 0), B = (l0 11 - b 2 + vt¿, 0, 0) a where b = b


v
c

Suppose B hits a stationary target in K¿ after time t¿B while A hits it after time tB + ¢t.
Then,

l0 11 - b 2 + vt¿B = v (t¿B + ¢t)

v¢t
l0 =
11 - v 2>c2
So,

= 17 m (on substituting values)

1.349 In the reference frame fixed to the ruler the rod is moving with a velocity v and
suffers Lorentz contraction. If l0 is the proper length of the rod, its measured
length will be

¢x1 = l0 11 - b 2 awhere b = b
v
c

In the reference frame fixed to the rod, the ruler suffers Lorentz contraction and we
must have

¢x2 11 - b 2 = l0

Thus, l0 = 1¢x1 ¢x2 = 6.0 m (on substituting values)

¢x1
and 1 - b2 =
¢x2

¢x1
or v = c 1 - = 2.2 * 108 m/s (on substituting values)
A ¢x2
206 PART ONE PHYSICAL FUNDAMENTALS OF MECHANICS

1.350 The coordinates of the ends of the rods in the frame fixed to the left rod are shown
in the figure. The points B and D coincide when

c1 - l0
l0 = c1 - vt0 or t0 =
v
A B D E
The points A and E coincide when (0,0,0) (l0,0,0) (c1–vt,0,0)
(c1+l0 1−β2−vt,0,0)
0 = c1 + l0 11 - b 2 - vt1

c1 + lo 11 - b 2
t1 =
v

l0
Thus, ¢t = t1 - t0 = 11 + 11 - b 22
v

a - 1 b = 1 - b2 = 1 - 2
v¢t 2 v2
or
l0 c

2c2 ¢t>l0 2l0> ¢t


v = =
c 2 ¢t 2>l02 1 + (l0>c¢t)2
From this
1 +

1.351 In K0, the rest frame of the particles, the events corresponding to the decay of the
particles are

A : (0, 0, 0, 0) and B : (0, l0, 0, 0)

In the reference frame K, the corresponding coordinates are by Lorentz transforma-


tion

vl0 l0
A : (0, 0, 0, 0), B : a , , 0, 0 b
c2 11 - b 2 11 - b 2

Now, l0 11 - b 2 = l , by Lorentz Fitzgerald contraction formula.


Thus the time lag of the decay time of B is

vl0 vl vl
¢t = = = 2
c 2 11 - b2 c 2(1 - b)
2 c - v2

= 20 ms (on substituting values)

B decays later. (B is the forward particle in the direction of motion.)


1.8 RELATIVISTIC MECHANICS 207

1.352 (a) In the reference frame K with respect to which the rod is moving with velocity
v, the coordinates of A and B are

A : t, xA + v 1t - tA2, 0, 0
B : t, xB + v 1t - tB2, 0, 0

l = xA - xB - v 1tA - tB2 = l0 11 - 2
B A
Thus,

xA - xB - v 1tA - tB2
So, l0 =
21 - v 2/c 2

(b) l0 - v 1tA - tB2 = l = l0 21 - v 2/c 2

1Since xA - tB can be either + l0 or - l0 .2

Thus, v 1tA - tB2 = 11 - 21 - v 2/c 2)l0

A 1 - 21 - v 2/c 2 B
l0
i.e., tA - tB =
v
l0
or tB - tA = v 11 + 11 - v 2/c22

1.353 At the instant the picture is taken the coordinates of A, B, A¿, B¿ in the rest frame
of AB are A' B'
A : 10, 0, 0, 02 v

B : 10, l0, 0, 02
B¿: (0, 0, 0, 0)
A¿: (0, or - l0 21 - v 2>c 2, 0, 0) A B

(a) In this frame the coordinates of B¿ at other times are B¿ : (t, vt, 0, 0). So B¿ is
opposite to B at time t (B)  l0 /v. In the frame in which B¿, A¿ is at rest, the time
corresponding to this is given by Lorentz transformation.

l0 vl0 l0
a b =
1
t0 (B¿) = - 2
21 - v 2/c2
21 - v 2/c 2 v c v

(b) Similarly in the rest frame of A , B, the coordinates of A at other times are

A¿ : a t, - l0 21 - v 2/c 2 + vt, 0, 0 b
208 PART ONE PHYSICAL FUNDAMENTALS OF MECHANICS

l0
A¿ is opposite to A at time t (A) = 21 - v 2/c 2
v
The corresponding time in the frame in which A¿, B¿ are rest is
l0
t (A¿) = gt (A) =
v

at - b
1 vx
1.354 By Lorentz transformation t¿ =
21 - v 2/c 2 c2

vx 1
So at time t = 0, t¿ =
c2 21 - v2>c2

If x 7 0,t¿ 6 0 and if x 6 0,t¿ 7 0, so we get the diagram given below “in terms of
the K-clocks”.

K:

K:

The situation in terms of the K ¿ clock is reversed.

1.355 Suppose x (t) is the locus of points in the frame K at which the readings of the clocks
of both reference system are permanently identical, then by Lorentz transformation

2> 2 a t b = t
1 Vx (t)
t¿ = -
21 - V c c2

a1 - 21 - V 2>c 2 b
c2
On differentiating x (t) =
V

11 - 21 - b 22 awhere b = b
c V
=
b c

Let b = tan h u, 0 … u 6 q . Then,

11 - 21 - tanh2u 2 = c a1 - b
c cosh u 1
x (t) =
tanh u sinh u cosh u
cosh u - 1 cosh u - 1 u
= c = c = c tanh … V
sinh u A cosh u + 1 2

( tan h u is a monotonically increasing function of u.)


1.8 RELATIVISTIC MECHANICS 209

1.356 We can take the coordinates of the two events to be

A : (0, 0, 0, 0); B : (¢t, a, 0, 0)


|a|
For B to be the effect and A to be cause we must have ¢t 7 .
c
In the moving frame, the coordinates of A and B become

A : (0, 0, 0, 0); B: c g a ¢t - b , g (a - V ¢t), 0, 0 d ¢where g =


aV 1

21 - 1V 2>c22
c2

Since,

2 c a ¢t - b 2 d = (¢t)2 -
a¿ 2 aV 2 1 a2
(¢t¿)2 - = g - (a - V ¢t) 7 0
c2 c2 c2 c2
|a| ¿
we must have ¢t¿ 7 .
c

1.357 (a) The four-dimensional interval between A and B (assuming ¢y = ¢z = 0) is:


52 - 32 = 16 units ct
7
Therefore the time interval between these two B
6
events in the reference frame in which the 5
events occurred at the same place is 4
C
3
c (t¿B - t¿A ) = 216 = 4 m
2
4 4 1
or t¿B - t¿A = = * 10-8 s A
x
c 3 0 1 2 3 4 5 6 7

(b) The four dimensional interval between A and C (assuming ¢y = ¢z = 0) is:

32 - 52 = - 16
So the distance between the two events in the frame in which they are simulta-
neous is 4 units = 4 m.

1.358 By the velocity addition formula


vx - V
vx¿ =
1 - Vvx >c 2

vy 21 - V 2>c 2
v y¿ =
1 - vxV>c 2
210 PART ONE PHYSICAL FUNDAMENTALS OF MECHANICS

2(vx - V )2 + v y2 (1 - V 2>c 2)
and v¿ = 2v x¿ 2 + v ¿y2 = .
1 - vxV>c 2

1.359 (a) By definition, the velocity of approach is


dx1 dx2
vapproach = - = v1 - ( - v2) = v1 + v2 = 1.25c
dt dt

in the reference frame K .


(b) The relative velocity is obtained by the transformation law

v1 - (–v2) v1 + v2
vr = =
v1 ( - v2) v1v2
1 - 1 + 2
c2 c
= 0.19c (where c is the velocity of light )

1.360 The velocity of one of the rods in the reference frame fixed to the other rod is
v + v 2v
V = =
1 + v 2>c 2 1 + b2
The length of the moving rod in this frame is

4v 2>c 2 1 - b2
l = l0 1 - = l0
C (1 + b 2)2 1 + b2

1.361 The approach velocity is defined by

d r1 d r2
Vapproach = - = v1 - v2
dt dt

in the laboratory frame.

So, Vapproach = 2v12 + v22

On the other hand, the relative velocity can be obtained by using the velocity addition
formula and has the components

v12 v1v2
B - v1, v2 1 - a bR so , Vr = v12 + v22 -
C c2 A c2
1.8 RELATIVISTIC MECHANICS 211

1.362 The components of the velocity of the unstable particle in the frame K are
y
a V, v¿
V2 y'
1 - , 0b
A c2
So the velocity relative to K is v'

v¿ 2V 2
V 2 + v¿ 2 -
A c2
The life time in this frame dilates to

¢t0 n
V 2 v¿ 2 v¿ 2V 2
1 - - +
A c2 c2 c4
and the distance traversed is
2V 2 + v¿ 2 - (v¿ 2V 2)>c 2
s = ¢t0
21 - V 2>c 2 21 - v¿ 2>c 2

1.363 In the frame K ¿ the components of the velocity of the particle are
v cos u - V K'
v¿x = K
vV cos u
1 - v
c2
v
v sin u 21 - V 2>c 2
v¿y = θ x
vVcos u
1 -
c2
v y¿
21 - V 2>c 2
v sin u
Hence, tan u¿ = =
v¿x v cos u - V

1.364 In K ¿ the coordinates of A and B are


A : (t¿, 0, - v¿ t¿, 0); B : (t¿, l, - v¿ t¿, 0)
After performing Lorentz transformation to the frame K we get

B : t = g a t¿ + b
Vl
A : t = gt ¿;
c2
x = g Vt ¿; x = g (l + Vt ¿)
y = v ¿t ¿; y = - v ¿t¿
z = 0; z = 0
Vl
By translating t ¿ : t ¿ - , we can write the coordinates of B as B : t = gt ¿.
c2
v ¿2
x = gl a 1 - b + Vt¿g = l 1 - 2 + Vt ¿g
V2
c 2 A c
212 PART ONE PHYSICAL FUNDAMENTALS OF MECHANICS

y = - v¿ a t¿ - b, z = 0
Vl K K'
c2
v

1 - a b, ¢y = 2
V2 v¿Vl A
Thus, ¢x = l v' B
C c 2 c
v¿V
Hence, tan u¿ =
c 2 21 - 1V>c22

t t + dt
1.365 - - K
v v + wdt

(a) In K the velocities at time t and t + dt are respectively v and v + wdt along x-
axis which is parallel to the vector V. In the frame K¿ moving with velocity V
with respect to K, the velocities are, respectively,
v - V v + wdt - V
and
1 - vV>c 2 1 - (v + wdt) V>c 2
The latter velocity is written as

v - V wdt v - V wV
+ + dt
1 - vV>c2 1 - vV>c2 (1 - vV>c ) c 2
2

v - V wdt 11 - V 2>c 22
= +
1 - vV>c 2 11 - vV>c 222

Also by Lorentz transformation

dt - Vdx>c2 1 - vV/c 2
dt¿ = = dt
21 - V 2>c 2 21 - V 2>c 2

Thus the acceleration in the K ¿ frame is

a1 b
dv¿ w V 2 3>2
w¿ = = -
dt¿ 11 - vV>c223 c2

(b) In the K frame, the velocities of the particle at the time t and t + dt are, respec-
tively (0, v, 0) and (0, v + wdt, 0), where V is along x-axis.
1.8 RELATIVISTIC MECHANICS 213

In the K¿ frame the velocities are 1 - V, v 21 - V 2>c 2, 02

and 1- V, (v + wdt) 21 - V 2>c 2, 02, respectively.

Thus the acceleration

wdt 2(1 - V 2>c 2)


= w a1 - b
V2
w¿ = along the y-axis
dt¿ c2

dt
aWe have used dt¿ = .
21 - V 2>c 2
a

1.366 In the instantaneous rest frame v = V and


w
w¿ =
11 - V 2>c 223>2
(from solution problem 1.365(a))

dv
= = w¿dt
11 - V 2>c223>2
So,

w¿ is constant by assumption. Thus, integration gives


w¿t
v =
21 + 1w ¿t>c22

¢ 1 + a b - 1≤
c2 w¿t 2
Integrating once again x =
w¿ B c
= 0.91 light year
The percentage difference of rocket velocity from velocity of light is given by
c - v
= a b = 0.47%
1 c 2
c 2 w¿t

1.367 The boost time t0 in the reference frame fixed to the rocket is related to the time t
elapsed on the Earth by

1w¿t>c22
t t
v2
t0 = 1 - dt = D1 - T dt
3A c2 3 1 + 1w¿t>c2
0 0
214 PART ONE PHYSICAL FUNDAMENTALS OF MECHANICS

t w ¿t /c

ln c + 1 + a b d
dt c dj c w¿t w¿t 2
= = =
3 21 + 1w¿t>c2 w¿ 3 21 + j2 w¿ c B c
0 0

= 3.5 months (on substituting values)

m0
1.368 m =
21 - b 2
m 1 1
For b L 1, L =
m0 22 (1 - b) 22h
L 70 (on substituting values)

1.369 We define the density r in the frame K is such a way that r dx dy dz is the rest mass
dm0 of the element. That is r dxdydz = r0dx0dy0dz0, where r0 is the proper density,
dx0, dy0, dz0 are the dimensions of the element in the rest frame K0. Now,

dy = dy0, dz = dzo, dx = dx0 21 - v2>c2

if the frame K is moving with velocity v relative to the frame K0. Thus
r0
r =
21 - v 2>c 2
Defining h by r = r0 (1 + h)

1 v2 1 h(2 + h)
We get 1 + h = or = 1 - =
21 - v 2>c 2 (1 + h) 1(1 + h)2
c 2 2

h(2 + h) c 2h (2 + h)
or v = c =
A (1 + h)2 1 + h

= 0.6c (where c is the velocity of light)

m0v m0 p2
1.370 We have = p or = m0 2 +
21 - v 2>c 2 21 - v 2>c 2 A c2

v2 m20c 2 p2
or 1 - = = 1 -
c2 m20c 2 + p 2 p 2 + m20c 2
cp c
or v = =
2p 2 + mo2c 2 21 + (moC> p 2 2
1.8 RELATIVISTIC MECHANICS 215

c - v m0c 2 -1>2
So, = c1 - a1 + a b b d * 100 % = 0.44% (on substituting values)
c p

1.371 By definition of h,

m0v v2 1
= hm0v or 1- = 2
21 - v 2>c 2 c 2 h
1 c 1
or v = c 1 - = 2h2 - 1 = c 23 (on substituting for h)
A h2 h 2

1.372 The work done is equal to change in kinetic energy which is different in the two
cases. Classically, i.e., in non-relativistic mechanics, the change in kinetic energy is
1 1
m0c 2 ((0.8)2 - (0.6)2) = m0c2 0.28 = 0.14 m0c 2
2 2
Relativistically, it is
m0c 2 m0c 2 m0c 2 m0c 2
- = - = m0c 2 (1.666 - 1.250)
21 - (0.8)2 21 - (0.6)2 0.6 0.8

= 0.416 m0c 2 = 0.42 m0 c 2

m 0c 2
1.373 = 2m0c 2
v2
1 - 2
A c
v2 1 v2 1
or 1 - = or 1 - =
A c 2 2 c 2 4
v 23 23
or = , i.e., v = c = 2.6 * 108 m/s
c 2 2

1.374 Relativistically,

= a - 1 b = b 2 + b4
T 1 1 3
m0c 2 21 - b 2 2 8

- 1b rel
2 22 = - a b
2T 3 2T 3 2T 2
So, b 2rel ⬵ 2 2
m0c 4 m0c 4 m0c 2

- b rel = c d = a1 - b
2T T 2 1>2 2T 3 T
Thus, - 3
m0c 2 2
m0c 4 A m0c 2 4 m0c 2
216 PART ONE PHYSICAL FUNDAMENTALS OF MECHANICS

2T
But classically, b cl =
A m0c 2

b rel - b cl 3 T
so , = = e
b cl 4 m0c 2
T 4
Hence if, 2
6 e L 0.013, then the velocity b is given by the classical formula
m0c 3
with an error less than e.

1.375 From the formula


m 0c 2 m0v
E = , p =
21 - v 2/c 2 21 - v 2>c 2

we find E 2 = c 2p 2 + m02c 4 or (m0c 2 + T ) 2 = c 2p 2 + m02c 4


1
or T (2 m0c 2 + T) = c 2p 2 i.e. p = 2T (2m0c 2 + T ) = 1.09 GeV/c
c

1.376 Let the total force exerted by the beam on the target surface be F and the power lib-
erated there be P. Then, using the result of the previous problem, we see

N I
F = Np = 2T (T + 2m0c 2) = 2T (T + 2m0c 2)
c ec
since, I = Ne, N being the number of particles striking the target per second. Also,
m0c 2
P = N a - m0c2 b =
I
T
21 - v 2>c 2 e

These will be, respectively, equal to the pressure and power developed per unit area
of the target if I is current density.

1.377 In the frame fixed to the sphere, the momentum transferred to the elastically scat-
tered particle is
2mv
21 - v 2>c 2
1
The density of the moving element is n (from solution of problem 1.369)
21 - v 2>c2
1.8 RELATIVISTIC MECHANICS 217

and the momentum transferred per unit time per unit area is the pressure, given by
2mv 1 #v 2mnv2
p =
1 - v 2>c2
n
21 - v 2>c 2 21 - v 2>c2
In the frame fixed to the gas, when the sphere hits a stationary particle, the latter
recoils with a velocity
v + v 2v
= =
1 + v >c
2 2 1 + v 2>c2
m # 2v
1 + v 2>c 2 2mv
4v 2>c 2
=
1 - v 2>c2
The momentum transferred is
1 -
A (1 - v 2>c 2)2
2
2mv # n # v = 2mnv2 2
1 - v >c 1 - v >c
and the pressure is 2 2

1.378 The equation of motion is


m0v
a b = F
d
dt 21 - v2>c2

v>c b Ft
Integrating, = = (using v = 0 for t = 0)
21 - v 2>c 2 21 - b 2 m0 c

= a b or b 2 =
b2 Ft 2 (Ft)2 Fct
or v =
1 - b 2 m0c (Ft) + (m0c)2
2
2(m0c)2 + (Ft)2

Fct dt c jdj c
or x = = = 2F 2t 2 + m 20 c 2 + constant
3 2F 2t 2 + m20c 2 F 3 2j 2 + (m c) 2
0
F

m0c 2 m0c 2
c 2t 2 + a b -
2
or using x = 0 at t = 0, we get, x =
B F F

1.379 Since, x = 2a 2 + c 2t 2

# c 2t
so, x = v =
a 2 + c 2t 2

v c 2t
or =
21 - v 2>c2 a
218 PART ONE PHYSICAL FUNDAMENTALS OF MECHANICS

m0v m0c 2
a b =
d
Thus, = F
dt 21 - v 2>c2 a

m0v #
#
a b = m0 + m0 2 v # v
d v v 1
1.380 F =
dt 21 - v 2>c 2 21 - v >c
2 2 c 11 - v 2>c 223>2

#
Thus, F ⬜ = m0w 21 - b 2, w = v, w ⬜ v

w #
F7 = m0 , w = v, w7 v
(1 - b 2)3>2

1.381 By definition,

c2 m0c 3dt vx cm0dx


E = m0 = , px = m0 =
21 - vx2>c2 ds 21 - v 2>c2 dx

where ds 2 = c 2 dt 2 - dx 2 is the invariant interval (dy = dz = 0).

dx¿ (dx - Vdt) px - VE>c 2


Thus, px¿ = cm0 = cm0g =
ds ds 21 - V 2>c2

dt¿ 1dt - Vdx >c22 E - Vpx


E ¿ = m0c 3 - c 3 m0g =
ds ds V2
1 - 2
A c

1.382 For a photon moving in the x direction


e = cpx
and p = p = 0
y z

1 - V>c
ae - V b = e
1 e
In the moving frame, e¿ =
21 - b 2 c A 1 + V>c
e 1 1 - b 3 3c
Note that e¿ = if, = or b = , V =
2 4 1 + b 5 5

1.383 As before
dt
E = m0c 3
ds
dx
px = m0c
ds
1.8 RELATIVISTIC MECHANICS 219

dy dz
Similarly py = m0c , pz = m0c
ds ds
Then E - c p = E - c 1px + py + pz22
2 2 2 2 2 2 2

(c 2dt 2 - dx 2 - dy 2 - dz 2)
= m02c 4 = m02c 4 is invariant
ds 2

1.384 (b) In the C.M. frame, the total momentum is zero, Thus

V cp1x 2T (T + 2m0c 2) T
= = =
c E1 + E2 T + 2m0c 2 A T + 2m0c 2
T
V = c = 2.12 * 108 m/s
A T + 2m0c 2
where we have used the result of problem 1.375.
(a) Then
1 1 T + 2m0c 2
= =
21 - V 2>c 2 1 -
T C 2m0c 2
A T + 2m0c 2
Total energy in the C.M. frame is

2m0c 2 T + 2m0c 2 '


= 2m0 c2 = 22m0c 2 ( T + 2m0c 2) = T + 2m0c 2
21 - V 2>c 2 C 2m0c 2

T = 2m0c 2 a - 1 b = 777 MeV


' T
So, 1 +
A 2m0c 2
' '
Also 2 2c 2 p 2 + mc 4 = 22m20c 2(T + 2m0c 2) Q 4c 2p 2 = 2m0c 2T

' 1
or p = m T = 940 MeV>c
A2 0

1.385 M0c 2 = 2E 2 - c 2p 2

2(2m0c 2 + T ) 2 - T (2m0c 2 + T ) = 22m0c 2 (2m0c 2 + T ) = c 22m0 (2m0c 2 + T )

c 2p T
Also cp = 2T (T + 2m0c 2), v = = c
E A T + 2m0c 2
220 PART ONE PHYSICAL FUNDAMENTALS OF MECHANICS

1.386 Let T ¿ = kinetic energy of a proton striking another stationary particle of the same
rest mass. Then, combined kinetic energy in the C.M. frame

T¿ T¿
= 2m0c 2 a b a b
T 2
1 + - 1 = 2T, + 1 = 1 +
A 2m0c 2 m0c 2 2m0c 2

T¿ T (2m0c 2 + T) 2T (T + 2m0c 2)
= or T¿ =
2m0c 2 m20c4 m0c 2

1.387 We have
E1 + E2 + E3 = m0c 2, p1 + p2 + p3 = 0

Hence, (m0c 2 - E1) 2 - c 2 p12 = (E 2 + E3) 2 - (p2 + p3) 2c 2

L.H.S. = (m 20 c 4 - E1) 2 - c 2p12 = (m02 + m12) c 4 - 2m0c 2E1

The R.H.S. is an invariant. We can evaluate it in any frame. Choose the C.M. frame
of the particles 2 and 3.

In this frame, R.H.S. = (E 2¿ + E 3¿ ) 2 = (m2 + m3) 2c 4

Thus, (m02 + m12) c 4 - 2m0c 2E1 = (m2 + m3)2c 4

m02 + m12 - (m2 + m3)2


or 2m0c 2E1 … E m02 + m12 - (m2 + m3)2 F c4 or E1 … c2
2m0

1.388 The velocity of ejected gases is u relative to the rocket. In an Earth centered frame
it is
v - u
1 - vu>c2
in the direction of the rocket. The momentum conservation equation then reads
v - u
(m + dm) (v + dv) + ( - dm) = mv
1 - uv>c2

v - u
or mdv - a - vb dm = 0
1 - uv>c2
Here, - dm is the mass of the ejected gases. So,
- u + uv>c2
dm = 0 or mdv + u a 1 - b dm = 0
v2
mdv -
1 - uv2>c2 c2
1.8 RELATIVISTIC MECHANICS 221

(neglecting 1 - uv/c 2, since u is non-relativistic).


v
Integrating, using b = , we get
c
db u dm
+ = 0
31 - b
2 c 3 m
1 + b u
or ln + ln m = constant
1 - b c
u
The constant = ln m0, since b = 0 initially.
c
1 - b 1 - 1m>mo2u>c
= a b or b =
m u>c
1 + 1m>mo2u>c
Thus,
1 + b m0
THERMODYNAMICS AND
MOLECULAR PHYSICS
PART 2
2.1 Equation of the Gas State. Processes

2.1 Let m1 and m2 be the masses of the gas in the vessel before and after the gas is
released. Hence mass of the gas released,
¢m = m1 – m2
Now from ideal gas equation
R R
p1V = m1 M T0 and p2V = m2 M T0

as V and T are same before and after the release of the gas.
R R
So, (p1 - p2)V = (m1 - m2) T = ¢m T0
M 0 M
(p1 - p2)VM ¢pVM
or ¢m = = (1)
RT0 RT0
R M r
We also know p = r T so, = (2)
M RT0 p0
(where p0 = standard atmospheric pressure and T0 = 273 K).
From Eqs. (1) and (2), we get

¢p 0.78
¢m = rV = 1.3 * 30 * = 30 g
p0 1

2.2 Let V is volume of each vessel. For the vessel which contained the ideal gas,
p1V = n1RT1
When heated, some gas, passes into the evacuated vessel till pressure difference be-
comes ¢p. Let p¿1 and p¿2 be the pressure on the two sides of the valve. Then
p¿1 V = n¿1 RT2

and p¿2 V = n¿2 RT2 = (n1 - n¿1)RT2


224 PART TWO THERMODYNAMICS AND MOLECULAR PHYSICS

Putting the values of n1 and n2 from the first two equations


p1V p¿1V p1 p¿1
p¿2 V = a - b RT2 or p¿2 = a - bT
RT1 RT2 T1 T2 2

But, p¿1 - p¿2 = ¢p


p1 p¿2 + ¢p
So, p¿2 = a - b T2
T1 T2
p1T2
= - p¿2 - ¢p
T1

1 p1T2
or p¿2 = a - ¢p b = 0.08 atm
2 T1

2.3 Let the mixture contain n1 and n2 moles of H2 and He, respectively. If molecular weights
of H2 and He are M1 and M2, then their respective masses in the mixture are
m1 = n1M1 and m2 = n2M2
Therefore, for the total mass of the mixture we get,
m = m1 + m2 or m = n1M1 + n2M2 (1)
Also, if n is the total number of moles of the mixture in the vessels, then we know,
n = n1 + n2 (2)
Solving Eqs. (1) and (2) for n1 and n2, we get
(nM2 - m) m - nM1
n1 = , n2 =
M2 - M1 M2 - M1

Therefore, we get
(nM2 - m) (m - nM1)
m1 = M1 and m2 = M2
M2 - M1 M2 - M1
m1 M1 (nM2 - m)
or =
m2 M2 (m - nM1)
One can also express the above result in terms of the effective molecular weight M of
the mixture, defined as
m RT
M = n = m
pV
m1 M1 M2 - M 1 - M>M2
Thus, = # =
m2 M2 M - M1 M>M1 - 1
2.1 EQUATION OF THE GAS STATE. PROCESSES 225

Using the data and table, we get


m1
M = 3.0 g and m = 0.50
2

2.4 We know, for the mixture, N2 and CO2 (being regarded as ideal gases, their mixture
too behaves like an ideal gas) , p0V = nRT , where, n is the total number of moles of
of the gases (mixture) present, p0 is the pressure and V is the volume of the vessel.
If n1 and n2 are number of moles of N2 and CO2, respectively present in the mixture,
then n = n1 + n2.
Now number of moles of N2 and CO2 is, by definition, given by
m1 m2
n1 = and n2 =
M1 M2
where, m1 is the mass of N2 (molecular weight = M1) in the mixture and m2 is the
mass of CO2 (molecular weight = M2) in the mixture.
Therefore density of the mixture is given by
m1 + m2 m1 + m2
r = =
V (nRT> p0)
p0 m1 + m2 p0(m1 + m2)M1M2
= =
RT n1 + n2 RT (m1M2 + m2M1)
= 1.5 kg/m3 (on substituting values)

2.5 (a) The mixture contains n1, n2 and n3 moles of O2, N2 and CO2, respectively. Then
the total number of moles of the mixture
n = n1 + n2 + n3
We know, ideal gas equation for the mixture
nRT
pV = nRT or p =
V
(n1 + n2 + n3)RT
or p = = 1.968 atm (on substituting values)
V

(b) Mass of oxygen (O2) present in the mixture: m1 = n1M1


Mass of nitrogen (N2) present in the mixture: m2 = n2M2
Mass of carbon dioxide (CO2) present in the mixture: m3 = n3M3
So, mass of the mixture:
m = m1 + m2 + m3 = n1M1 + n2M2 + n3M3
226 PART TWO THERMODYNAMICS AND MOLECULAR PHYSICS

Molecular mass of the mixture

mass of the mixture


M =
total number of moles
n1M + n M + n M
1 2 2 3 3
= = 36.7 g/mol (on substituting values)
n1 + n2 + n3

2.6 Let p1 and p2 be the pressure in the upper and lower part of the cylinder, respectively
at temperature T0. At the equilibrium position for the piston:

p1S + mg = p2S
mg
or p1 + = p2 (where m is the mass of the piston)
S
RT0
But, p1 = (where V0 is the initial volume of the lower part)
hV0

RT0 RT0 RT0


a1 - b
mg mg 1
So, + = Q = (1)
hV0 S V0 S V0 h

Let T ¿ be the sought temperature and at this temperature the volume of the lower part be-
comes V ¿, then according to the problem the volume of the upper part becomes h¿V .

a1 - b
mg RT 1
Hence, = (2)
S V¿ h¿

From Eqs. (1) and (2),


RT0 RT ¿ T011 - 1>h2V ¿
a1 - b= a1 - b or T ¿ =
1 1
V011 - 1>h¿2
(3)
V0 h V¿ h¿

As, the total volume must be constant, so


V0(1 + h)
V0(1 + h) = V ¿(1 + h¿) or V ¿ =
1 + h¿

Putting the value of V ¿ in Eq. (3), we get


(1 + h)
T011 - 1>h2V0
(1 + h¿)
T =
V0 11 - 1>h¿2

T0(h2 - 1)h¿
= = 0.42 kK
(h¿ 2 - 1)h
2.1 EQUATION OF THE GAS STATE. PROCESSES 227

2.7 Let r1 be the density after the first stroke. The mass remains constant.
Vr
So, Vr = (V + ¢V ) r1 Q r1 =
(V + ¢V )
Similarly, if r2 is the density after second stroke
Vr1 = (V + ¢V ) r2

r2 = a br1 = a b r0
V V 2
or
V + ¢V V + ¢V
In this way after nth stroke,

rn = a b r0
n
V
V + ¢V
Since pressure r density,

pn = a b p0 (because temperature is constant)


n
V
V + ¢V
It is required by pn >p0 to be 1>h,

a b or h = a b
n
1 V V + ¢V n
So, =
h V + ¢V V
ln h
n =
ln 11 + ¢V >V2
Hence,

m RT
2.8 From the ideal gas equation p =
M V
dp RT dm
= (1)
dt MV dt
In each stroke, volume v of the gas is ejected, where v is given by
V
v = m [mN - 1 - mN ]
N

In case of continuous ejection, if mN - 1 corresponds to mass of gas in the vessel at time


t, then mN is the mass at time t + ¢t, where ¢t, is the time in which volume v of the
gas has come out.
The rate of evacuation is therefore v >¢t, i.e.,

C =
v
= -
V # m (t + ¢t) - m (t)
¢t m (t + ¢t) ¢t
In the limit ¢t : 0, we get
V dm
C = (2)
m dt
228 PART TWO THERMODYNAMICS AND MOLECULAR PHYSICS

From Eqs. (1) and (2)


dp C mRT C dp C
= - = - p or = - dt
dt V MV V p V
p0 0
dp C
Integrating we get, = - dt
3 p V3
p t
p C
or ln = - t
p0 V

Thus, p = p0e - Ct>V

2.9 Let r be the instantaneous density, then instantaneous mass = Vr. In a short interval
dt the volume is increased by Cdt.
So, Vr = (V + Cdt) (r + dr) (because mass remains constant in a short interval dt).
dr C
So, = - dt
r V
Since pressure r density,
dp C
p = - V dt
p2
dp C
or - p = t
3 V
p1

V p1 V 1
or t = ln p = ln h = 1.0 min
C 2 C

2.10 The physical system consists of one mole of gas confined in the smooth vertical tube.
Let m1 and m2 be the masses of upper and lower pistons and S1 and S2 be their
respective areas.
For the lower piston p0
pS2 + m2g = p0S2 + T
T
or T = (p - p0) S2 + m2g (1)
p
Similarly for the upper piston
T
p0S1 + T + m1g = pS1
p0
or T = (p - p0)S1 - m1g (2)
2.1 EQUATION OF THE GAS STATE. PROCESSES 229

From Eqs. (1) and (2)


(p - p0)(S1 - S2) = (m1 + m2) g
or (p - p0) ¢S = mg
mg
So, p = + p0 = constant
¢S
From the gas law, pV = nRT or p¢V = nR¢T (because p is constant) .

a p0 + b ¢Sl = R¢T
mg
So,
¢S

1
Hence, ¢T = (p0 ¢S + mg)l
R
= 0.9 K

2.11 (a) Given that p = p0 - aV 2 = p0 - a1RT>p22 (as V = RT>p for one mole of gas).

1 1
Thus, T = p 1p0 - p = 2p0 p 2 - p 3 (1)
R 1a R 1a

d
For Tmax, (p p 2 - p 3) = 0
dp 0

which yields, p = 2>3 p0


Hence, using this value of p in Eq. (1), we get

2 p0 p0
p0 p0 - p0 = a b
1 #2 2
Tmax =
R 1a 3 A 3 3 R C 3a

(b) Given that p = p0e - bV = p0e - bRT>p

b RT p0 p p0
So, = ln and T = ln p (2)
p p bR

For Tmax, the condition is dT >dp = 0, which yields p = p0 >e.


Hence using this value of p in Eq. (2), we get
p0
Tmax =
ebR

2.12 Given that T = T0 + aV 2 = T0 + a (R 2T 2>p 2) (as V = RT>p for one mole of gas).

So, p = 1a RT (T - T0)-1>2 (1)


230 PART TWO THERMODYNAMICS AND MOLECULAR PHYSICS

dp
For Pmin, = 0, which gives
dT
T = 2T0 (2)
From Eqs. (1) and (2), we get
pmin = 1a R 2T0 (2T0 - T0 )-1>2 = 2R 1aT0

2.13 Consider a thin layer at a height h and thickness dh. Let p and dp + p be the pres-
sures on the two sides of the layer. The mass of the layer is Sdhr. Equating vertical
downward force to the upward force acting on the layer, we get
Sdhrg + (p + dp)S = pS
dp
So, = - rg (1)
dh
(p+dp)S
r
But, p = RT dh
M
rR pS
So, dp = dT h
Sdh
M
rR
or - dT = rgdh (using Eq. 1)
M
dT gM
So, = - = - 34 K>km
dh R
That means, temperature of air drops by 34 K at a height of 1 km above bottom.
2.14 From the previous problem we have,
dp
= - rg (1)
dh
But, from p = Crn (where C is a constant), we get
dp >dr = Cnrn - 1 (2)
We have from gas law,

T or Crn = r # T
R R
p = r
M M
M
So, T = Crn - 1 (using Eq. 2)
R
dT M#
Thus, = C (n - 1) rn –2 (3)
dr R
dT dT # dr # dp
But, =
dh dr dp dh
2.1 EQUATION OF THE GAS STATE. PROCESSES 231

Substituting from Eqs. (1), (2) and (3) we get


dT M 1 - Mg (n - 1)
= C (n - 1) rn - 2 n - 1
( - rg) =
dh R Cnr nR

2.15 We have, dp = - rgdh and from gas law r = (M>RT )p.


dp Mg
Thus, = - dh
p RT
Integrating, we get
p h
dp Mg
= - dh
3 p RT 3
p0 0

p Mg
or ln = - h (where p0 is the pressure at the surface of the Earth)
p0 RT

So, p = p0e-Mgh>RT
Under standard conditions, p0 = 1 atm, T = 273 K
Pressure at a height of 5 km = 1 * e-28 * 9.81 * 5000>8.314 * 273 = 0.5 atm
Pressure in a mine at a depth of 5 km = 1 * e-28 * 9.81 * (-5000)>8.314 * 273 = 2 atm

2.16 We have dp = - rgdh and from gas law p = ( r>M )RT .


dr
Thus, dp = RT (at constant temperature)
M
dr gM
So, = dh
r RT
Integrating within limits
r h
dr gM
= dh
3 r 3 RT
r0 0

r gM
or ln = h
r0 RT
RT r
So, r = r0e-Mgh>RT and h = - ln
Mg r0

(a) Given that T = 273 K and r0 >r = e.


232 PART TWO THERMODYNAMICS AND MOLECULAR PHYSICS

RT
Thus, h = - ln e-1
Mg
RT
= = 8 km.
Mg

(b) Given that T = 273 K and (r0 - r)>r0 = 0.01 or r>r0 = 0.99.

RT r
Thus, h = - ln = 0.09 km (on substituting values)
Mg r0

2.17 From the Barometric formula, we have

p = p0e-Mgh>RT
pM
and from gas law r =
RT
So, at constant temperature from these two equations
Mp0
r = e-Mgh>RT = r0e-Mgh>RT (1)
RT
Eq. (1) shows that density varies with height in the same manner as pressure. Let us
consider the mass element of the gas contained in the column.
Mp0
dm = r (Sdh) = e-Mgh>RT Sdh
RT
Hence the sought mass,
h
Mp0S p0S
m = e - Mgh>RT dh = (1 - e-Mgh>RT )
RT 3 g
0

2.18 As the gravitational field is constant the centre of gravity and the centre of mass
(C.M.) are same. The location of C.M.
q q

hdm h rdh
3 3
0 0
h = q
= q

dm rdh
3 3
0 0

But from Barometric formula and gas law r = r0e-Mgh>RT


2.1 EQUATION OF THE GAS STATE. PROCESSES 233

h(e-Mgh>RT )dh
3
0 RT
So, h = q
=
Mg
(e-Mgh>RT )dh
3
0

2.19 (a) We know that the variation of pressure with height of a fluid is given by
dp = - rgdh
r pM
But from gas law p = RT or r =
M RT

From these two equations


pMg
dp = - dh (1)
RT
dp - Mgdh
or =
p RT0(1 - ah)
p h
dp - Mg dh
Integrating we get, =
3 p RT0 3 (1 - ah)
p0 0

p
or ln = ln (1 - ah) Mg>aRT0
p0

p = p0(1 - ah)Mg>aRT0 aobviously h 6 b.


1
Hence,
a

(b) Proceed up to Eq. (1) of part (a), and then put T = T0(1 + ah) and proceed fur-
ther in the same fashion to get
p0
p =
(1 + ah)Mg>aRT0

2.20 Let us consider the mass element of the gas (thin layer) in the w
cylinder at a distance r from its open end as shown in the figure.
Using Newton’s second law for the element
Fn = mwn r
dr
(p + dp) S - pS = ( r S dr) ␻2r
234 PART TWO THERMODYNAMICS AND MOLECULAR PHYSICS

pM
or dp = rv2rdr = v2rdr
RT
dp Mv2
So, = rdr
p RT
p r
dp
Mv2
or = rdr
3 p RT 3
p0 0

p Mv2 2
= r or p = p0e Mv r >2RT
2 2
Thus, ln
p0 2RT

2.21 For an ideal gas law


r
p = RT
M
500
On substitution, we get p = 0.082 * 300 * atm = 279.5 atm
44
From Van der Waal gas equation

ap + b (V - nb) = n RT (where V = nVM )


n2a
V2
nRT an2 mRT>M am2
or p = - 2 = - 2 2
V - nb V V - mb>M V M
rRT ar2
= - 2 = 79.2 atm
M - rb M

2.22 (a) p = c - 2 d (1 + h) =
RT a RT
, (where VM is the molar volume)
VM - b VM VM
(The pressure is less for a Van der Waal gas than for an ideal gas.)

a(1 + h) -1 -1 + h hVM + b
or = RT c + d = RT
2
VM VM VM - b VM (VM - b)

a (1 + h)(VM - b)
or T =
RVM (hVM + b)
1.35 * 1.1 * (1 - 0.039)
= L 125 K
0.082 * (0.139)
2.1 EQUATION OF THE GAS STATE. PROCESSES 235

(b) The corresponding pressure is


RT a a (1 + h) a
p = - 2 = - 2
VM - b VM VM (hVM + b) VM
a (VM + hVM - hVM - b) a (VM - b)
= = 2
2
VM (hVM + b) V M (VM + b)
1.35 0.961
= * L 9.3 atm
1 0.139

1 a 1 a
2.23 Since, p1 = RT1 - 2 and p2 = RT2 - 2
V - b V V - b V
R (T2 - T1)
So, p2 - p1 =
V - b
R (T2 - T1) R (T2 - T1)
or V - b = Qb = V -
p2 - p1 p2 - p1
p2 - p1 a
Also, p1 = T1 -
T2 - T1 V2
a T1 ( p2 - p1) T1 p2 - p1 T2
Q = - p1 =
V 2 T2 - T1 T2 - T1
T1 p2 - p1T2
or a = V2
T2 - T1
Using T1 = 300 K, p1 = 90 atm, T2 = 350 K, p2 = 110 atm,V = 0.250, we get a = 1.87 atm
l2>mol2, and b = 0.045 l/mol.

0p
- 2 - V a b =
RT a RTV 2a
2.24 p = - 2
V - b V 0V T (V - b)2 V
- 1 0V
or k = a b
V 0p T
RTV 3 - 2a (V - b)2 -1 V 2(V - b)2
= c d =
V 2(V - b)2 [RTV 3 - 2a (V - b)2]

2.25 For an ideal gas, k0 = V>RT.

(V - b)2 2a (V - b)2 -1 b 2 -1
e1 - f a b e a b f
b 2 2a
Now, k = = k0 1 - 1 - 1 -
RTV RTV 3 V RTV V

= k0 e 1 - f (to leading order in a, b)


2b 2a
+
V RTV
236 PART TWO THERMODYNAMICS AND MOLECULAR PHYSICS

2a 2b a
Now, k 7 k0 if 7 or T 6
RTV V bR
If a and b do not vary much with temperature, then the effect at high temperature is
clearly determined by b and its effect is repulsive so compressibility is less.

2.2 The First Law of Thermodynamics. Heat Capacity

2.26 Internal energy of air, treating it as an ideal gas is given by


m m R pV
U = CVT = T = (1)
M M g - 1 g - 1
(using (CV = R>(g - 1), since Cp - CV = R and Cp >CV = g).
Thus at constant pressure U = constant, because the volume of the room is a constant.
Putting the value of p = patm and V in Eq. (1), we get U = 10 M J.

2.27 From energy conservation


1
Ui + (nM ) v 2 = Uƒ
2
or ¢U = (1>2)nMv 2 (1)
nRT
Using, U = (from previous problem) (2)
g - 1
nR
We get ¢U = ¢T
g - 1
Hence from Eqs. (1) and (2),
Mv 2 (g - 1)
¢T =
2R

2.28 On opening the valve, the air will flow from the vessel at higher pressure to the ves-
sel at lower pressure till both vessels have the same air pressure. If this air pressure
is p, the total volume of the air in the two vessels will be (V1 + V2). Also if n1 and n2
be the number of moles of air initially in the two vessels, we have
p1V1 = n1RT1 and p2V2 = n2RT2 (1)
After the air is mixed, the total number of moles are (v1 + v2) and the mixture is at tem-
perature T.
Hence, p (V1 + V2) = (n1 + n2)RT (2)
Let us look at the two portions of air as one single system. Since this system is con-
tained in a thermally insulated vessel, no heat exchange is involved in the process.
That is, total heat transfer for the combined system Q = 0.
2.2 THE FIRST LAW OF THERMODYNAMICS. HEAT CAPACITY 237

Moreover, this combined system does not perform mechanical work either. The walls of
the containers are rigid and there are no pistons, etc to be pushed, looking at the total system,
we know A = 0.
Hence, internal energy of the combined system does not change in the process.
Initially, energy of the combined system is equal to the sum of internal energies of the
two portions of air:

n1RT1 n2RT2
Ui = U1 + U2 = + (3)
g - 1 g - 1

Final internal energy of (n1 + n2) moles of air at temperature T is given by

(n1 + n2) RT
Uƒ = (4)
g - 1
Therefore, Ui = Uf implies
n1T1 + n2T2 p1V1 + p2V2 p1V1 + p2V2
T = = = T1T2
n1 + n2 (p1V1>T1) + (p2V2>T2) p1V1T2 + p2V2T1
From Eq. (2) therefore, final pressure is given by
n1 + n2 R p1V1 + p2V2
p = RT = (n1T1 + n2T2) =
V1 + V2 V1 + V2 V1 + V2
This process in an example of free adiabatic expansion of ideal gas.

2.29 By the first law of thermodynamics, Q = ¢U + A.


Here A = 0, as the volume remains constant.
nR
So, Q = ¢U = ¢T
g - 1
From gas law, p0V = nRT0

p0V¢T
So, ¢U = - = - 0.25 k J
T0(g - 1)
Hence amount of heat lost = - ¢U = 0.25 k J

2.30 By the first law of thermodynamics, Q = ¢U + A.


p ¢V A
But, ¢U = = (as p is constant)
g - 1 g - 1
A g#A 1.4
So, Q = + A = = * 2 = 7J
g - 1 g - 1 1.4 - 1
238 PART TWO THERMODYNAMICS AND MOLECULAR PHYSICS

2.31 Under an isobaric process,


A = p¢V = R¢T (as n = 1) = 0.6 k J.
From the first law of thermodynamics,
¢U = Q - A = Q - R¢T = 1 k J
R¢T
Again increment in internal energy, ¢U = (for n = 1)
g - 1

R ¢T Q
Thus, Q - R ¢T = or g = = 1.6
g - 1 Q - R ¢T

2.32 Given n = 2 moles of the gas. In the first phase, under isochoric process, A1 = 0, there-
fore from gas law if pressure is reduced n times, the temperature, i.e., new temperature
becomes T0>n.
Now from first law of thermodynamics,
nR¢T
Q1 = ¢U1 =
g - 1
T0 nRT0 (1 - n)
a - T0 b =
nR
=
g - 1 n n (g - 1)
During the second phase (under isobaric process), A2 = p¢V = nR ¢T.
Thus from first law of thermodynamics,
nR¢T
Q2 = ¢U2 + A2 = + nR¢T
g - 1
T0
nR a T0 - bg
n nRT0(n - 1)g
= =
g - 1 n (g - 1)
Hence the total amount of heat absorbed,
nRT0(1 - n) nRT0(n - 1)g
Q = Q1 + Q2 = +
n (g - 1) n (g - 1)
nRT0(n - 1)
( - 1 + g) = nRT0 a 1 - b = 2.5 kJ (on substituting values)
1
=
n (g - 1) n

2.33 Number of moles in the mixture n = n1 + n2


At a certain temperature, U = U1 + U2 or nCV = n1CV + n2CV
1 2

a n1 b
R R
n1CV + n2CV + n2
g1 - 1 g2 - 1
CV = =
1 2
Thus,
n n
2.2 THE FIRST LAW OF THERMODYNAMICS. HEAT CAPACITY 239

n1Cp + n2Cp
Cp =
1 2
Similarly,
n
g1R g2R
n1g1CV + n2g2CV a n1 + n2 b
g1 - 1 g2 - 1
= =
1 2

n n
g1 g2
n1 R + n2 R
Cp g1 - 1 g2 - 1
Thus, g = =
CV R R
n1 + n2
g1 - 1 g2 - 1
n1g1(g2 - 1) + n2g2(g1 - 1)
=
n1(g2 - 1) + n2(g1 - 1)
= 1.33 (on substituting values)

2.34 From the previous problem,


R R
n1 + n2
g1 - 1 g2 - 1
CV = = 15.2 J>mol K (on substituting values)
n1 + n2
g1R g2R
n1 + n2
g1 - 1 g2 - 1
and Cp = = 23.85 J>mol K (on substituting values)
n1 + n2

Total mass 20 + 7
Now molar mass of the mixture (M ) = = = 36
Total number of moles (1>2) + (1>4)
CV Cp
Hence, cV = = 0.42 J>gK and cp = = 0.66 J>gK
M M

2.35 Let S be the area of the piston and F be the force exerted by the external agent.
Then, F + pS = p0S (see figure) at an arbitrary instant of time. Here p is the pressure at
the instant the volume is V. (Initially the pressure inside is p0 ). Work done by the agent,
hV0
F
A = F dx
3
V0
p0S
hV0 hV0

= (p0 - p)S # dx = (p0 - p)dV pS


3 3 V
V0 V0
240 PART TWO THERMODYNAMICS AND MOLECULAR PHYSICS

hV0 hV0

nRT #
dV
= p0(h - 1)V0 - pdV = p0(h - 1)V0 -
3 3 V
V0 V0
= (h - 1) p0V0 - nRT ln h = (h - 1) nRT - nRT ln h
= nRT (h - 1 - ln h) = RT (h - 1 - ln h) (for n = 1 mole)

2.36 Let the agent move the piston to the right by x. In equilibrium position,
p1S + Fagent = p2S or Fagent = (p2 - p1)S

Work done by the agent in an infinitesimal change dx is


Fagent # dx = (p2 - p1) Sdx = (p2 - p1)dV

By applying pV = constant, for the two parts,


p1 (V0 + Sx) = p0V0 and p2 (V0 - Sx) = p0V0

p0V0 2Sx 2p0V0V


So, p2 - p1 = - S 2x 2 = (where Sx = V )
V 02 V 02 - V 2
When the volume of the left end is h times the volume of the right end,
h - 1
(V0 - V) = h (V0 - V ) or V = V
h + 1 0
v V
2p0V0V
A = (p2 - p1)dV = dV = - p0V0 [ln (V02 - V 2)]V0
3 V0 - V
2 2
3
0 0
= - p0V0 3ln1V 02 - V 22 - ln V 024
h - 1 2 2
= - p0V0 c ln eV 02 - a b V 0 f - ln V02 d
h + 1
(h + 1)2
= - p0V0 a ln b
4h
= p V ln
(h + 1)2 0 0
4h

2.37 In the isothermal process, heat transfer to the gas is given by


V2 V2 p1
Q1 = nRT0 ln = n RT0 ln h a for h = = b
V1 V1 p2
In the isochoric process, A = 0. Thus heat transfer to the gas is given by

¢T a for CV = b
nR R
Q2 = ¢U = nCV ¢T =
g - 1 g - 1
2.2 THE FIRST LAW OF THERMODYNAMICS. HEAT CAPACITY 241

p2 T0 p1 p1
But, = Q T = T0 = hT0 a for h = b
p1 T p2 p2
or ¢T = hT0 - T0 = (h - 1)T0
nR
So, Q2 = . (h - 1)T0
g - 1
Thus, net heat transfer to the gas is
nR
Q = nRT0 ln h + . (h - 1)T0
g - 1
Q h - 1 Q h - 1
or = ln h + Q - ln h =
nRT0 g - 1 nRT0 g - 1
h - 1 6 - 1
or g = 1 + = 1 + = 1.4
Q 80 * 103
- ln h a b - ln 6
nRT0 3 * 8.314 * 273

2.38 (a) From ideal gas law, p = 1nR>V2T = kT 1where k = nR>V 2.


For isochoric process, V = constant, thus p = kT, represents a straight line pass-
ing through the origin and its slope is equal to k.
For isobaric process p = constant, thus on p -T curve, it is a horizontal straight line
parallel to T-axis, if T is along horizontal (or x-axis).
For isothermal process, T = constant, thus on p -T curve, it represents a vertical
straight line if T is taken along horizontal (or x -axis).
For an adiabatic process T gp 1 - g = constant.
On differentiating, we get
(1 - g) p-gdp T g + gp1 - g # T g - 1 # dT = 0
p1-g T g-1
= a b a -g b a g b = a b
So, dp g g p
dT 1 - g p T g - 1 T

The approximate plots of isochoric, isobaric, isothermal, and adiabatic processes


are drawn in the answer sheet.
(b) As p is not considered variable, we have from ideal gas law

T = k¿T awhere k¿ = b
nR nR
V =
p p
On V- T co-ordinate system let us, take T along x -axis.
For an isochoric process, V = constant, thus k ¿ = constant and V = k¿T obviously
represents a straight line passing through the origin of the coordinate system and
k¿ is its slope.
242 PART TWO THERMODYNAMICS AND MOLECULAR PHYSICS

For isothermal process T = constant. Thus on the stated coordinate system it repr-
esents a straight line parallel to the V -axis.
For an adiabatic process, T V g - 1 = constant.
On differentiating, we get
(g - 1)V g - 2 dVT + V g - 1dT = 0

= -a b#
So, dV 1 V
dT g - 1 T
The approximate plots of isochoric, isobaric, isothermal and adiabatic processes
are drawn in the answer sheet.
2.39 (a) According to T- p relation in adiabatic process, T g = kp g - 1 (where k  constant).
Therefore, 1T2 >T12g = 1p2 >p12g - 1
So, T g>T0 g = hg - 1 1for h = p2 >p12
Hence, T = T0 # hg - 1>g = 290 * 10(1.4 - 1)>1.4 = 0.56 kK
(b) Using the solution of part (a), sought work done
nRT0
1h(g–1)/g - 12  5.61 kJ (on substituting values)
nR¢T
A = =
g - 1 g - 1
2.40 Let (p0, V0, T0) be the initial state of the gas.
We know that work done by the gas in adiabatic process is
- nR ¢T
Aadia =
g - 1
But from the equation T V –1  constant, we get T  T0 (1 – 1).
- nRT0(hg - 1 - 1)
Thus, Aadia =
g - 1
On the other hand, work done by the gas in isothermal process is
A iso = nRT0 ln (1>h) = - nRT0 ln h
A adia hg - 1 - 1 50.4 - 1
Thus, A iso = = = 1.4
(g - 1) ln h 0.4 * ln 5

2.41 Since here the piston is conducting and it is moved slowly, the temperature on the two
sides increases and is maintained at the same value.
Elementary work done by the agent  work done in compression – work done in
expansion, i.e., dA = p2dV – p1dV = (p2 – p1) dV, where p1 and p2 are pressures at any
instant of the gas on expansion and compression side, respectively.
2.2 THE FIRST LAW OF THERMODYNAMICS. HEAT CAPACITY 243

From the gas law p1 (V0 + Sx) = nRT and p2 (V0 – Sx) = nRT, for each section
(x is the displacement of the piston towards section 2).

= nRT # 2
2Sx 2V
So, p2 – p1 = nRT (as Sx  V )
V - S x
2
0
2 2 V0 - V2
2V
So, dA = nRT dV
V - V2
2
0
Also, from the first law of thermodynamics p1S
R
dA  dU  2 n dT (as dQ  0) p2S
g - 1
Fagent
R
So, work done on the gas  dA  2 n dT
g - 1

R 2V # dV
Thus, 2n dT  nRT 2
g - 1 V 0 - V2
dT VdV
or = g - 1 2
T V 0 - V2
When the left end is  times the volume of the right end,
h - 1
(V0 V )  (V0  V ) or V  V
h + 1 0
T V
dT VdV
On integrating, = (g - 1)
3V0 - V
2 2
3 T
T0 0

 ( – 1) c - ln 1V 02 - V 22 d
T 1 V
or ln
T0 2 0

g - 1
- [ln 1V 02 - V 22 - lnV 02]
2
g - 1 h - 1 2 g - 1 (h + 1)2
 c ln V 02 - ln V 02 e 1 - a b fd = In
2 h + 1 2 4h
g-1
(h + 1)2
Hence, T = T0 a b 2
4h

2.42 From energy conservation as in the derivation of Bernoulli’s theorem it reads


p 1 2
v gz u Qd  constant (1)
r 2
244 PART TWO THERMODYNAMICS AND MOLECULAR PHYSICS

In Eq. (1), u is the internal energy per unit mass and in this case is the thermal energy
per unit mass of the gas. As the gas vessel is thermally insulated Qd  0. Just inside
the vessel,
CVT RT p RT
u  also 
M M(g - 1) r M

Inside the vessel v = 0 and just outside p  0, and u  0. In general gz is not very
significant for gases.
Thus applying Eq. (1) just inside and outside the hole, we get
1 2 p
v  u
2 r
RT RT gRT
 + =
M M (g - 1) M(g - 1)
2gRT
Hence, v2 =
M(g - 1)
2gRT
or v = = 3.22 km/s
A M (g - 1)

Alternate:
From Eulerian equation
- ¥p + rg = ra
In projection form along a stream line for steady flow, it
becomes,
- 0p 0v
+ g cos a = rv
0s 0s
In this problem,
- dp dv
= rv (only because a becomes 90°)
ds ds
Experiments show, when the particles move sufficiently fast it can be assumed that
the process is adiabatic. Then the connection between the pressure and the density is
given by the relation,
p pi
= = constant
rg rgi
where g is the adiabatic exponent dependent on the nature of the gas and ri is the
density of the gas inside the vessel. The adiabatic relation is a consequence of the fact
that during the expansion of a gas there is no heat exchange with the surrounding,
that is there is no loss or gain of heat.
2.2 THE FIRST LAW OF THERMODYNAMICS. HEAT CAPACITY 245

On finding the density as a function of the pressure and substituting, we obtain the
equation
dp ri dv
- p-1>g = v
ds p 1>g
i
ds
This equation should be integrated along the stream tube. Since the pressure in the
vessel is pi and the pressure outside the vessel is p0, we must integrate from pi to p0
for the pressure and from zero to v = v0 (the outlet velocity) for the velocity.
p0 v0
ri
Thus, - p-1>g dp = v dv
3 pi1>g 3
pi 0

On performing the integration we find the outlet velocity,


2g pi p0 g g- 1
v = c 1 - a 1>g b d
C g - 1 ri pi
According the problem p0 = 0 because outside is vacuum.
2g pi 2g RT
Hence, v = = = 3.3 km/s (on substituting values)
A g - 1 ri Ag - 1 M
Note: The velocity here is the velocity of hydrodynamic flow of the gas into vacuum.
This requires that the diameter of the hole is not too small (D 7 mean free path l ). In
the opposite case (D V l ), the flow is called effusion. Then the above result does not
apply and kinetic theory methods are needed.

2.43 The differential work done by the gas,

a- 2 b dT = - nRdT 1as pV = nRT and V = a>T2


nRT 2 a
dA = pdV =
a T
T + ¢T

So, A =- nRdT = - nR¢T


3
T
From the first law of thermodynamics
nR
Q = ¢U + A = ¢T - nR¢T
g - 1
2 - g 2 - g
= nR¢T # = R ¢T # (for n = 1 mole)
g - 1 g - 1

2.44 According to the problem: A r U or dA = aU (where a is proportionality constant).


anRdT
So, pdV = (1)
g - 1
246 PART TWO THERMODYNAMICS AND MOLECULAR PHYSICS

From ideal gas law, pV = nRT, which on differentiating gives


pdV + Vdp = nRdT (2)
Thus, from Eqs. (1) and (2)
a
pdV = ( pdV + Vdp)
g - 1

pdV a
a a
or - 1b + Vdp = 0
g - 1 g - 1
a
or pdV (k - 1) + kVdp = 0 (where k = = another constant)
g - 1
k - 1
or pdV + Vdp = 0
k
k - 1
or npdV + Vdp = 0 awhere = n = ratiob
k
Dividing both the sides by pV, we get
dV dp
n + = 0
V p
On integrating we get,
n ln V + ln p = ln C (where C is constant)
or ln (pV n) = ln C or pV n = C

2.45 In the polytropic process work done by the gas is given by,
nR C Ti - Tƒ D
A =
n - 1
(where Ti and Tƒ are initial and final temperatures of the gas like in an adiabatic process).
nR
Also, ¢U = (T - Ti )
g - 1 ƒ
By the first law of thermodynamics,
Q = ¢U + A
nR nR
= (Tf - Ti ) + (T - Tƒ )
g - 1 n - 1 i
nR [n - g]
= (Tƒ - Ti )nR c d =
1 1
- ¢T
g - 1 n - 1 (n - 1) (g - 1)
According to definition of molar heat capacity when number of moles n = 1 and
¢T = 1, then Q = molar heat capacity.
2.2 THE FIRST LAW OF THERMODYNAMICS. HEAT CAPACITY 247

R (n - g)
Here, Cn = 6 0 for 1 6 n 6 g
(n - 1) (g - 1)

2.46 Let the process be polytropic, according to the law pV n = constant.


Thus, pƒVƒn = pi Vin
or 1pi >pƒ2 = b
ln b
So, an = b or ln b = n ln a or n =
ln a
In the polytropic process molar heat capacity is given by
R (n - g) R R
Cn = = -
(n - 1) (g - 1) g - 1 n - 1

a where n = b
R R ln a ln b
= -
g - 1 ln b ln a ln a
8.314 8.314 ln 4
So, Cn = - = - 42 J> mol K
1.66 - 1 ln 8 - ln 4

2.47 (a) Increment of internal energy for ¢T becomes


nR¢T R ¢T
¢U = = = - 324 J (as n = 1 mole)
g - 1 g - 1
From first law of thermodynamics
R¢T R¢T
Q = ¢U + A = - = 0.11 kJ
g - 1 n - 1
(b) Sought work done,

dV 1where pV n = k = piVin = pƒVƒn2


k
An = pdV = n
3 3V
Vi

A pƒ Vƒn V 1ƒ - n - piVin V 1i - n B
AV - Vi B =
k - -
= 1 n 1 n
1 - n ƒ 1 - n
pƒVƒ - piVi nR (Tƒ - Ti )
= =
1 - n 1 - n
nR¢T R¢T
= = - = 0.43 kJ (as n = 1 mole)
n - 1 n - 1

2.48 Law of the process is, p = aV or pV - 1 = a, so the process is polytropic with index
n = -1. As p = aV so, pi = aV0 and pf = ah V0.
248 PART TWO THERMODYNAMICS AND MOLECULAR PHYSICS

(a) Increment of the internal energy is given by


nR pƒVƒ - piVi
¢U = [Tƒ - Ti ] =
g - 1 g - 1
Putting pi = aV0 and pƒ = a hV0, we get

aV 02 (h2 - 1)
¢U =
g - 1
(b) Work done by the gas is given by
piVi - pƒVƒ aV02 - ahV0 # hV0
A = =
n - 1 -1 - 1
aV 02 (1 - h2) 1
= = a V 02 (h2 - 1)
-2 2
(c) Molar heat capacity is given by
R (n - g) R ( - 1 - g) R g + 1
Cn = = = #
(n - 1) (g - 1) ( - 1 - 1) (g - 1) 2 g - 1

nR
2.49 (a) ¢U = ¢T and Q = nCn ¢T , (where Cn is the molar heat capacity in the
g - 1
process.) It is given that Q = - ¢U.
R R
So, Cn ¢T = - ¢T or Cn = -
g - 1 g - 1
(b) By the first law of thermodynamics,
dQ = dU + dA
or 2dQ = dA (as dQ = - dU)
2R n
2nCn dT = pdV or dT + pdV = 0
g - 1
2Rv nRT 2dT dV
So, dT + dV = 0 or + = 0
g - 1 V (g - 1) T V
dT g - 1 dV
or + = 0 Q TV (g - 1)>2 = constant
T 2 V
(n - g) R
(c) We know Cn =
(n - 1) (g - 1)
But from part (a), we have Cn = - R>(g - 1).
2.2 THE FIRST LAW OF THERMODYNAMICS. HEAT CAPACITY 249

R (n - g) R
Thus, - =
g - 1 (n - 1) (g - 1)
1 + g
which yields n =
2

From part (b), we know TV (g - 1)>2 = constant.


T0 V (g - 1)>2
So, = a b = h (g - 1)>2 (where T is the final temperature)
T V0
Work done by the gas for one mole is given by
R (T0 - T )
C 1 - h(1 - g>2) D
2RT0
A = =
n - 1 g - 1

2.50 Given p = aT a (for one mole of gas).


-a
pT -a = a or p a b
pV
So, = a
R
Q p 1 - aV -a = aR -a or pV a>(a - 1) = constant
Here polytropic exponent n = a>(a - 1).
(a) In the polytropic process for one mole of gas,
R¢T R¢T
A = = = R¢T (1 - a)
1 - n 1 - [a>(a - 1)]
(b) Molar heat capacity is given by
R R R R
C = - = -
g - 1 n - 1 g - 1 [a>(a - 1)] - 1
R
= + R (a - 1)
g - 1

2.51 Given U = aV a
pV
or nCVT = aV a or nCV = aV a
nR
CV
aV a #
R # 1
Q = 1 or V a - 1 # p-1 =
CV pV Ra

or pV 1 - a = Ra>CV = constant = a (g - 1) [as CV = R>(g - 1)]


So polytropic index n = 1 - a.
250 PART TWO THERMODYNAMICS AND MOLECULAR PHYSICS

(a) Work done by the gas is given by


- nR¢T nR¢T
A = and ¢U =
n - 1 g - 1
- ¢U (g - 1) ¢U (g - 1)
Hence, A = = (as n = 1 - a)
n - 1 a
By the first law of thermodynamics
Q = ¢U + A
¢U (g - 1) g - 1
= ¢U + = ¢U c 1 + d
a a
(b) Molar heat capacity is given by
R R R R
C = - = -
g - 1 n - 1 g - 1 1 - a - 1
R R
= + (as n = 1 - a)
g - 1 a

2.52 (a) By the first law of thermodynamics


dQ = dU + dA = nCVdT + pdV
Molar specific heat according to definition is given by
dQ nCVdT + pdV
C = =
ndT ndT
nCV dT + (nRT>V )dV RT dV
= = CV +
ndT V dT
We have T = T0e aV
On differentiating, dT = aT0 e aV # dV
dV 1
So, =
dt a T0e aV
RT # 1 RT0e aV R
Hence, C = CV + aV
= CV + aV
= CV +
V aT0e aVT0e aV
(b) For the process, p = p0e aV.
So, p = RT>V = p0 e aV
or T = (p0>R)e aV # V

= CV + p0e aV #
RT dV R R
So, C = CV + = CV +
V dT p0e (1 + aV)
aV 1 + aV
2.2 THE FIRST LAW OF THERMODYNAMICS. HEAT CAPACITY 251

2.53 (a) Using solution of Problem 2.52,


RT dV pdV
C = CV + = CV + (for one mole of gas)
V dT dT
a RT a
We have, p = p0 + or = p0 + Q RT = p0V + a
V¿ V V¿
Therefore, RdT = p0dV
So, dV>dT = R>p0

C = CV + a p0 + b + a1 + b
a # R R a
Hence, =
V p0 g - 1 p0V
Rg aR
R = +
g - 1 p0V
(b) Work done is given by
V2
V2
a p0 + b dV = p0(V2 - V1) + a ln
a
A =
3 V V1
V1
p2V2 p1V1
¢U = CV (T2 - T1) = CV a - b (for one mole)
R R
R
= (p V - p1V1)
(g - 1)R 2 2
p0(V2 - V1)
c (p0 + aV2) V2 - a p0 + b V1 d =
1 a
=
g - 1 V1 g - 1

By the first law of thermodynamics,


Q = ¢U + A
p01V2 - V12 V2
= + p0(V2 - V1) + a ln
(g - 1) V1
gp0 (V2 - V1) V2
= + a ln
(g - 1) V1

2.54 (a) From solution of Problem 2.52, heat capacity is given by


RT dV
C = CV +
V dT
T T0
We have T  T0
V Q V  -
a a
252 PART TWO THERMODYNAMICS AND MOLECULAR PHYSICS

On differentiating, we get
dV 1
=
dT a

RT # 1 R (T0 + aV ) 1
Hence, C = CV + =
R
+ #
V a g - 1 V a
T0 RT0 RT0
+ R a + 1b =
R gR RT
= + = CV + = Cp +
g - 1 aV g - 1 aV aV aV

(b) Given that T  T0


V. and as T = pV>R for one mole of an ideal gas, so
R RT0
p = (T0 + aV ) = + aR
V V
V2 V2
RT0
Now, A = pdV = a + aR b dV (for one mole)
3 3 V
V1 V1
V2
= RT0 ln + a(V2 – V1)R
V1
Also, ¢U = CV (T2 - T1)
= CV [T0 + aV2 – (T0 + aV1)] = aCV (V2 - V1)

By the first law of thermodynamics


Q = ¢U + A
aR V2
= (V2 – V1) + RT0 ln + aR (V2 – V1)
g - 1 V1
V2
= aR (V2 – V1) c 1 + d + RT0 ln
1
g - 1 V1
V2
= aCp (V2 – V1) + RT0 ln
V1

2.55 Heat capacity is given by


RT dV
C = CV +
V dT
(a) Given that
C = CV + aT
2.2 THE FIRST LAW OF THERMODYNAMICS. HEAT CAPACITY 253

RT dV a dV
So, CV + aT = CV + or dT =
V dT R V
Integrating both sides, we get
a
T = ln V + ln C0 = ln VC 0 (where C0 is a constant)
R
or V # C0 = e -aT>R

V # e -aT>R =
1
Hence, = constant
C0

(b) According to the problem C = CV + bV .


RT dV
But, C = CV +
V dT
RT dV
So, = bV
V dT
R -2 dT
or V dV =
b T
Integrating both sides, we get

- V - 1 = ln T + ln C0 = ln T # C 0 (where C0 is a constant)
R
b

lnT # C0 = -
R
So,
bV
or T # C0 = e - R/bV
1
Hence, Te R/bV = = constant
C0

RT dV
(c) Given that C  CV ap and C  CV
V dT
RT dV RT dV
So, CV + ap = CV + Q ap =
V dT V dT
RT RT dV RT
or a = (as p = for one mole of gas)
V V dT V
or dV>dT = a Q dV  adT or dT = dV>a
V
So, T = + constant or V  aT  constant
a
254 PART TWO THERMODYNAMICS AND MOLECULAR PHYSICS

2.56 (a) By the first law of thermodynamics


A = Q - ¢U
or A  CdT  C V dT  (C  C V ) dT (for one mole)
Given that C = a>T
hT0
hT0
a
- CV b dT = a ln
a
So, A = - CV (hT0 - T0)
3 T T0
T0

= a ln h - CVT0 (h - 1)
RT0
= a ln h - (h - 1)
g - 1

dQ RT dV
(b) Given that C = + = + CV
dT V dT
and C = a>T

RT dV a
So, CV + =
V dT T
R 1 dV a
or + = dT
g - 1 RT V RT 2
dV a 1 # dT
or = dT -
V RT 2 g - 1 T
dV a(g - 1) dT
or (g - 1) = 2
dT -
V RT T

Integrating both sides, we get

a(g - 1)
(g - 1) lnV = - - ln T + ln K
RT
- a(g - 1)
ln c V g - 1 # d =
T
or
K RT
- a(g - 1)
d =
pV
lnc V g - 1 #
RK pV

or pV g>RK = e -a(g - 1)/pV


or pV ge a(g - 1)/pV = RK = constant
2.2 THE FIRST LAW OF THERMODYNAMICS. HEAT CAPACITY 255

2.57 The work done is


V2 V2

a - 2 b dV
RT a
A = p dV =
3 3 V - b V
V1 V1

V2 - b
+ aa b
1 1
= RT ln -
V1 - b V2 V1

2.58 (a) The increment in the internal energy is


V2
0U
¢U = a
b dV
3 0V T
V1

But from the second law of thermodynamics


0U 0s 0p
a b = T a b - p = Ta b - p
0V T 0V T 0T V
On the other hand
RT a
p = - 2
V -b V
0p 0U
Ta bV = a b =
RT a
or and
0T V - b 0V T V 2

¢U = a a b = 0.11 kJ (on substituting values)


1 1
So, -
V1 V2
(b) From the first law of thermodynamics
V2 - b
Q = A + ¢U = RT ln = 3.8 kJ (on substituting values)
V1 - b

2.59 (a) From the first law, for an adiabatic process, dQ = dU + pdV = 0.
From the previous problem
0U 0U
dU = a b dT + a b dV = CV dT + 2 dV
a
0T V 0V T V
RTdV
So, 0 = CV dT +
V - b
This equation can be integrated if we assume that CV and b are constant, then
R dV dT
+ = 0
CV V - b T
R
or ln T + ln (V – b) = constant or T (V – b)R>CV = constant
CV
256 PART TWO THERMODYNAMICS AND MOLECULAR PHYSICS

a
(b) We use dU = CV dT + dV
V2
RT
Now, dQ = CV dT + dV
V - b
0V
a b
RT
So along constant p, Cp = CV +
V - b 0T p

0V
a b
RT
Thus, Cp - CV =
V - b 0T p
RT a
But, p =
V - b V2

0V
0 = a- + 2b a b +
RT 2a R
On differentiating,
(V - b)2 V 0T P V - b

0V RT>V - b V - b
or T a b = =
0T P RT 2a 2a (V - b)2
- 3 1 -
(V - b)2 V RTV 3

R
and Cp - CV =
2a(V - b)2
1 -
RT V 3

2.60 From the first law

Q  Uf  Ui A  0 (as the vessels are thermally insulated)

As this is free expansion, A  0, so, Uf  Ui.

an2
But, U = nCVT -
V
- aV2 n
CV (Tf - Ti ) = a - bn =
a a
So,
V1 + V2 V1 V1(V1 + V2)

- a(g - 1)V2 n
or ¢T =
RV1(V1 + V2)
On substitution we get, T  3 K.

2.61 From the first law

Q  Uf  Ui A  Uf  Ui (as A  0 in free expansion).


2.3 KINETIC THEORY OF GASES. BOLTZMANN’S LAW AND MAXWELL’S DISTRIBUTION 257

So at constant temperature,
- an2 V2 - V1
- a- b = an 2
an2
Q =
V2 V1 V1 # V2
 0.33 kJ (on substituting values)

2.3 Kinetic Theory of Gases. Boltzmann’s Law and Maxwell’s


Distribution

2.62 From the formula p = nkT


p 4 * 10-15 * 1.01 * 105 - 3
n = = m
kT 1.38 * 1023 * 300
= 1 * 1011 per m3 = 105 cm - 3
Mean distance between molecules
110-5 cm321>3 = 101>3 * 10-2 cm = 0.2 mm

2.63 After dissociation each N2 molecule becomes two N-atoms and so contributes, 2 * 3
degrees of freedom. Thus the number of moles becomes

11 + h2
m
M

11 + h2 = 1.9 atm (on substituting values)


mRT
and p =
MV
(Here M is the molecular weight, in grams of N2.)

2.64 Let n1 = number density of He atoms, n2 = number density of N2 molecules.


Then, r = n1m1 + n2m2 (1)
(where m1 and m2 are masses of helium and nitrogen molecules).

Also, p = 1n1 + n22 kT (2)


From Eqs. (1) and (2), we get

a b
pr
-
kT m2
n1 = = 1.6 * 10-19 cm-3
m1
a1 - b
m2
258 PART TWO THERMODYNAMICS AND MOLECULAR PHYSICS

2.65 Number of nitrogen molecules coming to strike the differential area ds


of the wall per unit time is given by
n
= n | 1v # dS2 | = 1nv dS cosu2
dN
dt
where n = N>V = number density or concentration of N2 molecules in the beam.
The change in linear momentum of each of N2 molecule is
¢pmolecule = 2mv cosu n (see figure)
So ¢p for all the molecules striking the wall per unit time, i.e., the force exerted by
the wall on incoming N2 molecules is
1nvds cosu)¢pmolecule = 2nv 2 cos2udS n
Therefore from Newton’s law, the force exerted by N2 molecules on the wall is
2nmv 2 cos2u dS 1- n2
Thus the sought pressure i.e. normal force per unit differential is
dFnormal
= 2nmv 2cos2 u
ds

2.66 From the formula


gp rv 2
v = or g =
Ar p
If i = number of degrees of freedom of the gas, then
i
Cp = CV + RT and CV = RT
2
Cp 2 2
g = = 1 + or i =
CV i g - 1

= 5 1on substituting values2


2
=
( v >p) - 1
r 2

gp gRT 3kT 3RT


2.67 vsound = = and vrms = =
Ar A M A m A M
vsound g i + 2
So, = =
vrms A3 A 3i

(a) For monoatomic gases i = 3, therefore


vsound 5
= = 0.75
vrms A9
2.3 KINETIC THEORY OF GASES. BOLTZMANN’S LAW AND MAXWELL’S DISTRIBUTION 259

(b) For rigid diatomic molecules i = 5, therefore


vsound 7
= = 0.68
vrms A 15

2.68 For a general non-collinear, non-planar molecule, mean energy

kT (translational) + kT (rotational) + 13N – 62kT (vibrational)


3 3
=
2 2
= 13N – 32kT (per molecule)

For linear molecules, mean energy

3
= kT (translational) + kT (rotational) + (3N – 5)kT (vibrational)
2

= a 3N - b kT (per molecule)
5
2
1 1
Translational energy corresponds to fraction and in non-linear and
linear molecules, respectively. 2(N - 1) 2N - 5>3

2.69 (a) A diatomic molecule has 3 translational, 2 rotational and one vibrational degrees
of freedom. The corresponding energy per mole is
3 1
RT (for translational) + 2 * RT (for rotational) + 1 * RT (for vibrational)
2 2
7
= RT
2
7 Cp 9
Thus, CV = R and g = =
2 CV 7
(b) For linear N-atomic molecules, energy per mole

= a 3N - b RT (as before)
5
2
6N - 3
CV = a 3N - b R and g =
5
So,
2 6N - 5
(c) For non-collinear N-atomic molecules
CV = 31N - 12 R (as before from Problem 2.68)
3N - 2 N - 2>3
and g = =
3N - 3 N - 1
260 PART TWO THERMODYNAMICS AND MOLECULAR PHYSICS

2.70 In the isobaric process, work done is A = pdV = RdT (per mole).
On the other hand heat transferred is Q = C p dT .
Now Cp = (3N - 2) R for non-collinear molecules and C p = [3N - (3>2)] R for linear
molecules.
1
non-collinear
A 3N - 2
Thus, = d
Q 1
linear
3N - (3>2)
For monoatomic gases, Cp = 5>2 and A>Q = 2>5

2.71 Given specific heats cp , cV (per unit mass), then

M1cp - cV2 = R
R
or M = = 32 g/mol (on substituting values)
cp - cV
cp 2
Also, g = = + 1
cV i
2 2cV
i = = = 5 (on substituting values)
(cp >cV ) - 1
or
cp - cV

29
2.72 (a) Given, Cp = 29 J>K mol = R
8.3
20.7 29 7
CV = R, g = = 1.4 =
8.3 20.7 5
Cp
i = 2a - 1b = 5
R
(b) In the process, pT = constant and (T 2>V ) = constant.
dT dV
So, 2 - = 0
T V
RT 2RT
Thus, CdT = CV dT + pdV = CV dT + dV = CV dT + dT
V T

C = CV + 2R = a bR Q CV =
29 12.4 3
or R = R
8.3 8.3 2

d = 3 (monoatomic)
C 1
i = 2c +
1n - 12
Hence,
R
(where n = 1>2 is the polytropic index).
2.3 KINETIC THEORY OF GASES. BOLTZMANN’S LAW AND MAXWELL’S DISTRIBUTION 261

2.73 We know that,


1 3 5
C = n1 + n2
R V 2 2
(since a monoatomic gas has CV = (3>2)R and a diatomic gas has CV = (5>2)R )
1 3 5
and C = n1 + n2 + n1 + n2
R p 2 2
Cp 5 n1 + 7 n2
Hence, g = =
CV 3 n1 + 5 n2

2.74 The internal energy of the molecules is

mN 6 1u - v22 7 = mN 6 u2 - v 2 7
1 1
U =
2 2
(where v = velocity of the vessel, N = number of molecules, each of mass m).
When the vessel is stopped, internal energy becomes (1>2)mN 6 u2 7.
So, there is an increase in internal energy of ¢U = (1>2)mNv 2 . This will lead to a rise in
temperature of
(1>2)mNv 2
¢T =
(i>2)R
mNv 2
=
iR
Since there is no flow of heat, this change of temperature will lead to an excess pr-
essure, given by
R¢T mNv 2
¢p = =
V iV
¢p Mv 2
and finally = = 2.2%
p iRT
(where M = molecular weight of N2, i = number of degrees of freedom of N2 = 5).

2.75 (a) From the equipartition theorem


3 3kT 3RT
e = kT = 6 * 10-21 J and vrms = = = 0.47 km/s
2 A m A M
(b) In equilibrium, the mean kinetic energy of the droplet will be equal to that of a
molecule.

1p 3 2 3 2kT
So, d rvrms = kT or vrms = 3 = 0.15 m/s
26 2 A pd 3r
262 PART TWO THERMODYNAMICS AND MOLECULAR PHYSICS

2.76 Given that, i = 5, CV = 5>2 R, g = 7>5.

3 RT 1 1 3RT 1
So, v ¿rms = = vrms = or T ¿ = T
A M h hA M h2
Now, in an adiabatic process

TV g-1 = TV 2>i = constant or VT i>2 = constant

V¿a
1 i>2
So, Tb = VT i>2 or V ¿h-i = V or V ¿ = hiV
h2
The gas must be expanded hi times, i.e., 7.6 times.

R 1as i = 52
5 m
2.77 Here, CV =
2 M
(where m = mass of the gas, M = molecular weight of the gas).
If vrms increases h times, the temperature will have increased h2 times. This will require
(neglecting expansion of the vessels), a heat flow of amount

R 1h2 - 12 T = 10 kJ
5 m
2 M

2.78 The root mean square angular velocity is given by

I v = 2 * kT 1for 2 degrees of rotations2


1 2 1
2 2
2kT
or v = = 6.3 * 1012 rad/s
A I

2.79 Under compression, the temperature will rise, as

TV g-1 = constant or TV 2>i = constant


So, T ¿1h-1 V022>i = T0V 2>i
0 or T ¿ = h+2>i T0

So mean kinetic energy of rotation per molecule in the compressed state is given by

kT ¿ = kT0h2>i = 0.72 * 10-20 J


2.80 Number of collisions is given by
1
n 6v 7 = v
4
v¿ n ¿ 6v ¿ 7 1 T¿
Now, = =
v n 6v 7 hA T
2.3 KINETIC THEORY OF GASES. BOLTZMANN’S LAW AND MAXWELL’S DISTRIBUTION 263

(When the gas is expanded h times, n decreases by a factor h.)


Also T ¿1hV 22>i = TV 2>i or T ¿ = h2>i T
v¿ 1
So, = h-1>i = h-(i + 1)>i
v h

i.e., collisions decrease by a factor h(i + 1)>i (where i = 5 h).

2.81 In a polytropic process pV n = constant, where n is the polytropic index. For this
process
pV n = constant or TV n-1 = constant

+ 1n - 12
So, dT dV
= 0
T V
Then, dQ = CdT = dU + pdV = CV dT + pdV

RdT = a - b RdT
i RT i 1 i 1
= RdT + dV = RdT -
2 V 2 n - 1 2 n - 1
i 1
Now, C = R so - = 1
2 n - 1
1 i i - 2 i
or = - 1 = or n =
n - 1 2 2 i - 2
n-1
v¿ n ¿ 6v ¿ 7 1 T¿
= a b
1 V 2
Now, = =
v n 6v 7 hA T h V¿
1 1 1>(i - 2) 1 (i - 1)>(i - 2)
= a b = a b
h h h

= 1h2-(i -1)>(i -2) times =


1
times
2.52

2.82 If a is the polytropic index, then


pV a = constant, TV a-1 = constant
v¿ n ¿ 6v ¿ 7 V T¿ V T -1>2
Now, = = = = 1
v n 6v 7 V¿ A T V ¿T ¿ -1>2
1 1
Hence, = - or a = -1
a - 1 2
iR R
Then, C = + = 3R
2 2
264 PART TWO THERMODYNAMICS AND MOLECULAR PHYSICS

2kT 2RT 2p
2.83 vp = = = = 0.45 km/s
A m A M Ar
8p 3p
vavg = = 0.51 km>s and vrms = = 0.55 km>s
Ap r Ar

2.84 (a) The formula is

df 1u2 = u2e-u du 1where u = v>vp2


4 2

2p
1 + dh
v - vp
Now, Prob ¢ ` ` 6 dh≤ = df 1u2
vp 3
1 - dh

4 8
= e-1 * 2 dh =
dh = 0.0166 = 1.66%
2p 2pe
v - vrms vrms vrms
(b) Prob ¢ ` ` 6 dh≤ = Prob ¢ ` ` 6 dh
v
- ≤
vrms vp vp vp

= Prob ¢ ` u - ` 6
3 3
dh≤
A2 A2
3 3
+ dh
A2 A2

4 2 -u2
= u e du
3 p
3 3
- dh
A2 A2
4 3 -3>2 3
= * e * 2 dh
2p 2 A2

12 23
= e - 3>2dh = 0.0185 = 1.85%
22p

2.85 (a) vrms - vp = A 23 - 22 B


kT
= ¢v
Am
2
m ¢v
T = ¢ ≤ = 384 K
k 113 - 122
(b) Clearly v is the most probable speed at this temperature.
2kT mv 2
So, = v or T = = 342 K
A m 2k
2.3 KINETIC THEORY OF GASES. BOLTZMANN’S LAW AND MAXWELL’S DISTRIBUTION 265

2.86 (a) We have,


v 21 v 22 v1 v 21 - v 22
e-v 2 >v p Q a b = e v1
2 2kT
e-v 1 >v p = - v 22 >v p2 or v 2p = =
2 2 2 2 2

vp2 vp 2 v2 m (ln v 21>v 22)

m1v 21 - v 222
T = = 330 K
2 k ln v 21>v 22
So,

(b) F 1v2 = a comes from F 1v2dv = df 1u2, du = b


4 v 2 -v 2>v 2 1 1 dv
e p *
2 vp vp vp
2p v p

v 2 -v 2>v 2p v2 2kT0 2kT0


Thus, e 1 = = , v 2 = h
2 p
v 3p vp m 2 m
1 2

A B = a 1 - b = ln h
mv 2
- 2kT 1 - h1
1 mv 2 1 3
Now, e 0 or
h3>2 2kT0 h 2
3kT0 ln h
Thus, v =
B m A 1 - 1>h

2kT 2RT 2RT


2.87 vp = = ,v =
N A mN A MN p O A MO

MN
a1 - b
2RT
vp - vp = ¢v =
N o A MN B MO
MN 1¢v2 2 mN 1¢v22
T = = = 363 K
MN mN
2R a 1 - b 2k a 1 - b
2 2

A MO A mO

mHe
v2 )= a b
3>2
v 2 -v 2>v 2
= 3 e-v >vpHe or e v ( 2kT
2 2 2 m He m
- 2kTH
2.88 3
e p
H
vp vp mH
H He

ln mHe >mH
v 2 = 3kT
mHe - mH
On substitution, we get, v = 1.60 km/s.

N 4 v 2dv -v 2>v 2
2.89 dN (v) = e p
1p v 3p
H

For a given range v to v + dv , (dN (v)) is maximum when

a be
d dN (v) -4 +
2v 2 -v 2>v 2
= 0 = - 3v p
p
dvp Nv 2dv v 6p
266 PART TWO THERMODYNAMICS AND MOLECULAR PHYSICS

3 2 3kT
or v2 = vp =
2 m
1 mv 2
Thus, T =
3 k

2.90 Given d 3v = 2pv ⬜ dv ⬜ dvx

dn 1v2 = N a b e 2kT x ⬜ 2pv ⬜ dv ⬜ dvx


m 3>2 - m 1v 2 + v 2 2
Thus,
2pkT

2.91 6vx 7 = 0 (by symmetry)


q q

| vx | e -mv x >2kT vxe -mv x >2kT dvx


2 2
dvx
3 3
0 0
6| vx | 7 = q = q

e -mv 2x >2kT dvx e -mv x >2kT dvx


2

3 3
0 0
q q
2kT 2kT 1 -x
ue-u du
2
e dx
A m 3 A m 32
0 0
= q = q
dx
e-u du e -x
2

3 3 2 1x
0 0

2kT
≠(1)
B m 2kT
= =
B mp
≠a b
1
2
q q
2kT dx
vx2 e-mv x >2kT dvx xe-x
≠a b
2
2kT 3
3 m 3 2 2x
0 0 m 2 kT
2.92 6v 2x 7 = q = q = =
m
≠a b
1
dx
e-mv x >2kT dvx e-x
2
2
3 3 2 2x
0 0
2.93 Here vdA is equal to number of molecules hitting an area dA of the wall per second
q

i.e., vdA = dN 1vx2 vxdA


3
0
2.3 KINETIC THEORY OF GASES. BOLTZMANN’S LAW AND MAXWELL’S DISTRIBUTION 267

q
dA
b e x
m 1>2 -mv 2 >2kT
or v = na vxdvx
3 2pkT
0
q

a
b e udu
2kT 1>2 -u2
n
=
3 2p m
0

= n 6 v 7 a where 6 v 7 = b
1 2kT kT 1 8kT
= n = n
2 A mp A 2mp 4 A mp

2.94 Let the number of molecules per unit volume with x component of velocity in the
range vx to vx + dvx , be

dn1vx2 = na b e
m 1>2 - mv 2 /2kT
x dvx
2pkT
q

Then, p = 2mvx # vx dn 1vx2


3
0
q

2mv x2 n a b e x
m 1>2 -mv 2 >2kT
= dvx
3 2pkT
0
q
2kT 1
= 2mn u2e-u du
2

2p m 3
0
q
4 dx
= nkT xe -x = nkT
2p 3 2 2x
0
q

a b e x
m 3>2 -mv 2 >2kT 1
2.95 61>v7 = 4pv 2dv
3 2pkT v
0
q

= a b 4p
m 3>2 1 2kT
e - xdx
2pkT 2 m 3
0

b = a b = a 2 b =
m 1>2 2m 1>2 16 mp 1>2 4
= 2a
2pkT pkT p 8kT p6 v7
dN 1e2
2.96 dN 1v2 = N a b e
m 3>2 -mv 2>2kT
4pv 2dv = dN (e) = de
2pkT de
dN 1e2
= Na b e
m 3>2 -mv 2>2RT dv
or 4pv 2
de 2pkT de
268 PART TWO THERMODYNAMICS AND MOLECULAR PHYSICS

1 dv 1
Now, e =mv 2 so =
2 de mv
dN 1e2
= Na b e
m 3>2 -e>kT 2e 1
or 4p
de 2pkT Am m

1kT 2-3>2 e-e>kT e1>2


2
= N
2p
dN 1e2 = N 1kT 2-3>2 e -e>kT e1>2 de
2
i.e.,
2p
The most probable kinetic energy is given by

d dN 1e2
= 0
de de
1 − 1>2 - e>kT e1>2 e>kT 1
or e e − e = 0 or e = kT = ep
2 kT 2
kT
The corresponding velocity is v = Z vp
Am
2.97 The mean kinetic energy is
q

e3>2 e-e>kTde
3 ≠15>22
0 3
6e 7 = q = kT = kT
≠13>22 2
e1>2 e-e>kTde
3
0
Thus,
2 11 + dh2kT
3

1kT 2-3>2 e - e>kTe1>2 de


dN 2
=
N 3 2p
2 kT 11 - dh2
3

e-3>2 a b 2dh = 3
2 3 3>2 6 -3>2
= e dh
2p 2 Ap
If dh = 1%, the fraction of molecules is 0.9%.
q
¢N
1kT 2-3>2 1e e-e>kT de
2
2.98 =
N 2p 3
e0
q

1kT 2-3>2 1e0 e-e>kTde 1e0 W kT 2


2
=
2p 3
e0
e0
1kT 2-3>2 1e0kT e-e0 >kT = 2
2
= e-e0 >k T
2p A pkT
2.3 KINETIC THEORY OF GASES. BOLTZMANN’S LAW AND MAXWELL’S DISTRIBUTION 269

(In evaluating the integral, we have taken out 1e as 1e0 since the integral is domi-
nated by the lower limit.)

2.99 (a) F 1v2 = Av 3e-mv >2k T


2

For the most probable value of the velocity


dF 1v2 2mv -mv 2>2kT
= 0 or 3 Av 2e -mv >2kT - Av 3 = 0
2
e
dv 2kT
3kT
So, vp =
A m
2kT
This should be compared with the value vp = for the Maxwellian distribution.
A m
1 2
(b) In terms of energy, e = mv
2
F 1e2 = Av 3 e-mv >2kT
2 dv
de
= Aa b e
2e 3>2 -e>kT 1 2e
= A 2 e-e>kT
m 12me m
From this the probable energy comes out as follows:

F 1e2 = 0 Q ae b = 0 or ep = kT
2A -e>kT e -e>kT
- e
m2 kT

2.100 The number of molecules reaching a unit area of wall at angle between u and u + du
to its normal per unit time is
v=q

dv = dn1v2 v cos u
3 4p
v=0 v
q

na b e
m 3>2 -mv 2>2kT 3
= v dv sin u cos u du * 2p
3 2pkT
0
q

= na b e-x xdx sin u cos u du = n a b sin u cos u du


2kT 1>2 2kT 1>2
mp 3 mp
0

2.101 The number of molecules reaching per unit area of the wall with velocities in the
interval v to v + dv per unit time is
u = p>2

dv = dn1v2 v cosu
3 4p
u=0
270 PART TWO THERMODYNAMICS AND MOLECULAR PHYSICS

u = p>2

b e
m 3>2 -mv 2>2kT 3
= na v dv sin u cos u du * 2p
3 2pkT
u=0

= np a b e
m 3>2 -mv 2>2kT 3
v dv
2pkT

2.102 If the force exerted is F, then by the law of variation of concentration with height
n 1Z2 = n0 e-FZ>kT

kT ln h
So, h = eF¢h>kT or F = = 9 * 10-20 N
¢h
p 3 RT lnh 6RT lnh
2.103 Here, F = d ¢rg = or Na =
6 Nah pd 3g¢rh
Given T = 290 K, h = 2, h = 4 * 10-5 m, d = 4 * 10-7m, g = 9.8 m>s2,
¢r = 0.2 * 103 kg>m3 and R = 8.31 J>K.

6 * 8.31 * 290 * ln2


Hence, Na = * 1026 = 6.36 * 1023 mol-1
p * 64 * 9.8 * 200 * 4

concentration of H2 e -MN2gh>RT
2.104 h = = h0 = h0e (-MN2 + MH2)gh>RT
concentration of N2 e -MH2gh>RT
h
= e (-MN2 + MH2)gh>RT = 1.39
h0
2.105 We know that

n1(h) = n1e-m1gh>kt and n21h2 = n2e-m2gh> kt


They are equal at a height h, where n1 >n2 = e gh (m1 - m2)>kT
kT ln n1 - ln n2
or h =
g m1 - m2

2.106 At a temperature T the concentration n1z- 2 varies with height according to


n1z- 2 = n0e-mgz>kT
This means that particles per unit area of the base of cylinder are
q

- 2 d z-
n1z
3
0
2.3 KINETIC THEORY OF GASES. BOLTZMANN’S LAW AND MAXWELL’S DISTRIBUTION 271

q
n0kT
= n0e-mgz>kT dz- =
3 mg
0

Clearly this cannot change. Thus n0kT = p0 = pressure at the bottom of the cylin-
der, must not change with change of temperature.
q q

mgz e-mgz>ktdz xe-xdx


3 3
0 0 ≠(2)
2.107 6U 7 = q = kT q = kT = kT
≠(1)
e-mgz>kTdz e-xdx
3 3
0 0
When there are many kinds of molecules, this formula holds for each kind and the
average energy is given by
a FikT
6U 7 = = kT
a fi
where fi r fractional concentration of each kind of molecule at the ground level.

2.108 The constant acceleration is equivalent to a pseudo force wherein a concentration


gradient is set up. Then
e -MAwl>RT = 1 - h

RT ln11 - h2 hRT
or w = - L L 70 g
MAl MAl

2.109 In a centrifuge rotating with angular velocity ␻ about an axis, there is a centrifugal
acceleration v2r , where r is the radial distance from the axis. In a fluid if there are sus-
pended colloidal particles they experience an additional force. If m is the mass of
each particle then its volume is m/r and the excess force on this particle is
m>r 1r - r02v2r outward, corresponding to a potential energy of - (m>2r) ( r - r0 ) v2r 2.

This gives rise to a concentration variation, given by

n1r2 = n0 exp a + 1r - r02 ␻2r 2 b


m
2 r kT

n1r22
= h = exp a + 1r - r02␻21r 22 - r 212b
M
n1r12
Thus,
2 r RT

m M
(where = and M = NAm is the molecular weight).
k R
272 PART TWO THERMODYNAMICS AND MOLECULAR PHYSICS

2 r RT ln h
M =
1r - r02␻21r 22 - r 212
Thus,

2.110 The potential energy associated with each molecule is - (1>2 )m v2r 2 and there is a
concentration variation, given by

n 1r2 = n0 exp a b = n0 exp a b


m␻2r 2 M␻2r 2
2kT 2RT

h = exp a b
Mv2l 2 2RT
Thus, or v = ln h
2RT A Ml 2
Using M = 12 + 32 = 44 g, l = 100 cm, R = 8.31 * 107 erg/K, T = 300 K, we get,
␻ = 280 rad/s
2.111 Here, the concentration variation is
n1r2 = n0exp a - b
ar 2
kT
(a) The number of molecules located at the distance between r and r + dr is

4 pr 2dr n1r2 = 4pn0exp a - b r dr


ar 2 2
kT
r n 1r2 = 0
d 2 2ar 3 kT
(b) rp is given by or 2r - = 0 or rp =
dr kT A a
(c) The fraction of molecules lying between r and r + dr is

dN 4pr 2dr n0 exp 1 - ar 2>kT 2


= q
N
4pr 2drn0 exp 1ar 2>kT 2
3
0
q q

4pr 2dr exp a - b = a b 4p x exp 1 - x2


ar 2 kT 3>2 dx
3 kT a 3 2 2x
0 0

= a b 2p≠ a b = a b
kT 3>2 3 pkT 3>2
a 2 a
- ar 2
= a b 4pr 2dr exp a b
dN a 3>2
Thus,
N pkT kT
- ar 2
dN = N a b 4pr 2dr exp a b
a 3>2
(d)
pkT kT
- ar 2
n1r2 = N a b exp a b
a 1>2
So,
pkT kT
When T decreases h times, n102 = n0 will increase h3>2 times.
2.4 THE SECOND LAW OF THERMODYNAMICS. ENTROPY 273

U
2.112 (a) Let U = ar 2 or r =
Aa
1 dU dU
So, dr = =
A a 2 1U 2 1aU

exp a b
U dU U
So, dN = n04p
a 2 1aU kT

= 2pn0 a-3>2 U 1>2 exp a b dU


U
kT
(b) The most probable value of U is given by
-U
a b = 0 a b exp a b
d dN 1 U 1>2 1
- or Up = kT
dU dU 2 1U kT kT 2

From 2.111 (b), the potential energy at the most probable distance is kT.

2.4 The Second Law of Thermodynamics. Entropy

2.113 The efficiency is given by


T1 - T2
h = (for T1 7 T2)
T1
Now, in the two cases the efficiencies are
T1 + ¢T - T2
hh = (T1 increased)
T1 + ¢T
T1 - T2 + ¢T
hl = (T2 decreased)
T1
Thus, hh 6 n1
p1V1
Q1
2.114 (a) For H2, g = 7>5. From the figure
p2V2
p1V1 = p2V2, p3V3 = p4V4 adiabatic
and p2V2g = p3V3g, p1V1g = p4V4g p4,V4
p3V3
Q'2
Define n by V3 = n V2.
Then p3 = p2n-g
So, p4V4 = p3V3 = p2V2n1-g = p1V1n1-g
since, p4V4g = p1V1g so, V41-g = V11-g n1–g or V4 = nV1
274 PART TWO THERMODYNAMICS AND MOLECULAR PHYSICS

V2 V3 V3
Also, Q1 = p2V2 ln , Q 2¿ = p3V3 ln n 1 - g = p2V2 ln
V1 V4 V4
Q ¿2
Finally h = 1 - = 1 - n1 - g = 0.242
Q1

(b) Define n by p3 = p2>n

p2
Then, p2V g2 = V3 g or V3 = n1/gV2
n
So we get the formulae here by substituting n by n1/g in the previous case.
Then, h = 1 - n(1/g)-1 = 1 - n-(2/7) = 0.18

2.115 Used as a refrigerator, the refrigerating efficiency of a heat engine is Q1


given by
Q¿2 Q¿2 Q¿2 >Q1 1 - h A=Q1–Q'2
e = = = = = 9
A Q1 - Q¿ 2 Q ¿2 h Q'2
1 -
Q1
(where h is the efficiency of the heat engine).

2.116 Given V2 = nV1,V4 = nV3


Q1 = Heat taken at the upper temperature
= RT1 ln n + RT2 ln n = R (T1 + T2) ln n
p1,V1 T1 p2,V2
Now, T1V g-1
2 = T2V g-1
3
T2 p4,V4
p3,V3
T1
V3 = a b
1>g-1
or V2 p6,V6 p5,V5
T2 T3

T2 1>g - 1
Similarly, V5 = a b V4,
T3
T1 1>g - 1
V6 = a b V1
T3
Thus, heat ejected at the lower temperature
V6
Q2 = -RT3 ln
V5
T1 1>g - 1 V T1 1>g - 1 V2
= –RT3 ln a b = - RT3 in a b
1
T2 V4 T2 n2V3
T1 1>g - 1 1 T1 -1>g - 1
= –RT3 ln a b a b = 2RT3 ln n
T2 n2 T2
2.4 THE SECOND LAW OF THERMODYNAMICS. ENTROPY 275

2T3
Thus, h = 1-
T1 + T2
CV
Q ¿2 = CV 1T2 - T32 = V2 1p2 - p32
p1,V1
2.117 Since, p2,V2
R
Q1
CV Q'2
V11p1 - p42
p4,V1
and Q1 = p3,V2
R
1V2 p2 - p32
h = 1-
V1 1p1 - p42
So,

On the other hand,


p1V g1 = p2V g2, p3V g2 = p4V g1 also , V2 = nV1

Thus, p1 = p2ng and p4 = p3ng


So, h = 1 - n1-g (where g = 7>5 for N2 )
Therefore, h = 0.602 = 60.2%

Cp
2.118 Since, Q1 = p1 1V2 - V12 Q1
R p1,V1 p1,V2

p2 1V3 - V42
Cp
and Q 2¿ =
R
p2 1V3 - V42 p2,V4 Q'2
p3,V2
h = 1 -
p11V2 - V12
So,

Now, p1 = np2 or V3 = n1/gV2

p2V 4g = p1V1g or V4 = n1>gV1


1 # 1>g
So, h= 1 - n = 1 - n(1>g) - 1
n
2.119 Since the absolute temperature of the gas rises n times both in the T1 nT1
Q11
isochoric heating and the isobaric expansion, we have p1,V1 p1,V2

p1 = np2 and V2 = nV1 Q12 Q'21

Heat taken is Q1 = Q11 + Q12 p2,V1 p2,V2


Q'22
Q11 = CP 1n - 12T1
(T1/n) T1
where,

Q12 = CVT1 a 1 - b
1
and
n
Heat rejected is Q¿2 = Q¿21 + Q¿22
Q¿21 = CVT1 (n - 1) Q¿22 = Cp T1 a1 - b
1
where,
n
276 PART TWO THERMODYNAMICS AND MOLECULAR PHYSICS

Q ¿2 CV (n - 1) + Cp (1 - 1>n)
Thus, h = 1 - = 1 -
Q1 Cp(n - 1) + CV (1 - 1>n)
g
1 +
h - 1 + g(1 - 1>n) n + g
= a1 - b = 1 -
n
= 1 -
g(n - 1) + (1 - 1>n) 1 1 + ng
g +
n

2.120 (a) Here, p2 = np1, p1V1 = p0V0, Temp nT0

and np1V1g = p0V g0 Q1 adiabatic


p1,V1 p0,V0
V0
Also, Q¿2 = RT0 ln , Q1 = CVT0 (n - 1) isothermal
V1 Q'2 Temp T
0

But, nV1g - 1 = V0g - 1 or V1 = V0n-1>g - 1


RT0
Q¿2 = RT0 ln n-1>g - 1 ln n
g - 1

aon using CV = b
ln n R
Thus, h =1 -
n - 1 g - 1
(b) Here, V2 = nV1, p1V1 = p0V0

and p1(n V1)g = p0V0g Q1


p1,V2, nT0
i.e., ngV1g - 1 = V1g - 1 or V1 = ng>g - 1 V0 p1,V1
T0 adiabatic
Also, Q1 = C pT0 (n - 1), Q'2
isothermal p0,V0
V0 T0
Q 2¿ = RT0 ln
V1
Rg
or Q 2¿ = RT0 ln ng>(g - 1) = T ln n = C pT0 ln n
g - 1 0
ln n
Thus, h = 1 -
n - 1

2.121 Here the isothermal process proceeds at the maximum p0,V0 isothermal
T0 Q1
temperature instead of at the minimum temperature of p1,V1,T0
the cycle as in Problem 2.120. isochoric
p1 Q'2
(a) Here p1V1 = p0V0, p2 = adiabatic
n
p1,V1,T0 /n
and p2V g1 = p0V g0 or p1 V1g = np0V0g
(a)
2.4 THE SECOND LAW OF THERMODYNAMICS. ENTROPY 277

i.e., V1g - 1 = nV0g - 1 or V1 = n1>g - 1 V0

V1 RT0
Q¿2 = CVT0 a 1 - b , Q1 = RT0 ln
1
= ln n = CVT0 ln n
n V0 g - 1
p0,V0,T0
Q¿2 n - 1
Thus, h = 1 - = 1 - Q1
Q1 n ln n adiabatic isothermal
V1
(b) Here, V2 = , p0V0 = p1V1 p1,V2,T0 /n Q'2 p1,V1,T0
n
(b)
p0V0g = p1V2g = p1n-g V1g = V0g - 1 n-g V1g - 1 or V1 = n(g>g - 1) V0
V1
Q¿2 = C pT0 a 1 - b , Q1 = RT0 ln
1 Rg
= T ln n = C pT0 ln n
n V0 g - 1 0
n - 1
Thus, h = 1 -
n ln n

2.122 The section from (p1, V1, T0) to (p2, V2, T0>n) is a polytropic process of index a. We shall
assume that the corresponding specific heat C is positive.
p0 ,V0 , T0
Here, dQ = CdT = CVdT + pdV Q1 isothermal

Now, pV a = constant or TV a - 1 = constant p1 ,V1, T0


Polytropic of
RT R
So, pdV = dV = - dT Q' index
2
V a - 1 adiabatic p2 ,V2 ,T0 /n

= R a b
R 1 1
Then, C = CV - -
a - 1 g - 1 a - 1
RT0 p1V1
We have p1V1 = RT0, p2V2 = =
n n
p0V0 = p1V1 = np2V2, p0V0g = p2V2g
1 g-1
p1V1a = p2V2a or V0g - 1 = V or V2 = V0n1>(g - 1)
n 2
1 a-1
V1a - 1 = V or V1 = n-1>(a - 1) V2 = n1>(g - 1) -1>(a - 1) V0
n 2
V1
Now, Q¿2 = CT0 a1 - b , Q1 = RT0 ln = RT0 a b ln n = CT0 ln n
1 1 1
-
n V0 g - 1 a - 1
n - 1
Thus, h = 1 -
n ln n
278 PART TWO THERMODYNAMICS AND MOLECULAR PHYSICS

T1 T1
Q 2¿ = Cp a T1 - b = CpT1 a 1 - b, Q1 = CV a T0 - b
1
2.123 (a) Here,
n n n
Along the adiabatic line p0 ,V0 , T0

T0Vg0 - 1 = T11n V02g-1 or T0 = T1ng - 1


adiabatic
T1
So, Q1 = CV (ng - 1) Q1
n
p1 ,V1, T1 /n Q'2 p1 , nV0 , T1
g (n - 1)
Thus, h = 1 - (a)
ng-1
(b) Here Q 2¿ = CV (nT1 - T0), Q1 = C p # T1(n - 1) p1 ,V0 /n, T0 p1 ,V0 , nT1

Along the adiabatic line TV g-1 = constant. Q1

V0 g-1 Q 2'
So, T0V g-1
0 = T1 a b or T1 = ng - 1 T0 adiabatic
n
p0 ,V0 , T0
ng - 1
Thus, h = 1 - (b)
gng - 1(n - 1)

Q 2¿ = CpT0 a 1 - b , Q –1 = RT0 ln n
1
2.124 (a) Here, p0,V0,T0
n

Q ¿1 = CVT0 a 1 - b
1 adiabatic
and
n Q1"
Now, Q1 = Q ¿1 + Q –1 Q1

Q¿2 C p(1 - 1>n) p0/n,V0,T0/n Q2


p0/n,nV0,T0
So, h = 1 - = 1 -
Q1 CV (1 - 1>n) + R ln n (a)
g g (n - 1)
= 1 - = 1 -
R n ln n n - 1 + (g - 1)n lnn
1 +
CV n - 1
Q1
(b) Here, Q1 = CpT0 (n - 1), Q 2– = CVT0 (n - 1), np0,V0 /n ,T0 np0,V0,nT0n

and 2 = RT0 ln n
Q‡
Q2''
Now, Q 2¿ = Q 2– + Q 2‡
Q¿2 n - 1 + (g - 1) ln n Q2"'=RT0ln n
So, h = 1 - = 1 - p0,V0,T0
Q1 g(n - 1)
(b)
2.125 We have T0
Q¿1 = tRT0 ln ν, Q¿2 = CVT0(t - 1) Q1 '

and Q2¿ = RT0 ln ν , Q1– = CVT0 (t - 1)


Q1'' Q2''
as well as Q1 = Q1¿ + Q1¿ V0
T0
and Q2¿ = Q2– + Q2‡ vV0
Q2'''
2.4 THE SECOND LAW OF THERMODYNAMICS. ENTROPY 279

Q¿2
So, h = 1 +
Q1
CV (t - 1) + R ln ν
= 1 -
CV (t - 1) + t R ln ν
t - 1
+ ln ν
g - 1 (t - 1) ln ν
= 1 - =
t - 1 t - 1
+ t ln ν t ln ν +
g - 1 g - 1
2.126 Here, Q1– = CpT0(t - 1), Q1¿ = tRT0 ln n
nT0
and Q2– = CpT0 (t - 1), Q2‡ = RT0 ln n V0
V0
In addition, we have Q1'
T0
Q1 = Q1¿ + Q1–
T0 Q1''
and Q2¿ = Q2– + Q2‡
Q2''' n V0
nV0
Q ¿2 Cp(t - 1) + R ln n Q2'' P0
So, h = 1 - = 1 -
Q1 Cp (t - 1) + tR ln n

t - 1 + a1 - b ln n
1
g (t - 1) ln n
= 1 - =
g(t - 1)
t - 1 + a1 - b t ln n
1
t ln n +
g g - 1
2.127 Because of the linearity of the section BC, whose equation is
p νV
= * ( K p = aV ) C
p0 V0 p0/V0,V0, T0/V

t Q1
We have ν = v or ν = 1t Q2''

Here, Q–2 = CVT0 1 1t - 12, B A p0,V0,T0


Q2'''
p0,V0/V,T0 /V
T0
Q‡2 = CpT0 a 1 - 11t - 12
1
= Cp
1t 1t
RT0
1 1t - 12 a 1 + b
g
Thus, Q¿2 = Q2– + Q2‡ =
g - 1 1t
Along BC, the specific heat C is given by

CdT = CV dT + pdV = CV dT + d a aV 2 b = a CV + R b dT
1 1
2 2
1 g + 1 t - 1
Thus, Q1 = RT0
2 g - 1 1t
Q ¿2 1t + g 1 (g - 1) 11t - 12
Finally h = 1 - = 1 - 2 =
Q1 1t + 1 g + 1 (g + 1)1 1t + 12
280 PART TWO THERMODYNAMICS AND MOLECULAR PHYSICS

2.128 We write Claussius inequality in the form


d1 Q d2 Q
- … 0
3 T 3 T

where d1Q is the heat transferred to the system but d 2 Q is heat rejected by the system,
both are positive and this explains the minus sign before d 2 Q .
In this inequality Tmax 7 T 7 Tmin and we can write
d1 Q d2 Q
- 6 0
3 Tmax 3 Tmin

Q1 Q¿2 Tmin Q¿2


Thus, 6 or 6
Tmax Tmin Tmax Q1
Q ¿2 Tmin
or h = 1 - 6 1 - = hcarnot
Q1 Tmax

2.129 We consider an infinitesimal carnot cycle with isothermal process at temperatures


T + dT and T. Let dA be the work done in the cycle and dQ, be the heat received at
the higher temperature. Then by Carnot’s theorem
Q1
dA dT p+dp 1 2
=
dQ1 T p 4 3
On the other hand,
0p
dA = dpdV = a b dTdV
0T V
V V+dV
0U
while, dQ1 = dU12 + pdV = c a b + p d dV
0V T
0U 0p
Hence, a b + p = T a b
0V T 0T V

2.130 (a) In an isochoric process, the entropy change will be



CVdT Tƒ R ln n isotherm
¢S = = CV ln = CV ln n =
3 T Ti g - 1
Ti

For carbon dioxide g = 1.30, so


¢S = 19.2 J/K mol
2.4 THE SECOND LAW OF THERMODYNAMICS. ENTROPY 281

(b) For an isobaric process, entropy change will be


Tƒ gR ln n
¢S = Cp ln = Cp ln n =
Ti g - 1
= 25 J>K mol

2.131 In an isothermal expansion, entropy change is



¢S = νR ln
Vi

So, = e ¢S>νR = 2.0 times
Vi
2.132 The entropy change depends on the final and initial states only, so we can calculate
it directly along the isotherm, it is ¢S = 2 R ln n = 20 J>K, assuming that the final
volume is n times the initial volume.

2.133 If the initial temperature is T0 and volume is V0 then in an adiabatic expansion,

TV g - 1 = T0V0g - 1

So, T = T0n1 - g = T1 (where n = V1>V0 )


V1 being the volume at the end of the adiabatic process. There is no entropy change
in this process. Next the gas is compressed isobarically and the net entropy change is

¢S = a Cp b ln
m
M T1
V1 V0 V0
But, = or Tƒ = T1 = T0n-g
T1 Tƒ V1

¢S = a C b ln = - Cp ln n = -
m 1 m m Rg
So, ln n = - 9.7 J>K
M p n M M g - 1
2.134 The entropy change depends on the initial and final state only so can be calculated for
any process. We choose to evaluate the entropy change along the pair of lines shown
in the figure. Then p0,V0,T0
T0>b aT0>b
CV dT dT
¢S = + nCp
3 T 3 T
T0 T0>b
nR p0 T0 p0 T0
= (- CV ln b + Cp ln a) n = (g ln a - ln b) ,V0 , , V0 ,
g - 1
L - 11 J>K
282 PART TWO THERMODYNAMICS AND MOLECULAR PHYSICS

2.135 To calculate the required entropy difference we only have to calculate the entropy differ-
ence for a process in which the state of the gas in vessel 1 is changed to that in vessel 2.
T1>ab T1>b
p1,V1,T1
¢S = n ± Cp ≤
dT dT
CV +
3 T 3 T
T1 T1>ab

= n(Cp ln a - CV ln ab)
p0 T0 p1 T1
,V , = , aV1,
ln bb = n R aln a - b
a 0 a
= n aR ln a -
R ln b
g - 1 g - 1
¢S = 0.85 J>K
(using g = 5>3, a = 2, b = 1.5 and n = 1.2).
2.136 For the polytropic process with index n, pV n = constant.
Along this process (as in solution of Problem 2.122),
n - g
C = Ra b =
1 1
- #R
g - 1 n - 1 1g - 121n - 12
tT0
dT n - g
So, ¢S = =C R ln t
3 T (g - 1)(n - 1)
T0

2.137 The process in question may be written as


p V
= a
p0 V0

(where a is a constant and p0, V0 are some reference values).


For this process (from solution of Problem 2.127), the specific heat is

1 g + 1
R = Ra + b = R
1 1 1
C = CV +
2 g - 1 2 2 g - 1
Along the line volume increases a times then so does the pressure. The temperature
must then increase a2 times. Thus,
a2T0
dT nR g + 1 g + 1
¢S = nC = ln a2 = nR ln a
3 T 2 g - 1 g - 1
T0

¢S = 46.1 J/K.
5
(using n = 2, g = and a = 2)
3
2.4 THE SECOND LAW OF THERMODYNAMICS. ENTROPY 283

2.138 Let p1, V1 be a reference point on the line and let 1p, V2 be any other point.
p1,V
Then, p = p0 - aV

The entropy difference


¢S = S1p, V) - S 1p1, V12
p,V
p V p1,V1
= CV ln + Cp ln
p1 V1
p0 - aV V
= CV ln + Cp ln
p1 V1

For an extremum of ¢S
0 ¢S - aCV Cp
= + = 0
0V p0 - aV V1
or Cp1p0 - aV2 - aVCV = 0
gp0
or g1p0 - aV2 - aV = 0 or V = Vm =
a1g + 12
0 2 ¢S
This gives a maximum of ¢S because 6 0
0 V2
Note that a maximum of ¢S is a maximum of S (p,V ).

2.139 For the process, S = aT + CV ln T


dS
and specific heat C = T = aT + CV
dT
On the other hand dQ = CdT = CV dT + pdV for an ideal gas.
RT
Thus, pdV = dV = aTdT
V
R dV R
or = dT or ln V + constant = T
aV a
Using T = T0 when V = V0
R V
we get T = T0 + ln
a V0

2.140 For a Van der Waal gas

ap + b1V - b2 = RT
a
V2
284 PART TWO THERMODYNAMICS AND MOLECULAR PHYSICS

The entropy change along an isotherm can be calculated from


V2
0S
¢S = b dV
a
3 0V T
V1

It follows from solution of Problem 2.129 that

0S 0p
a b = a b =
R
0V T 0T V V - b

(assuming a, b to be known constants).

V2 - b
Thus, ¢S = R ln
V1 - b

V2,T2 T2 V2
0S 0S
2.141 We use, ¢S = dS (V,T ) = a b dT + a b dV
3 3 0T V1 3 0V T = T2
V1,T1 T1 V1

T2 V2
CV dT R T2 V2 - b
= + dV = CV ln + R ln
3 T 3V - b T1 V1 - b
T1 V1

(assuming CV, a, b to be known constants).

2.142 We can take S : 0 as T : 0.


T T
dT 1
Then, S = C = a T 2dT = aT 3
3 T 3 3
0 0

T2 T2
m(a + bT) T2
dT = mb 1T2 - T12 + ma ln
CdT
2.143 ¢S = =
3 T 3 T T1
T1 T1

= 2.0 kJ (on substituting values)


1

T = aS n or S = a b
T n
2.144 Here,
a
1
1 T n-1 S
Then, C = T =
n a1>n n
Clearly, C 6 0 if n 6 0
2.4 THE SECOND LAW OF THERMODYNAMICS. ENTROPY 285

2.145 We know, T
C>0
T
CdT T
S - S0 = = C ln T0
3 T T0
T0 C<0
(assuming C to be a known constant).
S0 S
S - S0
Then, T = T0 exp a b
C

dS a
2.146 (a) C = T = -
dT T
T2
T1
(b) Q = CdT = a ln
3 T2
T1

(c) W = ¢Q - ¢U
T1
= a ln + CV (T1 - T2)
T2
Since for an ideal gas CV is constant and ¢U = CV (T2 - T1) (as U does not depend
on V ).

2.147 (a) We have from the definition


Q1
Q = TdS = area under the curve T S0,T0 S1,T0
3
Q1 = T0 1S1 - S02

1T + T1 21S1 - S02
1 Q2'
Q 2¿ =
2 0 S1,T1
T0 S
Thus, using T1 = (a)
n
T0 + T1 1 + 1>n n - 1
We get h = 1 - = 1 - =
2T0 2 2n T S0 ,T0

1S - S021T1 + T02
1
(b) Here, Q1 = Q1
2 1
Q¿2 = T1 1S1 - S02
2T1 T0 - T1 n - 1 S0 ,T1 Q ' S1,T1
h = 1 - = = 2
T1 + T0 T0 + T1 n + 1 S
(b)
286 PART TWO THERMODYNAMICS AND MOLECULAR PHYSICS

2.148 In this case, known as free expansion, no work is done and no heat is exchanged. So
internal energy must remain unchanged, i.e., Uƒ = Ui . For an ideal gas this implies co-
nstant temperature, i.e., Tƒ = Ti . The process is irreversible but the entropy change
can be calculated by considering a reversible isothermal process. Then, as before
V2
dQ pdV
¢S = = = nR ln n = 20.1 J/K
3 T 3 T
V1

2.149 The process consists of two parts. The first part is a free expansion in which Uƒ =Ui . The
second part is an adiabatic compression in which work done results in change of
internal energy. Obviously,
V0

0 = UF - Uƒ + pdV and Vƒ = 2V0


3

Now in the first part
1
pƒ = p an Vƒ = 2V0
2 0
(because there is no change of temperature).

In the second part,

p 12V02 g = 2 g - 1p0V g0
1
p Vg =
2 0
V0 V0
2 g - 1p0V0g 2 g - 1p0V g0
dV = c V 1-g d
V0
pdV =
3 3 Vg -g + 1 2V0
2V0 2V0

2g-1 - 1
= 2 g - 1p0V0gV 0-g + 1
g - 1
RT0
= 11 - 2 g - 12
g - 1
RT0
Thus, ¢U = UF - Ui = 12 g - 1 - 12
g - 1
The entropy change ¢S = ¢SI + ¢SII.
¢SI = R ln 2 and ¢SII = 0 as the process is reversible adiabatic. Thus, ¢S = R ln 2.

2.150 In all adiabatic processes, by virtue of first law of thermodynamics


Q = Uƒ - Ui + A = 0

Thus, Uƒ = Ui - A
2.4 THE SECOND LAW OF THERMODYNAMICS. ENTROPY 287

For a slow process,


V

A¿ = pdV (where for a quasistatic adiabatic process pV g = constant)


3
V0

On the other hand for a fast process, the external work done is A– 6 A¿. In fact
A– = 0 for free expansion. Thus U ¿ƒ (slow) 6 U –ƒ (fast).
Since U depends on temperature only, if T ƒ¿ 6 T –ƒ , consequently p– ƒ 7 p¿ƒ from the
ideal gas equation pV = RT.
Thus the pressure will be higher after the fast expansi on.

2.151 Let V1 = V0, V2 = hv0.


Since the temperature is the same, the required entropy change can be calculated by
considering isothermal expansion of the gas in either parts into the whole vessel.
Thus, ¢S = ¢SI + ¢SII
V1 + V2 V1 + V2
= ν1R ln + ν2 R ln
V1 V2
1 + n
= ν1R ln (1 + n) + ν2 R ln a b
n
= 5.1 J>K

2.152 Let c1 be the specific heat of copper and c2 be the specific heat of water.
.
T0 97 + 273
c2m2dT m1c1dT
Then, ¢S = -
3 T 3 T
7 + 273 T0
T0 370
= m2c2 ln - m1c1 ln
280 T0

T0 can be found from,


280m2c2 + 370m1c1
c2m2 1T0 - 2802 = m1c1 1370 - T02 or T0 =
c2m2 + m1c1

Using c1 = 0.39 J>g K, c2 = 4.18 J>g K,


T0 L 300 K and ¢S = 28.4 - 24.5 ⬵ 3.9 J>K

2.153 For an ideal gas the internal energy depends on temperature only. We can consider
the process in question to be one of simultaneous free expansion, then the total en-
ergy U = U1 + U2.
288 PART TWO THERMODYNAMICS AND MOLECULAR PHYSICS

Since U1 = CVT1, U2 = CV T2 so, U = 2CV (T1 + T2)>2 and 1T1 + T22>2 is the final
temperature.
The entropy change is obtained by considering isochoric process because in effect,
the gas remains confined to its vessel.
1T1 + T22>2
1T1 + T222
T2
CV dT dT
So, ¢S = - CV = CV ln
3 T 3 T 4T1T2
T1 1T1 + T22>2

Since, 1T1 + T222 = 1T1 - T222 + 4T1T2,

We get ¢S 7 0

2.154 (a) Each atom has probability 1>2 to be in either compartment. Thus P = 2-N.
(b) Typical atomic velocity at room temperature is ' 105 cm/s so it takes an atom
10-5 s to cross the vessel. This is the relevant time scale for our problem. Let
T = 10-5 s, then in time T there will be t>T crossing or arrangements of the
atoms. This will be large enough to produce the given arrangement if
t -N ln t>t
2 L 1 or N L L 75
t ln 2

2.155 The statistical weight is


N! 10 * 9 * 8 * 7 * 6
NC = = = 252
N>2 N 8 * 4 * 3 * 2
N>2 ! !
2
The probability distribution is
NC 2-N = 252 * 2-10 = 24.6%
N>2

2.156 The probabilities that the half A contains n molecules is


N! 1 5 10 5 1
NC * 2-N = 2-N; , , , , , respectively.
n n !(N - n) ! 32 32 32 32 32
2.157 The probability of one molecule being confined to the marked volume is
V
P =
V0

We can choose this molecule in many 1NC ways. The probability that n molecules get
1
confined to the marked volume is clearly

NC P n 11 - P2N - n = P n 11 - P2N - n
N!
n n !1N - n2 !
2.5 LIQUIDS. CAPILLARY EFFECTS 289

2.158 In a sphere of diameter d, the number of molecules are


pd 3
N = n
6 0
where n0 = Loschmidt’s number = number of molecules per unit volume (1 cc) under NTP.
The relative fluctuation in this number is
0N 1N 1
= = = h
N N 1N

a b
1 p 3 6 6 1>3
or = d n0 or d 3 = Q d = = 0.41 mm
h2 6 pn0h2 ph2n0
The average number of molecules in this sphere is
1
= 106
h2

2.159 For a monoatomic gas , CV = 3>2R per mole.


The entropy change in the process is
T0 + ¢T
¢T
b
dT 3
¢S = S - S0 = CV = R lna 1 +
3 T 2 T0
T0

Now from the Boltzmann equation, S = k ln Æ .


3NA 3*6
Æ ¢T 2 * 10
23

= e 1S - S02>k = a 1 + b = a1 + b
2 1
= 101.31 * 10
21
So,
Æ0 T0 300

Thus the statistical weight increases by this factor.

2.5 Liquids. Capillary Effects

2.160 (a) ¢p = a a b =
1 1 4a
+
d>2 d>2 d
4 * 490 * 10-3 N N
= 2
= 1.307 * 106 2 = 13 atm
1.5 * 10-6 m m
(b) The soap bubble has two surfaces.

¢p = 2a a b =
1 1 8a
So, +
d>2 d>2 d
8 * 45
= = 1.2 * 10-3 atm
3 * 10-3
290 PART TWO THERMODYNAMICS AND MOLECULAR PHYSICS

2.161 The pressure just inside the hole will be less than the outside pressure by 4a>d. This
can support a height h of Hg where,
4a 4a
r gh = or h =
d rgd
4 * 490 * 10-3 200
So, h = = L 0.21 m of Hg
13.6 * 103 * 9.8 * 70 * 10-6 13.6 * 70

2.162 By Boyle’s law


p0
a p0 + b a b = a + b a b
8a 4p d 3 8a 4p hd 3
d 3 2 n hd 3 2

p0 a 1 - b = 1h - 12
h3 8a 2
or
n d

p0d a1 - b n 1h2 - 12
1 h3
Thus, a =
8 n

2.163 The pressure has terms due to hydrostatic pressure and capillarity and they add as
4a
p = p0 + rgh +
d
5 * 9.8 * 10 4 * 0.73 * 10-3
= a1 + * 10-5 b atm = 2.22 atm
3
+
105 4 * 10-6

2.164 By Boyle’s law

a p0 + hgr + b d = a p0 + b nd
4a p 3 4a p 3 3
d 6 nd 6

1n - 12
4a 2
or hgr - p0 (n3 - 1 ) =
d

h = c p01n3 - 12 + 1n - 12 d ngr = 4.98 m of water.


4a 2
or
d

2.165 Clearly

¢hrg = 4a| cos u | a b


1 1
-
d1 d2
4a| cos u |1d2 - d12
¢h = = 11 mm
d1d2rg
2.5 LIQUIDS. CAPILLARY EFFECTS 291

2.166 In a capillary with diameter d = 0.5 mm, water will rise to a height
R
2a 4a
h = =
rgr rgd
4 * 73 * 10-3
= = 59.6 mm
103 * 9.8 * 0.5 * 10-3

Since this is greater than the height (= 25 mm) of the tube, a meniscus of radius
R will be formed at the top of the tube, where

2a 2 * 73 * 10-3
R = = 3 L 0.6 mm
rgh 10 * 9.8 * 25 * 10-3

2.167 Initially the pressure of air in the capillary is p0 and it’s length is l. When submerged
under water, the pressure of air in the portion above water must be p0 + 4a>d, since
the level of water inside the capillary is the same as the level outside. Thus by
Boyle’s law

a p0 + b 1l - x2 = p0l
4a
d

1l - x2 = p0x
4a
or
d
l
or x = = 1.4 cm (on substituting values)
1 + p0d>4a

2.168 We have by Boyle’s law

ap0 - rgh + b 1l - h2 = p0 l
4a cosu
d
4a cos u p0h
or = rgh +
d l - h
p0h
a = a rgh + b
d
Hence,
l - h 4 cos u

2.169 Suppose the liquid rises to a height h. Then the total energy of the liquid in the
capillary is

E 1h2 = A d 22 - d 12 B h * rg * - p 1d2 + d12 ah


p h
4 2
4a
h = = 6 cm
rg1d2 - d12
Minimising E , we get
292 PART TWO THERMODYNAMICS AND MOLECULAR PHYSICS

2.170 Let h be the height of the water level at a distance x from the edge. Then the total
energy of water in the wedge above the level outside is

xdw # dx # h # rg - 2 dx # h # a cos u
h
E =
L 2 L
h
xrgdw a h2 - 2 hb
1 2a cos u x
= dx
L 2 xrgdw

xrgdw c ah - b - 2 2 2 2
1 2a cos u 2 4a2 cos2 u
= dx
L 2 xrgdw x r g dw

2a cos u
This is minimum when h =
xrgdw

2.171 From the equation of continuity

d v = a b #V
p 2# p d 2
or V = n 2v
4 4 n

We then apply Bernoulli’s theorem

p 1
+ v 2 + £ = constant
r 2
The pressure p differs from the atmospheric pressure by capillary effects. At the
upper section, p = p0 + 2 a>d neglecting the curvature in the vertical plane.

p0 + 2a>d 1 2 p0 + 2na>d 1
Thus, + v + gl = + n4v 2
r 2 r 2
2gl - 14a>rd21n - 12
or v =
B n4 - 1
Finally, the liquid coming out per second is,

1 2 2gl - 14a>rd21n - 12
V = pd = 0.9 cm3>s (on substituting values)
4 B n4 - 1

2.172 The radius of curvature of the drop is R1 at the upper end of the drop and R2 at the lower
end. Then the pressure inside the drop is p0 + 12a>R12 at the top end and p0 + 12a>R22
at the bottom end. Hence,
2a 2a 2a1R2 - R12
p0 + = p0 + + rgh or = rgh
R1 R2 R1R2
2.5 LIQUIDS. CAPILLARY EFFECTS 293

To a first approximation R1 L R2 L h>2.


So, R2 - R1 L 1>8 rgh3>a L 0.20 mm
and if h = 2.3 mm, a = 73 mN>m
2.173 We must first calculate the pressure difference inside the film from that outside. This is
given by

p = aa + b
1 1
r1 r2
h
Here, 2r1| cos u | = h and r2 L R, where R is the radius of
the tablet and can be neglected. Thus the total force exert-
ed by mercury drop on the upper glass plate is typically,
2pR 2a | cos u |
h
We should put h/n for h because the tablet is compressed n times. Then since Hg
is nearly incompressible, pR 2h = constant, so R : R 1n. Thus, total force
2pR 2a | cos u | 2
= n
h
Part of the force is needed to keep the Hg in the shape of a tablet rather than in the shape
of infinitely thin sheet. This part can be calculated being putting n = 1 above. Thus
2pR 2a | cos u | 2pR 2a | cos u |
mg + = n2
h h
2pR a | cos u |
2
or m = 1n2 - 12 = 0.7 kg
hg
2.174 The pressure inside the film is less than that outside by an amount
1 1
p = aa + b
r1 r2
(where r1 and r2 are the principal radii of curvature of the meniscus).
One of these is small, being given by h = 2r1 cos u while the other is large and will
be ignored.
2A cos u
Then, F L a
h
(where A  area of the water film between the plates)
m
Now, A =
rh
2ma
So, F = [when u (the angle of contact)  0]
rh2
= 1.0 N (on substituting values)
294 PART TWO THERMODYNAMICS AND MOLECULAR PHYSICS

2.175 This is analogous to the previous problem except that, A = pR 2.


2pR 2a
So, F = = 0.6 kN
h

2.176 The energy of the liquid between the plates is


h 1
E = ldh rg - 2 alh = rgldh2 - 2alh
2 2 h

rgld a h - b -
1 2a 2 2a2l
=
2 rgd rgd

This energy is minimum when, h = 2a>rgd and the minimum potential energy is then
2a2l
Emin = -
rgd
The force of attraction between the plates can be obtained from this as
- 0Emin 2a2l
F = = - (minus sign means the force is attractive)
0d rgd 2

alh
Thus, F = = 13 N
d

2.177 Suppose the radius of bubble is x at some instant. Then the pressure inside is
p0 + 4a>x. The flow through the capillary is given by Poiseuille’s equation,

pr 4 4a dx
Q = = - 4px 2
8hl x dt

Integrating we get,

t = p1R 4 - x 42
pr 4a
2hl

(where we have used the fact that t = 0 and x = R ).


The life time of the bubble corresponding to x = 0 is

2hlR 4
t =
ar 4

2.178 If the liquid rises to a height h, the energy of the liquid column becomes

- 2prha = rgp a rh - 2 b -
a 2 2pa2
E = rgpr2h #
h 1
2 2 rg rg
2.5 LIQUIDS. CAPILLARY EFFECTS 295

This is minimum when rh = 2a>rg and that is relevant height to which water must
rise.
2pa2
At this point, Emin = -
rg

Since E = 0 in the absence of surface tension, heat liberated must be


2pa2
Q =
rg

2.179 (a) The free energy per unit area being a,

F = pad 2 = 3 mJ (on substituting values)

(b) F = 2pad 2 because the soap bubble has two surfaces.


Substitution gives, F = 10 mJ.

2.180 When two mercury drops each of diameter d merge, the resulting drop has diameter d1
p 3 p
where, d 1 = d 3 * 2 or d1 = 21>3d
6 6
The increase in free energy is
¢F = p 22>3 d 2 a - 2pd 2 a = 2pd 2 a 12-1>3 - 12 = –1.43 mJ

2.181 Work must be done to stretch the soap film and compress the air inside. The former
is simply 2a * 4pR 2 = 8pR 2 a, there being two sides of the film. To get the latter
we note that the compression is isothermal and work done is
Vf = V

- pdV
3
Vi = V0

V0 p0 = a p0 + b V
4a # 4p 3
where, and V = R
R 3
pV 4a
or V0 = and p = p0 +
p0 R

and minus sign is needed because we are calculating work done on the system. Thus
since pV remains constant, the work done is
V0 p
pV ln = p V ln
V p0
p
So, A¿ = 8p R 2 a + p V ln
p0
296 PART TWO THERMODYNAMICS AND MOLECULAR PHYSICS

2.182 When heat is given to a soap bubble, the temperature of the air inside rises and the
bubble expands but unless the bubble bursts, the amount of air inside does not
change. Further we shall neglect the variation of the surface tension with tempera-
ture. Then from the gas equations

a p0 + b
4a 4p 3
r = nRT (n = constant)
r 3
Differentiating we get,

ap0 + b 4pr 2dr = nRdT


8a
3r

nRdT
or dV = 4pr 2dr =
8a
p0 +
3r
Now from the first law of thermodynamics

a p0 + b
nRdT 4a
d Q = nCdT = nCV dT +
p0 + 8a>3r r

p0 + 4a>r
or C = CV + R
p08a>3r

using C p = CV + R, we get
1>2 R
C = Cp +
1 + 3p0r>8a

2.183 Consider an infinitesimal Carnot cycle with isotherms at T - dT and T. Let A be the
work done during the cycle. Then

A = [a1T - dT 2 - a1T 2] ds = -
da
dT ds
dT T – dT

(where ds is the change in the area of film; we are consider-


ing only one surface here).
T
q
Then by Carnot theorem σ

A dT
h = =
Q1 T

da
- dT ds
dT dT da
or = or q = - T
q ds T dT
2.6 PHASE TRANSFORMATIONS 297

2.184 As before we can calculate the heat required. It is taking into account two sides of the
soap film. So,
da
dq = - T ds * 2
dT
dq da
Thus, ¢S = = -2 ds
T dT
Now, ¢F = 2ads

¢U = ¢F + T¢S = 2 a a - T b ds
da
So,
dT

2.6 Phase Transformations

2.185 The condensation takes place at constant pressure and temperature and the work
done is A = p ¢V , where ¢V is the volume of the condensed vapour phase. It is

¢m
p ¢V = RT = 1.2 J
M

(where M = 18 g is the molecular weight of water).

2.186 The specific volume of water (the liquid) will be written as V ¿l . Since V ¿vap 7 7 V ¿l ,
most of the weight is due to water. Thus if ml is mass of the liquid and mvap that of
the vapour then

m = ml + mvap

V = mlV ¿l + mvapV ¿ vap or V - mV ¿l = mvap 1V ¿ vap - V ¿2


l

V - mV ¿l
So, mvap = = 20 g
V ¿ vap - V ¿l
Its volume is V ¿ vap = 1.0 l

2.187 The volume of the condensed vapour was originally V0 - V at temperature T = 373
K. Its mass will be given by
Mp01V0 - V 2
p0 1V0 - V 2 =
m
RT or m = = 2g
M RT
(where p0 = standard atmospheric pressure).
298 PART TWO THERMODYNAMICS AND MOLECULAR PHYSICS

2.188 We let V ¿l = specific volume of liquid. V vap


¿ = NV ¿l = specific volume of vapour.
Let V = original volume of the vapour.

= 1ml + Nmvap2 V ¿l
pV V V
Then, M = ml + mvap = or
RT NV ¿l n
mlV ¿l n - 1
So, 1N - 12 ml V ¿l = V a 1 - b = 1n - 12 or h =
1 V
=
n n V>n N - 1
In the case when the final volume of the substance corresponds to the midpoint of
a horizontal portion of the isothermal line in the pV diagram, the final volume must
be 11 + N 2 V ¿l >2 per unit mass of the substance. Of this, the volume of the liquid is
V ¿>2
l per unit total mass of the substance.
1
Thus, h =
1 + N
2.189 From the first law of thermodynamics
¢U + A = Q = mq
(where q is the specific latent heat of vaporization).
So, increment in entropy is given by
Q mq
¢S = = = 6.0 kJ/K (on substituting values)
T T

A = p 1V ¿vap - V ¿2
RT
Now, l m = m
M

¢U = m a q - b
RT
Thus,
M
For water, this gives L 2.08 * 106 J.

2.190 Some of the heat used in heating water to the boiling temperature
T = 100°C = 373 K. The remaining heat = Q - mc¢T (here c = specific heat of
water, ¢T = 100 K) is used to create vapour. If the piston rises to a height h then
the volume of vapour will be L Sh (neglecting water). Its mass will be
p0Sh
* M
RT
Heat of vapourization will be
p0Sh Mq
RT
. To this must be added the work done in creating the saturated vapour = p0Sh.
Q - mc¢T
Q - m c¢T L p0S h a 1 + b
qM
Thus, or h L = 20 cm
p0S a 1 + b
RT qM
RT
2.6 PHASE TRANSFORMATIONS 299

2.191 A quantity mc1T - T02>q of saturated vapour must condense to heat the water to
bolling point T = 373 K. (Here c = specific heat of water, T0 = 295 K = initial water
temperature.)

The work done in lowering the piston will then be


mc1T - T02 RT
* = 25 J
q M
Since work done per unit mass of the condensed vapour is pV = RT>M .

2.192 Given
rvap 2a rvap 4a
¢P = = * = h pvap
rl r rl d
1m>M2 RT hRT
= h Pvap = h = r
Vvap M vap

4aM
or d =
rl RTh

For water, a = 73 dynes/cm, M = 18 g, rl = 1 g/cc and T = 300 K. Using these va-


lues and with h = 0.01, we get d L 0.2 m m.

2.193 At equilibrium the number of “liquid” molecules evaporating must equal the number
of “vapour” molecules condensing. By kinetic theory, this number is
1 1 8 kT
h * n 6 v 7 = h * n *
4 4 A m
p
The required mass is
kT m
m = m * h * n * = h nkT
A 2pm A 2pkt
M
= h p0 = 0.35 g/cm2
A 2pRT
where p0 is standard atmospheric pressure, T = 373 K and M = molecular weight of
water.

2.194 Here we must assume that m is also the rate at which the tungsten filament loses
mass when in an atmosphere of its own vapour at this temperature and that h (of the
previous problem) L 1. Then
2pRT
p = m = 0.9 nPa
A M
where p = pressure of the saturated vapor.
300 PART TWO THERMODYNAMICS AND MOLECULAR PHYSICS

2.195 From the Van der Waal’s equation


RT a
- 2
p =
V - b V
(where V = volume of one gram mole of the substances).
For water V = 18 cm3 per mol = 1.8 * 10-2 l mol-1 and a = 5.47 atm l2/mol 2.
If molecular attraction vanishes, the equation will be
RT
p¿ = (for the same specific volume)
V - b
Thus,
a 5.47
¢p = m = * 104 atm L 1.7 * 104 atm
V2 1.8 * 1.8

2.196 The internal pressure being a>V 2, the work done in condensation is
Vg
a a a a
2
dV = - L
3V Vl Vg Vl
Vl

This by assumption is Mq, M being the molecular weight, q being specific talent heat of
vaporization and Vl , Vg being the molar volumes of the liquid and gas, respectively
a Mq
Thus, pi = 2 = = rq
Vl Vl
(where r is the density of the liquid).
For water pi = 2 * 104 atm

2.197 The Van der Waal’s equation can be written as (for one mole)

p 1V 2 =
RT a
- 2
V - b V
0p 0 2p
At the critical point a b and a b vanish. Thus,
0V T 0V 2 T

RT 2a RT 2a
0 = - 3 or = 3
1V - b22 V 1V - b22 V

2RT 6a RT 3a
0 = - =
1V - b2 1V - b2
3
or 3
V4 V4
Solving these simultaneously, we get on division
2
V - b = V, V = 3 b L VMCr
3
2.6 PHASE TRANSFORMATIONS 301

This is the critical molar volume. Putting this back, we get


RTCr 2a 8a
2
= 3
or TCr =
4b 27b 27bR
RTCr a 4a a a
Finally pcr = - = - =
VMCr - b 2
VMCr 27b2 9b 2 27b 2
From these we see that
pCrVMCr a>9b 3
= =
RTCr 8a>27b 8

pCr a>27b2 1
2.198 We have, = =
RTCr 8a>27b 8b
TCr 0.082 * 304
Thus, b = R = = 0.043 l/mol
8pCr 73 * 8
1RTCr22 64a 27 1RTCr2
2
and = or a = = 3.59 atm l 2/mol
pCr 27 64 pCr

2.199 Specific volume is molar volume divided by molecular weight. Thus


VMCr 3RTCr 3 * 0.082 * 562
V ¿Cr = = = l>g = 4.71 cm3>g
M 8MpCr 8 * 78 * 47

ap + b 1VM - b2 = RT
a
2.200 We have,
VM2
p + a>VM2 VM - b 8 T
or * =
pCr VMCr 3 TCr

ap + b * an - b = t
a b 8
or 2
pCrVM VMCr 3
p VM
awhere p =
T a
,n = ,t =
pCr VMCr TCr

ap + b a n - b = t or ap + b an - b = t
27b 2 1 8 3 1 8
or
V M2 3 3 n2 3 3

When p = 12 and n = 1/2, we get


3 1 3
t = * 24 * =
8 6 2
302 PART TWO THERMODYNAMICS AND MOLECULAR PHYSICS

2.201 (a) The critical volume VMCr is the maximum volume in the liquid phase and the
minimum volume in the gaseous. Thus,
1000
Vmax = * 3 * 0.030 l = 5 l
18
(b) The critical pressure is the maximum possible pressure in the vapour phase in
equilibrium with liquid phase. Thus,
a 5.47
pmax = = = 225 atm
27b 2 27 * 0.03 * 0.03

8 a 8 3.62
2.202 We have, TCr = = * L 304 K
27 bR 27 0.043 * 0.082
M 44
rCr = = g/cm3 = 0.34 g/cm3
3b 3 * 43

2.203 The vessel is such that either vapour or liquid of mass m occupies it at critical point.
Then its volume will be
m 3 RTCr m
VCr = VMCr =
M 8 pCr M

The corresponding volume in liquid phase at room temperature is


m
V =
r

(where r = density of liquid ether at room temperature).


V 8MpCr
Thus, h = = L 0.254
VCr 3RTCr r

(using the given data and r = 720 g/l).

2.204 We apply the following relation, at constant T to the p


4
cycle 1234531
II
1 5
T dS = dU + pdV 3
I
C C C
2
Here, dS = dU = 0 V
C

So, pdV = 0
C
2.6 PHASE TRANSFORMATIONS 303

This implies that the areas I and II are equal. This reasoning is inapplicable to the
cycle 1231, for example. This cycle is irreversible because it involves the irreversible
transition from a single phase to a two-phase state at the point 3.

2.205 When a portion of super-cool water turns into ice some heat is liberated, which
should heat it upto ice point. Neglecting the variation of specific heat of water, the
fraction of water turning into ice is clearly
c|t|
f = = 0.25
q
(where c = specific heat of water and q = latent heat of fusion of ice).
Clearly f = 1 at t = - 80°C.

2.206 From the Clausius–Clapeyron (C - C ) equation


dT T 1V 2¿ - V 1¿ 2
=
dp q12

Here, q12 is the specific latent heat absorbed in 1 : 2 and 1 = solid, 2 = liquid.

T 1V 2¿ - V 1¿ 2 273 * 0.091
¢T = ¢p = - * 1 atm cm3 K>J
q12 333

atm * cm3 105N/m2 * 10-6m3


1 = = 10-1 Nm/J, ¢T = 0.0075 K
J J

2.207 Here 1 = liquid, 2 = steam.


T 1V 2¿ - V 1¿ 2
So, ¢T = ¢p
q12

q12 ¢T 2250 0.9


or V ¿2 L = * * 10-3 m3/g = 1.7 m3/kg
T ¢p 373 3.2

2.208 From C-C equation

dp q12 q12
= =
dT T 1V 2¿ - V 1¿ 2 TV 2¿

Assuming the saturated vapour to be ideal gas, we can write


304 PART TWO THERMODYNAMICS AND MOLECULAR PHYSICS

1 mp
=
V 2¿ RT
Mq
Thus, ¢p = p ¢T
RT 2 0

p = p0 a 1 + ¢T b L 1.04 atm
Mq
and
RT 2

2.209 From C-C equation, neglecting the volume of the liquid


q12
1as q = q122
dp Mq
L L p
dT TV ¿ 2 RT 2
dp Mq dT
or =
p RT T
m MpV
Now, pV = RT or m = (for an ideal gas)
M RT

1V is constant = specific volume)


dm dp dT
So, = -
m p T
18 * 2250
= a = a - 1b *
Mq dT 1.5
- 1b L 4.85%
RT T 8.3 * 373 375

2.210 From C-C equation


dp q Mq
L L p
dT TV 2¿ RT 2
Mq
Integrating ln p = constant -
RT

p = p0 exp c a - bd
Mq 1 1
So,
R T0 T
This is reasonable for |T - T0|<<T0, and far below critical temperature.

2.211 As before from solution of Problem 2.206, the lowering of melting point is given by
C¢V ¿
¢T = - p
q
The superheated ice will then melt in part. The fraction that will melt is
CT¢V ¿
h = p = 0.03
q2
where C is the specific heat capacity of ice, T  273 K and q is the specific heat of
melting.
2.6 PHASE TRANSFORMATIONS 305

2.212 (a) The equations of the transition lines are


1800
log p = 9.05 - (solid-gas)
T
1310
= 6.78 - (liquid-gas)
T
At the triple point they intersect. Thus,
490 490
2.27 = or Ttr = = 216 K
Ttr 2.27
The corresponding ptr is 5.14 atm.
In the formula log p = a - b>T , we compare b with the corresponding term in the
equation in Problem 2.210. Then
2.303 b Mq
ln p = a * 2.303 - Q 2.303 =
T R
2.303 * 1800 * 8.31
or q solid-gas = = 783 J/g
44
2.303 * 1310 * 8.31
q liquid-gas = = 570 J/g
44
Finally q solid-liquid = 213 J/g (on subtraction)
T2
T2
L m a c ln b
dT mq q
2.213 ¢S = mc + +
3 T T2 T1 T2
T1

= 103 a 4.18 ln b L 7.2 kJ/K


373 2250
+
283 373

qm T2 qv
2.214 ¢S L + c ln +
T1 T T2
333 373
= + 4.18 ln L 8.56 J/K
273 283

2.215 Specific heat of copper, c = 0.39 J/gK. Suppose all the ice does not melt, then

heat rejected  90 0.39 (90  0)  3159 J


and heat gained by ice  50 2.09 3 x 333

This gives, x  8.5 g


306 PART TWO THERMODYNAMICS AND MOLECULAR PHYSICS

The hypothesis is correct and final temperature will be T  273 K.


Hence change in entropy of copper piece
T 273
¢S = mc ln = mc ln = - 10 J/K
T1 363

2.216 (a) Here t2 = 60°C. Suppose the final temperature is t °C. Then,
heat lost by water = m2 c 1t2 - t2
heat gained by ice = m1qm + m1 c 1t - t12 (if all the ice melts)
In this case, m1qm = m2 * 4.18 160 - t2 (for m1 = m2 )
So the final temperature will be 0 C and only some ice will melt.
Then 100 * 4.18 (60) = m¿1 * 333
or m¿1 = 75.3 g = amount of ice that will melt
333 273
Finally ¢S = 75.3 * + 100 * 4.18 ln
273 333

m¿ 1qm T1
¢S = + m2c ln
T1 T2
(T2 - T1) T2
= m2c - m2c ln
T1 T1
T2 T2
= m2c c - 1 - ln d = 8.8 J/K
T1 T1

(b) If m2ct2 > m1qm , then all the ice will melt as one can check and the final temper-
ature can be obtained like this
m2c 1T2 - T2 = m1qm + m1c 1T - T12
1m2T2 + m1T12 c - m1qm = 1m1 + m22 cT
m1qm
m2T2 + m1T1 -
c
or T = L 280 K
m1 + m2

m1q T2
+ c a m1 ln - m2ln b = 19 J/K
T
and ¢S =
T1 T1 T

mq2 T2 Mqice
2.217 ¢S = - - mc ln +
T2 T1 T1
2.6 PHASE TRANSFORMATIONS 307

where, mq ice = m 1q2 + c 1T2 - T122


T2 T2
= mq2 a b + mc a - 1 - ln b
1 1
So, ¢S -
T1 T2 T1 T1
= 0.2245 + 0.2564 = 0.48 J/K

2.218 When heat dQ is given to the vapour its temperature will change by dT, pressure by
dp and volume by dV, it being assumed that the vapour remains saturated.
Then by C-C equation,
dp q q
= ¿ 7 7 V l ¿) or dp =
(V vap dT
dT TV ¿ TV ¿

RT
On the other hand, pV ¿ =
M
RdT
So, pdV ¿ + V ¿dp =
M

pdV ¿ = a - b dT
R q
Hence,
M T

Finally dQ = CdT = dU + pdV ¿

= CvdT + a - b dT = CpdT - dT
T q q
M T T

(Cp , CV refer to unit mass here).


q
Thus, C = Cp -
T

For water, Cp =
Rg .1 (with g = 1.32 and M = 18)
g - 1 M
So, Cp = 1.90 J/g K

and C = - 4.13 J/g K = - 74 J/mol K

2.219 The required entropy change can be calculated along a process in which the water is
heated from T1 to T2 and then allowed to evaporate. The entropy change for this
is
T2 qM
¢S = Cp ln +
T1 T2

where q = specific latent heat of vaporization.


308 PART TWO THERMODYNAMICS AND MOLECULAR PHYSICS

2.7 Transport Phenomena

2.220 (a) The fraction of gas molecules which traverses distances exceeding the mean free
path without collision is just the probability to traverse the distance s = l with-
out collision.
1
Thus, P = e-1 = = 0.37
e

Hence the sought fraction h = 0.37.


(b) This probability is
P = e-1 - e-2 = 0.23

Hence the sought fraction h = 0.23.

1 ¢l .
2.221 From the formula = e ¢l/l or l =
h ln h

2.222 (a) Let P (t)  probability of no collision in the interval (0, t). Then

P (t + dt) = P (t) (1 - adt)

dP
or = - aP (t) or P (t) = e-at
dt

where we have used P (0) = 1.


(b) The mean interval between collision is also the mean interval of no collision. Then,
q

te-atdt
3
0 1 ≠(2) 1
6 t7 = q = =
a ≠(1) a
e-atdt
3
0

1 kT
2.223 (a) l = =
22pd 2n 22pd 2p
1.38 * 10 - 23 * 273
= = 6.2 * 10-8 m
22p (0.37 * 10-9)2 * 105
2.7 TRANSPORT PHENOMENA 309

l 6.2 * 10-8
t = = s = 0.136 ns
6v 7 454
(b) l = 6.2 * 106 m
t = 1.36 * 104 s = 3.8 h

2.224 The mean distance between molecules is of the order

22.4 * 10-3 1/3


a b a b * 10-9 m « 3.34 * 10-9 m
224 1/3
=
6.0 * 1023 6

This is about 18.5 times smaller than the mean free path calculated in Problem
2.223(a) above.

2.225 We know that the Van der Waal’s constant b is four times the molecular volume. Thus,

d or d = a b
p 3 3d 1/3
b = 4NA
6 2p NA
kT0 2pNA
l = a ba b
2/3
Hence, = 84 nm (on substituting values)
22pP0 3b

2.226 The velocity of sound in N2 is


gp gRT
=
Ar A M
1 gRT0 RT0
So, = =
n A M 22p d 2p0NA

= 5.5 GHz (on substituting values)

kT
2.227 (a) l 7 l if p 6
12p d 2l
L 0.7 Pa for O2

(b) The corresponding molecular concentration n is obtained by dividing by kT and


is 1.84 * 1020 per m3 = 1.84 * 1014 per cm3 and the corresponding mean dis-
tance is given by
l 10-2
= = 1.8 * 10-7 m L 0.18 mm
n1>3 (0.184)1>3 * 105
310 PART TWO THERMODYNAMICS AND MOLECULAR PHYSICS

1 1 6 v7
2.228 (a) v = = =
t l> 6v 7 l
= 12pd 2 n 6 v7 = 0.74 * 1010 s-1
p0 8 RT
where n = and 6 v7 =
kT0 A pM
(b) Total number of collisions is
1
nv = 1.0 * 1029 s-1 cm-3
2
Note, the factor 1>2, which come because when two molecules collide we must
not count it twice.

1
2.229 (a) l =
12pd 2n
d is a constant and n is a constant for an isochoric process, so l is constant for
an isochoric process.
6 v7
nl
8RT
v = =
l B Mp
i.e., v r 1T
1 kT
(b) l = r T (for an isobaric process)
12pd 2 P
6v7 1T 1
v = r = (for an isobaric process)
l T 1T
2.230 (a) In an isochoric process l is constant and
v r 1T r 1pV r 1p r 1n
so v increases n times
kT
(b) l = must decrease n times in an isothermal process and v must in-
12pd 2p
crease n times because 6v 7 is constant in an isothermal process.
1 1 V
2.231 (a) l r Ú =
n N>V N
T 1>2
Thus, l r V and v r
V
But in an adiabatic process TV g - 1 = constant, so TV 2>5 = constant (as g = 7>5 here).
or T 1>2 r V -1>5
Thus, v rV -6>5
2.7 TRANSPORT PHENOMENA 311

T
(b) Given l r
p

p a b = constant
T g
But,
p
T
or r p-1>g or T r p 1 - 1>g
p
Thus, l r p -1>g = p-5>7
6 v7 p
When, v = r r p1>2 + 1>2g = pg + 1>2g = p6>7
l 1T
(c) Given l r V
But, TV 2/5 = constant or V r T -5>2

Thus, l r T -5>2
T 1>2
v r r T3
V
2.232 In the polytropic process of index n , pV n = constant, TV n -1 = constant and p 1–n T n =
constant.
(a) When l r V,
T 1>2
v r = V 1 - n>2 V -1 = V -n + 1>2
V
T
(b) When l r ,
p
T n r p n - 1 or T r p 1 - 1>n

So, l r p -1>n
6 v7 p
v = r r p 1 - 1>2 + 1>2n = p (n + 1)>2n
l 1T
T
(c) When l r ,
p
p r T n>n - 1

l r T 1 - n>n - 1 = T - 1>n - 1 = T 1>1 - n


p
v r r T 1n>n - 12 - 11>22 = T 1n + 12>21n - 12
1T

2.233 (a) The number of collision between the molecules in a unit volume is
1 1 1T
nn = pd 2 n 2 6 v 7 r 2
2 12 V
312 PART TWO THERMODYNAMICS AND MOLECULAR PHYSICS

This remains constant is polytropic process , pV -3 = constant.


Using solution of Problem 2.122, the molar specific heat for the polytropic pro-
cess, pV a = constant, is

C = Ra b
1 1
-
g - 1 a - 1

C = R a + b = R a + b =
1 1 5 1 11
Thus, R
g - 1 4 2 4 4
1
It can also be written as R (1 + 2i) (where i = 5 )
4
On substituting values C = 23 J/K mol.
1T
(b) In this case = constant and so pV -1 = constant.
V

C = R a + b = R a + b = 3R
1 1 5 1
So,
g - 1 2 2 2
R
which can also be written as (i + 1) (where i = 5).
2
On substituting values C = 29 J/K mol.

2.234 We can assume that all molecules incident on the hole, leak out. Then,
1
- dN = - d1nV2 = n 6 v 7 S dt
4
dt dt
or dn = - n = -n
4v>S 6 v7 t

Integrating, n = n0 e-t>t

8RT
Hence, 6v 7 =
A pM

2.235 If the temperature of the compartment 2 is h times more than that of compartment
1, it must contain 1>h times less number of molecules since pressure must be the
same when the big hole is open. If M = mass of the gas in 1 than the mass of the
gas is 2 must be M>h. So immediately after the big hole is closed,

M M
n 01 = and n 02 =
mV mVh
2.7 TRANSPORT PHENOMENA 313

where m = mass of each molecule and n01, n02 are concentrations in 1 and 2. After
the big hole is closed the pressures will differ and concentration will become n1 and
n2, where

11 + h2
M
n1 + n2 =
mVh
On the other hand

n1 6v1 7 = n2 6v2 7 , i.e. , n1 = 1h n2

n2 A 1 + 1h B = 11 + h2 = n0211 + h2
m
Thus,
mVh
1 + h
So, n2 = n 02
1 + 1h

2.236 We know
1 1 1
h = 6v 7lr = 6v 7 m a 1T
3 3 12 p d 2

Thus h changing a times implies T changing a2 times.


On the other hand

1 1 8kT kT
D = 6v 7l =
3 3A pm 12pd 2p

T 3>2
Thus D changing b times means changing b times.
p
a3
So p must change times = 2 times (on substituting values).
b

1T
2.237 D r r V 1T and h r 1T
n
(a) D will increase n times, and h will remain constant if T is constant.

T 3>2 1pV2 3>2


(b) D r r = p 1>2 V 3>2
p p
Also, h r 1pV

Thus D will increase n 3>2 times, h will increase n1>2 times, if p is constant.
314 PART TWO THERMODYNAMICS AND MOLECULAR PHYSICS

2.238 D r V 1T and h r 1T
In an adiabatic process

TV g - 1 = constant or T r V 1-g
Now V is decreased 1>n times.

1 3 - g>2
D r V 3 - g>2 = a b = a b
1 4>5
Thus,
n n
1 -1>5
h r V 1g>2 = a b = n 1>5
n
So D decreases n 4>5 times and h increase n1>5 times L 6.3 and 1.6 times, respectively.

2.239 (a) D r V 1T r 1pV 3


Thus D remains constant in the process pV 3 = constant. So polytropic index
of the process n = 3.

(b) h r 1T r 1pV
So h remains constant in the isothermal process. pV = constant, n = 1, here.

(c) Heat conductivity k = hCV and CV is a constant for the ideal gas.
Thus n = 1 here also.

1 8kT m 2 mkT 1
2.240 h = =
3A pm 12 p d 2 3 A p3 d 2

1>2 4 * 8.31 * 273 * 10-3 1>4


or d = a b a 3 b = a * 106 b a b
2 1>2 mkT 1>4 2
3h p 3 * 18.9 p3 * 36 * 1046
1>2 4 * 83.1 * 273 1>4
= 10-10 a b a b L 0.178 nm
2
3 * 18.9 p3 * 0.36

1
2.241 k = 6 v7 lrcV
3
1 8kT 1 CV
= mn
3 A mp 12 p d n
2 M
2.7 TRANSPORT PHENOMENA 315

cV is the specific heat capacity which is CV >M. Now CV is the same for all monoatom-
ic gases such as He and A.
1
Thus, k r
1Md 2

kHe 1MA dA2 dA2


or = 8.7 = = 110
kA 1MHe dHe
2 2
dHe

dA 8.7
= = 1.658 L 1.7
dHe A 110

2.242 In this case


r 22 - r 21
N1 = 4ph␻
r 21r 22
2R ¢R 2p h vR 3
or N1 L 4p h v or N1 =
R4 ¢R

To decrease N 1 n times, h must be decreased n times. Now h does not depend on


pressure until the pressure is so low that the mean free path equals, say, 1>2 ¢R.
Then the mean free path is fixed and h decreases with pressure. The mean free path
equals 1>2 ¢R , when
1
= ¢R (where n0 = concentration)
22pd 2n0

Corresponding pressure is
22kT
p0 =
pd 2 ¢R
The sought pressure is n times less, given by

22kT 10-23
p = = 70.7 * -18
L 0.71 Pa
2
pd n¢R 10 * 10-3
The answer is qualitative and depends on the choice 1>2 ¢R for the mean free path.

2.243 We neglect the moment of inertia of the gas in a shell. Then the moment of friction
forces on a unit length of the cylinder must be a constant as a function of r.
N1 1
- 2b
dv 1
4ph a r 1
So, 2pr 3h = N1 or v(r) = 2
dr r
N1 N1
a- 2b a - 2b
1 1 1 1
and v = or h =
4ph r 21 r2 4pv r 21 r2
316 PART TWO THERMODYNAMICS AND MOLECULAR PHYSICS

2.244 We consider two adjoining layers. The angular velocity gradient is v>h. So the mo-
ment of the frictional force is
a
␻ pha 4␻
N = r 2prdr hr =
3 h 2h
0

2.245 In the ultra-rarefied gas we must determine h by taking l = 11>22h.


1 8kT 1 mp 1 2M
Then, h = * h * = hp
3 A mp 2 kT 3 A pRT
1 pM
So, N = va 4p
3 A 2RT

2.246 Take an infinitesimal section of length dx and apply Poiseuille’s equation to this.
dV - pa 4 0p
Then, =
dt 8h 0x

pV = RT #
m
From the formula,
M
RT
pdV = dm
M
dm pa 4M pdp
or = m = -
dt 8hRT dx

This equation implies that if the flow is isothermal, then


dp |p 22 - p 21|
p = = constant
dx 2l

pa 4M |p 2 - p 1|
2 2
Thus, m =
16hRT l

2.247 Let T be the temperature of the interface. Then heat flowing from left is equal to the
heat flowing into right in equilibrium.
T1 - T T - T2 1k1T12>l1 + 1k2T22>l2
k1 = = k2 = or T =
k1>l1 + k2>l2
Thus,
l1 l2

2.248 We have
T1 - T T - T2 T1 - T2
k1 = k2 = k
l1 l2 l1 - l2
2.7 TRANSPORT PHENOMENA 317

or using the previous result


k1T1 k2T2
+
k1 l1 l2 T1 - T2
§T1 - ¥ = k
l1 k1 k2 l1 + l2
+
l1 l2
k2
(T1 - T2)
k1 l2 T1 - T2 l1 + l2
or = k Q k =
l1 k1 k2 l1 + l2 l1 l2
+ +
l1 l2 k1 k2
2.249 By definition the heat flux per unit area is
#
Q = -K
dT
= -a
d
ln T = constant = +
a ln T /T
1 2
dx dx l
x T2
Integrating ln T = ln + ln T1
l T1
(where T1  temperature at the end x  0)
T2 #
T = T1 a b
x/l a
So, and Q = ln T1/T2
T1 l

2.250 Suppose the chunks have temperatures T1, T2 at time t and T1  dT1, T2 dT2 at
time dt t.
kS
Then, C1dT1 = C2dT2 = (T1 - T2) dt
l

a b ¢T dt 1where ¢T = T1 - T22
kS 1 1
Thus, d¢T = – +
l C1 C2
k#S 1
¢T = (¢T )0e-t/t a where a bb
1 1
Hence, = +
t l C1 C2

# 0T 0T
2.251 Q = k = - A 2T (where A  constant)
0x 0x
2 0T 3/2
= - A
3 0x

2 (T 1
3/2 - T 3/2
2 )
= A
3 l

x
Thus, T 3/2 = constant - (T13/2 - T23/2)
l
318 PART TWO THERMODYNAMICS AND MOLECULAR PHYSICS

Using T = T1, at x = 0, we get

x T2 3/2
a b = 1 + aa b - 1 b
x 3/2 T 3/2
T 3/2 = T13/2 + (T - T13/2 ) or
l 2 T1 l T
T2 3/2
T = T1 c1 + e a b - 1 fd
x 2/3

l T1

where x is the distance from the plate maintained at the temperature T1.

1 8RT 1 R (i>2) R 3>2iT 3>2


2.252 k = mn =
3 A pM 22pd 2n M 3p3>2d 2 2MNA
Then from the previous problem

2iR 3>2(T 3>2


2 - T 3>2
1 )
q = ,
9p3>2d 2 2MNAl
i  3 here and d is the effective diameter of helium atom.

2.253 At this pressure and average temperature ( 27°C  300 K)


(T1 + T2)
T =
2
1 kT -5
l = = 2330 * 10 m = 23.3 mm 77 5.0 mm = l
22pd 2 p
The gas is ultra-thin and we write l = (1>2) l here.

dT T2 - T1
Then, q = k = k
dx l
1 1 MP R 1 p 6v 7
where, k = 6 v7 * l * * * = l
3 2 RT g - 1 M 6T (g - 1)
p 6v 7
and q = (T - T1) = 22 W/m2 (on substituting values)
6T (g - 1) 2
8RT
where, 6v 7 = .
A Mp
T2 + T1
We have used T2 - T1 6 6 here.
2
dT
2.254 At equilibrium, 2pr k = - A = constant.
dr
A
So, T = B - ln r
2pk
2.7 TRANSPORT PHENOMENA 319

But T  T1 , when r  R 1 and T  T2 , when r  R2 .


T2 - T1 r
From this we find T  T1
ln R2>R1
ln
R1

dT
2.255 At equilibrium 4pr2k = - A = constant
dr
A 1
T = B +
4pk r
Using T  T1 when r  R1 and T  T2 when r = R2,
T2 - T1
a - b
1 1
T = T1 +
(1>R2) - (1>R1) r R1

2.256 The heat flux vector is - k§T and its divergence equals w. Thus,
w
§2T = –
k
1 0 0T
a b ar b = – in cylindrical coordinates.
w
or
r 0r 0r k
w 2
or T = B + A ln r - r
2k
Since T is finite at r  0, A  0. Also T  T0 at r  R
w 2
So, B = T0 + R
4k
w 2
Thus, T = T0 + (R - r 2)
4k
where r is the distance from the axis of the wire also called axial radius.

2.257 Here again w


§2T = –
k
So in spherical polar coordinates,
1 0 0T 0T 0T
ar 2 b = –
w w w
or r 2 = - or r 2 = - r3 + A
r 2 0r 0r k 0r k 0r 3k
A w 2
or T = B - - r
r 6k
w 2
Again A = 0 and B = T0 + R
6k
w
So finally, T = T0 + * (R 2 - r 2)
6k
ELECTRODYNAMICS
PART 3
3.1 Constant Electric Field in Vacuum

q2 gm 2
3.1 Fele (for electrons) = and Fgr =
4pe0r 2 r2

Fele q2
Thus, (for electrons) =
Fgr 4pe0gm 2

(1.602 * 10 –19 C) 2
= = 4 * 1042
a b * 6.67 * 10 –11m 3>(kg # s 2) * (9.11 * 10 –31 kg) 2
1
9 * 10 9

Fele q2
Similarly (for proton) =
Fgr 4pe0gm 2

(1.602 * 10-19 C)2


= = 1 * 1036
a b * 6.67 * 10 - 11 m3> (kg # s2) * (1.672 * 10 - 27 kg)2
1
9 * 109

For Fele = Fgr,


q2 gm 2 q
= or = 24pe0g
4pe0r 2 r 2 m

6.67 * 10 –11m 3 (kg # s 2)


= = 0.86 * 10 –10 C>kg
B 9 * 10 9

1
3.2 Total number of atoms in the sphere of mass 1 gram = * 6.023 * 10 23.
63.54

6.023 * 10 23
So the total nuclear charge l = * 1.6 * 10 -19 * 29.
63.54
322 PART THREE ELECTRODYNAMICS

Now the charge on the sphere = total nuclear charge - total electronic charge
6.023 * 10 23 29 * 1
= * 1.6 * 10 -19 * = 4.298 * 10 2 C
63.54 100

Hence, force of interaction between these two spheres,

1 # [4.398 * 10 2] 2
F = N = 9 * 10 9 * 10 4 * 19.348 N = 1.74 * 10 15 N
4pe0 12

3.3 Let the balls be deviated by an angle u from the vertical when separation between
them equals x.
Applying Newton’s second law of motion for any one of the spheres, we get
T cos u = mg (1)
and T sin u = Fele (2) T

x Fele
From the Eqs. (1) and (2)
mg
Fele
tan u = (3)
mg
But from the figure
x x
tan u = ⬵ as x 66 l
2l
2 l2 - a b
x 2
(4)
B 2
From Eqs. (3) and (4)
mg x q2 mg x
Fele = or =
2l 4pe0x 2 2l

2pe0mgx 3
Thus q2 = (5)
l
Differentiating Eq. (5) with respect to time
dq 2pe0mg dx
2q = 3x 2
dt l dt

aapproach velocity is b
dx a dx
According to the problem = v =
dt 1x dt
2pe0mg 3pe0mg
a x3b
1>2 dq a
so, = x2
l dt l 2x
3.1 CONSTANT ELECTRIC FIELD IN VACUUM 323

dq 3 2pe0mg
Hence, = a
dt 2 B l
Note: If the plane of figure is x-y plane and point of suspension is O, Eq. (3) can be
obtained more easily using net torque equal to zero about OZ-axis , i.e., t0Z = O .
3.4 Let us choose coordinate axes as shown in the figure and fix three charges, q1, q2 and
q3 having position vectors r1, r2 and r3, respectively.
Now, for the equilibrium of q3
q2q3 (r2 - r3) q1q3 (r1 - r3)
+ = 0 y q1
| r2 - r3 |3 | r1 - r3 |3 r1−r3
r1 q3
q2 q1 r2−r3
or = r3 q
| r2 - r3 | 2 | r1 - r3 | 2 r2 2
O x
r2 - r3 r1 - r3
because = -
| r2 - r3 | | r1 - r3 | z

So, 1q2 (r1 - r3) = 1q1 (r3 - r2)


1q2 r1 + 1q1 r2
or r3 =
1q1 + 1q2
Also for the equilibrium of q1
q3 (r3 - r1) q2 (r2 - r1)
+ = 0
| r3 - r1 |3 | r2 - r1 | 3
- q2
or q3 = | r1 - r3 | 2
| r2 - r1 | 2
Substituting the value of r3, we get
- q1q2
q3 =
1 1q1 + 1q222

3.5 When the charge q0 is placed at the center of the ring, the wire dFele
gets stretched. The component of extra tension ¢T towards the ∆T
center will balance the electric force dFele due to the charge q0 on d
a differential part of the ring which subtends angle du at the ∆T
center, having the charge q0

dq = a b du
q
2p
du
2¢T sin = dFele
2
As du is very small
du
2¢T sin L Tdu
2
324 PART THREE ELECTRODYNAMICS

q0dq 1 q0 q
Using dFele = = a b du
4pe0r 2 4pe0 r 2 2p
1 q0
a b du
q
¢Td u =
4pe0 r2 2p
qq0
Hence, ¢T =
8p 2e0r 2
q
3.6 Sought field strength E =
1 # (r - r0 ) = 2.7i - 3.6j kV/m
4pe0 | (r - r0 ) | 3
1 q
So, |E| =
4pe0 | r - r0 | 2
= 4.5 kV>m (on substituting values)
3.7 Electric field strength due to a point charge (q) at a field point (P ) in vacuum is given
by
1 qr
E = q
4pe0 r 3 r
q r0
1
= P
4pe0 | (r - r0 ) | 3 O r
Taking into account the expresion for E, let us fix the coordinate system by taking the
point of intersection of the diagonals as the origin and let k be the unit vector directed
normally, emerging from the plane of figure.
Hence the sought field strength:
q li + x k - q l ( - i) + x k +q y
E = + +q(−l,0,0)
4pe0 (l 2 + x 2)3>2 4pe0 (l 2 + x 2)3>2 (0,l,0)
–q lj + xk q l ( - j) + x k k
+ +
4pe0 (l 2 + x 2)3>2 4pe0 (l 2 + x 2)3>2 (l,0,0)
-q
q (0,−l,0) −q
x
= [2 l i - 2l j]
4pe0 (l 2 + x 2)3>2
ql
Thus, E = dl
12pe0 (l 2 + x 2)3>2 d
R
O x
3.8 From the symmetry of the problem the sought field strength
C E
E = dEx dE
3
where the projection of field strength along x-axis due to an elemental charge is
dq cos u q R cos ud u
dEx = =
4pe0R 2 4p2e0R 3
3.1 CONSTANT ELECTRIC FIELD IN VACUUM 325

p>2
q q
Hence, E = cos u d u =
4p2e0R 2 3 2p 2e0R 2
- p>2

= 0.10 kV>m (on substituting values)

3.9 It can be easily shown that vector E in this case must be directed n
along the axis of the ring (see figure). Let us isolate differential lin- dE
E q
ear charge element dq on the ring in the vicinity of point A. We
write the expression for the component dEn of the field created by
this element at a field point P as
R
l
1 dq dq l
dEn = cos u = A r O
4pe0 R 2 4pe0 (r + l 2) 3>2
2

(where n is the unit vector perpendicular to the plane of ring or along the axis of the ring).
1 l q l
Hence, E = dEn = dq =
3 4pe0 (r + l ) 3
2 2 3>2 4pe0 (r + l 2)3>2
2

It can be seen that for l W r , the field E L q>4pe 0l 2, i.e., at large distances the system
behaves as a point charge.
From the expression of E = E (l ), the electric field is zero at the center of the ring
(l = 0) and also zero when l is very large. Hence, there must be a value of l for which
the electric field is maximum. This value of l can be obtained by setting the first de-
rivative of E to zero as follows
dE q (r 2 - 2l 2)
= 0 or = 0
dl 4pe0 (r 2 + l 2)5>2
r
which yields, l =
12
q
Emax =
6 13pe0r 2

3.10 The electric potential at a distance x from the given ring is given by
q q
w (x) = -
4pe0x 4pe0 (R + x 2)1/2
2

Hence, the field strength along x-axis (which is the net field strength in our case) is

c 2 - d
dw 1 q qx
Ex = - =
dx 4pe0 x (R + x 2)3>2
2

x 3 c a1 + 2 b - 1 d
q R 2 3>2
4pP0 x
=
x 2(R 2 + x 2)3>2
326 PART THREE ELECTRODYNAMICS

x3 c1 + + Ád
q 3 R2 3 R4
2
+
4pe0 2 x 8 x4
=
x 2 ( R 2 + x 2) 3>2
Neglecting the higher power of R>x, as x W R
3qR 2
E =
8pe0x 4
Note: Instead of w (x), we may write E (x) directly using solution of problem 3.9.

3.11 From the solution of Problem 3.9, the electric field strength due to ring at a point on
its axis (say x-axis) at distance x from the center of the ring is given by
qx
E (x) =
4pe0 (R 2 + x 2) 3>2
and from symmetry, E at every point on the axis is directed along the x-axis (see figure).
Let us consider an element dx on thread which carries the charge l dx . The electric
force experienced by the element in the field of ring
l qx dx
dF = (l dx) E (x) =
4pe0 (R 2 + x 2)3>2
Thus the sought interaction
q
R x
l qx dx
F = O x E
3 4 pe0 (R + x )
2 2 3/2
0
dx
On integrating we get
lq
F =
4pe0R
3.12 (a) The given charge distribution is shown in the figure. The symmetry of this distri-
bution implies that vector E at the point O is directed to the right, and its magni-
tude is equal to the sum of the projection onto the direction of E of vectors dE
from elementary charges dq. The projection of vector dE onto vector E is
1 dq
dE cos w = cos w
4pe0 R 2
where dq = lRdw = l0R cos wdw. (1)
Integrating Eq. (1) over w between 0 and 2p dq d

we find the magnitude of the vector E as R


2p O
E
l0 l0
E = cos2 wdw =
4pe0R 3 4e0R dE
0
3.1 CONSTANT ELECTRIC FIELD IN VACUUM 327

It should be noted that this integral is evaluated in the most simple way if we take
into account that 6cos2 w 7 = 1>2. Then

2p

cos2 wdw = 6 cos2 w7 2p = p


3
0

(b) Let us take differential length element of the ring at an azimuthal angle w from the
x-axis, the element subtends an angle dw at the center, and carries charge
dq = l Rdw = (l0 cos w) Rdw.
Taking the plane of ring as x-y plane and center of the ring x
as origin O, locations of field point r, of charge element R (Field Point)
and of field point relative to charge element r are shown in
j r
the figure. R P z
O r
Electric field strength at the field point due to considered
charge element
y
1 dq
dE = r
4pe0 r3

Using r = r - R = x k - (R cos w i + R sin w j) and r 3 = (R 2 + x 2) 3>2

1 (l0 cos w) Rdw


dE = {x k - (R cos w i + R sin w j)}
4pe0 (R 2 + x 2) 3>2

So, sought net electric field strength

E = dE
3
2p 2p 2p
l0R
= xk cos wdw - R i cos2wdw - R j sin 2wdw
4pe0 (R 2 + x 2) 3>2 J 3 3 3 K
0 0 0

Taking into account


2p 2p 2p

cos wdw = 0, sin 2wdw = 0 and cos2wdw = p


3 3 3
0 0 0

1 pl0R ( - i)
2

We get E =
4pe0 (R 2 + x 2) 3>2
l0R 2
Hence, E =
4e0(R 2 + x 2) 3>2
328 PART THREE ELECTRODYNAMICS

For x W R ,
p
Ex = where p = l0pR 2
4pe0x 3

Alternate:
Take an element S at an azimuthal angle w from the x-axis, the element subtending
an angle dw at the center. The elementary field at P due to the element is
l0 cos wdwR
(along SP with components )
4pe0 (x 2 + R 2)
l0 cos wdw R
* {cos u along OP, sin u along OS }
4pe0 (x 2 + R 2)
x
where cos u =
(x 2 + R 2) 1>2
R
and sin u =
(x 2 + R 2) 1>2
2p

The component along OP vanishes on integration as cos wdw = 0.


3
0
The component along OS can be broken into the parts x
along Ox and Oy and given by

S
l0R 2 cos wdw R z
* {cos w along Ox, sin w along Oy } O x
4pe0 (x 2 + R 2) 3>2 (Field Point)

On integration, the part along Oy vanishes. y


2R
l0R 2
So, Ex = cos 2wdw
4pe0 (x 2 + R 2) 3>2 3
0

2R

as cos 2w sin wdw = 0


3
0
l0R 2
Finally E = Ex =
4e0 (x 2 + R 2)3>2
3.1 CONSTANT ELECTRIC FIELD IN VACUUM 329

3.13 (a) It is clear from symmetry considerations that vector E must be directed as shown
in the figure. This shows the way of solving this problem: we must find the com-
ponent dEr of the field created by the element dl of the rod, having the charge dq
and then integrate the result over all the elements of the rod. In this case
1 ldl
dEr = dE cosa = cos a
4pP0 r 20

where l = q / 2a is the linear charge density. Let us red- dl


da
uce this equation to the form convenient for integration. r0da
Figure shows that dl cos a = r0da and r0 = r / cos a. r0 da
a E
Consequently, O r
1 lr0da l dE
dEr = 2
= cos a da
4pe0 r 0 4pe0r
This expression can be easily integrated to yield
a0
l l
E = 2 cos ada = (2 sin a0)
4pe0r P 3 Q 4pe0r
0

where a0 is the maximum value of the angle a.


a
sin a0 =
1a2 + r2

a2 b
q>2a a
Thus, E =
4pe0r 2a 2 + r 2
q
=
4pe0r 2a 2 + r 2
q
Note that in this case also E L for r W a, as for the field of a point charge.
4pe0r 2
(b) Let, us consider the element of length dl at a distance l from the center of the rod,
as shown in the figure.
dl
Then field at P, due to this element is given by r P
l
ldl
dE = 2a
4pe0 (r - l) 2
if the element lies on the side as shown in the figure,
ldl
and dE =
4pe0 (r + l) 2
if the element lies on the other side.
330 PART THREE ELECTRODYNAMICS

a a
l dl l dl
Hence, E = dE = +
3 3 4pe0 (r - l )2
3 4pe0 (r + l )
2
0 0
q
On integrating and putting l = , we get
2a
q 1
E =
4pe0 (r 2 - a 2)
q
For r W a, E L
4pe0r 2

3.14 The problem is reduced to finding Ex and Ey , which are the projections of E, under
the assumption l 7 0 (see figure).
Let us start with Ex. The contribution to Ex from the charge element of the segment dx is
1 l dx dE
dEx = sin a (1)
4pe0 r 2 Ey E
Let us reduce this expression to the form convenient for integra-
tion. Thus, Ex
da
r da y a
dx = ,r =
cos a cos a y
r
l
Then, dEx = sin a da
4pe0y dx x

Integrating this expression over a between 0 and p>2, we find


l
Ex =
4pe0y
In order to find the projection Ey it is sufficient to recall that dEy differs from dEx in
that sin a in Eq. (1) is simply replaced by cos a.
This gives
dEy = (l cos a da)>4 pe0y and Ey = l>4pe0 y

We have obtained an interesting result that Ex = Ey independently of y, i.e., E is orie-


nted at the angle of 45° to the rod. The modulus of E is
l 12
E = 2E 2x + E 2y =
4pe0 y
3.15 For Fig. (a): Using the solution of Problem 3.14, the net electric field strength at the
point O due to straight parts of the thread equals zero. For the curved part (arc), let
us derive a general expression i.e. let us calculate the field strength at the center of
arc of radius R and linear charge density l and which subtends angle u0 at the center.
3.1 CONSTANT ELECTRIC FIELD IN VACUUM 331

From the symmetry the sought field strength will be directed along dl
the bisector of the angle u0 and is given by
d
+ u0>2 0 /2

l (Rdu) l u0
E = 2
cos u = sin
3 4pe0R 2pe0R 2
-u0>2
dE E
In our problem u0 = p>2, thus the field strength due to the turned part at the point
12l
E0 =
4pe0R
which is also the sought result. For Fig. (b): Using the solution of Problem 3.14 (a),
net field strength at O due to straight parts equals
12 l
12 a b =
l
(directed vertically downward)
4pe0R 2pe0R
Now using the solution of Problem 3.15 (a) field strength due to the given curved
part (semi-circle) at the point O becomes
l
(directed vertically upward)
2pe0R
Hence the sought net field strength becomes zero.
3.16 Given charge distribution on the surface s = a # r is shown in the figure. Symmetry of
this distribution implies that the sought E at the center O of the sphere is opposite to
a. For the ring element
dq = s (2pr sin u) rdu = (a # r)
d
2pr 2 sin u du = 2par 3 sin u cos u du
O a
Again from symmetry, field strength due to any ring E r
element d E is also opposite to a, i.e., d E cT a. Hence,
dqr cos u –a
dE = (using the result of Problem 3.9)
4pe0 (r sin u + r cos u)
2 2 2 2 3>2 a

(2par 3 sin u cos u du) r cos u ( - a)


=
4pe0r 3 a
-ar
= sin u cos2 d u
2e0
p
( - a)r
Thus, E = dE = sin u cos2u du
3 2e0 3
0
Integrating, we get
ar 2 ar
E = = -
2e0 3 3e0
332 PART THREE ELECTRODYNAMICS

3.17 We start from two charged spherical balls each of radius R with equal and opposite
charge densities + r and - r. The center of the balls are at + a/ 2 and - a/ 2, resp-
ectively, so the equation of their surfaces are

`r - ` = R or r - cos u ⬵ R
a a
2 2
a
and r + cos u ⬵ R
2

considering a to be small. The distance between the two surfaces in the radial dir-
ection at angle is u | a cos u | and does not depend on the azimuthal angle. It is
seen from the diagram that the surface of the sphere has in effect a surface density
s = s0 cos u when s0 = ra.

Inside any uniformly charged spherical ball, the field is radial and has
z
the magnitude given by Gauss’s theorem
θ
4p 3
4pr 2E = r r/e0
3
rr –a
or E =
3e0
In vector notation, using the fact that V must be measured from the center of the ball,
we get, for the present case

ar - b - ar + b
r a r a
E =
3e0 2 3e0 2
s0
= - r a/3e0 = - k
3e0
where k is the unit vector along the polar axis from which u is measured.

3.18 Let us consider an elemental spherical shell of thickness dr. Thus surface charge
density of the shell s = rdr = (a # r) dr.
Thus using the solution of Problem 3.16, field strength due to this spherical shell

ar
dE = - dr
3e0
Hence, the sought field strength
R
a aR 2
E = - rdr = -
3e0 3 6e0
0
3.1 CONSTANT ELECTRIC FIELD IN VACUUM 333

3.19 Due to the line charge


l l
E = ( - k) + (er) (see solution of Problem 3.14)
4pe0 r 4pe0 r
Distribution of field E allows us to take a finite ring element
of radius 0 6 r 6 R and thickness dr. Hence area vector of
the taken ring element dS = (2prdr) ( - k).
Now flux across the considered ring element
er Ee
-k
d£ = E # dS = c d # 2prdr ( - k)
l
+ EZK
4pe0 r r
2pl dr
d£ = + (because er ⬜ k) z
4pe0

lR
So, £ = d£ =
3 2e0
3.20 Take the line joining the point charges as E+q
z-axis, In this problem at a normal distance r
E
(from z-axis) E is the same and is directed to-
wards positive z-axis. So, symmetry of the field E–q
–q
E allows us to take a finite ring element of l +q
radius r and thickness dr.
E at all the locations of taken ring element is d

2q 2ql
E = cos u k = k
4pe0 (l 2 + r 2) 4pe0 (l 2 + r 2) 3/2
2ql qlrdr
So, d£ = E # d S = k # 2prdr k =
4pe0 (l 2 + r 2) 3/2 e0 (l 2 + r 2) 3/2
R

c1 - d
ql rdr q l
Hence, £ = d£ = =
3 e03 (l 2 + r 2 ) 3/2 e0 2l + R 2
2
0

It can also be solved by considering a ring element or by E


using solid angle.
dS

3.21 From Gauss’ theorem, electric field strength at an inside r0 x


point at a distance r from the center of uniformly charged
rr R
ball of volume density r is E = . So E is not uniform
3 e0
334 PART THREE ELECTRODYNAMICS

over the given section (a disk of radius 2R 2 - r0 2 ) of the ball. Thus the section is
considered to be made up of ring elements whose radii varies from zero to 2R 2 - r0 2.
Let us consider a ring element of radius x and thickness dx, as shown in the figure,
so that magnitude of E takes same value over the considered ring element.
Flux over the considered ring element
d£ = E # d S = ErdS cos u
Using Er = rr/ 3e0 , ds = 2pxdx, and cos u = r0 /r , we get
rr r0 rr0
d£ = 2px dx = 2pxdx
3e0 r 3e0
Hence, sought flux
2R 2 - r 2
2prr0 (R 2 - r20)
0
2pr r0 prr0
£ = xdx = = (R 2 - r 20)
3e0 3 3e0 2 3e0
0
3.22 The field at P due to each of the threads at A and B are is the same having the ma-
gnitude
l
E0 = (directed along AP and BP )
2pe0 (x 2 + l 2/4) 1/2
The resultant is along OP , given by

E = 2E0 cos u E
P
lx
=
pe0 (x 2 + l 2/4) E0 E0
x
l
= A
O B

pe0 c x + # 2x + l d
l2 l /2 l /2
- 2#
l
4x 2 2x
l
=
pe0 c a 2x - b + ld
1 2

2 2x

l l
This is the maximum when x = and E = Emax = .
2 pe0l

3.23 Take a section of the cylinder perpendicular to its axis through the point where the
electric field is to be calculated. (All points on the axis are equivalent.) Consider an
element S with azimuthal angle w. The length of the element is Rdw, R being the
3.1 CONSTANT ELECTRIC FIELD IN VACUUM 335

radius of cross section of the cylinder. The element itself is a section of an infinite
strip. The electric field at O due to this strip is
s0 cos w (Rdw)
along SO y Rd
2pe0R
This can be resolved into x
O
s0 cos w dw dE
e f
cos w along Ox
2pe0 sin w along y O

On integration the component along yO vanishes. What remains is


2x
s0 cos 2 wdw s0
=
3 2pe0 2 e0
0

along xO, i.e., along the direction w = p.

3.24 Since the field is axis-symmetric (as the field of a uniformly charged filament),
we conclude that the flux through the sphere of radius R is equal to the flux
through the lateral surface of a cylinder having the same radius and the height R
2R, as shown in the figure.

Now, £ = E # d S = ErS
L
a
But, Er =
R

£ = S = 2 p R # 2R = 4paR
a a
Thus,
R R
Alternate:
The distribution of electric field is given by z

a (x i + y j)
a␳ dS
E= = r
x + y
2 2 r2
where ␳ = x i + y j , which is the vector component of position q y
vector r normal to the z-axis. Let us take a differential area el-
ement dS on the surface of the given sphere of radius R, as
shown in the figure. x

d£ = E # d S = 2 # d S = a b = dS
a␳ a a dS r a
Then, dS sin u =
r r r R R

a a
So, £ = dS = (4pR 2) = 4paR
R3 R
336 PART THREE ELECTRODYNAMICS

3.25 (a) Let us consider a sphere of radius r 6 R , then charge enclosed by the considered
sphere
r r

4pr 2 r0 a1 - b dr
r
q enclosed = 4pr 2drr = (1)
3 3 R
0 0

Now, applying Gauss’ theorem


qenclosed
Er4pr 2 =
e0
(where Er is the projection of electric field along the radial line)
r
r0
4pr 2 a1 - b dr
r
=
e0 3 R
0
r0r
c1 - d
3r
or Er =
3e0 4R
For a point outside the sphere r 7 R, so
R

4pr 2dr r0 a 1 - b (as there is no charge outside the ball)


r
qenclosed =
3 R
0

Again from Gauss’ theorem

4pr 2dr r0 a 1 - b
R
r
R
Er4pr 2 =
3 e0
0

r0 r0R 3
Er = 2 c d =
R3 R4
or -
r e0 3 4R 12 r 2e0

(b) As magnitude of electric field decreases with increasing r for r 7 R, field will be
maximum for r 6 R. Now, for Er to be maximum

ar - b = 0 or 1 -
d 3r 2 3r 2R
= 0 or r = rm =
dr 4R 2R 3

r0R
Hence, Emax =
9e0

3.26 Let the charge carried by the sphere be q, then using Gauss’ theorem for a spheri-
cal surface having radius r 7 R, we can write
3.1 CONSTANT ELECTRIC FIELD IN VACUUM 337

r
qenclosed q 1 a
E 4pr 2 = = + 4pr 2dr
e0 e0 e0 3 r
R
On integrating, we get

(q - 2paR 2) 4par 2
E 4pr 2 = +
e0 2e0

The intensity E does not depend on r when the expression in the parentheses is equal
to zero. Hence,
a
q = 2paR 2 and E =
2e0

3.27 Let us consider a spherical layer of radius r and thickness dr, having its center coin-
ciding with the center of the system. Then using Gauss’ theorem for this surface,
r
qenclosed rdV
Er 4pr 2 = =
e0 3 e0
0
r
1
= r e –ar 3 4pr 2dr
e0 3 0
0

After integration
r 4p
Er4pr 2 = [1 - e-ar ]
3

3e0a
r0
Er = [1 - e-ar ]
3
or 2
3e0ar

r0
now when ar 3 W 1, Er L
3e0ar 2
r0r
and when ar 3 V 1, Er L
3e0
3.28 Using Gauss’ theorem we can easily show that the electric field
strength within a uniformly charged sphere is

E = a b r
r r¢
3e0 r+ r–
a
The cavity, in our problem, may be considered as the superposition
of two balls, one with the charge density r and the other with - r.
338 PART THREE ELECTRODYNAMICS

Let P be a point inside the cavity such that its position vector with respect to the cen-
ter of cavity be r- and with respect to the center of the ball be r + . Then from the
principle of superposition, field inside the cavity, at an arbitrary point P is given by
E = E+ + E-
r r
= (r + - r - ) = a
3e0 3e0

Note: The expression obtained for E shows that it is valid regardless of the ratio bet-
ween the radii of the sphere and the distance between their centers.

3.29 Let us consider a cylindrical Gaussian surface of radius r and height h inside an infi-
nitely long charged cylinder with charge density r. Now from Gauss ’ theorem
qenclosed
Er 2prh =
e0

(where Er is the field inside the cylinder at a distance r from its axis) r+
rpr 2h rr r– h
or Er 2prh = Q Er =
e0 2e0 r

Now, using the method of Problem 3.28, field at a point P inside the
cavity is
r r
E = E + + E– = (r + - r–) = a
2e0 2e0
3.30 The arrangement of the rings is as shown in the figure. Now, potential at the point
1 is equal to sum of potential at 1 and due to the ring 1 and potential at 1 due to the
ring 2. So,
q - q +q –q
w1 = +
4pe0R 4pe0 (R 2 + a 2) 1/2
R R
Similarly, the potential at point 2 a
1 2
–q q
w2 = +
4pe0R 4 pe0 (R 2 + a 2) 1/2
Hence, the sought potential difference
-q
b +
q
w1 - w2 = ¢w = 2a
4pe0R 4pe0(R 2 + a 2) 1/2

a1 - b
q 1
=
2pe0R 21 + (a/R) 2
3.1 CONSTANT ELECTRIC FIELD IN VACUUM 339

3.31 We know from Gauss ’ theorem that the electric field due to an infinitely long straight
wire, at a perpendicular distance r from it is
l
Er = .
2pe0r
So, the work done is
2 hx
l
Erdr = dr
3 3 2pe0r
1 x
(where x is perpendicular distance from the thread by which point 1 is removed from it).
l
Hence, ¢w12 = ln h
2pe0
= 5 kV (on substituting values)

3.32 Let us consider a ring element as shown in the figure. Then the charge carried by the
element, dq = (2pR sin u) Rd u s.
Hence, the potential due to the considered element at the center of the hemisphere is

1 dq 2 psR sin ud u sR
dw = = = sin ud u
4 pe0 R 4pe0 2e0

So potential due to the whole hemisphere is y


p/2
Rs sR
w = sin u d u =
2e0 3 2e0 d
0
x
O R
Now from the symmetry of the problem, net electric field of
the hemisphere is directed towards the negative y-axis. So, we have

1 dq cos u s
dEy = 2
= sin u cos u d u
4pe0 R 2e0

p/2 p/2
s s s
Thus, E = E y¿ = sin u cos u d u = sin 2ud u = (along yO )
2e0 3 4e0 3 4e0
0 0

3.33 Let us consider an elementary ring of thickness dy and radius y as shown in the fig-
ure. Then potential at a point P, at distance l from the center of the disk is

s 2 pydy
dw =
4pe0 (y 2 + l 2) 1/2
340 PART THREE ELECTRODYNAMICS

Hence potential due to the whole disk is given by dy


R
s 2pydy sl y E
w = = ( 21 + ( R/l ) 2 - 1)
3 4pe0 (y2 + l 2) 1>2 2e0 l P
0
From symmetry
dw
E = El = -
dl
c - 1d = c1 - d
s 2l s 1
= -
2e0 2 2R + l
2 2 2e0 21 + (R/l ) 2

sR s
when l : 0, w = and E =
2e0 2e0
sR 2 sR 2
when l W R, w L and E L
4e0l 4e0l 2

3.34 By definition the potential in the case of a surface charge distribution is defined by
1 sdS
w = (1)
4pe0 3 r
In order to simplify integration, we shall choose the area element dS in the form of
a part of the ring of radius r and width dr (see figure). Then dS = 2ur dr, r = 2R
cos u and dr = - 2R sin u du . After substituting these expresion into integral in Eq. (1),
we obtain the expression for w at the point O as
0
sR
w = - u sin u du r
pe0 3
p>2 dr
O
We integrate by parts, denoting u = u and sin u du = dv to get

u sin u du = - u cos u + cos u d u


3 3
= - u cos u + sin u
which gives - 1 after substituting the limits of integration. As a result, we obtain
sR
w =
pe0

3.35 In accordance with the problem w = a # r. Thus from the equation E = - §w , we


get
3.1 CONSTANT ELECTRIC FIELD IN VACUUM 341

0 0 0
E = -c (ax x) i + (ay y) j + (a z) k d = - [ax i + ay j + azk] = - a
0x 0y 0x z

3.36 (a) Given, w = a (x 2 - y 2)

So, E = - §w = - 2a (x i - y j)
The sought shape of field lines is as shown in the figure (a) of answer sheet,
assuming a 7 0.
(b) Since, w = axy
So, E = - §w = - ay i - ax j
The plot of field lines is as shown in the figure (b) of answer sheet.

3.37 Given, w = a (x 2 + y 2) + bz 2

So, E = - §w = - [2 ax i + 2ay j + 2 bz k]

Hence, | E | = 2 2a 2 (x 2 + y 2) + b 2z 2

Shape of the equipotential surface:


Put r = xi + yj or r2 = x 2 + y 2
Then the equipotential surface has the equation
ar2 + bz 2 = constant = w

If a 7 0, b 7 0, then w 7 0 and the equation of the equipotential surface is


r z2
+ = 1
w>a w>b
which is an ellipse in r, z coordinates. In three dimensions, the surface is an ellipsoid
of revolution with semi-axes 1w>a and 1w>b .
If a 7 0, b 6 0, then w can be Ú 0. If w 7 0 then the equation is
r2 z2
- = 1
w>a w>| b |
This is a single cavity hyperboloid of revolution about z axis.
If w = 0, then the equation can be written as

ar2 - | b | z 2 = 0
a
or z =  r
A|b|
342 PART THREE ELECTRODYNAMICS

This is the equation of a right circular cone.


If w 6 0, then the equation can be written as
| b | z 2 - ar2 = | w |
z2 r2
- = 1
|w|> |b| | w | >a
or

This is a two cavity hyperboloid of revolution about z-axis.

3.38 From Gauss’ theorem, intensity at a point inside the sphere, at a distance r from the
rr 1 q
center is given by, Er = and outside it, is given by Er = .
3e0 4pe0 r 2
(a) Potential at the center of the sphere
q R q
q r R2 q
E # dr =
rr
w0 = dr + 2
dr = +
3 3 3e0 3 4pe0r 3e 2 4pe0R
0 0 R

a as r = b
q q 3q 3q
= + =
8pe0R 4pe0R 8pe0R 4pR 3

(b) Now, potential at any point inside the sphere, at a distance r from it’s center
R q
r q dr
w(r) = r dr + 2
3 3e0 3 4pe0 r
r r

c1 - d = w0 c 1 - d
3q r2 r2
On integration, w(r) = 2
8e0R 3R 3R 2

3.39 Let two charges + q and - q be separated by a distance l. Then electric potential at
a point at distance r W l from this dipole

+q -q r- - r+
a b
q Er
w(r) = + = (1)
4pe0r + 4pe0r- 4pe9 r+ r- E
r+
+q
E
q r
But r- - r+ L l cos u and r+ r- L r 2 (2)
l O
r–
From Eqs. (1) and (2),
–q
ql cos u p cos u p#r
w(r) = = w =
4pe0r 2 4pe0r 2 4pe0r 3
3.1 CONSTANT ELECTRIC FIELD IN VACUUM 343

where p is the electric moment vector.

- 0w 2 p cos u
Now, Er = =
0r 4pe0r 3

0w p sin u
and Eu = - =
r 0u 4pe0r 3

p p
So, E = 2E 2r + E 2u = 24 cos2 u + sin2 u = 21 + 3 cos2 u
4pe0 r3 4pe0r 3

3.40 From the results obtained in the previous problem

2p cos u p sin u
Er = and Eu =
4pe0r 3 4pe0r 3

From the given figure in the probem book, it is clear that,


p
Ez = Er cos u - Eu sin u = (3 cos 2 u - 1)
4pe0r 3

3 p sin u cos u
and E ⬜ = Er sin u + Eu cos u =
4pe0r 3
When, E⬜ p, | E | = E ⬜ and Ez = 0
1
So, 3 cos 2 u = 1 and cos u =
13
Thus E⬜ p at the points located on the lateral surface of the cone, with its axis coin-
ciding with the direction of z-axis and semi vertex angle u = cos -1 1> 13.

3.41 Let us assume that the dipole is at the center of the one equipotential surface which
is spherical (see figure). On an equipotential surface, the net electric field strength
along the tangent of it becomes zero. Thus,
p sin u
- E0 sin u + Eu = 0 or - E0 sin u + = 0
4pe0r 3

where E is electric field generated by the dipole.

r = a b
p 1>3
Hence,
4pe0E0
344 PART THREE ELECTRODYNAMICS

Alternate:
Potential at the point, near the dipole is given by
E0
p#r
w = - E0 # r + constant,
4pe0r 3
t Er
= a - E0 b cos u + constant
p
E
4pe0r 3
For w to be constant,
p p
- E0 = 0 or = E0
4pe0r 3 4pe0r 3

r= a b
p 1>3
Thus,
4pe0E0

3.42 Let P be a point, at distance r W l and at an angle u to the vector l (see figure).
Thus E at P
l l
r + r -
l 2 l 2
= -
`r + ` `r - `
2pe0 l 2 2pe0 l 2
2 2
r + l/2 r - l/2
l -
= C l 2 l2 S
2pe0 r 2 + + r l cos u r2 + - r l cos u P
4 4

a 2 - 3 cos u b
l l 2l r r
=
2pe0 r r
A O B
ll
Hence, E = |E| = (for r W l )
2pe0r 2

ln | r + l >2 | - ln | r - l >2 |
l l
Also, w =
2pe0 2pe0

l r 2 + rl cos u + l 2 >4 ll cos u


= = (for r W l )
r - rl cos u + l >4
ln 2 2
4pe0 2pe0r

3.43 The potential can be calculated by superposition. Choose the plane of the upper ring
as x = l/2 and that of the lower ring as x = - l >2.
q q
Then, w = -
4 pe0 [R 2 + (x - l/2) 2] 1/2 4pe0 [R 2 + (x + l/2) 2] 1/2
3.1 CONSTANT ELECTRIC FIELD IN VACUUM 345

q q
L -
4pe0 [R 2 + x2 - lx] 1/2 4pe0 [R 2 + x 2 + lx] 1/2

a1 + b - a1 - b
q lx q lx
L
4pe0 (R 2 + x 2) 1/2 2(R 2 + x )
2 4pe0 (R + x )
2 2 1/2 2(R + x 2)
2

qlx
L
4pe0 (R 2 + x 2) 3/2
ql
For | x | W R, w L .
4pe0x 2
The electric field is
0w
E = -
0x
ql 3 ql ql (2x 2 - R 2)
= - + * 2x =
4pe0 (R 2 + x 2) 3/2 2 (R 2 + x 2) 5/2 4pe0 4pe0 (R 2 + x 2) 5/2
ql
For | x | W R, E L .
2pe0x 3
The plot is as shown in the answer sheet.

3.44 The field of a pair of oppositely charged sheets with holes can by superposition be
reduced to that of a pair of uniform opposite charged sheets and disks with opposite
charges. Now the charged sheets do not contribute any field outside them. Thus, using
the result of the previous problem
R R2+x2
(- s) l 2prdr x sxl dy
w = = - (putting y = x 2 + r 2 ) -s
3 4pe0 (r + x )
2 2 3/2 4e0 3 y 3/2
0 2 x

sxl
=
2e0 2R 2 + x 2 +s l

0 w
c d
sl 1 x2 slR 2
Ex = - = - - = -
0 x 2e0 2R 2 + x 2 (R 2 + x 2) 3/2 2e0(R 2 + x 2) 3/2

The plot is as shown in the answer sheet.

3.45 For x 7 0 , we can use the result as given above and write

|x |
a1 - b
sl
w L 
2e0 (R + x 2) 1/2
2
346 PART THREE ELECTRODYNAMICS

for the solution that vanishes at a. There is a discontinuity in potential for | x | = 0.


The solution for negative x is obtained by s : - s. Thus,
slx
w = - + constant
2e0(R + x 2) 1/2
Hence, ignoring the jump
0w slR 2
E = - =
0x 2e0(R 2 + x 2) 3/2
For large | x |,
p p
w L  and E L (where p = pR 2 sl)
4pe0 x2 2pe0| x | 3
3.46 In cylindrical coordinates r, u, z,
l
Er = , Eu = 0, Ew = 0
2pe0r
0E
and F =p
0l
or (p # § )E

(a) For p along the thread, F = 0


as E does not change as the point of observation is moved along the thread.
0 (E r er ) 0 Er 0er
(b) For p along r F = pr = p a b er abecause = 0b
0r 0r 0r
lp lp
= er = -
2pe0 r2 2pe0r 2
0 (Er er ) 0 0
(c) For p along e u F = pu = p a eb
r0u r 0u 2pe0r r
0er 0 er
a because = eu b
pl pl pl
= = eu =
2 pe0 r2 0u 2pe0r 2 2pe0 r2 0u

3.47 Force on a dipole of moment p is given by

F = `p `
0E
0l
In our problem, field due to a dipole at a distance l is given by

p
|E| =
2pe0l 3
3.1 CONSTANT ELECTRIC FIELD IN VACUUM 347

Hence, the force of interaction


3p2
F = = 2.1 * 10-16 N
2pe0l 4

3.48 Given, - dw = E # dr = a (y dx + xdy) = ad (xy)

On integrating, w = - a xy + constant

3.49 Given, - dw = E # d r = [2 a xy i + 2(x 2 - y 2) j] # [dx i + dy j]

or dw = 2a x y dx + a 1x 2 - y 22dy = ad 1x 2y2 - ay 2 dy

y2
On integrating, w = ay a - x 2 b + constant
3

3.50 Given,

- d w = E # d r = 1ay i + 1ax + bz2 j + by k2 # 1dx i + dy j + dz k2


= a 1y dx + ax dy2 + b 1zdy + ydz2 = ad 1 xy2 + bd 1yz2

On integrating, w = - 1a xy + b yz2 + constant

3.51 Field intensity along x-axis


0w
Ex = - = 3 ax 2
0x
Then using Gauss’ theorem in differential form

wEx
r 1x2 = 6a e0x
r1x2
= so,
wx e0

3.52 In the space between the plates, we have by the Poisson equation
0 2w r0
= -
0x 2 e0

r0
or w = - x 2 + Ax + B
2e0

where r0 is the constant space charge density between the plates.


We can choose w (0) = 0 so, B = 0
348 PART THREE ELECTRODYNAMICS

r0d 2 ¢w r0d
Then w (d ) = ¢w = Ad - or A = +
2e0 d 2e0

0w r0
Now, E = = x - A = 0 (for x = 0)
0x e0

¢w r0d
if A = + = 0
d 2e0

2e0 ¢w
then, r0 = -
d2
r0d
Also, E 1d2 =
e0

3.53 As w = w(r), so field intensity is along the radial line and is given by
0w
Er = - = - 2ar (1)
0r

Due to spherical symmetry of field of E, electric flux over a spherical Gaussian sur-
face of radius r is 4pr 2Er .
From Gauss’ theorem,
q
4pr 2Er =
e0

On differenting,
dq
4p d (r 2 Er ) =
e0
where dq is the charge contained between the sphere of radii r and r dr .
Therefore, dq = r # 4pr 2dr

r # 4pr 2 dr
So, 4p 1r 2dEr + 2rEr dr2 =
e0
0Er 2 r
+ Er =
0r r e0
2 r
- 2a + ( - 2ar) =
r e0
Hence, r = - 6e0a
3.2 CONDUCTORS AND DIELECTRICS IN AN ELECTRIC FIELD 349

Alternate:
Here E = - §w = - 2ar er = Er er , where er is unit vector towards radius vector
and Er = - 2ar.
From Gauss’ law in the differential form

§#E =
r
e0
1 0 2 r
(r Er) =
r 0r
2 e0

1 0 2 r
- (r 2ar) = (using Er = - 2ar )
r 0r
2 e0

Hence, r = - 6e0a

3.2 Conductors and Dielectrics in an Electric Field

3.54 When the ball is charged, for the equilibrium of ball, electric force on it must counter
balance the excess spring force exerted on the ball due to the extension in the spring.
Thus, Fele = Fspring
q2
or = kx
4pe0 (2l )2 +q
l
(the force on the charge q might be considered to arise from attraction
by the electrical image),
l
–q
or q = 4> 1pe0kx
which is the sought charge on the sphere.

3.55 By definition, the work of this force done upon an elementary


displacement dx (see figure) is given by
q2 F q
dA = Fxdx = - dx, x
4pe0 (2x)2 x dx

where the expression for the force is obtained with the help of
the image method, Integrating this equation over x between l
and q, we find
q
q2 dx q2
A = - = -
16 pe0 3 x 2 16pe0l
l
350 PART THREE ELECTRODYNAMICS

3.56 (a) Using the concept of electrical image, it is clear that the magnitude of the force
acting on each charge,
q2 q2 Fq F–q
|F | = 12 - +q
l/2
–q
4pe0l 2 4pe0 ( 12 l )2 P
l /2 Fq
q2
= (2 12 - 1)
8pe0l 2

(b) Also, from the figure, magnitude of electrical field


strength at P –q l +q

E = 2 a1 - b
1 q
5 15 pe0l 2

3.57 Using the concept of electrical image, it is easily seen that


the force on the charge q is –q l l q
12 q 2 ( - q)2 l
F = +
4pe0 (2l)2 4pe0 (2 12 l)2
(2 12 - 1) q 2
= (It is an attractive force.) l
32pe0l 2
q –q
2l
3.58 Using the concept of electrical image, force on the dipole p,
0E
F = p
0l
(where E is field at the location of p due to (– p))
0E 3p2
|F| = p ` ` =
p –p
or
0l 32pe0l 4 l l

p
as |E| =
4 pe0 (2l )3

3.59 To find the surface charge density, we must know the electric field at the point P (see
figure) which is at a distance r from the point O.
Using the image mirror method, the field at P,
q 1 ql
E = 2E cos a = 2 =
4pe0x 2 x 2pe0(l + r 2)3>2
2
3.2 CONDUCTORS AND DIELECTRICS IN AN ELECTRIC FIELD 351

Now from Gauss’ theorem, the surface charge density +q


on conductor is connected with the electric field near its
surface (in vaccum) through the relation s = e0En, l P
where En is the projection of E onto the outward nor- r
O
mal n (with respect to the conductor). E– q E E+q

As our field strength E c T n, so –q

ql
s = - e0E = -
2p(l 2 + r 2)3>2

3.60 (a) The force F1 on unit length of the thread is


given by

F1 = lE1 x
l
where E1 is the field at the thread due to image
charge given by
-l
E1 =
2pe0 (2l )
l2
Thus, F1 = -
4pe0l

Here, minus sign means that the force is one of attraction.

(b) There is an image thread with charge density - l behind the conducting plane.
We calculate the electric field on the conductor considering the thread and its
image. It is
ll
E (x) = En(x) =
pe0 (x 2 + l 2)
ll
Thus, s(x) = e0En =
p (x 2 + l 2)

3.61 (a) At O
q
l dx l dx
En(O) = 2 2
=
3 4pe0x 2pe0l x
l
O r
l
So, s (O) = e0En =
2pl
352 PART THREE ELECTRODYNAMICS

q q
ldx x l x dx
(b) En (r) = 2 =
3 4pe0 (x + r ) (x + r ) 2pe0 3 (x + r 2)3>2
2 2 2 2 1>2 2
l l
q
l dy
= (putting y = x 2 + r 2)
4pe0 3 y3>2
2 2l +r

l
=
2pe0 1l 2 + r 2

Hence,
l
s (r) = e0En =
2p 1l 2 + r 2

3.62 (a) It can be easily seen that in accordance with the image method, a charge - q must
be located on a similar ring but on the other side of the conducting plane (see
figure) at the same perpendicular distance. From the solution of Problem 3.9,
electric field strength on axis of a ring of radius R at a distance l from it center is
ql
4pe0 (R 2 + l 2) 3>2
In our case this expression must be doubled. Hence, net field strength at point O

ql
E = 2 R q
4pe0(R 2 + l 2)3>2 ( - n)
l
O
where n is unit vector outward normal to the conducting plane.

s
Now, En = –q
e0

- ql
Hence, s =
2 p(R 2 + l 2) 3>2

where minus sign indicates that the induced charge is opposite in sign to that
of charge q 7 0.

(b) The net field strength at the center of the ring is vector sum of field strengths setup
by the charges q and - q and is given by
q (2l)
E = 0 + ( - n) (using result of Problem 3.9)
4pe0 [R 2 + (2l ) 2] 3>2
3.2 CONDUCTORS AND DIELECTRICS IN AN ELECTRIC FIELD 353

The potential at the center of the ring is equal to the algebraic sum of the poten-
tials at this point created by the charges q and - q , given by
q q
w = -
4pe0 R 4pe0 2R 2 + (2l)2

a - b
1 q q
=
4pe0 R 2R 2 + 4l 2

3.63 Potential w is the same for all the points of the sphere. Thus we can calculate its value
at the center O of the sphere, because only for this point, it can be calculated in the
most simple way.
So, 1 q
w = + w¿ (1)
4pe0 l
where the first term is the potential of the charge q, while the
second is the potential due to the charges induced on the sur-
face of the sphere. But since all induced charges are at the
same distance equal to the radius of the circle from the point +q O
C and the total induced charge is equal to zero, so w¿ = 0 .
Thus Eq. (1) is reduced to the form,
1 q
w =
4pe0 l

3.64 Point charge q is not placed at the center of the sphere, so the induced negative
charge - q on the inner surface is not distributed uniformly. The left over charge + q
on the outer surface of the conductor distributes itself uniformly so that the field
strength due to the left over charge at inside point of the conductor becomes zero.
Hence, the potential at O is given by
q 1 ( - s) ds q
w0 = + + R1
4pe0r 4pe0R1 4pE0R2 q
o r
q (- q) q R2
w0 = + +
4pe0r 4pe0R1 4e0pR2 –q
q

It should be noticed that the potential can be found in such a simple way only at O,
since all the induced charges on the inner surface are at the same distance from this
point, and their distribution (which is unknown to us) does not play any role.

3.65 Potential at the inside sphere is given by


q1 q2
wa = +
4pe0a 4pe0b
b
Obviously, wa = 0 for q2 = - q1 (1)
a
354 PART THREE ELECTRODYNAMICS

When r Ú b,
q1 q2 q1
a1- b 冫 r
b
wr = + = (using Eq. 1)
4pe0r 4pe0 4pe0 a

And when r … b ,
q1 q2 q1
a - b
1 1
wr = + =
4pe0r 4pe0b 4pe0 r a

3.66 (a) As the metallic plates 1 and 4 are isolated and connected by means of a conduc-
tor, w1 = w4. Plates 2 and 3 have the same amount of positive and negative
charges and due to induction, plates 1 and 4 are respectively negatively and pos-
itively charged and in addition to it all the four plates are located at a small but
equal distance d relative to each other. Hence, electric field strength between 1–2
and 3–4 is the same (say E). Let E be the field strength between the plates 2
and 3, which is directed from 2 to 3. Hence E c T E (see figure).
According to the problem
E ¿d = ¢w = w2 - w3 (1)
In addition, O 1
w1 - w4 = (w1 - w2) + (w2 - w3) + (w3 - w4) E +
2
E'
or 0 = - Ed + ¢w - Ed 3

¢w E¿ E
or ¢w = 2Ed Q E = = (2) 4
2d 2 x
Distribution of potential w is also shown in the figure.

(b) Since E r s, we can state according to Eq. (2) for part (a), that the charge on
rge on the plate 2 is divided into two parts; such that 1/3rd of it lies on the
upper side and 2/3rd on its lower face.
Thus charge density of upper face of plate 2 or of plate 1 or plate 4 and lower
face of 3 is
e0 ¢w
s = e0E =
2d
and charge density of lower face of 2 or upper face of 3 is
¢w
s¿ = e0E ¿ = e0
d
Hence, the net charge density of plate 2 or 3 becomes
3e0 ¢w
s + s¿ =
2d
which is obvious from the argument.
3.2 CONDUCTORS AND DIELECTRICS IN AN ELECTRIC FIELD 355

3.67 The problem of point charge between two conducting planes is more easily tackled
(if we want only the total charge induced on the planes) if we replace the point charge
by a uniformly charged plane sheet.
Let s be the charge density on this sheet and E1, E2 be the outward electric field on the
on the two sides of this sheet.
s
Then, E1 + E2 =
e0
The conducting planes will be assumed to be grounded.
Then, E1x = E2 (l - x)
q
s s
Hence, E1 = (l - x) and E2 = x
le0 le0
This means that the induced charge densities on the plane conductors are
s s
s1 = - (l - x) and s2 = - x
l l
q q
Hence, q1 = - (l - x) and q2 = - x
l l

3.68 Near the conductor


s
E = En
e0
This field can be written as the sum of two parts E1 and E2. E1 is the electric field due
to an infinitesimal area dS. Very near it
s
E1 = 
2e0
The remaining part contributes
s
E2 = (on both sides)
2e0
In calculating the force on the element dS, we drop E1 (because it is a ds En
self-force). Thus,
dF s s2
= s =
dS 2e0 2e0
3.69 The total force on the hemisphere is
p>2
s2
F = . cos u # 2pR sin uR du
3 2e0
0
p>2
2p R 2s2
= cos u sin u d u
2e0 3
0
356 PART THREE ELECTRODYNAMICS

q2
* * a b
2pR 2 1 q 2
= =
2e0 2 4pR 2 32pe0R 2
= 0.5 kN (on substituting values)

3.70 We know that the force acting on the area element dS of a conductor is,
s2
dF = dS (1)
2e0
If follows from symmetry considerations that the resultant force F is directed along the
z-axis, and hence, it can be represented as the sum (integral) of the projection of el-
ementary forces given in Eq. (1) onto the z-axis, so

dFz = dF cos u (2)

For simplicity, let us consider an element area dS = 2pR sin u du (see figure).
Now, Eq. (2) takes the form
z
ps2R 2 F
dFz = sin u cos u du dF
e0
ps20R 2 ds
= -a b cos3 u d cos u
R
e0 O

Integrating this expression over the half sphere, i.e., with respect to
cos u between 1 and 0, we obtain

ps20R 2
F = Fz =
4e0

3.71 The total polarization is P = (e - 1) e0E. This must be equal to n0p / N , where n0 is
the concentration of water molecules. Thus,

n0P
N = = 2.93 * 103 (on substituting values)
(e - 1) e0E

3.72 From the general formula in vector form


1 (3p # er) er - p
E =
4pe0 r3
1 2p
E = (where r = l and r c c p)
4pe0 l 3
3.2 CONDUCTORS AND DIELECTRICS IN AN ELECTRIC FIELD 357

This will cause the induction of a dipole moment


1 2p
pind = b * e0
4pe0 l 3
Thus, the force
b 2p d 1 2p 3bp2
|F| = =
4p l 3 dl 4pe0 l 3 4p2e0 l 7

3.73 The electric field E at distance x from the center of the ring is
qx
E (x) =
4pe0(R 2 + x 2 )3>2
The induced dipole moment is
q bx
p = be0E =
4p(R 2 + x 2)3>2
The force on this molecule is
0 q bx q 0 x q 2 b x (R 2 - 2x 2)
F =p E= =
0x 4p(R 2 + x 2)3>2 4pe0 0x (R 2 + x 2)3>2 16p2e0 (R 2 + x 2)
R
This vanishes for x = (apart from x = 0, x = q).
12
It is maximum when
0 x (R 2 - x 2 * 2)
= 0
0x (R 2 + x 2)4
or (R 2 - 2x 2)(R 2 + x 2) - 4x 2(R 2 + x 2) - 8x 2 (R 2 - 2x 2) = 0
R2
or R 4 - 13x 2R 2 + 10x 4 = 0 or x 2 = (13  2129)
20
R
or x = 313 2129 (on either side).
220
Plot of Fx(x) is as shown in the answer sheet.

3.74 Inside the ball


q r
D (r) = = e e0E
4p r 3
e - 1 e - 1 q r
Also e0E + P = D or P = D =
e e 4p r 3
e - 1 q e - 1
Also, q ¿ = - P # dS = - dÆ = q
C e 4p L e
358 PART THREE ELECTRODYNAMICS

s
3.75 Ddiel = ee0Ediel = Dconductor = s or Ediel =
ee0
e - 1
Pn = (e - 1)e0 Ediel = s
e
e - 1
s¿ = - Pn = - s
e
This is the surface density of bound charges.
3.76 From the solution of the Problem 3.74, charge on the interior surface of the conduc-
tor is given by
e - 1 e - 1
q ¿ in = - s dS = - q
e L e
Since the dielectric as a whole is neutral, there must be a total charge equal to
e - 1
q ¿ outer = + q
e
on the outer surface of the dielectric.

3.77 (a) Positive extraneous charge is distributed uniformly over the internal surface layer.
Let s0 be the surface density of the charge.

Clearly, for r 6 a E = 0

By Gauss’ theorem:

For a 6 r , e0E * 4pr 2 = 4pa2s0

s0
, a b (a 6 r 6 b )
a 2
or E =
e0 e r
For r 7 b, similarly
s0
a b
a 2
E =
e0 r
0w
Now, E = -
0r
So by integration from infinity, where w 1q2 = 0, we get
s0a 2
w = (r 7 b )
e0r
s0a2
For a 6 r 6 b, w = + B (where B is a constant)
eer
3.2 CONDUCTORS AND DIELECTRICS IN AN ELECTRIC FIELD 359

s0a2 s0a2
a - b +
1 1
By continuity, w =
e0e r b e 0b

For r 6 a, w = A = constant
s0a2 s0a2
a - b +
1 1
By continuity, w =
e0e a b e0b

(b) Positive extraneous charge is distributed uniformly over the internal volume of the
dielectric. Let r0 = volume density of the charge in the dielectric, for a 6 r 6 b.
Then,

E = 0 (for r 6 a)
4p 3
and e0 e 4pr 2E = (r - a 3) r0 (for a 6 r 6 b)
3
r0
ar -b
a3
or E =
3 e0e r2
4p 1b - a 2r0
3 3
E = (for r 7 b )
3 e0 4pr 2

1b 3 - a 32r0
or E = (for r 7 b )
3e0r 2
By integration,
1b 3 - a 32
w = (for r 7 b )
3e0r

r0
a b (for a 6 r 6 b )
r2 a3
or w = B - +
3e0e 2 r
By continuity,
b3 - a3 r0 b 2
a b
a3
r0 = B - +
3e0b 3e0e 2 b

r0 e1b 3 - a 32
e + a
bf
b2 a3
or B = +
3e0e b 2 b
r0 r0 a 2
ea + a2b = B -
a2
Finally w = B - (for r 6 a)
3e0e 2 2e0e

On the basis of obtained expressions, E 1r2 and 1w2 1r2 can be plotted as shown
in the answer sheet.
360 PART THREE ELECTRODYNAMICS

3.78 Let us take points 1 and 2 just left and right of the interface. Point 2 can be treated like A
given in the problem book. Continuity of tangential component of E at the interface gives
E1t = E2t
E1t = E0 sin a 0
As there is no free charge at the interface, so normal component of D has continuity
at the interface.
Hence, D1n = D2n
e0eE1n = e0 E0 cos a 0 n
E0 cos a0 2 A0
E 1n = 1 E0
e
E
So, E 1 = 2E 1t 2 + E 1n 2
E 1t
tan a = = e tan a0
E 1n
From the boundary condition of polarization vector P, we get
P2n–P1n = - s¿
0 - P1n = - s¿
P1n = s¿
e0(e - 1) E 1n = s¿
E0 cos a0
e0 (e - 1) = s¿
e
e0 (e - 1)
s¿ = E0 cos a 0
e
3.79 From the previous problem, we know
e - 1
s¿ = e0 E 0 cos u
e

E0

n 1 Length l

R 2

e - 1
E # dS =
1
(a) Then, Q = pR 2 E 0 cos u
I e0 e
3.2 CONDUCTORS AND DIELECTRICS IN AN ELECTRIC FIELD 361

(b) D # dr = 1D1t - D2t2 l = 1e0E0 sin u - ee0E0 sin u2


I
l = - 1e - 12 e0E0l sin u

0Dx
3.80 (a) div D = = r and D = rl
0x
rl
Ex = (for l 6 d )
ee0
rd
and Ex = constant (for l 7 d )
e0

(for l 6 d ) and w 1x2 = A -


rl 2 rld
Also, w1x2 = - (for l 7 d )
2ere0 e0

w 1x2 = ad -
rd d
By continuity, - 1b
e0 2e

On the basis of obtained expressions, E x (x) and w (x) can be plotted as shown
in the figure of answer sheet.
(e - 1)
(b) r¿ = - div P = - div (e - 1) e0 E = - r
e
s¿ = P1n - P2n (where n is the normal from 1 to 2)
= P1n (P2 = 0, as 2 is vacuum)
e - 1
= ( rd - rd>e) = rd
e

l 0 2
3.81 (a) div D = (r Dr ) = r
r 2 0r
r3 1 A
r 2Dr = r + ADr = rr + 2 , r 6 R
3 3 r
A = 0 as Dr Z q at r = 0,
rr
Thus, Er =
3ee0
B
For r 7 R, Dr =
r2
By continuity of Dr at r = R,
rr 3
B =
3
362 PART THREE ELECTRODYNAMICS

rR 3
So, Er = (for r 7 R )
3e0r 2
rR 3 rr 2
w = (for r 7 R ) and w = - + C (for r 6 R )
3e0r 6ee0

rR 2 rR 2
By continuity of w C = + +
3e0 6ee0
See answer sheet for graphs of E (r) and w (r).
1 0 r3 r (e - 1)
e r a1 - b f = -
1
(b) r¿ = div P = -
r 0r 3
2 e e

rR a 1 - b
1 1
s¿ = P1r - P2r = P1r =
3 e

3.82 Because there is a discontinuity in polarization at the boundary of the dielectric disk,
a bound surface charge appears, which is the source of the electric field inside and
outside the disk. We have for the electric field at the origin,

s¿dS
E = - 3
r
3 4pe0r
where r = radius vector to the origin from the element dS. s¿ = Pn = P cos u on the
curved surface (as Pn = 0 on the flat surface). Here u = angle between r and P. By
symmetry, E will be parallel to P.

2p
P cos u Rd u # cos u #
Thus, E⬵ - d
3 4pe0r 2
0
where, r = R , if d V R.

Pd
So, E⬵ -
4e0R

Pd
and E = -
4e0R

3.83 Since there are no free extraneous charges anywhere


0Dx
div D = = 0 or Dx = constant
0x
But Dx = 0 at q so, Dx = 0, everywhere
3.2 CONDUCTORS AND DIELECTRICS IN AN ELECTRIC FIELD 363

P0 P0
a1 - b a1 - b
x2 x2
Thus, E = - or Ex = -
e0 d2 e0 d2
P0x P0x 3
So, w = - + constant
e0 3e0d 2
2P0d 2P0d 3 4P0d
Hence, w ( + d) - w ( - d) = - =
e0 3d 2e0 3e0

3.84 (a) We have D1 = D2 or eE2 = E1


d d
Also, + E2 = E0 d Q E1 + E2 = 2E0
E1
2 2
2E0 2eE0
Hence, E2 = and E1 =
e + 1 e + 1
2ee0E0
and D1 = D2 =
e + 1
s
(b) We have D1 = D2 or e E2 = E1 = = E0
e0
E0
Thus, E1 = E0 and E2 =
e
and D1 = D2 = e0E0
3.85 (a) Constant voltage across the plates implies
E1 = E2 = E0, D1 = e0E0, D2 = e0eE0
(b) Constant charge across the plates implies
E1 = E2 D1 = e0E1, D2 = e e0E2 = eD1

2E0
So, E1(1 + e) = 2 E0 or E1 = E2 =
e + 1
1
3.86 At the interface of the dielectric and vacuum,
E1t = E2t
The electric field must be radial and
A 2
E1 = E2 = (for a 6 r 6 b)
e0er 2
A A
Now, q = (2pR 2) + (2pR 2)
R2 eR 2

= A a1 + b 2p
1
e
q
or E1 = E2 =
2pe0r 2(1 + e)
364 PART THREE ELECTRODYNAMICS

3.87 In air the forces are as shown. In K–oil,


r0
F : F ¿ = F>e and mg : mg a 1 - b
r T T
where r0 is the density of K–oil and r that of the material of F F
which the balls are made.
mg mg
Since the inclinations do not change
1 r0
= 1 -
e r
r0 1 e - 1
or = 1 - =
r e e
e
or r = r0
e - 1
= 1.6 g>cm3 (on substituting values)

3.88 Within the ball, the electric field can be resolved into normal and tangential components,
En = E cos u and Et = E sin u n
Then, Dn = ee0 E cos u
and Pn = (e - 1) e0 E cos u t
E
or s¿ = (e - 1) e0 E cos u
So, smax = (e - 1) e0 E,
= 3.5 nC>m2 (on substituting values)
and total charge of one sign,
1

q¿ = (e - 1) e0E cos u 2pR 2d (cos u)


3
0

= pR 2e0 (e - 1) E = 10 pC (on substituting values)


(Since we are interested in the total charge of one sign, we must integrate cos u from
0 to 1 only).

3.89 The charge is at A in the medium 1 and has an image point at A¿ in the medium 2.
The electric field in the medium 1 is due to the actual charge q at A and the image
charge q ¿ at A¿ . The electric field in 2 is due to a corrected charge q – at A. Thus, on
the boundary between 1 and 2
q¿ q
E1n = cos u - cos u
4pe0 r2 4pe0r 2
-q –
E2n = cos u
4pe0r 2
3.2 CONDUCTORS AND DIELECTRICS IN AN ELECTRIC FIELD 365

q¿ q
E1t = sin u + sin u
4pe0 r2 4pe0r 2
q–
E2t = sin u A' A
4pe0r 2 l l
2
The boundary conditions are
D1n = D2n and E1t = E2t
eq– = q - q¿
q– = q + q¿
2q e - 1
So, q– = and q¿ = - q
e + 1 e + 1
(a) The surface density of the bound charge on the surface of the dielectric is
s¿ = P2n = D2n - e0E2n = (e - 1) e0E2n
e - 1 q e - 1 ql
= - cos u = -
e + 1 2pr 2 e + 1 2pr 3
(b) Total bound charge is
q
e - 1 l e - 1
- q 2pxdx = - q
e + 1 3 2p ( l + x )
2 2 3>2 e + 1
0
3.90 The force on the point charge q is due to the bound charges. This can be calculated
from the field at this charge after extracting out the self field. This image field is
e - 1 q
Eimage =
e + 1 4pe0 (2l )2
e - 1 q2
Thus, F =
e + 1 16pe0l 2
q r1 q¿r2
3.91 Given EP = 3
+ (for P in 1)
4pe0r 1 4pr 32e0
r2 P
q– r1 r1
and EP = (for P in 2) A' A
4pe0r 31 l l
2q 2 1
where q– = and q¿ = q– - q
e + 1
In the limit l : 0
(q + q¿) r qr
EP = = (in either part)
4pe0r 3 2pe0(1 + e) r 3
q
Thus, EP =
2pe0 (1 + e)r 2
366 PART THREE ELECTRODYNAMICS

q
w =
2pe0 (1 + e)r

* e
q 1 in vacuum
D =
2pe0 (1 + e) r2 e in dielectric

q r2 q¿r1
3.92 EP = + (for P in 2)
4pe0er 32 4pe0r 31 1 2
P
q–r2
EP = (for P in 1) r1 r2
4pe0r 32
Using the boundary conditions, q
E1n = e E2n, E1t = E2t
This implies
q - eq ¿ = q – and q + eq ¿ = eq –
2q e - 1 q
So, q– = and q¿ =
e + 1 e + 1 e
Then, as earlier,
e - 1 #1
a b
ql
s¿ = #
2pr 3 e + 1 e

3.93 To calculate the electric field, first we note that an image charge will be needed to en-
sure that the electric field on the metal boundary is normal to the surface.
The image charge must have magnitude - q /e , so that the tangential component of the
electric field may vanish. Now,

a 2 b 2 cos u =
1 q ql
En = air
4pe0 er 2pe0er 3

(e - 1) ql E
Then, Pn = Dn - e0En = = s¿ q –q
2per 3 l
This is the density of bound charge on the surface.

3.94 Since the condenser plates are connected,


E1h + E2 (d - h) = 0 E2
E1 h d
and P + e0E1 = e0E2
P
or E1 + = E2
e0
3.2 CONDUCTORS AND DIELECTRICS IN AN ELECTRIC FIELD 367

Ph Ph
Thus, E2d - = 0 or E1 =
e0 e0d

a1 - b
P h
E2 = -
e0 d

3.95 Given P = ar, where r = distance from the axis. The space density of charges is
given by, r¿ = - div P = - 2a . On using these, we get

1 0 (r # r)
div r = = 2
r 0r

3.96 In a uniformly charged sphere


r0
E = r
3e0 r
+ –
The total electric field is dr
1 1
E = rr - (r - dr) r0
3e0 0 3e0

1 P
= r0 dr = - (where r dr = - P)
3e0 3e0

The potential outside is


P0 # r
a - b =
1 Q Q
w = (for r 7 R )
4pe0 r | r - dr | 4pe0r 3

4p 3
where P0 = - R r00r, is the total dipole moment.
3

3.97 The electric field E0 in a spherical cavity in a uniform dielectric of permittivity e is re-
lated to the far away field E, in the following manner. Imagine the cavity to be filled
up with the dielectric. Then there will be a uniform field E everywhere and a polar-
ization P, given by P = (e - 1) e0E.
Now take out the sphere making the cavity, the electric field
inside the sphere will be
P
-
3e0
P
By superposition, E0 - = E E0
3e0
E
368 PART THREE ELECTRODYNAMICS

1 1
or E0 = E + (e - 1) E = (e + 2) E
3 3

3.98 By superposition the field E inside the ball is given by


P
E = E0 -
3e0
On the other hand, if the sphere is not too small, then the macroscopic equation
P = (e - 1) e0 E must hold. Thus,

3E0
E a1 +
1
(e - 1)b = E0 or E =
3 e + 2
e - 1
Also, P = 3e0 E
e + 2 0

3.99 This is to be handled by the same trick as in Problem 3.96. We have effectively a two
dimensional situation. For a uniform cylinder full of charge, with charge density r0,
the electric field E at an inside point is along the (cylindrical) radius vector r and
is given by
1
E = rr
2e0
Since l 0 r r
div E = (rEr) = hence , Er = r
r 0r e0 2e0
Therefore the polarized cylinder can be thought of as two equal and opposite charge
distributions displaced with respect to each other. So,
1 1 1 P
E = rr - r (r - dr) = r dr = -
2e0 2e0 2e0 2e0
using P = - rdr (direction of electric dipole moment vector being from the negative
charge to positive charge).
3.100 As in solution of Problem 3.98, we can write
P
E = E0 -
2e0
Also, P = (e - 1) e0E

e + 1
So, Ea b = E0
2

2E 0 e - 1
or E = and P = 2 e0 E
e + 1 e + 1 0
3.3 ELECTRIC CAPACITANCE ENERGY OF AN ELECTRIC FIELD 369

3.3 Electric Capacitance. Energy of an Electric Field

3.101 Let us impart an imaginary charge q to the conductor, then


R2 q
q q
w+ - w- = dr + dr R2
3 4pe0 er 2 3 4pe0 r
2
–q
R1 R2
R1

c d +
q 1 1 q 1 +q
= -
4pe0 e R1 R2 4pe0 R 2
(e - 1)
c d
q 1
= +
4pe0 e R2 R1
Hence the sought capacitance,
q q 4pe0e 4pe0eR1
C = = =
w+ - w- (e - 1) R1
qc d
1
+ (e - 1) + 1
R2 R1 R2

3.102 From the symmetry of the problem, the voltage across each capacitor ¢w = j>2 and
charge on each capacitor q = C j>2 in the absence of dielectric.
Now when the dielectric is filled up in one of the capacitors, the equivalent capac-
itance of the system is given by
Ce
C ¿0 =
1 + e
and the potential difference across the capacitor, which is filled with dielectric is
q¿ Ce j j
¢w¿ = = =
eC (1 + e) Ce (1 + e)
But w r E .
So, as w decreases 1/ 2 (1 + e) times, the field strength also decreases by the same
factor and flow of charge
¢q = q ¿ - q
Ce C 1 (e - 1)
= j - j = Cj
(1 + e) 2 2 (e + 1)

3.103 (a) Since it is a series combination of two capacitors,


1 d1 d2 e0S
= + or C =
C e0e1S e0e 2S (d1/e1) + (d 2/e 2)
370 PART THREE ELECTRODYNAMICS

(b) Let s be the initial surface charge density, then density of bound + −
charge on the boundary plane is given by

s¿ = s a 1 - b - s a1 - b = sa - b
1 1 1 1
e1 e2 e2 e1 1 2

q CV e0S e1e 2 V
But, s = = =
S S e 2d1 + e1d 2 S d1 d2
e0V (e1 - e 2)
So, s¿ =
e 2d1 + e1d 2

3.104 (a) We point the x-axis towards the right and place the origin on the left hand side plate.
The left hand side plate is assumed to be positively charged. Since e varies lin-
early, we can write, + –
e(x) = a + bx
where a and b can be determined from the boundary condition. O x
We have,
e = e1 at x = 0 and e = e 2 at x = d
d
e 2 - e1
Thus, e (x) = e1 + a bx
d

Now potential difference between the plates


d d

E # dr =
s
w+ - w- = dx
3 3 e0e (x)
0 0
d
s sd e2
= dx = ln
e 2 - e1 (e 2 - e1) e0 e1
e0 a e1 + xb
3
0
d
Hence, the sought capacitance,
sS (e 2 - e1) e0S
C = =
w+ - w- (ln e 2 /e1) d
q q q
(b) Since, D = and P = -
S S Se (x)
and the space density of bound charge is
q (e 2 - e1)
r¿ = - div P = -
Sde 2(x)
3.3 ELECTRIC CAPACITANCE ENERGY OF AN ELECTRIC FIELD 371

3.105 Let us impart an imaginary charge q to the conductor. Now, the potential difference
between the plates will be
R2

w+ - w- = E # dr
3
R1
R2
q 1 q
= 2
dr = ln R 2 /R1
3 4pe0a/r r 4pe0a
R1

Hence, the sought capacitance


q q 4p e0a 4 pe0a
C = = =
w+ - w- q ln R 2 /R1 ln R 2 /R1

3.106 Let l be the linear charge density.


l
Then, E 1m = (1)
2pe0R1e1
l
and E2m = (2)
2pe0R 2e 2
The breakdown in either case will occur at the smaller value of r for a simultaneous
breakdown of both dielectrics. From Eqs. (1) and (2)

E1mR1e1 = E2mR 2e 2
which is the sought relationship.

3.107 Let l be the linear charge density, then, the sought potential difference
R2 R3
l l
w+ - w- = dr + dr
3 2pe0 e1r 3 2pe0 e2r
R1 R2

c ln R 2 /R1 + ln R3 /R 2 d
l 1 1
=
2pe0 e1 e2

As, E1R1e1 6 E 2R 2e 2, so
l
= E1R1e1
2pe0
is the maximum acceptable value, and for values greater than E1R1e1, dielectric
breakdown will take place.
372 PART THREE ELECTRODYNAMICS

Hence, the maximum potential difference between the plates

w+ - w- = E1R1e1 c ln R3 /R 2 d
1 1
ln R 2 /R1 +
e1 e2
e1
= E1R1 c ln R 2 /R1 + ln R3 /R 2 d
e2

3.108 Let us suppose that linear charge density of the wires be l, then the po- + –
tential difference will be w+ - w- = w - ( - w) = 2w. The intensity of
the electric field created by one of the wires at a distance x from its axis
can be easily found with the help of the Gauss’ theorem as
b
l
E =
2pe0x
a
b -a
l b - a
Then, w = E dx = ln
3 2pe0 a
a

Hence, capacitance per unit length is given by


l l pe0 pe0
= = ⬵ (as b W a )
w+ - w- 2w ln (b - a )/a ln b/a

3.109 The field in the regions between the conducting plane and the wire can be obtained
by using an oppositely charged wire as an image on the other side.
Then the potential difference between the wire and the plane is

E # dr
+ –
¢w =
3 a
b
E
c d dr
l l
= +
3 2pe0r 2pe0 (2b - r)
a
b b
l b l b
= ln - ln
2pe0 a 2pe0 2b - a
l 2b - a
= ln
2pe0 a
l 2b
= ln (as b W a )
2pe0 a
3.3 ELECTRIC CAPACITANCE ENERGY OF AN ELECTRIC FIELD 373

Hence, the sought mutual capacitance of the system per unit length of the wire
l 2pe0
=
¢w ln 2b /a

3.110 When b W a, the charge distribution on each spherical + –


conductor is practically unaffected by the presence of
the other conductor. Then, the potential w + (w-) on the
positive (respectively negative) charged conductor is a a
b
a- b
q q
+
4pe0ea 4pe0ea

q q
Thus, w+ - w- = and C = = 2pe0ea
2pee0a w+ - w-

Note: If we require terms which depend on a / b , we have to take account of dis-


tribution of charge on the conductors.
+q –q
3.111 Let us apply the method of image. Then the potential dif-
ference between the positively charged sphere and the
conducting plane is one half the nominal potential differ- E a
ence between the sphere and its image and is defined as
l l
1 q
¢w = (w+ - w-) ⬵
2 4pe0a
q
Thus, C = L 4pe0a (for l W a )
¢w

3.112 (a) Since w1 = wB and w 2 = wA . The arrangement of capacitors shown in the prob-
lem is equivalent to the arrangement shown in the figure.

1 2 C1
A B A, 2 B, 1
C1 C2 C3 C2

C3
Hence, the capacitance between A and B is
C0 = C1 + C 2 + C3

(b) From the symmetry of the problem, there is no potential difference between D
and E. So, the combination reduces to a simple arrangement shown in the figure.
374 PART THREE ELECTRODYNAMICS

D D
C C C C
A C B A B
C C C C
E E

Hence, the net capacitance between A and B is


C C
C0 = + = C
2 2
3.113 (a) In the given arrangement, we have three capacitors of equal capacitance
C = e0S /d and the first and third plates are at the same potential.
Hence, we can resolve the network into a simple form C
using series and parallel grouping of capacitors, as shown
in the figure. C
Thus, the equivalent capacitance is A B

1C + C2 C 2 2 e0S
C0 = = C=
1C + C2 + C
C
3 3 d
(b) Let us imagine that plates 1 and 2 have the charges q and  q and these are
distributed to the other plates using charge conservation and electric induction
(see figure).
Since the potential difference between the plates 1 and 2 is zero,
q1 q2 q1 e 0S
- + - = 0 awhere C = b
C C C d

or q2 = 2q1

The potential difference between A and B


q2
w = wA - wB =
C
Hence, the sought capacitance
q q1 + q2 3q1 3 3e0S
C0 = = = = C =
w q2 >C 2q1 >C 2 2d

3.114 The amount of charge that the capacitor of capacitance C1 can withstand would be
q1 = C1V1 and similarly the charge that the capacitor of capacitance C2 can withstand
will be q2 = C2V2. But in a series combination, charge on both the capacitors will be
same, so, qmax that the combination can withstand = C1V1.
Also, C1V1 6 C2V2, from the numerical data given.
3.3 ELECTRIC CAPACITANCE ENERGY OF AN ELECTRIC FIELD 375

Now, net capacitance of the system


C1C2
C0 =
C1 + C2

qmax C1V1 C1
Vmax = = = V1 a 1 + b = 9 kV
C1C2 >C1 + C2
Hence,
C0 C2

3.115 Let us distribute the charges, as shown in the figure. Now, we know that in a clos-
ed circuit, - ¢w = 0.
So, in the loop DCFED,
q1 q2 q2
= j or q1 = C1 c j + q2 a + bd
1 1
- - (1)
C1 C1 C2 C1 C2
Again in the loop DGHED,
q1 q1 + q2
+ = j (2)
C1 C2
q1 –q1 G –q 2 q2 C
D A
C1 q1 + q2 C1 –q 2
x C2 –(q + q2) C2 +q
1 2
E B
H F

Using Eqs. (1) and (2), we get


C1 jC1
q2 c + 2d = -
1 3
+
C1 C2 C2 C2

- q2 j 1
wA - wB = =
C 22 >C1 1
Now,
C1
c + 2d
C2 3
+
C1 C2 C2
j j
or wA - wB = = = 10 V
C 22 3C2 h2 + 3h + 1
c 2 + + 1d
C1 C1

3.116 The infinite circuit may be reduced to the circuit shown in the figure where C0 is the
net capacitance of the combination.
1 1 1
So, + =
C + C0 C C0
376 PART THREE ELECTRODYNAMICS

Solving the quadratic equation, C

CC0 + C 20 - C 2 = 0
C C0
we get,

(15 - 1)
C0 = C (taking only + ve value as C0 cannot be negative)
2
= 0.62 C

3.117 Let us make the charge distribution as shown in the figure.


q q
Now, wA - wB = - j + +q –q x +q –q
C1 C2 A B
C1 C2
(wA - wB) + j
or q = C1C2
C1 + C2
Hence, voltage across the capacitor C1
q (wA - wB) + j
= = C2 = 10 V
C1 C1 + C2
and voltage across the capacitor, C2
q (wA - wB) + j
= = C1 = 5 V
C2 C1 + C2

3.118 Let j2 7 j1, then using - ¢w = 0 in the closed circuit +q –q


(see figure), A B
C1
-q q
+ j2 - - j1 = 0
C1 C2 x1 – – x2
+ +
(j2 - j1) C1C2
or q = C2
(C1 + C2) D C
–q q
Hence the potential difference (P.D.) across the left and
right plates of capacitors,
q (j2 - j1) C2
w1 = =
C1 C1 + C2
-q (j1 - j2) C1
and similarly, w2 = =
C2 C1 + C2
3.3 ELECTRIC CAPACITANCE ENERGY OF AN ELECTRIC FIELD 377

3.119 Using the solution of the foregoing problem, the amount of charge on each capaci-
tor is given by
| j2 - j1 | C1C2
|q| =
C1 + C2

3.120 Make the charge distribution as shown in the –q2 +q 2 –q2 +q 2


figure. In the circuit 12561, using - ¢w = 0 4 3
C1 C2
q1 q1 jC3C4
- - j = 0 or q1 = 5 –q1 +q 1 –q1 +q 1 2
C4 C3 C3 + C4 C3 C4

and in the circuit 13461, 6 1


x
q2 q2 j C1C2
+ - j = 0 or q2 =
C2 C1 C1 + C2
q2 q1
Now, wA - wB = -
C1 C3
C2 C4 C2C3 - C1C4
= j c - d = jc d
C1 + C2 C3 + C4 (C1 + C2) (C3 + C4)
It becomes zero, when
C1 C3
(C2C3 - C1C4) = 0 or =
C2 C4

3.121 Let us assume the charge q flows through the connect- +q


ing wires, then at the state of equilibrium, charge dis- +(CV – q) C2 – q
tribution will be as shown in the figure. In the closed C1 +q
circuit, using - ¢w = 0, we get –(CV – q) C3
–q
(C1V - q) q q
- + + = 0
C1 C2 C3
V
or q = = 0.06 mC
(1>C1 + 1>C2 + 1>C3 )

3.122 Initially, charge on the capacitor C1 or C2, is given by


jC1C2
q =
C1 + C2
since they are in a series combination Fig. (a).
378 PART THREE ELECTRODYNAMICS

When the switch is closed, in the circuit CDEFC, from - ¢ w = 0 Fig. (b).
q2
j- = 0 or q2 = C2 j (1)
C2

And in the closed loop BCFAB, from - ¢w = 0


- q1 q2
+ - j = 0 (2)
C1 C2

Sw –q +q C +q1 –q1
3 D B
C1 C1
+q +q2
x C2 x x C2 x
–q –q2

E A
1 2 1 F 2
(a)
(b)

From Eqs. (1) and (2), q1 = 0.


Now, charge flowing through section 1 = (q1 + q2) - 0 = C2 j
jC1C2
and charge flowing through section 2 = - q1 - q =
C1 + C2

3.123 When the switch is open Fig. (a),


2 jC1C2
q0 =
C1 + C2

1 1

x + q0 x C1
+q1
C1 – q0 –q1
Sw Sw
+q0 3 +q2
x 3 C2 x C2
– q0 –q2

2 2
(a) (b)

and when the switch is closed Fig. (b),

q1 = jC1 and q2 = jC2


3.3 ELECTRIC CAPACITANCE ENERGY OF AN ELECTRIC FIELD 379

Hence, the flow of charge, due to the shorting of switch


C1 - C2
through section 1 = q1 - q0 = jC1 c d = - 24 mC
C1 + C2
C1 - C2
through section 2 = - q2 - (q0) = jC2 c d = - 36 mC
C1 + C2

and through the section 3 = q2 - (q2 - q1) - 0 = j (C2 - C1) = 60 mC

3.124 First of all, make the charge distribution as shown in the figure.
In the loop 12341, using - ¢w = 0 –q 1 +q 1 1 A–q2 +q 2
2 6
q1 q1 - q2 C1 C2
- j1 + = 0 (1) –q 1+q 2
C1 C3 x C3
q1– q2
Similarly in the loop 61456, using - ¢w = 0
3 5
x1 4 B x2
q2 q2 - q1
+ - j2 = 0 (2)
C2 C3

From Eqs. (1) and (2) we have


j2C2 - j1C1
q2 - q1 =
C2 C1
+ + 1
C3 C3

q2 - q1 j2C2 - j1C1
Hence, wA - wB = =
C3 C1 + C2 + C3

3.125 In the loop ABDEA, using - ¢w = 0


q1 q1 + q2 – (q1 +q2) q +q
1
- j3 + + + j1 = 0 (1) E 1 2
D
C3 C1 x1 C1
+q2 –q 2
Similarly in the loop CDEFC F C
x2 2 C2
q1 + q2 q2 +q 1 –q 1
+ j1 - j2 + = 0 (2) A B
C1 C2 3
x3 C3
Solving Eqs. (1) and (2), we get
j2C2 - j1C2 - j1C3 + j3C3
q1 + q2 =
C3 C2
+ + 1
C1 C1
380 PART THREE ELECTRODYNAMICS

q1 + q2
Now, w1 - w0 = w1 = - (as w0 = 0)
C1
j1(C2 + C3) - j2C2 - j3C3
=
C1 + C2 + C3
j2(C1 + C3) - j1C1 - j3C3
Using symmetry, w2 =
C1 + C2 + C3
j3(C1 + C2) - j1C1 - j2C2
and w3 =
C1 + C2 + C3
The answers have wrong sign in the answer sheet.

3.126 Taking advantage of the symmetry of the problem, we can make a diagram for
charge distribution as shown in the figure.
q1 –q1 5 q2 –q 2
6 4
C1 C2
A q1 – q2 B
C3
+ q2 – q1 –
q2 –q 2 q1 –q 1
1 C2 2 C1 3

In the loop, 12561, using - ¢w = 0


q2 q2 - q1 q1
+ - = 0
C2 C3 C1

q1 C1(C3 + C2)
or = (1)
q2 C2(C1 + C3)

Now, capacitance of the network


q1 + q2 q1 + q2
C0 = =
wA - wB q2>C2 + q1>C1
(1 + q1>q2)
= (2)
q1
a b
1
+
C2 q2C1
From Eqs. (1) and (2)
2C1C2 + C3(C1 + C2)
C0 =
C1 + C2 + 2C3
3.3 ELECTRIC CAPACITANCE ENERGY OF AN ELECTRIC FIELD 381

q1q2
3.127 (a) Interaction energy of any two point charges q1 and q2 is given by , where
4pe0r
r is the separation between the charges.
+q +q +q –q +q +q

+q +q –q +q –q –q
a
(a) (b) (c)

Hence, interaction energy of the system is given by


q2 q2
Ua = 4 + 2
4pe0a 4pe01 12 a)
- q2 q2
Ub = 4 + 2
4pe0 a 4pe0 112 a2
q2 2q 2 2q 2 12 q 2
Uc = 2 - - = -
4pe0 112 a2
and
4pe0a 4pe0 a 4pe0 a

3.128 Since the chain is of infinite length, any two charges of the same sign will occur sym-
metrically to any other charges of the opposite sign. So, interaction energy of each
charge with all the others is given by
q2
c1 - + - Á up to q d
1 1 1
U = -2 (1)
4pe0 a 2 3 4
1 2 1
But, ln (1 + x) = x - x + x 3 Á up to q
2 3

Putting x = 1, we get 1 1
ln 2 = 1 - + + Á up to q (2)
2 3
From Eqs. (1) and (2),
- 2q 2 ln 2
U =
4pe0a

3.129 Using electrical image method, interaction energy of the charge q with those induced
on the plane is given by
-q2 q2
U = = -
4pe0 (2l ) 8pe0l
382 PART THREE ELECTRODYNAMICS

3.130 Consider the interaction energy of one of the balls (say 1) and thin spherical shell of
the other. This interaction energy can be written as
dQ
R
r θ
l

wd q
3
q1
= r2 (r) 2pr 2 sin u d u dr
3 4pe0 R
p l+r
r2 (r) q1r 2 sin u du dr q 1r
= = dr dx r2 (r)
3 2 e0 (l 2 + r 2 + 2lr cos u)1>2 2e0l 3
0 l-r
q1r q1
= dr j 2r r2 (r) # 2 = 4 pr 2 dr r2 (r)
2e0l 4pe0l

Then, finally integrating


q
q1q2
Uint = where q2 = 4pr 2 r2 (r) dr
4pe0l 3
0

3.131 Charge contained in the capacitor of capacitance C1 is q = C1w and the energy,
stored in it is given by
q2 1
Ui = = C1w 2
2C1 2
Now, when the capacitors are connected in parallel, the equivalent capacitance of
the system C = C1 + C2 and hence, energy stored in the system is
C1 2 w 2
Uƒ = (since charge is conserved during the process)
2(C1 + C2)
So, increment in the energy
C1 2 w 2 - C2C1w 2
a b =
1 1
¢U = - = - 0.03 m J
2 C1 + C2 C1 2 (C1 + C2)

3.132 The charge on the condensers in position 1 is as shown in Fig. (a).


q q0 q + q0
Here, = =
C C0 C + C0
3.3 ELECTRIC CAPACITANCE ENERGY OF AN ELECTRIC FIELD 383

C (C + C0)
(q + q0) a + b = j or q + q0 =
1 1
and
C + C0 C C0 + 2C

C 2j CC0j
Hence, q = and q0 =
C0 + 2C C0 + 2C

q0 q0
+q +q 0 –q–q 0
C C0 C +q + q 0 –q 0 –q
–q –q 0 +q+q0 C C0 C
–q – q 0 +q 0 +q
2q 0
1 2 1 2

ξ q0 x q0
(a) (b)

After the switch is thrown to position 2, the charges change as shown in Fig. (b).
A charge q0 has flown in the right loop through the two condensers and a charge q0
through the cell, and because of the symmetry of the problem there is no change in
the energy stored in the condensers. Thus,
H (Heat produced) = Energy delivered by the cell
CC0 j 2
= ¢qj = q0j =
C0 + 2C
3.133 Initially, the charge on the right plate of the capacitor, q = C (j1 - j2) and finally,
when switched to the position 2, charge on the same plate of capacitor is
q¿ = C j1.
So, ¢q = q¿ = C j2
Now, from energy conservation,
¢U + Heat liberated = Acell
(where ¢U is the electrical energy),
1 1
or C j21 - C (j1 - j2)2 + Heat liberated = ¢q j1
2 2
as only the cell with e.m.f. j1 is responsible for redistribution of the charge.
1
So, C j1j2 - C j22 + Heat liberated = C j2 j1
2
1
Hence, heat liberated = C j22 .
2
384 PART THREE ELECTRODYNAMICS

qw
3.134 Self energy of each shell is given by , where w is the potential of the shell, created
2
only by the charge q on it.
Hence, self energies of the shells 1 and 2 are:
q1 2 q 22
W1 = and W2 =
8pe0R1 8 pe0R2
The interaction energy between the charged shells equals charge q of one shell, mul-
tiplied by the potential w created by other shell, at the point of location of charge q.
q2 q1q2
So, W12 = q1 =
4pe0R2 4pe0R2
Hence, total energy of the system
U = W1 + W2 + W12
q12 q22 q1q2
c d
1
= + +
4pe0 2R1 2R2 R2

3.135 Electric fields inside and outside the sphere is obtained with the help of Gauss ’ theorem.
qr q 1
E1 = (for r … R) and E2 = ( for r 7 R)
4pe0r 2 4pe 0 r 2
Sought self energy of the ball
U = W1 + W2
R q
e0E 21 e0E 22 q2
a + 1b
1
= 4pr 2dr + 4pr 2dr =
3 2 3 2 8pe0R 5
0 R
3q 2 W1 1
Hence, U = and =
4pe0 5R W2 5
3q 2
=
20pe0R
3.136 For a spherical layer,
1 1
e0eE 2 dV = ee0 E2 4pr 2dr
3 2 3 2
To find the electrostatic energy inside the dielectric layer, we have to integrate the
above expression in the limit [a, b]. We get,
b
q2
a b c - d = 27 mJ (on substituting values)
1 q 2
2 dr =
1 1
U = e0e 2
4pr
2 3 4pe0er 8pe0e a b
a
3.3 ELECTRIC CAPACITANCE ENERGY OF AN ELECTRIC FIELD 385

3.137 Since the field is conservative, the total work done by the field force
1
A = Ui - Uf = q (w1 - w2)
2
1 q2 q2
c d = c d
1 1 1 1
= - -
2 4pe0 R1 R2 8pe0 R1 R2

3.138 Initially, energy of the system, Ui = W1 + W12 where, W1 is the


self energy and W12 is the mutual energy. q
R1
1 q2 qq0 q0
So, Ui = +
2 4pe0R1 4pe0R1
R2
and on expansion, energy of the system is given by
Uf = W ¿ 1 + W ¿ 12
1 q2 qq0
= +
2 4pe0R1 4pe0R1

Now, work done by the field force, A equals the decrease in the electrical energy,
q (q0 + q/2)
a b
1 1
i.e., A = (Ui - Uf ) = -
4pe0 R1 R2

Alternate:

The work of electric forces is equal to the decrease in electric energy of the system,

A = Ui - Uf

In order to find the difference Ui - Uf , we note that upon expansion of the shell, the
electric field and hence the energy localized in it, changed only in the hatched
spherical layer consequently (see figure).
R2

A E 21 - E 22 B # 4pr 2dr
e0
Ui - Uf =
3 2
R2

where E1 and E2 are the field intensities (in the hatched region at a distance r from
the center of the system) before and after the expansion of the shell. By using Gauss’
theorem, we find

1 q + q0 1 q0
E1 = and E2 =
4pe0 r 2 4pe0 r 2
386 PART THREE ELECTRODYNAMICS

As a result of integration, we obtain


q (q0 + q/2) 1
a b
1
A = -
4pe0 R1 R2
3.139 Energy of the charged sphere of radius r can be obtained from the equation

1 1 q q2
U = qw = q =
2 2 4pe0r 8pe0r
If the radius of the shell changes by dr, then work done is
q2
4 pr 2 Fudr = - dU =
8pe0r 2
Thus sought force per unit area,
q2 (4pr 2 s)2 s2
Fu = = =
4pr 2 (8pe0r 2) 4pr 2 * 8 pe0r 2 2e0

3.140 Initially, there will be induced charges of magnitude - q and + q on the inner and
outer surfaces of the spherical layer, respectively. Hence, the total electrical energy
of the system is the sum of self energies of spherical shells having radii a and b, and
their mutual energies including the point charge q.
1 q2 1 ( - q)2 - qq qq - qq
Ui = + + + +
2 4pe0 b 2 4pe0 a 4pe0 a 4pe0b 4pe0b

q2
c - d
1 1
or Ui =
8pe0 b a
Finally, charge q is at infinity, hence Uf = 0.
Now, work done by the agent = increment in the energy
q2
c - d
1 1
= Uf - Ui =
8pe0 a b

3.141 (a) Sought work is equivalent to the work performed against the electric field creat-
ed by one plate held at rest, and moving the other plate away. Therefore the re-
quired work,
Aagent = qE (x2 - x1)

where E = s/ 2e0 is the intensity of the field created by one plate at the location
of other.
s q2
So, Aagent = q (x2 - x1) = (x - x1)
2e0 2e0S 2
3.3 ELECTRIC CAPACITANCE ENERGY OF AN ELECTRIC FIELD 387

Alternate:
Aext = ¢U (as field is potential)
q2 q2 q2
= x2 - x = (x2 - x1)
2e0S 2e0S 2e0S
(b) When voltage is kept constant, the force acting on each plate of capacitor will
depend on the distance between the plates.
So, elementary work done by agent, in its displacement over a distance dx, rel-
ative to the other is given by
dA = - Fxdx
e0V
Fx = - a b Ss (x) and s (x) =
s(x)
But,
2e0 x
x2
e0SV 2 1
c d
1 SV 2 1
Hence, A = dA = e0 2 dx = -
3 3 2 x 2 x1 x2
x1
Alternate:
From energy conservation,
Uf - Ui = Acell + Aagent

1 e0S 2 1 e0S 2 e0S e0S


or V - V = c - d V 2 + Aagent
2 x2 2 x1 x2 x1

(as Acell = (qf - qi )V = (Cf - Ci ) V 2).

Aagent = c d
1 1
So, -
x1 x2

3.142 (a) When a metal plate of thickness hd is inserted inside the capacitor, the capaci-
tance of the system becomes
e0S
C0 =
d (1 - h)
Initially, charge on the capacitor,
e0SV
q0 = C0V =
d (1 - h)
Finally, capacitance of the capacitor,
e0S
C =
d
As the source is disconnected, charge on the plates will remain same during the
process. From energy conservation,
Uf - Ui = Aagent (as the cell does no work)
388 PART THREE ELECTRODYNAMICS

2 2
1 q0 1 q0
or - = Aagent
2 C 2 C0

e0SV (1 - h)
c d c - d =
1 2 1 1 CV 2 h
Hence, Aagent = = 1.5 mJ
2 d (1 - h) C C 2 (1 - h)2

(b) Initially, capacitance of the system is given by


Ce
C0 = (since the two capacitors are in series)
h(1 - e) + e

So, charge on the plate q0 = C0V .


Capacitance of the capacitor, after the glass plate has been removed equals C.
From energy conservation,

Aagent = Uf - Ui
1 CV 2eh (e - 1)
q0 c - d =
1 2 1 1
= = 0.8 mJ
2 C C0 2 [e - h(e - 1)]2

3.143 When the capacitor which is immersed in water is connected to a constant voltage
source, it gets charged. Suppose s0 is the free charge density on the condenser
plates. Because water is a dielectric, bound charges also appear in it. Let s¿ be the
surface density of bound charges. (Because of homogeneity of the medium and uni-
formity of the field, when we ignore edge effects no volume density of bound
charges exists.) The electric field due to free charges is only s0 / e0 ; that due to bo-
und charges is s¿/ e0 and the total electric field is s0 / ee 0 . Recalling that the sign
of bound charges is opposite of the free charges, we have
s0 s0 e - 1
s¿ = a b s0
s¿
= - or
ee0 e0 e0 e

Because of the field that exists due to the free charges (not the total field; the field
due to the bound charges must be excluded for this purpose as they only give rise
to self energy effects), there is a force attracting the bound charges to the nearby
plates. This force is

1 s0 (e - 1) 2
s¿ = s0 (per unit area)
2 e0 2 ee0
The factor 1/2 needs an explanation. Normally the force on a test charge is qE in
an electric field E. But if the charge itself is produced by the electric filed then the
force must be constructed bit by bit and is
3.3 ELECTRIC CAPACITANCE ENERGY OF AN ELECTRIC FIELD 389

F = q (E ¿) dE ¿
3
0

if q (E ¿) r E ¿ , then we get
1
F = q (E ) E
2

This factor of 1/2 is well known. For example, the energy of a dipole of moment p in
an electric field E0 is - p # E0, while the energy per unit volume of a linear dielectric
in an electric field is - 1/2 P # E 0, where P is the polarization vector (i.e., dipole moment
per unit volume). Now the force per unit area manifests itself as excess pressure of
the liquid.
V s0
Noting that =
d ee0

e0e (e - 1)V 2
We get, ¢p =
2d 2

Using e = 81 for water, gives ¢p = 7.17 kPa = 0.07 atm.

3.144 One way of solving this problem will be exactly as in the previous case, so let us try
an alternative method based on energy. Suppose the liquid rises by the distance h.
Then let us calculate the extra energy of the liquid as a sum of the polarization ener-
gy and the ordinary gravitational energy. The latter is
1 # # 1
h rg Sh = rgSh2
2 2
If s is the free charge surface density on the plate, the h
bound charge density is, from the previous problem, a

e - 1
s¿ = s
e
This is also the volume density of induced dipole moment, i.e., polarization. Then
the energy is, as before
-1 # - (e - 1)s2
p E0 = - # s¿ E0 =
1 # 1 s
- s¿ =
2 2 2 e0 2e0e
where p is polarization vector and the total polarization energy is
(e - 1) s2
- S (a + h)
2e0e
390 PART THREE ELECTRODYNAMICS

Then, total energy is


(e - 1) s2 1
U (h) = - S (a + h) + rgSh2
2 e0e 2
The actual height to which the liquid rises is determined from the formula
dU
= U ¿(h) = 0
dh
(e - 1) s2
This gives h =
2e0erg

Alternate:
To find the stable height of the liquid, the process of its rise should be assumed to
be very slow (in equilibrium). From energy conservation in differential form,
dUele + dUgr = 0

dUele (h) dUgr (h)


So, + = 0
dh dh
d q2
a b + a mg b = 0
d h
dh 2C dh 2

Using m = rSh and q = sS = constant, where S is area of each of the plates,


q2 d 1
a b + a rShg b = 0
d h
2 dh C dh 2
1 a h d - h
But, = + + (where d is separation between the plates)
C ee0S ee0S e0S
d - h
a b = c d
d 1 d h
So, + (because initial liquid height a is constant)
dh C dh ee0S e0S
d ed - (e - 1)h (e - 1)
a b = c d = -
d 1
dh C dh e0 Se ee0S

q 2 (e - 1)
Hence, - + (rgS) h = 0
2 ee0S
s2 (e - 1)
= rgh
2 e0
s2 (e - 1)
h =
2rgee0
3.3 ELECTRIC CAPACITANCE ENERGY OF AN ELECTRIC FIELD 391

Note: Assuming that the lower plate is just immersed, the answer remains the same.

3.145 We know that the energy of the capacitor,

q2
U =
2C
0U
Hence, from Fx = `
0x q = constant

we have,
q 2 0C
Fx = nC 2 (1)
2 0x
Now, since d V R, the capacitance of the given capacitor can be calculated by the
formula of a parallel plate capacitor. Therefore, if the dielectric is introduced upto a
depth x and the length of the capacitor is l, we have,
2pe0 eRx 2pRe0 (l - x)
C = + (2)
d d
From Eqs. (1) and (2), we get,
pRV 2
Fx = e0 (e - 1)
d
3.146 When the capacitor is kept at a constant potential difference V, the work performed
by the moment of electrostatic forces between the plates, when the inner movable
plate is rotated by an angle dw, equals the increase in the potential energy of the
system. This comes about because when changes are made, charges flow from the
battery to keep the potential constant and the amount of the work done by these
charges is twice in magnitude and opposite in sign to the change in the energy of
the capacitor. Thus,
0U 1 0C
Nz = = V2
0w 2 0w
Now the capacitor can be thought of as made up of two parts (with and without the
dielectric) in parallel.
e 0 R 2w e0e(p - w) R 2
Thus, C = +
2d 2d
as the area of a sector of angle w is 1/2 R2
w. Differentiation then gives
(e - 1) e0R 2V 2
Nz = -
4d
The negative sign of Nz indicates that the moment of the force is acting clockwise
(i.e., trying to suck in the dielectric).
392 PART THREE ELECTRODYNAMICS

3.4 Electric Current

3.147 The convection current is


dq
I = (1)
dt

here, dq = ldx, where l is the linear charge density.


But, from the Gauss ’ theorem, electric field at the surface of the cylinder

l
E =
2 pe0a

Hence, substituting the value of l and subsequently of dq in Eq. (1), we get


2E pe0adx
I =
dt
= 2 pe0Eav aas = vb
dx
dt
= 0.5 mA (on substituting values)

3.148 Since d V r, the capacitance of the given capacitor can be calculated using the for-
mula for a parallel plate capacitor. Therefore, if water (permittivity e ) is introduced
up to a height x and the capacitor is of length l, we have

ee0 # 2prx e0(l - x) 2pr e02pr


C = + = (ex + l - x)
d d d

Hence charge on the plate at that instant, q = CV .


Again we know that the electric current intensity,
dq d (CV )
I = =
dt dt
Ve0 2pr d (ex + l - x) V 2pre0 dx
= = (e - 1)
d dt d dt

dx
But, = v
dt

2pre0(e - 1)V
So, I = v L 0.11 mA
d
3.4 ELECTRIC CURRENT 393

3.149 We have,
Rt = R0 (1 + at) (1)
where Rt and R0 are resistances at t °C and 0°C, respectively and a is the mean tem-
perature coefficient of resistance.
So, R1 = R0(1 + a1t) and R2 = hR0(1 + a2t)
(a) In case of series combination, R = ©Ri

So, R = R1 + R2 = R0 [(1 + h) + (a1 + ha2) t]


a1 + ha2
= R0 (1 + h)c 1 + td (2)
1 + h
Comparing Eqs. (1) and (2), we conclude that temperature coefficient of resista-
nce of the circuit
a1 + ha2
a =
1 + h
(b) In parallel combination
R0(1 + a1t) R0 h (1 + a2t) hR0
R = = R ¿(1 + a¿t), where R ¿ =
R0(1 + a1t) + h R0 (1 + a2t) 1 + h

Now, neglecting the terms proportional to the product of temperature coeffic-


ients, as they have very small values, we get
ha1 + a2
a¿ =
1 + h
3.150 Using the property of symmetry, the currents are flowing as shown in the figures.
(a) In the figures, Ohm’s law is applied between 1 and 7 via 1487 (say). So,
I I I 5 6 I/3
IReq = R + R + R = RI I/6
7 I
3 6 3 6 I/6 I/3
2 3 I/3
5R I/6
Thus, Req = I/6
6 I/3 8
5
I/3 I/6 I/6
(b) Between 1 and 2 from the loop 14321,
I 1 I/3 4
I1R = 2I2R + I3R or I1 = I3 + 2I2 (a)

From the loop 48734, 6 I2−I3 7


I I −I
(I2 - I3) R + 2(I2 - I3) R + (I2 - I3) R = I3R I2 2 3
2 3 2(I2−I3)
4 I3 I3
or 4 (I2 - I3) = I3 Q I3 = I I1 5 I2−I3
5 2 8
I2
14 1 I−I3
So, I1 = I I1+2 I3 I2 4
5 2
(b)
394 PART THREE ELECTRODYNAMICS

24
Then, (I1 + 2I2) Req = IR
5 2 eq
14
= I1R = IR
5 2
7
or Req = R
12
(c) Between 1 and 3 from the loop 15621 6 I1/2 7
I1 I1 0 I1/2 I1
I2R = I1R + R or I2 = 3 2
I2
I1/2
I1+2I2
2 2 3
I2
I1/2
Then, (I1 + 2I2)Req = 4I1Req I2
8
I1 5
0
= I2R + I2R = 3I1R 1
I2+2I2 I2 4
3 (c)
Hence, Req = R
4
3.151 Total resistance of the circuit will be independent of the number of cells,

(Rx + 2R)R 2R
if Rx =
Rx + 2R + R
R Rx Rx
or Rx2 + 2RRx - 2R 2 = 0
On solving and rejecting the negative root of the quadratic equation, we have
Rx = R ( 23 - 1)

3.152 Let R0 be the resistance of the network. A A


R1
R0R2 R2 R0 R0
Then, R0 = + R1
R0 + R2 B B
or R 20 - R0R1 - R1R2 = 0

On solving we get,

b = 6Æ
R1 R2
R0 = a1 + 1 + 4
2 A R1

3.153 Suppose that the voltage V is applied between the points A and B, then
V = IR = I0R0
where R is the resistance of the whole grid, I is the current through the grid and I0 is
the current through the segment AB. Now from symmetry, I/4 is the part of the cur-
rent flowing through all the four wire segments, meeting at the point A and similarly
3.4 ELECTRIC CURRENT 395

the amount of current flowing through the wires meeting at B is also I/4. Thus a
current I/2 flows through the conductor AB, i.e.,
I
I0 =
2
R0
Hence, R =
2
3.154 Let us imagine a thin cylindrical layer of inner and
– l
outer radii r and r + dr, respectively. As lines of
dr
current at all the points of this layer are perpendi-
cular to it, such a layer can be treated as a cylin-
r
drical conductor of thickness dr and cross-section- + –
al area 2prl. So, we have,
dr dr
dR = r = r
S (r) 2prl

and integrating between the limit [a, b], we get


r b
R = ln
2pl a
3.155 Let us imagine a thin spherical layer of inner and outer radii r and r + dr. Lines of cur-
rent at all the points of this layer are perpendicular to it and therefore such a layer can
be treated as a spherical conductor of thickness dr and cross sectional area 4pr 2. So,
dr
dR = r
4pr 2
and integrating between the limits [a, b], we get,

c - d
r 1 1
R =
4p a b
Now, for b : q, we have r
R =
4pa

3.156 In this case, resistance of the medium is given by

c - d (where r is the resistivity of the medium)


r 1 1
R =
4p a b
The current is given by
w w
I = = (1)
c - d
R r 1 1
4p a b
- dq d(Cw) dw
Also, I= = - = -C (as capacitance is constant) (2)
dt dt dt
396 PART THREE ELECTRODYNAMICS

Equating Eqs. (1) and (2), we get

w dw
= -C
c - d
r 1 1 dt
4p a b

dw ¢t
or – =
L w
c - d
Cr 1 1
4p a b

¢t 4pab
or In h =
Cr(b - a)
Hence, resistivity of the medium

4p¢t ab
r =
C (b - a) ln h
3.157 Let us imagine that the balls have charge +q and - q , respectively. The electric fi-
eld strength at the surface of a ball will be determined only by its own charge and
the charge can be considered to be uniformly distributed over the surface, because
the other ball is at an infinite distance. Magnitude of the field strength is given by
q
E =
4pe0 a 2
So, current density

1 q
j =
r 4pe0a 2

and electric current

q q
I = j # d S = jS = # 4pa 2 =
L r4 pe0a 2 re0

But, potential difference between the balls


q
w+ - w- = 2
4 pe0 a
Hence, the sought resistance

w + - w– 2q/4pe0a r
R = = =
I q/r e0 2pa
3.4 ELECTRIC CURRENT 397

3.158 (a) The potential in the unshaded region beyond the conductor has the potential of
the given charge and its image has the form

w = A a - b
1 1
r1 r2
where r1, r2 are the distances of the point from the E
charge and its image, respectively. The potential
P q
has been taken to be zero on the conducting plane
and on the ball. r
x
w L A a - b =V
1 1
q q
a 2l image l
So, A L Va. In this calculation, the condition a 66 l ,
is used to ignore the variation of w over the ball.
The electric field at P can be calculated similarly. The charge on the ball is
Q = 4p e0Va
Va 2alV 1 2alV
and EP = 2 cosu = then j = E =
r2 r3 r rr 3
(b) The total current flowing into the conducting plane is
q q
2alV
I = 2 pxdx j =
r1 x 2 + l 22 3>2
2pxdx
3 3
0 0

Putting y = x2 + l , we get
2
q
2 p alV dy 4paV
I = =
r 3 y 3>2 r
l2
V r
Hence, R = =
I 4pa
3.159 (a) The wires can be assumed to be perfect conductors, so the resistance is entire-
ly due to the medium. If the wire is of length L, the resistance R of the medium
is ^ 1/L because different sections of the wire are connected in parallel (by
the medium) rather than in series. Thus if R1 is the resistance per unit length of
the wire, then R = R1>L. Unit of R1 is ohm-meter.
E
The potential at a point P is by symmetry and P
superposition for (l W a ), is given by
q q
r1 =r r2 =r
A r1 A r2
w⬵ ln - ln
2 a 2 a
1 2
A r1
= ln +V/ 2 –V/2
2 r2 l
398 PART THREE ELECTRODYNAMICS

V A a
Then, w1 = = ln (for the potential of 1)
2 2 l
-V
or A =
ln l /a

ln r1>r2
V
and w = -
2 ln l>a

We then calculate the field at a point P which is equidistant from 1 and 2 and
at a distance r from both:

a b * 2 sin u
V 1
E =
2 ln l /a r
Vl 1
=
2 ln l>a r 2

1 Vl
and j = sE =
r 2r 2 ln l/a
V 1
(b) Near either wire E =
2 ln l>a a

1 V
and j = sE =
r 2 ln l/a
V V
Then I = = L = j2p aL
R R1

r
which gives R1 = ln l>a
p

3.160 Let us imagine that the plates of the capacitor have the charges +q and -q.
The capacitance of the network

ee0 En dS
q L
C = = (1)
w w
Now, electric current

I = 1 j # d S = 1sEndS, as j c c E (2)

Hence, using Eq. (1) in (2), we get


Cw Cw
I = s = = 1.5 m A
ee0 ree0
3.161 Let us imagine that the conductors have the charges q and - q. As the medium
3.4 ELECTRIC CURRENT 399

is a poor conductor, the surfaces of the conductors are equipotential and the field
configuration is same as in the absence of the medium.
Let us surround, for example, the positively charged conductor, by a closed surface
S, just containing the conductor.
w w w
Then, R = = = (as j c c E )
I 1j # dS 1 sEndS
q ee0 1 EndS
and C = =
w w
ee0
So, RC = = ree0
s
3.162 The dielectric ends in a conductor. It is given that, on one side (the dielectric side)
the electric displacement D is as shown in figure. Within the conductor, at any point
A, there can be no normal component of electric field. For if there were such a field, a
current will flow towards depositing charge there which in turn will set up countering
electric field causing the normal component to vanish. Then by Gauss’ theorem, we can
easily derive s = Dn D cos a , where s is the
surface charge density it A.
The tangential component is determined D
from the circulation theorem as
a
Dielectric n A (Conductor)
E # dr = 0
C
It must be continuous across the surface of
the conductor. Thus, inside the conductor
there is a tangential electric field of magnitude
D sin a
at A
e0e
This implies a current, by Ohm’s law, of density
E D sin a
j = =
r e0er

3.163 The resistance of a layer of the medium, of thickness dx and at a distance x from
the first plate of the capacitor is given by

1 dx
dR = (1)
s1x2 S

Now, since s varies linearly with the distance from the plate, it may be represented
400 PART THREE ELECTRODYNAMICS

s2 - s1
as, s = s1 + a bx, at a distance x from any one of the plates.
d
1 dx
From Eq. (1) dR =
s2 - s1
s1 + a bx
S
d
d
1 dx d s2
or R = = ln
s2 - s1 S (s2 - s1) s1
s1 + a bx
S 3
0
d
V SV (s2 - s1)
Hence, I = = = 5 nA
R s2
d ln
s1

3.164 By charge conservation current density j, leaving the medium 1 must enter the
medium 2.
Thus, j1 cos a1 = j2 cos a2
Another relation follows from
E1t = E2t n
2
which is a consequence of a2 j2

E # dr = 0. 1
C a1
j1
1 1
Thus, j sin a1 = j sin a2
s1 1 s2 2
tan a1 tan a2
or =
s1 s2
tan a1 s1
or =
tan a2 s2

3.165 The electric field in conductor 1 is


r1I
E1 =
pR 2
r 2I
and that in 2 is E2 =
pR 2
Applying Gauss’ theorem to a small cylindrical pill-box at the boundary,
r1I r2I sdS
- dS + dS =
pR 2 pR 2 e0
3.4 ELECTRIC CURRENT 401

1
Thus, s = e0 (r2 - r1)
pR 2
and charge at the boundary = e0 (r2 - r1) I

3.166 We have, E1d1 + E2d2 = V and by current conservation


1 1
E = E E1 E2
r1 1 r2 2 +V O
r1V
Thus, E1 = d1 d2
r1d1 + r2d2
r2V
E2 =
r1d1 + r2d2
At the boundary between the two dielectrics,
s = D2 - D1 = e0e2E2 - e0e1E1

e0V
= (e2r2 - e1r1)
r1d1 + r2d2

3.167 By current conservation


E (x) E (x) + dE (x) dE (x) dx
= =
r (x) r (x) + dr (x) dr (x)
O
Solving we get, I
+V E E+dE
I r (x)
E (x) = Cr (x) = A
Hence, charge induced in the slice per unit area is given by

l I
ds = e0 [{e (x) + de (x)} {r (x) + dr (x)} - e (x) r (x)] = e0 d [e (x) r (x)]
A A
Thus, dQ = e0Id [e (x) r (x)]

Hence total charge induced obtained by integration is


Q = e0I (e2r2 - e1r1)

3.168 As in the previous problem,

E (x) = Cr (x) = C (r0 + r1x)


(h - 1) r0
where r0 + r1d = h r0 or r1 =
d
402 PART THREE ELECTRODYNAMICS

d
h - 1
Cr (x) dx = Cr0d a 1 + b = Cr0d (h + 1)
1
By integration, V =
3 2 2
0
2V
Thus, C =
r0d (h + 1)

Thus volume density of charge present in the medium

dQ
= = e0 dE (x)/dx
Sdx
2e0V (h - 1) r0 2e0 V (h - 1)
= * =
r0d (h + 1) d (h + 1) d 2

3.169 (a) Consider a cylinder of unit length and divide it into shells of radius r and thick-
ness dr. Different sections are parallel. For a typical section

da b =
1 2pr dr 2pr 3dr
=
R1 (a/r 2) a

1 pR 4 S2
Integrating, = =
R1 2a 2pa
2pa
or R1 =
(where S = pR 2)
S2
(b) Suppose the electric field inside is Ez = E0 (z-axis is along the axis of the con-
ductor). This electric field cannot depend on r in steady conditions when other
components of E are absent, otherwise it would violate the circulation theorem

E # dr = 0
I
The current through a section between radii (r and r +dr ) is
1 E
2prdr E = 2pr 3dr
a/r 2 a
R
E pR 4E
Thus, I = 2pr 3dr =
3 a 2a
0
2 apI
Hence, E = (where S = pR 2)
S2
3.170 The formula is,
q = CV0 (1 - e -t/RC )
3.4 ELECTRIC CURRENT 403

q V
or V = = V0 (1 - e -t/RC ) or = 1 - e - t/RC
C V0

V V0 - V
or e -t/RC = 1 - =
V0 V0

V0
Hence, t = RC ln = RC ln 10, if V = 0.9 V0
V0 - V

Thus, t = 0.6 ms
3.171 The charge decays according to the formula

q = q0e -t/RC

Here, RC = mean life = half-life/ln 2

So, half life T = RC ln 2


ee0A rd
But, C = and R =
d A
T
Hence, r = = 1.4 * 10 13 Æm
ee0 ln 2

3.172 Suppose q is the charge at time t. Initially q = C j, at t = 0.


Then at time t, from Loop rule
hq
IR = - j (1)
C
Differentiating Eq. (1) with respect to time and considering that I = - dq /dt (-sign
because charge decreases)
dI h ξ
R = - I
dt C
dI -h # I
i.e., = dt
I RC –IR η q/c
Integration of this equation gives
I -ht
ln =
I0 RC
So, I = I0 e-ht /RC
where I0 is determined by condition of Eq. (1).
Indeed we can write
hq 0
I0R = - j
C
404 PART THREE ELECTRODYNAMICS

where q0 = C j is the charge of the capacitor before its capacitance has charged.
j
Therefore, I0 = (h - 1)
R
dq j1h - 12
Finally, I = - = e -ht/RC
dt R

3.173 Let r = internal resistance of the battery. We shall take the resistance of the amme-
ter to be = 0 and that of voltmeter to be G.
j
Initially, V = j - Ir, I =
r + G
G
So, V = j (1)
r + G

After the voltmeter is shunted x


V jr
= j - (Voltmeter) (2)
h RG V A
r +
R + G
R
j j
and = h (Ammeter) (3)
RG r + G
r +
R + G
From Eqs. (2) and (3) we have
V hr j
= j - (4)
h r + G
From Eqs. (1) and (4)
G
= r + G - h r or G = hr
h

Then Eq. (1) gives the required reading


V j
=
n h + 1
= 2.0 V (on substituting values)

3.174 Assume the current flow to be as shown in figure. Then potentials are as shown.
Thus,
w1 = w1 - IR1 + j1 - IR2 - j2
3.4 ELECTRIC CURRENT 405

j1 - j2 j1 –IR1 j1 –IR1 + ξ1
or I = ξ1
R1 + R2
R1
and w2 = w1 - IR1 + j1 1 j1 j2 2
j1 - j2 I R2
So, w1 - w2 = - j1 + R1 ξ2 –IR1 + ξ1–IR2
R1 + R2 j
(j1R2 + j2R1)
= -
(R1 + R2)
= - 4V
3.175 Let us consider the current I, flowing through the circuit, as shown in the figure.
Applying loop rule for the circuit, - ¢w = 0.
So, - 2j + IR 1 + IR2 + IR = 0 1 x R1 2 x R2 3
or 2j = I 1R1 + R2 + R2
2j
or I = i
R + R1 + R2 R
Now, if w1 - w2 = 0
- j + IR 1 = 0
2jR1
or = j and 2R1 = R2 + R + R1
R + R1 + R2
or R = R1 - R2 (which is not possible as R2 7 R1 )
Thus, w2 - w3 = - j + IR2 = 0
2jR2
or = j
R + R1 + R2

So, R = R2 - R1
which is the required resistance.
Nj NaR
3.176 (a) Current, I = = = a (as j = aR)
NR NR
(b) wA - wB = nj - nI R = n a R - n a R = 0
3.177 Since the capacitor is fully charged, no current flows through it. x1 R1
So, current is given by
j2 - j1
I = (as j2 7 j1 ) x1 C
R1 + R2 1
A B
2
I
Hence, wA - wB = j1 - j2 + IR2 x2
R2
406 PART THREE ELECTRODYNAMICS

j2 - j1
= j1 - j2 + R2
R1 + R2
1j1 - j22R1
= = - 0.5 V
R1 + R2

3.178 Let us make the current distribution, as shown in the figure.


j I1 I2
Current I = (using loop rule)
R1R2 I1
R +
R1 + R2
x R R1 R2
So, current through the resistor R1,
j R2
I1 =
R1R2 R1 + R2
R +
R1 + R2
jR2
= = 1.2 A
RR1 + RR2 + R1R2
and similarly, current through the resistor R2,
j R1 jR1
I2 = = = 0.8 A
R1R2 R1 + R2 RR1 + R1R2 + RR2
R +
R1 + R2

3.179 Total resistance


xR0
R #
l - x l
= R0 +
l xR0
R +
l
l - x xRR0 V0
= R0 +
l lR + xR0 l
x R0
l - x
= R0 c d
xR
+
l lR + xR0 V
R

n a1 - b
xR x xR
Then V = V0 +
lR + xR0 l xR0 + lR

= V0Rx n e lR + R0 x a 1 - bf
x
l
x
For R 77 R0, V ⬵ V0
l
3.4 ELECTRIC CURRENT 407

3.180 Let us connect a load of resistance R between the points A and B (see figure).

From the loop rule, ¢w = 0, so x1,R1


I1
IR = j1 - I1R1 (1) A B
and IR = j2 - 1I - I12R2 (2)
x2,R2
Solving Eqs. (1) and (2), we get I
j1R2 + j2R1 R1R2 j0
I = nR + = R
R1 + R2 R1 + R2 R + R0
j1R2 + j2R1 R1R2
where j0 = and R0 =
R1 + R2 R1 + R2
Thus one can replace the given arrangement of the cells by a single cell having the
e.m.f. j0 and internal resistance R0.

3.181 Make the current distribution, as shown in the figure.


Now, in the loop 12341, applying - ¢w = 0, we get R2 x2
6 + – 5
IR + I1R1 + j1 = 0 (1) – + R1 I – I1
1 4
and in the loop 23562, x1 I1
2 3
IR - j2 + (I - I1)R2 = 0 (2) I R
Solving Eqs. (1) and (2), we obtain current through the resistance R,
1j2R1 - j1R22
I = = 0 .02 A
RR1 + RR2 + R1R2
and it is directed from left to the right.

3.182 At first indicate the currents in the branches using charge conservation (which also
includes the point rule).
In the loops 1 BA61 and B34AB, from the loop rule using of - ¢w = 0, we get,
respectively
- j2 + 1I - I12R2 + j1 - I1R1 = 0 (1) I
A
IR3 + j3 - 1I - I12R2 + j2 = 0
6 4
(2) x1 x
R2
R3
On solving Eqs. (1) and (2), we obtain R1 I2 =(I–I1)
1j1 - j22R3 + R21j1 + j32
x2
1 3
I1 = I1 B I1
R1R2 + R2R3 + R3R1
L 0.06 A
Thus, wA - wB = j2 - I2R2 = 0 .9 V
408 PART THREE ELECTRODYNAMICS

3.183 Indicate the currents in all the branches using charge conservation as shown in the
figure. Applying loop rule and using - ¢ w = 0 in the loops 1A781, 1B681 and B456B,
respectively, we get R 2 I1
8 7 6 5
j0 = 1I0 - I12R1 (1)

I3R3 + I1R2 - j0 = 0 (2) x0 +
R1 R3 x1

1I1 - I32R - j - I3 R3 = 0
(I0 – I1) I3 R
(3) 1 4
I0 A I1 B I 4 = (I1–I3)
Solving Eqs. (1), (2) and (3), we get the sought current

j1R2 + R32 + j0R3


1I1 - I32 =
R 1R2 + R32 + R2R3

3.184 Indicate the currents in all the branches using charge conservation as shown in the figure.
Applying the loop rule and using - ¢w = 0 in the loops 12341 and 15781, we get
- j1 + I3R1 - (I1 - I3)R2 = 0 (1) R3 7
8 6
I1 A B
- j2 + (I1 - I3)R2 + I1R3 = 0 (2) I1 x2
R2
1 4
Solving Eqs. (1) and (2), we get I2 5 R1 I3

j11R2 + R32 + j2R2


2 3
x1 I3
I3 =
R1R2 + R2R3 + R3R1

Hence, the sought potential difference


wA - wB = j2 - I3R1
j2R31R1 + R22- j1R11R2 + R32
= = - 1V (on substituting values)
R1R2 + R2R3 + R3R1

3.185 Let us distribute the currents in the paths as shown in the figure.

Now, w1 - w2 = IR1 + I1R2 (1)


2
and w1 - w3 = IR1 + (I - I1)R3 (2)
R2
I1
R1 I
Simplifying Eqs. (1) and (2) we get 1

R31w1 - w22 + R21w1 - w32 R3


I = = 0.2A I2
R1R2 + R2R3 + R3R1 3
3.4 ELECTRIC CURRENT 409

3.186 Distribution of current is as shown in figure. From R1 C I1 −I3


Kirchhoff’s second law
R2
I1R1 = I2R3 I3
A B
R4
I1R1 + (I1 - I3)R2 = V
I2 R3 D I2 +I 3
I2R3 + (I3 + I2) R4 = V

Eliminating I2, we get

I1(R1 + R2) - I3R2 = V


R1
I1 1R3 + R42 + I3R4 = V
R3

R1R2 R1
Hence, I3 c R41R1 + R22 + 1R3 + R42 d = V c1R1 + R22 - 1R3 + R42 d
R3 R3

[R31R1 + R22 - R11R3 + R42]V


I3 =
R3R41R1 + R22 + R1R21R3 + R42

On substitution, we get I3 = 1.0 A from C to D .

3.187 From the symmetry of the problem, current I1 r 5 R I–I1


flow is indicated, as shown in the figure. 6 4
A I –2I1 B
Now, wA - wB = I1r + (I - I1)R (1) r
I I–I1 I
1 3
In the loop 12561, from - ¢w = 0 R 2 r I1
(I - I1) R + (I - 2I1) r - I1r = 0
(R + r)I
or I1 = (2)
3r + R
Equivalent resistance between the terminals A and B using Eqs. (1) and (2),
is given by
R + r
c 1r - R2 + R d I
wA - wB 3r + R r 13R + r2
R0 = = =
I I 3r + R

3.188 Let, at any moment of time, charge on the plates be + q and - q , respectively, then
voltage across the capacitor
w = q /C (1)
410 PART THREE ELECTRODYNAMICS

Now, from charge conservation, 4 I 3 6


dq I1 I2
I = I1 + I2, where I2 = (2) ξ C
+q
dt R –q
In the loop 65146, using - ¢w = 0 1
R 2
5
I I

+ aI1 + bR - j = 0
q dq
(3)
C dt

using Eq. (1) and Eq. (2).


In the loop 25632, using - ¢w = 0
q q
- + I1R = 0 or I1R = (4)
C C
Using Eq. (4) in Eq. (3)
dq 2q dq dt
R = j1 - or = (5)
dt C 2q R
j -
C

On integrating Eq. (5) between suitable limits,


q

2q
3 dq
t j -
0 1 C C t
= dt or - ln =
2q R 3 2 j R
j- 0
C

= V = j11 - e 2t/RC2
q 1
Thus,
C 2

3.189 (a) Since current I is a linear function of time, and at t = 0 and ¢t, it equals I0 and
zero, respectively, it may be represented as

I = I0 a1 - b
t
¢t
¢t ¢t
I0 ¢t
I0 a 1 - b dt =
t
Thus, q = I dt =
3 3 ¢t 2
0 0
2q
So, I0 =
¢t

a1 - b
2q t
Hence, I =
¢t ¢t
The heat generated is given by
3.4 ELECTRIC CURRENT 411

¢t ¢t
4q 2R
c a1 - b d R dt =
2q t 2
H = I 2Rdt =
3 3 ¢t ¢t 3¢t
0 0

(b) Obviously the current through the coil is given by

I = I0 a b
1 t /¢t
2
q q
I0 ¢t
Then, q = Idt = I02 -t /¢t dt =
3 3 ln 2
0 0

q ln 2
So, I0 =
¢t
Hence, heat generated in the circuit in the time interval t [0, q]
q q
q 2 ln 2
c 2 -t /¢t d Rdt = -
q ln 2 2
H = I 2Rdt = R
3 3 ¢t 2¢t
0 0

3.190 The equivalent circuit may be drawn as shown in the figure.


Resistance of the network = R0 + 1R >32.
Let us assume that e.m.f. of the cell is j, then current
R
j
I =
R0 + 1R>32 R
R
Now the thermal power generated in the circuit
j
P = I 2R >3 = 1R>32
I
1R0 + 1R>3222
For P to be maximum, x, R0
dP
= 0, which yields
dR
R = 3R0

3.191 We assume current conservation but not Kirchhoff’s second law. Then thermal
power dissipated is
P 1I12 = I 21 R1 + 1I - I122R2 R1

= I 12 1R1 + R22 - 2II1R 2 + I 2R2 I1

R2 R1R2
= [R1 + R2] cI1 - Id + I2
2 I I
R2
R1 + R2 R1 + R2
I2 = I – I1
412 PART THREE ELECTRODYNAMICS

The resistances being positive, we see that the power dissipated is minimum when
R2
I1 = I
R1 + R2
This corresponds to the usual distribution of currents over resistances joined in parallel.

3.192 Let internal resistance of the cell be r, then


V = j - Ir (1)
where I is the current in the circuit. We know that thermal power generated in the
battery is,
Q = I 2r (2)

Putting r from Eq. (1) in Eq. (2), we obtain


Q = 1j - V 2I = 0.6 W

In a battery, work is done by electric forces whose origin lies in the chemical
processes going on inside the cell. The work so done is stored and used in the elec-
tric circuit outside. Its magnitude just equals the power used in the electric circuit.
We can say net power developed by the electric force is
P = - IV = - 2.0 W
Minus sign means that power is generated and not consumed.

3.193 As far as the motor is concerned, the power delivered is displaced and can be rep-
resented by a load R0. Thus,
V R
I =
R + R0
V R0
V 2R0
and P = I 2R0 =
(R0 + R) 2
This is maximum when R0 = R and the current I is then,
V
I =
2R
The maximum power delivered is
V2
= Pmax
4R
V2 V2
The power input is and its value when P is maximum is .
R + R0 2R
1
The efficiency then is = 50% .
2
3.4 ELECTRIC CURRENT 413

3.194 If the wire diameter decreases by d, then by the information given, power input
V2
P= = H (heat lost through the surface)
R
Now, H r (1 - d) like the surface area and R r (1 - d) -2.

V2
So, (1 - d) 2 = A(1 - d) or V 2 (1 - d) = constant
R0

But , V r 1 + h

So, (1 + h) 2 (1 - d) = constant = 1

Thus, d = 2h = 2%

3.195 The equation of heat balance is


V2 dT
- k (T - T0) = C
R dt
Put T - T0 = j

V2 k V2
So, Cj + k j = or j+ j =
R C CR
d V 2 kt>C
or je kt>C = e
dt CR
V 2 kt>C
or je kt>C = e + A
kR
where A is a constant. Clearly,
V2
j = 0 at t = 0, so A = -
kR

Hence, V2
T = T0 + (1 - e –kt>C)
kR

3.196 Let wA - wB = w. I R
A I'
Now, thermal power generated in the resistance Rx
R2 Rx
w R2 2
P = I ¿ 2Rx = Rx
C R2Rx R2 + Rx S B
R1 +
R2 + Rx
414 PART THREE ELECTRODYNAMICS

For P to be independent of Rx
dP
= 0, which yields
dRx
R1R2
Rx = = 12 Æ
R1 + R2
3.197 Indicate the current flow in the circuit as shown in the figure.
Applying loop rule in the closed loop 12561 and using - ¢w = 0, we get
I1R + IR1 = j1 (1) 6 4
x1 x2
and in the loop 23452,
(I - I1)R2 - I1R = - j2 (2)
R
Solving Eqs. (1) and (2), we get R1 R2
I
j1R2 + j2R1
I1 =
RR1 + R1R2 + RR2 1 I 2 I – I1 3
So, thermal power generated in the resistance R is given by
j1R2 + j2R1
P = I12 R = c d R
2

RR1 + R1R2 + RR2


For P to be maximum,
dP
= 0, which yields
dR
R1R2
R =
R1 + R2
(j1R2 + j2R1)2
Hence, Pmax =
4R1R2 (R1 + R2)

3.198 Suppose that there are x number of cells connected in series, in each of the n par-
allel groups, then
nx = N or x = N /n (1)
Now, for any one of the loop, consisting of x cells and the resistor R, we have from
the loop rule R
I
IR + xr - x j = 0 x xr
n

x xr
xj N/n j
So, I = = (using Eq. 1)
R + xr/n R +Nr/n 2
3.4 ELECTRIC CURRENT 415

Heat generated in the resistor R, is given by

Q = I 2R = a b R
Nnj 2

n2R + NR
and for Q to be maximum,
dQ
= 0, which yields
dn
Nr
n = = 3
AR
3.199 When switch 1 is closed, maximum charge accumulated on the capacitor is given by
qmax = C j (1)

and when switch 2 is closed at any arbitrary instant of time, C

(R1 + R2) a - b =
q
dq
R2
dt c
R1
(because the capacitor is discharging), Sw
q t
2
1 1 1
or dq = - dt
3 q (R1 + R2)C 3
qmax 0 x

Integrating, we get
q -t -t
ln = or q = qmax e (R + R )C (2)
(R1 + R2) C
1 2
qmax

Differentiating with respect to time, we get

= qmax e (R + R )C a- b
dq -t 1
I(t) =
(R1 + R2)C
1 2
dt
Cj -t
or I(t) = e (R + R )C
(R1 + R2)C
1 2

Negative sign is ignored, as we are not interested in the direction of the current.

j -t
Thus, I (t) = e (R + R )C (3)
(R1 + R2)
1 2

When the switch (Sw) is at the position 1, the charge (maximum) accumulated on
the capacitor is, q = C j.
When the switch (Sw) is thrown to position 2, the capacitor starts discharging and
as a result, the electric energy stored in the capacitor totally turns into heat energy
416 PART THREE ELECTRODYNAMICS

through the resistors R1 and R2 (during a very long interval of time). Thus from the
energy conservation, the total heat liberated through the resistors is given by
q2 1
H = Ui = = C j2
2C 2

During the process of discharging of the capacitor, the current through the resistors
R1 and R2 is the same at all moments of time, thus

H1 r R1 and H2 r R2
HR1
So, H1 = (as H = H1 + H2)
(R1 + R2)
1 CR1
Hence, H1 = j2
2 R1 + R2
= 60 mJ (on substituting values)

3.200 When the plate is absent, the capacity of the condenser is


e0S
C =
d

When the plate is present, the capacity is


e0S C
C¿ = =
d (1 - h) 1 - h

(a) The energy increment is clearly


1 1 Ch
¢U = CV 2 - C ¿V 2 = V2
2 2 2(1 - h)
(b) The charge on the plate is
CV
qi = (initially) and qƒ = CV (finally)
1 - h
A charge CV h / 1 - h has flown through the battery, charging it and withdr-
awing CV 2 h / 1 - h units of energy from the system into the battery. The
energy of the capacitor has decreased by just half of this. The remaining half, i.e.,
(1/2) CV 2h / 1 - h is the work done by the external agent in withdrawing the
plate. This ensures conservation of energy.
3.4 ELECTRIC CURRENT 417

3.201 Initially, capacitance of the system = C e.


1
So, initial energy of the system Ui = (C e)V 2
2
1
and final energy of the capacitor Uƒ = CV 2
2
Hence capacitance energy increment
1 1 1
¢U = CV 2 - (Ce)V 2 = - CV 2 (e - 1) = - 0.5 mJ
2 2 2
From energy conservation
¢U = Acell + Aagent
as there is no heat being released.
But, Acell = (Cf - Ci)V 2 = (C - Ce)V 2
Hence, Aagent = ¢U - Acell
1
= C (e - 1)V 2 = 0.5 mJ
2

3.202 If C0 is the initial capacitance of the condenser before water rises in it then
1 e02lpR
Ui = C0V 2, where C0 =
2 d
(R is the mean radius and l is the length of the capacitor plates.)
Suppose the liquid rises to a height h in it. Then capacitance of the condenser is
ee0h2pR e(l - h) 2pR e02pR
C = + = (l + (e - 1) h)
d d d
and energy of the capacitor and the liquid (including both gravitational and electro-
static contributions) is

1 e02pR h
(l + (e - 1) h)V 2 + rg (2pR hd)
2 d 2

If the capacitor were not connected to a battery this energy would have to be min-
imized. But the capacitor is connected to the battery and, in effect the potential en-
ergy of the whole system has to be minimized. Suppose we increase h by dh. Then
the energy the capacitor and the liquid increases by
e0 2pR
dh a (e - 1) V 2 + rg (2pRd) h b
2d
418 PART THREE ELECTRODYNAMICS

and that of the cell diminishes by the quantity Acell which is the product of charge
flown and V
e0 (2pR)
dh (e - 1) V 2
d
In equilibrium, the two must balance, so
e0 (e - 1) V 2
rgdh =
2d
e0 (e - 1) V 2
Hence, h =
2rgd 2
3.203 (a) Let us imagine a thin spherical layer with inner and outer radii r and r + dr, re-
spectively. Lines of current at all the points of this layer are perpendicular to it
and therefore such a layer can be treated as a spherical conductor of thickness
dr and cross sectional area 4pr 2, Now, we know that resistance,
dr dr
dR = r = r (1)
S (r) 4pr 2
Integrating Eq. (1) between the limits,
R b

c - d
dr r 1 1
dR = r or R = (2)
3 3 4pr 2 4p a b
0 a
Capacitance of the network,
4pe0e
C = (3)
c - d
1 1
a b

q = Cw c d
where q is the charge
and (4)
at any arbitrary moment
- dq
also, w = a b R, as capacitor is discharging. (5)
dt
From Eqs. (2), (3), (4) and (5), we get

c- drc - d
dq 1 1
4 p e0e dt a b dq dt
q = or =
ree0
c - d
1 1 4p q
a b
q t
dq 1 dt
Integrating - = dt =
3 q re0e 3 ree0
q0 0

Hence, q = q0e-t/re0e
3.4 ELECTRIC CURRENT 419

(b) From energy conservation, heat generated during the spreading of the charge
H = Ui - Uf (because Acell = 0)
2 q02 b - a
1 q0
c - d - 0 =
1 1
=
2 4pe0e a b 8pe0e ab

3.204 (a) Let, at any moment of time, charge on the plates be (q0 - q) then, current through
the resistor
d (q0 - q)
I = -
dt
because the capacitor is discharging.
dq –(q0 – q) (q0 – q)
or I =
dt C
Now, applying loop rule in the circuit
q0 - q R
IR - = 0
C dq
I=
dq q0 - q dt
or R =
dt C
dq 1
or = dt
q0 - q RC
At t = 0, q = 0 and at t = t, q = q .
q0 - q -t
So, ln =
q0 RC
Thus, q = q0 (1 - e –t>RC) = 0.18 mC

(b) Amount of heat generated = decrease in capacitance energy

1 3q0 - q0 (1 - e
2 2
–t>RC )4
1 q0
= -
2 C 2 C
2
1 q0
= [1 - e-2t>RC ] = 82 mJ
2 C
3.205 Let at any moment of time, charge flown be q, then current I = dq /dt .
Applying loop rule in the circuit and using ¢w = 0, we get
dq (CV0 - q) q
R - + = 0
dt C C
dq 1
or = dt
CV0 - 2q RC
420 PART THREE ELECTRODYNAMICS

CV0 - 2q t R
So, ln = -2 (for 0 … t … t ) –(CV0 – q)
CV0 RC –q
C C
CV0 +q
or q = (1 - e -2t>RC ) CV0 – q
2 dq
iI =
CV0 2 V0 dt
dq
Hence, I = = e –2t>RC = e –2t>RC
dt 2 RC R
Now, heat liberated will be given by
q q
V02 1
Q = I 2Rdt = R e -4t>RC dt = CV02
3 R2 3 4
0 0

3.206 In a rotating frame, to first order in v, the main effect is a Coriolis force given by
2m (v * v). This unbalanced force will cause electrons to react by setting up a mag-
netic field B so that the magnetic force e (v * B) balances the Coriolis force.
e 2m
Thus, - B = v or B = - v
2m e
The flux associated with this is
2m
£ = Npr 2B = Npr 2 v
e

where N = l / 2pr is the number of turns of the ring. If v changes (and there is
time for the electron to rearrange) then, B also changes and so does £ . An e.m.f.
will be induced and a current will flow, which is given by
2m
I = N pr 2
v>R
e
The total charge flowing through the ballastic galvanometer, as the ring is stopped, is

q = N pr 2 n
2m
v>R
e
e 2Npr 2v lvr
So, = =
m qR qR
= 1.8 * 1011 C>kg (on substituting values)

3.207 Let n0 be the total number of electrons then, total momentum of electrons,

p = n0mevd (where vd is drift velocity) (1)

n0e ne
Now, I = rSxvd = Sxvd = v (2)
V l d
3.4 ELECTRIC CURRENT 421

Here Sx = cross sectional area, r = electron charge density, V = volume of sample.


From Eqs. (1) and (2)
me
p = Il = 0.40 m Ns
e
3.208 By definition nevd = j , where vd is the drift velocity, n is number density of electr-
ons.

l nel
Then t = =
vd j
So distance actually travelled
nel 6 v 7
S = 6 v7 t = = 107 m
j
(6 v 7 = mean velocity of thermal motion of an electron.)

3.209 (a) Let n be the volume density of electrons, then from I = rSvd , we get
l
I = neS | 6 v7 | = neS
t
neSl
So, t = = 3 Ms
I
(b) Sum of electric forces is given by
| (nv) e E | = | n Slerj | (where r is resistivity of the material)
I
= nSler = nel rI = 1.0 MN
S

3.210 From Gauss’ theorem, field strength at a surface of a cylindrical shape equals,
l / 2pe0 r , where l is the linear charge density.

1 2eV
Now, eV = m v 2 or v = (1)
2 e A me
dq dx
Also, dq = ldx so, = l
dt dt
I I
or I = lv or l = = (using Eq. 1)
v 2eV
A me
I me
Hence, E = = 32 V/m
2pe0r A 2eV
422 PART THREE ELECTRODYNAMICS

For the point inside the solid charged cylinder, applying Gauss’ theorem
q
2prhE = pr 2 h
e0pR 2l
q>l lr
or E = r =
2pe0R 2 2pe0R 2
dw
So, from E = -
dr
w2 R
l
we get - dw = 2
rdr
3 3 2pe0R
w1 0

c d =
l r2 R l
or w1 - w2 = 2
2pe0R 2 0 4pe0
I me
Hence, w1 - w2 = = 0.80 V
4pe0 A 2eV

3.211 (a) Between the plates


w = ax 4>3
0w 4
or = a * x l>3
0x 3
0 2w 4 -r
= ax -2>3 =
0x 2 9 e0
4e0a
or r = - x -2>3
9
(b) Let the charge on the electron be –e.
1
Then mv 2 - ew = constant = 0
2
(as the electron is initially emitted with negligible energy),

2e w 2e w
or v2 = and v =
m A m
4e0a 2we -2>3 4 2e
So, j = - rv = x = e0a 3>2
9 A m 9 Am
( j is measured from the anode to cathode, hence the - ve sign.)

3.212 We know that, E =V


d
3.4 ELECTRIC CURRENT 423

So by the definition of the mobility


V V
v + = u0+ , v - = u0– -
d d
eV
and j = (n + u0+ + n– u –0)
d
The negative ions move towards the anode and the positive ions towards the catho-
de and the total current is the sum of the currents due to them.
On the other hand, in equilibrium
n+ = n-

n (u 0+ + u –0)
I eV
So, n+ = n- =
S d
Id
= = 2.3 * 10 8 cm -3
eVS (u 0+ + u –0)
3.213 Velocity = mobility * field
V0
or v = u sin vt (which is positive for 0 … vt … p)
l
So, the maximum displacement in one direction is
p
V0 2uV0
xmax = u sin vtdt =
3 l lv
0
2uV0
At v = v0, xmax = l so, = l
lv0
v0l 2
Thus, u =
2V0
3.214 When the current is saturated, all the ions produced reach the plate.
# Isat
Then, ni = = 6 * 10 9 cm -3 s -1
eV
(Both positive ions and negative ions are counted here.)
The equation of balance is
dn #
= ni - rn 2
dt
The first term on the right is the production rate and the second term is the recom-
bination rate which by the usual statistical arguments is proportional to n 2 (no of
positive ions * no. of –ve ions). In equilibrium
dn
= 0
dt
#
ni
So, neq = = 6 * 10 7 cm -3
Br
424 PART THREE ELECTRODYNAMICS

#
3.215 Initially, n = n0 = 2ni >r
Since we can assume that long exposure to the ionizer has caused equilibrium to be
set up. After the ionizer is switched off,
dn
= - rn 2
dt
dn 1
or rdt = - or rt = + constant
n2 n
1 1
But, n = n0 at t = 0 so, rt = -
n n0
The concentration will decrease by a factor h when
1 1 h - 1
rt0 = - =
n0 >h n0 n0
h - 1
or t0 = # = 13 ms
2r ni

3.216 Ions produced will cause charge to decay. Clearly,


# e0A
hCV = decrease of charge = ni eAdt = Vh
d
e0Vh
or t = # = 4.6 days
ni ed 2
Note that ni here, is the number of ion pairs produced.

3.217 If n = number of electrons moving to the anode at distance x, then


dv
= av or v = v0 e ax
dx
Assuming saturation,
I = ev0e ad

3.218 Since the electrons are produced uniformly through the volume, the total current at-
taining saturation is clearly,
d
# # e ad - 1
I = e (ni Adx) e ax = eni A a b
3 a
0

# e ad - 1
j = A = eni a b
I
Thus,
a
3.5 CONSTANT MAGNETIC FIELD. MAGNETICS 425

3.5 Constant Magnetic Field. Magnetics

3.219 (a) From Biot-Savart’s law


m0 d l * r
dB = I
4p r3
m0 1Rd u2R
So, dB = I (as d l ⬜ r)
4p R3
From symmetry
2p
m0 I m0 I
B = dB = du = = 6.3 mT
L 3 4p R 2 R
0

(b) From Biot-Savart’s law n


m0
dl * r dB
B = I (here r = x - R)
4p L r 3
m0 I
So, B = c dl * x - dl * Rd
4p r 3 C C x
r
Since x is a constant vector and |R| is also constant, so,

dl * x = a dlb * x = 0 abecause dl = 0 b O dl
R
I I I
I
and - dl * R = R dl n = nR dl = 2pR 2 n
I I I
Here n is a unit vector perpendicular to the plane containing the current loop
(see figure) and in the direction of x.
m0 2 pR 2I
B =
4p 1x 2 + R 223>2
Thus, we get n

= 2.3 mT

3.220 Since l AOB = 2p/n ,OC or perpendicular distance of any segment from center equals
R cos p/n . Now magnetic induction at O, due to the right current carrying element AB
m0 I p
= 2 sin
4p R cos p/n n
From Biot–Savart’s law, the magnetic field at O due to any section such as AB is per-
pendicular to the plane of the figure and has the magnitude
p
n
m0 dx m0 I R cos p/n sec2udu m0I 1 p
B = I 2 cos u = 2 2
cos u = 2 sin
L 4p r 3p 4p R cos 2p/n sec u 4p R cos p/n n
-n
426 PART THREE ELECTRODYNAMICS

As there are n number of sides and magnetic induction vectors, due to each side at
O, are equal in magnitude and direction. So
m0
2 sin # n
nI p
B0 =
4p p n
R cos
n I O
m0 nI p 2
= tan A n dq R
2p R n
m0 I Lim tan pn
a b
C B
For n : q B0 =
2 R n : q p>n
m0 I
=
2 R

3.221 We know that magnetic induction due to a straight current carrying wire at any point,
at a perpendicular distance from it is given by
m0 I
B = 1sin u1 + sin u22 (as shown in figure) I
4p r q2
r
m0 q1
e cos + cos f
I w w
Here, B1 = B3 =
4p w 2 2
(d /2) sin
2
m0
asin + sin b
I w w
and B2 = B4 = 4
4p w 2 2
(d>2) cos
2 p –j j
2 2
1 3
Hence, the magnitude of total magnetic induction at O, O
d
2
B0 = B1 + B2 + B3 + B4 2

w w
m0 4I cos 2 sin
2
= D + T
4p d>2 w w
sin cos
2 2
4 m0 I
= = 0.10 mT
pd sin w

3.222 Magnetic induction due to the arc segment at O is given by I

m0 I O
Barc = (2p - 2 w)
4p R 2j R
3.5 CONSTANT MAGNETIC FIELD. MAGNETICS 427

and magnetic induction due to the line segment at O is given by


m0 I
Bline = [2 sin w]
4p R cos w
So, total magnetic induction
m0 I
B0 = Barc + Bline = [p - w + tan w] = 28 mT
2p R
3.223 (a) From the Biot-Savart law
m0 (d l * r)
dB = I
4p r3
So, magnetic field induction due to the arc of radius a at O is given by
m0 I
Ba = (2p - w) (directed inward)
4p a
and magnetic field induction due to the arc of radius b at O is given by
m0 I
Bb = w (directed inward)
4p a I
2
Due to each of straight current segments 1 and 2, magnetic
induction is zero, because d l 7 r. j b
a
m0 2p - w
I c + d
w O I
Hence, B0 = I
4p a b
(b) Due to each of straight currents directed towards (away)
center, magnetic induction vector at the center is zero. From b
the symmetry, direction of magnetic induction vector due to
rest of two straight wires and the arc current, all are directed
normally into the plane of the figure. Magnetic induction due b
a 45°
to each of straight current is O ƒ
m0 I
sin 45°
4p b
and due to arc current is
m0 I 3p
a b
4p a 2
Hence, the total magnetic induction at O is given by
m0 I 3p m0 I m0 I 3p 22
B0 = a b + 2 * sin 45° = c + d
4p a 2 4p h 4p n 2a b
3.224 The thin walled tube with a longitudinal slit can be considered equivalent to a full tube
and a strip carrying the same current density in the opposite direction. Inside the tube,
the former does not contribute so the total magnetic field is simply due to the strip. It is
m0 (I>2pR) h m0 Ih
B = =
2p r 4p 2 Rr
where r is the distance of the field point from the strip.
428 PART THREE ELECTRODYNAMICS

3.225 First of all let us find out the direction of vector B at point O. For this purpose, we
divide the entire conductor into elementary fragments with current dI. It is obvious
that the sum of any two symmetric fragments gives a resultant along B as shown in
the figure and consequently, vector B will also be directed as shown.
O
So, | B| = dB sin w j j
3 dj
m0 dB
= dI sin w B
3 2pR
p
m0
I sin w dw aas dI = dwb
I
=
3 2p 2R p
0

m0 I
Hence, B =
p2R

3.226 From symmetry, for Fig. (a)


2
B0 = B1 + B2 + B3 1 R
O 3
m0 I m0 I
= 0 + p + 0 =
4p R 4 R (a)

From symmetry, for Fig. (b)


B0 = B1 + B2 + B3 2
m0 I m0 I 3p R 3
= + + 0 O
4p R 2p R 2
m0 I 1
c1 + d
3p
= (b)
4p R 2

From symmetry, for Fig. (c)


3
B0 = B1 + B2 + B3
2 R
m0 I m0 I m0 I O
= + p +
4p R 4p R 4p R 1
m0 I (c)
= (2 + p)
4p R

3.227 B0 = B1 + B2
l
m0I 1 /2
or | B0 | = 12 = 2.0 mT (using Problem 3.221) /2
4pl
2
3.5 CONSTANT MAGNETIC FIELD. MAGNETICS 429

3.228 (a) B0 = B1 + B2 + B3
m0 I m0 I m0 I
= ( - k) + p ( - i) + ( - k)
4p R 4p R 4p R
m0 I
= - [2k + p i]
4p R
m0 I
So, | B0 | = 2p 2 + 4 = 0.30 mT
4p R
(b) B0 = B1 + B2 + B3
m0 I m0 I m0 I
= ( - k) + p ( - i) + ( - i)
4p R 4p R 4p R
m0 I
= - [k + (p + 1) i]
4p R
m0 I
So, | B0 | = 21 + (p + 1) 2 = 0.34 mT
4p R
(c) Here, using the law of parallel resistances
I1 1
I1 + I2 = I and =
I2 3
I1 + I2 4
So, =
I2 3
3 1
Hence, I2 = I and I1 = I
4 4
m0 I m0 I m0 3p I1 m0 (p>2) I2
Thus, B0 = ( - k) + (- j) + c a b ( - i) + id
4p R 4p R 4p 2 R 4p R
- m0 I
= ( j + k) + 0
4p R
m0 12I
Thus, | B0 | = = 0.11 mT
4p R
3.229 (a) We apply circulation theorem as shown in the figure. The current is vertically up-
wards in the plane and the magnetic field is horizontal and parallel to the plane.

m0I
B # d l = 2 Bl = m0 Il or
B
Then, B =
C 2

(b) Each plane contributes m0 I / 2 between the planes and P


outside the plane that is cancelled out.
430 PART THREE ELECTRODYNAMICS

B = e
m0 I between the plane
Thus,
0 outside
3.230 We assume that the current flows perpendicular to the plane of the paper, x=0
then by circulation theorem, inside the plate

2Bdl = m0 (2 x dl) j
or B = m0 x j, | x | … d
Outside the plate, 2Bdl = m0 (2d dl) j
or B = m0 dj | x | Ú d

3.231 It is easy to convince oneself that in both the regions 1 and 2, there can only be a
circular magnetic field (i.e., the component Bw). Any radial field in region 1 or any Bz
away from the current plane will imply a violation of Gauss’ law of magnetostatics.
Bw must obviously be symmetrical about the straight wire. Then in 1,

Bw 2pr = m0I
m0 I 1 I
or Bw =
2p r
O
Bw # 2pr = 0 or
2
In 2, Bw = 0

3.232 Along the axis of loop,


m0IR 2
B = = Bx (along the axis)
2 (R 2 + x 2) 3>2
q q
m0IR 2
B # dr =
dx
Thus, Bxdx =
3 3 2 3q (R 2 + x 2) 3>2
-q -

p>2
m0 IR 2 R sec 2 ud u
= 3 3
(on putting x = R tan u)
2 3 R sec u
-p>2

p>2
1
= m0I cos u d u = m0I
2 3
-p>2
q

The physical interpretation of this result is that Bx dx can be thought of as the


L- q
circulation of B over a closed loop by imaging that the two ends of the axis are con-
nected by a line at infinity (e.g., a semicircle of infinite radius).
3.5 CONSTANT MAGNETIC FIELD. MAGNETICS 431

3.233 By circulation theorem, inside the conductor


Bw2pr = m0 jzp r 2 or Bw = m0 jz r/2

1
i.e., B = m j * r
2 0

Similarly, outside the conductor

1 R2
Bw 2pr = m0 jz pR 2 or Bw = m0 jz
2 r
1 R2
So, B = m0 (j * r) 2
2 r

2.234 The given current, which will be assumed uniform, arises due
to a negative current flowing in the cavity superimposed on
the true current everywhere including the cavity. Then from r
the previous problem, by superposition, r'
O l O'
m0 m0 m0
B = [j * r] - [j * r¿] = [j * (r - r¿)]
2 2 2
m0
B = (j * l)
2
If l vanishes so that the cavity is concentric with the conductor, then there is no
magnetic field in the cavity.
3.235 By circulation theorem
r

Bw # 2 p r = m0 j 1r ¿2 * 2pr ¿dr ¿
3
0

Using Bw = br a, inside the stream,


r

br a + 1 = m0 j 1r ¿2r ¿dr ¿
3
0
So by differentiation,
(a + 1)br a = m0 j (r) r
b (a + 1) -1
Hence, j (r) = r–
m0

3.236 On the surface of the solenoid there is a surface current density, given by
js = nI ew
432 PART THREE ELECTRODYNAMICS

m0 ew * r0
Then, B = - nI R dw dz
4p L r03

where r0 is the vector from the current element to the field point, which is the center
of the solenoid,
Now, - ew * r0 = R ez

r0 = 1z 2 + R 221>2

l>2
m0nI dz
B = Bz = * 2pR 2
1R 2 + z 22 3>2
Thus,
4p 3
-l>2
l
+ tan-1 2R
1
= m nI cos a d a (on putting z = R tan a)
2 0 L l
− tan
-1
2R

l /2
= m0n I sin a = m0 nI
3(l /2) 2 + R 2

1 + a b
2R 2
= m0 n I>
B l

3.237 (a) We proceed exactly as in the previous problem. Then the magnetic induction
on the axis at a distance x from one end is clearly,
q q
m0 nl dz 1 dz
B = * 2pR 2 = m0nI R 2
4p 3 [R 2 + (z - x) 2] 3/2 2 3 1z 2 + R 22 3/2
0 x

p>2

cos u d u = m0nI a 1 - b
1 1 x
= m0 nI
2 3
-1 x
2 2x 2 + R 2
tan R

x 7 0 means that the field point is outside the solenoid. B then decreases with
( x ). x 6 0 means that the point gets more and more inside the solenoid. B then
increases with (x) and eventually becomes constant, equal to m0nI. The B - x
graph is as given in the answer sheet.

(b) We have,
B0 - dB x0
c1 - d = 1 - h
1
=
B0 2 2R 2 + x0 2
3.5 CONSTANT MAGNETIC FIELD. MAGNETICS 433

x0
or – = 1 - 2h
2R 2 + x0 2
Since h is small (L 1%), x0 must be negative.
Thus, x0 = - | x0 |
| x0 |
and = 1 - 2h
2R 2 + | x0 | 2
| x0 | 2 = (1 - 4 h + 4 h2) (R2 + | x0 | 2)
0 = (1 - 2h)2 R2 - 4 h (1 - h) | x0 | 2
(1 - 2 h) R
or | x0 | =
2 2h(1 - h)
L 5R

3.238 If the strip is tightly wound, it must have a pitch of h. This means that the current
will flow obliquely, partly along ew and partly along ez. Obviously, the surface cur-
rent density is

c 21 - (h/2 pR )2 ew + ez d
I h
js =
h 2pR
By comparison with the case of a solenoid and a hollow straight conductor, we see
6
that field inside the coil (r R ) is
I
= m0 21 - (h/2pR)2
h
(Cf. B = m0 nI )

For the field outside the coil (r 6 R ), only the other term contributes, so

I h
Bw * 2pr = m0 * * 2pR
h 2pR
m0 2I
or Bw = #
4p r
= 0 .3 mT
Note: Surface current density is defined as current flowing normally across a unit
length over a surface.

3.239 Suppose a is the radius of cross-section of the core. The winding has a pitch 2pR/N,
so the surface current density is
I I
js ⬵ e1 + e
2pR/N 2pa 2
434 PART THREE ELECTRODYNAMICS

where e1 is a unit vector along the cross section of the core and e2 is a unit vector
along its length.
The magnetic field inside the cross section of the core is due to first term above, and
is given by
Bw # 2 pR = m0 NI
(NI is total current due to the above surface current (first term).)
Thus, Bw = m0 NI/2 pR.
The magnetic field at the center of the core can be obtained from the basic formula:
m0 js * r0
dB = dS
4p r 30
and is due to the second term.
m0 I 1
So, B = Bzez = ez Rdw * 2pa
4p 2pa L R 3
m0I
or Bz =
2R
N
The ratio of the two magnetic fields, h = = 8 * 102
p

3.240 We need the flux through the shaded area.


Now by Ampere’s theorem,
I # 2
Bw 2pr = m0 pr R
pR 2
m0 r
or Bw = I
2p R2
The flux through the shaded region is
R

£1 = 1 # d r#Bw(r)
3
0
R
m0 r m0
= dr I 2
= I
3 2p R 4p
0

= 1.0 mWb/m

3.241 Using solution of Problem 3.237, the magnetic field is given by,

m nI a 1 - b
1 x
B =
2 0 2x + R 2
2
3.5 CONSTANT MAGNETIC FIELD. MAGNETICS 435

1 1
At the end, B = m n I = B0 (where B0 = m0 nI )
2 0 2
is the field deep inside the solenoid. Thus,
1 £
£ = m0 nIS = where £ = m0nIS
2 2
is the flux of the vector B through the cross-section deep inside the solenoid.

3.242 Bw2 pr = m0NI


m0NI
or Bw =
2pR
a

Then, £ = Bwhdr (for a … r … b )


3
b
m0
awhere h = b
b
= 2NIh ln h
4p a
= 8 m Wb
3.243 Magnetic moment of a current loop is given by pm = n I S , where n is the number of
turns and S is the cross sectional area. In our problem,
m0 I
n = 1, S = pR 2 and B =
2 R
2BR 2pBR 3
So, pm = pR 2 = = 30 mA m2
m0 m0
N #
3.244 Take an element of length r d u containing rd u turns. Its magnetic moment is
pr

d u # d 2I
N p
p 4
A
normal to the plane of cross section. We resolve it along OA
and OB. The moment along OA integrates to q
p O
N 2 B
pm = d I cos u d u = 0
3 4
0 q
while that along OB gives
p
Nd 2I 1
pm = sin u d u = Nd 2I = 0.5 Am2
3 4 2
0
3.245 (a) From Biot-Savart’s law, the magnetic induction due to a circular current carrying
wire loop at its center is given by
m0
Br = I
2r
436 PART THREE ELECTRODYNAMICS

The plane spiral is made up of concentric circular loops, having different radii,
varying from a to b. Therefore, the total magnetic induction at the center,
m0
B0 = I dN (1)
m0 3 2r
where I is the contribution of one turn of radius r and dN is the number of
2r
turns in the interval (r, r + dr), i.e.,
N
dN = dr
b - a
Substituting in Eq. (1) and integrating the result over r between a and b, we
obtain
b
m0IN m0IN b
B0 = dr = ln = 7 mT
3 2r (b - a) 2(b - a) a
a
(b) The magnetic moment of a turn of radius r is pm = I p r 2 and for all turns,
b
N pIN (b 3 - a 3)
p = pmdN = Ipr 2 dr =
3 3 b - a 3 (b - a)
a
= 15 mA m2
3.246 (a) Let us take a ring element of radius r and thickness dr. Then charge on the ring
element
dq = s 2prdr
and current due to this element
(s2prdr) v
dI = = s vr dr
2p
So, magnetic induction at the center, due to this element
m0 dI
dB =
2 r
and hence, from symmetry
R
m0 srdr m0
B = dB = = sR
3 3 r 2
0
(b) Magnetic moment of the element
dpm = (dI)pr 2 = sdr pr 2 = spr 3 dr
Hence, the sought magnetic moment
R
R4
pm = dpm = spr 3dr = sp
3 3 4
0
3.5 CONSTANT MAGNETIC FIELD. MAGNETICS 437

3.247 Since only the outer surface of the sphere is charged, consider the element as a ring,
as shown in the figure.
The equivalent current due to the ring element r dq

r sinq
dI = (2pr sin u rd u) s (1) dq`
2p q
w
O
and magnetic induction due to this loop element at the
center of the sphere O is given by
m0 2pr sin ur sin u m0 sin 2 u
dB = dI = dI
4p r3 4p r
Hence, the total magnetic induction due to the sphere at the center O is given by
p>2
m0
v 2pr 2 sin ud u sin 2u s
B = dB = (using Eq. 1)
3 3 4p 2p r
0

p>2
m0svr 2
Hence, B = sin 3 u d u = m svr = 29 pT
3 4p 3 0
0

3.248 The magnetic moment must clearly be along the axis of rotation. Consider a volume
element dV . It contains a charge 3 q /4 pR 3 dV. The rotation of the sphere causes this
charge to revolve around the axis and constitute a current given by
3q v
dV *
4pR 3 2p
Its magnetic moment will be

3q v
dV * * pr 2 sin2 u
4pR 3 2p
So the total magnetic moment is
R p
3q r 2 sin 2u
pm = 3
r 2 sin u d u * dr
3 3 2R 2
0 0
3q  R5 4 1
= * * * = qR 2 
2R 3 2 5 3 5
The mechanical moment is
2 pm q
M = mR 2 v so, =
5 M 2m
438 PART THREE ELECTRODYNAMICS

3.249 Because of polarization, a space charge is present within the cylinder. Its density is
rp = - div P = - 2a
Since the cylinder as a whole is neutral, a surface charge density sp , must be present
on the surface of the cylinder also. This has the magnitude (algebraically)
sp * 2pR = 2apR 2 or sp = aR
When the cylinder rotates, currents are set up which give rise to magnetic fields. The
contribution of rp and sp can be calculated separately and then added.
For the surface charge the current is (for a particular element),
v
sR * 2pR dx * = aR 2v dx
2p
Its contribution to the magnetic field at the center is
m0R 2 (aR 2  dx)
2(x 2 + R 2) 3/2
and the total magnetic field is
-q q
m0R 2 (aR 2 v dx) m0aR 4v dx
Bs = =
3 2(x 2 + R 2) 3/2 2 3 (x 2 + R 2) 3/2
q -q
m0aR 4v
2
= * 2 = m0aR 2v
2 R
For the volume charge density, consider a circle of radius r, radial thickness dr and
length dx.
The current is

- 2a * 2pr dr dx * = - 2 ardr  dx
2p
The total magnetic field due to the volume charge distribution is
R q R q
m 0r 2
Bv = - dr dx 2p r v = - am0 vr 3dr dx (x 2 + r 2) 3/2
3 3q 2(x 2 + r 2) 3/2 3 3q
0 - 0 -
R

= - am0 vr * 2 = - m0 a v R 2 so, B = Bs + Bv = 0
3
0

3.250 Force of magnetic interaction, Fmag = e (v * B)


m0 e (v * r)
where, B =
4p r3
3.5 CONSTANT MAGNETIC FIELD. MAGNETICS 439

m0 e 2
So, Fmag = [v * (v * r)]
4p r 3
m0 e 2 m0 e 2
= 3
[(v # r) * v - (v # v) * r] = ( - v 2r)
4p r 4p r 3
1 er
and Fele = eE = e
4pe0 |r|3

|Fmag|
= - v 2m0e0 = a b = 1.00 * 10 -6
v 2
Hence,
|Fele| c

3.251 (a) The magnetic field at O is only due to the curved path, as for the line element,
d l c c r.
y
m0I m0I
Hence, B = p(- k) = ( - k)
4pR 4R
I
m0I 2
Thus, Fu = IB ( - j) = ( - j)
4R R
p0I 2 O
So, Fu = = 0.20 mN/m x
4R
z
(b) In this part, magnetic induction B at O will be effective only due to the two semi
infinite segments of wire. Hence,
m0I p y
B = 2. sin ( - k)
4p a b
l 2
2 B
m0I l
= ( - k) x
pl O
Thus force per unit length,
m0l 2
Fu = ( - i)
pl
= 0.13 mN/m

3.252 Each element of length dl = Rda experiences a force BI Rda, where da is the angle
subtended by the element at the center. This causes a tension T in the wire.
For equilibrium, ampere force BI Rda must be balanced by
the radial component of tensile forces, which is
2T sin (da/2) = Tda da
T 2 T
So, Tda = BI Rda,
440 PART THREE ELECTRODYNAMICS

Hence, T = BIR
Thus, the wire experiences a stress
BIR
pd 2/4
This must equal the breaking stress sm for rupture. Thus,

pd 2 sm
Bmax =
4IR
= 8 kT
3.253 The Ampere forces on the sides OP and O ¿P ¿ at their mid points are directed along
the same line, in opposite directions and have equal values, hence the net force as
well as the net torque of these forces about the axis OO¿ is zero. The Ampere force
on the segment PP ¿ and the corresponding moment of this force about the axis OO¿
is effective and deflecting in nature.
The weight of each segment develops restoring torque. Let the length of each side
be l and r be the density of the material then, mass of each segment m = Slr. The
Ampere force on the segment PP ¿ is IlB, from the formula of Ampere force on a
straight wire in a uniform magnetic field F = I (l * B).
In equilibrium the deflecting torque must be equal to the restoring torque. So,
Fl cos w = 2mg (l/2) sin w + mgl sin w
B O'
I lB (l cos w) = 2Srgl 2 sin w
O
2Srg
Hence, B = tan w (Sl g )
(Sl g )
I
= 10 mT
P P'
3.254 We know that the torque acting on a magnetic dipole ilB
(Sl g )
N = pm * B
But, pm = ISn, where n is the normal on the plane of the loop and is directed in
the direction of advancement of a right handed screw, if we rotate the screw in the
sense of current in the loop.
On passing current through the coil, the torque
acting on the magnetic dipole is counterbalanced
by the moment of additional weight about O. N
l
Hence, the direction of current in the loop must be pm O
in the direction shown in the figure. S
B
pm * B = - l * ¢mg
3.5 CONSTANT MAGNETIC FIELD. MAGNETICS 441

or NI SB = ¢ mgl
¢mgl
So, B = = 0.4 T (on substituting values).
NIS

3.255 (a) As is clear from the condition, Ampere’s forces on the sides 2 and 4 are equal in
magnitude but opposite in direction. Hence, the net effective force on the fr-
ame is the resultant of the forces experienced by sides 1 and 3.
Now, the Ampere force on side 1,
m0 II0 ha
F1 = 2

ah - b
2p 1 I 3
2
I0
and that on side 3, 1
I
m0 I0I
F3 =
a
ah + b
2p 1 4
2
So, the resultant force on the frame = F1 - F3 (as they are opposite in nature).
2m0 II0
= = 0.40 mN
p(4h 2 - 1)
(b) Work done in turning the frame through some angle,
A = 1 Id £ = I (£ f - £ i )
where £f is the flux through the frame in final position and £ i in the initial
position.
Here, |£ f | = |£ i | = £ and £ i = - £ f

So, ¢£ = 2 £ and A = I 2 £
Hence, A = 2I 1 B # d S
a (h + 12 )
m0 I0a m0 II0a 2h + 1
= 2I dr = ln a b
3 2p r p 2h - 1
a (h - 12 )

= 0.10 m J

3.256 There are excess surface charges on each wire (irrespective of whether the current is
flowing through them or not). Hence, in addition to the magnetic force Fmag, we must
take into account the electric force Fele. Suppose that an excess charge l corresponds
442 PART THREE ELECTRODYNAMICS

to a unit length of the wire, then electric force exerted per unit length of the wire by
other wire can be found with the help of Gauss’ theorem.
1 2l 2l 2
Fele = lE = l = (1)
4pe0 l 4pe0l
where l is the distance between the axes of the wires. The mag-
++ + + + + + + + +
netic force acting per unit length of the wire can be found with
the help of the theorem on circulation of vector B E Fele R

m0 2I 2 –––––––––––
Fmag = F
4p l
where I is the current in the wire. (2)
Now, from the relation, l = C w, where C is the capacitance of the wires per unit
length, as given in solution Problem 3.108 and w = IR. Substituting we get,
pe0 I ln h
l = IR or = (3)
ln h l pe0R
I
Dividing Eq. (2) by (1) and then substituting the value of from Eq. (3), we get,
l
Fm m0 (ln h) 2
=
Fe e0 p 2R 2

The resultant force of interaction vanishes when this ratio equals unity. This is pos-
sible when R = R0, where
m0 ln h
R0 = = 0.36 kÆ
A e0 p

3.257 Using solution of Problem 3.225, the magnetic field due to the conductor with semi-
circular cross-section is given by,
m0I
B =
p 2R

0F m0I 2
Then = BI =
0l p 2R

3.258 We know that Ampere’s force per unit length on a wire element in a magnetic field
is given by
d Fn = I (n * B) (1)

where n is the unit vector along the direction of current.


3.5 CONSTANT MAGNETIC FIELD. MAGNETICS 443

Now, let us take an element of the conductor I2, as shown in the figure. This wire
element is in the magnetic field, produced by the current I1, which is directed nor-
mally into the sheet of the paper and its magnitude is given by

m0I1
| B| = (2)
2pr
From Eqs. (1) and (2)
I2
d Fn = dr (n * B) (because the current through the element = I2 /b dr)
b
m0 I1I2 dr
So, d Fn = (towards left, as n ⬜ B)
2p b r
Hence the magnetic force on the conductor:
a +b
m0 I1I2 r
dr dr
Fn = (towards left)
2p b 3 r I1
a I2
m0 I1I2 a + b a
= ln (towards left)
2p b a b
Then according to the Newton’s third law, the magnitude of sought magnetic inter-
action force
m0I1I2 a + b
= ln
2p b a
3.259 By the circulation theorem B = m0I, where I = current per unit length flowing alo-
ng the plane perpendicular to the paper. Currents flow in the opposite paper.
Currents flow in the opposite sense in the two planes and produce the given
field B by superposition. The field due to one of the plates is just 1/2 B. The force
on the plate is 1/2 B * I * length * breadth = B 2 / 2m0 per unit area.
−B
(Recall the formula F = Bll on a straight wire.)

3.260 In Fig. (a), the external field must be B1 + B2 / 2 , which when superposed with the
internal field B1 - B2 / 2 (of opposite sign on the two sides of the B1 B2
plate), must give the actual field.

B1 - B2 1
Now = mI
2 2 0
B1 - B2
or I = (a)
m0
444 PART THREE ELECTRODYNAMICS

B12 - B22
Thus, F =
2m0

In Fig. (b), the external field must be B1 - B2 / 2 upward B1 B2

with an internal field, B1 + B2 / 2 , upward on the left and do-


wnward on the right. Thus,
B1 + B2 B1 2 - B2 2
I = and F =
m0 2m0
(b)
In Fig. (c), the boundary condition from Gauss’ theorem is, B1 cos u1 = B2 cos u2. Also
(B1 sin u1 + B2 sin u2) = m0I , where I = current per unit length. The external field
parallel to the plate must be

B1 sin u1 - B2 sin u2
2
The perpendicular component B 1 cos u1, does not matter since the q1 q2
corresponding force is tangential.

B 12 sin 2 u1 - B 22 sin 2 u2
Thus, F = per unit area
2m0
B 12 - B 22
= per unit area B1 B2
2m0
(c)
The direction of the current in the plane conductor is perpendicu-
lar to the paper and beyond the drawing.

3.261 The current density is I /aL , where L is the length of the sec- B
tion. The difference in pressure produced must be
L I
I IB
¢p = * B * (abL)/ab = = 0.5 k Pa
aL a a
b
3.262 Let t = thickness of the wall of the cylinder. Then,
J = I/2 pRt (along z axis)
The magnetic field due to this at distance such that r (R - t / 2 6 r 6 R + t / 2 ), is
given by
3.5 CONSTANT MAGNETIC FIELD. MAGNETICS 445

p 5r 2 - (R - t /2)26
I
Bw (2pr) = m0
2pRT
m 0I
Bw = 5r 2 - (R - t /2) 26
4pRrt
R – t/2
Now, F = J * B dV R+t/2
L
Fr
and p =
2pRL
R + 2t
m0 I 2
e 2 - aR - b f * 2pr Ldr
1 t 2
= r
2pRL 3t 8p 2R 2t 2r 2
R-2

R + 2t
m0I 2
e r 2 - aR - b f dr
t 2
=
8p 2R 3t 2 3 2
R - 2t

aR + b - aR - b
m0I 2 t 3 t 3
= C t 2 S
- aR - b t
2 2
8p 2R 3t 2
3 2
m0I 2 m0I 2
= [Rt + 0 (t 2)] L
8p 2R 3t 8p 2R 2

3.263 When self-forces are involved, a typical factor of 1/2 comes into play. For example,
the force on a current carrying straight wire in a magnetic induction B is BIl. If the
magnetic induction B is due to the current itself then the force can be written as
I

F = B (I ¿) dI ¿l
3
0
1
If B (I ¿) ^ I ¿ , then this becomes, F = B (I ) Il.
2
In the present case, B (I ) = m0nI and this acts on nI ampere turns per unit length, so,
pressure is given by
F 1 I * nI * 1 * l 1
p = = m0n = m0n 2I 2
Area 2 1 * l 2
3.264 The magnetic induction B in the solenoid is given by B = m0nI. The force on an el-
ement dl = Rda of the current carrying conductor is
1 1
dF = m0n II Rda = m0 nI 2Rda
2 2
446 PART THREE ELECTRODYNAMICS

this is radially outwards. The factor 1/2 is explained dq


above. For equilibrium, it should be balanced by the ra- F dq F 2
dial component of the longitudinal tensile forces, Tda. 2
1
Hence, T = m0nI 2R
2

2 Flim
This equals Flim when, I = Im =
B m0nR
Note that Flim, here, is actually a force and not a stress.

rd
3.265 Resistance of the liquid between the plates =
S
Voltage between the plates = Ed = vBd
vBd
Current through the plates =
R + rd / S
Power generated in the external resistance R, is given by

v 2B 2R v 2B 2d 2 v 2B 2d 2
P = = =
aR + b a 1R + b
rd 2 rd 2
c e R 1>4 - a b f + 2 d
rd 1>2 2 rd 2
S S 1R s 1R B S
This is maximum when
rD v 2B 2Sd
R = and Pmax =
S 4r
3.266 The electrons in the conductor are drifting with a speed of
J I
vd = =
ne pR 2ne
where e = magnitude of the charge on the electron, n = concentration of the con-
duction electrons.
The magnetic field inside the conductor due to this current is given by
I m0 Ir
Bw (2pr) = pr 2 2
m0 or Bw =
pR 2p R 2
A radial electric field vBw must come into being in equilibrium. Its potential diff-
erence is
R
I m0 Ir
¢w = 2 2
dr
3 pR ne 2p R
0
3.5 CONSTANT MAGNETIC FIELD. MAGNETICS 447

m0I 1
a b R2
I
=
pR ne 2pR 2 2
2

m0I 2
=
4p 2R 2ne
= 2 pV

E
3.267 Here, vd = and j = ne vd
B
jB 200 * 10 4 A /m 2 * 1 T
So, n = =
eE 1.6 * 10 -19 C * 5 * 10 -4 V/m
= 2.5 * 10 28 per m 3 = 2.5 * 10 22 cm - 3

Atomic weight of Na being 23 and its density L 1, molar volume is 23 cm 3 . Thus,


number of atoms per unit volume is
6 * 10 23
L = 2.6 * 10 22 cm - 3
23
Thus, there is almost one conduction electron per atom.

Drift velocity
3.268 By definition, mobility =
Electric field component causing this drift
vd
or m =
EL
EL
On the other hand, ET = vd B = (as given)
h
1
So, m = = 3 .2 * 10 -3 m 2/(V # s)
hB
m 0I
3.269 Due to the straight conductor, Bw =
2pr

We use the formula, F = (pm # §) B

(a) The vector pm is parallel to the straight conductor, so


0
F = pmB = 0
0Z
because neither the direction nor the magnitude of B depends on z.
(b) The vector pm is oriented along the radius vector r, so
0
F = pm B
0r
448 PART THREE ELECTRODYNAMICS

The direction of B at r + dr is parallel to the direction at r. Thus only the w com-


ponent of F will survive.
0 m0I m0I pm
Fw = pm = -
0r 2pr 2pr 2
(c) The vector pm coincides in direction with the magnetic field, produced by the
conductor carrying current I.
0 m0I m0I pm 0ew
F = pm ew =
r 0w 2p 2pr 2 0w
m0Ipm 0ew
So, F = - e , as, = - er
2pr 2 r 0w

0Bx
3.270 We have, Fx = pm
0x
m0I Rdl m0IR 2
But, Bx = =
4p 3 + R 2) 3>2
(x 2 2(x 2 + R 2) 3>2
m0 I # 2 pR 2 3 m 2 pm
So, F = # 2x # pm = 0 6pR Ipmx x
4p (x 2 + R 2) 5>2 2 4p (x 2 + R 2) 5>2

#
0 ( m0 3p1m r) r - p1mr
2
3.271 F = p2m c d
0l 4p r5
0 m0 p1m - 3 m0 p1m p2m
= p2m c d = = 9 nN
0l 2p l 3 2 pl 4

3.272 From solution of Problem 3.270, for x W R,


m0I ¿R 2
Bx =
2x 3

2Bx x 3 2 * 3 * 10 -5T * (10 -1 m) 3


or I¿ L = L 0.5 kA
m0R 2 1.26 * 10 -6 * (10 -2 m) 4

3.273 B n¿ = B cos a, B'


B
1
H t¿ = B sin a, a
m0
H' n
B t¿ = mB sin a
Vacuum
So, B ¿ = B 2m 2 sin 2a + cos 2a
3.5 CONSTANT MAGNETIC FIELD. MAGNETICS 449

H # dS = a
- J b # d S = - J # d S, since B # d S = 0.
B
3.274 (a)
C C m0 C C

Now J is non-vanishing only in the bottom half of the sphere.

1 B
Here, Bn¿ = B cos u, H t¿ = B sin u, B t¿ = m B sin u, H n¿ = cos u
m0 mm0

m - 1
a 1 - b and Jt =
B cos u 1
Jn = B sin u
m0 m m0

Only Jn contributes to the surface integral, so

a1 - b
p R 2B cos u
J # dS = - J # dS =
1
- Jn d S =
C C C m0 m
lower lower

B
n
q l
S
r

R 2
h B' H'

(b) B # d r = (Bt - B ¿ t ) l = (1 - m)/B l sin u


C

3.275 Inside the cylindrical wire, there is an external current of density I / pR 2 . This gives
a magnetic field Hw with

r2 Ir
Hw2 pr = I or Hw =
R2 2pR 2

mm0Ir m - 1 Ir x Ir
From this Bw = and Jw = =
2pR 2 2p R 2 2pR 2

Hence, total volume of molecular current is


xI
Jw # d r = dl = xI
C L 2pR
r =R
450 PART THREE ELECTRODYNAMICS

The surface current is obtained by using the equivalence of the surface current den-
sity to J * n. This gives rise to a surface current density in the z-direction of
- xI
2pR
The total molecular surface current is
xI
I s¿ = - (2pR) = - xI
2pR
The two currents have opposite signs.
3.276 We can obtain the form of the curves required here, by qualitative arguments.

From H # dl = I
C
we get, H (x W 0) = H (x V 0) = nI

Then B (x W 0) = mm0nI
B (x V 0) = m0nI
Also,
B
B (x 6 0) = m0H (x 6 0) B

J (x 6 0) = 0 H H
J
B is continuous at x = 0, H is not. These give the re-
quired curves as shown in the answer sheet.
3.277 The lines of B as well as H field are circles around the wire. Thus,
I
H1pr + H2pr = I or H1 + H2 =
pr
Also, m0m1H1 = m2H2m0 = B1 = B2 = B
m2 I
Thus, H1 =
m1 + m2 pr
m1 I m1 m2
H2 =
m1 + m2 pr
m1m2 I
and B = m0
m1 + m2 pr
3.278 The medium I is vacuum and contains a circular current carrying coil with current I. The
medium II is magnetic with permeability m. The boundary is the plane z = 0 and the
coil is in the plane z = l. To find the magnetic induction, we note that the effect of the
magnetic medium can be written as due to an image coil in II as far as the medium I is
concerned. On the other hand, the induction in I can be written as due to the coil in I,
carrying a different current. It is sufficient to consider the far away fields and ensure that
the boundary conditions are satisfied there. Now for actual coil in medium I,
3.5 CONSTANT MAGNETIC FIELD. MAGNETICS 451

Br
Br x
Bq
q'
q'

r
Bq
q' q' q
II I I

2pm cos u¿ m0 pm sin u¿ m0


Br = - .a b anda b Bu =
r3 4p r3 4p
m0 pm m0 pm
So, B2 = (2 cos 2 u¿ - sin 2u¿) and Bx = ( - 3 sin u¿ cos u¿)
4p 4p
where, pm = I (pa 2), a = radius of the coil.

Similarly due to the image coil,


m0 P m¿ m0 P m¿
Bz = (2 cos 2 u¿ - sin 2 u¿), Bx = ¿ = I ¿(pa 2)
(3 sin u¿ cos u¿), P m
4p 4p
As far as medium II is concerned, we write similarly
m0 p –m m0 p –m
Bz = ( 2 cos 2 u¿ - sin 2 u¿), Bx = ( - 3 sin u¿ cos u¿), p –m = I –(pa 2)
4p 4p
The boundary conditions are
pm + P m¿ = p –m (from B1n = B2n)
1
- pm + P m¿ = - p –m (from H1t = H2t)
m
2m m - 1
Thus, I– = I and I¿ = I
m + 1 m + 1
In the limit when the coil is on the boundary, the magnetic field everywhere can be
obtained by taking the current to be
2m
I
m + 1
2m
Thus, B = B
m + 1 0
3.279 We use the fact that within an isolated uniformly magnetized ball, H ¿ = - J / 3 ,
B¿ = 2m0 J / 3 , where J is the magnetization vector. Then, in a uniform magnetic field
with induction B0 , we have by superposition,
2m0 J B0 J
Bin = B0 + , Hin = -
3 m0 3
452 PART THREE ELECTRODYNAMICS

or Bin + 2m0Hin = 3B0


Also, Bin = mm0 Hin
3B0 3 mB0
Thus, Hin = and Bin =
m0 (m + 2 ) m + 2

3.280 The coercive force Hc is just the magnetic field within the cylinder. This is by circu-
lation theorem,
NI
HC = = 6 kA/m
l
(from H # d r = I, total current, considering a rectangular contour).
C

3.281 We use, H # dl = 0
C H
Neglecting the fringing of the lines of force, we write this as

B
H (pd - b) + b = 0 B
m0
- Bb
or H L = 101 A/m
m0pd

The sense of H is opposite to B .

NI m0 - Bb
H # d l = NI or H (2pR) +
Bb
3.282 Here, = NI so, H =
C m0 2pRm0
B 2pRB
Hence, m = = = 3700
m0H m0NI - Bb

B
3.283 One has to draw the graph of m = versus H from the given graph. The m-H
m0H
graph starts out horizontally, and then rises steeply at about H = 0.04 kA/m before
falling again. It is easy to check that mmax L 10,000.

3.284 From the theorem on circulation of vector H

Bb m0NI m0pd
Hpd + = NI or B = - H = (1.51 - 0.987) H
m0 b b
where B is in Tesla and H in kA/m. Besides, B and H are interrelated as in the
Fig. 3.76 of the problem book. Thus we have to solve for B, H graphically by sim-
ultaneously drawing the two curves (the hysterisis curve and the straight line)
and find the point of intersection. It is at H L 0.26 kA/m, B = 1.25 T.
3.5 CONSTANT MAGNETIC FIELD. MAGNETICS 453

B
Then, m = L 4000
m0H

3.285 From the formula,

F = (pm.§ ) B : F = ( J # § ) B dV
3
x
Thus, F = (B # §) B dV
mm0 3

Since B is predominantly along the x-axis,


x =L
x 0Bx xS xSB 2 xSB 2
so, Fx = Bx Sdx = dB x2 = - =
m m0 3 0x 2m m0 3 2m m0 2m0
x =0

3.286 The force in question is


xBV dB
F = (pm # §) B =
m m0 dx
Since B is essentially in the x-direction,
xV dB 2 xB02V d xB02
Fx L = (e-2ax ) = - 4 ax e-2ax
2 2
so, V
2m0 dx 2m0 dx 2m0
This is maximum when its derivative vanishes, i.e.,
1
16a 2x 2 - 4a = 0 or xm =
14a
The maximum force is
1 xB02V xB02V a
Fmax = 4a e -1>2 =
14a 2m0 m0 A e
e
m0 Fmax
Aa
So, x = = 3.6 * 10 -4
VB02
3.287 The force is given by
xBV dB xV dB 2
Fx = (pm # § ) Bx = L
m m0 dx 2m0 dx
This force is attractive and an equal force must be applied for balance. The work
done by applied forces is
x =L
xV x =L xVB 2
A = - Fxdx = ( - B 2)x = 0 L
3 2m0 2m0
x =0
454 PART THREE ELECTRODYNAMICS

3.6 Electromagnetic Induction. Maxwell’s Equations

3.288 Taking the direction of vector B normally inward and direction on normal n to the
plane pointing outward. Here,
d£ = B # dS = - 2B xdy
y
y
Since y = ax 2 or x =
Aa B w
y dy
So, d£ = - 2B dy
Aa
From Faraday’s law of electromagnetic induction,
y

jin = -
dt x
0
y dy
Hence, jin = 2B
A a dt

ausing
8w dy a
= By = 22wy
A a dt
From Lenz’s law, and right handed screw rule, the induced current and hence the
induced e.m.f. in the loop is anticlockwise.

3.289 Let us assume B is directed into the plane of the loop. Then the motional e.m.f.

jin = (v * B) # d l = vBl I
3

and is directed in the sense of (v * B). R1 R2


xin ,R
R1+ R2
jin R1R2
a where Ru = b
Bvl
So, I = =
R1R2 R + Ru R1 + R2
R +
R1 + R2

as R1 and R2 are in parallel connections.


3.290 (a) As the metal disk rotates, any free electron also rotates with it with the same angu-
lar velocity v, and that is why an electron must have an acceleration of v 2r directed
towards the disk’s center, where r is the separation of the electron from the center
of the disk. We know from Newton’s second law that if a particle has some acceler-
ation then there must be a net effective force on it in the direction of acceleration.
We also know that a charged particle can be influenced by two fields-electric and
magnetic. In our problem, magnetic field is absent hence there must be an electric
field near an electron and is directed opposite to the acceleration of the electron.
3.6 ELECTROMAGNETIC INDUCTION. MAXWELL’S EQUATIONS 455

If E is the electric field strength at a distance r from the center of the disk, we
have from Newton’s second law
Fn = mwn
eE = mr v 2
mv 2r
or E =
e
and the potential difference is given by
a a
mv2r
wcen - wrim = E # dr = dr as E cT dr
3 3 e
0 0

mv 2 a2
Thus, wcen - wrim = ¢w = = 3.0 nV
e 2
(b) When field B is present, by definition of motional e.m.f.
2

w1 - w2 = - (v * B) # dr
3
1

Hence the sought potential difference,


a a

wcen - wrim = - vBdr = - vr Bdr (as v = vr)


3 3
0 0

1
Thus, wrim - wcen = w = vBa 2 = 20 mV
2

In general v 6 eB /m , so we can neglect the effect discussed in part (a) here.

3.291 By definition,
E = - (v * B)
C C d

So, E # dr = - (v * B) # dr = - vB dr
3 3 3
A A 0

But, v = vr, where r is the perpendicular distance of the point from A.


C d

E # dr =
1
Hence, - vBrdr = - vBd 2 = - 10 mV
3 3 2
A 0
456 PART THREE ELECTRODYNAMICS

This result can be generalized to a wire AC of arbitary planar shape. We have


C C

E # dr = - (v * B) # dr
3 3
A A A C
C

= - ((v * r) * B) # dr
3
A
C

= - (B # r␻ - B # ␻r) # dr
3
A
1
= - Bv d 2
2
= - 10 mV

d being AC and r being measured from A.

3.292 Magnitude of flux at any moment of time,

| £(t) | = B # S = B a R 2w b
1
2
where w is the sector angle, enclosed by the field.
Now, magnitude of induced e.m.f. is given by,
d£ t
jin = ` ` = ` ` =
BR 2 dW BR 2
v
dt 2 dt 2
where v is the angular velocity of the disk. But as it starts rotating from rest at t = 0
with an angular acceleration b , its angular velocity v(t) = bt. So,
BR 2
jin = bt
2
According to Lenz’s law the first half cycle current in the loop is in anticlockwise
direction, and in subsequent half cycle it is in clockwise direction.
BR 2
Thus, jin = ( - 1)n bt (where n is number of half revolutions)
2

The plot jm(t), where tn = 22pn/b , is as shown in the answer sheet.

3.293 Field, due to the current carrying wire in the region, right to it, is directed into the
plane of the paper and its magnitude is given by
3.6 ELECTROMAGNETIC INDUCTION. MAXWELL’S EQUATIONS 457

m0 I
B =
2pr r
where r is the perpendicular distance from the wire.
As B is same along the length of the rod thus, motional e.m.f.
2

jin = ` (v * B) # dl ` = vBl
3
1

and it is directed in the sense of (v * B).


So, current (induced) in the loop
jin 1 m0lvI
Iin = =
R 2 pRr
3.294 Field due to the current carrying wire at a perpendicular distance x from it is
given by
m0 I
B (x) =
2p x

There will be no induced e.m.f. in the segments (2) and (4) as, v c c d l . The magni-
tude of e.m.f. induced in 1 and 3, will be
m0 m0avI
j1 = vB (a + x) a = v a b a =
2I
4p a + x 2p (a + x)
m0 2I m0av I
j2 = vB (x)a = v a b a =
4p x 2p x

respectively, and their sense will be in the direction of (v * B).


So, e.m.f. induced in the network
= j1 - j2 (as j1 7 j2)

avm0I 1 va 2m0I
c - d =
1
=
2p x a + x 2px (a + x)

3.295 As the rod rotates, motional e.m.f. is induced in it


a
1 2
j = (vr) B dr = a Bv
3 2
0
The net current in the conductor is then
1 2
j(t) - a Bv
2
R
458 PART THREE ELECTRODYNAMICS

A magnetic force will then act on the conductor of magnitude BI per unit length. Its
direction will be normal to B and the rod and its torque will be
1 2
a j(t ) - a Bv
2
£ ≥ dx Bx
3 R
0

Obviously both magnetic and mechanical torque acting on the C.M. of the rod must
be equal but opposite in sense. Then for equilibrium at constant v ,
1
j(t ) - a 2Bv
2 Ba 2 1
= mga sin vt
R 2 2
1 mg R 1
or j(t ) = a 2Bv + sin vt = (a 3B 2v + 2mg R sin vt)
2 aB 2aB
(The answer given in the book is incorrect dimensionally.)

3.296 The rod behaves like a cell of e.m.f.  vBl, where v is the velocity of the rod at an
arbitrary instant of time. From Lenz’s law, current in the loop is in clockwise dire-
ction.
B
From Newton's second law for the rod, Fx = mwx R

g
ma
mg sin a - IlB = mw

F
or
I x in
For steady state, acceleration of the rod must be equal to zero. a

Hence, mg sin a = IlB (1) mg


a
jin vBl
But, I = = (2) x
R R
From Eqs. (1) and (2) mg sin aR
v =
B 2l 2

3.297 From Lenz’s law, the current through the copper bar is directed from 1 to 2 or in
other words, the induced current in the circuit is in clockwise direction.
Potential difference across the capacitor plates C B
1
ag

q
Fm

= jin or q = Cjin
C
l

2
Hence, the induced current in the loop
dq djin
I = = C x
dt dt
3.6 ELECTROMAGNETIC INDUCTION. MAXWELL’S EQUATIONS 459

But the variation of magnetic flux through the loop is caused by the movement
of the bar.
So, the induced e.m.f. jin = Blv
djin dv
On differentiation = Bl
= Blw
dt dt
dj
Hence, I = C = C Bl w
dt
Now, the forces acting on the bars are the weight and the Ampere’s force, where
Famp = IlB (CB lw) B = Cl 2B 2w.
From Newton’s second law, for the rod,
Fx = mwx
or mg sin a - Cl 2B 2w = mw
mg sin a g sin a
Hence, w = 2 2 =
Cl B + m l 2B 2C
1 +
m
3.298 Flux of B, at an arbitrary moment of time t is given by
pa 2
£t = B # S = B cos vt
2
From Faraday’s law, induced e.m.f.

jin = -
dt

d a Bp
a2
cos vtb
2 Bpa 2v
= - = sin vt
dt 2
and induced current
jin Bpa 2
Iin = = v sin vt
R 2R
Now, thermal power generated in the circuit at the moment at time  t

P (t) = jin * Iin = ab


Bpa 2v 2 1
sin 2vt
2 R
and mean thermal power generated is given by
T

c d
Bpa 2v 2 1
sin 2 vtdt
2 2 3
a b
0 1 Bpa 2v 2
6 P 7 = T
=
2R 2
dt
3
0
460 PART THREE ELECTRODYNAMICS

Note: The calculation of jin, which can also be checked by using motional e.m.f. is
correct even though the conductor is not a closed semicircle, because the flux
linked to the rectangular part containing the resistance R is not changing. The
answer given in the book is off by a factor 1>4.

3.299 The flux through the coil changes sign. Initially it is BS per turn. Finally it is - BS
per turn. Now if flux is £ at an intermediate state, then the current at that moment
will be

-N
dt
I =
R
So, charge that flows during a sudden turning of the coil is
N
q = I dt = - [£ - ( - £)]
3 R
BS
= 2N
R
1 qR
Hence, B = = 0.5 T (on substituting values)
2 NS

3.300 According to Ohm’s law and Faraday’s law of induction, the current I0 appearing in
the frame, during its rotation, is determined by the formula

d£ dI0 b O
I0 = - = -L
dt dt
I
Hence, the required amount of electricity (charge) is a

q = I 0 dt a O'
3
1
= - (d £ + LdI0)
R3
1
= - (¢ £ + L ¢I0)
R

Since the frame has been stopped after rotation, the current in it vanishes, and hence
¢I0 = 0. We have to find the increment of the flux ¢ £ through the frame
(¢£ = £ 2 - £ 1).
Let us choose the normal n to the plane of the frame, for instance, so that in the
final position, n is directed behind the plane of the figure (along B).
3.6 ELECTROMAGNETIC INDUCTION. MAXWELL’S EQUATIONS 461

Then it can be easily seen that in the final position, £ 2 7 0, while in the initial
position, £ 1 6 0 (the normal is opposite to B), and ¢w turns out to be simply equal
to the flux through the surface bounded by the final and initial positions of the frame:
b+a

¢£ = £ 2 + | £ 1 | = B adr
3
b-a
where B is a function of r, whose form can be easily found with the help of the the-
orem of circulation. Finally omitting the minus sign, we obtain
¢£ m0aI b + a
q = = ln
R 2pR b - 1
3.301 As B, due to the straight current carrying wire, varies along the rod (connector) and
enters linearly so, to make the calculations simple, B is made constant by taking its
average value in the range [a, b].
b b
m0 I0 Bƒ
Bdr dr
3 3 2p r Famp v
a a
R
6B 7 = b
= b
Fext
x
a b
dr dr
3 3 I0
a a
m0 I0 b
or 6B 7 = ln
2p (b - a) a
(a) The flux of B changes through the loop due to the movement of the connector.
According to Lenz’s law, the current in the loop will be anticlockwise. The mag-
nitude of motional e.m.f.
jin = v 6 B 7 (b - a)
m0 I0 m0
a ln b (b - a)
b dx b
= = I0 ln v
2p (b - a) a dt 2p a
So, induced current
jin bm0 I0v
Iin = = ln
R 2p R a
(b) The force required to maintain the constant velocity of the connector must be
the magnitude equal to that of Ampere’s force acting on the connector, but in
opposite direction.
m0 I0 m0 I0
Fext = Iin l 6 B 7 = a v ln b (b - a) a ln b
b b
So,
2p R a 2p (b - a) a
v m0
a I0 ln b
b 2
=
R 2p a
and will be directed as shown in the figure.
462 PART THREE ELECTRODYNAMICS

3.302 (a) The flux through the loop changes due to the movement of the rod AB.
According to Lenz’s law, current should be anticlockwise, since we have
assumed that B is directed into the plane of the loop. The motional e.m.f.
jin (t) = Bl v and induced current Iin = vBl /R .
From Newton’s law in projection form,
Fx = mwx
v dv A
So, - Famp = m
dx
v0
vB 2l 2
But Famp = Iin lB = R Famp
R Bƒ
vB 2l 2 dv
So, - = mv B
R dx
x 0
mR mRv0
or dx = - 2 2 dv or x =
3 B l 3 B 2l 2
0 0 v
(b) Sum of the works done by Ampere’s force and induced e.m.f. is zero, so from
energy conservation
¢E + Q = 0 (where Q is heat liberated)

c0 - mv 02 d + Q = 0
1
or
2
1
So, Q =
mv02
2
3.303 Using the calculation done in the previous problem, Ampere’s force on the connector,
vB 2l 2
Famp = (directed towards left)
R
Now from Newton’s second law, A
dv B R
F - Famp = m
dt Famp F
vB 2l 2 dv v
So, F = + m
R dt B
v
dv
or t = m 2 2
3 F - vB l
0
R
vB 2l 2
F -
t R R
or = - 2 2 ln P Q
m B l F
3.6 ELECTROMAGNETIC INDUCTION. MAXWELL’S EQUATIONS 463

RF
v = (1 - e -tB
2l 2
/Rm
Thus, )
B 2l 2

3.304 According to Lenz’s law, the sense of induced e.m.f. is such that it opposes the cause
of change of flux. In our problem, magnetic field is directed away from the reader
and is diminishing. So,

(a) (b) (c) (d)

In Fig. (a), in the round conductor, it is clockwise and there is no current in the
connector.
In Fig. (b), in the outside conductor, it is clockwise.
In Fig. (c), in both the conductors, it is clockwise; and there is no current in
the connector to obey the charge conservation.
In Fig. (d), in the left side of the figure, it is clockwise.
3.305 The loops are connected in such a way that if the current is clockwise in one, it is
anticlockwise in the other. Hence the e.m.f. in loop b opposes the e.m.f. in loop a.
d d
In the loop with side equal to a, e.m.f. is (a 2B) = a 2 (B0 sin vt)
dt dt
Similarly, e.m.f. in loop with side equal to b = b 2 B0 v cos vt.
Hence, net e.m.f. in the circuit = (a 2 - b 2) B0v cos vt, as both the e.m.f ’s are in op-
posite sense, and resistance of the circuit = 4 (a + b) r.
Therefore, the amplitude of the current
(a 2 - b 2) B0v
= = 0.5 A
4(a + b)r
3.306 The flat shape is made up of concentric loops, having different radii, varying from O to a.
Let us consider an elementary loop of radius r, then, e.m.f. induced due to this loop
is
- d (B # S)
= pr 2B0v cos vt
dt
and the total induced e.m.f.
a
jin = (pr 2B0v cos vt) dN (1)
Lo
464 PART THREE ELECTRODYNAMICS

where pr 2v cos vt is the contribution of one turn of radius r and dN is the number
of turns in the interval (r, r + dr ).

dN = a b dr
N
So, (2)
a
From Eqs. (1) and (2),
a
N pB0v cos vtNa 2
j = - (pr 2B0v cos vt) dr =
3 a 3
0

Maximum value of e.m.f. amplitude


1
jmax = pB0 vNa 2
3
3.307 The flux through the loop changes due to the variation in B with time and also due
to the movement of the connector.

d (B # S)
So, jin = 2 2 = 2 d (BS) 2 (as S and B are collinear)
dt dt

But, B, after t seconds of beginning of motion = Bt, and S becomes = 1 /2 l wt 2, as


connector starts moving from rest with a constant acceleration w.
3
So, jin = Bl wt 2
2
= 12 mV (on substituting values)

3.308 We use B = m0 n I
Then, from the law of electromagnetic induction

E # dl = -

C dt
So, for r 6 a
# 1 # #
Ew2pr = - pr 2m0n I or Ew = - m0 nrI (where I = dI/dt)
2
For r 7 a ,
# #
Ew 2pr = - pa 2m0nI or Ew = - m0nI a 2/2r

The meaning of minus sign can be deduced from Lenz’s law.


#
3.309 The e.m.f. induced in the turn is m0nI p d 2/ 4. The resistance is pd /S r. So, the cu-
#
rrent is m0nI Sd / 4 r = 2 mA, where r is the resistivity of copper.
3.6 ELECTROMAGNETIC INDUCTION. MAXWELL’S EQUATIONS 465

3.310 The changing magnetic field will induce an e.m.f. in the ring, which is obviously
equal in the two parts by symmetry (the e.m.f. induced by electromagnetic induc-
tion does not depend on resistance). The current that will flow due to this will be
different in the two parts. This will cause an acceleration of charge, leading to the
setting up of an electric field E which has opposite sign in the two parts. Thus,
j # j
- paE = rI and + paE = h rI
2 2
where j is the total induced e.m.f. From this,
j = (h + 1)rI
1 1 h - 1
and E = (h - 1)rI = j
2pa 2pa h + 1

But by Faraday’s law, j = pa 2b


1 h - 1
So, E = ab
2 h + 1

3.311 Consider the rotating frame with an instantaneous angular velocity v(t). In this
frame, a Coriolis force, 2 m v ¿ * v(t) acts, which must be balanced by the magnetic
force e v * B(t).
e
Thus, v(t) = - B(t)
2m
It is assumed that v is small and varies slowly, so v 2 and v can be neglected.

3.312 The solenoid has an inductance,


L = m0n 2pb 2I
where n = number of turns of the solenoid per unit length. When the solenoid is
connected to the source, an e.m.f. is set up, which, because of the inductance and
resistance, rises slowly according to the equation
#
RI + LI = V
This has the well-known solution
V
I = (1 - e –tR/L)
R
Corresponding to this current, an e.m.f. is induced in the ring. Its magnetic field
B = m0 nI , in the solenoid, produces a force per unit length given by
dF m02n 2pa 2II
= BI =
dl r
m02 pa 2V 2
a be
n 2 –tR/L
= (1 - e –tR/L),
r RL
466 PART THREE ELECTRODYNAMICS

acting on each segment of the ring. This force is zero initially and zero for a large t.
Its maximum value is for some finite t. The maximum value of

- a - e –tR/L b =
1 1 2 1
e –tR/L (1 - e –tR/L) =
4 2 4
dFmax m02pa 2V 2 n 2 m0a 2V 2
So, = =
dl r 4RL 4rRlb 2

3.313 The amount of heat generated in the loop during a small time interval dt

dQ = j2/R dt, but, j = - = 2at - at
dt
(2at - at) 2
So, dQ = dt
R
and hence, the amount of heat generated in the loop during the time interval 0 to t
t
(2at - at) 2 1 a 2t 3
Q = dt =
3 R 3 R
0

3.314 Take an elementary ring of radius r and width dr. The e.m.f. induced in this elemen-
tary ring is pr 2 b. Now the conductance of this ring is

da b =
1 h dr hrdr
so dI = b dr
R r 2pr 2r b
a
Integrating we get the total current as
b r
hrdr hb(b 2 - a 2)
I = b =
3 2r 4r
0

3.315 Given L = m0n 2V = m0n 2l0pR 2, where R is the radius of the solenoid.
L 1
Thus, n =
A m0I0p R
So, length of the wire required is
4pLl0
l = nl02pR = = 0.10 km
B m0

3.316 From the previous problem, we know that, length of the wire needed
Ll 4p
l¿ = (where l = length of solenoid)
A m0
3.6 ELECTROMAGNETIC INDUCTION. MAXWELL’S EQUATIONS 467

r0l ¿
Now, R =
S
where S = area of cross section of the wire, also m = rSl ¿.
RS Rm Rm
Thus, l¿ = = or l ¿ =
r0 rr0l ¿ A rr0
where r0 = resistivity of copper and r = density of copper.
Rm Ll
Equating, =
rr0 m0/4p
m0 mR
or L =
4p rr0l
3.317 The current at time t is given by
V
I(t) = (1 - e -tR/L )
R
The steady state value is
V
I0 =
R
I(t)
and = h = 1 - e -tR/L or e -tR/L = 1 - h
I0
R 1 L 1
or t0 = ln or t0 = ln = 1.49 s
L 1 - h R 1 - h
3.318 The time constant t is given by
L L
t = =
R l0
r0
S
where r0 = resistively, l0 = length of the winding wire, S = cross section of the wire.
But m = lr0S
L mL
So eliminating S, t = =
r0l0 rr0l 20
m/rl0
4plL
From solution of Problem 3.315, l0 =
A m0

(Note the interchange of l and l0 because of difference in notation here.)


mL m
Thus, t = = m0 4p = 0.7 ms
4p rr0l
rr0 Ll
m0
468 PART THREE ELECTRODYNAMICS

3.319 Between the cables, where a 6 r 6 b, the magnetic field H satisfies


I
Hw2pr = I or Hw =
2pr
mm0I
So, Bw =
2pr
The associated flux per unit length is
r =b
mm0 I mm0 I b
£ = * 1 * dr = ln
3 2pr 2p a
r =a

Hence, the inductance per unit length is


£ mm0
ln h awhere h =
b a
L1 = =
I 2p a
mH
We get L1 = 0.26
m
3.320 Within the solenoid
Hw # 2 pr = NI or Hw =
NI NI
, B = mm0
2pr w 2pr
a+b
mm0 a dr
and the flux is £ = N £1 = N NI
2p 3 r
b

mm0
N 2 a ln a 1 + b
a
Finally, L =
2p b

3.321 Neglecting end effects, the magnetic field B between the


plates, which is mainly parallel to the plates, is
I
B = m0
b
Thus, the associated flux per unit length of the plates is b

h
* h * 1 = a m0 b * I
I h
£ = m0
b b
h
So, L1 = inductance per unit length = m0 = 25 nH/m.
b
3.322 For a single current carrying wire,
m 0I
Bw = (for r 7 a)
2pr
For the double line cable with current flowing in opposite direction in the two con-
ductors Bw L m0I / pr between the cables, by superposition. The associated flux is
3.6 ELECTROMAGNETIC INDUCTION. MAXWELL’S EQUATIONS 469

d-a
m0I dr * 1 m0I d m0
£ = L ln = ln h * I per unit length
3 p r p a p
a
m0
Hence, L1 = ln h
p
is the inductance per unit length.
3.323 In a superconductor there is no resistance. Hence,
dI d£
L = +
dt dt
¢£ pa 2B
So integrating, I = =
L L
because ¢£ = £ f - £ i

£ f = pa 2B as £ i = 0

Also, the work done is


d£ 1 1 p 2a 4B 2
A = jIdt = I dt = LI 2 =
L L dt 2 2 L
3.324 In a solenoid, the inductance
N S
L = mm0 n 2V = mm0
l
where S = area of cross-section of the solenoid, l = its length, V = Sl, N = nl = total
number of turns.
When the length of the solenoid is increased, for example, by pulling it, its induc-
tance will decrease. If the current remains unchanged, the flux, linked to the sole-
noid, will also decrease. An induced e.m.f. will then come into play, which by Lenz’s
law will try to oppose the decrease of flux, for example, by increasing the current.
In the superconducting state, the flux will not change and so, I /l = constant.

I I0 l
Hence, = or I = I0 = I0 (1 + h) = 2A (on substituting values)
l l0 l0

3.325 The flux linked to the ring cannot change on transition to the super-conduction state,
for reasons similar to those given above. Thus, a current I must be induced in the
ring, where
£ pa 2B paB
I = = = = 50 A (on substituting values)
m0a a ln - 2b m0 a ln - 2b
L 8a 8a
b b
470 PART THREE ELECTRODYNAMICS

3.326 We write the equation of the circuit as


L dI
RI + = j
h dt

for t Ú 0. The current at t = 0 just after inductance is changed is I = h j / R , so


that the flux through the inductance is unchanged.

We look for a solution of the above equation in the form


I = A + Be –t/C
L j
Substituting C = , B = h - 1, A =
hR R

j
We get, I = (1 + (h - 1) e -hRt/L
R

3.327 In the left loop, j - I0R = (I0 - I)R


I
or j + IR = 2I0R
x R L
(j + IR) j - IR I0
So, j - I0 R = j - = R I0 – I
2 2
But in the bigger loop
dI
j - I0R = L
dt
j - IR dI
or = L
2 dt
dI
So, j - IR = L
dt

3.328 The equations are


L1
dI1 dI2
L1 = L2 = j - R (I1 + I2) I1
dt dt
I2 L2
d
Then, (L I - L2I2) = 0 I1+I2
dt 1 1

x R
or L1I1 - L2I2 = constant

But initially at t = 0, I1 = I2 = 0.
3.6 ELECTROMAGNETIC INDUCTION. MAXWELL’S EQUATIONS 471

So, constant must be zero and at all times. Therefore,


L1I1 = L2I2
In the final steady state, current must obviously be I1 + I2 = j / R . Thus in steady state,
jL2 jL1
I1 = and I2 =
R (L1 + L2) R (L1 + L2)
3.329 Here,
m0I
B = (at a distance r from the wire)
2pr
The flux through the frame is,
a+l
m0I m0b
I ln a 1 + b
a
£ 12 = bdr =
3 2pr 2p l
l I b
£ 12 m 0b
ln a1 + b
a l
Thus, L12 = = a
I 2p l
b
m 0I l hdr
3.330 Here, B = and £ = m0m N
2pr 2p 3 r
a
mm0 hN b
Thus, L12 = ln
2p a
3.331 The direct calculation of the flux £ 2 is a rather complicated problem, since the con-
figuration of the field itself is complicated. However, the application of the reciproc-
ity theorem simplifies the solution of the problem. Indeed, let the same current I
flow through loop 2. Then the magnetic flux created by this current through loop 1
can be easily found.
As the loop is very small, so, induction value of B at the center is for the whole loop.
Magnetic induction at the center of the loop is
m0I
B =
2b b
So, flux through loop 1 is
m0I a
£ 12 = pa 2 I
2b
£ 21 1 m0pa
2
(a) So, L12 = =
I 2 b
(b) From reciprocity theorem,
m0pa 2I
£ 12 = £ 21, £ 21 =
2b
472 PART THREE ELECTRODYNAMICS

3.332 Let pm be the magnetic moment of the magnet M.


Then, the magnetic field due to this magnet is
m0 3 (pm # r) r ρ r
pm
c 5
- 3d x
4p r r
The flux associated with this, when the magnet is
along the axis at a distance x from the center, is
m0 3 (pm # r) r pm
£ = c - d # d S = £1 - £2
4p 3 r5 r3
a
m0 m0 pm 1
a - b
2prd r 1
where, £ 2 = pm =
4p 3 (x 2+ r )
2 3/2 2 x 2x + a 2
2
0
a
3m0 pmx 2 2prdr
and £1 =
4p 3 (x 2+ r 2) 5/2
0
m0 pmx 2
a b
1 1
= - 2
2 x 3 (x + a 2) 5/2
- m0 pma 2
So, £ =
2 (x 2 + a 2) 3/2
d£ N dw
When the flux changes, an e.m.f. - N is induced and a current - flows.
dt R dt
The total charge q, flowing as the magnet is removed to infinity from x = 0, is
N N # m0 pm
q = £ (x = 0) =
R R 2a
2aqR
or pm =
N m0

3.333 If a current I flows in one of the coils, the magnetic field at the center of the other coil is
m0a 2I m0a 2I
B = = , as l 77 a
2(l 2 + a 2) 3/2 2l 3
The flux associated with the second coil is then approximately m0p a 4I/2 l 3.
m0pa 4
Hence, L12 =
2l 3
3.334 When the current in one of the loops is I1 = at, an e.m.f. L12 dI1 / dt = L12 a, is ind-
uced in the other loop. Then, if the current in the other loop is I2, we must have,
dI2
L2 + RI2 = L12a
dt
3.6 ELECTROMAGNETIC INDUCTION. MAXWELL’S EQUATIONS 473

This equation has the solution,


L12a
I2 = (1 - e - tR /L2 )
R
which is the required current.
3.335 Initially, after a steady current is set up, the current is flowing as shown in the figure.
In steady conditions, I20 = j / R and I10 = j /R 0 .
The current in the inductance cannot change suddenly. We then have the equation,
dI2
L + (R + R0) I2 = 0 L R
dt
This equation has the solution I2

I2 = I20e-t (R + R0)/L I1 R0
The heat dissipated in the coil is
q q I
Sw
Q = I22 Rdt = I 220R e -2 t ( R + R0 /L )dt
3 3
0 0

L L j2
= RI 220 * = = 3mJ
2(R + R0) 2R (R + R0)

3.336 To find the magnetic field energy we recall that the flux varies linearly with current.
Thus, when the flux is £ for current I, we can write f = AI. The total energy en-
closed in the field, when the current is I, is

W = jIdt = N I dt
3 3 dt
I
1 1
= Nd£I = NA IdI = NAI 2 = N £I
3 3 2 2
0
The characteristic factor 1/2 appears in this way.

3.337 We apply circulation theorem,

H # 2pb = NI
NI
or H =
2pb
Thus the total energy,

BH # 2pb # pa 2 = p 2a 2b BH
1
W =
2
Given N, I, b, we know H, and can find out B from the B - H curve. Then W can
be calculated = 2.0 J.
474 PART THREE ELECTRODYNAMICS

3.338 From H # d r = NI
C
H # pd +
B #
b L NI, (d W b)
m0
Also, B = mm0 H
NI
Thus, H =
pd + md
Since B is continuous across the gap, B is given by,
NI
B = mm0
pd + mb
both in the magnetic field and the gap.
B2
* S * b
Wgap 2m0 mb
(a) = = = 3.0
Wmag B2 pd
* S * pd
2mm0
2
B # d S = N mm0 # S = m0 SN I
NI
(b) The flux is N
3 pd + mb pd
b +
m
m0SN 2
So, L L
pd
b +
m
Total energy
2
1 m0N S # 2
a
B 2 pd 1
= + bb S = I = LI 2 = 0.15H
2m0 m 2 pd 2
b +
m
The L found in the one way is similar with the one found in the other way. Note
that in calculating the flux, we do not consider the field in the gap since it is not
linked to the winding. But the total energy includes that of the gap.
3.339 When the cylinder with a linear charge density l rotates
with a circular frequency v, a surface current density
(charge> length * time) of I = lv>2p is set up.
The direction of the surface current is normal to the plane ds
Q
of paper at Q and the contribution of this current to the mag-
netic field at P is dB
dB r
m0 I (e * r)
dB = dS (where e is the direction of the current ) P
4p r3 dB||
In magnitude, | e * r | = r, since e is normal to r and the
direction of d B is as shown in the figure.
3.6 ELECTROMAGNETIC INDUCTION. MAXWELL’S EQUATIONS 475

Its component, d B 7 , cancels out by cylindrical symmetry. The component that re-
mains is
m0 IdS m0I
| B⬜ | = 2
cos u = dÆ = m0I
4p 3 r 4p 3
where we have used,
dS cos u
= dÆ and dÆ = 4p
r2 3
which is the total solid angle around any point.
The magnetic field vanishes outside the cylinder by similar argument.

The total energy per unit length of the cylinder is


m0
m02 a b * pa 2 =
1 lv 2
W1 = a 2l 2v 2
2m0 2p 8p
1
3.340 wele = e0 E 2 (for the electric field)
2
1
wmag = B 2 (for the magnetic field)
2m0
1 1
Thus, B 2 = e0 E 2
2m0 2
B
or E = = 3 * 10 8 V/m
2e0m0
3.341 The electric field at P is
ql
EP =
4 pe0 1a 2 + l 22 3/2
To get the magnetic field, note that the rotating ring constitutes a current
I = q v/2p, and the corresponding magnetic field at P is
m0a 2I
BP =
21a 2 + l 22 3/2 a

wele e0m0E 2 ql * 2 l P
a b
2
Thus, = = e m
wmag B2 0 0
4p e0m0a 2I

a 2 b
1 l 2
=
e0 m 0 a v

wM e0m0 v 2a 4
or =
wE l2
= 1.1 * 10-15
476 PART THREE ELECTRODYNAMICS

3.342 The total energy of the magnetic field is

( B # H) dV = B# a - J b dV
1 1 B
23 23 m0

B # B dV - J # BdV
1 1
=
2m0 3 23
The second term can be interpreted as the energy of magnetization, and has the density
1
– J # B.
2
3.343 (a) In series, the current I flows through both coils, and the total e.m.f. induced
when the current changes is
dI dI
- 2L = - L¿
dt dt
or L¿ = 2L
I
(b) In parallel, the current flowing through either coil is, and the e.m.f. induced is
2

- L a b
1 dI
2 dt

- L a b = - L¿
Since, 1 dI dI
2 dt dt
1
We get, L¿ = L
2
3.344 We use L1 = m0n12 V, L2 = m0n22V

So, L12 = m0n1n2V = 2L1L2

3.345 The interaction energy is


1 2 1 2 1 2
| B + B2 | dV - | B | dV - | B | dV
2m0 L 1 2m0 L 1 2m0 L 2
1
= B # B dV
m0 3 1 2
Here, if B1 is the magnetic field produced by the first of the current carrying
loops, and B2 that of the second one, then the magnetic field due to both the
loops will be B1 + B2.
3.346 We can think of the smaller coil as constituting a magnet of dipole moment,
pm = pa 2I1
Its direction is normal to the loop and it makes an angle u with the direction of the
magnetic field, due to the bigger loop. This magnetic field is
m0I2
B2 =
2b
3.6 ELECTROMAGNETIC INDUCTION. MAXWELL’S EQUATIONS 477

The interaction energy has the magnitude


m0 I1 I2
|W | = pa 2 cos u
2b
Its sign depends on the sense of the currents.
3.347 (a) There is a radial, outward conduction current. Let Q be the instantaneous charge
on the inner sphere, then,
dQ 1 dQ
j * 4pr 2 = - or j = - r
dt 4pr 2 dt
0D
a rb = -j
d Q
On the other hand jd = =
0t dt 4pr 2
where D is electric desplacement vector.
(b) At the given moment,
q
E = r
4pe0 er 2
E q
and by Ohm’s law j = = r
r 4pe0e rr 2
q
Then, jd = - r
4pe0 e rr 2
q q
jd # d S = -
dS cos u
and 2
= -
I 4pe0 er 3 r e 0er

The surface integral must be - ve because jd , being opposite of j, is inward.


3.348 Here also we see that on neglecting edge effects, jd = - j. Thus Maxwell ¿ s equations
reduce to, div B = 0, Curl H = 0, B = mH.
A general solution of this equation is B = constant = B0. Here , B0 can be thought of
as an extraneous magnetic field. If it is zero, B = 0.

3.349 Given I = Im sin vt


Im 0D
We see that j = sin vt = - jd = -
S 0t
Im
or D = cos vt
vS
So, amplitude of electric field
Im
Em = = 7 V/cm
e0vS
3.350 The electric field between the plates can be written as
Vm Vm
E = Re e i vt instead of cos vt
d d
478 PART THREE ELECTRODYNAMICS

This gives rise to a conduction current,


s
jc = sE = Re Vme i vt
d
and a displacement current,
0D Vm
jd = = Re e0e iv e i vt
0t d
The total current is
Vm
jT = 2s 2 + (e0e v) 2 cos (vt + a)
d
s
where, tan a =
e0ev
The corresponding magnetic field is obtained by using circulation theorem,
H # 2pr = pr 2j T

or H = Hm cos (vt + a)
rVm
where, Hm = 2s 2 + (e0ev) 2
2d
3.351 Inside the solenoid, there is a magnetic field, given by
B = m0nIm sin vt
Since this varies in time, there is an associated electric field. This is obtained by using,
d
E # dr = - B #dS
C dt 3
s #
# # Br
For r 6 R, 2 p rE = - B p r 2 or E = -
2
# 2
BR
For r 7 R, E = -
2r
The associated displacement current density is
$
0E - e B r>2
jd = e0 = c 0 $ 2
0t - e0 B R >2r
(The answer given in the book is dimensionally incorrect without the factor e0.)

3.352 In the non-relativistic limit,


q
E = r
4pe0r 3
(a) On a straight line coinciding with the charge path,
0E q -v
c d a using = -vb
3rr dr
jd = e0 = -
0t 4p r 3 r 4 dt
3.6 ELECTROMAGNETIC INDUCTION. MAXWELL’S EQUATIONS 479

But in this case,


# r 2q v
r = - v and v = v so, jd =
r 4pr 3
#
(b) In this case, r = 0, as r ⬜ v.
qv
Thus, jd = -
4pr 3
qx
3.353 We have, Ep =
4pe0 (a 2 + x 2) 3>2
0D 0E qv
then, jd = = e0 = (a 2 - 2x 2)
0t 0t 4p(a + x 2) 5>2
2

This is maximum, when x = xm = 0, and minimum at some other value. The max-
imum displacement current density is
qv
( jd )max =
4pa 3
To check this, we calculate
0jd qv
= [( - 4x (a 2 + x 2) - 5x (a 2 - 2x 2)]
0x 4p
3
This vanishes for x = 0 and for x = a. The latter is easily shown to be a smaller
A2
local minimum (negative maximum).

3.354 We use Maxwell’s equations in the form


0
B # d r = e0m0 E # dS
C 0t 3
when the conduction current vanishes at the site.
r
We know that, a
q dS # r
E # dS = v x
3 4pe0 3 r 2
q q
= dÆ = 2p(1 - cos u)
4pe0 3 4pe0
where 2p (1 - cos u) is the solid angle, formed by the disk like surface, at the charge.
#
B # dr = 2paB = m0q # sin u # u
1
Thus,
C 2
On the other hand, x = a cot u
480 PART THREE ELECTRODYNAMICS

dx
Differentiating and using = -v
dt
#
we get, v = a cosec 2 u u
m0 qvr sin u
Thus, B =
4pr 3
This can be written as
m0q (v * r)
B =
4pr 3
q v * r
and H =
4p r 3
(The sense has to be checked independently.)
3.355 (a) If B = B(t), then,
- 0B
Curl E = Z 0
0t
So, E cannot vanish.

(b) Here also, curl E Z 0, so E cannot be uniform.


(c) Suppose for instance,
E = a ƒ(t)
where a is a spatially and temporally fixed vector. Then - 0B /dt = curl E = 0.
Generally speaking, this contradicts the other equation, curl H = 0D/0t Z 0 because
in this case the left hand side is time independent but right-hand side is depend-
ent on time. The only exception is when ƒ(t) is a linear function. Then the uni-
form field E can be time-dependent.
3.356 From the equation
0D
Curl H - = j
0t
We get on taking divergence of both sides,
0
- div D = div j
0t
But div D = r and hence,
0r
div j + = 0.
0t
0B
3.357 From the equation § * E = -
0t
On taking divergence of both sides,
0
0 = - div B
0t
This is compatible with div B = 0.
3.6 ELECTROMAGNETIC INDUCTION. MAXWELL’S EQUATIONS 481

3.358 A rotating magnetic field can be represented by


Bx = B0 cos vt; By = B0 sin vt and Bz = Bzo

- 0B
Then, Curl E =
0t
So, - (Curl E)x = - vB0 sin vt = - vBy

- (Curl E)y = vB0 cos vt = vBx and - (Curl E )z = 0

Hence, Curl E = - ␻ * B

3.359 Consider a particle with charge e, moving with velocity v, in frame K. It experiences
a force F = e (v * B).
In the frame K ¿ , moving with velocity v, relative to K, the particle is at rest. This means
that there must be an electric field E in K ¿ , so that the particle experiences a force
given by
F¿ = eE¿ = F = e v * B
Thus, E¿ = v * B

3.360 Within the plate, there will appear a (v * B) force, which will cause the charge in-
side the plate to drift, until a countervailing electric field is set up. This electric field
is related to B, as E = eB, since v and B are mutually perpendicular, and E is per-
pendicular to both. The charge density s, on the force of the plate producing this
electric field is given by
s
E = or s = e0vB = 0.40 pC/m 2
e0
3.361 Choose ␻ c c B along the z–axis, and choose r as the cylindrical polar radius vector of
a reference point (perpendicular distance from the axis). This point has the velocity
v = ␻ * r
and experiences a (v * B) force, which must be counterbalanced by an electric field
E = - (␻ * r) * B = - (␻ # B) r
There must appear a space charge density
r = e0 div E = - 2 e0 ␻ # B = - 8 pC/m 3
Since the cylinder as a whole is electrically neutral, the surface of the cylinder must
acquire a positive charge of surface density,
2 e0 (␻ # B) pa 2
s= + = e0 a ␻ # B = + 2p C/m 2
2pa
482 PART THREE ELECTRODYNAMICS

3.362 In the reference frame K ¿ moving with the particle,


qr
E¿ ⬵ E + v0 * B =
4pe0r 3
E
B¿ ⬵ B - v0 * = 0
c2
Here, v0 = velocity of K ¿ relative to the K frame, in which the particle has velocity
v. Clearly, v0 = v. From the second equation,

v * E q v * r m0 q (v * r)
B⬵ 2
= e0 m0 * 3
=
c 4pe0 r 4p r3

3.363 Suppose, there is only electric field E in K. Then in K ¿ , considering non-relativistic


velocity v,
v * E
E¿ = E, B¿ = -
c2
So, E ¿ # B¿ = 0
In the relativistic case,

E¿7 = E7 B¿7 = B 7 = 0
t
E⬜ - v * E>c 2
E¿⬜ = B¿⬜ =
21 - v 2>c 2 21 - v 2>c 2

Now, E¿ # B¿ = E7 # B¿7 + E¿⬜ B¿⬜ = 0, since


- E ⬜ # (v * E)>(1 - v 2>c 2)
E⬜¿ # B⬜¿ =
- E ⬜ # (v * E ⬜) n a 1 - b = 0
v2
c2

yi - xj
3.364 In K, B = b (where b = constant)
x2 + y2
yj - xi bvr
In K ¿, E¿ = v * B = bv =
x2 + y2 r2
The electric field is radial (r = x i+ y j).

r
3.365 In K, E = a, r = (x i + y j)
r2
v * E ar * v
In K ¿ , B¿ = - 2
=
c c 2r 2
The magnetic lines are circular.
3.6 ELECTROMAGNETIC INDUCTION. MAXWELL’S EQUATIONS 483

3.366 In the non-relativistic limit, we neglect v 2/c 2 and write

E¿7 = E7 B¿7 = B7
f
E¿ ⬜ ⬵ E⬜ + v * B B¿⬜ ⬵ B⬜ - v * E>c 2

These two equations can be combined to give


E¿ = E + v * B and B¿ = B - v * E>c 2

3.367 Choose E in the direction of the z–axis, E = (0, 0, E), The frame K ¿ is moving with
velocity v = (v sin a, 0, v cos a), in the x -z plane. Then in the frame K ¿,
E¿|| = E || B¿|| = 0
E⬜ - v * E/c 2
E¿⬜ = B¿⬜ =
21 - v 2/c 2 21 - v 2/c 2

The vector along v is e = (sin a, 0, cos a) and the perpendicular vector in the x -z
plane is f = ( - cos a, 0, sin a).

(a) Thus, using E  E cos a e + E sin a f


E sin a
We get, E ||¿ = E cos a and E ⬜¿ =
21 - v 2/c 2
1 - b 2 cos 2 a
So, E¿ = E = 9 kV/m
B 1 - b2
tan a
and tan a ¿ = (where b = v /c )
21 - v 2/c 2
v * E/c 2
(b) B ||¿ = 0, B¿⬜ =
21 - v 2/c 2
b E sin a
B¿ =
c 21 - b 2
= 14 mT

3.368 Choose B in the z direction and the velocity v = (v sin a, 0, v cos a) in the x - z
plane, then in the K ¿ frame
E ¿|| = E|| = 0 B ¿|| = B||

v * B B⬜
E¿⬜ = B ⬜¿ =
21 - v 2/c 2 21 - v 2/c 2
484 PART THREE ELECTRODYNAMICS

c b B sin a
We find similarly, E¿ =
21 - b 2

1 - b 2 cos 2 a
B¿ = B
B 1 - b2
= 1.4 n V/m
tan a
and tan a¿ =
21 - b 2
3.369 (a) We see that, E¿ # B¿ = E¿ 7 # B¿ 7 + E¿ ⬜ # B¿ ⬜
v * E
(E⬜ + v * B) # a B⬜ - b
c2
= E7 # B7 +
v2
1 -
c2
E⬜ # B⬜ - (v * B) # (v * E)>c 2
= E7 # B7 +
1 - v 2>c 2
E⬜B⬜ - (v * B⬜) # (v * E⬜)>c 2
= E7 # B7 +
v2
1 -
c2
But, A * B#C * D = A#C B # D - A # DB # C

a1 - b
v2
c2
So, E¿ # B¿ = E7 # B7 + E⬜ # B⬜ = E#B
v2
1 - 2
c
(b) We see that, E ¿ 2 - c 2 B¿ 2
= E ¿ 72 - c 2B ¿ 72 + E ¿ ⬜2 - c 2B ¿ ⬜2
v * E 2
c (E ⬜ + v * B) 2 - c 2 aB ⬜ - b d
1
= E 2 7 - c 2B 2 7 +
v2 c2
1 -
c2

c E 2⬜ - c 2B 2⬜ + (v * B ⬜ ) 2 - (v * E ⬜ ) 2 d
1 1
= E 2 7 - c 2B 2 7 +
v2 c2
1 -
c2

C E 2⬜ - c 2B 2⬜ D a1 - b = E 2 - c 2B 2
1 v2
= E 2 7 - c 2B 2 7 +
v2 c2
1 -
c2
(Since, (v * A⬜) = v A⬜2 .)
2 2
3.6 ELECTROMAGNETIC INDUCTION. MAXWELL’S EQUATIONS 485

3.370 In this case, E # B = 0, as the fields are mutually perpendicular. Also,

E 2 - c 2B 2 = - 20 * 10 8 a b
V 2
m
Thus, we can find a frame in which E ¿ = 0, and
1 E2
B¿ = 2c 2B 2 - E 2 = B 1 - 2 2
c B c B

4 * 10 4
1 - a b = 0.15 mT
2
= 0 # 20
B 3 * 10 8 * 2 * 10 –4

3.371 Suppose the charge q moves in the positive direction of the x–axis of the frame K.
Let us go over to the moving frame K¿ , at whose origin the charge is at rest. We take
the x and x¿ axes of the two frames to be coincident, and the y- and y ¿-axes to be
parallel.
In the K ¿ frame,
1 q r¿
E =
4pe0 r ¿ 3

and this has the following components

1 qx ¿ 1 qy ¿
E ¿x = and E ¿ y = .
4pe0 r ¿ 3 4pe0 r ¿ 3

Now let us go back to the frame K. At the moment when the origins of the two
frames coincide, we take t = 0. Then,

v2
x = r cos u = x ¿ 1 - and y = r sin u = y ¿
B c2

Also, Ex = E x¿ , Ey = E y¿ > 21 - v 2>c 2

r 2 (1 - b 2 sin 2 u)
From these equations, r ¿2 =
1 - b2

y¿
c (1 - b 2) 3>2 ax ¿i + jb d
q 1
E¿ =
4pe0 r3 (1 - b sin u)
2 2 3>2
21 - b 2
q r (1 - b 2)
=
4pe0r 3 (1 - b 2 sin 2 u) 3>2
486 PART THREE ELECTRODYNAMICS

3.7 Motion of Charged Particles in Electric and Magnetic Fields

3.372 Let the electron leave the negative plate of the capacitor at time t = 0.
dw w at
As, Ex = - , E = =
dx l l
therefore, the acceleration of the electron is
eE eat
w = =
m ml
dv eat
or =
dt ml
v t
ea
or dv = tdt
3 ml 3
0 0
1 ea 2
or v= t (1)
2 ml

But from, s = vdt


3 t
1 ea
l = t 2dt
2 ml 3
0
eat 3
or l =
6 ml
1/3
6 ml 2
t = a b
or ea
Putting the value of t in Eq. (1), we get
2/3 1/3

a b = a b = 16 km/s
1 ea 6 ml 2 9 ale
v =
2 ml ea 2 m
3.373 The electric field inside the capacitor varies with time as, E = at . Hence, electric
force on the proton, F = eat and subsequently, acceleration of the proton is
eat
w =
m
Now, if t is the time elapsed during the motion of the proton between the plates,
then t = l / v 7 , as no acceleration is effective in this direction. (Here v 7 is velocity alo-
ng the length of the plate.)
dv ⬜
From kinematics, = w
dt
3.7 MOTION OF CHARGED PARTICLES IN ELECTRIC AND MAGNETIC FIELDS 487

v⬜ t

So, dv ⬜ = wdt
3 3
0 0

as initially, the component of velocity in the direction ⬜ to plates was zero.


t
ea t 2 ea l 2
or v⬜ = =
3 m 2m 2m v 2 7
0

v⬜ eal 2
Now, tan a = =
v7 2mv 3 7
1

, as v 7 = a b (from energy conservation)


eal 2 2eV 2
= 3
m
2m a b
2eV 2
m

al 2 m
=
4 A 2eV 3

3.374 The equation of motion is


dv dv q
= v = (E0 - ax)
dt dx m
1 2 q 1
Integrating, v - (E0x - ax 2) = constant
2 m 2
But initially, v = 0 when x = 0, so “constant” = 0.

aE0x - ax 2 b
2q 1
Thus, v2 =
m 2
2E0
Thus, v = 0, again for x = xm =
a
The corresponding acceleration is
qE0
a b =
dv q
(E0 - 2E0) = -
dt x m m
m

3.375 From the law of relativistic conservation of energy


m0c 2
- e Ex = m0c 2
21 - (v 2/c 2)

as the electron is at rest (v = 0 for x = 0) initially.


488 PART THREE ELECTRODYNAMICS

Thus clearly, T = eEx


m0c 2
On the other hand, 21 - (v 2/c 2) =
m0c 2 + eEx

v 2(m0c 2 + eEx) 2 - m 20c 4


or =
c m0c 2 + eEx
(m0c 2 + eEx) dx
or ct = cdt =
3 3 2(m0c 2 + eEx) 2 - m 20c 4

1 dy 1
= = 2(m0c 2 + eEx) 2 - m 20c 4 + constant
2 eE 3 2y - m 2c 4 eE
0

The “constant” = 0, at t = 0, for x = 0.


1
So, ct = 2(m0c 2 + eEx) 2 - m 20 c 4
eE
Finally, using T = eEx, we get

ce # E t0 =, 2T (T + 2m0c 2)

2T (T + 2m0c 2)
or t0 = = 3.0 ns
eEc
3.376 As before, T = eEx . Now in linear motion,

d m0v m0w m 0w v
= + w
dt 21 - v 2/c 2 21 - v /c
2 2 (1 - v /c )
2 2 3>2 c 2

m0 (T + m0c 2) 3
= w = w = eE
(1 - v 2/c 2) 3>2 m 20c 6

eEm 20c 6 -3
a1 + b
eE T
So, w = =
(T + m0c 2) 3 m0 m0c 2

3.377 The equations are


m0vx m0vy
a b = 0 a b = eE
d d
and
dt 21 - (v 2/c 2) dt 21 - v 2/c 2

vx v0
Hence, = constant =
21 - v 2/c 2 21 - (v02/c 2)
3.7 MOTION OF CHARGED PARTICLES IN ELECTRIC AND MAGNETIC FIELDS 489

Also, by energy conservation


m0c 2 m0c 2
= + eEy
21 - (v 2/c 2) 21 - (v02/c 2)
v0e0 m0c 2
Dividing, vx = , e0 =
e0 + eEy 21 - (v20/c 2)
m0 e0 + eEy
Also, =
21 - (v 2/c 2) c2
Thus, (e0 + eEy)vy = c 2eEt + constant
This “constant” = 0 as vy = 0 at t = 0.
Integrating again,
1 1
e0 y + eEy 2 = c 2Et 2 + constant
2 2
Again, this “constant” = 0, as y = 0, at t = 0.

Thus, (ce E t) 2 = (eyE ) 2 + 2e0eEy + e20 - e20

or ceEt = 2(e0 + eEy)2 - e20

or e0 + eEy = 2e 20 + c 2e 2E 2t 2
v0e0 c 2eEt
Hence, vx = also, vy =
2e20 + c 2e 2E 2t 2 2e20 + c 2e 2E 2t 2
vy eEt
and tan u = = 21 - (v 20 /c 2)
vx m0v0

3.378 From the figure,


d dqB
sin a = =
R mv
as radius of the arc R = mv / qB , where v is the velocity of the particle when it
enters into the field. From initial condition of the problem

1 2qV
qV = mv 2 or v =
2 B m
dqB q
Hence, sin a = = dB
2qV A 2 mV
m
B m
a = sin -1 a dB b = 30° (on substituting values)
q
and
A 2 mV
490 PART THREE ELECTRODYNAMICS

3.379 (a) For motion along circle, the magnetic force acted on the particle will provide the
centripetal force necessary for its circular motion,
mv 2 eBr
i.e., = evB or v = = 100 km/s
r m
2p 2pr 2pm
and the period of revolution, T = = = = 6.5 ms
v v eB
dp
(b) Generally, = F
dt
# #
dp d m0v m0v m0 v (v # v )
But, = = +
dt dt 21 - (v 2/c 2 ) 21 - (v 2/c 2) (1 - (v 2/c 2)) 3>2 c2
#
For transverse motion, v # v = 0 so,
#
dp m0v m0 v2
= =
dt 21 - (v 2/c 2) 21 - (v 2/c 2) r
m0v 2 v/c Ber
Thus, = Bev or =
r 21 - v 2/c 2 21 - (v 2/c 2) m0c
v Ber
or =
c 2B 2e 2r 2 + m02c 2

2pr 2pm0 2p
Finally, T = = = 2B 2e 2r 2 + m20c 2 = 4.1 n s
v eB 21 - v 2/c 2 cBe

3.380 (a) As before, p = B qr


(b) T = 2c 2p 2 + m20 c 4 = 2c 2B 2q 2r 2 + m20c 4

v2 c2
(c) w = = -
r r [1 + (m0c /Bqr) 2]
using the result for v from the previous problem.

3.381 From solution of Problem 3.279,


2pe 2pm0c 2
T = 2 (relativistic), T0 = (non-relativistic)
c eB c 2eB

Here, m0c 2> 21 - v 2>c 2 = e

2pT
Thus, dT = , (T = K.E.)
c 2 eB
dT T
Now, = h = so, T = h m0c 2
T0 m0c 2
3.7 MOTION OF CHARGED PARTICLES IN ELECTRIC AND MAGNETIC FIELDS 491

3.382 The given potential difference is not large enough to cause significant deviations
from the non-relativistic formula. So,
1
T = eV = mv 2
2
2eV
Thus, v =
A m
2eV 2eV
So, v || = cos a v⬜ = sin a
A m A m
mv 2⬜ mv ⬜
Now, = Bev ⬜ or r =
r Be
2pr 2pm
and T = =
v⬜ Be
2pm 2eV 2mV
Pitch p = v || T = cos a = 2p cos a = 2.0 cm
Be A m A eB 2
3.383 The charged particles will traverse a helical trajectory and will be focused on the axis
after traversing a number of turns. Thus,

L n #
1 2pm 2pm
= (n + 1)
v0 qB1 qB2
n n + 1 1
So, = =
B1 B2 B2 - B1
1 2pm
Hence, =
v0 q (B2 - B1)
l2 (2 p) 2 1
= *
(q >m) 2
or
2 qV>m (B2 - B1) 2
q 8p 2V
or = 2
m l (B2 - B1) 2
3.384 Let us take the point A as the origin O and the axis of the solenoid as z-axis. At an
arbitrary moment of time let us resolve the velocity of electron into its two rectan-
gular components, v || along the axis and v ⬜ to the axis of solenoid. We know the
magnetic force does no work, so the kinetic energy as well as the speed of the elec-
tron | v ⬜ | will remain constant in the x-y plane. Thus v ⬜ can change only its direc-
tion as shown in the problem figure, v || will remain constant as it is parallel to B.
Thus at time = t ,
vx = v ⬜ cos vt = v sin a cos vt
vy = v ⬜ sin vt = v sin a sin vt
and vz = v cos a (where v = eB /m )
492 PART THREE ELECTRODYNAMICS

As at t = 0, we have x = y = z = 0, so the equation of motion of the electron is


z = v cos at
v sin a
x = sin vt
v
v sin a
y = (cos vt - 1)
v
The equation of the helix.
l
On the screen z = l, so t =
v cos a

a1 - cos b
2v 2 sin 2 a vl
Then, r2 = x2 + y2 = 2
v v cos a

2v sin a 2 vl 2 = 2 mv sin a 2 sin leB 2


r = sin
v 2v cos a eB 2 mv cos a

3.385 Choose the wire along the z–axis, and the initial direction of the electron, along the
x–axis. Then the magnetic field in the x –z plane is along the y–axis and outside
the wire it is
m0I
B = By = (as Bx = Bz = 0, if y = 0)
2px
The motion must be confined to the x - z plane. Then the equations of motion are
d
mvx = - evx By
dt
d (mvz )
= + evx By
dt

Multiplying the first equation by vx and the second by vz and then adding, we get
dvx dvz
vx + vz = 0
dt dt
or v 2x + v 2z = v 20, say or vz = 2v 20 - v 2x

dvx e m0I
Then, vx = - 2v 20 - v 2x
dx m 2px
vxdvx m0Ie dx
or - =
2v 20 - v x2 2pm x

m0Ie x
Integrating, 2v 20 - v 2x = ln
2pm a
3.7 MOTION OF CHARGED PARTICLES IN ELECTRIC AND MAGNETIC FIELDS 493

on using, vx = v0, if x = a (i.e., initially).


Now, vx = 0, when x = xm
m0Ie
So, xm = ae v0/b , where b =
2pm

3.386 Inside the capacitor, the electric field follows the 1 / r law, and so the potential can
be written as
V ln r /a -V 1
w = and E = ,
ln b /a ln b /a r
Here r is the distance from the axis of the capacitor.
mv 2 qV 1
Also, =
r ln b/ar r
qV
or mv 2 =
ln b/a
On the other hand, mv = qBr in the magnetic field.
V q v V
Thus, v = and = = 2 2
Br ln b/a m Br B r ln (b/a)

3.387 (a) The equations of motion are


dvx dvy dvz
m = - qBvz , m = qE and m = qvx B
dt dt dt
These equations can be solved easily.

qE qE
First, vy = t, y = t¿2
m 2m

Then, v 2x + v 2z = constant = v 20 (as before)

In fact, vx = v0 cos vt and vz = v0 sin vt as one can check.


Integrating again and using x = z = 0, at t = 0, we get
v0 v0
x = sin vt, z = (1 - cos vt)
v v
2p
Thus, x = z = 0 for t = tn = n
v
qE 2p 2p 2p 2 mEn 2
At that instant, yn = * * n2 * =
2m qB/m qB/m qB 2
494 PART THREE ELECTRODYNAMICS

vx
(b) Also, tan an = (vz = 0 at this moment)
vy
mv0 mv0 qB 1 Bv0
= = * * =
qEtn qE m 2pn 2pEn
3.388 The equation of the trajectory is
v0 v0 qE 2
x = sin vt, z = (1 - cos vt), y = t ( as in Problem 3.384)
v v 2m
Now on the screen x = l, so
vl vl
sin vt = or vt = sin -1
v0 v0

a sin -1 b
qE vl 2
At that moment, y =
2mv 2 v0

vl 2mv 2y 2qB 2y
So, = sin = sin
v0 B qE B Em
v0 vt vt
and z = 2 sin 2 = l tan
v 2 2
qB 2y
= l tan c sin -1 d = l tan
1 vl
2 v0 B 2mE

qB 2y
= a tan -1 b L 2
z 2 z2
For small z,
2 mE l l

or
2mE
y = # z2
qB 2l 2
This is the equation of a parabola.
E
3.389 In crossed field, eE = evB so v =
B
I E mIE
Then force exerted on the plate, F = * m = = 20 mN
e B eB
3.390 When the electric field is switched off, the path followed by the particle will be
helical and pitch
¢l = v || T
(where v || is the velocity of the particle, parallel to B and T the time period of re-
volution).
= v cos (90 - w) T = v sin w T

a as T = b
2pm 2p
= v sin w (1)
qB qB
3.7 MOTION OF CHARGED PARTICLES IN ELECTRIC AND MAGNETIC FIELDS 495

Now, when both the fields were present, qE = qvB sin (90 - w), as no net force was
effective on the system.
E
So, v = (2)
B cos w
E 2 pm
From Eqs. (1) and (2), ¢l = tan w = 6 cm
B qB
3.391 When there is no deviation,
- q E = q (v * B)
E
or in scalar from, E = vB (as v ⬜ B) or v = (1)
B
Now, when the magnetic field is switched on, let the deviation in the field be x.
Then,

a bt
1 q vB 2
x =
2 m
where t is the time required to pass through this region.
a
Also, t =
v

a ba b =
1 qvB a 2 1 q a 2B 2
Thus, x = (2)
2 m v 2m E

For the region where the field is absent, velocity in the upward direction

= a bt =
qvB q
aB (3)
m m

qaB
Now, ¢x - x = t¿
m
qa B 2b b bB
= , where t¿ = = (4)
m E v E
From Eqs. (2) and (4),
1 q a 2B 2 q aB 2b
¢x - =
2m E m E
q 2E ¢x
or =
m aB 2 (a + 2b)
3.392 (a) The equation of motion is
d 2r
m = q (E + v * B)
dt 2
496 PART THREE ELECTRODYNAMICS

i j k
3 # # # # #
Now, v * B = x y z 3 = iBy - jBx
0 0 B

So, the equation becomes,


dvx qBvy dvy qE qB dvz
= ; = - v and = 0
dt m dt m m x dt
# # #
(Here, vx = x, vy = y, vz = z ).
The last equation is easy to integrate as vz = constant = 0, since vz is zero ini-
tially. Thus integrating again, z = constant = 0, and motion is confined to the
x-y plane.

Method 1:
Let us put j = vx + ivy

# dvx dvy
or j = + i
dt dt

vy + i a v b
qB qE qB
= -
m m m x

= vvy + i a - vvx b awhere v =


vE qB a
B m
E
= vvy + i v - ivvx
B
E
= iv - iv (vx + ivy) (because i 2 = - 1)
B
This equation after being multiplied by e i vt can be rewritten as

d E
(j e i vt) = i ve i v t
dt B

and integrated to give,


E
j = + Ce -i vt - i a
B
where C and a are two real constants. Taking real and imaginary parts,
E
vx = + C cos (vt + a) and vy = - C sin (vt + a)
B
3.7 MOTION OF CHARGED PARTICLES IN ELECTRIC AND MAGNETIC FIELDS 497

Since vy = 0, when t = 0, we can take a = 0, then vx = 0 at t = 0 gives,


E
C = -
B
and we get,
E E
vx = (1 - cos vt) and vy = sin vt
B B
Integrating again and using x = y = 0, at t = 0, we get

at - b , y (t) =
E sin vt E
x (t) = (1 - cos vt)
B v vB
This is the equation of a cycloid.
Alternate of method 1, to find vx(t) and vy(t):
We have,
# qB # qE qB
vx = vy and v y = - v
m m m x
# #
and vy = v a - vx b
E
or vx = vvy
B
#
After differentiating vy with respect to time, we get
$ #
v y = - vvx = - v2vy
At t = 0, vy = 0, so its solution becomes
vy = vym sin vt (where vym is velocity amplitude)
# qE
At t = 0, vy =
m
vE E
So, v vym = Q vym =
B B
E
Hence, vy = sin vt
B
E
So ¢x = vy dt = [cos vt ]t0
L Bv
E
= [1 - cos vt ]
Bv
because at t = 0, x = y = 0.
#
Now differentiating vx = vvy with respect to time, we get
$ #
vx = vvy = v2 [E/B - vx ]
or v¶x + v2vx = v2E (1)
498 PART THREE ELECTRODYNAMICS

$
This equation is the form x + v2x = A

(where A is any constant.)


$
x + v2 a x - 2 b = 0
A
or
v
A
Putting x¿ = x -
v2
$ $
and x ¿ = x in the solutions

We get x ¿ = a cos (vt + d)

A
or x - = a cos (vt + d)
v2
A
or x = a cos (vt + d) +
v2
Thus the solution of Eq. (1) is
E
vx = vxm cos (vt + d) +
B
On differentiation
.
vx = - vvxm sin (vt + d)
.
At t = 0, vx = 0, d = 0, hence
E
vx = vxm cos vt +
B
Again at t = 0,
E
vx = 0, so, vxm = -
B
E
or vx = [1 - cos vt]
B

Alternate:
Suppose we have two inertial systems of reference: the system K and the system
K ¿ moving relative to the first system at a velocity v0. We know the magnitudes
of the fields E and B at a certain point in space and time in the system K. If
v0 V c the fields E¿ and B¿ at the same point in space and time in the system
E¿ are given by
E = E + (v0 * B), B¿ = B - (v0 * E)>c2
3.7 MOTION OF CHARGED PARTICLES IN ELECTRIC AND MAGNETIC FIELDS 499

The particle moves under the action of the Lorentz force. It can be easily seen
that the particle always remains in the plane xy. Its motion can be described
most easily in a certain system K ¿ where only the magnetic field is present. Let
us find this reference system. It follows from transformation
E¿ = E + (v0 * B), E¿ = 0 in a reference system that moves with a velocity v0
satisfying the relation E = - (v0 * B). It is more convenient to choose the sys-
tem K ¿ whose velocity v 0 is directed towards the positive values on the x-axis,
since in such system the particle will move perpendicular to vector B¿ and its
motion will be the simplest.
Thus in the system K ¿ that moves to the right at a velocity v0 = E /B, the field
E ¿ = 0 and only the field B¿ is observed. In accordance with transformation
B¿ = B - (v0 * E)>c 2 and figure, we have

B¿ = B - (v0 * E)>c2 = B(1 - v20>c2)

Since for a non-relativistic particle v0 V c , we can assume that B¿ = B.


In the system K ¿, the particle will move only in the magnetic field, it’s velocity
being perpendicular to this field. The equation of motion for this particle in the
system K ¿ will have the form
mv20
= qv0B
R
This equation is written for the instant t = 0, where the particle moved in the
syatem K ¿ as is shown in Fig. (a). Since the Lorentz force F is always perpendi-
cular to the velocity of the particle, v0 = constant, and it follows from above equa-
tion that in the system K ¿ the particle will move in a circle of radius
mv0
R =
qB

Thus the particle moves uniformly with the velocity v0 in a circle in the system K ¿,
which, in turn, moves uniformly to the right with the same velocity v0 = E/B. This
motion can be compared with the motion of the point q at the rim of a wheel Fig. (b)
rolling with the angular velocity v = v0/R = qB/m.

K′
y K

B R v0 = E
R ωt B
F q q
x
v0 O (a) O (b) x
500 PART THREE ELECTRODYNAMICS

Fig. (b) readily shows the coordinates of the particle q at the instant t are given by

x = v0t - R sin vt = a (vt - sin vt)

y = R - R cos vt = a (1 - cos vt)

mE qB
where a = 2
and v =
qB m

(b) The velocity is zero, when vt = 2n p. We see that

v 2 = vx2 + vy2 = a b (2 - 2 cos vt)


E 2
B

ds 2E 2 vt
or v = = sin 2
dt B 2

The quantity inside the modulus is positive for 0 6 vt 6 2p. Thus we can drop
the modulus and write for the distance traversed between two successive zeroes
of velocity as

a 1 - cos b
4E vt
S =
vB 2
Putting vt = 2p, we get
8E 8mE
S = =
vB qB 2

(c) The drift velocity is in the x-direction and has the magnitude

E E
6vx 7 = 6 (1 - cos vt) 7 =
B B

m0I
3.393 When a current I flows along the axis, a magnetic field Bw = is set up, where
2 pr
r = x + y . In terms of components,
2 2 2

m0Iy m0Ix
Bx = - 2
, By = and Bz = 0
2 pr 2 pr 2

Suppose a potential difference V is set up between the inner cathode and the outer
anode. This means a potential function of the form
ln r/b
w = V (a 7 r 7 b )
ln a/b
3.7 MOTION OF CHARGED PARTICLES IN ELECTRIC AND MAGNETIC FIELDS 501

as one can check by solving Laplace equation.


The electric field corresponding to this is
Vy a
Vx
Ex = - , Ey = - 2 ,E = 0
2
r ln a/b r ln a/b z b
The equations of motion are
d |e |Vz |e | m0I #
mvx = + 2 + xz
dt r ln a/b 2pr 2

d |e | Vy |e | m0I #
mvy = + 2 + yz
dt r ln a/b 2pr2
d m0I # # m0I d
and mvz = - | e | 2
(xx + yy ) = |e | ln r
dt 2 pr 2p dt
( - | e | ) is the charge on the electron.
Integrating the last equations, we get
m0I r #
mvz = - | e | ln = mz
2p a
#
Since vz = 0 when r = a , we now substitute this z in the other two equations to get

a mv 2x + mv 2y b
d 1 1
dt 2 2
# #
|e|V | e | m0I 2 r xx + y y
2
= c - a b In d #
ln a/b m 2p b r2

|e|V | e |2 m0I 2 r
= c a b ln d #
1 d 2
- r
ln a/b m 2p b 2r 2 dt
|e | V |e |2 m0I 2 r d r
= c - a b ln d ln
ln a/b m 2p a dt b

Integrating and using v 2 = 0, at r = b, we get,

|e | V m0I 2
mv 2 = c |e |2 a b a ln b d
1 r 1 r
ln -
2 ln a/b a 2m 2p b

The RHS must be positive, for all a 7 r 7 b. The condition for this is

1 | e | m0I 2
a b ln
a
V Ú
2 m 2p b
502 PART THREE ELECTRODYNAMICS

3.394 This differs from the previous problem in (a 4 b) and the magnetic field is along the
z-direction. Thus Bx = By = 0, Bz = B .
Assuming as usual the charge of the electron to be - | e | , we write the equation of
motion as

d | e | Vx # d | e | Vy #
mvx = 2 - | e | By, mvy = 2 + | e | Bx
dt r ln b>a dt r ln b>a

d
and mvz = 0 = 7 z = 0
dt
The motion is confined to the plane z = 0. Eliminating B from the first two equations,

# #
| e | V xx + yy
a mv 2 b =
we get d 1
dt 2 ln b/a r2

1 ln r/a
or mv 2 = |e | V
2 ln b/a

So, as expected, since magnetic forces do not work

2 |e | V
v = (at r = b )
B m

On the other hand, eliminating V, we also get


d # #
m (xvy - yvx) = | e | B (xx + yy )
dt

|e | B 2
i.e., (xvy - yvx) = r + constant
2m
The constant is easily evaluated, since v is zero at r = a. Thus,

|e|B 2
(xvy - yvx) = (r - a 2) 7 0
2m

At r = b, (xvy - yvx) … vb . Thus,

|e|B 2
vb Ú (b - a 2)
2m
3.7 MOTION OF CHARGED PARTICLES IN ELECTRIC AND MAGNETIC FIELDS 503

2mb 2 |e | V 1
or B … *
b2 - a B
2 m |e |
2b 2mV
or B …
b2 - a B |e |
2

3.395 The equations are as in solution of Problem 3.392.


dvx qB dvy qEm qB dvz
= vy, = cos vt - vx and = 0
dt m dt m m dt
qB
with v = , j = vx + ivy
m
dj Em
We get, = i v cos v t - ivj
dt B
Or multiplying by e i vt , we get
d Em
(je i vt ) = i v (e 2i vt + 1)
dt 2B
Em Em
Integrating, je i vt = e 2i vt + i vt
4B 2B
Em
or j = (e i vt + 2i vte i vt ) + Ce i vt
4B
Em
Since j = 0 at t = 0, C = -
4B
Em Em
Thus, j = i sin vt + i vt e i vt
2B 2B
Em Em Em
or vx = vt sin vt and vy = sin vt + vt cos vt
2B 2B 2B
Integrating again,
a a
x = 2
(sin vt - vt cos vt ), y = t sin vt
2v 2v

where a = qEm /m , and we have used x = y = 0, at t = 0.

The trajectory is an unwinding spiral.

3.396 We know that for a charged particle (proton) in a magnetic field,


mv 2
= Bev or mv = Ber
r
504 PART THREE ELECTRODYNAMICS

eB
But, v =
m
1 1
Thus, E = mv 2 = mv 2r 2
2 2
So, ¢E = mv 2r ¢r = 4p2n 2 mr ¢r
On the other hand ¢E = 2eV, where V is the effective acceleration voltage across
the dees, there being two crossings per revolution.
So, V Ú 2p 2 n2mr ¢r/e
= 0.10 MV
mv 2
3.397 (a) From = Bev or mv = Ber
r
(Ber ) 2 1
We get, T = = mv 2 = 12 MeV
2m 2
2p 2 pr
(b) From =
v v
v 1 T
We get, fmin = = = 20 MHz
2pr pr A 2m

3.398 (a) The total time of acceleration is


1
t = .n
2n
where n is the number of passages of the dees.
B 2e 2r 2
But, T = neV =
2m
B 2er 2
or n =
2mV
p B 2er 2 pBr 2 p 2m nr 2
So, t = * = = = 17 ms
eB/m 2mV 2V eV
(b) The distance covered is
1
s = a nn
2n
2eV
But, vn = 1n
B m
eV eV eV 2 3>2
So, s = a 1n = 1n dn = n
B 2m n2 B 2m n 3
2 B 2m n2 3
3.7 MOTION OF CHARGED PARTICLES IN ELECTRIC AND MAGNETIC FIELDS 505

B 2e 2r 2 2 p 2mn 2r 2
But, n = =
2 eVm eV
4p 3n 2mr 2
Thus, s L = 0.74 km
3eV

3.399 In the nth orbit,


2prn n
= nT0 =
nn n

We ignore the rest mass of the electron and write vn L c. Also W L cp = cBern.

2pW n
Thus, 2
=
Bec n
2pWn
or n = = 9
Bec 2
3.400 The basic condition is the relativistic equation
mv 2 m0v
= Bqv or mv = = Bqr
r 21 - v 2/c 2
Bq
Using, v =
m
v0 Bq
We get, v = and v0 = r
v 20 r 2 m0
1 +
C c2
The time of acceleration is
N 1 N p N pW
n
t = a = a = a 2
n=1 2n n v
n=1 n n qBc
N is the number of crossings of either Dee.
But, n¢W
Wn = m0c 2 +
2
there being two crossings of the Dees per revolution.

pm0c 2 p¢Wn
So, t = a 2
+ a
qBc 2qBc 2

p N (N + 1) p¢W p¢W
=N + 2
⬵ N2 (N W 1)
v0 4 qBc 4qBc 2
506 PART THREE ELECTRODYNAMICS

vN c 0t ¢W
Also, r = rN L ⬵ N
vN p 0N 2 qBc
v0
Hence finally, v =
q2B2 ¢W 2
1 + * N2
B m20 c 2 4q 2B 2c 2
v0 v0
= =
1¢W 2 2 4 qB c 2 21 + at
1 + * t
B 4m0 2 c 4 p¢W
qB ¢W
a =
p m02c 2

3.401 When the magnetic field is set up in the solenoid, an electric field is induced in it, which
will accelerate the charged particle. If B is the rate at which the magnetic field is
increasing, then

# 1 #
pr 2B = 2prE or E = r B
2

dv 1 # qBr
Thus, m = r B q or v =
dt 2 2m
After the field is set up, the particle will execute a circular motion of radius r, where
1
mv = B q r or r = r
2

3.402 The increment in energy per revolution is e £ , so the number of revolutions is

W
N = = 5 * 106 revolutions

The distance traversed is s = 2 prN = 8 * 103 km.

3.403 We know that,


r
dp e d£ e d
= eE = = 2pr ¿ B (r ¿) dr¿
dt 2pr dt 2pr dt 3
0

On the other hand, p = B (r) er (where r = constant)


dp d #
So, = er B (r) = er B (r)
dt dt
3.7 MOTION OF CHARGED PARTICLES IN ELECTRIC AND MAGNETIC FIELDS 507

# e d
Hence, er B (r) = pr 2 6B 7
2pr dt
# 1 d
So, B (r) = 6B7
2 dt
This equation is most easily satisfied by taking
1
B (r0) = 6B 7
2
r0
1 1 dr
3.404 The condition, B (r0) = 6B 7 = B # 2pr 2
2 23 pr0
0
r0
1
or B (r0 ) = Brdr
r20 3
This gives r0. 0

In the present case,


r0
1
B0 - ar 20 = 2 (B - ar 2) rdr
r0 3
0

a B0 - ar02 b
1 1
=
2 2
3 1
or ar 2 = B0
4 0 2
2B0
or r0 =
B 3a

3.405 The induced electric field (or eddy current field) is given by
r
1 d
E (r) = 2pr ¿ (r ¿) B (r ¿) dr ¿
2pr dt 3
0
r
dE 1 d dB (r)
Hence, = - 2pr ¿ B (r ¿) dr ¿ +
dr 2pr 2 dt 3 dt
0
1 d dB (r)
= - 6B 7 +
2 dt dt
This vanishes for r = r0 by the betatron condition, where r0 is the radius of the equi-
librium orbit.
dE # 1 #
Hence, = B (r0) - B = 0
dr 2
508 PART THREE ELECTRODYNAMICS

3.406 From the betatron condition,


1 d dB B
6B 7 = (r ) =
2 dt dt 0 ¢t
d 2B
Thus, 6B7 =
dt ¢t
d£ d 6B 7 2pr 2B
and = pr 2 =
dt dt ¢t
So, energy increment per revolution is
d£ 2pr 2eB
e = = 0.10 keV
dt ¢t

3.407 (a) Even in the relativistic case, know that p = Ber.


Thus, W = 2c 2p 2 + m20c 4 - m0c 2 = m0c 2 121 + (Ber/m0c) 2 - 12
(b) The distance traversed is
W W W¢t
2pr = 2pr 2
=
e£ 2pr eB/¢t Ber

on using the result of the previous problem.


CONTENTS

Preface vii

1BSU'PVSOscillations and Waves 1


4.1 Mechanical Oscillations 1
4.2 Electric Oscillations 59
4.3 Elastic Waves. Acoustics 90
4.4 Electomagnetic Waves. Radiation 113

1BSU'JWFOptics 129
5.1 Photometry and Geometrical Optics 129
5.2 Interference of Light 166
5.3 Diffraction of Light 179
5.4 Polarization of Light 216
5.5 Dispersion and Absorption of Light 241
5.6 Optics of Moving Sources 253
5.7 Thermal Radiation. Quantum Nature of Light 262

1BSU4JYAtomic and Nuclear Physics 287


6.1 Scattering of Particles. Rutherford–Bohr Atom 287
6.2 Wave Properties of Particles. Schrodinger Equation 313
6.3 Properties of Atoms. Spectra 338
6.4 Molecules and Crystals 366
6.5 Radioactivity 390
6.6 Nuclear Reactions 403
6.7 Elementary Particles 421
OSCILLATIONS AND WAVES
PART
PART 1
4
4.1 Mechanical Oscillations
¥ p´
4.1 (a) Given x  a cos ¦ vt µ
§ 4¶

p p
So, v x  x  av sin ¥¦ vt  ´µ and w x  x  a v2 cos ¥¦ vt  ´µ (1)
§ 4¶ § 4¶
On the basis of obtained expressions plots, x (t ), vx(t ) and wx(t ) can be drawn as
shown in the answer sheet, of the problem book.
(b) From Eq. (1)
¥ p´
v x  a v sin ¦ vt  µ
§ 4¶

¥ p´
So, v x2  a 2 v2 sin 2 ¦ vt µ (2)
§ 4¶

But from the given law of motion,


¥ p´ ¥ p´
x  a cos ¦ vt µ so, x 2  a 2 cos2 ¦ vt µ
§ 4¶ § 4¶

¥ p ´ x2 ¥ p´ x2
or cos2 ¦ vt µ  2 or sin 2 ¦ vt µ  1 2 (3)
§ 4¶ a § 4¶ a
Using Eq. (3) in Eq. (2),

¥ x2 ´
v x2  a2 v2 ¦ 1  2 µ or v x2  v2 (a 2  x 2 ) (4)
§ a ¶
2 2
or ¥ vx ´ ¥x´
¦ µ ¦ µ 1
§ av ¶ §a¶
2 PART FOUR OSCILLATIONS AND WAVES

Again from Eq. (4),


vx  av2 cos (vt  p/4) v2x

4.2 (a) From the given motion law of the particle


¥ p´ a¨ ¥ p ´·
x  a sin 2 ¦ vt  µ  ©1  cos ¦ 2vt  µ ¸
§ 4¶ 2ª § 2 ¶¹

a a ¥ p´ a a
or x  cos ¦ 2vt µ  sin 2vt  sin (2vt  p )
2 2 § 2 ¶ 2 2
a a
i.e., x  sin (2vt  p ) (1)
2 2
Now comparing this equation with the general equation of harmonic oscillations,
X  A sin (v0 t  a), we get amplitude, A  a/2 and angular frequency, v0  2v.
Thus, the period of one full oscillation, T  2pv0  pv.
(b) Differentiating Eq. (1) with respect to time, we get
v x  av cos (2vt  p )
or v x2  a2 v2 cos2 (2vt  p )  a2 v2 [1 sin  (2vt  p )] (2)
2
¥ a´ a2
Also, ¦§ x µ¶  sin 2 (2vt p ) (using Eq. 1)
2 4
x2 4x 4x ¥ x´
so, 4 1  sin 2 (2vt p ) or 1 sin 2 (2vt p )  ¦ 1 µ¶ (3)
a2 a a § a
From Eqs. (2) and (3),
4x ¥ x´
v x  a 2 v2 ¦§ 1 µ¶  4v2 x ( a x )
a a
Plot of vx (x) is as shown in the answer sheet.

4.3 Let the general equation of simple harmonic motion (S.H.M.) be


x  a cos (vt  a) (1)
So, vx  –av sin (vt  a) (2)
Let us assume that at t  0, x  x0 and v x  v x . 0

Thus, from Eqs. (1) and (2), for t  0, x0  a cos a, and v x  av sin a .
0
4.1 MECHANICAL OSCILLATIONS 3

2
vx ¥ vx ´ 0
Therefore, tan a  0
and a  x02 ¦ µ  35.35 cm
vx 0 § v¶

Under our assumption, Eqs. (1) and (2) give the sought x and vx if
2
¥ vx ´ ¥ v ´ p
t  2.40 s, a x ¦
2
0 µ
0
and a  tan 1 ¦ x µ 
§ v ¶ § vx 0 ¶ 4
Putting all the given numerical values, we get
x  29 cm and vx  81 cm/s

4.4 From the equation,


v x2  v2 (a 2 x 2 ) (see Eq. 4 of Problem 4.1)
v12  v2 ( a2 x12 ) and v22  v2 ( a2 x22 )
Solving these equations simultaneously, we get
(v12  v22 ) (v12 x22  v22 x12 )
v , a
( x22  x12 ) (v12  v22 )

4.5 (a) When a particle starts from an extreme position, you can write the motion law as
x  a cos vt (1)
(Here, x is the displacement from the equilibrium position.)
If t1 be the time taken to cover the distance a/2, then from Eq. (1), we get
a a 1 p ¥ T´
a   a cos vt1 or cos vt1   cos ¦ as t1  µ¶
2 2 2 3 § 4
p p T
Thus, t1   
3v 3(2pT ) 6
As x  a cos vt, so, vx  – av sin vt
Thus, v  | vx |  –vx  av sin vt (for t v t1  T/6).
Hence, sought mean velocity

° v dt  a ¥ 2p ´ sin vt dt  3a  0.5 ms


T 6

< v > ° ¦§ T µ¶ T 6 T
° dt 0

(b) In this case, it is easier to write the motion law in the form:
x  a sin vt (2)
If t2 is the time taken to cover the distance a/2, then from Eq. (2), we get
4 PART FOUR OSCILLATIONS AND WAVES

a 2p 2p 1 p ¥ T´
 a sin t2 or sin t2   sin ¦§ as t2 µ¶
2 T T 2 6 4
2p p T
Thus, t2  or t2 
T 6 12
Differentiating Eq. (2) with respect to time, we get
2p 2p
v x  av cos vt  a cos t
T T
2p 2p
So, v  vx  a cos t ( for t  t2  T /12)
T T
Hence, the sought mean velocity

° v dt  1 T 12
2p 2p 6a
<v> ° a cos t dt   1 ms
° dt (T 12) 0
T T T

4.6 (a) As x  a sin vt, so, vx  av cos vt


3  8T

° v dt  °
av cos (2p /T )t dt
x 0
Thus, < vx >
° dt  3T /8

2 2 av
 ( using T  2pv)
3p
(b) In accordance with the problem
v  vx i so, |< v >||< vx >|
Hence, using part (a),

2 2 av 2 2 av
| v | 
3p 3p
(c) We have, vx  av cos vt

T
So, v  |vx|  av cos vt, for t 
4
T 3
 – a v cos vt, for t  T
4 8
4.1 MECHANICAL OSCILLATIONS 5

T 4 3T  8

° v dt  ° °
av cos vt dt  av cos vt dt
0 T 4
Hence,  v 
° dt 3T  8

Using v  2p/T, and on evaluating the integral, we get


2( 4  2 )av
 v 
3p

4.7 From the given motion law, x  a cos vt, it is obvious that the time taken to cover the
distance equal to the amplitude (a), starting from extreme position equals T/4. Now,
one can write
T ¥ T ´
t n  t0 ¦§ where t0 and n  0, 1, 2, . . .µ¶
4 4
As the particle moves according to the law, x  a cos vt, so at n  1, 3, 5 . . . (or for
odd n values) it passes through the mean position and for even numbers of n it comes
to an extreme position (if t0  0).
Case 1: When n is an odd number.
In this case, from the equation x a sin vt, if t is counted from n T/4 then the distance
covered in the time interval t0 becomes,
¥ T´ ¥ np ´
s1  a sin vt0  a sin v ¦ t  n µ  a sin ¦ vt  µ
§ 4¶ § 2 ¶
Thus, the sought distance covered for odd n is
¥ np ´ ¨ ¥ np ´ ·
s  na  s1  na  a sin ¦ vt  µ  a ©n  sin ¦ vt  µ
§ 2 ¶ ª § 2 ¶ ¸¹
Case 2: When n is an even number.
In this case, from the equation x  a cos vt, if it is counted from nT/4 then the distance
covered (s2 ) in the interval t0 is given by
¥ T´ ¥ p´
a  s2  a cos vt0  a cos v ¦ t  n µ  a cos ¦ vt  n µ
§ 4¶ § 2¶

¨ ¥ np ´ ·
or s2  a ©1  cos ¦ vt  µ
ª § 2 ¶ ¸¹
Hence, the sought distance for even n is
¨ ¥ np ´ · ¨ ¥ np ´ ·
s  na  s2  na  a ©1  cos ¦ vt  µ¶ ¸  a ©n  1  cos ¦§ vt  µ
ª § 2 ¹ ª 2 ¶ ¸¹
6 PART FOUR OSCILLATIONS AND WAVES

In general,
« ¨ ¥ np ´ ·
®a ©n  1  cos ¦§ vt  2 µ¶ ¸ , if n is even
® ª ¹
s¬
® ¨ ¥ np ´ ·
®­a ©ªn  sin ¦§ vt  2 µ¶ ¸¹ , if n is odd

4.8 Obviously the motion law is of the form, x  a sin vt and vx  va cos vt. Comparing
vx  va cos vt with vx  35 cos pt, we get
35
v  p, a
p
2p T
Thus, T 2 and  0.5 s
v 4
Now, t  5 s T 4  0.3  2.8 s ( where T 4  0.5 s)
As n  5 is odd, like in Problem 4.7, we have to find the distance covered by the
particle starting from the extreme position in the time interval 0.3 s.
Thus, from the equation
35
x  a cos vt  cos p (0.3)
p
35 35 35
 s1  [ cos p (
(0.3) ] or s1  [1  cos p (
(0.3)]
p p p
Hence, the sought distance
35 35
s 5  {1  cos 0.3 p }
p p
35 35
 (6  cos 0.3 p )   7( 6 cos 54o)  60 cm
p 22

4.9 Since the motion is periodic, the particle repeatedly passes through any given region
in the range  a ≤ x ≤ a. The probability that it lies in the range (x, x  dx) is defined
as the fraction $t t (as t → ∞), where $t is the time when the particle lies in the range
(x, x  dx) out of the total time t. Because of periodicity, the probability is given by
dP dt 2dx
dP  dx  
dx T vT
where the factor 2 is needed because the particle is in the range (x, x  dx) during both
upward and downward phases of its motion. Now, in a harmonic oscillator
v  x  va cos vt  v a 2  x 2
4.1 MECHANICAL OSCILLATIONS 7

Since vT  2p (where T is the time period)


dP 1 dx
we get dP  dx 
dx p a  x2
2

a
dP
Note that ° dx dx  1
a

dP 1 1
So, 
dx p a x2
2

is properly normalized.

4.10 (a) We take a graph paper and choose an axis (x-axis) and an origin. Draw a vector
of magnitude 3 inclined at an angle p/3 with the x-axis. Draw another vector of
magnitude 8 inclined at an angle p/3 (since sin (vt  p/6)  cos (vt  p/3))
with the x-axis. The magnitude of the resultant of both these vectors (drawn from
the origin) obtained using parallelogram law is the resultant amplitude.
2p
Clearly, a 2  32  82  2 – 3 – 8 – cos
3 p/3
p/3 x
1
 9  64 48 s
2
 73  24  49
Thus, a  7 units
(b) One can follow the same graphical method here but the result can be obtained
more quickly by breaking into sines and cosines and adding:
¥ 5 ´ ¥ 5 ´
So, x  ¦3  µ cos vt ¦ 6 µ sin vt
§ 2¶ § 2¶

 A cos (vt  a)
2 2
¥ 5 ´ ¥ 5 ´ 30  60
Then a2  ¦ 3  µ¶  ¦§ 6  µ¶  9  25   36
§ 2 2 2

 70  15 2  70  21.2
So, a  6.985 ≈ 7 units
Note: In using the graphical method, convert all oscillations to either sines or co-
sines but do not use both.
8 PART FOUR OSCILLATIONS AND WAVES

4.11 Given, x1 a cos vt and x2  a cos 2vt


So, the net displacement

x  x1  x2  a (cos vt  cos 2vt)  a (cos vt  2 cos2vt  1)

and vx  x  a ( v sin vt  4v cos vt sin vt)


For x to be maximum,
x  av2 cos vt  4 av2 cos2 vt  4 av2 sin2vt  0
or 8 cos2 vt  cos vt  4  0
which is a quadratic equation for cos vt. Solving for acceptable value
cos vt  0.644
So, sin vt  0.765

and vmax |v xmax|   av [ 0.765  4 s 0.765 s 0.644 ]   2.73 av

4.12 We can write


a
a cos 2.1t cos 50.0t  (cos 52.1t  cos 47.9t)
2
Thus, the angular frequencies of constituent oscillations will be
52.1 s 1 and 47.9 s 1
To get the beat period note that the variable amplitude a cos 2.1t becomes maximum
(positive or negative), when
2.1t  np
Thus, the interval between two maxima is
p
y 1.5 s
2.1

4.13 If the frequency of A with respect to K  is n0 and K  oscillates with frequency n with
respect to K, the beat frequency of the point A in the K-frame will be n when
n  n0  n
In the present case n  20 or 24
This means n0  22 and n2
Thus, beats of 2n  4 will be heard when n  26 or 18.
4.1 MECHANICAL OSCILLATIONS 9

4.14 (a) From the equation x  a sin vt


x2 x2
sin 2 vt  2 or cos2 vt  1  2 (1)
a a
And from the equation y  b cos vt
y2
we have, cos2 vt  (2)
b2
From Eqs. (1) and (2), we get
y
x2 y2 x2 y2
1 2  2 or 2
 2 1
a b a b
which is the standard equation of ellipse
shown in the figure.
We observe that, b
a x
at t  0, x0 and yb O
and at t  p  2v, x   a and y  0

Thus, we observe that at t  0, the point is at O (see figure) and at the following
moments, the coordinate y diminishes and x becomes positive. Consequently, the
motion is clockwise.
(b) As x  a sin vt and y  b cos vt, so, we may write r  a sin vt i  b cos vt j.
Thus, r  w   v2 r

4.15 (a) From the equation x  a sin vt, we have


¥ x2 ´
cos vt  1 ¦ 2 µ
§a ¶
and from the equation y  a sin 2vt, we have
¥ x2 ´ ¥ x2 ´
y  2a sin vt – cos vt  2 x 1  ¦ 2 µ or y 2  4x 2 ¦1  2 µ
§a ¶ § a ¶
(b) From the equation, x  a sin vt
x2
sin 2 vt 
a2
From the equation, y  a cos 2vt
¥ x2 ´
y  a (1  2 sin 2 vt )  a ¦ 1  2 2 µ
§ a ¶
For the plots see answer sheet of the problem book.
10 PART FOUR OSCILLATIONS AND WAVES

4.16 As U (x)  U0 (1 – cos ax)


dU
so, Fx     U 0 a sin ax
dx
or Fx  – U0a (ax)
(because for small angle of oscillations sin ax  ax)
or Fx  – U0 a2x
But we know Fx  m v02 x , for small oscillations.
U 0 a2 U0
Thus, v02  or v0  a
m m
Hence, the sought time period
2p 2p m m
T   2p 2
v0 a U0 a U0

a b
4.17 If U (x ) 2

x x
then the equilibrium position is
x  x0 (when U ( x0 )  0 )
2a b 2a
or  3
 2  0  x0 
x0 x0 b
Now write x  x0  y
a b 1
Then, U (x )  2
  ( x  x0 ) U ( x0 )  ( x  x0 )2 U ( x0 )
x0 x0 2
3
6a 2b ¥ 2a ´ b4
But, U ( x0 )  ¦ µ  3b 2b 
x04 x03 § b ¶ 8a 3

1 ¥ b4 ´
So, finally U ( x )  U ( x0 )  ¦ 3 µ y 2  . . .
2 § 8a ¶
We neglect remaining terms for small oscillations and compare with the potential
energy for a harmonic oscillator, then
1 1 ¥ b4 ´ 2 b2
m v2 y 2  ¦ 3 µy so, v
2 2 § 8a ¶ 8a 3m

8ma 3
Thus, T  2p
b2
4.1 MECHANICAL OSCILLATIONS 11

Note: Equilibrium position is generally a minimum of the potential energy. Then


U  (x0 )  0, U  (x0 ) > 0. The equilibrium position can in principle be a maximum but
then U (x0) < 0 and the frequency of oscillations about this equilibrium position will be
imaginary. The answer given in the book is incorrect both numerically and dimension-
ally.

4.18 Let us locate and depict the forces acting on the ball at the position when it is at a
distance x below the normal (non-deformed) position of the string. At this position, the
unbalanced downward force on the ball will be mg – 2F sin u.
By Newton’s law,
mx  mg – 2F sin u
 mg – 2Fu (when u is small)
A B
x 4F
 mg  2 F  mg  x u
l 2 l x
F F
4F 4F ¥ mgl ´
Thus, x  g x  ¦x  µ
ml ml § 4F ¶
mg
mgl
Substituting x  x  , we get
4F
4F
x  x
ml
ml
Thus, T p  0.2 s
F

4.19 Let us depict the forces acting on the oscillating ball at an arbitrary angular position u
(see figure), relative to equilibrium position where FB is the force of buoyancy. For the
ball from the equation:
Nz  Ibz
(where we have taken the positive sense of z-axis in the direction of angular velocity,
i.e., u of the ball and passes through the point of suspension of the pendulum O), we
get
 mgl sin u  FB l sin u  ml 2 u (1) O
u
Using m  ( 4 3) pr 3 s,
,,FB  ( 4 3)pr 3 r and sin u  u (for small u), in T FB
Eq. (1), we get
g¥ r´
u   ¦ 1  µ u
l § s¶ mg
12 PART FOUR OSCILLATIONS AND WAVES

Thus, the sought time period


1 l g
T  2p  2p
gl (1  rs ) 1  1h

hl
Hence, T  2p  1.1 s
g (h  1)

4.20 Obviously for small b, the ball executes part of S.H.M. Due to the perfectly elastic col-
lision, the velocity of ball is simply reversed. As the ball is in S.H.M. (| u |< a on the
left), its motion law in differential form can be written as
g
u   u   v02u (1)
l
If we assume that the ball is released from the extreme position, u  b at t  0, the
solution of differential equation would be taken in the form
g
u  b cos v0t  b cos t (2)
l
If t is the time taken by the ball to go from the extreme position u  b to the wall, i.e.,
u  a, then Eq. (2) can be rewritten as
g
 a  b cos ta
l

l ¥ a´ l ¥ a´
or ta  cos1 ¦ µ  ¦ p  cos
1
µ
g § b¶ g§ b¶

l ¥ a´
Thus, the sought time T  2t a  2 ¦ p  cos
1
µ
g § b¶

l ¥p a´ 1 1
2 ¦  sin
1
µ (because sin x  cos x  p/2)
g§2 b¶

4.21 Imagine that the downward acceleration of the elevator car has continued for time t, then
the sought time t  2hw  t , where obviously 2hw is the time of upward accelera-
tion of the elevator. One should note that if the point of suspension of a mathematical
pendulum moves with an acceleration w, then the time period of the pendulum will be
l
2p
|g  w|
4.1 MECHANICAL OSCILLATIONS 13

In this problem the time period of the pendulum while it is moving upward with ac-
celeration w becomes 2p l ( g  w ) , and its time period, while the elevator moves
downward, with the same magnitude of acceleration, becomes
l
2p
gw
As the time of upward acceleration equals 2hw , the total number of oscillations dur-
ing this time is
2hw
2p l ( g  w )
Thus, the indicated time
2hw l 2h (g  w)
– 2p  –
2p l ( g  w ) g w g
Similarly, the indicated time for the time interval t
t l (g  w)
2p  t
2p l  ( g  w ) g g
We require that
2h ( g  w ) (g  w) 2h
 t   t
w g g w

2h g w g
or t 
w g  gw
Hence, the sought time
2h 2h gw  gw
t t 
w w g  gw

2h (1  h)  1  h
 (where h  w/g)
w 1 1h

4.22 If the hydrometer were in equilibrium or floating, its weight will be balanced by the
buoyancy force exerted on it by the fluid. During the small oscillation, let us locate
the hydrometer when it is at a vertically downward distance x from its equilibrium
position. Obviously, the net unbalanced force on the hydrometer is the excess buoy-
ancy force directed upward and equals pr 2x rg. Hence, for the hydrometer
mx  pr 2 rgx
pr 2 rg
or x   m x
14 PART FOUR OSCILLATIONS AND WAVES

Hence, the sought time period


m
T  2p  2.5 s
pr 2 rg

4.23 First, let us calculate the stiffness k1 and k2 of both parts of the spring. If we subject the
original spring of stiffness k having the natural length l0 (say), to the deforming forces
F–F (say) to elongate the spring by amount x, then
F  kx (1)
Therefore, the elongation per unit length of the spring is x/l0. Now, let us subject one
of the parts of the spring of natural length hl0 to the same deforming forces F–F. Then
the elongation of the spring will be
x
hl0  hx
l0
Thus, F  k1(h x) (2)
Hence, from Eqs. (1) and (2)
k  hk1 or k1  k/h (3)
k
Similarly, k2 
1 h
The position of the block m when both the parts of the spring are non-deformed, is
its equilibrium position O. Let us displace the block m towards right or in positive
x-axis by the small distance x. Let us depict the forces acting on the block when it is at
a distance x from its equilibrium position (see figure) From the second law of motion
in projection form, i.e., Fx  mwx, we have
 k1 x  k2 x  mx
k2x
¥k k ´ k 1x
µ x  mx
or ¦  k1 k2
§h 1 h ¶ m
O x
k 1
Thus, x   x
m h (1  h)
Hence, the sought time period

T  2p h (1  h) m k  0.13 s

4.24 Similar to the solution of Problem 4.23, the net unbalanced force on the block m when
it is at a small horizontal distance x from the equilibrium position becomes (k1  k2) x.
From Fx  mwx for the block
 (k1  k2 ) x  mx
4.1 MECHANICAL OSCILLATIONS 15

¥k k ´
Thus, x   ¦ 1 m 2 µ x
§ ¶
Hence, the sought time period

m
T  2p
k1 k2
Alternate:
Let us set the block m in motion to perform small oscillations. Let us locate the block
when it is at a distance x from its equilibrium position.
As the spring force is restoring conservative force and deformation of both the springs
is the same, so from the conservation of mechanical energy of oscillation of the spring–
block system
2
1 ¥ dx ´ 1 1
m ¦ µ  k1 x 2  k2 x 2  constant
2 § dt ¶ 2 2
Differentiating with respect to time
1 1
m 2x x  (k1  k2 ) 2x x  0
2 2
¥ k  k2 ´
or x   ¦ 1 µx
§ m ¶
Hence, the sought time period
m
T  2p
k1 k2

4.25 During the vertical oscillations let us locate the block at a vertically downward distance
x from its equilibrium position. At this moment if x1 and x2 are the additional or fur-
ther elongations of the upper and lower springs relative to the equilibrium position,
then the net unbalanced force on the block will be k2 x2 directed in upward direction.
Hence,
 k2 x2  mx (1)
We also have x  x1  x2 (2)
Since the springs are massless and initially the net force on the spring is also zero, so
for the spring
k 1x1  k2 x2 (3)
Solving the Eqs. (1), (2) and (3) simultaneously, we get
k k
 1 2 x  mx
k1 k2
k1 k2
Thus, x  x
m(k1 k2 )
16 PART FOUR OSCILLATIONS AND WAVES

Hence, the sought time period


(k1 k2 )
T  2p m
k1 k2

4.26 The force F, acting on the weight deflected from the position of equilibrium is 2T0 sin u.
Since the angle u is small, the net restoring force
x 2l
F  2T0
l
u u
or F  kx ( where k  2T0 l )
T0 T0
So, by using the formula,

k 2T0
v0  or v0 
m ml

4.27 If the mercury rises in the left arm by x, it must fall by a slanting length equal to x in
the other arm. Total pressure difference in the two arms will then be
rgx  rgx cos u  rgx (1  cos u)
This will give rise to a restoring force
rgSx (1  cos u)
This must equal mass times acceleration which can be obtained from work–energy
principle. The kinetic energy of the mercury in the tube is clearly 1 2 mx 2 . So mass
times acceleration must be mx. Hence,
mx  r gS (1  cos u) x  0
x
This is S.H.M. with a time period u u
x
m
T  2p  0.8 s
r gS (1 cos u)

4.28 In the equilibrium position, the centre of mass (C.M.) of the rod lies midway between
the two rotating wheels. Let us displace the rod horizontally by a small distance and
then release it. Let us depict the forces acting on the rod when its C.M. is at distance x
from its equilibrium position (see figure). Since there is no net vertical force acting on
the rod, Newton’s second law gives
N1  N2  mg (1)
For the translational motion of the rod, from the equation Fx  mwcx, we have
kN 1  kN 2  mx (2)
4.1 MECHANICAL OSCILLATIONS 17

As the rod experiences no net torque about an axis perpendicular to the plane of the
figure through the C.M. of the rod.

¥l ´ ¥l ´
N1 ¦  x µ  N 2 ¦  x µ (3)
§2 ¶ §2 ¶
l/2 l/2
Solving Eqs. (1), (2) and (3) simultaneously, we get s
s C s
2g
x   k x
l
Hence, the sought time period N1 C N2
s
l 2l s kN1 x kN2 s
T  2p p  1.5 s l/2
2kg kg mg

4.29 (a) The only force acting on the ball is the gravitational force F, of magnitude
g 4/3 pr mr, where g is the gravitational constant, r the density of the Earth and r
is the distance of the body from the centre of the Earth.
But, g  g 4p/3 rR, so the expression for F can be written as, F mg r/R, where
R is the radius of the Earth and the equation of motion in projection form has the
form, mx  mgRx  0.
(b) The equation obtained above has the form of an equation of S.H.M. having the time
period,
R
T  2p
g
Hence, the body will reach the other end of the shaft in the time
T R
t p  42 min
2 g
(c) From the conditions of S.H.M., the speed of the body at the centre of the Earth will
be maximum, having the magnitude
g
v  Rv  R  gR  7.9 kms
R

4.30 In the frame of point of suspension, the mathematical pendulum of mass m


(say) will oscillate. In this frame, it will experience the inertial force m ( w) in
addition to the real forces during its oscillations. Therefore, in equilibrium position m
is deviated by some angle, say a
T0 cos a  mg  mw cos (p – b) and T0 sin a  mw sin (p – b)
So, from these two equations
18 PART FOUR OSCILLATIONS AND WAVES

g  w cos b
tan a 
w sin b

m 2w 2 sin 2 b  (mg  mw cos b)2


and cos a  (1)
mg  mw cos b
Let us displace the bob from its equilibrium position by some small angle and then
release it. Now locate the bob at an angular position (a  u) from vertical as shown
in the figure.
From the equation N0z  Ibz, we have
–mgl sin (a  u) – mw cos (p – b)l sin (a  u)  mw sin (p – b)l cos (a  u)  m l 2 u
or – g (sin a cos u  cos a sin u) – w cos (p – b)(sin a cos u  cos a sin u)
 w sin b (cos a cos u – sin a sin u)  l u
But for small u, sin u  u, cos u  1. So,
– g (sin a  cos au) –w cos (p – b) (sin a  cos au)  w sin b (cos a – sin au)  l u
l 
or (tan a  u)(w cos b  g )  w sin b (1  tan a u)  u (2)
cos a
Solving Eqs. (1) and (2) simultaneously, we get
w w
(g 2  2wg cos b  w 2 ) u  l g 2  w 2 2wg cos b u O
b O
b
|g  w|
u  
Thus, a
u T0 au
l
T
Hence, the sought time period s s
mw mw
2p l
T  2p  0. 8 s
v0 |g  w| mg mg

4.31 Obviously the sleeve performs small oscillations mv2x


A kx B
in the frame of rotating rod. In the rod’s frame let m
us depict the forces acting on the sleeve along the O x
length of the rod while the sleeve is at a small dis-
tance x towards right from its equilibrium position.
The free-body diagram of block does not contain Coriolis force, because it is perpen-
dicular to the length of the rod. From Fx  mwx for the sleeve in the frame of rod
 kx  mv2 x  mx
¥k ´
or x   ¦  v2 µ x (1)
§ m ¶
4.1 MECHANICAL OSCILLATIONS 19

Thus, the sought time period


2p
T  0.7 s
km  v2
It is obvious from Eq. (1) that the sleeve will not perform small oscillations if
k
vr  10 rads
m

4.32 When the bar is about to start sliding along the plank, it experiences the maximum
restoring force which is being provided by the limiting friction.
Thus, kN  mv02 a or kmg  mv02 a
2
v2 a a ¥ 2p ´
or k  g0  g ¦ µ  0.4
§T ¶

4.33 The natural angular frequency of a mathematical pendulum equals v0  g l .


(a) The solution of S.H.M. equation in angular form will be
u  um cos (v0 t  a)
If at the initial moment, i.e., at t  0, u  um, then a  0. Thus, the above equation
takes the form
u  um cos v0 t

g 9.8
 um cos t  3o cos t
l 0.8
Thus, u  3°cos 3.5t
(b) The S.H.M. equation in angular form will be
u  um sin (v0 t  a)
If at the initial moment t  0, u  0, then a  0. Then the above equation takes
the form
u  um sin v0 t
Let v0 be the velocity of the lower end of pendulum at u  0, then from conserva-
tion of mechanical energy of oscillation
Emean  Eextreme or Tmean  Uextreme
1
or mv02  mgl (1  cos um )
2
¥ v2 ´
Thus, um  cos 1 ¦ 1  0 µ
§ 2 gl ¶
20 PART FOUR OSCILLATIONS AND WAVES

¨ (0.22)2 ·
 cos 1 ©1   4.5o
ª 2 s 9.8 s 0.8 ¸¹
Thus, the sought equation becomes
u  um sin v0t  4.5° sin 3.5t
(c) Let u0 and v0 be the angular deviation and linear velocity at t  0. As the mechanical
energy of oscillation of the mathematical pendulum is conserved
1
mv02  mgl (1 cos u0 )  mgl (1 cos um )
2
v02
or  gl (cos u0 cos um )
2
« v2 º
Thus, um  cos 1 ¬cos u0  0 »
­ 2 gl ¼

« (0.22)2 º
 cos 1 ¬cos 3o  »  5.4o
­ 2 s 9.8 s 0.8 ¼
Then from u  5.4° sin (3.5t  a), we see that sin a  3/5.4 and cos a < 0 because
the velocity is directed towards the centre. Thus, a  p/2  1.0 radians and we get
the answer, u 5.4° cos (3.5t  1.0).

4.34 While the body A is at its upper extreme position, the spring is obviously elongated by
the amount
¥ m1 g ´
¦§ a µ
k ¶
If we indicate y-axis in vertically downward direction, Newton’s second law of motion
in projection form, i.e., Fy  mwy for body A gives
¥ m g´ ¥ m g´
m1 g  k ¦ a  1 µ  m1 v2 a or k ¦ a  1 µ  m1 ( v2 a  g ) (1)
§ k ¶ § k ¶
(Since at any extreme position the magnitude of acceleration of an oscillating body
equals v2a and is restoring in nature.)
If N is the normal force exerted by the floor on the body B, while the body A is at its
upper extreme position, from Newton’s second law for body B
¥ m g´
N  k ¦ a  1 µ  m2 g
§ k ¶
¥ m g´
or N  m2 g  k ¦ a  2 1 µ  m2 g  m1( v2 a  g ) (using Eq. 1)
§ k ¶
4.1 MECHANICAL OSCILLATIONS 21

Hence, N  (m1  m2) g  m1v2a  40 N


When the body A is at its lower extreme position, the spring is compressed by the
distance (a  m1 gk). From Newton’s second law in projection form, i.e., Fy  mwy for
body A in this state
¥ m g´ ¥ m g´
m1 g  k ¦ a  1 µ = m1 ( v2 a ) or k ¦ a + 1 µ = m1(g + v2 a ) (2)
§ k ¶ § k ¶
In this case, if N  is the normal force exerted by the floor on the body B, from Newton’s
second law for body B, we get
¥ m g´
N a = k ¦ a  1 µ  m2 g = m1 (g  v2 a )  m2 g (using Eq. 2)
§ k ¶
Hence, N a  (m1  m2)g  m1v2a  60 N
From Newton’s third law, the magnitude of sought forces are N a and N, respectively.

4.35 (a) For the block from Newton’s second law in projection form Fy  mwy
N  mg  my (1)
But from y  a (1 – cos vt)
we get 
y  v2 a cos vt (2)
From Eqs. (1) and (2)
¥ v2 a ´
N  mg ¦ 1  cos vt µ (3)
§ g ¶
From Newton’s third law the force which the body m exerts on the block is directed
vertically downward and equals
¥ v2 a ´
N a  mg ¦ 1  cos vt µ
§ g ¶
(b) When the body m starts falling behind the plank or losing contact, N  0 (because
the normal reaction is the contact force). Thus, from Eq. (3)
¥ v2 a ´
mg ¦ 1  g cos vt µ  0
§ ¶
g
Hence, amin   8 cm
v2
(c) We observe that the motion takes place about the mean position y  a at the initial
instant y  0. As shown in (b) the normal reaction vanishes at a height (g/v2) above
the position of equilibrium and the body flies off as free body. The speed of the
22 PART FOUR OSCILLATIONS AND WAVES

body at a distance (g/v2) from the equilibrium position is v a 2  ( g/v2 )2 , so that the
condition of the problem gives
2
¨ª v a 2  ( g/v2 )2 ·¹ g
 2 ah
2g v
Hence, on solving the resulting quadratic equation and taking the positive root,
g 2hg
a    20 cm
v2 v2

4.36 (a) Let y (t)  displacement of the body from the end of the unstretched position of
the spring (not the equilibrium position). Then
my  ky  mg
This equation has the solution in the form
y  A  B cos (vt  a)
If mv2 B cos ( vt  a)   k[ A  B cos (vt  a) ] mg
k mg
then v2  and A
m k
and at t  0, we have y  0 and y  0. So,
– vB sin a  0 and A  B cos a  0
Since B > 0 and A > 0, we must have a  p
mg
BA
k
mg
and y (1  cos vt ); v  km
k
(b) Tension in the spring is
T  ky  mg (1  cos vt)
so, Tmax  2mg and Tmin  0

4.37 In accordance with the problem


F   amr
So, m ( x i 
y j)   am ( x i y j)
Thus, x   ax 
y   ay
and
Hence, the solution of the differential equation x   ax becomes
4.1 MECHANICAL OSCILLATIONS 23

x  a cos ( v0 t  d) ( where v02  a) (1)

So, x   av0 sin (v0 t  a) (2)


From the initial conditions of the problem, vx  0 and x  r0 at t  0. So from Eq. (2),
a  0, and equation takes the form
x
x  r0 cos v0t so, cos v0t  (3)
r0
One of the solutions of the other differential equation 
y   ay, becomes
y  a sin (v0t  d) (where v02  a ) (4)
From the initial condition, y  0 at t  0, so d  0 and Eq. (4) becomes
y  asin v0t (5)
Differentiating with respect to time, we get
y  aav0 cos v0t (6)
But from the initial condition of the problem, y  v0 at t  0. So, from Eq. (6)
v0
v0  av0 or a 
v0
Using it in Eq. (5), we get
v0
y sin v0 t (7)
v0
v0 y
or sin v0 t 
v0
Squaring and adding Eqs. (3) and (7), we get
v02 y 2 x2
sin 2 v0 t cos2 v0 t  2
 2
v0 r0
2 2
¥ x´ ¥ y´
or ¦§ r µ¶  a ¦§ v µ¶  1 (as a  v02 )
0 0

4.38 (a) Since the elevator car is a translating non-inertial frame, the body m will experi-
ence an inertial force mw directed downward in addition to the real forces in the
elevator’s frame. From the Newton’s second law in projection form Fy  mwy, for
the body in the frame of elevator car
¥ mg ´
k ¦  y µ mg  mw  my
§ k ¶
24 PART FOUR OSCILLATIONS AND WAVES

(because the initial elongation in the spring is mg/k).

¥ mw ´
So, my   ky  mw   k ¦ y  µ
§ k ¶

or d2 ¥ mw ´ k¥ mw ´ (1)
¦§ y  µ¶   ¦§ y  µ
dt 2 k m k ¶
Eq. (1) shows that the motion of the body m is S.H.M. and its solution becomes

mw ¥ k ´
y  a sin ¦ t  aµ (2)
k § m ¶
Differentiating Eq. (2) with respect to time

k ¥ k ´
y  a cos ¦ t  aµ (3)
m § m ¶
Using the initial condition y (0)  0 in Eq. (2), we get
mw
a sin a 
k
 0)  0 in Eq. (3), we get
and using the other initial condition y(
k
a cos a  0
m
a mw
Thus, a and a
2 k
Hence, using these values in Eq. (2), we get

mw ¥ k ´
y ¦§ 1  cos tµ
k m ¶
(b) Proceed upto Eq. (1) as in part (a). The solution of this differential equation will be

mw ¥ k ´
y  a sin ¦ t  dµ
k § m ¶

at ¥ k ´
or y  a sin ¦ t  dµ
km § m ¶

at ¥ k´
or y  a sin ( v0 t  d) ¦§ where v0  µ (4)
v02 m¶
From the initial condition that at t  0, y (0)  0, so 0  a sin d or d  0. Thus,
Eq. (4) takes the form
4.1 MECHANICAL OSCILLATIONS 25

at
y  a sin v0 t (5)
v02
Differentiating Eq. (5) we get
a
y   av0 cos vt (6)
v02
But from the other initial condition y (0)  0 at t  0. So, from Eq. (6)
a a
 2  av0 or a   3
v0 v0
Putting the value of a in Eq. (5), we get the sought y (t), i.e.,
at a a
y  2  3 sin v0 t or y  3 ( v0 t sin v0 t )
v0 v0 v0

4.39 There is an important difference between a rubber cord or steel coir and a spring.
A spring can be pulled or compressed and in both cases obeys Hooke’s law. But a
rubber cord becomes loose when one tries to compress it and does not then obey
Hooke’s law. Thus, if we suspend a body by a rubber cord, it stretches by a distance
mg/k in reaching the equilibrium configuration. If we further stretch it by a distance
$h it will execute harmonic oscillations when released if $h ≤ mg/k because only in
this case will the cord remain taut and obey Hooke’s law. Thus,
mg
$hmax   10 cm
k
The energy of oscillation in this case is
1 1 m2 g2
k ($hmax )2   4.8 mJ
2 2 k

4.40 Since the pan is of negligible mass, there is no loss of kinetic energy even though the
collision is inelastic. The mechanical energy of the body m in the field generated by
the joint action of both the gravity force and the elastic force is conserved, i.e., $E  0.
During the motion of the body m from the initial to the final position (position of maxi-
mum compression of the spring) $T  0, and therefore $U  $Ugr  $Usp  0 or
1 2
 mg (h  x ) + kx  0
2
On solving the quadratic equation, we get
mg m 2 g 2 2mgh
x p 
k k2 k
Since minus sign is not acceptable
mg m 2 g 2 2mgh
x  
k k2 k
26 PART FOUR OSCILLATIONS AND WAVES

If the body m were at rest on the spring, the corresponding position of m will be its
equilibrium position and at this position the resultant force on the body m will be zero.
Therefore the equilibrium compression $x (say) due to the body m will be given by
mg
k $x  mg or $x 
k
Therefore, separation between the equilibrium position and one of the extreme posi-
tions, i.e., the sought amplitude

m 2 g 2 2mgh mg 2kh
a  x  $x  2
  1
k k k mg
The mechanical energy of oscillation which is conserved, equals E  Uextreme, because
at the extreme position kinetic energy becomes zero.
Although the weight of body m is a conservative force, it is not restoring in this prob-
lem, hence Uextreme is only concerned with the spring force. Therefore,
1 2 m2 g2
E  U extreme  ka  mgh 
2 2k

4.41 Unlike the previous problem, the kinetic energy of body m decreases due to the per-
fectly inelastic collision with the pan. Obviously, the body m comes to strike the pan
with velocity v0  2 gh . If v be the common velocity of the “body m  pan” system
due to the collision then from the conservation of linear momentum
mv0  (M  m) v

or mv0 m 2 gh (1)
v 
( M  m) ( M  m)
At the moment the body m strikes the pan, the spring is compressed due to the weight
of the pan by the amount Mg/k. If l is the further compression of the spring due to
the velocity acquired by the “pan  body m” system, then from the conservation of
mechanical energy of the said system in the field generated by the joint action of both
the gravity and spring forces
2 2
1 1 ¥ Mg ´ 1 ¥ Mg ´
(M  m ) v 2  (M  m )gl  k ¦  lµ  k ¦ µ
2 2 § k ¶ 2 § k ¶
1 m 2 2 gh
or (M  m )  (M  m ) gl
2 ( M  m)
2 2
1 ¥ Mg ´ 1 1 ¥ Mg ´
 k¦ µ¶  kl 2  Mgl  k ¦§ µ (using Eq. 1)
2 § k 2 2 k ¶
4.1 MECHANICAL OSCILLATIONS 27

or 1 2 m 2 gh
kl  mgl  0
2 (m  M )

2 kghm 2
mg  m 2 g 2 
Thus, M m
l
k
Since minus sign is not acceptable
mg 1 2 km 2 gh
l  m2 g2 
k k ( M  m)
If the oscillating “pan  body m” system were at rest, it corresponds to the equilibrium
position, i.e., the spring was compressed by ( M  m ) g/k, therefore the amplitude of
oscillation
mg 1 2 km 2 gh mg 2hk
al   m2 g2   1
k k ( M  m) k ( M  m) g
The mechanical energy of oscillation which is only conserved with the restoring forces
becomes E  Uextreme  1/2 ka2 (because spring force is the only restoring force, not
the weight of the body).
1
Alternately E  Tmean  (M  m ) a2 v2
2
1 ¥ k ´ 1
Thus, E  ( M  m )a 2 ¦  ka 2
2 § M  m µ¶ 2

4.42 We have F  a (y i  x j)


or m (x i  
y j)  a (y i  x j)
So, mx  ay and my   ax (1)
From the initial condition, t = 0, x  0 and y  0. So, integrating equation mx  ay ,
we get
a
mx  ay or x  y (2)
m
Using Eq. (2) in the equation my  ax , we get
2
¥ a2 ´ ¥ a´
my   ¦ µ y or 
y ¦ µ y (3)
§ m¶ § m¶
One of the solutions of differential Eq. (3) is
y  A sin( v0 t  a) ( where v0  am )
28 PART FOUR OSCILLATIONS AND WAVES

As at t  0, y  0, so the solution takes the form y  A sin v0t. On differentiating with


respect to time
y  Av0 cos v0 t
From the initial condition of the problem, at t  0, y  v0 .
So, v0  Av0 or A  v0 v0

Thus, y  (v0 v0 ) sin v0 t


Thus, from Eq. (2)
x  v0 sin v0 t (4)
v0
On integrating xB cos v0 t (5)
v0
v0
On using x  0 at t  0, B
v0
v0
Finally x (1  cos v0 t ) (6)
v0
Hence, from Eqs. (4) and (6), we get
2 2
¨ ¥ v0 ´ · ¥ v0 ´
©x  ¦ v µ ¸ y  ¦ v µ
2

ª § ¶
0 ¹ § 0¶

which is the equation of a circle of radius (v0 /v0) with the centre at the point
x0 v0/v0, y0  0.

4.43 If water has frozen, the system consisting of the light rod and the frozen water in the hollow
sphere constitute a compound (physical) pendulum to a very good approximation because
we can take the whole system to be rigid. For such systems, the time period is given by
T1  2p 1 g 1  k 2 l 2 , where k 2  25 R 2 and k is the radius of gyration of the sphere.
The situation is different when water is unfrozen. When dissipative forces (vis-
cosity) are neglected, we are dealing with ideal fluids. Such fluids instantaneously re-
spond to (unbalanced) internal stresses. Suppose the sphere with liquid water actually
executes small rigid oscillations. Then the portion of the fluid above the centre of the
sphere will have a greater acceleration than the portion below the centre because the
linear acceleration of any element is in this case, equal to angular acceleration of the
element multiplied by the distance of the element from the centre of suspension (recall
that we are considering small oscillations). Then, as is obvious in a frame moving with
the centre of mass, there will appear an unbalanced couple (not negated by any pseudo
forces) which will cause the fluid to move rotationally so as to destroy differences in
acceleration. Thus, for this case of ideal fluids, the pendulum must move in such a way
4.1 MECHANICAL OSCILLATIONS 29

that the elements of the fluid all undergo the same acceleration. This implies that we have
a simple (mathematical) pendulum with the time period
l
T0  2p
g
2
2 ¥ R´
Thus, T1  T0 1 ¦ µ
5§ l ¶
(One expects that a liquid with very small viscosity will have a time period close to T0
while one with high viscosity will have period closer to T1.)

4.44 Let us locate the rod at the position when it makes an angle u from the vertical. In this
problem, both the gravity and spring forces are restoring conservative forces. Thus,
from the conservation of mechanical energy of oscillation of the oscillating system
1 ml 2  2 1 1
(u) + mg (1  cos u) k (l u)2  constant
2 3 2 2
Differentiating with respect to time, we get
1 ml 2   mgl 1
2 uu + sin u u kl 2 2 u u  0
2 3 2 2
Thus, for very small u
3g ¥ 2kl ´
u   1 u
¦
2l § mg µ¶

3g ¥ 2kl ´
Hence, v0  1
¦
2l § mg µ¶

4.45 (a) Let us locate the system when the threads are deviated through an angle a a, dur-
ing the oscillations of the system (see figure). From the conservation of mechanical
energy of the system
1 mL2 2
u  mgl (1  cos a)  constant (1)
2 12
where L is the length of the rod, u is the angular deviation of the rod from its equi-
librium position, i.e., u  0).
Differentiating Eq. (1) with respect to time
1 mL2  
2u u  mgl aa   0 ( for small a, sin a  a) (2)
2 12
30 PART FOUR OSCILLATIONS AND WAVES

But from the figure


¥ L´ ¥ L´
¦§ µ¶ u  1a or a  ¦ µ u
§ 2l ¶
2
¥ L´
So, a  ¦ µ u l
§ 2l ¶ a
Putting these values of a and d adt in Eq. (2), we
a
get C
u
d 2u 3g L/2
u
 u
dt 2 l
Thus, the sought time period
2p l
T  2p  1.1 s
v0 3g
(b) The sought oscillation energy
a
E  U extreme  mgl (1  cos a)  mgl 2 sin 2
2
a2 mgl a2 (for small angle a, sin a  a )
 mgl 2 
4 2
 0.05 J

4.46 The kinetic energy of the disk is


1 2 1 ¥ mR 2 ´ 2 1
I w  ¦ µ w  mR 2w 2
2 2§ 2 ¶ 4
The torsional potential energy is 12kw 2 . Thus, the total energy is
1 1 1 1
E  mR 2w 2  kw 2  mR 2w 02  kw02
4 2 4 2
By definition of the amplitude wm , w  0, when w  wm . Thus, the total energy is
1
E  kwm2
2
1 2 1 1
Hence, kwm  mR 2w 02 kw02
2 4 2
mR 2 w 02
or wm  w0 1
2k w02
4.1 MECHANICAL OSCILLATIONS 31

4.47 Moment of inertia of the rod equals ml 2 3 about its one end and perpendicular to its
length. Thus, rotational kinetic energy of the rod is
1 ¥ ml 2 ´ 2 ¥ ml 2 ´ 2
¦ µ u  ¦§ µu
2§ 3 ¶ 6 ¶
when the rod is displaced by an angle u its centre of gravity (C.G.) moves up by a distance
l l u2
1 cos u  (for small u)
2 4
Thus, the potential energy becomes
lu2
mg
4
Since the mechanical energy of oscillation of the rod is conserved
1 ¥ ml 2 ´ 2 1 ¥ mgl ´ 2
¦ µ u  ¦§ µ u  constant
2§ 3 ¶ 2 2 ¶
On differentiating with respect to time and then simplifying, we get u   3 g2l u for
small u. We see that the angular frequency v is 3 g2l .
We write the general solution of the angular oscillation as
u  A cos vt  B sin vt
But u  u0 at t  0, so A  u0 and B  u0 v.
u0
Thus, u  u0 cos vt  sin vt
v
Thus, the kinetic energy of the rod
ml 2 2 ml 2
T u  [ vu0 sin vt  u0 cos vt ]2
6 6
ml 2 2
 [u0 cos2 vt v2u02 sin 2 vt  2 v u0 u0 sin vt cos vt ]
6
On averaging over one time period, the last term vanishes and < sin2 vt >  < cos2 vt >  1/2.
Thus,
1 1 ¥ 3g ´
 T   ml 2 u02  mgl u02 ¦ where v2  µ
12 8 § 2l ¶

4.48 Let l  distance between the C.G. (C) of the pendulum and its point of suspension O.
Originally the pendulum is in inverted position and its C.G. is above O. When it falls
to the normal (stable) position of equilibrium, its C.G. has fallen by a distance 2l. In
the equilibrium position, the total energy is equal to K.E.  12I v2 and we have from
energy conservation
32 PART FOUR OSCILLATIONS AND WAVES

1 2 4mgl
I v  mg 2l or I 
2 v2
Angular frequency of oscillation for a physical pendulum is given by v02  mgl I .
Thus,
I 4mgl v2 4p
T  2p  2p 
mgl mgl v

4.49 Let moment of inertia of the pendulum about the axis be I, then using N Z  I bz for the
pendulum, we get
mgx
mgx sin u  I u or u   u (for small u)
I
which is the required equation for S.H.M. So, the frequency of oscillation
Mgx I
v1  or x  v12 (1)
I Mg
Now, when the mass m is attached to the pendulum at a distance l below the oscillat-
ing axis, we get
d 2u
Mgx sin u  mgl sin u  ( I  ml 2 ) 2
dt
g ( Mx  ml ) d 2u
or  u  2 ( for small u)
( I  ml )
2 dt
which is again the equation of S.H.M. So, the new frequency
g ( Mx  ml )
v2  (2)
( I  ml 2 )
Solving Eqs. (1) and (2), we get
g [( I g )v12  ml ]
v2 
( I  ml 2 )
I v12  mgl
or v22 
I  ml 2
or I ( v22  v12 )  mgl  mv22l 2
( v22  gl )
Hence, I  ml 2  0.8 g–m 2
v12  v22

4.50 When the two pendulums are joined rigidly and set to oscillate, each exerts torque on
the other; these torques are equal and opposite. We write the law of motion for the two
pendulums as
4.1 MECHANICAL OSCILLATIONS 33

I 1u   v12 I 1u  G

I 2u  v22 I 2u  G
where G is the torque of mutual interactions. We consider the restoring forces on each
pendulum in the absence of the other as v12 I 1u and v22 I 2u, respectively. Then,

I v2 I 2 v22
u   1 1 u  v2 u
I1 I 2

I 1 v12  I 2 v22
Hence, v
I1  I 2

4.51 Let us locate the rod when it is at small angular position u relative to its equilibrium
position. If a is the sought distance, then from the conservation of mechanical energy
of oscillation
1
mga (1  cos u)  I OO  (u )2  constant
2
Differentiating with respect to time, we get
1
mga sin uu    0
I OO  2 uu
2
ml 2
But, I OO   ma 2
12
Also, for small u, sin u  u, so
¨ ga ·
u   © 2 ¸u
ª (l 12)  a ¹
2

Hence, the time period of one full oscillation becomes

(l 2 12)  a 2 4p 2 ¥ l2 ´
T  2p or T2  ¦§  aµ
ag g 12a ¶

d ¥ l2 ´
For Tmin, ¦§  aµ  0
da 12a ¶
l2 l
So,  1 0 or a
12a2 2 3

l
Hence, Tmin  2p
g 3
34 PART FOUR OSCILLATIONS AND WAVES

4.52 Consider the moment of inertia of the triangular plate about AB.
I  °° x 2 dm  °° x 2 r dx dy A B

h h
h  x 2h 2r
 ° x 2 r dx –  ° x2 (h  x ) dx
0
h 3 0 3
2r ¥ h 4 h4 ´ rh 4 mh2 dx
 ¦§  µ¶  
3 3 4 6 3 6
on using the area of the triangle ABC , $  h2  3 and C
m  r$. Thus, kinetic energy
1 mh2 2
T u
2 6
h 1 u2
and potential energy U  mg(1  cos u)  mgh
3 2 3
Here u is the angle that the instantaneous plane of the plate makes with the equilibrium
position, which is vertical. (The plate rotates as a rigid body.)
1 mh2 2 1 mgh 2
E u  u
2 6 2 3
¥ mgh3 ´ 2 g
Hence, v2  ¦ 
§ mh2 6 µ¶ h

h 2h h
So, T  2p p and lreduced 
2g g 2

4.53 Let us consider the rotating frame, in which the disk is stationary. In this frame the rod
is subjected to Coriolis and centrifugal forces, Fcor and Fcf, where Fcor  ° 2 dm ( v  v0 )
and Fcf  ° dm v02 r, where r is the position of an elemental mass of the rod (see fig-
ure) with respect to point O (disk’s centre) and
ds mv2r2
d r
v  P
dt
As r  OP  OA  AP r
r
d r d ( AP)
so,   v (as OA is constant) Os A
dt dt
Since the rod is vibrating transversely, vis directed per-
v
pendicular to the length of the rod. Hence 2 dm ( v  v)
for each elemental mass of the rod is directed along PA.
4.1 MECHANICAL OSCILLATIONS 35

Therefore, the net torque of Coriolis about A becomes zero. The net torque of centrifu-
gal force about point A,
¥ m´
Tcf ( A )  ° AP  dm v02 r  ° AP  ¦ µ ds v02 (OA  AP)
§ l ¶

¥m ´ m
 ° AP  ¦ ds µ v02 OA  ° ds v02 s a sin u (k )
§ l ¶ l
l
m 2 l
 v0 a sin u (k ) ° s ds  mv02 a sin u (k )
l 0
2
l
So, t cf ( Z )  Tcf ( A )  k   mv02 a
sin u
2
According to the equation of rotational dynamics, tA ( z )  I A az .
l ml 2 
or  mv02 a sin u  u
2 3
3 v02 a
or u   sin u
2 l
3 v02 a
Thus, for small u u   u
2 l
This implies that the frequency of oscillation is
3 v2 a
v0 
2l

4.54 The physical system consists of a pulley and the block. Choosing an inertial frame, let
us direct the x-axis as shown in the figure.
Initially the system is in equilibrium position. Now from the condition of translation
equilibrium for the block
T0  mg (1)
T0 k$ l T T
Similarly for the rotational equilibrium of the pulley
k$lR  T0 R T0 T
O O
or T0  k$l (2) x
mg
mg mg
From Eqs. (1) and (2) $l  (3)
k x x
Suppose the equilibrium of the system is disturbed in some way, in order to analyze
its motion. At an arbitrary position shown in the figure, from Newton’s second law of
motion for the block
Fx  mw x
36 PART FOUR OSCILLATIONS AND WAVES

mg  T  mw  mx (4)
Similarly, for the pulley
N Z  I bZ
TR  k ( $l  x ) R  I u (5)
w  bR or x  R u (6)
I
From Eqs. (5) and (6) TR  k ( $l  x ) R  x (7)
R
Solving Eqs. (4) and (7) using the initial condition of the problem,
¥ I´
 kRx  ¦ mR  µ x
§ R¶
¥ k ´
or x   ¦ x
§ m  ( I R 2 ) µ¶
Hence, the sought time period
2p m  I R 2 k
T  2p and v0 
v0 k m  I R 2
Note: We may solve this problem by using the conservation of mechanical energy also.

4.55 At the equilibrium position,


Nz  0 (net torque about O)
So, mA gR  mg R sin a  0 or m A  m sin a (1)
From the equation of rotational dynamics of a solid body about the stationary axis
(say z-axis) of rotation, i.e., from N z  I bZ , when the
pulley is rotated by the small angular displacement u in
clockwise sense relative to the equilibrium position (see
figure), we get

¨ MR 2 ·
m A gR  mgR sin (a  u)  ©  mR 2  m A R 2 u
ª 2 ¹̧
Using Eq. (1) Os

mg sin a  mg (sin a cos u  cos a sin u) R


u a
ms
 [
MR  2m (1  sin a) R 
2
u ] A
But for small u, we may write cos u  1 and sin u  u
4.1 MECHANICAL OSCILLATIONS 37

Thus, we have
{ MR  2m (1  sin a) R } 
mg sin a  mg (sin a  cos au)  u
2
2mg cos a
Hence, u   u
[ MR  2m (1  sin a) R ]
Hence, the sought angular frequency,

2mg cos a
v0 
MR  2mR (1  sin a)

4.56 Let us locate a solid cylinder when it is displaced from its s


stable equilibrium position by the small angle u during its
R vc
oscillations (see figure). If v0 is the instantaneous speed of u
the C.M. (C) of the solid cylinder which is in pure rolling,
then its angular velocity about its own centre C is
v
v C (1)
r
Since C moves in a circle of radius (R  r), the speed of C at the same moment can
be written as
vC  u ( R  r ) (2)
Thus, from Eqs. (1) and (2)
(R  r )
v  u (3)
r
As the mechanical energy of oscillation of the solid cylinder is conserved, i.e.,
E  T  U  constant
1 1
So, mvC2  I C v2  mg ( R  r ) (1  cos u)  constant
2 2
(Here m is the mass of solid cylinder and IC is the moment of inertia of the solid cylin-
der about an axis passing through its C.M. (C) and perpendicular to the plane of figure
of solid cylinder.)
1 1 mr 2 2
or mv2r 2  v  mg ( R  r ) (1  cos u)  constant
2 2 2
(using Eq. 1 and I C  mr 2 2 )

3 2  2 ( R  r )2
r (u)  g ( R  r ) (1  cos u)  constant (using Eq. 3)
4 r2
38 PART FOUR OSCILLATIONS AND WAVES

Differentiating with respect to time


3
( R  r ) 2u u  g sin u u  0
4
2g
So, u   u (because for small u, sin u  u)
3( R  r )

2g
Thus, v0 
3 (R  r )
Hence, the sought time period

2p 3 (R  r )
T  2p
v0 2g

4.57 Let k1 and k2 be the spring constants of left and right side springs. As the rolling of
the solid cylinder is pure, its lowest point becomes the instantaneous centre of rotation.
If u is the small angular displacement of its uppermost point relative to its equilibrium
position, the deformation of each spring becomes 2Ru. Since the mechanical energy of
oscillation of the solid cylinder is conserved, E  T  U  constant, i.e.,
1 1 1
I P (u )2  k1 (2 R u)2  k2 (2 R u)2  constant
2 2 2
Differentiating with respect to time, we get
1 1
I P 2u u  ( k1  k2 ) 4 R 2 2 uu  0
2 2
¥ mR 2 ´
or ¦§  mR 2 µ u  4 R 2 ku  0
2 ¶

mR 2
(because I P  I C  mR 2   mR 2 ).
2
8 k
Hence, u  u
3m
8k
Thus, v0 
3m
and the sought time period,

2p 3m 3m
T  2p p
v0 8k 2k
4.1 MECHANICAL OSCILLATIONS 39

4.58 In the C.M. frame (which is rigidly attached to the centre of mass of the two cubes) the
cubes oscillate. We know that the kinetic energy of two-body system equals 12 mvrel 2 ,

where m is the reduced mass and vrel is the modulus of velocity of any one-body par-
ticle relative to other. From the conservation of mechanical energy of oscillation
2
1 1 «d º
kx 2  m ¬ (l0  x ) »  constant
2 2 ­ dt ¼
(Here l0 is the natural length of the spring.)
Differentiating the above equation with respect to time, we get
1 1 ¨ d ( l0  x ) ·
k 2xx  m 2xx
  0 ©ª because  x
2 2 dt ¹̧
k ¥ m1m2 ´
Thus, x   x ¦§ where m  m  m µ¶
m 1 2

Hence the natural frequency of oscillation

k
v0 
m

4.59 Suppose balls 1 and 2 are displaced by x1 and x2 respectively from their initial posi-
tions. Then the energy is
1 1 1 1
E m1 x12  m2 x 22  k ( x1  x2 )2  m1v12
2 2 2 2
Also total momentum is
m1 x1  m2 x 2  m1v1
m1 x1  m2 x2
If X and x  x1  x2
m1  m2
m2 m1
Then x1  X  x and x2  X  x
m1  m2 m1  m2
1 1 m1m2 1
E (m1  m2 ) X 2  x 2  kx 2
2 2 m1  m2 2
m1v1
Now, from total momentum X 
m1  m2

1 m1m2 1 1 1 m12v12 1 m1m2 2


So, x 2  kx 2  m1v12   v1
2 m1  m2 2 2 2 m1  m2 2 m1  m2
40 PART FOUR OSCILLATIONS AND WAVES

(a) From the above equation, we see


k 3  24 m1m2 2
v   6 s 1 , when m   kg
m 2 m1  m2 3
(b) The energy of oscillation is
1 m1m2 1 2
v12    (0.12)2  48  10 4  4.8 mJ
2 m1  m2 2 3
We have x  a sin ( vt  a)
Initially, x  0 at t  0, so a  0. Then, x  a sin vt. Also x  v1 at t  0.
So, va  v1 and hence,
v1 12
a   2 cm
v 6

4.60 Suppose the disk 1 rotates by angle u1 and the disk 2 by angle u2 in the opposite sense.
Then total torsion of the rod  u1  u2 and torsional potential energy is
1
k (u1  u2 )2
2
The kinetic energy of the system (neglecting the moment of inertia of the rod) is
1 2 1 2 (1) (2)
I 1u1  I 2u2
2 2
So, total energy of the rod
1 2 1 2 1
E I 1u1  I 2u2  k (u1  u2 )2
2 2 2
We can put the total angular momentum of the rod equal to zero since the frequency
associated with the rigid rotation of the whole system must be zero (and is known).
u1 u u  u2
Thus, I 1u1  I 2u2 or  2  1
1I 1 1I 2 1I 1  1I 2
I2 I1
So, u1  (u1  u2 ) and u2  (u1  u2 )
I1  I 2 I1  I 2
1 I1 I 2 1
and E (u1  u2 )2  k(u1  u2 )2
2 I1  I 2 2
The angular oscillation frequency corresponding to this is
k k Ia ¥ I1 I 2 ´
v2   and T  2p ¦§ where I a  I  I µ¶
I 1 I 2 ( I 1  I 2 ) I  k 1 2
4.1 MECHANICAL OSCILLATIONS 41

4.61 In the first mode, the carbon atom remains fixed and the oxygen atoms move in equal
and opposite steps. Then total energy is
O C O
1 1
2mo x  2kx
2 2
2 2
x
where x is the displacement of one of the O atoms x
(say left one). O C O
k
Thus, v12 
mo x y x
In the second mode, the oxygen atoms move in equal steps in the same direction but
the carbon atom moves in such a way so as to keep the centre of mass fixed.
2mo
Thus, 2mo x  mC y  0 or y   x
mC
2
1 1 ¥ 2m ´ 1 1 2mo 2 1 ¥ 2mo ´ 2
K.E.  2mo x  mC ¦ o x µ  2mo x 2  2mo x  2mo ¦ 1  x
2 2 § mC ¶ 2 2 mC 2 § mC µ¶
2 2 2
1 ¥ 2mo ´ 2 1 ¥ 2mo ´ 2 1 ¥ 2mo ´ 2
P.E.  k 1 x  k ¦1  x  2k ¦ 1  x
2 ¦§ mC µ¶ 2 § mC µ¶ 2 § mC µ¶

k ¥ 2mo ´ 2mo
Thus, v22  ¦§ 1  m µ¶ and v2  v1 1 
mo C mC

32 11
Hence, v2  v1 1   v1  1.91 v1
12 3

4.62 Let us displace the piston through small distance x towards right, then from Fx  mwx
( p1  p1 )S   mx (1)
g
But, the process is adiabatic, so from pV  constant
p0V0g p0V0g
p2  and p1 
(V0  Sx )g (V0  Sx )g
as the new volumes of the left and the right parts are now (V0  Sx) and (V0  Sx),
respectively.
So, Eq. (1) becomes p1S
p0V0g S « 1 1 º
¬ g
 g »
  x p2S
m ­ (V0  Sx ) (V0  Sx ) ¼ x

p0V0g S « (V0  Sx )g  (V0  Sx )g º


or ¬ »   x
m ­ (V02  S 2 x 2 )g ¼
42 PART FOUR OSCILLATIONS AND WAVES

¥¥ gSx ´ ¥ gSx ´ ´
¦
g ¦§ 1  V µ¶  ¦§ 1  V µ¶ µ
pV S 0 0
or 0 0
¦ µ   x
m ¦ g ¥
2
gS x ´ 2
µ
¦§ V0 ¦ 1  2 µ µ¶
§ V0 ¶
gS 2 x 2
Neglecting the term in the denominator, as it is very small, we get
V02
2 p0 S 2gx
x  
mV0
which is the equation for S.H.M. and hence the oscillating frequency will be
2 p0g
v0  S
mV0

4.63 In the absence of the charge, the oscillation period of the ball
l
T  2p
g
when we impart the charge q to the ball, it will be influenced by the induced charges
on the conducting plane. From the electric image method, the electric force on the ball
by the plane equals q 2 4pd0 (2h )2 and is directed downward. Thus, in this case the
effective acceleration of the ball will be
q2
ga  g 
16pd0mh2
and the corresponding time period
l l
T a  2p  2p
ga g  (q 16pd0mh2 )
2

From the condition of the problem


T  hT a
1 ¥ 1 ´
So, T 2  h2T a 2 or  h2 ¦ 2 16pd mh 2 ) µ
g § g  (q 0 ¶
Thus, on solving
q  4h pd0mg (h2  1)  2 MC

4.64 In a magnetic field of induction B the couple on the magnet is MB sin u MB u.
Equating this to I u, we get
I u  MBu  0
4.1 MECHANICAL OSCILLATIONS 43

MB I
or v2  or T  2p
I MB
T1
Given T2 
h
1 1 1 1 1 1
or  – or  – or B2  h2 B1
B2 B1 h B2 B1 h2
The induction of the field increased h2  25 times.

4.65 We have in the circuit at a certain instant of time (t), from Faraday’s law of electromag-
netic induction
dI dx
L  Bl or L dI  Bl dx
dt dt
As at t  0, x  0, so
Bl
LI  Blx or x I (1)
L
For the rod from the second law of motion, Fx  mwx, we have
 IlB  mx
¥ l 2 B2 ´
Using Eq. (1), we get x   ¦ µ x   v0 x
2 (2)
§ mL ¶
lB
where v0 
mL
The solution of the above differential equation is of the form
x  a sin ( v0 t  a)
From the initial condition, at t  0, x  0, so a  0.
Hence, x  a sin v0t (3)
Differentiating with respect to time,
x  av0 cos v0 t
But from the initial condition of the problem at t  0, x  v0 .
v
Thus, v0  av0 or a  0 (4)
v0
Putting the value of a from Eq. (4) into Eq. (3), we obtain
v
x  0 sin v0 t
v0
44 PART FOUR OSCILLATIONS AND WAVES

4.66 As the connector moves, an emf is set up in the circuit and a current flows, since the
emf is j  Blx, we must have
 Blx  dI dt  0 L
Blx
So, I
L x
provided x is measured from the initial position.
We then have,
Blx
mx   – Bl  mg
L
Since by Lenz’s law, the induced current will oppose downward sliding. Finally,
( Bl )2
x  xg
mL
Bl
on putting v0 
mL
A solution of this equation is x  gv02  A cos ( v0 t  a).
But x  0 and x  0 at t  0.
g
This gives x (1  cos v0 t )
v02

4.67 We are given x  a0 e bt sin vt.


(a) The velocity of the point at t  0 is obtained from
v 0  ( x )t 0  va0
The term “oscillation amplitude at the moment t  0” is meaningless. Probably the
implication is the amplitude for t  1/b. Then x  a0 sin vt and amplitude is a0.
(b) When the displacement is an extremum x  ( ba0 sin vt  va0 cos vt ) e bt  0
v
Then, tan vt 
b
v
or tn  tan1  np ( where n  0, 1, 2, . . .)
b

4.68 Given w  w0 e bt cos vt


We have w  bw  vw0 e bt sin vt
4.1 MECHANICAL OSCILLATIONS 45

   bw  bv w0 e bt sin vt  v2 w0 e bt cos vt


w
 b2w  2bv w0 e bt sin vt  v2w
So,
(a) (w )0   bw0 , (w
)0  ( b2  v2 )w0
(b) w   w0 ebt ( b cos vt  v sin vt ) becomes maximum (or minimum) when
  w0 ( b2  v2 )ebt cos vt  2 bvw0 eBr sin vt  0
w
v2  b2
or tan vt 
2 bv

1 ¨ 1 v2  b2 ·
and tn  © tan  np ¸ ( where n  0, 1, 2, . . .)
vª 2 bv ¹

4.69 We write x  a0 e bt cos ( vt  a)


(a) x (0)  0  a   p2  x   a0 e bt sin vt.
Since a0 is ve, we must choose the upper sign if x (0)  0 and the lower sign if
x (0)  0. Thus,
| x (0)|
a0 
v
and a [ pp 22 ifif xx ((00))  00
(b) We write
x  Re Aebt  i vt and A  a0 e ia
Then, x  v x  Re ( b  i v) Aebt  i vt
From vx(0)  0, we get Re ( b  i v) A  0.
This implies A i (b iv)B where B is real and positive. Also x0  Re A vB.
Thus, B  x0 v with ve sign in A if x0 0 and vesign in A if x0  0.
b  iv ¥ i b´
So, A  i | x0 | ¦  1   µ | x0 |
v § v¶
2
Finally ¥ b´
a0  1  ¦ µ | x0 |
§ v¶
b ¥  b´
tan a  or a  tan1 ¦
v § v µ¶
a is in the fourth quadrant (p2  a  0) if x0  0 and a is in the second quad-
rant (p2  a  p ) if x0 0.
46 PART FOUR OSCILLATIONS AND WAVES

4.70 We write x  a0 e bt cos ( vt  a)


Then, ( x )t  0   ba0 cos a  va0 sin a  0
b
or tan a  
v
a0
Also, ( x )t  0  a0 cos a
h
sec2 a  h2 and tan a   h2  1

Thus, b  v h2  1  5 s1
(We have taken the amplitude at t  0 to be a0.)

4.71 We write x  a0 e bt cos ( vt  a)


 Re Aebt  i vt , A  a0e ia
x  Re A ( b  i v) ebt  i vt
Velocity amplitude as a function of time is defined in the following manner.
Put t  t0  t, then
x  Re Aeb( t 0  t)
e iv (t 0  t)

 Re Aebt e i vt
0 0  i vt
 Re Aebt e i vt (for t  1/b)
0

This means that the displacement amplitude around the time t0 is a0 ebt and we can say 0

that the displacement amplitude at time t is a0 ebt . Similarly for the velocity amplitude
(a) Velocity amplitude at time t is
a0 b2  v2 ebt
Since A( b  i v)  a0 e ia ( b  i v)

 a0 b2  v2 eig (where g is another constant)


(b) x (0)  0  Re A  0 or A   ia0 (where a0 is real and positive)
Also, v x  (0)  x 0  Re  ia0 ( b  i v)
 va0
Thus, a0 | x 0 | /v and we take ve sign if x0 < 0 and ve sign if x0 > 0. Finally,
the velocity amplitude is obtained as
2
¥ b´
| x 0 | 1  ¦ µ ebt
§ v¶
4.1 MECHANICAL OSCILLATIONS 47

4.72 The first oscillation decays faster in time. But if one takes the natural time scale, the pe-
riod T for each oscillation, the second oscillation attenuates faster during that period.

4.73 By definition of the logarithmic decrement ( l  b 2pv), we get for the original decre-
ment
2p 2 pn b
l0  b and finally l 
v0  b
2 2 v02  n2 b2
b l b l0 2p
Now,  0 or 
v b
2
0
2 2p v0 1  ( l0 2p )2

l2p n( l0 2p )
So, 
1  ( l0 2p )2 1  ( l0 2p )2

l n l0 2p
Hence, 
2p 1  (n2  1)( l0 2p )2

For critical damping v0  nc b


2
1 b l0 2p ¥ 2p ´
So,   or nc  1  ¦ µ
nc v0 1  ( l0 2p )2 § l0 ¶

4.74 The equation for the dead weight is


mx  2bmx  mv02 x  mg
g g
So, $x  or v02 
v02 $x
2
2pb 2pb v0 ¥ l´
Now, l  or  1 ¦ µ
v v02  b2 v0  b
2 2 § 2p ¶

2
2p 2p ¥ l´
Thus, T  1 ¦ µ
v02  b2 v0 § 2p ¶

2
$x ¥ l´ $x
 2p 1 ¦ µ  ( 4p 2  l2 )  0.70 s
g § 2p ¶ g

4.75 The displacement amplitude decreases h times every n oscillations. Thus,


1
 e  b – 2 p v – n
h
48 PART FOUR OSCILLATIONS AND WAVES

2 pn b b ln h
or  ln h or
v v 2 pn
v pn
So, Q   499
2b ln h

4.76 From x  a0e bt cos ( vt  a), we get using ( x )t  0  l  a0 cos a


0  ( x )t  0   ba0 cos a  va0 sin a
b v l
Then, tan a   or cos a 
v v  b2
2
1

l v2  b2 ¥ b´ t
and x ebt cos ¦ vt  tan1 µ
v § v¶ 2
3
1¥ p b´
x 0 at t  ¦ np   tan1 µ
v§ 2 v¶
Total distance traveled in the first lap  l.
To get the maximum displacement in the second lap we note that
¨ ¥ b´ ¥ b´ ·
x  © b cos ¦ vt  tan1 µ  v sin ¦ vt  tan1 µ ¸
ª § v¶ § v¶ ¹

l v2  b2
Now, x e bt  0
v
when vt  p, 2 p, 3 p, . . ..
Thus, xmax   a0 e pb/v cos a   le pb/v (for t  p/v)
So, distance traversed in the second lap  2lepbv .
Continuing total distance traversed  l  2l epb/v  2le2pb/v  . . .
2lep b/v
l 
1  ebp/v
1  el/2
l
1  el/2
where l  2pbv is the logarithmic decrement. Substitution gives distance  2 m.

4.77 For an undamped oscillator, the mechanical energy E  12 mx 2  12 mv02 x 2 is con-
served. For a damped oscillator
x  a0 e bt cos ( vt  a), v  v02  b2
4.1 MECHANICAL OSCILLATIONS 49

1 1
and E (t )  mx 2  mv02 x 2
2 2
1
 ma02 e2 br b2 cos2 ( vt  a)  2 bv cos ( vt  a)
2
1
 sin ( vt  a)  v2 sin 2 ( vt  a)  ma02 v02 e 2 bt cos2 ( vt  a)
2
1 1
 ma02 v02 e 2 bt  ma02 b2 e 2 bt cos (2vt  2a)
2 2
1
 ma02 b ve 2 bt sin (2vt  2a)
2
If b  v, then the average of the last two terms over many oscillations about the time
t will vanish and
1
 E (t )   m a02 v02 e 2 bt
2
This is the relevant mechanical energy. In time t this deceases by a factor 1/h,
so
1 ln h
e2 bt  or t
h 2b
ln h
b
2t
2pb 2p 2p ¥ g´
and l   ¦§ since v02  µ¶
v0  b
2 2 ( v0 b)  1
2 [ 4 gt l (ln h)2 ]  1
2 l

p 1 4 gt 2
and Q   1  130
l 2 l (ln h)2

4.78 The restoring couple is


'   mgR sin w   mg Rw
w
The moment of inertia is
3mR 2
I
2
Thus, for undamped oscillations
3 mR 2
  mgR w  0
w mg
2
50 PART FOUR OSCILLATIONS AND WAVES

2 g
So, v02 
3 R
2pb 2pb
Also l 
v v02  b2
2
b l v0 ¥ l´
Hence,  or  1 ¦ µ
v0  b
2 2 2p v0  b2 § 2p ¶

Finally the period T of small oscillation comes to


2
2p 2 p v0 3R ¥ ¥ l´ ´
T   2p ¦ 1  ¦§ µ¶ µ¶
v v0 v02  b2 2g § 2p

3R
 ( 4p 2  l2 )  0.90 s
2g

4.79 Let us calculate the moment G1 of all the resistive forces on the disk. When the disk
 where w is the
rotates an element (r dr du) with coordinates (r, u) has velocity r w,
instantaneous angle of rotation from the equilibrium position and r is measured from
the centre. Then,
2p R
G1  °
0
du ° dr – r – ( F
0
1 r)

R
hpR 4
 ° hr w r 2d  2p  w
0
2

Also, moment of inertia  mR 2 2.


mR 2 phR 4
Thus,  
w w  aw  0
2 2
phR 2 2a
or   2
w w  w0
2m mR 2
2a phR 2
Hence, v02  and b
mR 2 2m
2
¥ 2a ´ ¥ phR 2 ´
and angular frequency v ¦ 
§ mR 2 µ¶ ¦§ m µ¶

(Normally by frequency we mean v2p.)


4.1 MECHANICAL OSCILLATIONS 51

4.80 From the law of viscosity, force per unit area  h dvdx , so when the disk executes
torsional oscillations the resistive couple on it is
R
rw hpR 4
° h – 2pr – – r – dr  2  w ( where w is torsion )
0
h h
(Here factor 2 is for the two sides of the disk; see the figure in the book.)
The equation of motion is
hpR 4
 
Iw w  cw  0
h
Comparing with   2bw  v02 w  0
w
hpR 4
we get, b
2hI
Now the logarithmic decrement l is given by
l  bT (where T  time period)
2 lhI
Thus, h
pR 4T

4.81 If w  angle of deviation of the frame from its normal position, then emf
d  Ba 2 w
is induced in the frame in the displaced position and a current d R  B a2w R flows
in it. A couple
B a 2w B 2a 4
– B –a–a  w
R R
then acts on the frame in addition to any elastic restoring couple cw. We write the
equation of the frame as
B 2a 4
 
Iw w  cw  0
R
Thus, b  B 2 a 4 2IR, where b is the damping coefficient.
Amplitude of oscillation dies out according to ebt , so time required for the oscillation
to decrease e-fold is
1 2 IR
 2 4
b B a

4.82 We shall denote the stiffness constant by k. Suppose the spring is stretched by x0. The
bar is then subjected to two horizontal forces: restoring force kx and friction kmg
opposing the motion of the bar. If
52 PART FOUR OSCILLATIONS AND WAVES

kmg
x0  $
k
the bar will come back. If x0  $ , the bar will stay put. The equation of the bar when
it is moving to the left is
mx  kx  kmg x
This equation has the solution
k
x  $   x0  $ cos t
m
where we have used x  x0 , x  0 at t  0 . This solution is only valid till the bar comes
to rest. This happens at
p
t1 
k/m
and at the time x  x1  2$  x0 . If x0  2$ the tendency of the bar will now be to
move to the right. (If $  x0  2$ the bar will stay put now.) Now the equation for
rightward motion becomes
mx  kx  kmg
(The friction force has reversed.)
We notice that the bar will move to the right only if
k ( x0  2$ )  kmg, i.e., x0  3$
In this case, the solution is
k
x   $  ( x0  3$ ) cos t
m
p
Since x  2$  x0 and x  0 at t  t1  the bar will next come to rest at
km
2p
t  t2 
km
and at that instant x  x2  x0  4$ . However, the bar will stay put unless x0  5$.
Thus,
2p
(a) Time period of one full oscillation   0.28 s.
k/m
(b) There is no oscillation if 0  x0  $; one half oscillation if $  x0  3$; two half
oscillations if 3$  x0  5$, and so on
We can say that the number of full oscillations is one half of the integer n,
4.1 MECHANICAL OSCILLATIONS 53

¨x  $·
n© 0
ª 2$ ¹̧
(where for any quantity a, [a]  the smallest non-negative integer greater than a).

4.83 The equation of motion of the ball is


m ( x  v02 x )  F0 cos vt
This equation has the solution
x  A cos ( v0 t  a)  B cos vt
where A and a are arbitrary and B is obtained by substitution in the above equation as
F0 m
B
v  v2
2
0

The condition x  0, x  0 at t  0 gives


F0 m
A cos a  0 and  v0 A sin a  0
v02  v2
F0 m F m
This gives for a  0, A   20 2
v v2
0
2 v  v0
F0 m
Finally, x (cos v0 t  cos vt )
v  v02
2

4.84 We have to look for solutions of the equation


mx  kx  F ( for 0  t  t )
mx  kx  0 ( for t  t )
subjected to x (0)  x (0)  0 , where F is constant.
The solution of this equation will be sought in the form
F
x  A cos ( v0 t  a) ( for 0  t  t )
k
x  B cos [ v0 (t  t )  b] ( for t  t )
A and a will be determined from the boundary condition at t  0.
F
0  A cos a
k
54 PART FOUR OSCILLATIONS AND WAVES

0 v0 A sin a
F
Thus, a0 and A 
k
F
and x (1  cos v0 t ) ( for 0  t  t )
k
B and b will be determined by the continuity of x and x at t  t. Thus,
F F
(1  cos v0 t )  B cos b and v0 sin v0 t   v0 B sin b
k k
2
Thus, ¥F´
B 2  ¦ µ (2  2 cos v0 t ) x(t)
§k¶

F vt t t
or B2 sin 0
k 2

4.85 For the spring, mg  k$l , where k is its stiffness coefficient. Thus,
k g
v02  
m $l
The equation of motion of the ball is
F0
x  2bx  v02 x  cos vt
m
2pb b l2 p
Here, l or 
v02  b2 v 1  ( l2 p )2
To find the solution of the above equation we look for the solution of the auxiliary
equation
F0 i vt
z  2 b z  v02 z  e
m
Clearly we can take Re z  x. Now we look for a particular integral for z of the form
z  Ae i vt
Thus, substitution gives A and we get
( F0 m ) e i vt
z 
v02  v2  2i bv
So, taking the real part
F0 [( v02  v2 ) cos vt  2b v sin vt ]
x
m ( v02  v2 )2  4 b2 v2
4.1 MECHANICAL OSCILLATIONS 55

F0 cos ( vt  w ) 2 bv
or x and w  tan1
m (v  v )  4 b v
2
0
2 2 2 2 v02  v2
The amplitude of this oscillation is maximum when the denominator is minimum.
This happens when
v4  2v02 v2  4 b2 v2  v04  ( v2  v02  2b2 )  4 b2 v02  4 b4
is minimum, i.e., for v2  v02  2 b2

2  v2 ¥ 1 
2 b2 ´
Thus, vres 0 ¦
§ v02 µ¶

g ¨ 2( l2p )2 · g 1  ( l2p )2
 ©1  
$l ª 1  ( l2p )2 ¸¹ $l 1  ( l2p )2
F0 m F0 m F0 m l
and ares    –
4b v 4b 2 2
0
4 2b v  b 2
0
2 2b2 2 p
F0 1  ( l2p )2 F0 $l l ¥ 4 p2 ´
 .  ¦ 1  µ
2 m v02 l2p 4p mg § l2 ¶

F0 m
4.86 Since a
( v  v  2b2 )2  4 b2 ( v02  b2 )
2 2
0

we must have v12  v02  2b2  ( v22  v02  2b2 )


v12  v22
or v02  2 b2   vres
2
2
v12  v22
Therefore, vres 
2
 5.1  102 s1

F0 ( v02  v2 ) cos vt  2 b v sin vt


4.87 We have x
m ( v02  v2 )2  4 b2 v2
F0 v 2 b v cos vt  ( v2  v02 ) sin vt
Then, x 
m ( v02  v2 )2  4 b2 v2
Thus, the velocity amplitude is
F0 v
v0 
m ( v02  v2 )2  4 b2 v2
56 PART FOUR OSCILLATIONS AND WAVES

F0

2 2
¥v 0 ´
m ¦  vµ  4 b2
§ v ¶

This is maximum when v2  v02  vres


2

F0
and then v0 res 
2m b
2
Now at half maximum ¥ v02 ´
¦§  vµ  12 b2
v ¶

or v2  2 3 bv  v02  0

or v  b 3  v02  3 b2
where we have rejected a solution with negative sign before.

Using v1  v02  3 b2  b 3 and v2  v02  3 b2  b 3


we get,
(a) Velocity resonance frequency is vres  v0 v1v2

| v1  v2 |
(b) Damping coefficient is b
2 3
and damped oscillation frequency
( v1  v2 )2
v02  b2  v1 v2 
12

4.88 In general for displacement amplitude


F0 1
a
m ( v  v )  4 b2 v2
2
0
2 2

F0 1

m ( v  v  2 b )  4 b2 ( v02  b2 )
2 2
0
2 2

ares v02 v02


Thus, h  
alow 4 b2 ( v02  b2 ) 2 b v02  b2

b l2p l b
But  , 
v0 1  ( l2p ) 2p
2 v0  b2
2
4.1 MECHANICAL OSCILLATIONS 57

v02 l 1 1  ( l2p )2
Hence, h –   2.1
2 b2 2p 2 l2p

4.89 The work done in one cycle is


T T
A  ° F dx  ° Fv dt  ° F0 cos vt ( va sin ( vt  w )) dt
0 0

T
 ° F0 va ( cos vt sin vt cos w  cos2 vt sin w ) dt
0

1 T
 F0 va sin w  paF0 sin w
2 2

4.90 In the formula x  a cos ( vt  w )


F0 1
we have a
m ( v  v )  4 b2 v2
2
0
2 2

2b v
tan w 
v  v2
2
0

Thus, ( v02  v2 ) tan w


b
2v
k
Hence, v0   20 s1
m
(a) The quality factor

p v02  b2 1 4 v2 v02
Q    1  2. 2
bT 2b 2 ( v02  v2 )2 tan 2 w
(b) Work done is
A  paF0 sin w

 p ma2 ( v02  v2 )2  4 b2 v2 sin w  pma2  2 bv


 p ma 2 ( v02  v2 ) tan w  6 mJ

4.91 Here as usual,


2bv
tan w  (where w is the phase lag of the displacement)
v02  v2
x  a cos (vt  w)
58 PART FOUR OSCILLATIONS AND WAVES

F0 1
and a
m ( v02  v2 )2  4 b2 v2
(a) Mean power developed by the force over one oscillation period
pF0 a sin w 1
<P >  F0 av sin w
T 2
F02 bv2 F02 b 1
 
m ( v0  v )  4 b v
2 2 2 2 2 m ( v0 v  v)2  4 b2
2

(b) Mean power < P > is maximum when v  v0 (because the denominator is then
minimum). So,
F2
 P  max  0
4 mb

4.92 Given b  v0 h. Then from the previous problem


F02 v0 1
 P  –
hm ( v02 v  v)2  4 v02 h2

At displacement resonance v  v02  2 b2


F02 v0 1 F02 v0 1
 P  res  
hm 4 b4 v02  2 b2 4 v02 h2 hm 4 v h
4
0
4 v02
 4
v02 (1  2h2 ) h2
F02 h2 F02 h h2  2
 
4 h m v0 (1h2  2) 1 4 m v0 h2  1

while F02 h
 P  max 
4m v0
 P  max   P  res 100
Thus,  2 %
 P  max h 1

4.93 The equation of the disk is


N m cos vt
  2 bw  v02 w 
w
I
Then, as before w  wm cos ( vt  a)
Nm 2bv
where wm  , tan a 
I [( v  v )  4 b2 v2 ]1/2
2
0
2 2 v02  v2
4.2 ELECTRIC OSCILLATIONS 59

(a) Work performed by frictional forces is


T
 ° N r dw ( where N r   2I bw   ° 2 bI w 2 dt   2pb vI wm2 )
0

  pI wm2 ;( v02  v2 )2  4 b2 v 2 12


= sin a   pN m wm sin a
(b) The quality factor

p p v02  b2 v v02  b2
Q    2
l bT 2b ( v0  v2 ) tan a
12
1 « 4 v2 v02 4 b2 v2 º
 ¬ 2  2 »
2 tan a ­ ( v0  v )
2 2 ( v0  v2 )2 ¼
12
1 « 4 v2 v02 I 2 wm2 º Nm
 ¬  tan 2 a» (since v02  v2  cos a)
2 tan a ­ N m2 cos2 a ¼ I wm
1 2
1 « 4 v2 v02 I 2 wm2 º
 ¬ 2
 sin 2 a»
2 sin a ­ Nm ¼
1 2
1 « 4 v2 I 2 wm2 ¥ 2 N m cos a ´ º
 ¬ ¦§ v  I w µ¶  1  cos a»
2
2 sin a ­ N m2 m ¼
1 2
1 « 4 I 2 wm2 4 4 I wm 2 º
 ¬ v  v cos a  cos2 a  1»
2 sin a ­ N m2 Nm ¼
2 1 2
1 «®¥ 2I wm v2 ´ º®
 ¬¦  cos aµ  1»
2 sin a ®­§ N m ¶ ®¼

4.2 Electric Oscillations

4.94 If the electrons (charge of each electron e) are shifted by a small distance x, a
net ve charge density (per unit area) is induced on the surface. This will result in an
electric field E  n ex d0 in the direction of x and a restoring force on an electron of
magnitude ne 2 x d0 .
ne 2 x
Thus, mx  
d0
ne 2
or x  x 0
md0
60 PART FOUR OSCILLATIONS AND WAVES

ne 2
This gives vp 
md0
 1.645  1016 s1
as the plasma frequency for the problem.

4.95 Since there are no sources of emf in the circuit,


q dI
L
C dt
where q  charge on the capacitor, I  dq dt current through the coil.
d 2q 1
Then,  v02 q  0, v02 
dt 2 LC
The solution of this equation is
q  qm cos ( v0 t  a)
qm
From the problem Vm 
C
Then, I   v0CVm sin ( v0 t  a)
and V  Vm cos ( v0 t  a)
I2
V2   Vm2
v02 C 2
LI 2
or V2   Vm2
C
By energy conservation
1 2 q2
LI   constant
2 2C
When the potential difference across the capacitor takes its maximum value Vm, the
current I must be zero.
1 2
Thus, CVm  constant
2
LI 2 2
Hence, once again  V 2  Vm
C

4.96 After the switch is closed, the circuit satisfies


dI q
L 
dt C
4.2 ELECTRIC OSCILLATIONS 61

d 2q
or  v02 q  0  q  CVm cos v0 t
dt 2
where we have used the fact that when the switch is closed, we must have
q dq
V  Vm , I 0 (at t  0)
C dt
dq
(a) Thus, I CVm v0 sin v0t
dt
C
Vm sin v0t
L
(b) The electrical energy of the capacitor is
q2
cos2 v0 t
2C
and of the inductor is
1 2
LI sin 2 v0 t
2
The two are equal when
p
v0 t 
4
At that instant, the emf of the self-inductance is
dI V
L  Vm cos v0t  m
dt 2

4.97 The required work can be represented as an increment of the energy of the circuit
qm2 ¥ 1 1´ ¥C ´
A W a W  ¦  µ  W ¦§  1µ¶
2 §Ca C ¶ Ca

On the other hand, v0 ^ 1 C and hence, h  v0a v0  C C a and consequently


A  W (h2  1) .
Alternate:
The required work can be represented as an increment of the energy of the circuit
1 1 1
A W a W  L (h2 v02 qm2 )  L ( v02 qm2 )  L v02 qm2 (h2  1)
2 2 2
taking into account I m  v0 qm .
62 PART FOUR OSCILLATIONS AND WAVES

4.98 Capacitors C1 and C2 are in par- Sw


allel connection whose equiva-
˚
lent capacitance C 0  C1  C 2 . C1 C2 ⇔ L C0 C1  C2
Equation of circuit becomes L

dI q
L 
dt C 0

¥ 1 ´
or q  v02 q  0 ¦§ where v0  LC µ¶
2

Its general solution is


q  qm cos ( v0 t  a)
At t  0 charge is maximum, i.e.,
q  qm  C 0V  (C1  C 2 ) V
Using the initial condition, i.e., a  0
q  qm cos v0 t
So, the frequency of oscillations
1 1
v0  
L C0 L (C1  C 2 )
2p
and period of oscillations T   2p L (C1  C 2 )  0.7 ms
v0
Differentiating the equation q  qm cos v0t with respect to time, we get
I  v0 qm sin v0 t
Thus, the peak value of current
1 C1  C 2
I m  v0 qm  (C1  C 2 ) V  V  8.05 A
L (C1  C 2 ) L

4.99 Assume that after closing the switch at t  0, the right-hand side capacitor acquired
charge q at t  t, equation of circuit will be I
dI q (CV0  q ) L
L   (CV0q) q
dt C C C [ C
(CV0q) q
or d 2q 2q V0 (1)
2
  Sw
dt LC L
It is a form of differential equation
4.2 ELECTRIC OSCILLATIONS 63

x  v02 x  A ( where v0 and A are constant )


General solution of such differential equation is
A
x  a cos ( v0 t  a) 
v2
Taking this into account, the general solution of Eq. (1) is
CV0
q  qm cos ( v0 t  a)  (2)
2
So, I   v0 qm sin ( v0 t  a) (3)
Using the initial condition I  0 and q  0 at t  0, in Eqs. (3) and (2), we get
CV0
a0 and qm 
2
Now Eq. (2) has the form
CV0
q [1  cos vt ]
2
So, voltage across the right-hand side capacitor
q V0
V  [1  cos vt ]
C 2
Thus, the charge on left-hand side capacitor
CV
CV0  q  0 [1  cos vt ]
2
and voltage across it is
V0
[1  cos vt ]
2
Alternate:
Initially q1  CV0 and q2  0. After the switch is closed, charge flows and we get
q1  q2  CV0
q1 dI q (1)
L  2 0
C dt C
Also I  q1   q2 I
2I
Thus, LI  0 q1 q2
C C C
q1 q2
2
Hence, I  v02 I  0, v02  Sw
LC 
64 PART FOUR OSCILLATIONS AND WAVES

The solution of this equation is subject to I  0 at t  0.


So, I  I 0 sin v0t
I0
Integrating, we get q1  A  cos v0t
v0
I0
and q2  B  cos v0t
v0
Finally substituting in Eq. (1)
A B 2I
 0 cos v0t  LI 0 v0 cos v0t  0
C v0C
CV0
Thus, A B
2
CV0 I
and  0 0
2 v0
CV0 q1 V0
So, q1  (1  cos v0 t ) and V1   (1  cos v0 t )
2 C 2
CV0 q2 V0
q2  (1  cos v0 t ) and V2   (1  cos v0 t )
2 C 2

4.100 The flux in the coil is

& (t )  [ & ( for t  0)


0 ( for t  0)
Due to sudden switching off of the external magnetic field at the moment t  0, an
induced current appears, but the capacitor still remains uncharged. In accordance
with Ohm’s law, we have
d& dI
 L 0
dt dt
Hence, &  LI  0
This gives &  LI 0 , where I0 is the initial current (immediately after switching off the
field).
After switching off external magnetic field, the equation for LC circuit becomes
q dI
 L 0 (1)
C dt
Differentiation of this equation with respect to time gives
4.2 ELECTRIC OSCILLATIONS 65

d2I I
 0
dt 2 LC
Its general solution will be
¥ 1 ´
¦§ where v0  L C µ¶
2
I  I m cos ( v0 t  a)

Since at initial instant t  0, the capacitor was uncharged, so I m  I 0 (initial current)


and a  0. As a result
I  I 0 cos v0 t

& & ¥ t ´
Finally, I  I 0 cos v0 t  cos v0 t  cos ¦
L L § LC µ¶

4.101 Given V  Vm ebt cos vt


(a) The phrase ‘peak values’ is not clear. The answer is obtained on taking
|cos vt | 1
pn
i.e., t
v
(b) For extremum dV  0
dt
b
 b cos vt  v sin vt  0 or tan vt 
v
¥ b ´
i.e., vtn  np  tan1 ¦
§ v µ¶

1 ¨ 1 ¥
b ´ ·
Hence, tn  ©np  tan ¦§ v µ¶ ¸
v ª ¹

4.102 The equation of the circuit is


d 2Q dQ Q
2
LR  0
dt dt C
where Q  charge on the capacitor.
This has the solution
Q  Qm ebt sin ( vt  a)
R 1
where b , v  v02  b2 , v02 
2L LC
66 PART FOUR OSCILLATIONS AND WAVES

dQ
Now, at t  0 I 0
dt
so, Qm ebt [ b sin ( vt  a)  v cos ( vt  a)]  0
v
Thus, v cos a  b sin a or a  tan1
b
Qm Qm
Now, Vm  and V0  sin a (where V0 is P.D. at t  0)
C C
V0 v v b2 R 2C
Therefore  sin a    1 2  1
Vm v2  b2 v0 v0 4L

4.103 We write dQ
  I  I m ebt sin vt
dt
 gm I m ebt  i vt (where gm means imaginary part)

ebt  i vt
Then, Q  gm I m
 b  iv
ebt  i vt
 gm I m
b  iv
( b  i v) ebt  i vt
 gm I m
b2  v2
b sin vt  v cos vt
 I m ebt
b2  v2
sin ( vt  d) v
 I m ebt , tan d 
b2  v2 b
(An arbitrary constant of integration is assumed to be equal to zero.)

Q L bt
Thus, V  Im e sin ( vt  d)
C C

L L v
V (0)  I m sin d  I m
C C v2  b2

L
 Im
C (1  b2 v2 )
4.2 ELECTRIC OSCILLATIONS 67

4.104 We know that, I  I m ebt sin vt


R 1
b , v0  , v  v02  b2
2L LC
I   q (where q  charge on the capacitor)
sin ( vt  d) v
Then, q  I m ebt and tan d 
v2  b2 b
1 2  2 bt
Thus, Wmag  LI m e sin 2 vt
2
I m2 e 2 bt sin 2 ( vt  d) LI m2  2 bt
Wele   e sin 2 ( vt  d)
2C v2  b2 2
Current is maximum when
d  bt
e sin vt  0
dt
Thus,  b sin vt  v cos vt  0
v
or tan vt   tan d
b
i.e., vt  np  d
Wmag sin 2 ( vt ) sin 2 d 1
Hence,   
Wele sin ( vt  d) sin 2d 4 cos2 d
2 2

1 v02 1 L2 L
     5
4 b v0
2 2 4b 2 LC R 2 CR 2
(Wmag is the magnetic energy of the inductance coil and Wele is the electric energy of
the capacitor.)

4.105 Clearly L  L1  L2 , R  R1  R2

p p
4.106 We know, Q or b
bT QT
Now, bt  ln h
ln h
so, t QT
p
Q ln h
  0.5 s
pn
68 PART FOUR OSCILLATIONS AND WAVES

4.107 Current decreases e-fold in time


1 2L 2L
t  s oscillations
b R RT
2L v

R 2p

L 1 R2 1 4L
  2   1  16 oscillations
pR LC 4L 2p R 2C

p v
4.108 Since, Q 
bT 2b
v
Therefore, v  2 bQ or b
2Q
1 v0
Now, v0  v 1  or v
4Q 2 1  1( 4Q 2 )
v0  v 1
So,  100%   100%  0.5%
v0 8Q 2

d R
4.109 At t  0, current through the coil  Sw
R r
d
P.D. across the condenser 
R r d, r C L
(a) At t  0, energy stored
2 2
1 ¥ d ´ 1 ¥ dR ´ 1 ( L  CR 2 )
W0  L¦ µ  C¦ µ  d2  2.0 mJ
2 § R r¶ 2 § R r¶ 2 ( R  r )2
(b) The current and the charge stored decrease as etR/2 L so energy decreases as etR/L .
Therefore, W  W0 etR/L  0.10 mJ

p pn v v02  b2
4.110 Since Q   
bT b 2b 2b
v0 v0
So,  1  4Q 2 or b
b 1  Q2
Now, W  W0 e2bt
4.2 ELECTRIC OSCILLATIONS 69

Thus, energy decreases h times in time


ln h
2b
1  4Q 2 Q ln h
 ln h  s  1.033 ms
2v0 2pv0

4.111 In a leaky condenser


dq ¥ V ´
 I  Ia ¦§ where I a   leak curent µ¶
dt R
q dI d ¥ dq V ´
Now V L L ¦§  µ
C dt dt dt R¶
d 2q L dq
L 2

dt RC dt
1 dq 1
or q   q 0
RC dt LC
Then q  qm ebt sin ( vt  a)
(a) For damped oscillations,
1 1
b , v02  , v  v02  b2
2RC LC
1 1
Therefore, v 
LC 4 R 2C 2
(b) The quality factor
v 1 1
Q  RC 
2b LC 4 R2 C 2

1 4CR 2
 1
2 L

4.112 Given V  Vm ebt sin vt , v  v0 bT  1


Energy loss per cycle
Power loss 
T
1
 CVm2  2 b
2
70 PART FOUR OSCILLATIONS AND WAVES

(Energy decreases as W0 ebt so loss per cycle is W0  2 bT .)


1 R
Thus,  P  CVm2 
2 L
2P  L
or R
Vm2 C

1 L C Vm2
Hence, Q   100 (on substituting values)
R C L 2P 

4.113 Energy is lost across the resistance and the mean power loss is
1 2
 P   R  I 2  RI m  20 mW
2
This power should be fed to the circuit to maintain undamped oscillations.

RCVm2
4.114 We have  P  (as in Problem 4.112)
2L
On substituting values, we get
 P   5 mW

4.115 Given q  q1  q2
I1 I1  I2
I 1   q1 , I 2   q2
q
LI 1  RI 2  L
C C
Thus, CL q1  ( q1  q2 )  0 R
RC q2  q1  q2  0
Putting q1  A e i vt and q2  Bei vt , we have
(1  v2 LC ) A  B  0
A  (1  i vRC ) B  0
A solution exists only if
(1  v2 LC ) (1  i vRC )  1
or i vRC  v2 LC  i v3 LRC 2  0
or LRC 2 v2  i vLC  RC  0
4.2 ELECTRIC OSCILLATIONS 71

1 1
and v2  i v  0
RC LC
i 1 1
So, v    i b  v0
2RC LC 4 R 2C 2
Thus, q1  ( A1 cos v0 t  A2 sin v0 t ) e bt
v0 is the oscillation frequency. Oscillations are possible only if v02  0,
1 C
i.e., 2

4R L

4.116 We have
L1 I1  R1 I 1  L2 I2  R2 I 2 I1 I2


° I dt R1, L1
I
R2, L2
C C
I  I1  I 2
On differentiating we get the equations
L1CI1  R1CI1  ( I 1  I 2 )  0

L2CI2  R2CI2  ( I 1  I 2 )  0
On solving, we get I 1  A1 e at , I 2  A2 e at
Then, (1  a2 L1C  aR1C ) A1  A2  0
A1  (1  a2 L2C  aR2C ) A2  0
This set of simultaneous equations has a non-trivial solution only if
(1  a2 L1C  aR1C ) (1  a2 L2C  aR2C )  1
L1 R2  L2 R1 L  L2  R1 R2C R  R2
or a3  a2 a 1  1 0
L1 L2 L1 L2C L1 L2C
This cubic equation has one real root which we ignore and two complex conjugate
roots. We require the condition that this pair of complex conjugate roots is identical
with the roots of the equation
a2 LC  aRC  1  0
The general solution of this problem is not easy. So, we look for special cases.
If R1  R2  0, then R  0 and L  L1 L2 ( L1  L2 ).
If L1  L2  0, then L  0 and R  R1 R2 ( R1  R2 ).
72 PART FOUR OSCILLATIONS AND WAVES

These are the quoted solutions but they are misleading.


We shall give the solution for small R1, R2. Then we put a   b  i v , to get
(1  v2 L1C  2 i bvL1C  bR1C  i vR1C )(1  v2 L2C  2 i bvL2 C  bR2C  i vR2C )  1
when b is small, neglecting b2 and bR1 , bR2 ,we get
L1  L2
(1  v2 L1C ) (1  v2 L2C )  1 or v2 
L1 L2C
1 L1 L2
This is identical with v2  if L 
LC L1  L2
Also (2bL1  R1 )(1  v2 L2C )  (2 bL2  R2 ) (1  v2 L2C )  0
R R L2  R2 L21 R1 L22  R2 L21
This gives b  1 2  R
2 L 2 L1 L2 ( L1  L2 ) ( L1  L2 )2

4.117 We have
q dI dq
0 L  RI , I 
C dt dt
C L R
L
For the critical case, R 2
C
Thus, LCq  2 LC q  q  0

Let us look for a solution with q ^ ebt . Sw


 
1
We know that, b
LC
An independent solution is t ebt .
Thus, q  ( A  Bt ) et LC

At t  0, q  CV0 , thus A  CV0 . Also at t  0, q  I  0 .

1 C
So, 0 B  A  B  V0
LC L
dq C t
Finally, I  V0 e LC
dt L

1 ¥ C ´ t
 ¦§ CV0  V0 tµ e LC
LC L ¶
4.2 ELECTRIC OSCILLATIONS 73

V0 t
 te LC
L
The current has been defined to increase the charge. Hence, the minus sign.
The current is maximum when
dI V ¥ t ´
  0 et LC
¦§ 1  µ 0
dt L LC ¶

This gives t  LC and the magnitude of the maximum current is


V0 C
| I max |
e L

4.118 The equation of the circuit is


dI
L  RI  Vm cos vt
dt
From the theory of differential equations
I  I P  IC
where IP is a particular integral and IC is the complementary function (solution of the
differential equation with the R.H.S  0).
Now, I C  I coil etR/L
and for IP we write I P  I m cos ( vt  w )
Vm vL
Substituting, we get Im  and w  tan1
R v L
2 2 2 R
Vm
Thus, Im  cos ( vt  w )  I coil etRL
R 2  v2 L2
Now, in an inductive circuit I  0 at t  0 because a current cannot change suddenly.
Vm
Thus, I coil   cos w
R  v2 L2
2

Vm
and so, I ;cos ( vt  w )  cos wetRL =
R  v2 L2
2

4.119 Here, the equation is


Q dQ
R  Vm cos vt (where Q is charge on the capacitor) (1)
C dt
74 PART FOUR OSCILLATIONS AND WAVES

A solution subjected to Q  0 at t  0 is of the form (as in the previous problem)

Q  Qm [cos ( vt  w )  cos wet /RC ]


Substituting in Eq.(1), we get
Qm
cos ( vt  w )  vRQm sin ( vt  w )
C
 Vm cos vt

 Vm [cos w cos ( vt  w )  sin w sin ( vt  w )]


So, Qm  CVm cos w
vR Qm  Vm sin w
This leads to
CVm
Qm  , tan w  vRC
1  ( vRC )2
dQ Vm ¨ cos2 w tRC ·
Hence, I   sin ( v t  w )  e
dt R 2  (1vC )2 ©ª sin w ¸
¹
The solution given in the book satisfies I  0 at t  0. When Q  0 at t  0, this
will not satisfy the equation at t  0. Thus, I p 0. (Equation will be satisfied with
I  0 only if Q p 0 at t  0.)
With the I obtained above
Vm
I (t  0) 
R

4.120 The current lags behind the voltage by the phase angle
vL
w  tan1
R
Now, L  m0n2pa 2l ( where l is length of the solenoid)
r – 2pan – l
R ( where 2b is diameter of the wire )
pb 2
1
But 2bn  1 so, b
2n
m0n2l pa2 – 2pn 1
Then, w  tan1 p
r – 2panl 4 n2
4.2 ELECTRIC OSCILLATIONS 75

m0 p 2 an
 tan1
4 rn

4.121 Here V  Vm cos vt


I  I m cos ( vt  w )
Vm 1
where Im  , tan w 
R  (1vC )
2 2 vRC
2
1 ¥V ´
Now R2  2
¦ mµ
( vC ) § Im ¶
2
1 ¥ Vm ´
 ¦§ RI µ¶  1
vRC m

Thus, the current is ahead of the voltage by


2
1 ¥ Vm ´
w  tan1  tan1 ¦§ RI µ¶  1  60o
vRC m

4.122 (a) Here


t

° Idt
0
R
V IR
C
1 V
R I  I  V   vV0 sin vt C V'
or
C
Ignoring transients, a solution has the form
I  I 0 sin ( vt  a)
I0
v R I 0 cos ( vt  a)  sin ( vt  a)   vV0 sin vt
C
  vV0 [ sin ( vt  a) cos a  cos ( vt  a) sin a ]
So, RI 0  V0 sin a
I0
 V0 cos a, a  tan1 ( vRC )
vC
V0
I0 
R  (1vC )2
2
76 PART FOUR OSCILLATIONS AND WAVES

I  I 0 sin ( vt  tan1 vRC  p )   I 0 sin ( vt  tan1 vRC )


t
I0
Then, Q  ° I dt  Q0  cos ( vt  tan1 vRC )
0
v

dQ Q
It satisfies V0 1  cos vt  R 
dt C
if V0 (1  cos vt )   RI 0 sin ( vt  tan1 vRC )
Q0 I
  0 cos ( vt  tan1 vRC )
C vC
Thus, Q0  CV0
I0 V0
and 
vC 1  ( vRC )2
V0 vRC
RI 0 
1  ( vRC )2
Q V0
Hence, Va  V0  cos ( vt  a)
C 1  ( vRC )2

(b) V0 V0

h 1  ( vRC )2
or h2  1  v2 ( RC )2
h2  1
or RC   22 ms
v

4.123 For Fig. (a) of the problem, the voltage vector diagram is

VR VLR Voltage

VL

w
VRO Current

For Fig. (b) of the problem, we have


vL  1vC ¥ 1 ´
tan w    ve ¦§ as v2  µ
R LC ¶
4.2 ELECTRIC OSCILLATIONS 77

The voltage vector diagram is


VR
VLR
VL
w Current
Voltage V

Vm
4.124 (a) Im   4.48 A
R  ( vL  1/vC )2
2

vL  1vC
(b) tan w  , w   60o
R
Current lags behind the voltage V by w.
Im
(c) VC   0.65 kV
vC
VL  I m R 2  v2 L 2  0.5 kV

1 Vm
4.125 (a) VC 
vC R 2  ( vL  1/vC )2
Vm Vm
 
( vRC )  ( v LC  1)
2 2 2 ( v v  1)2  4 b2 v2 v04
2 2
0

Vm

( v v  1  2 b v02 )2  4 b2 v02  4 b4 v04
2 2
0
2

This is maximum when


1 R2
v2  v02  2 b2   2
LC 2L
vL
(b) VL  I m vL  Vm
R  ( vL  1vC )2
2

Vm L Vm L
 
R v  ( L  1v C )
2 2 2 2 L  1v (2LC  R 2 )  1v4C 2
2 2

Vm L

(1v C  ( L  CR 2))2  L2  ( L  12CR 2 )2
2 2
78 PART FOUR OSCILLATIONS AND WAVES

This is maximum when


1 1
2
 L  CR 2
vC 2
1 1
or v2  
LC  12C R
2 2 1v0  2b2 v04
2

v04 v02
 or v
v02  2 b2 v02  2 b2

4.126 Vm R 2  v2 L2
VL  I m R 2  v2 L2 
R 2  ( vL  1vC )2
For a given v, L, R, voltage amplitude is maximum when
1
 vL
vC
1
or C  28.2 MF
v2 L
For calculated value of C,
2
V R 2  v2 L2 ¥ vL ´
VL  V 1  ¦ µ  0.540 kV
R § R¶

1 Vm Vm vL
Also, VC    0.509 kV
vC R R

4.127 We use the complex voltage V  Vm e i vt . Then the voltage across the capacitor is
1 II'
( I  I a)
i vC I

and that across the resistance is RI  and both equal V.


Poor
conductor

I
V
Thus, I a  m e i vt and I  I a  i vCVm e i vt R
R
Vm
Hence, I (1  i vRC )e i vt
R
The actual voltage is obtained by taking the real part.
V
Then, I  m 1  ( vRC )2 cos ( vt  w )
R
4.2 ELECTRIC OSCILLATIONS 79

where tan w  vRC .


Note: A condenser with poorly conducting material (dielectric of high resistance)
between the plates is equivalent to an ideal condenser with a high resistance joined in
parallel between its plates.

dI 1 ° I 1 dt dI I
4.128 L1    L 12 2
dt C dt I0
dI 2 dI C L1 L2
L2   L12 1
dt dt L12
From the second equation
L 2 I 2   L 12 I 1

¥ L 212 ´  I1
Then, ¦§ L1  L µ¶ I 1  C  0
2

Thus, the current oscillates with frequency


1
v
C ( L 1  L 212 L 2)

4.129 Given V  Vm cos vt


L, R
I  I m cos ( vt  w )
I
Vm
Here Im  V
R 2  ( vL  1vC )2

Then, VC 
° I dt  I m sin ( vt  w )
C vC
Vm
 sin ( vt  w )
(1  v LC )2  ( vRC )2
2

At resonance, the voltage amplitude across the capacitor is


Vm L
 Vm  nVm
RC 1 LC CR 2
L
So,  n2
CR 2
L 1 1
Now, Q   n2 
CR 2 4 4
80 PART FOUR OSCILLATIONS AND WAVES

4.130 For maximum current amplitude


Vm
Im 
R  ( vL  1vC )2
2

1 Vm
L and then Im0 
v2C R
Im0 Vm
Now, 
h (n  1)2
R2 
v2C 2
(n  1)2
So, h  1
( vRC )2
n 1
or vRC 
h2  1
2 2
¥ L ´ 1 ¥ 1 ´ 1 h2  1 1
Now, Q  ¦ 2µ   ¦ µ   
§ CR ¶ 4 § vRC ¶ 4 (n  1)2 4

1
4.131 At resonance v0 L  ( v0C )1 or v0 
LC
Vm
and ( I m )res 
R
Vm Vm Vm
Now,  
nR ¥ 1 ´
2
¥ 1 ´
2

R 2  ¦ v1 L  R 2  ¦ v2 L 
§ v1C µ¶ § v2C µ¶
1
Then, v1 L   n2  1 R
v1C
1
v2 L    n2  1 R (assuming v2  v1 )
v2C
v02 v2 R
or v1    v2  0   n2  1
v1 v2 L
v02
or v1  v2  ( v1  v2 )  v0  v1 v2
v1 v2
R
and v2  v1  n2  1
L
4.2 ELECTRIC OSCILLATIONS 81

R v  v1
b  2
2L 2 n2  1

v02 1 (n2  1) v1v2 1


and Q   
4 b2 4 ( v2  v1 )2 4

4.132 The quality factor


v v
Q  0 (for low damping)
2b 2b
Im RI m
Now 
2 R 2  ( vL  1vC )2
(where Im  current amplitude at resonance)
v2 R
or v  0    2 b
v L
Thus, v  v0  b
v0
So, $v  2 b and Q
$v

4.133 At resonance v  v0
Vm
I m ( v0 ) 
R
Vm
Then, I m (h v0 ) 
R 2  (hv0 L  1hv0C )2
Vm

R  (h  1h)2 L/C
2

I m ( v0 ) ¥ l ´ (h2  1)2
 1  ¦ Q2  µ
I m (hv0 ) § 4¶ h2
 2.2 (for Q  10)
and  1.9 (for Q  10)

4.134 The A.C. current must be


I  I 0 2 sin vt
Then D.C. component of the rectified current is
T /2
1
 I a  °I 0 2 sin vt dt
T 0
82 PART FOUR OSCILLATIONS AND WAVES

p
1
 I0 2
2p ° sin u d u
0

I0 2

p
Since the charge deposited must be the same
I0 2 pt 0
I 0 t0  t or t
p 2
(The answer is incorrect in the answer sheet.)

t
4.135 (a) In this case I (t )  I 1 ( for 0  t  T )
T
I (t  T )  I (t )
Now, mean current
T
1 t T 2 2 I
 I  °I 1 dt  I 1 2  1
T 0
T T 2
Then, I 1  2I 0 sin  I   I 0
Now, mean square current
T
1 t2 4 I 02
 I 2   4 I 02
T °T
0
2
dt 
3
2I 0
So, effective current  1.51I 0
3
(b) In this case I  I 1 |sin vt |
T
1
and I0 
T °I
0
1 |sin vt | dt

2p p
1 I1 2I1

2p
I1 ° |sin u | d u 
0
p ° sin u d u 
0
p
pI
So, I1  0
2
Then, mean square current
T
p 2 I 02
 I2  
4T ° sin
0
2 vt dt
4.2 ELECTRIC OSCILLATIONS 83

2p
p 2 I 02 1 p 2 I 02

4

2p °
0
sin 2 u d u 
8

So, effective current  pI 0  1.11I 0


8

V02
4.136 We have PD.C. 
R
V02 R
and PA.C.  .
R v L
2 2 2 R  v2 L 2
2

V02 R P
  D.C.
1  ( vLR ) 2 h
vL
Thus,  h 1
R
R
or v h 1
L
R
n h  1  2 kHz (on substituting values)
2pL

4.137 We have Z  R 2  X L2 or R  Z 2  X L2

XL
Then, tan w 
Z  X L2
2

2
Z 2  X L2 ¥X ´
So, cos w   1 ¦ L µ
Z § Z ¶
2
¥X ´
Therefore, w  cos1 1  ¦ L µ  37o
§ Z ¶

The current lags by w behind the voltage.


V2
Also P  VI cos w  Z 2  X L2  0.16 kW
Z2

V 2 (R  r )
4.138 We have P
( R  r )2  v 2 L 2
84 PART FOUR OSCILLATIONS AND WAVES

V2 V2
P 2
 2
¥ ( vL ) ´ ¨ vL ·
R r  ¦
§ R  r µ¶ © R  r  R  r ¸  2v L
ª ¹
This is maximum when R  r  vL or R  vL  r and thus, Pmax  V 2 2vL .
On substituting values, we get R  200 6 and Pmax  0.114 kW.

V 2R
4.139 We have P
R 2  ( X L  X C )2
Varying the capacitor does not change R, so if P increases n times
Z  R 2  ( X L  X C )2

must decrease n times.


Thus, cos w  RZ increases n times.
Therefore, percentage increase in cos w  ( n  1)  100 %  30.4 %

V 2R
4.140 We have P
R  ( X L  X C )2
2

1
At resonance X L  X C  v0 
LC
Power generated will decrease n times when
2
¥ 1 ´
( X L  X C )2  ¦ vL  µ  (n  1) R
2
§ vC ¶
v02 R
or v   n  1   n  1 2b
v L
Thus, v2  2 2 n  1 bv  v02  0

( v  n  1 b)2  v02  (n  1) b2

v b2 b
or  1  (n  1) 2  n  1
v0 v0 v0
(taking only the positive sign in the first term to ensure positive value for v/v0 ).
Now, quality factor
2
v 1 ¥ v0 ´
Q  ¦§ b µ¶  1
2b 2
4.2 ELECTRIC OSCILLATIONS 85

v0
 1  4Q 2
b

v n 1 n 1
Thus,  1 
v0 (1  4Q )
2
1  4Q 2
For large Q
v  v0 n 1
  100  0.5 %
v0 2Q

VR V R12  X L2
4.141 We have V1  and V2 
( R  R1 )2  X L2 ( R  R1 )2  X L2
2 2
2 ¥ VR ´ ¥V R´
So, ( R  R1 )  X L2  ¦ µ and R12  X L2  ¦ 2 µ
§ V1 ¶ § V1 ¶
R2
Hence, R 2  2 RR1  (V 2  V22 )
V12
R
or R1  (V 2  V22  V12 )
2V12
Heat power generated in the coil is
V 2 R1 V12 V2 R2
 2
 R1  12  2 (V 2  V12  V22 )
( R1  R2 )2  X L2 R R 2V1

V 2  V12  V22
  30 W
2R
I1

V R
4.142 Here I2  (V = effective voltage)
R L1R
V
V
I1 
R  X L2
2 I2 I1

V ( R  R1 )2  X L2 V
and I 
R R X2
1
2
L
Reff
Reff is the impedance of the coil and the resistance in parallel.
2
I 2  I 22 R 2  2RR1 ¥ I 1 ´ 2RR
Now,  ¦ µ  2 1 2
2
I2 R1  X L
2 2
§ I2 ¶ R  XL
86 PART FOUR OSCILLATIONS AND WAVES

I 2  I 22  I 12 2 RR
 2 1 2
I2 2 R  XL
The mean power consumed in the coil is
V 2R
I 12 R1  2 1 2  I 22 R
R  XL
I 2  I 12  I 22 1
  R ( I 2  I 12  I 22 )  2.5 W
2 I 22 2

1 1 1 1 1  i vRC
4.143 We have     i vC 
Z R 1i vC R R
R
| Z |  40 6
1  ( vRC )2

4.144 For Fig. (a): The resistance, the voltage and the current are in phase. For the coil, the
voltage is ahead of the current by less than 90°. The current is obtained by addition
because the elements are in parallel.
For Fig. (b): IC is ahead of the voltage by 90°.
For Fig. (c): The coil has no resistance, so IL is 90° behind the voltage.
IR IC IR
Axis of
w voltages w
w I0
I0
I0 IL
ILr ILR
(a) (b) (c)

4.145 When the coil and the condenser are in parallel, the equation is
I1
L
dI 1
 RI 1 
° I 2 dt  V cos vt I2
m
dt C I1
L,R C
And I  I1  I 2 V
I2
Using complex voltages
Vm e i vt
I1  and I 2  i vCVm e i vt
R  i vL

and ¥ 1 ´ ¨ R  i vL  i vC ( R 2  v2 L2 ) ·
I ¦  i vC µ Vm e i vt  © ¸ Vm e
i vt
§ R  i vL ¶ ª R 2  v2 L2
¹
4.2 ELECTRIC OSCILLATIONS 87

Thus, taking real parts


Vm
I cos ( vt  w )
| Z (v)|
1 [ R 2  { v C ( R 2  v2 L2 )  vL }2 ]
when 
| Z (v) | ( R 2  v2 L2 )1/ 2

vL  vC ( R 2  v2 L2 )
and tan w 
R
(a) To get the frequency of resonance, first we must define resonance. One definition
requires rapid change of phase with w passing through zero at resonance. For the
series circuit
Vm vL  (1vC )
Im  and tan w 
{ R 2  ( vL  1vC )2 }1/2 R
Both definitions give v2  1LC at resonance. In the present case, the two defini-
tions do not agree (except when R = 0). The definition that has been adopted in
the answer given in the book is the vanishing of phase. This requires
C ( R 2  v2 L2 )  L
1 R2
or v2   2  vres2 , so v
res  31.6  10 rads
3
LC L
Note that for small R, w rapidly changes from  p2 to  p2 as v passes through
vres from  vres  to vres .
Vm R CR
(b) At resonance Im   Vm
LC L
So effective value of total current
CR
I V  3.1 mA
L
V C
Similarly IL  V  0.98 A
L /C L

C R 2C 2
And I C  v CV  V  2  0.98 A
L L
Note: The vanishing of phase (its passing through zero) is considered a more basic
definition of resonance.

4.146 We use the method of complex voltage


Here, V  V0 e i vt
88 PART FOUR OSCILLATIONS AND WAVES

V0 e i vt ILR
Then, IC   i v C V0 e i vt
1i vC I I
2
C
Ve i vt
and I LR  0 V C L,R
R  i vL
R  i vL  i vC ( R 2  v2 L2 ) i vt
Then I  I C  I LR  V0 e
R 2  v2 L2
Then taking the real part

V0 R 2  { vC ( R 2  v2 L2 )  vL }2
I cos ( vt  w )
R 2  v2 L2
 vL  vC ( R 2  v2 L2 )
where tan w 
R

4.147 From the previous problem


R 2  v2 L2
Z
R 2  { vC ( R 2  v2 L2 )  vL }2
R 2  v2 L2

( R 2  v2 L2 ) (1  2v2 LC )  v2C 2 ( R 2  v2 L2 )2

R 2  v2 L2 R 2  v2 L2
 
(1  2v2 LC )  v2C 2  R 2  v2 L2 (1  v2 LC )2  ( vRC )2

4.148 (a) We have


d&
d   v&0 sin vt  LI  RI
dt
Using I  I m sin ( vt  w )

we get v&0 sin vt  v&0 [sin ( vt  w ) cos w  cos ( vt  w ) sin w]

 LI m v cos ( vt  w )  RI m sin ( vt  w )
RI m  v&0 cos w and LI m  &0 sin w
v&0 vL
or Im  and tan w 
R v L
2 2 2 R
(b) Mean mechanical power required to maintain rotation  energy loss per unit time
4.2 ELECTRIC OSCILLATIONS 89

T
1 1 2 1 v2&02 R

T ° RI 2 dt  2
RI m 
2 R 2  v2 L2
0

4.149 We consider the force F12 that a circuit 1 exerts on another closed circuit 2:
F12  ° I 2 d l 2  B12
Here B12 = magnetic field at the site of the current element d I2 due to the current I1
flowing in 1 and is given by
m0 I 1d I1  r12
4p ° r123
where r12  r2  r1  vector from current element d I1 to the current element d I2.
Now,
m0 d I2  (d I1  r12 ) m0 d I1 (d I2 – r12 )  (d I1 – d I2 )r12
F12 
4p °°I I
1 2
r123

4p °°I I
1 2
r123
In the first term, we carry out the integration over d It first. Then,
d I (d I – r ) dI – r 1
° ° 1 r1232 12  ° d I1 ° 2r123 12   ° d I1 ° d I2 – 2 r12  0
1 ¥ 1´
because ° d I 2 – 2
r12
 ° d S2 curl ¦  µ  0
§ r12 ¶
m0 r12
Thus, F12   °°I I
1 2 d I1 – d I2
4p r123
The integral involved will depend on the vector a that defines the separation of the
(suitably chosen) centre of the coils. Let C1 and C2 be the centres of the two coils
suitably defined.
Then, r12  r2  r1  R2  R1  a
where R1 (R2) is the distance of d l1(d l2) from C1(C2) and a stands for the vector C1C2.
r12 1
Then,   a
3
r12 r12
¨ m d I – d I2 ·
and F12   a © I 1 I 2 0 ° ° 1 ¸
ª 4p r12 ¹
The bracket defines the mutual inductance L12. Thus noting the definition of x
eL
 Fx   12  I 1 I 2 
ex
where < > denotes time average.
Now I 1  I 0 cos vt  Real part of I 0 e i vt
90 PART FOUR OSCILLATIONS AND WAVES

The current in the coil 2 satisfies


dI 2 dI
RI 2  L 2   L12 1
dt dt
 i vL12
or I2  I 0 e i vt (in the complex case)
R  i vL2
Taking the real part, we get
vL12 I 0 vL12
I2  ( vL2 cos vt  R sin vt )   I 0 cos ( vt  w )
R  v L2
2 2 2
R  v2 L2t
2

(where tan w  RvL2 ).


Taking time average, we get
eL12 vL12 I 0 1 v2 L2 L12 I 02 eL12
 Fx   I0 . cos w 
ex R 2  v2 L22 2 2( R 2  v2 L22 ) ex
The repulsive nature of the force is also consistent with Lenz’s law, assuming of
course, that L12 decreases with x.

4.3 Elastic Waves. Acoustics


4.150 Since the temperature varies linearly we can write the temperature as a function of x,
which is the distance from the point A towards B, i.e.,
T2  T1
T  T1  [ for 0  x  l ]
l
¥ T  T1 ´
Hence, dT  ¦ 2 dx (1)
§ l µ¶
In order to travel an elemental distance of dx which is at a distance of x from A, the
time taken will be
dx
dt  (2)
a T
From Eqs. (1) and (2), expressing dx in terms of dT, we get
l ¥ dT ´
dt 
a T ¦§ T2  T1 µ¶
which on integration gives
t T2
l dT
° dt 
0
a (T2  T1 ) °
T1 T
4.3 ELASTIC WAVES. ACOUSTICS 91

2l
or t
a (T2  T1 )
 T2  T1
Hence, the sought time
2l
t
a  T1  T2
4.151 Equation of plane wave is given by j (r , t )  a cos ( vt  k – r ), where k  vv n
called the wave vector and n is the unit vector normal to the wave surface in the
direction of the propagation of wave.
y

P(x,y,z)
s n

O x

z
or j ( x , y, z )  a cos ( vt  kx x  k y y  kz z )
 a cos ( vt  kx cos a  ky cos b  kz cos g )
Thus, j ( x1 , y1 , z1 , t )  a cos ( vt  kx1 cos a  ky1 cos b  kz1 cos g )
and j ( x2 , y2 , z 2 , t )  a cos ( vt  kx2 cos a  ky2 cos b  kz 2 cos g )
Hence, the sought wave phase difference
w2  w1  k [( x1  x2 ) cos a  ( y1  y2 ) cos b  ( z1  z 2 ) cos g ]

or $w  w2  w1 | k|[( x1  x2 ) cos a  ( y1  y2 ) cos b  ( z1  z 2 ) cos g ]


v
 [( x1  x2 ) cos a  ( y1  y2 ) cos b  ( z1  z 2 ) cos g ]
v

4.152 The phase of the oscillation can be written as


&  vt  k – r
When the wave moves along the x-axis
&  vt  kx x (on putting k y  kz  0)
Since the velocity associated with this wave is v1
v
we have kx 
v1
92 PART FOUR OSCILLATIONS AND WAVES

v v
Similarly, ky  and kz 
v2 v3
v v v ¥e e e ´
Thus, k e x  e y  ez  v ¦ x  y  z µ
v1 v2 v3 § v1 v2 v3 ¶

4.153 The wave equation propagating in the direction of +ve x-axis in medium K is given as
j  a cos ( vt  kx )
v
So, j  a cos k (vt  x ) (where k  and v is the wave velocity)
v
In the reference frame K, the wave velocity will be (v  V) propagating in the direc-
tion of ve x-axis and x will be x. Thus, the sought wave equation
j  a cos k [(v  V ) t  x a ]
¨¥ v ´ · ¨ ¥ V´ ·
or j  a cos ©¦ v  V µ t  kx a ¸  a cos © vt ¦§ 1  v µ¶  kx a ¸
ª § v ¶ ¹ ª ¹

4.154 This follows on actually putting


j  f (t  ax )
e2 j 1 e2 j
in the wave equation 
ex 2 v 2 et 2
(We have written the one-dimensional form of the wave equation.)
1
Then, f  (t  ax )  a2 f  (t  ax )
v2
The wave equation is satisfied if
1
a 
va
That is the physical meaning of the constant a.

4.155 The given wave equation


j  60 cos (1800 t  5.3 x )
is of the type j  a cos ( vt  kx )
where a  60  10 6 m, v  1800 s1 and k  5.3 m1.
2p 2p
As k so, l 
l k
v v
Also k so, v   340 ms
v k
4.3 ELASTIC WAVES. ACOUSTICS 93

a ak
(a) Sought ratio is   5.1  105
l 2p
(b) Since j  a cos ( vt  kx )
ej
  av sin ( vt  kx )
et
So, velocity oscillation amplitude
¥ ej ´
¦ µ or vm  av  0.11 ms (1)
§ et ¶m
and the sought ratio of velocity oscillation amplitude to the wave propagation
velocity
vm 0.11
  3.2  104
v 340
(c) Relative deformation
ej
 ak sin ( vt  kx )
ex
So, relative deformation amplitude
¥ ej ´
¦ µ  ak  ( 60  10  5.3) m  3.2  10 m
6 4
(2)
§ ex ¶m
From Eqs. (1) and (2)
¥ ej ´ av 1 ¥ ej ´
¦ µ  ak   ¦ µ
ex
§ ¶m v v § et ¶m

Thus, ¥ ej ´ 1 ¥ ej ´
¦ µ  ¦ µ
ex
§ ¶m v § et ¶m
where v  340 m/s is the wave velocity.

4.156 (a) The given equation is


j  a cos ( vt  kx )
So at t  0, j  a cos kx
dj
Now,   av sin ( vt  kx )
dt
dj
and  av sin kx (at t  0)
dt
dj
Also,   ak sin ( vt  kx )
dx
94 PART FOUR OSCILLATIONS AND WAVES

and at t  0,
dj
  ak sin kx
dx
Hence, all the graphs are similar but have different amplitudes, as shown in the
answer sheet of the problem book.
(b) At the points where j  0, the velocity direction is positive, i.e., along ve x-axis
in the case of longitudinal and ve y-axis in the case of transverse waves, where
dj/dt is positive and vice versa.
For sought plots see the answer sheet of the problem book.

4.157 In the given wave equation the particle’s displacement amplitude  aegx .
Let there be two points x1 and x2, between which the displacement amplitude differs by
h  1%.
So, aegx  aegx  haegx
1 2 1

or egx (1  h)  egx
1 2

or ln (1  h)  gx1   gx2
ln (1  h)
or x2  x1  
g
ln (1  h)
So, path difference 
g
2p
and phase difference   path difference
l
2p ln (1  h) 2ph
   0.3 rad
l g lg

4.158 Let S be the source whose position vector relative to P S Q


the reference point O is r. Since intensities are inversely
proportional to the square of distances,
r1 r
Intensity at P ( I 1 ) d22 r2

Intensity at Q ( I 2 ) d12
O
where d1  PS and d2  QS.
But intensity is also proportional to the square of amplitude.
a12 d22
So,  or a1d1  a2 d2  k ( say )
a22 d12
4.3 ELASTIC WAVES. ACOUSTICS 95

k k
Thus, d1  and d2 
a1 a2
Let n be the unit vector along PQ directed from P to Q.
k
Then, PS  d1n  n
a1
k
and SQ  d2 n  n
a2
From the triangle law of vector addition
k
OP  PS  OS or r1  nr
a1
or a1 r1  k n  a1 r (1)
k
Similarly, r n  r2 or a2 r2  k n  a2 r (2)
a2
Adding Eqs. (1) and (2), we get
a1 r1  a2 r2  ( a1  a2 ) r
a1 r1  a2 r2
Hence, r
a1  a2

4.159 (a) We know that the equation of a spherical wave in a homogenous absorbing me-
dium of wave damping coefficient g is
a0a egr
j cos ( vt  kr )
r
Thus particle’s displacement amplitude equals
a0a egr
r
According to the conditions of the problem,
a0a egr 0

at r  r0 , a0  (1)
r0

and when r  r , a0 aa egr (2)


 0
h r
Thus, from Eqs. (1) and (2)
r0
e g (r  r )  h
0

r
or g (r  r0 )  ln (hr0 )  ln r
96 PART FOUR OSCILLATIONS AND WAVES

or ln h  ln r0  ln r
g
r  r0
ln 3  ln 5  ln 10
  0.08 m1
5
a0a egr
(b) As j cos ( vt  kr )
r
ej aa egr
So,  0 v sin ( vt  kr )
et r
¥ ej ´ a0a egr
¦§ µ¶  v
et n r
a0a egr a
But at point A,  0
r h

So, ¥ ej ´ a0 v a0 2pn
¦§ µ¶  
et m h h
50  106 22
 2  1.45  103  15 cms
3 7

4.160 (a) When the waves are transverse:


Equation of the resultant wave is
¥ y  x´
j  j1  j2  2a cos k ¦
§ 2 µ¶
cos vt [k( x  y )
2 ]
[ k( x  y )
 a a cos vt 
2 ] ¨ ¥ y  x´·
© where a a  2a cos k ¦§ 2 µ¶ ¸
ª ¹
Now, the equation of wave pattern is
x  y  k (k is a constant)
For sought plots see the answer sheet of the problem book.
For antinodes, i.e., maximum intensity
k( y  x )
cos   1  cos np
2
2np
or  (x  y)   nl
k
or y  x  n l ( for n  0, 1, 2, . . .)
Hence, the particles of the medium at the points lying on the solid straight lines
( y  x  n l) oscillate with maximum amplitude. For nodes, i.e., minimum
intensity,
4.3 ELASTIC WAVES. ACOUSTICS 97

k ( y  x)
cos 0
2
k ( y  x) p
or   (2n  1)
2 2
l
or y  x  (2n  1)
2
and hence the particles at the points lying on dotted lines do not oscillate.
(b) When the waves are longitudinal:
For sought plots see the answer sheet of the problem book.
j j
k ( y  x )  cos1 1 cos1 2
a a

or
j1
a [ j
 cos k ( y  x )  cos1 2
a ]
j2 ¥ 1 j2 ´
 cos k ( y  x )  sin k ( y  x ) sin ¦§ cos µ
a a¶
j2 j2
 cos k ( y  x )  sin k ( y  x ) 1  22 (1)
a a
From Eq. (1), if sin k ( y  x )  0 sin (np )
j1  j2 (1)n
Thus, the particles of the medium at the points lying on the straight lines
( y  x  n l2) will oscillate along those lines (for even n) or at right angles to
them (for odd n).
Also from Eq. (1),
p
cos k ( y  x )  0  cos (2n  1)
2
j12 j22
 1  (a circle)
a2 a2
Thus, the particles at the points, where y  x  (n  14) l, will oscillate along
circles. In general, all other particles will move along ellipses.

4.161 The displacement of oscillations is given by j  a cos ( vt  kx ). Without loss of


generality, we confine ourselves to x  0. Then the displacement maxima occurs at
vt  np. Now the energy density is given by
w  ra 2 v2 sin 2 vt (at x  0)
At T/6 time later than t  0 (where T  2pv is the time period),
98 PART FOUR OSCILLATIONS AND WAVES

p 3
w  ra2 v2 sin 2  ra2 v2  w0
3 4
1 2 2 2w0
Thus,  w  ra v 
2 3

4.162 The power output of the source must be


4pl 2 I 0  Q l
R
The required flux of acoustic power is then a
O sS
6
Q
4p
where 6 is the solid angle subtended by the disk enclosed by the ring at S. This solid
angle is
6  2p (1  cos a) (1)
So, flux
¨ 1 ·
&  2pl 2 I 0 ©1  2 ¸
ª 1  ( R L ) ¹
¥ l ´
Substitution gives &  2p  30 ¦ 1  MW  20 MW
§ 1  14 µ¶
Eq. (1) is a well-known result which is derived as follows. Let SO be the polar axis.
Then the required solid angle is the area of that part of the surface of a sphere of such
radius so that its co-latitude is v a.
a

Thus, 6  ° 2p sin u d u  2p (1  cos a)


0

4.163 From the result of Problem 4.162, power flowing out through any one of the open-
ings is
P ¥ h2 ´
1
2 §¦ R  (h2) µ¶
2 2

P ¥ h ´
 1
2 ¦§ 4 R  h µ¶
2 2

As total power output equals P, so the power reaching the lateral surface must be
P ¥ h ´ Ph
P  2– 1   0.07 W
2 ¦§ 4R  h ¶
2 2 µ 4 R 2  h2
4.3 ELASTIC WAVES. ACOUSTICS 99

4.164 We are given j  a cos kx cos vt


ej ej
So,   ak sin kx cos vt and   av cos kx sin vt
ex et
Thus, ( j )t  0  a cos kx and ( j )t  T /2   a cos kx
¥ ej ´ ¥ ej ´
Therefore, ¦ µ   ak sin kx and ¦ µ  ak sin kx
§ ex ¶t  0 § ex ¶t  T /2
¥ ej ´
(a) The graphs of (j) and ¦ µ are as shown in Fig. 35 of the answer sheet (p. 332).
§ ex ¶
(b) We can calculate the density as follows:
Take a parallelopiped of cross-section unity and length dx with its edges at x
and x  dx. After the oscillation the edge at x goes to x  j (x) and the edge
at x  dx goes to
ej
x  dx  j ( x  dx )  x  dx  j ( x )  dx
ex
Thus, the volume of the element (originally dx) becomes
¥ ej ´
¦§ 1  µ¶ dx
ex
and hence the density becomes
r0
r
1  ejex
On substituting, we get for the density r (x), the curves shown in Fig. 35 referred
to above.
(c) The velocity v (x) at time t  T/4 is
¥ ej ´
¦§ µ¶   a v cos kx
et t  T 4
On plotting, we get the Fig. 36 given in the answer sheet (p. 332).

4.165 Given j  a cos kx cos vt


(a) The potential energy density (per unit volume) is the energy of longitudinal
strain. This is
2
¥1 ´ 1 ¥ ej ´
w p  ¦ stress  strainµ  E ¦ µ
§2 ¶ 2 § ex ¶

(where ejex is the longitudinal strain and E is the Young’s modulus).


1 2 2 1
wp  Ea k sin 2 kx cos2 vt  ra2 v2 sin 2 kx cos2 vt
2 2
100 PART FOUR OSCILLATIONS AND WAVES

(b) The kinetic energy density is


2
1 ¥ ej ´ 1
wk  r ¦ µ  ra 2 v2 cos2 kx sin 2 vt
2 § et ¶ 2
On plotting we get Fig. 37 given in the answer sheet (p. 332).
For example, at t  0
1 2 2
w  w p  wk  ra v sin 2 kx
2
and the displacement nodes are at x  p2k so we do get the figure.

4.166 Let us denote the displacement of the elements of the string by


j  a sin kx cos vt
Since the string is 120 cm long we must have k 120  np.
If x1 is the distance at which the displacement amplitude first equals 3.5 mm, then
a sin kx1  3.5  a sin (kx1  15 k )
p  15 k
Then, kx1  15 k  p  kx1 or kx1 
2
Let us assume that the string has the form shown below in the figure.
It shows that
7.5
p
k  120  4p so, k  cm1 7.5 15 15 15 15 15 15 15
30
Thus, we are dealing with the third overtone.
p
Also, kx1  so, a  3.5 2 mm  4.949 mm
4

4.167 We have
1 1
n T m  Tl  M (where M  total mass of the wire)
2l 2l
When the wire is stretched, total mass of the wire remains constant. For the first wire, the
new length  l  h1l and for the second wire, the length is l  h2l . Also T1  a (h1l ),
where a is a constant and T2  a (h2l ). Substituting in the above formula
1 (a h1l )(l  h1l )
n1 
2(l  h1l ) M

1 (ah2l )(l  h2l )


n2 
2(l  h2l ) M
4.3 ELASTIC WAVES. ACOUSTICS 101

n2 1  h1 h2 1  h2
Therefore,  –
n1 1  h2 h1 1  h1

h2 (1  h1 ) 0.04 (1  0.02)
   1.4
h1 (1  h2 ) 0.02 (1  0.04)

4.168 Let initial length and tension be l and T, respectively.


1 T
So, n1 
2l r1
In accordance with the problem, the new length
l  35
la  l   0.65l
100
and new tension,
T  70
TaT   1.7T
100
Thus, the new frequency
1 Ta 1 1. 7 T
n2  
2l a r1 2  0.65 l r1

n2 1.7 1.3
Hence,   2
n1 0.65 0.65

4.169 Obviously in this case the velocity of sound propagation will be


v  2 n (l2  l1 )
where l2 and l1 are consecutive lengths at which resonance occurs
In accordance with the problem,
(l2  l1 )  l
So, v  2 n l  2  2000  8.5 cms  0.34 kms

4.170 (a) When the tube is closed at one end


v
nn  (2n  1) ( where n  0, 1, 2, . . .)
4l
340
 (2n  1)  100 (2n  1)
4  0.85
Thus for n  0, 1, 2, 3, 4, 5, 6, . . ., we get
102 PART FOUR OSCILLATIONS AND WAVES

n1  100 Hz, n2  300 Hz, n3  500 Hz, n4  700 Hz,


n5  900 Hz, n6  1100 Hz, n7  1300 Hz
Since n should be > n0  1250 Hz, we need not go beyond n6.
Thus, 6 natural oscillations are possible.
(b) An organ pipe opened from both ends vibrates with all harmonics of the funda-
mental frequency. Now, the fundamental mode frequency is given as
v
n (where v is the velocity of sound)
l
v
or n
2l
Here also, end correction has been neglected. So, the frequencies of higher modes
of vibrations are given by
¥ v´
n n¦ µ (1)
§ 2l ¶
v ¥ v´ ¥ v´
or n1  , n2  2 ¦ µ , n3  3 ¦ µ
2l § 2l ¶ § 2l ¶
It may be checked by putting the values of n in the Eq. (1), that below 1285 Hz,
there are a total of six possible natural oscillation frequencies of air column in
the open pipe.

4.171 Since the copper rod is clamped at midpoint, it becomes a node and the two free
ends will be antinodes. Thus, the fundamental node formed in the rod is as shown
in Fig. (a).
l
In this case, l
2
v 1 E
So, n0   (a) (b)
2l 2l r
where E  Young’s modulus and r is the density of copper.
Similarly, the second node or the first overtone in the rod is as shown above in
Fig. (b).
3l
Here, l
2
3v 3 E
Hence, n1  
2l 2l r
2n  1 E
n ( where n  0, 1, 2, . . .)
2l r
4.3 ELASTIC WAVES. ACOUSTICS 103

Putting the given values of E and r in the general equation


n  3.8(2 n  1) kHz
Hence, n0  3.8 kHz, n1  (3.8  3)  11.4 kHz, n2  (3.8)  5  19 kHz,
n3  (3.8  7)  26.6 kHz, n4  (3.8  9)  34.2 kHz,
n5  (3.8  11)  41.8 kHz, n6  (3.8 )  13  49.4 kHz and
n7  (3.8)  14  50 kHz
Hence, the sought number of frequencies between 20 kHz to 50 kHz equals 4.

4.172 Let two waves, j1  a cos ( vt  kx ) and j2  a cos ( vt  kx ), superimpose and as


a result, we have a standing wave (the resultant wave) in the string of the form
j  2 a cos kx cos vt.
According to the problem 2a  am . Hence, the standing wave excited in the string is
j  am cos kx cos vt
ej
or   vam cos kx sin vt
et
(a) So, the kinetic energy confined in the string element of length dx is given by
2
1 ¥ m ´ ¥ ej ´
dT  ¦ dx µ¶ ¦§ µ¶
2§ l et
1 ¥m ´ 2 2
or dT  ¦ dx µ¶ am v cos2 kx sin 2 vt
2§ l
mam2 v2 2p
or dT  sin 2 vt cos2 x dx
2l l
Hence, the kinetic energy confined in the string corresponding to the fundamen-
tal tone
l/2
mam2 v2 2p
T  ° dT  sin 2 vt ° cos 2 x dx
2l 0
l
(because for the fundamental tone, length of the string l  l2 ).
1
Integrating we get, T  mam2 v2 sin 2 vt
4
Hence, the sought maximum K.E. is
1
Tmax  mam2 v2
4
(because for maximum K.E. sin 2 vt  1 ).
104 PART FOUR OSCILLATIONS AND WAVES

(b) Mean K.E. averaged over one oscillation period


2pv

° Tdt  1 ma ° sin 2 vt dt
0
T  2
m v2 2pv
° dt 4 ° dt
0

1
or T  mam2 v2
8

4.173 We have a standing wave given by the equation


j  a sin kx cos vt
ej
So,   av sin kx sin vt
et
ej
and  ak cos kx cos vt
et
The K.E. confined in an element of length dx of the rod
2
1 ¥ ej ´ 1
dT  ( rS dx ) ¦ µ  rSa 2 v2 sin 2 vt sin 2 kx dx
2 § et ¶ 2
So the total K.E. confined in the rod
l/2
1 2p
T  ° dT  rSa 2 v2 sin 2 vt ° sin 2 x dx
2 0
l
2 2
pSa v r sin vt 2
or T
4k
The P.E. in the above rod element
j
¥ e2 j ´
dU  ° eU   ° Fj d j ¦§ where Fj   rS dx 2 µ¶
0 et
or Fj  ( rS dx ) v2 j
j
So, dU  v rS dx ° j d j
2

rv2 S j 2 rv2 Sa 2 cos2 vt sin 2 kx dx


or dU  dx 
2 2
Thus, the total P.E. stored in the rod
U  ° dU
4.3 ELASTIC WAVES. ACOUSTICS 105

l/2
2p
or U  rv2 Sa 2 cos2 vt ° sin
0
2
l
x dx

2
prSa v cos vt 2 2

4k
To find the P.E. stored in the rod element we may adopt an easier way. We know that the
potential energy density confined in a rod under elastic force
1 1 1
U D  (stress  strain )  sd  E d2
2 2 2
1 2
1 rv 2
 rv 2d2  d
2 2 k2
2
1 rv2 ¥ ej ´ 1
 ¦ µ  ra 2 v2 cos2 vt cos2 kx
2 k 2 § ex ¶ 2
Hence, the total P.E. stored in the rod will be
l/2
1
U  ° U D dV  ° 2 ra v 2 2 cos2 vt cos2 kx S dx
0

prSa 2 v2 cos2 vt

4k
Hence, the sought mechanical energy confined in the rod between the two adjacent
nodes
prv2 a 2 S
E T U 
4k

4.174 Receiver R1 registers the beating due to the sound waves reaching it directly from
source and the other due to the reflection from the wall.
Frequency of sound reaching directly from S to R1
v
n S m R  n0 (when S moves towards R1)
1
v u
v
and nSa m R  n0 (when S moves towards the wall)
1
v u
Now, frequency reaching R1 after reflection from wall
v
n W m R  n0 (when S moves towards R1)
1
v u s s s
R2 S R1
v
and nWa m R  n0 (when S moves towards the
1
v u wall)
106 PART FOUR OSCILLATIONS AND WAVES

Thus, the sought beat frequency


$n  (nS m R  nW m R )
1 1

v v 2n vu 2un0
 n0  n0  2 0 2  1 Hz
v u v u v u v

4.175 Let the velocity of tuning fork be u. Thus, frequency reaching the observer due to the
tuning fork that approaches the observer
v
n a  n0
v u
Frequency reaching the observer due to the tuning fork that recedes from the ob-
server
v
n   n0
v u
So, beat frequency
¥ 1 1 ´
n  n  n   n0 v ¦ 
§ v  u v  u µ¶
2 n0 vu
or n
v 2  u2
So, n u 2  (2v n0 )u  v 2 n  0

 2v n0  4n02 v 2  4n 2 v 2
Hence, u
2n
Hence, the sought value of u, on simplifying and noting that u > 0, is

v n0 ¥ ´
2
¥n´
u ¦ 1  ¦ µ  1µ  0.5 ms
n § § n0 ¶ ¶

4.176 Obviously, the maximum frequency will be heard when the source is moving with
maximum velocity towards the receiver and minimum frequency will be heard when
the source recedes with maximum velocity. As the source swings harmonically, its
maximum velocity equals av. Hence,
v v
nmax  n0 and nmin  n0
v av v av
So, the frequency bandwidth
¥ 2a v ´
$n  nmax  nmin  n0 v ¦ 2
§ v  a2 v2 µ¶
4.3 ELASTIC WAVES. ACOUSTICS 107

or ( $ n a 2 ) v2  (2n0 va ) v  $nv 2  0

 2 n0 va  4 n02 v 2 a2  $ n 2 a2 v 2
So, v
2 $n a 2
On simplifying and taking ve sign as v m 0 if $n m 0

vn0 ¥ ´
2
¥ Dn ´
v ¦ 1 ¦  1µ  34 s1
$n a § § n0 µ¶ ¶

4.177 It should be noted that the frequency emitted by the source at time t could not be
received at the same moment by the receiver, because till that time the source will
cover the distance 1/2wt2 and the sound wave will take further time 1/2 wt2/v to reach
the receiver. Therefore, the frequency noted by the receiver at time t should be emit-
ted by the source at the time t1 < t. Therefore,
¥ 12wt12 ´
t1  ¦ t (1)
§ v µ¶
and the frequency noted by the receiver
v (2)
n  n0
v  wt1
Solving Eqs. (1) and (2), we get
n0
n  1.35 kHz
1  2wt v

4.178 (a) When the observer receives the sound, the source is x
the closest to him. It means that frequency is emitted S s
by the source sometime before (see figure). Figure u
shows that the source approaches the stationary ob-
server with velocity v s cos u.
Hence, the frequency noted by the observer
¥ v ´
n  n0 ¦
§ v  v s cos u µ¶ O

¥ v ´ n0
 n0 ¦ µ  (1)
§ v  hv cos u ¶ 1  h cos u

x l2  x2
But  (2)
vs v
108 PART FOUR OSCILLATIONS AND WAVES

x v
so,  s h
l2  x2 v
or cos u  h
Hence, from Eqs. (1) and (2), the sought frequency
n0
n  5 kHz
1  h2
(b) When the source is right in front of O, the sound emitted by it will not be Doppler
shifted because u  90°. This sound will be received at O at time t  l/v after the
source has passed it. The source will by then have moved ahead by a distance
vs t  lh. The distance between the source and observer at this time will be
l 1  h2  0.32 km.

4.179 Frequency of sound when it reaches the wall


v u
na  n
v
The wall will reflect the sound with same frequency n. Thus, frequency noticed by a
stationary observer after reflection from wall
v
n   n
v u
(since wall behaves as a source of frequency n).
v u v v u
Thus, n  n . n
v v u v u
or v u laa v  u
laa  l or 
v u l v u
laa v u 2v
So, 1 1 
l v u v u
Hence, the sought percentage change in wavelength is
l  l 2u
 100   100 %  2 % decrease
l v u

4.180 Frequency of sound reaching the wall


¥ v  u´ (1)
n  n0 ¦
§ v µ¶
For the observer, the wall becomes a source of frequency n receding from it with
velocity u.
4.3 ELASTIC WAVES. ACOUSTICS 109

Thus, the frequency reaching the observer


¥ v ´ ¥ v  u´
na  n ¦ µ  n0 ¦ (using Eq. 1)
§ v  u ¶ § v  u µ¶
Hence, the beat frequency registered by the receiver (observer)
2un0
$n  n 0  n a   0.6 Hz
v u

4.181 Intensity of a spherical sound wave emitted from a point source in a homogenous
absorbing medium of wave damping coefficient g is given by
1
I  ra 2 e2gr v2 v
2
So, intensity of sound at a distance r1 from the source
I1 1 ra 2 e2gr v2 v 1


r12 2 r12
and intensity of sound at a distance r2 from the source
I2 1 ra2 e2gr v2 v 2

2

r2 2 r22
But according to the problem
1 I1 I
2
 22
h r1 r2
hr12 hr12
So,  e 2g ( r
2  r1 ) or ln  2 g (r2  r1 )
r22 r22
ln (hr12 r22 )
or g  6  103 m1
2 (r2  r1 )

4.182 (a) Loudness level in bels  log I I 0 , where I 0 is the threshold of audibility.
So, loudness level in decibels
I
L  10 log
I0
Thus, loudness level at x  x1
Ix
Lx  10 log 1
1
I0
Ix
Similarly, Lx  10 log 2
2
I0
110 PART FOUR OSCILLATIONS AND WAVES

Ix
Thus, Lx  Lx  10 log 2
2 1
Ix 1

12ra 2 v2ne2gx 2
or Lx  Lx  10 log
2
12ra 2 v2ne2gx
1
1

 Lx  10 log e2g ( x
1
2  x1 )

 Lx  20 g ( x2  x1 ) log e
1

Hence, L a  L  20 g x log e ( since x2  x1  x )


 20  20  0.23  50  0.4343
 60  10  50 dB
(b) The point at which the sound is not heard anymore, the loudness level should
be zero. Thus,
L 60
0  L  20 g x log e or x   0.3 km
20 g log e 20  0.23  0.4343

4.183 (a) Since there is no damping, so


I (12)ra 2 v2 nr02
Lr  10 log  10 log   20 log r0
0
I0 (12)ra 2 v2n
Similarly, Lr  20 log r

¥r ´
So, Lr  Lr  20 log ¦ 0 µ
0 §r¶

¥r ´
or Lr  Lr  20 log ¦ 0 µ
0
§r¶
20
 30  20  log  36 dB
10
(b) Let r be the sought distance at which the sound is not heard.
r0 r r
So, Lr  Lr  20 log 0 or Lr  20 log or 30  20 log
0
r 0
r0 20
r 3 r
So, log10  or 10( 3/2) 
20 2 20
Thus, r  200 10  0.63 km
Therefore, for r > 0.63 km no sound will be heard.
4.3 ELASTIC WAVES. ACOUSTICS 111

4.184 We treat the fork as a point source. In the absence of damping, the oscillation has
the form
constant
cos ( vt  kr )
r
Because of the damping of the fork, the amplitude of oscillation decreases exponen-
tially with the retarded time (i.e., the time at which the wave started from the source).
Thus, we write for the wave amplitude
constant  b ( t  r /n )
j e
r
(t  t  rAn ) (t  t  rB n )
e b e b
This means that 
rA rB
¥ rB  rA ´
¦t 
b § n ¶µ rA
Thus, e 
rB
ln rB rA
or b  0.12 s1
t  (rB rA )n

4.185 (a) Let us consider the motion of an element of the medium of thickness dx and unit
area of cross-section. Let j  displacement of the particles of the medium at loca-
tion x. Then, by the equation of motion
r dx j   dp
where dp is the pressure increment over the length dx.
Recalling the wave equation
e2 j
j  v 2 2
ex
we can write the above equation as p p+dp
ej2
rv 2 2 dx   dp
ex x x+dx
Integrating this equation, we get
ej
$ p  surplus pressure  rv 2  constant
ex
In the absence of a deformation (a wave), the surplus pressure is $p  0. So,
constant  0 and thus,
ej
$ p  rv 2
ex
(b) We have found earlier that
w  wk  w p  total energy density
112 PART FOUR OSCILLATIONS AND WAVES

2 2 2
1 ¥ ej ´ 1 ¥ ej ´ 1 ¥ ej ´
wk  r¦ µ and wp  E ¦ µ  rv 2 ¦ µ
2 § et ¶ 2 § ex ¶ 2 § ex ¶
It is easy to see that the space-time average of both densities is the same and the
space-time average of total energy is then
2
¥ ej ´
 w   rv ¦ µ2
§ ex ¶
The intensity of the wave is
( $p ) 2
I v  w 
rv
1
Using ($p )2  ($p )m2
2
we get ($p )m2
I
2rv

4.186 The intensity of the sound wave is


( $p )m2 ($p )m2
I 
2 rv 2rNl
(here, r is the density of air and we have used v
 nl).
Thus, the mean energy flow reaching the ball is
 &   °  j   dS   j   S
(as mean energy flow density vector < j > is constant).
 &    I pR 2
(Notice that |< j >| is nothing but intensity I, negative sign is due to energy inflow,
and pR 2 being the effective area (area of cross-section) of the ball.)
($p )m2
Hence,  &   p R 2 I  pR 2
2rnl
 10.9 mW (on substituting values)

4.187 (a) We have P ($p )m2


 intensity 
4pr 2 2rv

rv P
or ($ p )m 
2pr 2
4.4 ELECTOMAGNETIC WAVES. RADIATION 113

1.293 kg/m 3  340 m/s  0.80 W



2 p  1.5  1.5 m 2
1/2
1.293  340  0.8 ¥ kg kg m 2 s3 ms1 ´
 ¦ µ¶
2 p  1.5  1.5 § m5
 4.9877 ( kg m1 s2 )  5 Pa
($p )m
 5  105
p
ej
(b) We have $ p   r v2
ex
( $ p )m  r v 2 k jm  r v 2pnjm
($ p )m
jm  a 
2pr v n
5
  3 Mm
2p  1.293  340  600
jm 3  106 1800
   106  5  106
l 340600 340

4.188 Given that loudness level L  50 dB  5 bels.


Then the intensity at the relevant point (at a distance r from the source) is I 0 – 10 L.
Had there been no damping, the intensity would have been e 2gr I 0 – 10 L.
Now this must equal the quantity
P
(where P  sonic power of the source)
4p r 2
P
Thus,  e 2gr I 0 – 10 L
4pr 2
or P  4pr 2 e 2gr I 0 – 10 L  1.39 W

4.4 Electomagnetic Waves. Radiation

4.189 The velocity of light in a medium of relative permittivity d is c d. Thus, the change
in wavelength of light (from its value in vacuum to its value in the medium) is
c d c c ¥ 1 ´
$l    ¦  1µ   50 m
n n n § d ¶
114 PART FOUR OSCILLATIONS AND WAVES

4.190 From the data of the problem, the relative permittivity of the medium varies as
d( x )  d1 e ( xl ) ln d d
1 2

Hence, the local velocity of light


c c ¥x d ´
v( x )   exp ¦ ln 1 µ
d (x ) d1 § 2l d2 ¶
Thus, the required time t
t l
dx d ¥ x d1 ´
t°  1 ° exp ¦§ 2l ln d µ dx
0
n (x ) c 0 2¶

¥1 d ´
 exp ¦ ln 1 µ  1
d § 2 d2 ¶ 2l d1  d2
 1 
c 1 d1 c d
ln ln 1
2l d2 d2

4.191 Conduction current density jcond  sE


eD eE
Displacement current density of jdis   dd0  i vdd0 E
et et
jcond s
Ratio of amplitudes   2 (on substituting values)
jdis v d d0

eB eH
4.192 We have   E    m0
et et
  cos ( vt  k – r )  Em  k  Em sin ( vt  k – r )
eH k  Em
At r  0  sin vt
et m0
1
So integrating (ignoring a constant) and using c  , we get
d0 m0

k  Em 1 d0 k  Em
H cos ckt   cos ckt
m0 ck m0 k

4.193 As in the previous problem


k  Em E
H cos ( vt  k – r ) m e z cos (kx  vt )
m0 v m0c
d0
 Em e z cos (kx  vt )
m0
4.4 ELECTOMAGNETIC WAVES. RADIATION 115

Thus,
d0
(a) At t  0, H Em e z cos kx   0.3e z
m0

d0
(b) At t  t0, H Em e z cos (kx  vt0 )  0.18 e z
m0

4.194 We have jind 


° E – d l  E l [cos vt  cos(vt  kl )]
m

vl ¥ vl ´
  2Em l sin
sin ¦ vt  µ
2c § 2c ¶
1
Putting the values Em  50 mV/m, l  m, we get
2
vl 2pvl p  108 p
  
c c 3  10 8 3

¥ p´ ¥ p´
so, jind  50 mV ¦sin µ sin ¦ vt  µ
§ 6 ¶ § 6¶
¥ p p´ ¥ p´
  25 sin ¦ vt  µ  25 cos ¦ vt  µ mV
§ 6 2 ¶ § 3¶

4.195 We have E  jE (t, x) and B  kB (t1x)


eE eB eB
and Curl E  k   k
ex et et
eE eB
So,  
ex et
eE 1 eE
Also, Curl B  d00 m0  2
et c et
eB eB 1 eE
and Curl B  j so,  2
ex ex c et

4.196 Given E  Em cos (vt  k r)


d 0 k s Em
Then, as before, H cos ( vt k – r )
m0 k
d0 1
So, S  EH  Em  ( k  Em ) cos2 ( vt  k – r )
m0 k
116 PART FOUR OSCILLATIONS AND WAVES

d0 2 k
 Em cos2 ( vt  k – r )
m0 k

1 d0 2 k
 S  Em
2 m0 k

4.197 We have E  Em cos (2pNt  kx )


eD
(a) jdis   2pd0NEm sin ( vt  kx )
et
Thus, ( jdis )rms  jdis
2 1 2

 2pd0 NEm  0.20 mAm 2


(b) As in Problem 4.196, we can write
1 d0 2
 Sx   Em
2 m0
Thus, S x   3.3 MWm 2

4.198 For the Poynting vector, we can derive as in Problem 4.196


1 dd0 2
S  Em (along the direction of propagation)
2 m0
Hence, in time t (which is much longer than the time period T of the wave), the
energy reaching the ball is
1 dd0 2
pR 2  Em  t  5 kJ
2 m0

4.199 Here E  Em cos kx cos vt


From div E  0, we get Em  0, so, Em is in the y –z plane.
x

eB
Also,     E    cos kx  Em cos vt
et
 k  Em sin kx cos vt
k  Em
So, B sin kx sin vt  Bm sin kx sin vt
v
where Bm  Em c and Bm > Em in the y –z plane.
At t  0, B  0, so E  Em cos kx .
4.4 ELECTOMAGNETIC WAVES. RADIATION 117

T
At t  , E  0, so B  Bm sin kx .
4

4.200 Here E  Em cos kx vt


k  Em
H sin kx sin vt (exactly as in Problem 4.199)
m0 v
Em  ( k  Em ) 1
S E H  sin 2 kx sin 2vt
m0 v 4
1 ¥ 1 ´
Thus, Sx  d0 cEm2 sin 2kx sin 2vt ¦§ as m c  d0 c µ¶
4 0

Sx   0

4.201 Inside the condenser, the peak electrical energy


Wele  12 CVm2
1 d pR 2
 Vm2 0
2 d
where d  separation between the plates, pR2  area of each plate and V  Vm sin vt ,
where Vm is the maximum voltage.
Changing electric field causes a displacement current
uD
jdis   d0 Em v cos vt
ut
d vV
 0 m cos vt
d
This gives rise to a magnetic field B (r) (at a radial distance r from the centre of the
plate)
d vV
B (r ) – 2pr  m0 pr 2 jdis  m0 pr 2 0 m cos vt
d
1 r
B  d0 m0 v Vm cos vt
2 d
Energy associated with this field is
B2
° 2m0 dv
B2
° (2pr dr )d
2m0
118 PART FOUR OSCILLATIONS AND WAVES

B2
° (2pd )r dr
2m0
R
pd20 m0 v2 2
 Vm cos2 vt ° r 3 dr
4d 0

1 v2 R 4 2

pd20 m0 v2 Vm cos2 vt
16 d
Thus, the maximum magnetic energy
d20 m0 pR 2 2
Wmag  ( v R )2 Vm
16 d
2
Wmag 1 1 ¥ vR ´
Hence,  d0 m0 ( vR )2  ¦ µ  5  1015
Wele 8 8§ c ¶
The approximation is valid only if vR  c.

4.202 Here I  I m cos vt , then the peak magnetic energy is


1 2 1
Wmag  LI m  m0n2 I m2 pR 2 d
2 2
Changing magnetic field induces an electric field which by Faraday’s law is
d
dt °
E – 2pr   B – d S  pr 2 m0nI m v sin vt

1
E  r m0nI m v sin vt
2
The associated peak electric energy is
1
Wele  ° d0 E 2 (2pr dr ) d
2
R
1
 d0 m02n2 v2 I m2 sin 2 vt (2pd )° r 3 dr
8 0

1 pR 4
 d0 m02n2 v2 I m2 sin 2 vt  d
8 2
2
Wele 1 1 ¥ vR ´
Hence,  d0 m0 ( vR )2  ¦ µ  5  10
15
Wmag 8 8§ c ¶
Again, we expect the results to be valid if and only if
¥ vR ´
¦§ µ 1
c ¶
4.4 ELECTOMAGNETIC WAVES. RADIATION 119

4.203 If the charge on the capacitor is Q, the rate of increase of the capacitor’s energy is
d ¥ 1 Q 2 ´ QQ d
¦ µ  QQ
dt § 2 C ¶ C d 0 pR 2
Now, electric field between the plates (inside it) is E  QpR 2d0 .
So, displacement current is
eD Q

et pR 2
This will lead to a magnetic field (circuital) inside the plates. At a radial distance r
Q 
Qr
2prH u (r )  pr 2 or H u 
pR 2 2pR 2
Q
Hence, Hu (R )  (at the edge)
2pR
Thus, inward Poynting vector
Q Q
S 
2pR pR 2d0
Total flow is

QQd
2pRd  S 
pR 2 d0

4.204 Suppose the radius of the conductor is R0. Then the conduction current density is
I I rI
jc  2
 sE or E  
pR0 pR0 s pR02
2

1
(where r  is the resistivity).
s
Inside the conductor there is a magnetic field given by
I
H – 2pR0  I or H (at the edge)
2pR0
Therefore, energy flowing in per second in a section of length l is
rI 2l
EH  2pR0 l 
pR02
rl
But the resistance R
pR02
Thus, the energy flowing into the conductor  I 2 R.
120 PART FOUR OSCILLATIONS AND WAVES

I
4.205 Here nev 
pR 2
where R  radius of cross-section of the conductor and n  charge density (per unit
volume).
1 2eU
Also, mv 2  eU or v
2 m
Thus, the moving protons have a charge per unit length
m
nepR 2  I
2eU
This gives rise to an electric field at distance r given by
1 m2eU
E
d0 2pr
I
The magnetic field is H (for r > R)
2pr
I2 m
Thus, S (radially outward from the axis)
d0 4p 2r 2 2eU
This is the Poynting vector.

4.206 Within the solenoid, B  m0nI and the rate of change of magnetic energy
d ¥ 1 ´
W mag  ¦  m0n2pR 2lII
dt § 2m0n I pR l µ¶
2 2 2

where R  radius of cross-section of the solenoid and l  length.


Also, H  Bm0  nI along the axis within the solenoid.
So, Eu 2pr  pnr 2 B  pr 2 m0nI
1 
or Eu  m0nIr
2
So at the edge,
1  (circuital)
Eu ( R )  m0nIR
2
Then Sr  Eu H z (radially inward)
1
and W mag  m0n2 IIR
  2pRl  m0n 2pR 2lII (as before)
2
4.4 ELECTOMAGNETIC WAVES. RADIATION 121

4.207 Given w2  w1.


The electric field is as shown by the dashed lines ( ).
The magnetic field is as shown () emerging w1
out of the paper. S  E  H is parallel to the
wires and towards right.
Hence, the source must be on the left. w2

4.208 The electric field ( ) and the magnetic field (H ) are as shown. The electric
field by Gauss’ theorem is
A O
Er 
r
r +V
Integrating, we get w  A ln 2
r
V
So, A (r2  r1 )
ln r2 r1
V
Then, E
r ln r2 r1
I
Magnetic field Hu 
2pr
The Poynting vector S is along the z-axis and non-zero between the two wires
(r1 < r < r2). The total power flux is
r2
IV
° 2pr 2 ln r2 r1
– 2pr dr  IV
r1

4.209 As in the previous problem


V0 cos vt I 0 cos ( vt  w )
Er  and Hu 
r ln r2 r1 2pr
Hence, time averaged power flux (along the z-axis) is
1
V0 l0 cos w
2
1
(on using cos vt cos ( vt  w )   cos w ).
2

4.210 Let n be along the z-axis.


Then, S1n  E1x H1y  E1y H1x
122 PART FOUR OSCILLATIONS AND WAVES

and S2n  E2x H2y  E2y H2x


Using the boundary condition E1t  E2 t , H1t  H2t at the boundary (t  x or y), we
see that
S1n  S2n

4.211 We know  |2 when


P ^| p
ei e
p  ¤ ei ri  ¤ mi ri  ¤ mi ri
mi m
¥ ei e ´
¦§ if m  m  fixedµ¶ .
i

d2
But ¤ mi ri  0 (for a closed system)
dt 2
Hence, P  0.

 )2
1 2( p
4.212 We have P
4pd0 3c 3
 |2  (e v2a )2 cos2 vt
|p
1 2 1
Thus, P  ( e v 2 a )2 s
4pd0 3c 3 2
e 2 v4 a2
  5.1 s 101.5 W
12pd0c 3

e e 2q 1
4.213 Here,  
p s force 
m mR 2 4pd0
2
Thus,
1 ¥ e 2q ´ 2
P ¦ µ
( 4pd0 )3 § mR 2 ¶ 3c 3

4.214 Most of the radiation occurs when the moving particle is closest to the stationary
particle. In that region, we can write
R2  b2 + v2t2
and apply the previous problem’s formula.
@ 2
1 2 ¥ qe 2 ´ dt
Thus, $W z 3 3 °
( 4pd0 ) 3c @ m
¦§ µ¶
(b  v 2t 2 )2
2
4.4 ELECTOMAGNETIC WAVES. RADIATION 123

(the integral can be taken between @ with little error).


@ @
dt 1 dx p
Now, ° (b 2  v 2t 2 )2  v ° (b 2  x 2 )2  2vb 3
@ @

1 pq 2 e 4
Hence, $W z
( 4pd0 )3 3c 3m 2vb 3

4.215 For the semicircular path on the right


mv 2 BeR
 Bev or v
R m
1 B 2e 2 R 2
Thus, K.E. T  mv 2 
2 2m
Power radiated
2
1 2 ¥ ev 2 ´
P ¦ µ
4pd0 3c 3 § R ¶
Hence, energy radiated
2
1 2 ¥ B 2 e 3 R ´ pR B 3e 5 R 2
$W  ¦ µ – m 
4pd0 3c 3 § m 2 ¶ BeR 6d0m 3c 3
$W Be 3
So,   2.06  1018
T 3d0c 3m 2
(neglecting the change in v due to radiation, the value is correct if $W T  1).

mv
4.216 We have R
eB
2 2
Then, 1 2 ¥ ev 2 ´ 1 2 ¥ e 2 Bv ´
P ¦§ µ¶  ¦ µ
4pd0 3c 3 R 4pd0 3c 3 § m ¶
1 ¥ B 2e 4 ´
 ¦ µT
3pd0 c 3 § m 3 ¶
This is the radiated power, so
dT B 2 e 4
 T
dt 3pd0m 3 c 3
Integrating, we get T  T0 et/t
3pd0m 3 c 3
t
B 2e 4
124 PART FOUR OSCILLATIONS AND WAVES

t is (1836)3 y 1010 times less for an electron than for a proton, so electrons radiate
their energy much faster in a magnetic field.

4.217 P is a fixed point at a distance l from the equilibrium position of the particle. Because
l a, to first order in a/l, the distance between P and the instantaneous position of
the particle is still l. For the first case y  0 so t  T/4.
The corresponding retarded time is
T l
ta  
4 c
¥I l´ vl
Now, 
y (t a )  v2 a cos v ¦  v2 a sin
§ 4 c µ¶ c
For the second case, y  a at t  0 so at the retarded time t a   vl c .
vl
Thus, 
y (t a) v2a cos
c
The radiation fluxes in the two cases are proportional to ( 
y (t a))2 , so
S1 vl
 tan 2  3.06 (on substituting values)
S2 c
Note: The radiation received at P at time t depends on the acceleration of the charge
at the retarded time.

4.218 (a) Along the circle, x  R sin vt, y  R cos vt y


where v  v/R. If t is the parameter in x (t),
y (t) and t is the observer time then 1
v
l  x (t )
ta  t  vt
c R
x
where we have neglected the effect of the y-
coordinate which is of second order. The observed
coordinates are v
x(t)  x (t) and y(t)  y (t) 2

dy a dy dt dy vR sin vt vx vx cR


Then,     
dt a dt a dt a dt 1  (vRc )cos vt 1  vyc 1 vycR
d2 ya dt d ¥ vx R ´
and 
dt a 2 dt a dt ¦§ 1  vycR µ¶

1 « v 2 R 2 y vx R (v 2cR 2 x ) º v 2R (vc  y R )


 ¬  »
1  vycR ­1  vycR (1  vycR )2 ¼ (1  vycR )3
4.4 ELECTOMAGNETIC WAVES. RADIATION 125

This is the observed acceleration.


(b) Energy flow density of electromagnetic radiation S is proportional to the square of
the y -projection of the observed acceleration of the particle (i.e., d 2 y a/dt a2 ).
2
S1 ¨ (vc  1) (vc  1) · (1  vc )4
Thus, © ¸ 
S2 ª (1  vc )3 (1  vc )3 ¹ (1  vc )4

4.219 We know that S0 (r) ^ 1/r2. At other angles S (r, u) ^ sin2 u, thus
S (r, u)  S0 (r) sin2 u  S0 sin2 u
Average power radiated
u = p/2
2 8p
P  S0  4pr 2   S 0r 2 s S0
3 3
(Average of sin2 u over whole sphere is 2/3.)

4.220 From the previous problem


8pS0r 2
P0 
3
3P0
or S0 
8pr 2
S0 3 P0
Thus, w  
c 8pcr 2
(Poynting flux vector is the energy contained in a box of unit cross-section and
length c.)

4.221 The rotating dipole has moments


px  p cos vt and py  p sin vt
1 2 4 2 p 2 v4
Thus, P v p 
4pd0 3c 3 6p d 0 c 3

4.222 If the electric field of the wave is


E  E0 cos vt
This induces a dipole moment whose second derivative is
e 2 E0
 
p cos vt
m
126 PART FOUR OSCILLATIONS AND WAVES

Hence, radiated mean power


2
1 2 ¥ e 2 E0 ´ 1
P  ¦ µ 
4pd0 3c 3 § m ¶ 2
On the other hand, the mean incident Poynting flux is
d0 1
Sinc    E02
m0 2
2
P 1 2 ¥ e2 ´ m
Thus,  – (d0 m0 )3/2 ¦ µ s
Sinc  4pd0 3 § m¶ d0
2
m2 ¥ e 2 ´
 0 ¦ µ
6p § m ¶

4.223 For the elastically bound electron


mr  mv02 r  e E0 cos vt
This equation has a particular integral (i.e., neglecting the part which does not have
the frequency of the impressed force), which gives
e E0 cos vt
r
m v02  v2
e 2 E0 v2
So,   
p cos vt
( v02  v2 ) m
Hence, mean radiated power
2
1 2 ¥ e 2 v2 ´ 1 2
P E0
4pd0 3c 3 § m ( v02  v2 ) µ¶ 2
¦

The mean incident Poynting flux is


d0 1 2
Sinc   E0
m0 2
2
P m2 ¥ e 2 ´ v4
Thus,  0 ¦ µ
Sinc  6p § m ¶ ( v02  v2 )2

4.224 Let r  radius of the ball


R  distance between the ball and the Sun (r  R)
Ms  mass of the Sun
g  gravitational constant
4.4 ELECTOMAGNETIC WAVES. RADIATION 127

gM s 4p 3 P 1
Then, r r pr 2 –
2
R 3 4pR 2 c
(The factor 1/c converts the energy received on the right into momentum received.
Then the right-hand side is the momentum received per unit time and must equal the
negative of the impressed force for equilibrium.)
3P
Thus, r  0.606 Mm
16 pc rg Ms
OPTICS
PART 5
5.1 Photometry and Geometrical Optics

5.1 (a) The relative spectral response V(l) shown in Fig. 5.11 of the book is so defined
that A/V(l) is the energy flux of light of wavelength l needed to produce a unit lu-
minous flux at that wavelength. (A is the conversion factor defined in the book.)
At l  0.51 Mm, as per the figure
V( l)  0.50
and energy flux corresponding to a luminous flux of 1 lumen
1.6
  3.2 mW
0.50
At l  0.64 Mm, we have
V( l)  0.17
and energy flux corresponding to a luminous flux of 1 lumen
1. 6
  9.4 mW
0.17
&e
(b) Here, d &e ( l) 
l2  l1
dl in the interval l1  l  l2, since energy is distributed uniformly.
l2 l2
( l) &e
&  ° V ( l) d &2
A( l2  l1 ) l°
Then,  V ( l) d l
l A
1 1

Since V(l) is assumed to vary linearly in the interval l1  l  l2 , we have


l2
1 1
l1  l2 ° V ( l) d l  2 (V ( l )  V ( l ))
1 2
l1
130 PART FIVE OPTICS

&e
Thus, &
(V ( l1 )  V ( l2 ))
2A
Using V(0.58 Mm)  0.85 and V(0.63 Mm)  0.25, we get
&e
&  1.1  1.55 lm
2  1.6

5.2 We have &A


&e 
V( l)
1 d0 2 4p r 2
But &e  Em 
2 m0 area
n
mean energy
flux vector

&A m0
or Em2 
2pr V ( l) d0
2

For l  0.59 Mm and V ( l)  0.74, we have


Em  1.14 Vm
d0
Also, Hm  Em  3.02 mAm
m0

Total luminous flux incident


5.3 (a) Mean illuminance 
Total area illuminated
Now, to calculate the total luminous flux incident on the sphere, we note that the illu-
minance at the point of normal incidence is E0. Thus, the incident flux is E0 – pR 2 . So,
pR 2 – E 0
mean illuminance l2  R2
2pR 2
R
1 a 90 a
or  E   E0
2 l

(b) The sphere subtends a solid angle


¥ l 2  R2 ´
2p (1  cos a)  2p ¦ 1  µ¶
§ l
at the point source and therefore receives a total flux of
5.1 PHOTOMETRY AND GEOMETRICAL OPTICS 131

¥ l 2  R2 ´
2pI ¦ 1  µ¶
§ l
The area irradiated is
90  a
¥ R´
2pR 2 ° sin u d u  2pR 2 (1  sin a)  2pR 2 ¦ 1  µ
§ l¶
0

I 1  1  ( Rl )2
Thus, E
R2 1  Rl
Substituting, we get
 E   50 lx

5.4 Luminance L is the light energy emitted per unit area of the emitting surface in a given
direction per unit solid angle divided by cos u. Luminosity M is simply energy emitted
per unit area, so, we have
M  ° L cos u d 6
where the integration must be in the forward hemisphere of the emitting surface (as-
suming light is being emitted in only one direction, say, outward direction of the sur-
face). But
L  L0 cos u
p/2
2
Thus, M  ° L0 cos2 u d 6  2p ° L0 cos2 u sin u d u  pL0
0 3

5.5 (a) For a Lambert source, L  constant. The flux emitted into the cone is
&  L$S ° cos a d 6
u
u
 L$S ° 2p cos a sin a d a u
0

 L$S p(
(1  cos2 u)  pL$S sin 2 u
(b) The luminosity is obtained from the previous formula for u  90°. So,
&(u  90o)
M  pL
$S

5.6 The equivalent luminous intensity in the direction OP is LS cos u and the illuminance
at P is
132 PART FIVE OPTICS

LS cos u LSh2 O
cos u 
( R 2  h2 ) ( R 2  h 2 )2
u
LS LS
  2 h
( R 2 h  h )2 u
ª̈( R h  h )  2 R ·¹
2

P
This is maximum when R  h and the maximum illuminance is R

LS 1.6  102
2
  40 lx
4R 4

5.7 The illuminance at P is


I (u) I (u) cos3 u
EP  cos u 
(x  h )
2 2 h2
since this is constant at all x, we must have
I (u) cos3 u  constant  I 0 u
I0 h
or (u)  u
cos3 u
x P
The luminous flux reaching the table is
I0
&  pR 2   314 lm
h2

5.8 The illuminated area acts as a Lambert source of luminosity M  pL where


MS  rES  total reflected light. Thus, the luminance
rE
L
p R
The equivalent luminous intensity in the direction making u
an angle u with the vertical is
rES
LS cos u  cos u
p
90 – u
and the illuminance at point P is P
rES cos u sin u rES
 cos u sin 3 u
p R cosec u pR 2
2 2

This is maximum when


d
(cos u sin3 u)   sin 4 u  3 sin 2 u cos2 u  0
du
5.1 PHOTOMETRY AND GEOMETRICAL OPTICS 133

or tan 2 u  3  tan u  3
Then the maximum illuminance is
3 3 rES
16p R 2
This illuminance is obtained at a distance R cot u  R 3 from the ceiling. Substitution
gives the value 0.21 lx.

5.9 From the definition of luminance, the energy emitted in the radial direction by an ele-
ment dS on the surface of the dome is
d &  L dS d 6
Here L is constant. The solid d6 is given by
u
dA cos u
d6 
R2 O
where dA is the area of an element on the plane illuminated by the radial light.
L dS dA
Then, d&  cos u
R2
The illuminance at O is then
p2
d& L
E °  ° 2pR 2 sin u d u cos u
dA 0
R2
l
 2pL ° sin u d (sin u)  pL
0

5.10 Consider an element of area dS at point P. It emits light of flux


d &  L dS d 6 cos u
dA
 L dS cos2 u
h2 sec2 u
L dS dA
 cos4 u
h2
in the direction of the surface element dA at O.
The total illuminance at O is then
L dS
E° cos4 u
h2
134 PART FIVE OPTICS

But dS  2pr dr  2ph tan u d (h tan u)


h
 2ph2 sec2 u tan u d u dA
u O
p2
r
Substitution gives E  2pL ° sin u cos u d u  pL
0
u
P
5.11 Consider an angular element of area
2px dx  2ph2 tan u sec2 u d u
Light emitted from this ring is
d &  L d 6(2ph2 tan u sec2 u d u) cos u x

dA cos u
Now, d6  h
h2 sec2 u
u
where dA  an element of area of the table just below
the centre of the illuminant.
Then, the illuminance at the element dA will be
ua

E0  ° 2pL sin u cos u d u


u0

R
where sin a 
h  R2
2

Finally, using luminosity M  pL , we get


R2
E0  M sin 2 a  M
h2  R 2
¥ h2 ´ ¥ lm ´
or M  E0 ¦ 1  2 µ  700 lmm 2  ¦ 1 lx  1 2 dimensionally µ
§ R ¶ § m ¶

5.12 The light emitted by an element of the illuminant towards the point O under consid-
eration is (see figure)
d &  L dS d 6 cos ( a  b)
R
The element dS has the area a
A
dS  2pR 2 sin a d a
b
The distance a b
12
OA  ;h2  R 2  2hR cos a=
5.1 PHOTOMETRY AND GEOMETRICAL OPTICS 135

We also have
OA h R
 
sin a sin (a  b) sin b
From the figure, we have
h cos a  R
cos (a  b) 
OA
h  R cos a
cos b 
OA
If we imagine a small area d O at O, then
d 3 cos b
 d6
OA2
Hence, the illuminance at O is
d& (h cos a  R )(h  R cos a)
° d 3  ° L2pR 2 sin a d a
(OA)4
The limit of a is a  0 to that value for which a + b  90, for then light is emitted
tangentially. Thus,
R
amax  cos1
h
cos1 R h
(h  R cos a)(h cos a  R )
So, E ° L 2pR 2 sin a d a
(h2  R 2  2hR cos a)2
0

We put y  h2  R 2  2hR cos a


So, dy  2hR sin a d a

¥ h  h2  R 2  y ´ ¥ h2  R 2  y  ´
µ¶ ¦§ Rµ
dy ¦§
h2  R 2
2h 2R ¶
Therefore, E  ° L 2pR 2 2
( h  R )2 2hR y

h2  R 2
L 2pR 2 (h2  R 2  y )(h2  R 2  y )
 2 2
8h R ° y2
dy
( h  R )2

h2  R 2 h2  R 2
pL ¨ ( h 2  R 2 )2 · pL ¨ ( h 2  R 2 )2 ·
 2
4h ° 2 ©ª y 2  1¸¹ dy  4h2 ª© y  y ¸¹
(h  R ) ( h  R )2
136 PART FIVE OPTICS

pL

4h2
; ( h  R ) 2  ( h 2  R 2 )  ( h 2  R 2 )  ( h  R )2 =
pL pLR 2
 2 ;2h2  2R 2  2h2  2R 2 =  2
4h h
Substitution gives E  25.1 lx.

5.13 We see in the figure that because of the law of reflection, the component of the inci-
dent unit vector e along n changes sign on reflection while the component parallel to
the mirror remains unchanged.
n
So, e  e  e >
where e >  n( e – n )
e e
and e  e  n( e – n )
We see that the reflected unit vector is
e a  ea  e a>  e  e >
 [ e  n(e – n )]  n(e – n )  e  2n(e – n )

5.14 We choose the unit vectors perpendicular to the mirror as the x-, y-, z-axes in space.
Then after reflection from the mirror with normal along x-axis, we have
e a  e  2i ( i – e )   e x i  e y j  e z k
where i, j, k are the basic unit vectors. After a second reflection from the 2nd mirror,
say, along y-axis
e aa  e a  2j( j – e a )   e x i  e y j  ez k
Finally, after the third reflection
e aaa  e x i  e y j  ez k e

5.15 Let PQ be the surface of water and n be the refractive index (R.I.) of water. Let AO be
the shaft of light with incident angle u1 and OB and OC be the reflected and refracted
light rays at angles u1 and u2, respectively (see figure). N
From the figure
A
u2  p2  u1 u1 B
u
From the law of refraction at the interface PQ
P Q
sin u1 sin u1 O
n 
sin u2 sin(p2  u1 )
u2
sin u1
or n  tan u1
cos u1
C
5.1 PHOTOMETRY AND GEOMETRICAL OPTICS 137

Hence, u1  tan1 n
So, n  53o (on substituting values)

5.16 Let two optical mediums of R.I., n1 and n2, respectively be such that n1 > n2. When
angle of incidence is u1cr (see figure), from the law of reflection
n1 sin u1 cr  n2 N
(1)
When the angle of incidence is u1, from the law of refrac-
tion at the interface of mediums 1 and 2,
n1 sin u1  n2 sin u2 n
P n2 Q
1
But in accordance with the problems
¥p ´ u1cr
u2  ¦  u1 µ (2)
§2 ¶
So, n1 sin u1  n2 cos u1
Dividing Eq. (1) by Eq. (2)
sin u1cr 1

sin u1 cos u1
1
or h
cos u1
1 h2  1
so, cos u1  and sin u1  (3)
h h
n1 cos u1
But, 
n2 sin u1
n1 1 h
So,  (using Eq. 3)
n2 h h  1
2

Thus, n1 1
  1.25
n2 h 1
2

u
O n
5.17 From the figure, the sought lateral shift u
b b
x  OM sin (u  b) d N
 d sec b sin (u  b)
x

M
 d sec b (sin u cos b  cos u sin b)
 d(sin u  cos u tan b)
138 PART FIVE OPTICS

But from the law of refraction

sin u
sin u  n sin b or sin b 
n
n2  sin 2 u
So, cos b 
n
sin u
and tan b 
n  sin 2 u
2

Thus, x  d (sin u  cos u tan b)

¥ sin u ´
 d ¦ sin u  cos u µ
§ n  sin u ¶
2 2

¨ 1  sin 2 u ·
 d sin u ©1  ¸  3.1 cm
ª n2  sin 2 u ¹

5.18 From the figure,


MP MN cos a u
sin d a   M
OM h sec ( a  d a)
du N
Since da is very small, we have h' a
2
h
MN cos a MN cos a
da   (1)
h sec a h da
O
MN cos2 u
Similarly, du  (2)
ha
From Eqs. (1) and (2)
d a ha cos2 a h cos2 d a
 or ha  (3)
du h cos2 u cos2 a d u
From the law of refraction
n sin a  sin u (A)
sin u n2  sin 2 u
sin a  so, cos a  (B)
n n2
Differentiating Eq. (A)
da cos u
n cos a d a  cos u d u or  (4)
d u n cos a
5.1 PHOTOMETRY AND GEOMETRICAL OPTICS 139

Using Eq. (4) in Eq. (3), we get


h cos3 u
ha  (5)
n cos3 a
h cos3 u n2h cos3 u
Hence, ha  32
 (using Eq. B)
¥ n2  sin 2 u ´ (n2  sin 2 u)32
n¦ µ¶
§ n2

5.19 The figure shows the passage of a monochromatic ray through the given prism, placed
in air medium. From the figure, we have
u  b1  b2 (1) a u
and a  ( a1  a2 )  ( b1  b2 )
a1 a2
 (a1  a2 )  u (2) b1 b2
From Snell’s law,
sin a1  n sin b1
or a1  n b1 (for small angles) (3)
and sin a2  n sin b2
or a2  n b2 (for small angles) (4)
From Eqs. (2), (3) and (4), we get
a  n( b1  b2 )  u
So, a  n(u)  u  (n  1)u (using Eq. 1)

5.20 In general, for the passage of a monochromatic ray through a prism as shown in the
figure of the solution of Problem 5.19.
a  ( a1  a2 )  u (1)
Also, from Snell’s law,
sin a1  n sin b1 or a1  sin1 (n sin b1 ) (2)
Similarly,
a2  sin1 (n sin b2 )  sin1 [n sin (u  b1 )] (as u  b1  b2 )
Using Eq. (2) in Eq. (1), we get

a  [sin1 (n sin b1 )  sin1 (n sin (u  b1 ))]  u


da
For a to be minimum, 0
d b1
140 PART FIVE OPTICS

or n cos b1 n cos (u  b1 )
 0
1  n sin b1
2 2 1  n2 sin 2 (u  b1 )
cos2 b1 cos2 (u  b1 )
or 
(1  n2 sin 2 b1 ) 1  n2 sin 2 (u  b1 )
or cos2 b1 (1  n2 sin 2 (u  b1 ))  cos2 (u  b1 )(1  n2 sin 2 b1 )
or (1  sin 2 b1 )(1  n2 sin 2 (u  b1 ))  (1  sin 2 (u  b1 ))(1  n2 sin 2 b1 )
or 1  n2 sin 2 (u  b1 )  sin 2 b1  sin 2 b1 n2 sin 2 (u  b1 )
 1  n2 sin 2 b1  sin 2 (u  b1 )  sin 2 b1 n2 sin 2 (u  b1 )
or sin 2 (u  b1 )  n2 sin 2 (u  b1 )  sin 2 b1 (1  n2 )
or sin 2 (u  b1 )(1  n2 )  sin 2 b1 (1  n2 )
u
or u  b1  b1 or b1 
2
u
But b1  b2  u so, b2   b1
2 a
u
which is the case of symmetric passage of the ray.
In the case of symmetric passage of the ray a a
b b
a1  a2  aa (say)
u
and b1  b2  b 
2
Thus, the total deviation a  ( a1  a2 )  u
au
a  2aa  u or aa 
2
But from Snell’s law,
sin a  n sin b

au u
so, sin  n sin
2 2

5.21 In this case, we have


au u
sin  n sin (see solution of Problem 5.20)
2 2
In our problem a  u.
5.1 PHOTOMETRY AND GEOMETRICAL OPTICS 141

¥ u´ ¥ u´ ¥ u´ ¥ u´
So, sin u  n sin ¦ µ or 2 sin ¦ µ cos ¦ µ  n sin ¦ µ
§ 2¶ § 2¶ § 2¶ § 2¶
¥ u´ n
Hence, cos ¦ µ 
§ 2¶ 2
¥ n´
or u  2 cos1 ¦ µ  83o ( where n  1.5)
§ 2¶

5.22 In the case of minimum deviation


au u
sin  n sin
2 2

So, [ u
a  2 sin1 n sin
2 ]
 u  37o ( for n  1.5)

Passage of ray for grazing incidence and grazing emergence is a


the condition for maximum deviation. From the figure
a  p  u  p  2ucr u
(where ucr is the critical angle). ucr ucr
¥ 1´
So, a  p  2 sin ¦ µ  58o
§ n¶
for n  1.5  R.I. of glass.

5.23 The smallest deflection angle is given by the formula,


d  2a  u (1)
where a is the angle of incidence at first surface and u is the prism angle.
Also from Snell’s law, n1 sin a  n2 sin (u2), as the angle of refraction at first surface is
equal to half the angle of prism for least deflection.
n ¥ u´ 1.5
So, sin a  2 sin ¦ µ  sin 30o  0.5639
n1 § 2 ¶ 1.33
or a  sin1 (0.5639)  34.3259o
Substituting the values in Eq. (1), we get d  8.65o.

5.24 From Cauchy’s formula, and also experimentally, the R.I. of a medium depends upon
the wavelength of the monochromatic ray, i.e., n  f (l). In the case of least deviation
of a monochromatic ray passing through a prism, we have
u au
n sin  sin (1)
2 2
142 PART FIVE OPTICS

The above equation tells us that we have n  n (a), so we may write


dn
$n  $a (2)
da u a
From Eq. (1) Da
u 1 au
dn sin  cos da
2 2 2
dn cos (a  u)2
or  (3)
da 2 sin u2
From Eqs. (2) and (3)
cos (a  u)2
$n  $a
sin u2

1  sin 2 (a  u)2 1  n2 sin 2 u2


or $n  $a  $a (using Eq. 1)
2 sin u2 2 sin u2
2 sin u2
Thus, $a  $n  0.44
1  n2 sin 2 u2

5.25 Fermat’s principle: The actual path of propagation of light (trajectory of a light ray) is
the path which can be followed by light within the least time, in comparison with all
other hypothetical paths between the same two points.
The above statement is the original wording of Fermat (a famous French scientist of
17th century). Deduction of the law of refraction from Fermat’s principle:
Let the plane S be the interface between medium 1 and medium 2 [Fig. (a)] with the
refractive indices n1  c/v1 and n2  c/v2. Assume, as usual, that n1 < n2. Two points
are given — one above the plane S (point A), the other under plane S (point B). The
various distances are: AA1  h1, BB1  h2, A1B1  l. We must find the path from A to
B which can be covered by light faster than it can cover any other hypothetical path.
Clearly, this path must consist of two straight lines, viz., AO in medium 1 and OB in
medium 2. The point O in the plane S has to be found.

sA A P

{
s
h1 s A P
a lx
A1 B1 S S B
s s A O2 s s 1
{ s 1 A1 O
{

s
l b h2
h2 s x
O1 s
Bs B
s
B
(a) (b) (c)
5.1 PHOTOMETRY AND GEOMETRICAL OPTICS 143

First of all, it follows from Fermat’s principle that the point O must lie on the intersec-
tion of S and a plane P, which is perpendicular to S and passes through A and B.
Indeed, let us assume that this point does not lie in the plane P; let this be point O1
in Fig. (b). Drop the perpendicular O1 O2 from O1 onto P. Since AO2 < AO1 and BO2
< BO1, it is clear that the time required to traverse AO2B is less than that needed to
cover the path AO1B. Thus, using Fermat’s principle, we see that the first law of re-
fraction is observed: the incident and the refracted rays lie in the same plane as the
perpendicular to the interface at the point where the ray is refracted. This plane is the
plane P in Fig. (b); it is called the plane of incidence.
Now let us consider light rays in the plane of incidence [Fig. (c)]. Designate A1O as x and
OB1  l  x. The time it takes a ray to travel from A to O and then from O to B is

AO OB h 2  ( l  x )2 h2  x 2
T   2  1 (1)
v1 v2 v2 v1
The time depends on the value of x. According to Fermat’s principle, the value of x
must minimize the time T. At this value of x, the derivative dT/dx equals zero, i.e.,
dT x lx
  0 (2)
dx v1 h1  x
2 2 v2 h2  (l  x )2
2

x lx
Now,  sin a and  sin b
h12  x 2 h22  (l  x )2
Consequently,
sin a sin b sin a v1
 0 or 
v1 v2 sin b v2
sin a cn1 n2
So,  
sin b cn2 n1
Note: Fermat himself could not use Eq. 2, because mathematical analysis was devel-
oped later by Newton and Leibniz. To deduce the law of the refraction of light, Fermat
used his own maximum and minimum method of calculus, which, in fact, corre-
sponded to the subsequently developed method of finding the minimum (maximum) of
a function by differentiating it and equating the derivative to zero.

5.26 (a) Join F to P. Draw PQ parallel to OO. Reflected ray makes an angle (say a) with PQ
equal to that made by the incident ray with PF.
\
\ \\\

\\\ \
\ \ \ \\ \\ \\ \\\ \ \ \ \\ \\\ \

\ \ \\ \

O F
O O O
\ \ \ \ \ \ \ \ \ \ \ \ \ \ \ \ \\

F F
a
a
Q Q
a
\ \\ \ \

P P
\ \\ \

a
\ \\ \

\
144 PART FIVE OPTICS

(b) In Fig. (a) of the problem book, P is the object and P  is its (real) image. Look for a point
O  on the axis such that O P  and O P make equal angles with OO. This determines the
position of the mirror. Draw a ray from P parallel to the axis. This must on reflection pass
through P . The intersection of the reflected ray with principal axis determines the focus.
\ P

\ \\\
\ \ \ \\ \\ \\ \\\ \ \ \ \\ \\\ \
O O
F C
P

\ \\ \ \
\ \\ \
\
(a)
In Fig. (b) of the problem book, suppose P is the object and P  is the image. Then the
mirror is convex because the image is virtual, erect and diminished. Look for a point X
(between P and P ) on the axis such that PX and P X  make equal angle with the axis.
P \ \\\
\ \ \ \ \ \ \ \\ \\ \ \ \\ \ \

P'

X F C
\\ \ \ \ \
\ \\ \

(b)

5.27 (a) From the mirror formula,


1 1 1
 
sa s f
s as
we get, f  (1)
sa  s
But from the problem
sa
s  sa  l b
s
From these two relations, we get
l lb
s and sa 
1 b 1 b
Substituting it in the Eq. (1), we get
b((l 1  b)2 lb
f    10 cm
l (1  b1  b) (1  b2 )
(b) Again we have,
1 1 1 s s
  or 1
sa s f sa f
5.1 PHOTOMETRY AND GEOMETRICAL OPTICS 145

1 s s f
or  1
b1 f f
f
or b1  (2)
s f
Now, it is clear from the above equation that for smaller b, s must be large, so the
object is displaced away from the mirror in second position
f
i.e., b2  (3)
s l  f
Eliminating s from Eqs. (2) and (3), we get
l b1 b2
f    2.5 cm
( b2  b1 )

5.28 For a concave mirror


1 1 1 sf
  so sa 
sa s f s f
(In coordinate convention, s  s is negative and f  | f | is also negative.)
If A is the area of the mirror (assumed small) and the object is on the principal axis,
then the light incident on the mirror per second is I0 A/s2. This follows from the defini-
tion of luminous intensity as light emitted per second per unit solid angle in a given
direction and the fact that A/s2 is the solid angle subtended by the mirror at the source.
Of this a fraction r is reflected so if I is the luminous intensity of the image, then
IA A
 rI 0 2
sa 2 s
2
¥ |f| ´
Hence, I  rI 0 ¦
§| f | s µ¶
(Because our convention makes f ve for a concave mirror, we have to write | f |.)
Substitution gives I  2.0  103 cd.

5.29 For O1 to be the image, the optical paths of all rays OAO1 must be equal upto terms
of leading order in h. Thus,
A
nOA  naAO1  constant
h
But, OP | s |, O1 P | s a| and PM  d, so
O P dM C O1
OA  h2  (| s | d2 )
R
h2
| s | d 
2| s |
146 PART FIVE OPTICS

h2
Similarly, O1 A  h2  (| s a | d) | s a | d 
2| s a |
(Ignoring products h2d.) Then,
h2 ¥ n1 na ´
n | s | na| s a| nd  nad  ¦  µ  constant (1)
2 §| s | | s a|¶
From the triangle AMC,
( R  d) 2  h 2  R 2
or h2  2 R d (where CP  R, radius of curvature)
h 2
So, d (2)
2R
h2 « n  n a n na º
Hence, n | s | n a | s a |
¬   »  constant (using Eq. 2 in Eq. 1)
2 ­ R | s | | s a |¼
Since this must hold for all h, we have
na n na  n
 
a
|s | |s| R
From our sign convention, s > 0, s < 0 so we get
na n na  n
 
sa s R

5.30 All rays focusing at a point must have traversed the same optical path. Thus,
x x
A P

r
F O F
O ( f – x)

x  n r 2  ( f  x )2  nf
or (nf  x )2  n2r 2  n2 ( f  x )2
2
or n2r 2  (nf  x )2  ;n( f  x )=
 (nf  x  nf  nx )(nf  x  nf  nx )
 x (n  1)(2nf  (n  1) x )
 2n(n  1) fx  (n  1)(n  1) x 2
5.1 PHOTOMETRY AND GEOMETRICAL OPTICS 147

Thus, (n  1)(n  1) x 2  2n(n  1) fx  n2r 2  0

n(n  1) f  n2 (n  1)2 f 2  n2r 2 (n  1)(n  1)


So, x
(n  1)(n  1)

nf ¨ n 1 r2 ·
 ©1  1  ¸
n 1 ª n 1 f 2 ¹
The ray must move forward so x < f, for ve sign x > f for small r, so ve sign. (Also
x → 0 as r → 0.) x > f means ray is turning back in the direction of incidence (see
figure).
nf ¨ n 1 r2 ·
Hence, x ©1  1  ¸
n 1 ª n 1 f 2 ¹
For the maximum value of r,
n 1 r2
1 0 (1)
n 1 f 2
because the expression under the radical sign must be non-negative, which gives the
maximum value of r.
Hence, rmax  f (n  1)(n  1) (using Eq. 1)

5.31 Since the given lens has significant thickness, the thin lens formula cannot be used.
For refraction at the front surface from the formula
na n na  n
 
sa s R
1.5 1 1.5  1
 
sa 20 5
On simplifying, we get, s a  30 cm.
Thus, the image I produced by the front surface behaves as a virtual source for the
rear surface at distance 25 cm from it, because the thickness of the lens is 5 cm. Again,
from the refraction formula at curve surface
na n na  n
 
sa s R
O I'
1 1.5 1  1.5 I
 
s a 25 5
On simplifying, s a  6.25 cm.
Thus, we get a real image I at a distance 6.25 cm beyond the rear surface (see figure).
148 PART FIVE OPTICS

5.32 (a) The formation of the image of a source S, placed at a distance u from the pole of
the convex surface of plano-convex lens of thickness d is shown in the figure. On
applying the formula for refraction through a spherical surface, we get
n 1 (n  1)
  (here n2  n and n1  1)
sa s R
n 1 (n  1) 1 n (n  1)
So,   or  
d s R s d R N

or
sa
s
 sa [
n (n  1)
d

R ] O R O'
But in this case, optical path of the light correspond-
ing to the distance v in the medium is v/n, so the s
magnification produced will be d

b
sa

ns n d [
s a n (n  1)

R
 ] [
d n (n  1)
n d

R
1
nR ]
d (n  1)

Substituting the values, we get magnification b  0.20.


(b) If the transverse area of the object is A (assumed small), the area of the image is
b2A. We shall assume that pD 2 4  A . Then light falling on the lens is
pD 2 4
LA
s2
from the definition of luminance (see Eq. 5.1c) of the book. Here,
¥ pD 2 4 ´
cos u  1 (if D 2  s 2 ) and d6  ¦ 2 µ
§ s ¶
Then the illuminance of the image is
(pD 2 4)s 2 Ln2pD 2
LA 2

bA 4d 2
Substitution gives 42 lx.

5.33 (a) Optical power of a thin lens of R.I. n in a medium with R.I. n0 is given by
¥ 1 1´
&  (n  n0 ) ¦  µ (1)
§ R1 R2 ¶
From Eq. (1), when the lens is placed in air
¥ 1 1´
&0  (n  1) ¦  µ (2)
§ R1 R2 ¶
Similarly, from Eq. (1), when the lens is placed in liquid
5.1 PHOTOMETRY AND GEOMETRICAL OPTICS 149

¥ 1 1´
&  (n  n0 ) ¦  µ (3)
§ R1 R2 ¶
Thus, from Eqs. (2) and (3)
n  n0
& &0  2 D
n 1
The second focal length is given by
na
fa
&
(where n is the R.I. of the medium in which it is placed).
n
f a  0  85 cm
&
(b) Optical power of a thin lens of R.I. n placed in medium of R.I. n0 is given by
¥ 1 1´
&  (n  n0 ) ¦  µ (4)
§ R1 R2 ¶
For a biconvex lens placed in air medium from Eq. (4)
¥1 1 ´ 2(n  1)
&0  (n  1) ¦  
§ R R µ¶ R
where R is the radius of each curve surface of the lens.
Optical power of a spherical refractive surface is given by
na  n
&
R
For the rear surface of the lens which divides air and glass medium
n 1
&a  (here n is the R.I. of glass)
R
Similarly, for the front surface which divides water and glass medium
n  n n  n0
&l  0  (5)
R R
Hence, the optical power of the given optical system
n  1 n  n0 2n  n0  1
&  &a  &l    (6)
R R R
From Eqs. (4) and (6)
& 2n  n0  1 (2n  n0  1)
 so, & &0
&0 2(n  1) 2(n  1)
Focal length in air,
150 PART FIVE OPTICS

1
f  15 cm
&
or &  6.7 D
and focal length in water
n0 ¥ 4´
  20 cm ¦§ for n0  µ¶
& 3

5.34 (a) Clearly, the media on the sides are different. The front focus F is the position of the
object (virtual or real) for which the image is formed at infinity. The rear focus F is
the position of the image (virtual or real) of the object at infinity.
(a) For Figs. 5.7 (a) and (b) of the problem book:
C
D h'
* D
}h'
C

a a h'  B
b b O
a
F'
b
O'
b
{
h'
B
a F
O F A F' O' A

(a) Convex lens (b) Concave lens


This geometrical construction ensured that the second expression in Eq. (5.1g) is
obeyed. In the figure above, FA is parallel to the incident ray. Join F to D where
CD  AB. Emergent ray is parallel to DF.
(b) For Figs. 5.5 (a) and (b) of the problem book replacing mirror with lens:
P'
Ps D

P'
O F' O'
P F

(a) Convex lens (b) Concave lens


where P is the object.
(c) For Figs. 5.8 (a) and (b) of the problem book:
Clearly, it is important to note that when the rays (1)
and (2) are not symmetric about the principal axis, the O' OI
figure can be completed by reflection in the principal O
OI'
axis. Knowing one path we know the path of all rays a a
connecting the two points. For a different object, we
proceed as shown in the figure, we use the fact that a
ray incident at a given height above the optic centre suffers a definite deviation.
5.1 PHOTOMETRY AND GEOMETRICAL OPTICS 151

The concave lens can be discussed similarly.

5.35 Since the image is formed on the screen, it is real, so for a converging lens, the object
is on the incident side. Let s1 and s2 be the magnitudes of the object distance in the
first and second cases, respectively. We have the lens formula
1 1 1
  (1)
sa s f
In the first case, from Eq. (1)
1 1 1 f (l )
  or s1   26.31 cm
( l ) ( s1 ) f l f
Similarly, from Eq. (1) in the second case
1 1 1 lf
  or s2   26.36 cm
(l  $l ) ( s2 ) f ( l  $l )  f
Thus, the sought distance
$lf 2
$x  s2  s1   0.5 mm
(l  f 2 )

5.36 The distance between the object and the image is l. Let x  distance between the ob-
ject and the lens. Then, since the image is real, we have as per convention,
s   x, sa  l  x
1 1 1
So,  
x lx f
or x (l  x )  lf or x 2  xl  lf  0
Solving, we get the roots
1
x  ¨ªl  l 2  4lf ·¹
2
(We must have l > 4f for real roots.)
(a) If the distance between the two positions of the lens is $l, then clearly
$l  x2  x1  difference between roots  l 2  4lf
l 2  $l 2
So, f   20 cm
4l
(b) The two roots are conjugate in the sense that if one gives the object distance, the other
gives the corresponding image distance (in both cases). Thus, the magnifications are
l  l 2  4lf l  l 2  4lf
 (enlarged) and  (diminished)
l  l 2  4lf l  l 2  4lf
152 PART FIVE OPTICS

The ratio of these magnifications being h, we have


l  l 2  4lf l 2  4lf h 1
 h or 
l  l  4lf
2 l h 1
2
4f  h  1 h
or 1   1 4
l  h  1 (1  h )2

h
Hence, f l  20 cm
(1  h )2

5.37 We know from the previous problem that the two magnifications are reciprocal of
each other ( ba baa  1). If h is the size of the object, then
ha  bah and haa  baah
Hence, h  hahaa
 3.0 mm

5.38 Refer to Problem 5.32 b. If A is the area of the object, then provided the angular
diameter of the object at the lens is much smaller than other relevant angles like D/f,
we calculate the light falling on the lens as LA(pD 2 4 s 2 ), where s2 is the object distance
squared. If b is the transverse magnification ( b  s/s ) then the area of the image is b2A.
Hence, the illuminance of the image (also taking account of the light lost in the lens)
pD 2 1 (1  a)pD 2 L
E  (1  a) LA 2 2

4s b A 4f2
(since s  f for a distant object.)
Substitution gives E  15 lx

5.39 (a) If s  object distance, s  average distance, L  luminance of the source,


$S  area of the source assumed to be a plane surface held normal to the principal
axis, then we find for the flux incident on the lens
$&  ° L $S cos u d 6
@
 L $S ° cos u 2p sin u d u
0

pD 2
 L $S p sin 2 a  L $S
4s 2
5.1 PHOTOMETRY AND GEOMETRICAL OPTICS 153

Here, we are assuming D  s, and ignoring the variation of L since a is small.


If L is the luminance of the image and $S a  (S aS )2 $S is the area of the image,
then similarly
D2 D2 D2 D/2
L a $S a p  L a $S p  L $ S p a
4 s a2 4s 2 4s 2 S S'
or L  L (irrespective of D)
(b) In this case, the image on the white screen is from a Lambert source. Then, if its
luminance is L0, its luminosity will be pL0 and
s a2 D2
pL0 2 $S  L $S 2 p
s 4s
or L0 ^ D 2
since s depends on f and s but not on D.

5.40 Focal length of the converging lens, when it is submerged in water of R.I. n0 (say)
1 ¥n ´¥1 1 ´ 2(n1  n0 )
 ¦ 1  1µ ¦  µ (1)
f1 § n0 ¶ R R ¶
§ n0 R
Similarly, the focal length of diverging lens in water
1 ¥n ´¥ 1 1 ´ 2(n2  n0 )
 ¦ 2  1µ ¦  µ (2)
f2 § n0 ¶ § R R ¶ n0 R
Now, when they are put together in the water, the focal length of the system, will be
1 1 1
 
f f1 f2
2(n1  n0 ) 2(n2  n0 ) 2(n1  n2 )
  
n0 R n0 R n0 R
n0 R
or f   35 cm
2(n1  n2 )

\
5.41 C is the centre of curvature of the silvered surface and O is the
\\\\

effective centre of the equivalent mirror in the sense that an


\\\\ \\\

object at O forms a coincident image. From the figure, using the


\\\\\\\\\\\\\ \\\\ \\

formula for refraction at a spherical surface, we have C O


n 1 n 1 R
  or f 
\\\\

R 2 f R 2(2n  1) \

(In our case f is ve.)


Substitution gives f 10 cm.
154 PART FIVE OPTICS

5.42 (a) Path of a ray, as it passes through the lens system is as shown in the figure. Focal
length of all the three lenses
1
f m  10 cm (neglecting their signs)
10
3.33
Applying lens formula for the first lens, consider- cm
5 cm 5 cm 5 cm
ing a ray coming from infinity
1 1 1
  or s a  f  10 cm
sa @ f
and so the position of the image is 5 cm to the right of the second lens, when
only the first one is present. The ray again gets refracted while passing through the
second lens, so
1 1 1 1
  
sa 5 f 10
or s  10 cm, which is now 5 cm left to the third lens. So, for this lens
1 1 1 1 3
  or 
s aa 5 10 s aa 10

or s aa  103  3.33 cm (from the last lens)

(b) This means that if the object is x cm to the left


of the first lens on the axis OO then the image
is x on to the right of the third (last) lens. Call
the lenses L1, L2, L3 from the left and let O be O O2 O1 O3
the object, O1 its image by the first lens, O2 the
image of O1 by the second lens and O3 the im-
age of O2 by the third lens.
O1 and O2 must be symmetrically located with respect to the lens L2 and since this
lens is concave, O1 must be at a distance 2 | f2 | to be the right of L2 and O2 must
be 2 | f2 | to be the left of L2. One can check that this satisfies lens equation for
the third lens L3
s   (2| f2 | 5)   25 cm

s a  x, f3  10 cm

1 1 1
Hence,   so, x  16.67 cm
x 25 10
We have written | f2 | because f2 is ve in our convention.
5.1 PHOTOMETRY AND GEOMETRICAL OPTICS 155

5.43 (a) Angular magnification for the Galilean telescope in normal adjustment is given as
f
' o
fe
fo
or 10  or fo  10 fe (1)
fe
The length of the telescope in this case is given by
l  fo  fe  45 cm
O O' Fe
So, using Eq. (1), we get
S'
fe  5 and fo  50 cm
fe
(b) Using lens formula for the objective, l'
1 1 1 s f
  or s0a  0 o  50.5 cm
s0a s0 fo s0  f o
From the figure, it is clear that,
s0a  l a  fe (where l is the new tube length)
or l a  N0  fe  50.5  5  45.5 cm
So, the displacement of ocular is,
$l  l a  l  45.5  45  0.5 cm

5.44 In the Keplerian telescope, in normal adjustment, the distance between the objec-
tive and eyepiece is fo  fe. The image of the mounting produced by the eyepiece is
formed at a distance v to the right where
1 1 1
 
sa s fe
But s   ( fo  f e )

1 1 1 fo
so,   
sa fe fo  f e f e ( fo  f e )
The linear magnification produced by the eyepiece of the mounting is in magnitude,
sa f
| b|  e
s fo
This equals d/D according to the problem so,
fo D
' 
fe d
156 PART FIVE OPTICS

5.45 It is clear from the figure that a parallel beam of light, originally of intensity I0 has, on
emerging from the telescope, an intensity
2
¥f ´
I  I0 ¦ o µ
§ fe ¶
because it is concentrated over a section whose diameter is fe /fo of the diameter of the
cross-section of the incident beam.
2
¥f ´
Thus, h¦ oµ
§ fe ¶
fo
So, '  h
fe
tan #a #a
Now, ' 
tan # #
Hence, #  # a h  0.6 a, on substitution.

5.46 When a glass lens is immersed in water its focal length increases approximately four
times. We check this as follows:
1 ¥ 1 1´
 (n  1) ¦  µ
fa § R1 R2 ¶

1 ¥n ´¥ 1 1´
 ¦  1µ ¦  µ
fw § n0 ¶ § R1 R2 ¶
(nn0 )  1 1 n  n0 1
 – 
n 1 fa n0 (n  1) fa
Now, back to the problem, originally in air
fo
'  15 so, l  fo  fe  fe ( '  1)
fe
n0 (n  1)
In water, fe a  fe
n  n0
and the focal length of the replaced objective is given by the condition
foa  fea  l  ('  1) fe

or foa  ('  1) fe  fea

f oa n  n0
Hence, 'a   ('  1) 1
fea n0 (n  1)
5.1 PHOTOMETRY AND GEOMETRICAL OPTICS 157

Substitution gives (n  1.5, n0  1.33), 'a  3.09.

5.47 If L is the luminance of the object, A is its area, s  distance of the object, then light
falling on the object is
LpD 2
A
4s 2
The area of the image formed by the telescope (assuming that the image coincides
with the object) is '2 A and the area of the final image on the retina is
2
¥f ´ 2
¦ µ ' A
§s¶
where f  focal length of the eye lens. Thus the illuminance of the image on the retina
(when the object is observed through the telescope) is
LpD 2 A LpD 2

4 s 2 ( f s )2 ' 2 A 4 f 2 ' 2
When the object is viewed directly, the illuminance is, similarly
Lpd02
4f2
LpD 2 Lpd02
We want 
4 f 2 '2 4f2
D
So, 'a   20 (on substituting values)
d0

5.48 Obviously, fo  1 cm and fe  5 cm. Now, we know that magnification of a mi-


croscope is
¥ sa ´D
'  ¦ 0  1µ (for distinct vision)
§ fo ¶ fe
¥ sa ´ 25
or 50  ¦ 0  1µ or s0a  11 cm
§1 ¶ 5
Since distance between objective and ocular has increased by 2 cm, it will cause an
increase in the tube length by 2 cm.
So, s0aa  s0a  2  13 cm
¥ s aa ´D
and hence, 'a  ¦ 0  1 µ  60
§ fo ¶ fe
158 PART FIVE OPTICS

5.49 It is implied in the problem that the final image of the object is at infinity (otherwise
light coming out of the eyepiece will not have a definite diameter).
(a) We see that
s0a 2b | s0 | 2a
| s0 |
or b a 2a 2b d
s0a
Then, from the figure
s0a
d  2 fe b  2 fe a
| s0 |
But when the final image is at infinity, the magnification ' in a microscope is given by
sa l
' 0 – (l  least distance of distinct vision)
| s0 | f e
2l a
So, d
T
Since d  d0 when '  '0
2l a
d  15 (on substituting values)
d0
(b) If ' is the magnification produced by the microscope, then the area of the image
produced on the retina (when we observe an object through a microscope) is
2
¥f ´
'2 ¦ µ A
§s¶
where s  distance of the image produced by the microscope from the eye lens,
f  focal length of the eye lens and A  area of the object. If &  luminous flux
reaching the objective from the object and d  d0 so that the entire flux is admitted
into the eye), then illuminance of the final image on the retina is
&
' 2 ( f s ) 2 A
But if d  d02 , then only a fraction (d0 /d)2 of light is admitted into the eye and the
illuminance becomes
2
& ¥ d0 ´ &d02
¦ µ 
A( f s )2 '2 § d ¶ A( f s )2 (2l a)2
independent of '. The condition for this is then
d  d0 or '  '0  15
5.1 PHOTOMETRY AND GEOMETRICAL OPTICS 159

5.50 The primary and secondary focal lengths of a thick lens are given as
f   (n& ){1  (d n a )&2 }
and f a   (n aa& ){1  (d n a )&1 }
where & is the lens power and n, n and n are the refractive indices of first medium,
lens material and the second medium beyond the lens, respectively. &1 and &2 are the
powers of first and second spherical surface of the lens.
Here n  1 for lens, n  n for air and n  n0 for water.
f   1&1 º
So, » (as d  0) (1)
and f a   n0 &¼
Now, power of a thin lens
&  &1  &2
(n  1)
where &1 
R
(n0  n )
and &2 
R
(2n  n0  1)
So, & (2)
R
From Eqs. (1) and (2), we get
R
f   11.2 cm
(2n  n  1)
n0 R
and fa  14.9 cm
(2n  n0  1)
Since the distance between the primary principal point and primary nodal point is
given as
x  f a [(n aa  n )n aa]
So, in this case
n0 n0  1
x   (n0  1)&
& n0
n0 1
   f a  f  3.7 cm
& &

5.51 See the answer sheet of the problem book.


160 PART FIVE OPTICS

5.52 (a) Draw PX parallel to the axis OO and let PF in-
tersect it at X. That determines the principal point P X'
H. Since the medium on both sides of the system
H'
is the same, the principal point coincides with the O F O
H
nodal point. Draw a ray parallel to PH through P. P
That determines H. Draw a ray PS parallel to the X
axis and join PX. That gives F .

(b) We let H stand for the principal point (on the P


axis). Determine H by drawing a ray PH passing S'
through P and parallel to PH. One ray (conjugate H H' F'
O O
to SH) can be obtained from this. To get the other
ray one needs to know F or F . This is easy be- P'
S
cause P and P are known. Finally we get S.

(c) From the incident ray we determine Q. A line paral- Q Q'


lel to OO through Q determines Q and hence H. H
and H are also the nodal points. A ray parallel to the O O'
incident ray through H will emerge parallel to itself F H F'
through H. That determines F . Similarly, a ray par-
allel to the emergent ray through H determines F.

5.53 Here we do not assume that the media on the two sides of the system are the same.

S' S'
S
S
F H H' F'
H F F H
(a) (b)

H F' F H S
S'
S'
F' H' H F

(c) (d)
5.1 PHOTOMETRY AND GEOMETRICAL OPTICS 161

5.54 (a) Optical power of the system of combination of two lenses


&  &1  &2  d &1&2
on substituting values, & 4 D
1
or f   25 cm
&
Now, the position of primary principal plane with respect to the vertex of converg-
ing lens is
d &2
X  10 cm
&
Similarly, the distance of secondary principal plane with respect to the vertex of
diverging lens is
d &1
X a    10 cm (i.e., 10 cm left to it)
&
(b) The distance between the rear principal focal point F  and the vertex of converg-
ing lens is
¥ 1´ &d ¥ d &1 ´
l  d  ¦ µ (d &1 )  ¦
§ &¶ & § & µ¶

f ¥ 1´ &d d &1 ¥ 1´
and ¦ µ  ¦§ as f  µ¶
l § &¶ & & &
1
  d &1
d&
1 1
 (&1  &2  d &1&2 )  d &1   d 2 &1&2
d d &2
Now, if f/l is maximum for a certain value of d then l/f will be minimum for the
same value of d. And for minimum l/f
d (l  f ) dd  &2  2d &1&2  0
or d  &2 2&1&2
or d  12&1  5 cm
So, the required maximum ratio of f/l  4/3.

5.55 The optical power of first convex surface is


P (n  1)
& 5 D (as R1  10 cm )
R1
and the optical power of second concave surface is
162 PART FIVE OPTICS

(1  n )
&1    10 D
R2
So, the optical power of the system will be
d
&  &1  &2  &1&2   4 D
n
Now, the distance of the primary principal plane from the vertex of convex surface is
given as
¥ 1 ´ ¥d´
x  ¦ µ ¦ µ &2 (here n1  1 and n2  n)
§ &¶ § n ¶
d &2
  5 cm
&n
and the distance of secondary principal plane from the vertex of second concave sur-
face will be
¥ 1 ´ ¥d´ d&
x a  ¦ µ ¦ µ &1  1  2.5 cm
§ &¶ § n ¶ &n

5.56 The optical power of the system of two thin lenses placed in air is given as
&  &1  &2  d &1&2
1 1 1 d
or    (where f is the equivalent focal length)
f f1 f2 f1 f2
1 f  f1  d
So,  2
f f1 f2
f1 f2
or f
f1  f2  d
This equivalent focal length of the system of two lenses is measured from the primary
principal plane. As is clear from the figure, the distance of the primary principal plane
from the optical centre of the first is
O1 H  x   (n & )( d n a )&1
d &1 G L2
 (as n  n  1 for air) L1
&
df F2 F1 O1 H O2

f1
f
¥d´¥ f1 f2 ´
¦ µ ¦ d
§ f1 ¶ § f1  f2  d µ¶
5.1 PHOTOMETRY AND GEOMETRICAL OPTICS 163

df2

f1  f2  d
If we place the equivalent lens at the primary principal plane of the lens system, it will
provide the same transverse magnification as the system. So, the distance of equivalent
lens from the vertex of the first lens is
df2
x
f1  f2  d

5.57 The plane mirror forms the image of the lens and water, L1 L2
filled in the space between the two, behind the mirror,
as shown in the figure. So, the whole optical system is
equivalent to two similar lenses, separated by a distance
2l and thus, the power of this system
d &1&2
&  &1  &2 
n0
where &1  &2  &1 is the optical power of individual l l
lens and n0 is the R.I. of water.
Now, &  optical power of first convex surface  optical power of second concave
surface, so
(n  1) n0  n
&   (since n is the refractive index of glass)
R R
(2n  n0  1)

R
and so, the optical power of whole system will be
2d & a 2
&  2& a   3.0 D (on substituting values)
n0

5.58 (a) A telescope in normal adjustment is a zero-power combination of lenses. Thus,


we require
d
&  0  &1  &2  &1&2
n
n 1
But, &1  power of the convex surface 
R0  $R
n 1
&2  power of the concave surface 
R0

(n  1)$R d (n  1)2


Thus, 0 
R0 ( R0  $R ) n R0 ( R0  $R )
164 PART FIVE OPTICS

n$ R
So, d  4.5 cm (on substituting values)
n 1
0. 5 0. 5 d 0.5  0.5
(b) Here, &  1    
0.1 0.075 1.5 0.1  0.075
20 d  2 5  20 5 200d
5    
3 3 3 3 9
200d 2 ¥ 3 ´
or  or d¦ m  3 cm
9 3 § 100 µ¶

5.59 (a) The power of the lens is (as in the previous problem)
n  1 n  1 d ¥ n  1´ ¥ n  1´ d (n  1)2
&   ¦ µ ¦ µ 0
R R n§ R ¶§ R ¶ nR 2
The principal planes are located on the side of the convex surface at a distance
d from each other, with the front principal plane being removed from the convex
surface of the lens by a distance R/(n  1).
n  1 n  1 R2  R1 (n  1)2
(b) Here, &     R1
R1 R2 n R1 R2
(n  1)( R2  R1 ) ¨ n  1· R2
 ©1 
R2 R1 ª n ¹̧
n 1¥ 1 1 ´
 ¦  µ0
n § R1 R2 ¶
Both principal planes pass through the common centre of curvature of the sur-
faces of the lens.

5.60 Let the optical powers of the first and second surfaces of the ball of radius R1 be &1
and &1, respectively then
(n  1) (n  1)
&1   and &1   (1  n )(R1 ) 
R1 R1
This ball may be treated as a thick spherical lens of thickness 2R1. So, the optical
power of this sphere is
2R1&1a &1aa 2(n  1)
&  &1   
n nR1
Similarly, the optical power of second ball
2(n  1)
&2 
nR2
5.1 PHOTOMETRY AND GEOMETRICAL OPTICS 165

Let the distance between the centres of these balls be d. Then, the optical power of
whole system
&  &1  &2  d &1&2
2(n  1) 2(n  1) 4d (n  1)2
  
nR1 nR2 n2 R1 R2
2(n  1) ¨ 2d (n  1) ·
 © ( R1  R2 )  ¸¹
nR1 R2 ª n
Now, since this system serves as a telescope, the optical power of the system must be
equal to zero. So,
2d (n  1) ¥ 2(n  1) ´
( R1  R2 )  ¦ as p 0µ
n § nR1 R2 ¶

n( R1  R2 )
or d  9 cm
2(n  1)
Since the diameter D of the objective is 2R1 and that of the eyepiece is d  2R2, so,
the magnification
D 2R R
'  1  1 5
d 2R2 R2

5.61 Optical powers of the two surfaces of the lens are


(n  1) (1  n ) n  1
&1  and &2  
R R R
So, the power of the lens of thickness d will be
d &1& 2 n  1 n  1 d (n  1)2  R 2 n2  1
& a  &1  &2     
n R R n2 nR
and optical power of the combination of these two thick lenses will be
2(n2  1)
&  & a  & a  2& a   37 D
nR

5.62 Consider a ray QPR in a medium of gradually varying


R
refractive index n. At P, the gradient of n is a vector
with the given direction which is nearly the same at u u (du/2)
P
neighboring points Q and R. The arc length QR is ds. n
We apply Snell’s formula n sin u  constant, where u Q
is to be measured from the direction n. The refrac- N (Normal)
u (du/2)
tive indices at Q, R whose mid-point is P are
1
n  | n |d u cos u
2
166 PART FIVE OPTICS

¥ 1 ´¥ 1 ´
So, ¦§ n  | n |d u cos uµ¶ ¦§ sin u  cos u d uµ¶
2 2

¥ 1 ´¥ 1 ´
 ¦ n  | n |d u cos uµ ¦ sin u  cos u d uµ
§ 2 ¶ § 2 ¶

or n cos u d u |n |ds cos u sin u

¥ 1 ´ 1
(We have used sin ¦ u  d uµ  sin u  cos u d u here.)
§ 2 ¶ 2
Now, using the definition of the radius of curvature
1 du

r ds
1 1
we get,  |n |sin u
r n
The quantity |n| sin u can be called dn/dN , i.e., the derivative of n along the nor-
mal N to the ray. Then,
1 d
 ln n
r dN

5.63 From the previous problem


1 1 ˆ
 p – n  pˆ – n | n | 3  108 m1
r n
(since p̂  n both being vertical).
So, r  3.3  107 m
For the ray of light to propagate all the way round the earth, we must have
r  R  6400 km  6.4  106 m

Thus, | n | 1.6  107 m1

5.2 Interference of Light

5.64 (a) In this case the net vibration is given by


x  a1 cos vt  a2 cos ( vt  d)
where d is the phase difference between the two vibrations which varies rapidly
and randomly in the interval (0, 2p). (This is what is meant by incoherence.)
5.2 INTERFERENCE OF LIGHT 167

Then, x  ( a1  a2 cos d) cos vt  a2 sin d sin vt


The total energy will be taken to be proportional to the time average of the square
of the displacement.
Thus, E  ( a1  a2 cos d)2  a22 sin 2 d   a12  a22
as < cos d >  0 and we have put < cos2 vt >  < sin2 vt >  1/2 and this has been
absorbed in the overall constant of proportionality. In the same units the energies
of the two oscillations are a12 and a22, respectively, so the proposition is proved.
(b) Here r  a1 cos vt i  a2 cos ( vt  d) j
and the mean square displacement is
aa12  a22
If d is fixed but arbitrary, then as in (a), we see that E  E1  E2.

5.65 It is easier to do it analytically. Given that


j1  a cos vt , j2  2a sin vt
3 ¥ p p ´
and j3  a ¦ cos cos vt  sin sin vt µ
2 § 3 3 ¶
Resultant vibration is
7a ¥ 3 3´
j cos vt  a ¦ 2  sin vt
4 § 4 µ¶
This has an amplitude
a
49  (8  3 3 )2  1.89a
4

5.66 We use the method of complex amplitudes. Then the amplitudes are
A1  a, A2  ae iw ,. . . , AN  ae i ( N  1)w
and the resultant complex amplitude is
A  A1  A2  . . .  AN  a (1  e iw  e 2iw  . . .  e i ( N  1)w )
1  e iN w
a
1  e iw
The corresponding ordinary amplitude is
1 /2
1  e iN w ¨ 1  e iN w 1  eiN w ·
| A | a a© 
1 e i w
ª 1 e
iw 1  eiw ¸¹
1 2
¨ 2  2 cos N w · sin N w2
a© ¸ a
ª 2  2 cos w ¹ sin w2
168 PART FIVE OPTICS

5.67 (a) With dipole moment perpendicular to plane, there is no variation with u of indi-
vidual radiation amplitude. Then the intensity variation is due to interference only.
In the direction given by angle u, the phase difference is
P
2p
( d cos u)  w  2kp ( for maxima)
l
1
¥ w´
Thus, d cos u  ¦ k  µl
§ 2p ¶ u
Here k  0,  1,  2, . . .. d d cos u
We have added w to 2p/l d cos u because the
extra path that the wave from 2 has to travel
in going to P (as compared to 1) makes it lag
more than it already has (due to w). 2

(b) Maximum for u  p gives


¥ w´
d  ¦ k  µl
§ 2p ¶
Minimum for u  0 gives
¥ w 1´
d  ¦ka   µl
§ 2p 2 ¶
¥ w 1´
Adding we get ¦ k  ka   µ l  0
§ p 2¶
This can be true only if k a   k, w  p/ 2 , since 0 < w < p.
¥ 1´
Then, d  ¦ k  µ l
§ 4¶
Here k  0,  1,  2,3, . . . .
(Otherwise R.H.S. will become ve.)
Putting k   k , k  0,  1,  2,3, . . . , we get
¥ 1´
d  ¦k  µ l
§ 4¶

5.68 If $w is the phase difference between neighboring radiations, then for a maximum in
the direction u we must have
2p
d cos u  $w  2pk u u u u
l
For scanning u  vt  b
5.2 INTERFERENCE OF LIGHT 169

d $w
Thus, cos ( vt  b)  k
l 2p
¨ d ·
or $w  2p ©k  cos ( vt  b)
ª l ¹̧
To get the answer same as the book, put b  a  (p/2).

ll
5.69 From the general formula $x 
d
$x ll
we find that 
h d  2$h
Since d increases to d  2$h when the source is moved away from the mirror plane
by $h. Thus,
2$h
hd  d  2$h or d 
(h  1)
2$h$x
So, l  0.6 Mm
(h  1)l

5.70 We can think of the two coherent plane waves as emitted from two coherent point
sources very far away. Then
ll l
$x  
d d l
d
But  # ( if #  1)
l
l
so, $x 
#

5.71 (a) Here, S S   d  2r a


(b  r )l
Then, $x 
2ar
1
Putting b  1.3 m, r  0.1 m, l  0.55 Mm, a  12  radian, we get
5  57
$x  1.1 mm
Number of possible maxima
2ba
 1  8. 3  1  9
$x
(Here 2ba is the length of the spot on the screen which gets light after reflection
from both the mirrors. We add 1 above to take account of the fact that in a distance
$x there are two maxima.)
170 PART FIVE OPTICS

(b) When the slit moves by dl along the arc of radius r the incident ray on the mirror
rotates by dl /r; this is also the rotation of the reflected ray. There is then a shift of
the fringe of magnitude. Thus,
dl
b  13 mm (on substituting values)
r
(c) If the width of the slit is d then we can imagine the slit to consist of two narrow
slits with separation d. The fringe pattern due to the wide slit is the superposition
of the pattern due to these two narrow slits. The full pattern will not be sharp at
all if the pattern due to the two narrow slits are (1/2)$x apart because then the
maxima due to one will fill the minima due to the other. Thus,
b dmax 1 (b  r ) l
 $x 
r 2 4r a
¥ r´ l
or dmax  ¦ 1  µ  42.4 Mm
§ b ¶ 4a

5.72 To get this case we must let r → @ in the formula for $x of the last example.
(b  r )l l
So, $x  m
2ar 2a
(A plane wave is like light emitted from a point source at @.)
Then, l  2a$x  0.64 Mm

5.73 (a) We show the upper half on the lens. The emergent light is at an angle a/2f from
the axis. Thus, the divergence angle of the two incident light beams is
a
#
f
When they interfere, the fringes produced have
a width
l fl
$x    0.15 mm
# a
The patch on the screen illuminated by both
rays of light has width b # and this contains a
2
{

b # ba 2
f
 2  13 fringes
$x f l
(if we ignore 1 in comparison with b#/$x (as in solution of Problem 5.71a)
(b) We follow the logic of Problem 5.71c. From one edge of the slit to the other edge
the distance is of magnitude d (i.e., a/2 to a/2  d). If we imagine the edge to shift
by this distance, the angle #/2 will increase by $#/2  d/2f and the light will shift
 b – d/2 f . The fringe pattern will therefore shift by d. b/f.
5.2 INTERFERENCE OF LIGHT 171

Equating this to $x/2  f l/2a, we get


f 2l
dmax   37.5 Mm
2ab

ll
5.74 We know $x 
d
s
lab
d  2 (n  1) ua
d  (n  1) u
d  2d .a
{
d s

s
d

d
a b

n  R.I. of glass
2(n  1) ua $x
Thus, l  0.64 Mm
ab

5.75 It will be assumed that the space between the biprism and the
glass plate filled with benzene constitutes complementary prisms u
as shown in the figure. Then the two prisms being oppositely
placed, the net deviation produced by them is u

d  (n  1)u  (n  1)u  (n  n)u


Hence, as in the previous problem
d  2ad  2au (n  na )

So, (a  b ) l
$x 
2au (n  n a )
For plane incident wave, we let a → @
l
So, $x   0.2 mm
2u(n  n a )

5.76 Extra phase difference introduced by the glass plate is


2p
 (n  1)h
l
This will cause a shift equal to (n  1)hl fringe width,
h l l (n  1)hl
i.e., by (n  1)    2 mm
l d d
The fringes move down if the lower slit is covered by the plate to compensate for the
extra phase shift introduced by the plate.
172 PART FIVE OPTICS

1
5.77 No. of fringes shifted N  (n a  n ) N
l
Nl
So, na  n   1.000377
l

5.78 (a) Suppose the vector E, E, E correspond to the incident, reflected and the transmit-
ted waves respectively. Due to the continuity of the tangential component of the
electric field across the interface, it follows that
Et  Et   Et  (1)
where the subscript t means tangential.
The energy flux density is
EHS
Since H mm0  E ε ε 0
d0 d d
and H E n 0 E
m0 m m0
Now S ~ nE2 and since the light is incident normally
n1 E t2  n1 E t2  n2 E taa 2 (2)
or n1 ( E  E ta )  n2 E taa
2
t
2 2

So, n1 ( E t  E ta )  n2 E taa (3)


2n1
or E taa  Et
n1  n2
Since Et and Et have the same sign, there is no phase change involved in this case.
(b) From Eqs. (1) and (3)
(n2  n1 ) E t a  (n2  1) E t  0
n1  n2
or E ta  Et
n1  n2
If n2 > n1, then Et and Et have opposite signs. Thus, the reflected wave has an
abrupt change of phase by p if n2 < n1, i.e., on reflection from the interface be-
tween two media when light is incident from the rarer to denser medium.

5.79 Path difference between rays 1 and 2 is


2nd sec u2  2d tan u2 sin u1
n  (sin 2 u1 n )
 2d  2d n2  sin 2 u1
1  (sin 2 u1 n2 )
5.2 INTERFERENCE OF LIGHT 173

For bright fringes this must equal (k  12) l, where


u1
1/2 comes from the phase change of p for ray 1. u2
Here k  0, 1, 2, . . .. u2
Thus, 4d n2  sin 2 u1  (2k  1) l u1 1
l(1  2k )
or d  0.14(1  2k ) Mm d
4 n2  sin 2 u1
2
5.80 Given that
¥ 1´ ¥ 1´
2d n2  ¦ µ  ¦ k  µ  0.64 Mm ( bright fringe)
§ 4¶ § 2¶
¥ 1´
2d n2  ¦ µ  k a  0.40 Mm (dark fringe )
§ 4¶
(where k, k are integers.)
¥ 1´
Then, 64 ¦ k  µ  40k or 4(2k  1)  5k
§ 2¶
This means, for the smallest integer solutions, k  2 and k  4
4  0.40
Hence, d  0.65 Mm
2 n2  14

5.81 When the glass surface is coated with a material of R.I. n a  n (n  R.I. of glass)
of appropriate thickness, reflection is zero because of interference between various
multiplied reflected waves, as shown in the figure.
Let a wave of unit amplitude be normally incident from the left. The reflected ampli-
tude is r, where
Incident ray 1
n 1 t
r
n 1 r
tt'r tr
Its phase is ve so we write the reflected wave as
r. The transmitted wave has amplitude t, where tr 2
2
t
1 n +tr 3
+tt'r3
This wave is reflected at the second face and has
amplitude tr, because
n n n 1

n n n 1
174 PART FIVE OPTICS

The emergent wave has amplitude ttr.


It is proved below that tt  1  r2. There is also a reflected part of amplitude
trr  tr2, where r is the reflection coefficient for a ray incident from the coating
towards air. After reflection from the second face wave of amplitude
ttr 3   (1  r 2 )r 3
emerges. Let d be the phase of the wave after traversing the coating both ways.
Then, the complete reflected wave is
r  (1  r 2 )re i d  (1  r 2 )r 3 e 2i d  (1  r 2 )r 5 e 3i d . . .
1
  r  (1  r 2 )re i d
1  r 2e id
1
  r [1  r 2 e i d  (1  r 2 )e i d ]
1  r 2e id
1  e id
 r
1  r 2e i d
This vanishes if d  (2k  1)p. But
2p
d 2 nd
l
l 2
so, d  2k  1 r
4 n 1 r r
2
We now deduce that tt  1  r and r  r. This fol- tt'
lows from the principle of reversibility of light path as
shown in the figure.
tt  r 2  1
rt  r t  0 t
rt t
Therefore, tt  1  r 2 tr'
r  r
(r is the reflection ratio for the wave entering a denser medium.)

5.82 We have the condition for maxima


¥ 1´
2d n2  sin 2 u1  ¦ k  µ l
§ 2¶
This must hold for angle u  du2 with successive values of k. Thus,

¥ du ´ ¥ 1´
2d n2  sin 2 ¦ u  µ  ¦ k  µ l
§ 2 ¶ § 2¶
5.2 INTERFERENCE OF LIGHT 175

¥ du ´ ¥ 1´
and 2d n2  sin 2 ¦ u  µ  ¦ k  µ l
§ 2 ¶ § 2¶

Therefore, l  2d [ n2  sin 2 u  du sin u cosu  n2  sin 2 u  du sin u cos u ]


du sin ucos u
 2d
n2  sin 2 u

n2  sin 2 ul
or d  15.2 Mm
sin 2u du

5.83 For small angles u, we write for dark fringes


¥ sin 2 u ´
2d n2  sin 2 u  2d ¦ n  µ  (k ) l
§ 2n ¶
For the first dark fringe u  0 and 2dn  (k0 )l.
For the ith dark fringe u1
2
sin ui ´
¥
2d ¦ n  µ  (k0  i  1) l
§ 2n ¶
l l
nl r2
or sin ui 
2 (i  1)  i 2
d 4l
nl r2 r2
Then (i  k )  i 2 k
d 4l
d (ri2  rk2 )
so, l
4l 2n(i  k )

5.84 We have the usual equation for maxima


¥ 1´ Da
2hk a n2  sin 2 u1  ¦ k  µ l rk
§ 2¶
Here, hk  distance of the fringe from top and
hk a  dk  thickness of the film.
Thus, on the screen placed at right angles to the hk
reflected light a

$x  (hk  hk  1 ) cos u1
l cos u1
 dk
2a n2  sin 2 u1
176 PART FIVE OPTICS

5.85 (a) For normal incidence, we have, using the above formula,
l
$x 
2na
l
so, a  3 (on substituting values)
2n$x
(b) In a distance l on the wedge there are N  l/$x fringes. If the fringes disappear
there, it must be due to the fact that the maxima due to the component of wave-
length l coincides with the minima due to the component of wavelength l  $l.
Thus,
¥ 1´ l
N l  ¦ N  µ ( l  $l) or $l 
§ 2¶ 2N
$l 1 $x 0.21
so,     0.007
l 2N 2l 30
(The answer given in the answer sheet is off by a factor 2.)

1
5.86 We have r 2  k lR
2
So, for k differing by 1 ($k  1), we have
1 1
2r $r  $k lR  lR
2 2
lR
or $r 
4r

5.87 The path traversed in air film of the wave constituting the kth ring is
r2 1
 kl
R 2
When the lens is moved a distance $h the ring radius changes to r and the path
length becomes
r a2 1
 2$h  k l
R 2
Thus, r a  r 2  2R $h  1.5 mm

5.88 In this case, the path difference is r 2  r02 R for r > r0 and zero for r  r0. This must
equal (k  12) l (where k  6 for the sixth bright ring).

¥ 1´
Thus, r  r02  ¦ k  µ lR  3.8 mm
§ 2¶
5.2 INTERFERENCE OF LIGHT 177

5.89 From the formula for Newton’s rings, we derive for dark rings
d12 d22
 k1 R l and  k2 R l
4 4
d22  d12
So,  l  0.5 Mm (on substituting values)
4(k2  k1 ) R

5.90 Path difference between waves reflected by the two convex surface is
¥ 1 1 ´
r2 ¦  µ
§ R1 R2 ¶
Taking account of the phase change at the 2nd surface we write the
condition of bright rings as
¥ 1 1 ´ 2k  1 s
r2 ¦  µ  l
R
§ 1 R2 ¶ 2
In this case, k  4 for the fifth bright ring.
1 1 9 4 18 l
Thus,   l,  2
R1 R2 2 d2 d
1 1 1 1
Now  (n  1) ,  (n  1)
f1 R1 f2 R2
1 1 1 18 l
So,    (n  1) 2  &  2.40 D
f f1 f2 d
(Here n  R.I. of glass  1.5.)

¥ 2 2´
5.91 (a) Here &  (n  1) ¦  µ
§ R1 R2 ¶
1 1 &
so,  
R1 R2 2(n  1)
As in the previous problem, for the dark rings, we have
¥ 1 1´ &
rk2 ¦  µ  rk2  k l
§ 1
R R2¶ 2(n  1)
As k  0 is a dark spot; excluding it, we take k  10 here.
20 l(n  1)
Then, r  3.49 mm
&
178 PART FIVE OPTICS

(b) Path difference in water film will be


¥ 1 1´
n0r 2  ¦  µ
§ R1 R2 ¶
(where r  new radius of the ring.) Thus,
n0r 2  r 2
r
or r   3.03 mm
n0
(Here n0  R.I. of water  1.33.)

5.92 The conditions for minima is


r2 ¥ 1´
n2  ¦ k  µ l
R § 2¶
(Phase changes occur at both surfaces on reflection, hence, minima when path differ-
ence is half integer multiple of l.)
In this case k  4 for the fifth dark ring, counting from k  0 for the first dark ring.
(2k  1) lR
Thus, r , k5
2n2
r  1.17 mm (on substituting values)

5.93 Sharpness of the fringe pattern is the worst when the maxima and minima intermingle
¥ 1´
So, n1 l1  ¦ n1  µ l2
§ 2¶
Using l1  l, l2  l  $l, we get
l
n1$l 
2
l l1
or n1    140
2$l 2( l2  l1 )

5.94 Interference pattern vanishes when the maxima due to one wavelength mingle with
the minima due to the other. Thus,
2$h  k l2  (k  1) l1
(where $h  displacement of the mirror between the sharpest patterns of rings).
Thus, k( l2  l1 )  l1
l1
or k
l2  l1
5.3 DIFFRACTION OF LIGHT 179

l1 l2 l2
So, $h    0.29 mm
2( l2  l1 ) 2$l

5.95 (a) The path difference between rays 1 and 2 can be seen to be
$  2d secu  2d tan u sin u
 2d cosu  k l (for maxima)
(Here k  half-integer.)
d
The order of interference decreases as u increases, i.e., as
the radius of the rings increases. u
(b) Differentiating 2d cos u  kl u
2d sin u du  dkL
or 2d sin u du  l
(On putting dk 1.)
l
Thus, du 
2d sin u
du decreases as u increases.

2d
5.96 (a) We have kmax  ( for u  0)
l
(b) We must have
2d cos u  k l  (k  1)( l  $l)
1 l l l2
Thus,  and $l    5 pm (on substituting values)
k 2d k 2d

5.3 Diffraction of Light


5.97 The radius of the periphery of the N th Fresnel zone is
rN  Nb l
Then, by conservation of energy
@
I 0 p ( Nb l )2  ° 2pr dr I (r )
0
(Here r is the distance from the point P.)
@
2
Nb l °0
Thus, I0  r dr I (r )
180 PART FIVE OPTICS

5.98 By definition
ab l
rk2  k (for the periphery of the k th zone)
a b
Then, ark2  brk2  kab l
ark2 ar 2
So, b   2 m (on substituting values)
ka l  rk2 3a l  r 2
(It is given that r  rk for k  3.)

5.99 Suppose maximum intensity is obtained when the aperture contains k zones. Then a
minimum will be obtained for k  1 zones. Another maximum will be obtained for
k  2 zones. Hence,
ab
r12  k l
a b
ab
and r22  (k  2)l
ab
ab 2
Thus, l (r2  r12 )  0.598 Mm (on substituting values)
2ab

5.100 (a) When the aperture is equal to the first Fresnel zone: The amplitude is A1 and
should be compared with the amplitude A/2 when the aperture is very wide. If I0
is the intensity in the second case, the intensity in the first case will be 4I0.
When the aperture is equal to the internal half of the first zone: Suppose Ain and
Aout are the amplitudes due to the two halves of the first Fresnel zone. Clearly,
Ain and Aout differ in phase by p/2 because only half the Fresnel zone is involved.
Also, in magnitude
|Ain|  |Aout|
2
Then, A12  2| Ain |
2 A2
so, |Ain |  1
2
Hence, following the argument of the first case, Iin  2I0.
(b) The aperture was made equal to the first Fresnel zone and then half of it was closed
along a diameter. In this case the amplitude of vibration is A1/2. Thus, I  I 0 .

5.101 (a) Suppose the disk does not obstruct light at all. Then,
1
Adisk  Aremainder  Adisk
2
(because the disk covers the first Fresnel zone only).
5.3 DIFFRACTION OF LIGHT 181

1
So, Aremainder 
Adisk
2
Hence, the amplitude when half of the disk is removed along a diameter is
1
Adisk  Aremainder
2
1 1
 Adisk  Adisk  0
2 2
Hence, I  0.
(b) In this case
1
A Aexternal  Aremainder
2
1 1
 Aexternal  Adisk
2 2
We write Adisk  Ain  iAout
where Ain (Aout) stands for Ainternal (Aexternal). The factor i takes account of the p/2
phase difference between two halves of the first Fresnel zone.
1 1
Thus, A  Ain and I  Ain2
2 4
1 1 2
On the other hand I 0  ( Ain2  Aout 2 ) Ain
4 2
1
So, I  I0
2

5.102 When the screen is fully transparent, the amplitude of vibrations is (1/2)A1 with in-
tensity I 0  14 A12 .
(a) In case of screen (1)
3 ¥1 ´ 9
A ¦ A1 µ and I  I 0
4§2 ¶ 16
In case of screen (2), half of the plane is blocked out so
1¥1 ´ 1
A  ¦ A1 µ and I I0
2§2 ¶ 4
In case screen (3)
1 ¥ A1 ´ 1
A ¦ µ and I I0
4§ 2 ¶ 16
In case of screen (4)
1¥1 ´ I
A  ¦ A1 µ and I I0, so I4  I2
2§2 ¶ 4
182 PART FIVE OPTICS

In general we get
2
¥ ¥ w ´´
I w  I 0 ¦ 1  ¦ µµ
§ § 2p ¶ ¶
where w is the total angle blocked out by the screen.
(b) In case of screen (5)
3 ¥1 ´ 1
A ¦ A1 µ  A1
4§2 ¶ 4
A1 being the contribution of the first Fresnel zone.
5 25
Thus, A  A1 and I  I0
8 16
In case of screen (6)
1¥1 ´ 1 3 9
A  ¦ A2 µ  A1  A1 and I I0
§
2 2 ¶ 2 4 4
In case of screen (7)
1 ¥1 ´ 3 7 49
A ¦§ A1 µ¶  A1  A1 and I I0
4 2 4 8 16
In case of screen (8)
1¥1 ´ 1 3 9
A  ¦ A1 µ  A1  A1 and I  I 0 , so I 8  I 6
2§2 ¶ 2 4 4
In screens (5) to (8) the first term in the expression for the amplitude is the
contribution of the plane part and the second term gives the expression for the
Fresnel zone part. In general in screens (5) and (8)
2
¨ ¥ w ´·
I  I 0 ©1  ¦ µ¸
ª § 2p ¶ ¹
where w is the angle covered by the screen.

5.103 We would require the contribution to the amplitude P


of a wave at a point from half a Fresnel zone. For this,
we proceed directly from the Fresnel-Huygen’s prin-
ciple. The complex amplitude is written as
a0 ikr b
E  ° K (w ) e dS
r
Here K (w) is a factor which depends on the angle w
between a normal n to the area dS and the direction
from dS to the point P and r is the distance from the
element dS to P.
5.3 DIFFRACTION OF LIGHT 183

We see that for the first Fresnel zone


bl
a0
°
2
E eikb  ik r 2b 2pr d r ( where K (w )  1)
b 0

¥ r2 2 ´
¦§ using r  b  ( for r  b )µ¶ .
2
2b
For the first Fresnel zone, r  b  (l/2) so, r2  b2  bl and r2  bl.
( b l 2 ) kx
a0 ikb i
Thus, E
b
e 2p ° e b dx
0

a0 eik l2  1
 2peikb
b ikb
a0 4p
 2pieikb (2)   ia0 eikb  A1
k k
For the next half-zone
3b l4
a0 ikb
E e 2p ° eikx /b dx
b b l2

a0
 2pieikb (ei ( 3k l/4 )  eik l2 )
k
a0 A (1  i )
 2pieikb (1  i )   1
k 2
If we calculate the contribution of the full second Fresnel zone we will get A1. If
we take account of the factors K(w) and 1/r which decrease monotonically, we expect
the contribution to change to A2. Thus, we write for the contribution of the half
zones in the second Fresnel zone as
A2 (1  i ) A (1  i )
 and  2
2 2
The part lying in the recess has an extra phase difference
2p
d   (n  1)h
l
Thus, the full amplitude is (note that the correct form is eikr)
¥ A2 ´ A
¦§ A1  (1  i )µ ei d  2 (1  i )  A3  A4  . . .
2 ¶ 2
¥A ´ A A
 ¦ 1 (1  i )µ ei d  2 (1  i )  3
§ 2 ¶ 2 2
184 PART FIVE OPTICS

¥A ´ A
 ¦ 1 (1  i )µ ei d  i 1 (as A2  A3  A1 )
§ 2 ¶ 2
A3
and A3  A4  A5 . . . 
2
The corresponding intensity is
A12 ¨ i·
I ©ª(1  i )e  e ¸¹ [(1  i )e  i ]
i d i d
4

¨ ¥ p ´·
 I 0 [3  2 cos d  2 sin d]  I 0 ©3  2 2 sin ¦ d  µ ¸
ª § 4 ¶¹
(a) For maximum intensity
¥ p´
sin ¦ d  µ  1
§ 4¶
p p
or d  2kp  ( where k  0, 1, 2, . . .)
4 2
3p 2p
d  2kp   (n  1)h
4 l
l ¥ 3´
So, h ¦k  µ
n 1§ 8¶
(b) For minimum intensity
¥ p´
sin ¦ d  µ  1
§ 4¶
p 3p 7p
d  2kp  or d  2kp 
4 2 4
l ¥ 7p ´
So, h ¦k  µ
n 1§ 8 ¶

(c) For I  I 0 , cos d  0


sin d  1 ] or [ sin d  0
cos d  1

kl
Thus, d  2kp and h
n 1
3p l ¥ 3p ´
or d  2k  and h ¦§ k  µ
2 n 1 4 ¶
5.3 DIFFRACTION OF LIGHT 185

5.104 The contribution to the wave amplitude of the inner half-zone is


b l2
2pa0 eikb
°
2
eik r /2b r2r
d
b 0
h
b l4
2pa0 eikb

b ° eikx /b dx
0

2pa0 eikb ik l4 1


 (e  1) 
b ikb

2pa0 eikb A
 (i  1)   1 (1  i )
b 2
With phase factor this becomes
A1 2p
(1 + i )e id ( where d  (n  1)h )
2 l
The contribution of the remaining aperture is A1/2(1  i) (so that the sum of the two
parts when d  0 is A1).
Thus, the complete amplitude is
A1 A
(1  i )e i d + 1 (1  i )
2 2
and the intensity is
I  I 0 [( l  i )e i d  ( l  i )][( l  i )ei d  ( l  i )]

 I 0 [2  2  (1  i )2 ei d  (1  i )2 e i d ]

 I 0 [ 4  2iei d  2ie i d ]  I 0 ( 4  4 sin d)

Here I 0  A12 4 is the intensity of the incident light which is the same as the intensity
due to an aperture of infinite extent (and no recess). Now, I is maximum when
sin d  1
3p
or d  2kp 
2
l ¥ 3´
So, h ¦ k  µ¶
n 1§ 4
The maximum intensity Imax  8I0.
186 PART FIVE OPTICS

5.105 We follow the argument of Problem 5.103. We find that the contribution of the first
Fresnel zone is
4pi
A1   a0 eikb
k
For the next half zone it is
A2
 (1  i )
2
¥
¦§ The contribution of the remaining part of the second Fresneel zone will be

A2 ´
 (1  i ).µ
2 ¶
If the disk has a thickness h, the extra phase difference suffered by the light wave in
passing through the disk will be
2p
d (n  1)h
l
Thus, the amplitude at P will be
¥ A ´ A
E P  ¦ A1  2 (1  i ) µ ei d  2 (1  i )  A3  A4  A5  …
§ 2 ¶ 2
¥ A (1  i ) ´ i d iA1 A1
¦ 1 µe   [( l  i )ei d  i ]
§ 2 ¶ 2 2
The corresponding intensity will be
¥ ¥ p´´
I  I 0 (3  2cos d  2sin d)  I 0 ¦ 3  2 2 sin ¦ d  µ µ
§ § 4 ¶¶
The intensity will be a maximum when
¥ p´
sin ¦ d  µ  1
§ 4¶
p 3p
or d  2kp 
4 2
¥ 5´
i.e., d  ¦ k  µ – 2p
§ 8¶
l ¥ 5´
So, h ¦k  µ ( where k  0, 1, 2, …)
n 1§ 8¶
Note: It is not clear why k  2 for hmin. The normal choice will be k  0. If we take
k  0, we get hmin  0.59 Mm.
5.3 DIFFRACTION OF LIGHT 187

5.106 Here the focal point acts as a virtual source of light. This
means that we can take spherical waves converging to-
wards F. Let us divide these waves into Fresnel zones r P
just after they emerge from the stop. So, h F
r 2  f 2  ( f  h )2  (b  m l2)2  (b  h )2

(where r is the radius of the mth Fresnel zone and h is the distance to the left of the
foot of the perpendicular).
Thus, r 2  2 f h  bm l  2bh
So, h  bm l2 (b  f )
and r 2  fbm l(b  f )
The intensity maxima are observed when an odd number of Fresnel zones are ex-
posed by the stop. Thus,

kbf l
rk 
b f
 0.90 k mm ( where k  1, 3, 5, . . .)

5.107 For the radius of the periphery of the k th zone, we have


ab
rk  k l  k lb ( if a  @ )
ab
If the aperture diameter is reduced h times, it will produce a similar diffraction pat-
tern (reduced h times) if the radii of the Fresnel zones are also h times less. Thus,
r
rka  k
h
b
This requires b   1.0 m
h2

5.108 (a) If a point source is placed before an opaque ball, the diffraction pattern consists
of a bright spot inside a dark disk followed by fringes. The bright spot is on the
line joining the point source and the centre of the ball. When the object is a finite
source of transverse dimension y, every point of the source has its corresponding
image on the line joining that point and the centre of the ball. Thus, the trans-
verse dimension of the image is given by
b
y  y  9 mm
a
188 PART FIVE OPTICS

(b) The minimum height of the irregularities covering the surface of the ball at ran-
dom, at which the ball obstructs light is, according to the note at the end of the
problem, comparable with the width of the Fresnel zone along which the edge
of opaque screen passes.
So, hmin  $r
To find $r, we note that
k lab
r2 
a b
lab
or 2r $r  D $r  $k
a b
where D  diameter of the disk ( diameter of the last Fresnel zone) and
$k  1.
lab
Thus, hmin   0.099 mm
D (a  b )

5.109 In a zone plate, an undarkened circular disk is followed by a number of alternately un-
darkened and darkened rings. For the proper case, these correspond to 1st, 2nd, 3rd , …
Fresnel zones.
Let r1  radius of the central undarkened circle. Then for this to be the first Fresnel
zone in the present case, we must have
SL  LI  SI  l2
Thus, if r1 is the radius of the periphery of the first zone
l L
a 2  r12  b 2  r12  ( a  b ) 
2
r12 s
¥ 1 1´ l 1 1 1 S a b I
or ¦§  µ¶  or   2
2 a b 2 a b r1 l
It is clear that the plate is acting as a lens of focal length
r12 ab 1.5
f1    m  0.6 m
l a  b 2.5
This is the principal focal length. Other maxima are obtained when
l l
SL  LI  SI  3 , 5 , . . .
2 2
Thus, focal lengths are also
r12 r12 . . .
, ,
3 l 5l
5.3 DIFFRACTION OF LIGHT 189

5.110 Just below the ledge, the amplitude of the wave is given by
1 1
A  ( A1  A2  A3  A4  . . .)ei d  ( A1  A2  A3  A4  . . .)
2 2
Here the quantities in the brackets are the contributions of various Fresnel zones; the
factor 1/2 is to take account of the division of the plate into two parts by the ledge;
the phase factor d is given by
l
2p
d= h(n  1)
l
and takes into account the extra length traversed by the
waves on the left. h
A
Using A1  A2  A3  A4 . . .  1
2
A1
we get A (1  ei d )
4
and the corresponding intensity is
2
1  cos d ¥ ¥ A1 ´ ´
I  I0 ¦§ where I 0 ^ ¦§ µ¶ µ
2 2 ¶
(a) This is minimum when
cos d 1
So, d  (2k  1)p
l
and h  (2k  1) ( where k  0, 1, 2, . . .)
2(n  1)
Using n  1.5, l  0.60 Mm, we get
h  0.60 (2k  1) Mm
(b) Intensity will be twice as low as (I  I0 /2) when
cos d  0
p p
or d  kp   (2k  1)
2 2
Thus, in this case
h  0.30 (2k  1) Mm

5.111 (a) From Cornu’s spiral, the intensity of the first maximum is given as
I max  1.37 I 0
190 PART FIVE OPTICS

and the intensity of the first minimum is given by


I min  0.78 I 0
So, the required ratio is
I max
 1.76
I min
(b) The value of the distance x is related to the parameter n in Fresnel’s integral by
2
vx
bl
For the first two maxima, the distances x1, x2 are related to the parameters v1, v2
by
bl bl
x1  v1 and x2  v2
2 2
bl
Thus, (v2  v1 )  x2  x1  $x
2
2 x2
2 ¥ $x ´
or l b P
b ¦§ v2  v1 µ¶
From Cornu’s spiral, the positions of the maxima
are x1
v1  1.22, v2  2.34, v3  3.08, etc.
2
2 ¥ $x ´ I Screen
Thus, l ¦ µ  0.7 Mm
b § 1.12 ¶

5.112 We shall use the equation written down in Problem 5.103, the Fresnel-Huygen’s
formula.
Suppose we want to find the intensity at P which is such that the coordinates of the
edges (x-coordinates) with respect to P are x2 and x1. Then, the amplitude at P is
a0 ikr
E  ° K (w ) e dS
r
We write dS  dx dy, y is to be integrated from @ to @. So,
x2  y2
r b (1)
2b
where r is the distance of the element of surface on I from P. It is b 2  x 2  y 2 and
hence, approximately given by Eq. (1). We then get
5.3 DIFFRACTION OF LIGHT 191

x
¨@ 1
·
E  A0 (b ) © ° e ikx 2 b2
dx  ° eikx 2b dx ¸ 2

©ª x 2 @ ¸¹
n
¨@ 1
·
 A0a (b ) © ° eipu 2 du  ° eipu 2 du ¸
2 2

©ªn 2 @ ¸¹
2 2
where v2 
x2 and v1  x1
bl bl
The intensity is the square of the amplitude. In our case, at the centre
2 a a2
v1  v2  –   0.64
bl 2 2b l
(where a  width of the strip  0.7 mm, b  100 cm, l  0.60 Mm).
At, say, the lower edge v1  0, v2  1.28.
Thus,
@ 0.64 2 2
¥1 ´ ¥1 ´
° °
2 2
eipu 2 du  eipu 2 du ¦§  C (0.64)µ¶  ¦§  S ( 0.64 )µ¶
I centre
 0.64
@
@
0
4 2 2
I edge 1  C (1.28) 2  1  S (1.28) 2
° °
2 2
eipu 2 du  eipu 2 du
1.28 @

v
pu 2
where C (v )  ° cos du
0
2
v
pu 2
and S (v )  ° sin
du
0
2
Rough evaluation of the integrals using Cornu’s spiral gives
I centre
2.4
I edge
@ @
¥ pu 2 pu 2 1´
¦
§
We have used ° cos
2
du  ° sin
2
du  .µ

0 0

Thus, C (0.64)  0.62, S (0.64)  0.15


and C (1.28)  0.65, S (1.28 )  0.67

5.113 If the aperture has width h then the parameters (v, v) associated with (h/2, h/2)
are given by
h 2 h
v 
2 bl 2b l
192 PART FIVE OPTICS

The intensity of light at O on the screen is obtained as the square of the amplitude
A of the wave at O which is
v
h/2
A ~ constant °e ipu 2 /2 du
v
O
b
Thus, I  2I 0 [(C (v ))2  ( S (v ))2 ]
where C (v) and S (v) have been defined above and I0 is h/2
the intensity at O due to an infinitely wide (v  @) aper-
ture, for then
¥ ¥ 1´ 2 ¥ 1´ 2 ´ 1
I  2 I 0 ¦ ¦ µ  ¦ µ µ  2I 0   I 0
§
§ 2 ¶ § ¶
2 ¶ 2
By definition v corresponds to the first minimum of the intensity. This means
v  v1  0.90
When we increase h to h  $h, the corresponding second minimum of intensity
h  $h
v2 
2b l
From Cornu’s spiral v2  2.75.

Thus, $h  2b l(v2  v1 )  0.85 2b l


2 2
¥ $h ´ 1 ¥ 0.70 ´ 1
or l ¦ µ ¦ µ  0.565 Mm
§ 0.85 ¶ 2b § 0.85 ¶ 2  0.6

5.114 Let a  width of the recess and h


a 2 a
v 
2 bl 2b l A
a O
0.6
  0.60
2  0.77  0.65 V
be the parameter along Cornu’s spiral corresponding to
the half-width of the recess. The amplitude of the diffracted wave is given by
@
¨ v v
·
A constant ©e i d ° eipu /2 du  ° eipu /2 du  ° eipu /2 du ¸
2 2 2

ª v v @ ¹
where d  2pl(n  1)h is the extra phase due to the recess. (Actually an extra
phase eid appears outside the recess. When we take it out and absorb it in the con-
stant we get the expression written.)
5.3 DIFFRACTION OF LIGHT 193

¨ ¥1 ´ ¥1 ´·
Thus, A constant  ©C (v )  iS (v )e i d  ¦  C (v ) µ  i ¦  S (v ) µ ¸
ª §2 ¶ §2 ¶¹
From Cornu’s spiral, the coordinates corresponding to the parameter v  0.60 are
C v  0.57, S v  0.13. So, the intensity at O is proportional to
|[(0.57  0.13i )e id  0.07  0.37i ]|2
 (0.572  0.132 )  0.072  0.372
(0.57  0.13i )(0.07  0.37i )e id
(0.57 0.13i )(0.07  0.37i )ei d
So,
0.57 p 0.13i  0.585e p ia , a  12.8
0.07 p 0.37i  0.377e p ib , b  100.7
Thus, the cross-term is
2  0.585  0.377 cos ( d  88 )
¥ p´
 2  0.585  0.377 cos ¦ d  µ
§ 2¶
For maximum intensity
p
d   2k ap ( where k a  1, 2, 3, 4, . . .)
2
 2 (k  1)p ( where k  0, 1, 2, 3, . . .)
3p
or d  2kp 
2
l ¥ 3´
So, h ¦k  µ
n 1§ 4¶

5.115 Using the method of Problem 5.103 we can write down the amplitudes at 1 and 2 as,
@
¨0 ·
A1 ~ constant © ° eipu 2 du  ei d ° eipu 2 du ¸
2 2
h
ª@ v ¹
a
¨ v @
· b
A2 ~ constant © ° eipu 2 du  ei d ° eipu 2 du ¸
2 2

ª@ 0 ¹
Screen
where v  a 2b l 1 2
is the parameter of Cornu’s spiral and constant factor is common to 1 and 2.
194 PART FIVE OPTICS

Using the usual notation


v
pu 2
C  C (v )  ° cos du
0
2
v
pu 2
S  S (v )  ° sin du
0
2
@ @
pu 2 pu 2 1
and the result ° cos 2
du  ° sin
2
du 
2
0 0

We find the ratio of intensities as


2
¥1 ´ ¥1 ´ 1  i
¦§  C µ¶  i ¦§  S µ¶  e
i d
I2 2 2 2

I1 1 i «¥ 1 ´ ¥1 ´º
 ei d ¬¦  C µ  i ¦  S µ »
2 §
­ 2 ¶ § 2 ¶¼

(The constants in A1 and A2 must be the same by symmetry.)


In our case, a  0.30 mm, l  0.65 Mm, b  1.1 m .

2
So, v  0.30   0.50
1.1  0.65

Therefore, C (0.50)  0.48 and S (0.50)  0.06


2
(1  i )
0.02  0.44i  ei d 2

Also, I2 2 1  (0.02  0.44i ) 2e i d  ip4


 
I1 1  i id 1  (0.02  0.44i ) 2e i d  ip4
e  0.02  0.44i
2
But 0.02  0.44i  0.44eia and a  1.525 radians (  87.4  ).
2
I2 1  0.44  2  e i ( d  0.740 ) 1  2 (0.44)2  2 2  0.44 cos ( d  0.740) 2
So,  
I1 1  0.44  2  e  i ( d  0. 740 ) 1  2 (0.44)2  2 2  0.44 cos ( d  0.740)

I2 is maximum when d  0.740  0 ( modulo 2p ).


Thus, in that case

I2 1.387  1.245 2.632


   1.9
I 1 1.387  1.245 cos (1.48 ) 1. 5
5.3 DIFFRACTION OF LIGHT 195

5.116 We apply the formula of Problem 5.103 and calculate


a
° r0 eikr dS  °  °
Aperture Semicircle Slit

The contribution of the full first Fresnel zone has been evaluated in Problem 5.103.
The contribution of the semicircle is one-half of it and is
2p
 ia0 eikb  ia0 leikb
k
The contribution of the slit is
0.90 b l @
a0 2

° °
2
eikb eikx 2b dx eiky  2b dy
b 0 @
@ @

° iky 2  2b
°
2
Now, e dy  eipy b l dy
@ @

@
bl
° eipu 2 du  b l eip4
2
or
2 @
Thus, the contribution of the slit is
0.9  2
a0 bl
°
2
b l eikb  ip4 eipu 2 du
b 0
2
1.27
1 2
 a0 leikbip4
2
° eipu 2 du
0

Thus, the intensity at the observation point P on the screen is


2
1i
a02 l2 i  (C (1.27 )  iS (1.27))
2
2
(1  i )(0.67  0.65i )
 a02 l2 i 
2
(On using C (1.27)  0.67 and S (1.27)  0.65.)
2
 a02 l2| i  0.01  0.66i|
2
 a02 l2|0.01  1.66i|
 2.76 a02 l2
Now a02 l2 is the intensity due to half of first Fresnel zone and is therefore equal to I0.
(It can also be obtained by doing the x-integral over @ to  @.)
Thus, I  2.76I 0
196 PART FIVE OPTICS

5.117 From the statement of the problem, we know that the width of the slit  diameter of
the first Fresnel zone  2 bl, where b is the distance of the observation point from
the slit.
We calculate the amplitudes by evaluating the inte- 1 2 3
gral of Problem 5.103. We get
bl @
a0
b  °b l
A1  eikb eikx 2b dx ° eiky 2b dy
2 2

2 @
a0 ikb b l 2 2

2 ° 2
 e eipu 2 du  ° eipu 2 du
b 0

a0 l
 (1  i )eikb (C ( 2 )  iS ( 2 ))
2
bl @
a0
b  °b l
A2  eikb eikx 2b dx ° eiky 2b dy
2 2

@

 2 A1
a0 l(1  i )
A3   ia0 leikb  (C ( 2 )  iS ( 2 ))eikb
2
where the contribution of the first half Fresnel zone (in A3, first term) has been ob-
tained from the last problem.
2
Thus, (1  i )(0.53  0.72i )
I 1  a02 l2
2

( using C ( 2 )  0.53 and S ( 2 )  0.72.)


2
 a02 l2| 0.095  0.625i|  0.03996 a02 l2
I 2  4I 1
2
I 3  a02 l2| 0.095  0.625i  i|
2
 a02 l2| 0.095  1.625i|
 2.6496 a02 l2
So, I 3  6.6I 1
Thus, I1 : I 2 : I 3  1 : 4 : 7
5.3 DIFFRACTION OF LIGHT 197

5.118 The radius of the first-half Fresnel zone is bl2 and the amplitude at P is obtained
using Problem 5.103. So, we have

a0 ¨
h b l 2 @ ·
A © °  ° ¸ eikb  ikx 2b dx 2

b © @ ¸¹
ª h b l 2

@ b l 2
2 a0 ikb 2
P
° eiky  2b dy 
b
e ° eik r  2b 2prd r
@ 0

We use
h b l 2 @
2 2

° eikx  2b dx  ° eikx  2b dx
@ h b l 2

°
2
 eipx b l dx
h b l 2

@
2 bl
 ° eipu 2 du
h
2
@ h
bl ¥ ´
° °
2
 ¦  µe
ipu  2 du
2 §0 0 ¶

bl ¥ ¥ 1 ´ ¥1 ´´
 ¦§ ¦§  C (h)µ¶  i ¦§  S (h)µ¶ µ¶
2 2 2
Thus,

l ¨¥ 1 ´ ¥1 ´·
A  a0  2  ( l  i )eikb ©¦§ 2  C (h)µ¶  i ¦§ 2  S (h)µ¶ ¸  a0 l(1  i )e
ikb
2 ª ¹
where we have used
b l 2
2pib 2pb
°
2
eik r 2b 2pr d r  (1  i )  ( l  i )  lb (1  i )
0
k k

Thus, the intensity is


2
2
¨ ¥1 ´ ·
I |A|  a02 l2  2 ©(32  C (h))2  ¦  S (h)µ ¸
ª §2 ¶ ¹
198 PART FIVE OPTICS

From Cornu’s spiral,


C (h)  C (1.07 )  0.76
and S (h)  S (1.07 )  0.50
I  a02 l2  2  (0.74 )2  1.09a02 l2
As before I 0  a02 l2 so, I  I0

5.119 If a plane wave is incident normally from the left on a


slit of width b and the diffracted wave is observed at a
large distance, the resulting pattern is called Fraunhofer
diffraction. The condition for this is b 2  ll, where l is
the distance between the slit and the screen. In prac- x
u
tice, light may be focused on the screen with the help
of a lens (or a telescope).
Consider an element of the slit which is an infinite strip
of width dx. We use the formula of Problem 5.103 with $ = x sin w
the following modifications:
The factor 1/r characteristic of spherical waves will be omitted. The factor K(w) will
also be dropped if we confine ourselves to not too large w. In the direction defined
by the angle w the extra path difference of the wave emitted from the element at x
relative to the wave emitted from the centre is
$   x sin w
Thus, the amplitude of the wave is given by
b 2 ¥ i 12 kb sin w 1
i kb sin w ´
e e 2
a ° e ik sin w dx  ¦ µ
b 2
§ ik sin w ¶

sin (pbl sin w )



pbl sin w

sin 2 a
Thus, I  I0
a2
where a  pbl sin w and I0 is a constant.
Minima are observed for sin a  0 but a5
 0. Thus, we find minima at angles given
by

b sin w  k l ( where k  p 1, p 2, p 3, . . .)
5.3 DIFFRACTION OF LIGHT 199

5.120 Since I(a) is ve and vanishes for b sin w  k l, i.e., for a  kp, we expect maxima
of I(a) between a  p and a 2p, etc. We can get these values by
d sin a d sin a
( I (a))  I 0 2 0
da a da a
a cos a  sin a
or 0
a2
or tan a  a
Solutions of this transcendental equation can be obtained graphically. The first three
solutions are a1  1.43p, a2  2.46p, a3  3.47p, on the ve side.
Thus, b sin w1  1.43l
b sin w2  2.46l
b sin w3  3.47l
(On the negative side the solutions are a1 ,  a2 ,  a3 , . . . )
Asymptotically the solutions are
¥ 1´
b sin wm  ¦ m  µ l
§ 2¶

5.121 The relation b sin u  k l for minima (when light is incident normally on the slit) has
a simple interpretation: b sin u
is the path difference between
extreme rays emitted at angle u.
When light is incident at an angle u0
u0, the path difference is x u u
u
b(sin u  sin u0 )
and the condition of minima is
b (sin u  sin u0 )  k l
For the first minima
l
b (sin u  sin u0 )  p l or sin u  sin u0 p
b
Putting in values u0  30, l  0.50 Mm, b  10 Mm, we get
1 1
sin u  p = 0.55 or 0.45
2 20
and u+1  33 20a and u1  26  44a
200 PART FIVE OPTICS

5.122 (a) This case is analogous to the previous one except that the incident wave moves
in glass of R.I. n. Thus, the expression for the path difference for light diffracted
at angle u from the normal to the hypotenuse of the wedge is
b (sin u  n sin 1)
We write u  1  $u
Then, for the direction of Fraunhofer maximum u
u
b (sin (1  $u)  n sin 1)  0
or $u  sin1 (n sin 1)  1
Using 1  15, n  1.5, we get
$u  7.84 
(b) The width of the central maximum is obtained from
b (sin u1  n sin 1)  p l ( where l  0.60 Mm, b  10 Mm )

¥ l´
Thus, u+1  sin1 ¦ n sin 1  µ  26.63
§ b¶
¥ l´
and u1  sin1 ¦ n sin 1  µ  19.16 
§ b¶
Therefore, du  u+1  u1  7.47 

5.123 The path difference between waves reflected at A and B is


d(cos a0  cos a)
and for maxima
d (cos a0  cos a)  k l (k  0, p 1, p 2, . . .) a0 a a0 a
A d B
In our case, k  2 and a0 , a are small in radians. Then,

¥ a2  a02 ´
2l  d ¦ µ
§ 2 ¶
(a2  a02 )d
or l
4
 0.61 Mm
( for a  3p180, a0  p180 and d  103 m ).
5.3 DIFFRACTION OF LIGHT 201

5.124 The general formula for diffraction from N slits is


sin 2 a sin 2 N b
I  I0
a2 sin 2 b
pa sin u
where a
l
p (a  b ) sin u
b
l
and N  3 in the two cases given here.
(a) In this case a  b  2a
so, b  2a
sin 2 a
and I  I0 (3  4 sin 2 2a)2
a2
On plotting we get a curve that qualitatively looks like the given figure.
qI/I0
(b) In this case a  b  3a
so, b  3a

sin 2 a
and I  I 0 (3  4 sin 2 3a)2
a2
This has 3 minima between the
principal maxima. a

5.125 From the diffraction formula


d sin u  m l

we have d sin 45  2l1  2  0.65 Mm

or d  2 2  0.65 Mm
Then, for l2  0.50 in the third order

2 2  0.65 sin u  3  0.50


1.5
sin u   0.81602
1.3  2
This gives u  54.68   55
202 PART FIVE OPTICS

5.126 The diffraction formula is


d sin u0  n0 l
where u0  35 is the angle of diffraction corresponding to order n0 (which is not
yet known).
n0 l
Thus, d  n0  0.9327 Mm
sin u0
on using l  0.535 Mm.
For the nth order, we get
n n
sin u  sin u0  (0.573576 )
n0 n0
If n0  1, then n > n0 is at least 2 and sin u > 1, so, n  1 is the highest order of
diffraction.
If n0  2, then n  3, 4, but sin u > 1 for n  4, thus, the highest order of diffrac-
tion is 3.
If n0  3, then n  4, 5, 6. For n  6, sin u  2  0.57  1, so not allowed, while for
n  5, sin u  53  0.573576 1 is allowed. Thus, in this case the highest order of
diffraction is 5 as given. Hence, n0  3 and d  3  0.9327  2.7981  2.8 Mm .

5.127 Given that


d sin u1  l
and d sin u2  d sin (u1  $u)  2 l
Thus, sin u1 cos $u  cos u1 sin $u  2 sin u1
or sin u1 (2  cos $u)  cos u1 sin $u
sin $u
or tan u1 
2  cos $u
sin $u
or sin u1 
sin $u  (2  cos $u)2
2

sin $u

5  4 cos $u
d sin $u
Thus, l
5  4 cos $u
Substitution gives l  0.534 Mm
5.3 DIFFRACTION OF LIGHT 203

5.128 (a) Here the simple formula d sin u  m1 l holds.


m  0.530
Thus, 1.5 sin u  m  0.530 and sin u 
1.5
Highest permissible value of m  2 because sin u > 1 if m  3.
1.06
Thus, sin u  ( for m  2, u  45 )
1.50
(b) Here d (sin u0  sin u)  n l
u
nl
Thus, sin u  sin u0 
d
0.53 u
 sin 60  n 
1.5
 0.86602  n  0.353333
For n  5, sin u   0.900645.
For n  6, sin u  1. Thus, the highest order is n  5 and we get
u  sin1 (0.900645)   64

5.129 For the lens


1 ¥1 1´ R
 (n  1) ¦  µ or f 
f § R @¶ n 1
For the grating
l
d sin u1  l or sin u1 
d

d
cosec u1  , cot u1  (d l)2  1
l

1
and tan u1 
(dl)2  1

Hence, the distance between the two symmetrically placed first order maxima is

2R
2 f tan u1 
(n  1) ( d l)2  1

On putting R  20, n  1.5, d  6.0 Mm, l  0.60 Mm, we get this distance as 8.04 cm.
204 PART FIVE OPTICS

5.130 The diffraction formula is easily obtained on taking account of the fact that the opti-
cal path in the glass wedge acquires a factor n (refractive index). We get
d (n sin 1  sin (1uk ))  k l
Since n > 0, 1  u0 > 1 so, u0 must be negative. We get, using 1  30
3 1
  sin (30o  u0 )  sin 48.6o
2 2
1
Thus, u0  18.6 uk
Also, for k  1
3 l 0.5 1
 sin (30o  u1 )   
4 d 2.0 4
Thus, u+1  0
We calculate uk for various values of k by the above formula. For k  6
3
sin (uk  30o)   uk  78.6o
4
For k  7,
sin (uk  30o)  1  uk  120o

This is inadmissible. Thus the highest order that can be observed is k  6 corre-
sponding to uk  78.6.
(For k  7 the diffracted ray will be grazing the wedge.)

5.131 The intensity of the central Fraunhofer maximum will be zero if the waves from
successive grooves (not in the same plane) differ in phase by an odd multiple of p.
Then, since the phase difference is
2p
d (n  1)h
l
for the central ray, we have

2p ¥ 1´
(n  1)h  ¦ k  µ 2p ( where k  1, 2, 3, . . .)
l § 2¶
l ¥ 1´
or h ¦§ k  µ¶
n 1 2
The path difference between the rays 1 and 2 is approximately (neglecting terms of
order u2).
5.3 DIFFRACTION OF LIGHT 205

a sin u  a  na
a
 a sin u  (n 1)a u
u ~a
~a
Thus, for a maximum
2
¥ 1´ 1
a sin u  ¦ k a  µ l  m l
§ 2¶

¥ 1´
or a sin u  ¦ m  k a  µ l
§ 2¶
(where k a  0, 1, 2, . . . and m  0,  1,  2, . . . ).
The first maximum after the central minimum is obtained when m  k  0. We then
get,
1
a sin u1  l
2

5.132 When standing ultrasonic waves are sustained in the tank, they behave like a grating
whose grating element is equal to wavelength of the ultrasonic, i.e.,
v
d
n
and v  velocity of ultrasonic. Thus, for maxima
v
sin um  m l
n
On the other hand
f tan um  m$x

v
Assuming um to be small ( because l  ) , we get
n

f tan um f tan um lnf


$x   
m vnl sin um v

lnf
or v
$x
Putting the values l  0.55 Mm, n  4.7 MHz, f  0.35 m and $x  0.60  103 m,
we get v  1.51 kms.
206 PART FIVE OPTICS

5.133 Each star produces its own diffraction pattern in the


focal plane of the objective and these patterns super-
pose. The zeroeth order maxima of these patterns are
separated by angle #. As the distance d decreases the u
angle u between the neighboring maxima in either dif-
d
fraction pattern increases (sin u  l/d). When u be-
comes equal to 2#, the first deterioration of visibility #
occurs because the maxima of one system of fringes
coincide with the minima of the other system. Thus,
from the condition u  2# and sin u  l/d, we get
1 l
# u ( radians)
2 2d
 0.06(on substituting values)

5.134 (a) For normal incidence, the maxima are given by


d sin u  n l
l 0.530
So, sin u  n n
d 1.500
Clearly, n  2 as sin u  1 for n  3. Thus, the highest order is n  2. Then,
du k k 1
D  
d l d cos u d 1  (k ld )2

Putting k  2, l  0.53 Mm, d  1.5 Mm  1500 nm, we get


2 1 180
D   60  6.47 ang minnm
1500 ¥ 1.06 ´
2 p
1 ¦
§ 1.5 µ¶

(b) We write the diffraction formula as


d (sin u0  sin u)  k l
l
So, sin u0  sin u  k
d
Here u0  45 and sin u0  0.707, so
sin u0  sin u  1.707
l 0.53
Since   0.353333
d 1.5
we see that k  4.
5.3 DIFFRACTION OF LIGHT 207

Thus, highest order corresponds to k  4.


du
Now, as before D
dl
k kd
so, D 
d cos u 1 (k ld  sin u0 )2
 12.948 ang min/nm

5.135 We have d sin u  kl


du k tan u
so, D 
dl d cos u l

5.136 For the second order principal maximum


d sin u2  2 l  k l
Np
or d sin u2  2N p
l
Minima adjacent to this maximum occur at
Np
d sin(u2  $u)  (2N  1)p
l
l
or d cos u2 $u 
N
Finally, angular width of the second principal maximum is
2l
2$u 
Nd cos u2
2l tanu2
 
Nd 1  (k ld ) 2 N

 11.019 of arc (on substituting values)

l Nd sin u
5.137 Using R  kN 
dl l
l sin u

l
l
or R
l
208 PART FIVE OPTICS

5.138 For the just resolved waves, the frequency difference


c dl c c
dn   
l lR lkN
c 1
 
Nd sin u dt
since Nd sin u is the path difference between waves emitted by the extremities of the
grating.

5.139 It is given that dl  0.050 nm. So,


l 600
R   12000 ( nearly )
dl 0 – 05
On the other hand
d sin u  k l
l
Thus, sin u  l
kN
where l  102 m is the width of the grating.
l
Hence, sin u  12000 
l
 12000 s 600 s 107  0.72
or u  46

5.140 (a) We see that


N  6.5 s 10 s 200  13000
Hence, to resolve lines with dl  0.015 nm and l  670.8 nm, we must have
670.8
R  44720
0.015
Since 3N < R < 4N, one must go to the fourth order to resolve the said compo-
nents.
1
(b) We have d mm  5 Mm
200
k l k s 0.670
So, sin u  
d 5
5.3 DIFFRACTION OF LIGHT 209

Since sin u  1, we must have


k  7.46
d
so, kmax  7 
l
Nd l
Thus, Rmax  kmax N  91000  
l l
(where l  6.5 cm is the grating width).
l 670 l2
Finally, dlmin    0.007 nm  7 pm 
Rmax 91000 l

l 589.3
5.141 (a) Here, R   kN  5 N
dl 0.6
589.3 102
so, N 
3 d
3  102
d m  0.0509 mm
589.3
(b) To resolve a doublet with l  40.0 nm and dl  0.13 nm in the third order, we
must have
R 460
N   1179
3 3 s 0.13
This means that the grating is
Nd  1179  0.0509  60.03 mm wide = 6 cm

5.142 (a) From d  sin u  k l, we get


k dl
du 
d cos u
On the other hand, x  f sin u
kf
so, dx  f cos u du  dl
d
For f  0.80 m, dl  0.03 nm and d  1/250 mm, substituting we get
« 6 Mm if k 1
dx  ¬12 Mm if k  2
­
210 PART FIVE OPTICS

(b) Here N  25  250  6250


l 310.169
and   10339 – – N
dl 0.03
and so to resolve we need k  2 because k  1 gives an R.P. of only 6250.

5.143 Suppose the incident light consists of two wavelengths l and l  dl which are just
resolved by the prism. Then by Rayleigh’s criterion, the maximum of the line of wave-
length l must coincide with the first minimum of the line of wavelength l  dl. Let us
write both conditions in terms of the optical path differences for the extreme rays.
For the light of wavelength l C

bn  (DC  CE)  0
D E
For the light of wavelength l  dl
+ +
b (n  dn )  ( DC  CE )  l  dl A B
because the path difference between extreme rays equals l for the first minimum in
a single slit diffraction (from the formula a sin u  l).
Hence, b dn  l
l dn dn
and R b b
dl dl dl

l dn
5.144 (a) We know  R b  2Bbl3
dl dl
For b  5 cm, B  0.01 Mm 2 , l1  0.434 Mm  5  104 Mm, we get
R1  1.223  104
For l2  0.656 Mm, we get
R2  0.3542  104
(b) To resolve the D-lines we require
5893
R  982
6
0.02  b
Thus, 982 
(0.5893)3
982  (0.5893)3
b Mm  1.005  104 Mm  1.005 cm
0.02
5.3 DIFFRACTION OF LIGHT 211

dn
5.145 Given that b  kN  2  10, 000
dl
or b  0.10 Mm1  2  104
Thus, b  2  105 Mm  0.2 m  20 cm

5.146 Resolving power of the objective is


D 5 s 102
  7.45  104
1.22 l 1.22 s 0.55 s 106
Let ($y)min be the minimum distance between two points at a distance of 3.0 km
which the telescope can resolve. Then,
( $y )min 1.22 l 1
 
3 s 10 3 D 7.45 s 104
3 s 103
or ( $y )min   0.04026 m  4.03 cm
7.45 s 104

5.147 The limit of resolution of a reflecting telescope is determined by diffraction from the
mirror and obeys a formula similar to that from a refracting telescope. The limit of
resolution is
1 1.22 l ( $y )min
 
R D L
(where L  distance between the Earth and the Moon  384000 km).
Then, putting the values l  0.55 Mm, D  5.0 m, we get
($y )min  51.6 m

5.148 By definition, the magnification


angle subtended by the image at the eye #a
' 
angle subtended by the objectt at the eye #
At the limit of resolution
1.22 l
#
D
where D  diameter of the objective.
On the other hand, to be visible to the eye,
1.22 l
#a 
d0
212 PART FIVE OPTICS

where d0  diameter of the pupil.


Thus, to avail of the resolution offered by the telescope we must have
1.22 ld0 D
' 
1.22 lD d0
D 50 mm
Hence, ' min    12.5
d0 4 mm

5.149 Let A and B be two points in the field C


of a microscope which is represented
by the lens CD. Let Aand B be their 90aA B'
image points which are at equal dis-
tances from the axis of the lens CD.
B A'
Then all paths from A to A are equal
and the extreme difference of paths D
from A to B is equal to
ADB ACB
 AD  DB (AC  CB)
 AD  DB BD  DB  BC  CB AC  CB
(As BD  DB  BC  CB.)
 AD  BD  BC  AC
 2AB cos (90  a)  2AB sin a
From the theory of diffraction by circular apertures, this distance must be equal to
1.22l when B coincides with the minimum of the diffraction due to A and A with
the minimum of the diffraction due to B. Thus
1.22 l l
AB   0.61
2 sin a sin a
Here 2a is the angle subtended by the objective of the microscope at the object.
Substituting values
0.61 s 0.55
AB  Mm  1.40 Mm
0.24

5.150 Suppose dmin  minimum separation resolved by the microscope, #  angle sub-
tended at the eye by this object when the object is at the least distance of distinct
vision l0 ( 25 cm). The minimum angular separation resolved by the eye
1.22 l
# 
d0
5.3 DIFFRACTION OF LIGHT 213

From the previous problem


0.61l
dmin 
sin a
dmin 0.61l
and # 
l0 l0 sin a

angle subteneded at the eye by the image


'  magnifying power 
btended at the eye by the object
angle sub
#a
When the object is at the least distance of distinct vision  .
#
¥l ´ 25
Thus, ' min  2 ¦ 0 µ sin a  2   0.24  30
§ d0 ¶ 0.4

5.151 Path difference is given by


BC  AD  a (cos 60o  cos a) D
C
For diffraction maxima a
A a B
a (cos 60o  cos a)  k l 60
Since l  2/5a, we get
1 2
cos a   k
2 5
1 2
If k  1, cos a    0.9 and a  26o
2 5
1
If k  0, cos a   0.5 and a  60o
2
1 2
If k  1, cos a    0.1 and a  84o
2 5
1 4
If k  2, cos a     0.3 and a  107.5o
2 5
1 6
If k  3, cos a     0.7 and a  134.4o
2 5
Other values of k are not allowed as they lead to |cos a| > 1.
214 PART FIVE OPTICS

5.152 We give here a simple derivation of the conditions for diffraction maxima, known
as Laue equations. It is easy to see from the figure that the path difference between
waves scattered by nearby scattering centres P1 and P2 is
P2 A  P1 B  r – s0  r – s
 r – ( s0  s )  r – S s0
a cos a  h l A
P2 s0
b cos b  k l
Here, cos a and cos b are the direction cosines of
the ray with respect to the x- and y-axes of the two-
s0
dimensional crystal. So,
$x P1 B s0
cos a 
($x )2  4l 2

¥ $x ´
 sin ¦ tan1 µ  0.28735
§ 2l ¶
Using h  k  2, we get
40  2
a pm  0.278 nm
0.28735
$y ¥ $y ´
Similarly, cos b   sin ¦ tan1 µ  0.19612
( $ y )  4l
2 2 § 2l ¶

80
Thus, b pm  0.408 nm
cos b

5.153 Suppose a, b and g are the angles between the direction of the diffraction maximum
and the directions of the array along the periods a, b, and c, respectively (call them
x-, y-, and z-axes). Then the value of these angles can be found from the following
familiar conditions
a(1  cos a)  k1 l
b cos b  k2 l and c cos g  k3 l
where k1, k2, k3 are integers.
(These formulas are, in effect, Laue equations, see any text book on modern physics.)
Squaring and adding, we get, on using cos2 a  cos2 b  cos2 g  1
¨¥ k ´ 2 ¥ k ´ 2 ¥ k ´ 2 · 2k l
2  2 cos a  ©¦ 1 µ  ¦ 2 µ  ¦ 3 µ ¸ l2  1
©ª§ a ¶ §b¶ § c ¶ ¸¹ a
5.3 DIFFRACTION OF LIGHT 215

Thus, 2k1 a
l
;(k1 a)  (k2 a)2  (k3 a)2 =
2

Knowing a, b, c and the integers k1, k2, k3, we can find a, b, g as well as l.

5.154 The unit cell of NaCl is shown in the figure. In an infinite crystal, there are four Na
and four Cl ions per unit cell. (Each ion on the middle of the edge is shared by four
unit cells; each ion on the face centre by two unit cells, the ion in the middle of the
cell by one cell only and finally each ion on the corner by eight unit cells.) Thus,
M
4  r – a3
NA
(where M  molecular weight of NaCl in grams  58.5 g
NA  Avogadro’s number  6.023  1023 ).
1 M
Thus, a  2.822 Å
2 2NA r
The natural facet of the crystal is one of the faces of the
unit cell. The interplanar distance
1
d  a  2.822 Å
2
Thus, 2d sin a  2 l
3
So, l  d sin a  2.822 Å   244 pm
2

5.155 When the crystal is rotated, the incident monochromatic beam is diffracted from a
given crystal plane of interplanar spacing d, whenever in the course of rotation the
value of u satisfies the Bragg equation.
We have the equations
2d sin u1  k1 l and 2d sin u2  k2 l
But, p  2u1  p  2u2  a or 2u1  2u2  a
a
So, u2  u1 
2 u2
Thus,
u1
k1 = 2
« a aº a
2d ¬sin u1 cos  cos u1 sin »  k2 l
­ 2 2¼ p2u1
a ¥ a´ p2u2 k2 = 3
Hence, 2d sin cos u1  ¦ k2  k1 cos µ l
2 § 2¶
216 PART FIVE OPTICS

a a
Also 2d sin sin u1  k1 l sin
2 2
1/2
a ¥ 2 a´
Squaring and adding 2d sin  ¦ k1  k22  2k1k2 cos µ l
2 § 2¶
1/2
l ¥ 2 a´
Hence, d ¦§ k1  k2  2k1k2 cos µ¶
2
2 sin a2 2
Substituting a  60o, k1  2, k2  3, l  174 pm, we get
d  281 pm  2.81 Å
(Lattice parameters are typically in Ås and not in fractions of a pm so the answer is
not 0.281 pm as given in the book.)

5.156 In a polycrystalline specimen, microcrystals are oriented at various angles with re-
spect to one another. The microcrystals which are oriented at certain special angles
with respect to the incident beam produce diffraction maxima that appear as rings.
The radius of these rings are given by
r  l tan 2a
a
Bragg’s law gives l r
a 2a
2d sin a  k l
In this case k  2, d  155 pm, l  17.8 pm, so
17.8
a  sin1  6.6o and r  3.52 cm
155

5.4 Polarization of Light

5.157 Natural light can be considered an incoherent mixture of two plane polarized light
rays of intensity I0 /2 with mutually perpendicular planes of vibrations. The screen
consisting of the two polaroid half-planes acts
as an opaque half-screen for one or the other of
these light waves. The resulting diffraction pattern
has the alteration in intensity (in the illuminated
region) characteristic of a straight edge on both
sides of the boundary. boundary
At the boundary the intensity due to either component is (I0 /2)/4 and the total inten-
sity is I0 /4. (Recall that when light of intensity I0 is incident on a straight edge, the
illuminance in front of the edge is I0 /4.)
5.4 POLARIZATION OF LIGHT 217

5.158 (a) Assume first that there is no polaroid and the amplitude due to the entire hole
which extends over the first Fresnel zone is A1. Then, we know, as usual,
I 0  A12 4.
When the polaroid is introduced as shown above, each
half transmits only the corresponding polarized light. If
the full hole were covered by one polaroid, the ampli-
tude transmitted will be ( A1  2 ).
Therefore, the amplitude transmitted in the present case
will be A1 2 2 through either half. Since these trans-
mitted waves are polarized in mutually perpendicular
planes, the total intensity will be
2 2
¥ A1 ´ ¥ A1 ´ A12
¦§ µ¶  ¦§ µ¶  4  I 0
2 2 2 2
(b) We interpret the problem to mean that the two polaroid pieces are separated
along the circumference of the circle limiting the first half of the Fresnel zone.
(This however is inconsistent with the polaroids being identical in shape; how-
ever no other interpretation makes sense.)
From Problem 5.103 and the previous problems we see that the amplitudes of
the waves transmitted through the two parts is
A1 A1
(1  i ) and (1  i )
2 2 2 2
and the intensity is
2 2
A12 A1
(1  i )  (1  i )
2 2 2 2
A12
  2I 0
2

5.159 When the polarizer rotates with angular velocity v, its instantaneous principal direction
makes angle vt from a reference direction which we choose to be along the direction
of vibration of the plane polarized incident light. The transmitted flux at this instant is
&0 cos2 vt
and the total energy passing through the polarizer per revolution is
T
¥ 2p ´
° & cos
0
2 vt dt ¦ where T 
§ v¶
µ
0
p
 &0  0.6 mJ
v
218 PART FIVE OPTICS

5.160 Let I0  intensity of the incident beam. Then the intensity of the beam transmitted
through the first Nicol prism is
1
I1  I 0
2
and through the second prism is
¥1 ´
I 2  ¦ I 0 µ cos2 w
§2 ¶
Then, through the N th prism it will be
I N  I N  1 cos2 w
1
 I 0 cos2( N  1) w
2
Hence, fraction transmitted
IN 1
 h  cos( 2 N  1) w  0.12 ( for N  6)
I0 2
and w  30

5.161 When natural light is incident on the first polaroid, the fraction transmitted will be
1/2t (only the component polarized parallel to the principal direction of the polaroid
will go).
The emergent light will be plane polarized and on passing through the second pola-
roid will be polarized in a different direction (corresponding to the principal direction
of the second polaroid) and the intensity will have decreased further by t cos2 w.
In the third polaroid the direction of polarization will again have to change by w, thus
only a fraction t cos2 w will go through.
1
Finally I  I 0  t 3 cos4 w
2
Thus, the intensity will have decreased. So,
I0 2
 3  60.2 times
I t cos4 w
where t  0.81, w  60.

5.162 Suppose the partially polarized light consists of natural light of intensity I1 and plane
polarized light of intensity I2 with direction of vibration parallel to, say, x-axis.
Then when a polaroid is used to transmit it, the light transmitted will have a maxi-
mum intensity (1/2)I1  I2, when the principal direction of the polaroid is parallel
to x-axis, and will have a minimum intensity (1/2)I1 when the principal direction is
perpendicular to x-axis.
5.4 POLARIZATION OF LIGHT 219

I max  I min I2
Thus, P 
I max  I min I1  I 2
I2 P 0.25 1
So,   
I 1 1  P 0.75 3

5.163 If, as above, I1 is the intensity of natural component and I2 is the intensity of plane
polarized component, then
1
I max  I 1  I 2
2
I max 1
and I  I 1  I 2 cos2 w
h 2
¥ 1´
So, I 2  I max ¦ 1  µ cosec2 w
§ h¶

¨ ¥ 1´ · 2I ¨1 ·
I 1  2 I max ©1  ¦ 1  µ cosec2 w ¸  max ©  cos2 w ¸
ª § h¶ 2
¹ sin w ª h ¹
I2 1  1h h 1
Then, P  
I 1  I 2 2(1h  cos2 w )  1  1h 1  h cos 2w
On putting h  3.0, w  60, we get
2 4
P   0.8
1  3  12 5

5.164 Let us represent the natural light as a sum of two mutually perpendicular compo-
nents, both with intensity I0. Suppose that each polarizer transmits a fraction a1 of
the light with oscillation plane parallel to the principal direction of the polarizer and
a fraction a2 with oscillation plane perpendicular to the principal of the polarizer.
Then, the intensity of light transmitted through the two polarizers is
I   a12 I 0  a22 I 0
when their principal directions are parallel and
I >  a1a2 I 0  a2a1 I 0  2a1a2 I 0
when they are crossed. But,
I> 2a a 1
 2 1 22 
I a1  a2 h

a1  a2 h 1
So, 
a1  a2 h 1
220 PART FIVE OPTICS

(a) Now, the degree of polarization produced by either polarizer when used singly is
I  I min a1  a2
P0  max 
I max  I min a1  a2
(assuming, that a1 > a2).
h 1 9
Thus, P0    0.905
h 1 11

(b) When both the polarizers are used with their principal directions par-
allel, the transmitted light, when analyzed, has the maximum intensity,
I max  a12 I 0 and the minimum intensity, I min  a22 I 0 .
a12  a22 a1  a2 (a1  a2 )2
So, P  –
a12  a22 a1  a2 a12  a22

h 1 ¥ 2a1a2 ´
 ¦1  2 µ
h 1 § a1  a22 ¶

h 1 ¥ 1´ h2  1 1
 ¦ 1  µ   1  2  0.995
h 1 § h¶ h h

5.165 If the principal direction N of the Nicol prism is along A or B, the intensity of light
transmitted is the same whether the light incident is one with oscillation plane N1 or
N2. If N makes an angle dw with A as shown in the figure, then the fractional differ-
ence in intensity transmitted (when the light incident is N1 or N2) is
¥ $I ´ cos2 (90o  w2  dw )  cos2 (90o  w2  dw ) A
¦§ µ¶  N
I A cos2 (90o  w2)
dw
sin 2 (w2  dw )  sin 2 (w2  dw ) N1

sin 2w2 w/2
B
2 sin w2 – 2 cos w2dw w w/2
  4 cot dw
sin w2
2 2 N2
If N makes an angle dw ( w) with B, then
¥ $I ´ cos2 (w2  dw )  cos2 (w2  dw ) 2 cosw2 – 2 sinw2dw w
¦§ ¶ µ    4 tan dw
I B cos w2
2 cos w2
2 2
( $I / I ) A w
Thus, h  cot 2
( $I / I ) B 2
1
or w  2 tan1
h
This gives w  11.4 for h  100.
5.4 POLARIZATION OF LIGHT 221

5.166 Fresnel equations read


sin 2 (u1  u2 ) tan 2 (u1  u2 )
I >a  I > and I a  I 
sin 2 (u1  u2 ) tan 2 (u1  u2 )
At the boundary between vacuum and a dielectric u1 p u2, since by Snell’s law
sin u1  n sin u2 .
Thus I >a I > cannot be zero. However, if u1  u2  90, I a  0 and the reflected light
is polarized in this case. The condition for this is
sin u1  n sin u2  n sin(90o  u1 )
or tanu1  n u2
is called Brewster’s angle.
The angle between the reflected and refracted light is 90 in this case.

5.167 (a) From Fresnel’s equations


sin 2 (u1  u2 )
I >a  I >
sin 2 (u1  u2 )
and I a  0 at Brewsters angle
I >a  I > sin 2 (u1  u2 )
1
 I (sin u1 cos u2  cos u1 sin u2 )2
2
n
Now, tan u1  n, sin u1 
n 1
2

1
cos u1  , sin u2  cos u1
n2  1

and cos u2  sin u1


2
1 ¥ n 2  1´
Therefore, I >a  I ¦ 2
2 § n  1µ¶
Thus reflection coefficient
I >a
r
I
2
1 ¥ n2  1 ´
 ¦ 2  0.074
2 § n  1 µ¶
(on putting n  1.5).
222 PART FIVE OPTICS

(b) For the refracted light


2
1 ®« ¥ n2  1 ´ ®º
I >aa  I >  I >a  I ¬1  ¦ 2 µ »
2 ®­ § n 1¶ ®¼
1 4n2
 I 2
2 (n  1)2
1
and I a  I
2
at the Brewster’s angle.
Thus, the degree of polarization of the refracted light is
I aa I >aa (n2  1)2  4n2
P 
I aa I >aa (n2  1)2  4n2
(n2  1)2 r
 
2(n  1)  (n  1)
2 2 2 2 1 r
On putting r  0.074, we get P  0.080.

5.168 The energy transmitted is, by conservation of energy, the difference between incident
energy and the reflected energy. However the intensity is affected by the change of
the cross section of the beam by refraction. Let Ai, Ar, At be the cross sections of the
incident, reflected and transmitted beams, respectively.
Then Ai  Ar
cos r
and At  Ai I0
cos i
rI0
But at Brewster’s angle r  90  i, so i
r
At  Ai tan i  nAi r
(1  r ) I 0
Thus, It   0.721I 0
n

5.169 The amplitude of the incident component whose oscillation vector is perpendicular
to the plane of incidence is
A>  A0 sin w
and similarly A  A0 cos w
sin 2 (u1  u2 ) 2
Then, I >a  I 0 sin w
sin 2 (u1  u2 )
5.4 POLARIZATION OF LIGHT 223

2
¨ sin u1 cos u2  cos u1 sin u2 ·
 I0 © ¸ sin W
2

ª sin u1 cos u2  cos u1 sin u2 ¹


2
¨ n2  1 ·
 I0 © 2 2
¸ sin w
ª n  1¹
Putting n  1.33 for water, we get r  0.0386.

5.170 Since natural light is incident at the Brewster’s angle, the reflected light 1 is com-
pletely polarized and P1  1.
Similarly, the ray 2 is incident on glass air surface at Brewster’s angle (tan11/n) so 3
is also completely polarized. Thus, P3  1.
Now as in Problem 5.167b
r
P2   0.087 ( if r  0.080)
1 r
I0 1 I0
rI0

2 3 || → 21 I0 2
1 r(12r)I0
| → 2 (12r)I0

4
|| → 21 I0
1
| → 2 (12r)I0
Finally, as shown in the figure
12  12 (1  2r )2 2r (1  r )
P4    0.173
12  12 (1  2r ) 2 1  2r (1  r )

5.171 (a) In this case, from Fresnel’s equations


sin 2 (u1  u2 )
I >a  I >
sin 2 (u1  u2 )
2
¥ n2  1 ´
we get I1  ¦ 2 µ I 0  rI 0 (say)
§ n 1¶
Then I 2  (1  r ) I 0 , I 3  r(1  r ) I 0
(Here r is invariant under the substitution n → 1/n.)
224 PART FIVE OPTICS

Thus, I 4  (1  r )2 I 0
16n 4
 I 0  0.726 I 0
(n2  1)4
(b) Suppose r is the coefficient of reflection of the component of light whose electric
vector oscillates at right angles to the incidence plane. From Fresnel’s equations
2
¥ n2  1 ´
ra  ¦ 2 µ
§ n 1¶
Then, in the transmitted beam we have a partially polarized beam which is a
superposition of two (I and ) components with intensities
1 1
I 0 and I 0 (1  r a )2
2 2
1  (1  r a )2 (n2  1)4  16n 4 1  0.726
Thus, P    0.158
1  (1  r a )2 (n2  1)4  16n 4 1  0.726

5.172 (a) When natural light is incident on a glass plate at Brewster’s angle, the transmit-
ted light has
I 16n 4 I 0 a4 I 0
I\\aa 0 and Iaa  2 
2 (n  1)4 2 2
where I0 is the incident intensity (see Problem 5.171a).
After passing through the second plate, we find
1 1
I\\aaaa  I 0 and Iaaaa  ( a4 )2 I 0
2 2
1
Thus, after N plates I\\trans  I0
2
1
and Itrans  a4N I0
2
1  a4 N ¥ 2n ´
Hence, P
1 a4 N ¦§ where a  1  n2 µ¶

(b) We have a4  0.726 for n  3/2.


Thus, P (N  1)  0.158, P (N  2)  0.310 P (N  5)  0.663 and P (N  10)
 0.922

5.173 (a) We decompose the natural light into two components with intensity I\\  I0 /2 
I where first component has its electric vector oscillating parallel to the plane of
incidence and second component has the same perpendicular to it. By Fresnel’s
equations for normal incidence
5.4 POLARIZATION OF LIGHT 225

2 2
Ia sin 2 (u1  u2 ) ¥ u  u2 ´ ¥ n  1´
 lim  lim ¦ 1 µ ¦ r
I u1m 0 sin (u1  u2 )
2 u m 0 § u1  u2 ¶
1 § n  1µ¶
2
I\\a ¥ n  1´
Similarly, r¦
I\\ § n  1µ¶
2 I1 I3
Ia ¥ 0. 5 ´ 1 I4
Thus, r¦ µ   0.040
I § 2. 5 ¶ 25
(b) The reflected light at the first surface has the intensity I0
I2
I1  rI0
Then the transmitted light has the intensity
I2  (1  r)I0
At the second surface where light emerges from glass into air, the reflection coef-
ficient is again r because r is invariant under the substitution n → 1/n.
Thus, I 3  r (1  r ) I 0 and I 4  (1  r )2 I 0
For N lenses the loss in luminous flux is then
$&
 1  (1  r )2 N  0.335 ( for N  5)
&

5.174 Suppose the incident light can be decomposed into waves with intensity I\\ and I with
oscillations of the electric vectors parallel and perpendicular to the plane of incidence.
For normal incidence, we have from Fresnel equations
2 2
¥ u  u2 ´ ¥ n 1´ I0 I1
Ia  I ¦ 1 µ m I ¦ µ
§ u1  u2 ¶ § n 1¶
(where we have used sin u  u for small u).
2 I2
¥ n a  1´
Similarly, I\\a  I\\ ¦ I3
§ n a  1µ¶
Then the refracted wave will be
4n a 4n a
I\\aa I\\ and Iaa  I
(n a  1)2 (n a  1)2
At the interface with glass
2
¥ na  n ´
Iaaa Iaa ¦ (similarly for I\\aaa )
§ n a  n µ¶
226 PART FIVE OPTICS

we see that
Ia I aaa
  (if n a  n , similarly for \\ component)
I Iaa
This shows that the light reflected as a fraction of the incident light is the same on
the two surfaces if na  n .
Note: The statement of the problem given in the book is incorrect. Actual amplitudes
are not equal; only the reflectance is equal.

5.175 Here u1  45. So, we have


1 1 2 2
sin u2      0.4714
2 n 3 2 3
u2  sin1 0.4714  28.1
sin 2 (u1  u2 )
Hence, Ia  I
sin 2 (u1  u2 )
2
1 ¥ sin 16.9 ´ 1
 I0 ¦ µ  I 0  0.0923
2 § sin 73.1 ¶ 2
2
1 ¥ tan 16.9 ´ 1
Similarly, I\\a  I0 ¦ µ  I 0  0.0085
2 § tan 73.1¶ 2
(a) Degree of polarization P of the reflected light
0.0838
P  0.831
0.1008
(b) By conservation of energy
1
Iaa  I 0  0.9077
2
1
I\\aa I 0  0.9915
2
0.0838
Thus, P  0.044
1.8982

5.176 The wave surface of uniaxial crystal consists of two


sheets of which one is a sphere while the other is an
ellipsoid of revolution. The optic axis is the line joining
the points of contact.
5.4 POLARIZATION OF LIGHT 227

To make the appropriate Huygen’s construction we must draw the relevant section of
the wave surface inside the crystal and determine the directions of the ordinary and
extraordinary rays. The result is as shown in Fig. 42 (a, b and c) of the answer sheet.

5.177 In a uniaxial crystal, an unpolarized beam of light (or


even a polarized one) splits up into O (for ordinary)
and E (for extraordinary) light waves. The direction of O
vibration in the O and E waves are most easily specified
in terms of the O and E principal planes. The princi-
pal plane of the ordinary wave is defined as the plane
containing the O ray and the optic axis. Similarly, the
principal plane of the E wave is the plane containing u
the E ray and the optic axis. In terms of these planes the
following is true: E
The O vibrations are perpendicular to the principal of
the O ray while the E vibrations are in the principal
plane of the E ray.
When we apply this definition to the Wollaston prism we find the following:
When unpolarized light enters from the left, the O and E waves travel in the same
direction but with different speeds. The O ray on the left has its vibrations normal
to the plane of the paper and it becomes E ray on crossing the diagonal boundary
of the two prisms, similarly the E ray on the left becomes O ray on the right. In this
case Snells law is applicable only approximately. The two rays are incident on the
boundary at an angle u and in the right prism the ray which we have called O ray
on the right emerges at
¥n ´ ¥ 1.658 1 ´
sin1 ¦ E sin uµ  sin1 ¦   33.91
n
§ O ¶ § 1.486 2 µ¶

where we have used


nE  1.1658, nO  1.486 and u  30
Similarly, the E ray on the right emerges within the prism at
¥n ´
sin1 ¦ O sin uµ  26.62
§ nE ¶
This means that the O ray is incident at the boundary between the prism and air at
33.91  30  3.91
and will emerge into air with a deviation of
sin1 nO sin 3.91  sin1 (1.658 sin 3.91 )  6.49 
228 PART FIVE OPTICS

The E ray will emerge with an opposite deviation of


sin1 (nE sin (30  26.62 ))
 sin1 (1.486 sin 3.38  )  5.03
Hence, d  6.49   5.03  11.52
This result is accurate to first order in (nE  nO) because Snell’s law holds when
nE  nO.

5.178 The wave is moving in the direction of z-axis.


(a) Here E x  E cos ( vt  kz ) and E y  E sin ( vt  kz )

E x2 E y2
or  1
E2 E2
So the tip of the electric vector moves along a circle. For the right-handed coor-
dinate system, this represents circular anticlockwise polarization when observed
towards the incoming wave.
¥ p´
(b) Here, E x  E cos ( vt  kz ) and E y  E cos ¦ vt  kz  µ
§ 4¶
Ey 1 1
or  cos ( vt  kz )  sin (vt  kz )
E 2 2
2
¥ Ey 1 Ex ´ 1¥ E x2 ´
or ¦  µ  ¦1  2 µ
§ E 2 E ¶ 2§ E ¶
E y2
E x2 E y Ex 1
or 2 2

 2 2

E E E 2
This is clearly an ellipse. By comparing with the previous case (compare the
phase of Ey in the two cases) we see this represents elliptical clockwise polariza-
tion when viewed towards the incoming wave.
We write the equations as
¥ p´ p
E x  E y  2E cos ¦ vt  kz  µ cos
§ 8 ¶ 8
¥ p´ p
E x  E y  2E sin ¦ vt  kz  µ sin
§ 8 ¶ 8
2 2
¥ Ex  E y ´ ¥ Ex  E y ´
Thus, ¦§ 2 E cos p 8 µ¶  ¦§ 2 E sin p 8 µ¶  1

Since cos p 8 > sin p 8, the major axis is in the direction of the straight line y  x.
5.4 POLARIZATION OF LIGHT 229

(c) Here, E x  E cos ( vt  kz )


and E y  E cos ( vt  kz  p )   E cos ( vt  kz )
Thus, the tip of the electric vector traces the curve
E y   Ex
which is a straight line (y  x). It corresponds to plane polarization.

5.179 For quartz


nE 1.553 º
for l  589 nm
nO 1.544 »
¼
In a quartz plate cut parallel to its optic axis, plane polarized light incident normally
from the left divides itself into O and E waves which move in the same direction with
different speeds and as a result acquire a phase difference. This phase difference is
2p
d (nE  nO )d
l
where d  thickness of the plate. In general, this makes the emergent light ellipti-
cally polarized.
(a) For emergent light to experience only rotation of polarization plane
d  (2k  1)p (where k  0, 1, 2, 3 . . .)
l
For this d  (2k  1)
2(nE  nO )
0.589
 (2k  1) Mm
2  0.009
0.589
 (2k 1) mm
18
The maximum value of (2k  1) for which this is less than 0.50 is obtained from
0.50  18
 15.28
0.589
Then we must take
0.589
k  7 and d  15   0.4908 mm
18
(b) For circular polarization
p
d (modulo p)
2
p
i.e., d  ( 4k  1)
2
230 PART FIVE OPTICS

l 0.589
So, d  ( 4k  1)  ( 4k  1)
4(nE  nO ) 36

0.50  36
Now  30.56
0.589
The nearest integer less than this which is of the form 4k  1 is 29 for k  7. For
this value of k, d  0.4749 mm.

5.180 As in the previous problem, the quartz plate introduces a phase difference d between
the O and E components. When d  p/2 (modulo p), the resultant wave is circularly
polarized. In this case intensity is independent of the rotation of the rear prism. Now,
2p
d (nE  nO )d
l
2p
 0.009  0.5  103 m
l
9p
 (here l is in Mm)
l
For l  0.50 Mm, d  18p. The relevant values of d have to be chosen in the form

¥ 1´
¦§ k  µ¶ p
2
For k  17, 16, 15, we get l  0.5143 Mm, 0.5435 Mm and 0.5806 Mm.
These are the values of l which lie between 0.50 Mm and 0.60 Mm.

5.181 As in the previous two problems, the quartz plate will introduce a phase differ-
ence d. The light on passing through the plate will remain plane polarized only for
d  2kp or (2k  1)p. In the latter case the plane of polarization of the light incident
on the plate will be rotated by 90°, so light passing through the analyzer (which was
originally crossed) will be maximum. Thus, dark bands will be observed only for
those l for which d  2kp.
2p
Now d (nE  nO )d
l
2p
  0.009  1.5  103 m
l
27p
 ( here l is in Mm )
l
For l  0.55, we get d  49.09p.
5.4 POLARIZATION OF LIGHT 231

Choosing d  48p, 46p, 42p, we get l  0.5625 Mm, 0.5870 Mm, 0.6136 Mm and
0.6429 Mm. These are the only values of l between 0.55 Mm and 0.66 Mm. Thus there
are four bands.

2p
5.182 Here, d  0.009  0.25 m
l
4.5p
 ( here l is in Mm)
l
We see that, for l  428.6 nm, d  10.5p
for l  529.4 nm, d  8.5p
for l  692.3 nm, d  6.5p
These are the only values of l for which the plate acts as a quarter wave plate.

5.183 Between crossed Nicols, a quartz plate, whose optic axis makes 45° angle with the
principal directions of the Nicols, must introduce a phase difference of (2k  1)p
so as to transmit the incident light (of that wavelength) with maximum intensity. In
this case, the plane of polarization of the light emerging from the polarizer will be
rotated by 90° and will go through the analyzer undiminished. Thus, we write for
light of wavelength 643 nm
2p  0.009
d  d ( mm )  103
0.643  106
18pd
  (2k  1)p (1)
0.643
To nearly block the light of wavelength 564 nm we require
18pd
 (2k a )p (2)
0.564
We must have 2k > 2k  1. For the smallest value of d, we take 2k  2k  2.
Thus, 0.643 (2k  1)  0.564  (2k  2)
or 0.079  2k  0.564  2  0.643
or 2k  6.139
This is not quite an integer but is close to it. This means that if we take 2k  6
Eq. (1) can be satisfied exactly while Eq. (2) will hold approximately.
7  0.643
Thus, d  0.250 mm
18
232 PART FIVE OPTICS

5.184 If a ray traverses the wedge at a distance x below the joint, then the distance that the
ray moves in the wedge is 2x tan 1/2 and this causes a phase difference
2p 1
d (nE  nO ) 2x tan
l 2
1
between the E and O wave components of the ray. For a gen-
eral x the resulting light is elliptically polarized and is not
completely quenched by the analyzer polaroid. The condition
for complete quenching is
x
d  2kp  dark fringe
That for maximum brightness is
d  (2k  1)p  bright fringe
The fringe width is given by
l
$x 
2(nE  nO ) tan 1/2
l
Hence, (nE  nO ) 
2$x tan 12
¥ 1´
Using tan ¦ µ  tan 175  0.03055 l  0.55 Mm and $x  1 mm, we get
§ 2¶

nE  nO  9.001  103

5.185 Light emerging from the first polaroid is plane polarized N1 Optic
with amplitude A where A2  I0 /2. N1 is the principal di- axis
N2
rection of the polaroid and a vibration of amplitude can be w
resolved into two vibrations: E wave with vibration along w
the optic axis with amplitude A cos w and the O wave
with vibration perpendicular to the optic axis and having
an amplitude A sin w. These acquire a phase difference d
on passing through the plate. The second polaroid trans- N2
mits the components A cos w cos wa and A sin w sin wa. N1

What emerges from the second polaroid is a set of two plane polarized waves in
the same direction and same plane of polarization but with phase difference d. They
interfere and produce a wave of amplitude squared
R 2  A2 [cos2 w cos2 wa  sin 2 w sin 2 wa  2 cos w cos wa sin w sin wa cos d]
Using cos2 (w  w a )  (cos w cos w a  sin w sin w a )2
 cos2 w cos2 w a  sin 2 w sin 2 w a  2 cos w cos w a  2 cos w cos w a sin w sin w a
5.4 POLARIZATION OF LIGHT 233

we easily find
¨ d·
R 2  A2 ©cos2 (w  w a )  sin 2w sin 2w a sin 2
ª 2 ¹̧
Now A2  I0 /2 and R2  I, so the result is

1 ¨ 2 d·
I I 0 ©cos (w  w a )  sin 2w sin 2w a sin 2
2 ª 2 ¹̧
Special cases:
Crossed polaroids: Here w  w  90° or w  w  90° and 2w  2w  180°. Thus,
in this case
1 d
I  I  I 0 sin 2 2w sin 2
2 2
Parallel polaroids: Here w  w. Thus,
1 ¥ d´
I  I\\  I 0 ¦ 1  sin 2 2w sin 2 µ
2 § 2¶
2p
With d  $, the conditions for the maximum and minimum are easily found to be
l
those shown in the answer sheet.

5.186 Let the circularly polarized light be resolved into plane N


polarized components of amplitude A0 with a phase e
v E wave
difference p/2 between them. wa w
O
On passing through the crystal, the phase difference
becomes d  p/2 and the components of the E and O
waves in the direction N are, respectively,
A0 cos w and A0 sin w
They interfere to produce a wave of amplitude squared
¥ p´
R 2  A02 cos2 w  A02 sin 2 w  2 A02 cos w sin w cos ¦ d  µ
§ 2¶
 A02 (1  sin 2w sin d)
Hence, I  I 0 (1  sin 2w sin d)
Here I0 is the intensity of the light transmitted by the polaroid when there is no crys-
tal plate.
234 PART FIVE OPTICS

5.187 (a) The light with right circular polarization (viewed against the oncoming light, this
means that the light vector is moving clockwise) becomes plane polarized on
passing through a quarter wave plate. In this case the direction of oscillations of
the electric vector of the electromagnetic wave forms an angle of 45° with the
axis of the crystal OO (see Fig. a). In the case of left-hand circular polarizations,
this angle will be 45° (see Fig. b).
(b) If for any position of the plate the rotation of the polaroid (located behind the
plate) does not bring about any variation in the intensity of the transmitted light,
the incident light is unpolarized (i.e., natural). If the intensity of the transmitted
light can drop to zero on rotating the analyzer polaroid for some position of the
quarter wave plate, the incident light is circularly polarized. If it varies but does
not drop to zero, it must be a mixture of natural and circularly polarized light.

O O

45 45

O'
O
(a) (b)

5.188 (a) The light from P is plane polarized with its electric vector vibrating at 45° with the
plane of the paper. At first the sample S is absent. Light from P can be resolved into
components vibrating in and perpendicular to the plane of the paper. The former is
the E ray in the left half of the Babinet compensator (B.C.) and the latter is the O ray.
In the right half the nomenclature it is the opposite. In the compensator the two
components acquire a phase difference which depends on the relative position of
the ray. If the ray is incident at a distance x above the central line through the com-
pensator then the E ray acquires a phase
2p
[nE (l  x )  nO (l  x )] tan 1 1
l
while the O ray acquires E

2p
[nO (l  x )  nE (l  x )] tan 1
l 1
so the phase difference between the two rays is
2p P S B.C. A
(nO  nE )2x tan 1  d
l
5.4 POLARIZATION OF LIGHT 235

We get dark fringes whenever d  2kp because

}
then the emergent light is the same as that coming
from the polarizer and is quenched by the analyzer.
(If d  (2k  1)p, we get bright fringes because in l
this case, the plane of polarization of the emergent x
light has rotated by 90° and is therefore fully trans-
mitted by the analyzer.)
It follows that the fringe width $x is given by
l
$x 
2 nE  nO 1

(b) If the fringes are displaced upwards by dx, then the path difference introduced
by the sample between the O and the E rays must be such so as to be exactly
cancelled by the compensator. Thus,
2p
[ d (nOa  nEa )  (nE  nO )2dx tan 1]  0
l
or d (nOa  nEa )   2(nE  nO )dx 1

using tan1  1.

5.189 Light polarized along the x-direction (i.e., one whose electric vector has only an x-
component) and propagating along the z-direction can be decomposed into left and
right circularly polarized light in accordance with the formula
1 1
E x  ( E x  iE y )  ( E x  iE y )
2 2
On passing through a distance l of an active medium these acquire the phases
2p 2p
dR  nR l and dL  nL l
l l
So we get for the complex amplitude
1 1
E a  ( E x  iE y )e i d  ( E x  iE y )e i d
R L

2 2
dR  dL
i ¨1 1 i d 2 ·
e 2
©ª 2 ( E x  iE y )e  2 ( E x  iE y )e
i d 2
¹̧
dR  dL
i ¨ d d·
e 2
©ª E x cos 2  E y sin 2 ¹̧ ( where d  dR  dL )

Apart from an overall phase (dR  dL )/2 (which is irrelevant) this represents a wave
whose plane of polarization has rotated by
236 PART FIVE OPTICS

d p
 ($n )l ( where $n |nR  nL |)
2 l
By definition this equals al, so
al
$n 
p
589.5  106 mm  21.72 degmm p
  ( radians)
p 180
0.5895  21.72
  103
180
 0.71  104

5.190 Plane polarized light, on entering the wedge, decomposes into right and left circu-
larly polarized light which travels with different speeds in prism(P) and the emergent
light gets its plane of polarization rotated by an angle which depends on the distance
travelled. 1
Given that $x  fringe width, $x tan u  difference
in the path length traversed by two rays which form
successive bright or dark fringes.
2p
Then, |nR  nL| $x tan u  2p
l
p$n p
Thus, a 
l $x tan 1 P Pol
 20.8 ang degmm
Let x  distance on the polaroid Pol as measured from a maximum. Then, a ray that
falls at this distance traverses an extra distance equal to
p x tan u
and hence a rotation of
px
 ax tan u  
$x
By Malus law the intensity at this point will be
¥ px ´
cos2 ¦
§ $x µ¶

5.191 If I0  intensity of natural light, then (1/2)I0  intensity of light emerging from the
polarizer Nicol (N1).
5.4 POLARIZATION OF LIGHT 237

Suppose the quartz plate rotates this light by w, then the analyzer (N2) will transmit
1
I 0 cos2 (90  w )
2
N1
1
 I 0 sin 2 w
2
of this intensity. w
90˚w
1 N2
Hence, hI 0  I 0 sin 2 w
2
or w  sin1 2h
But w  ad
1 1
so, dmin 
sin 2h
a
For minimum d we must take the principal value of inverse sine. Thus, using a  17
ang deg /mm we get dmin  2.99 mm.

5.192 For light of wavelength 436 nm


41.5  d  k  180  2k  90
(Light will be completely cut off when the quartz plate rotates the plane of polariza-
tion by a multiple of 180°.) Here d  thickness of quartz plate in mm.
For natural incident light, half the light will be transmitted when the quartz rotates
light by an odd multiple of 90°. Thus,
31.1  d  (2k a  1)  90
41.5 4
Now,  1.3344 
31.1 3
Thus, k  2 and k  1
4  90
so, d  8.67 mm
41.5

5.193 Two effects are involved here: rotation of plane of polarization by sugar solution and the
effect of that rotation on the scattering of light in the transverse direction. The latter is
shown in the given figure. It is easy to see from the figure that there will be no scattering
of light in this transverse direction if the incident light has its electric vector parallel to the
line of sight. In such a situation, we expect fringes to occur in the given experiment.
From the given data we see that in a distance of 50 cm, the rotation of plane of po-
larization must be 180°.
238 PART FIVE OPTICS

Thus, the specific rotation constant of sugar is


rotation constant
concentration
18050 e
 angdegcm
500 gl
180

5.0 dm  (0.500 gcm 3
No vibration along
 72 ang deg(dm – gcm ) 3 line of sight

5.194 (a) In passing through the Kerr cell, the two perpendicular components of the elec-
tric field will acquire a phase difference. When this phase difference equals 90°,
the emergent light will be circularly polarized because the two perpendicular
components O and E have the same magnitude since it is given that the direc-
tion of electric field E in the capacitor forms an angle of 45° with the principal
directions of the Nicols. In this case the intensity of light that emerges from this
system will be independent of the rotation of the analyzer prism.
Now the phase difference introduced is given by
2p
d (nE  nO )l
l
In the present case
p
d (for minimum electric field)
2
l
So, nE  nO 
4l
Now nE  nO  B lE 2

1
so, E min   105  88  10.66 kVcm
4 Bl
(b) If the applied electric field is
E  E m sin vt (where v  2pn )
then the Kerr cell introduces a time varying phase difference
d  2pB l Em2 sin 2 vt
 2p  2.2  1010  10  (50  103 )2 sin 2 vt
 11p sin 2 vt
In one half-cycle (i.e., in time pv  T 2  12n ) this reaches the value 2kp
when
5.4 POLARIZATION OF LIGHT 239

2 4 6 8 10 2 4 6 8 10
sin 2 vt  0, , , , , , , , , ,
11 11 11 11 11 11 11 11 11 11
i.e., 11 times. On each of these occasions light will be interrupted. Thus, light
will be interrupted
2n  11  2.2  108 times per second
[Light will be interrupted when the Kerr cell (placed between crossed Nicols)
introduces a phase difference of 2kp and in no other case.]

5.195 From Problem 5.189, we know that


al
$n 
p
where a is the rotation constant. Thus,
2a 2ac
$n  
2pl v
On the other hand, amag  VH
Thus, for the magnetic rotations
2cVH
$n 
v

5.196 Part of the rotation is due to Faraday effect and part of it is ordinary optical rotation.
The latter does not change sign when magnetic field is reversed. Thus,
w1  al  VlH
and w2  al  VlH
Hence, 2VlH  (w1  w2 )
(w1  w2 )2
or V
lH
Putting the values, we get
510 ang min
V  103  0.015 ang minA
2  0.3  56.5

5.197 As per the problem,


w  wchem  wmag
We look against the transmitted beam and count the positive direction clockwise. The
chemical part of the rotation is annulled by reversal of wave vector upon reflection.
240 PART FIVE OPTICS

Thus, wchem  al
since in effect there is a single transmission. On the other hand
wmag   NHVl
To get the signs right, recall that dextrorotatory compounds rotate the plane of vibra-
tion in a clockwise direction on looking against the oncoming beam. The sense of
rotation of light vibration in Faraday effect is defined in terms of the direction of the
field; positive rotation being that of right handed screw advancing in the direction of
the field. This is the opposite of the definition of wchem for the present case.
Finally,
w  ( a  VNH )l
Note: If plane polarized light is reflected back and forth through the same active me-
dium in a magnetic field, the Faraday rotation increases with each traversal.

5.198 There must be a Faraday rotation by 45° in the opposite direction so that light could
pass through the second polaroid. Thus,
VlH min  p4

p4 45  60
H min   Am
Vl 2.59  0.26
 4.01 kAm
If the direction of magnetic field is changed then the sense of rotation will also
change. Light will be completely quenched in the above case.

5.199 Let r  radius of the disk, then its moment of inertia about its axis  12mr 2 . In time
t, the disk will acquire an angular momentum
I
t – pr 2 –
v
when circularly polarized light of intensity I falls on it. By conservation of angular
momentum, this must equal to
1
mr 2 – v0
2
where v0  final angular velocity. Equating
mvv0
t
2pI
5.5 DISPERSION AND ABSORPTION OF LIGHT 241

v c mc v0
But n  so, t
2p l Il
Substituting the values of the various quantities, we get
t  11.9 h

5.5 Dispersion and Absorption of Light

5.200 In a travelling plane electromagnetic wave, the intensity is simply the time averaged
magnitude of the Poynting vector i.e.,
d0 2
I < EH >< E >  < c d0 E 2 >
m0
1
on using c and E d0  H m0
d0 m0

(see Section 4.4 of the book).


Now, time averaged value of E 2 is E02 2, so
1 2I
I  cd0 E02 or E0 
2 c d0

(a) Represent the electric field at any point by E  E0 sin vt. Then, for the electron
we have the equation
mx  eE 0 sin vt
eE0
So, x  sin vt
m v2
The amplitude of the forced oscillation is
eE0 e 2I
  5.1  1016 cm
mv 2 m v2 c d0
The velocity amplitude is clearly
eE0
 5.1  1016  3.4 1015  1.73 cms
mv
(b) For the electric force, amplitude of the electric force

Fele  eE0
For the magnetic force (which we have neglected above), it is
242 PART FIVE OPTICS

Fmag  (evB)

 (ev m0 H )

E
 evE d0 m0  ev
c
Writing v  v0 cos vt

eE 0
where v0 
mv
We see that the magnetic force is (neglecting the sign)
evE 0
Fmag  sin 2vt
2c
Hence, the ratio of amplitudes of the two forces
Fmag v0
  2.9  1011
Fele 2c
This is negligible and justifies ignoring the magnetic field of the electromagnetic
wave in calculating v0.

5.201 (a) It turns out that the spatial dependence of the electric field as well as the mag-
netic field is negligible. Thus, for a typical electron
mr  e E0 sin vt

e E0
So, r  sin vt (neglecting any non-sinusoidal part)
m v2
The ions will be practically unaffected. Then,
n0 e 2
P  n0 e r   E0
m v2

¥ n e2 ´
and D  d0 E 0  P  d0 ¦ 1  0 2 µ E 0
§ d0mv ¶

Hence, the permittivity


n0 e 2
d 1
d0 m v 2
5.5 DISPERSION AND ABSORPTION OF LIGHT 243

(b) The phase velocity is given by


v c
v 
K d

vP2 ¥ n e2 ´
2  0
So, ck  v 1  ¦§ where v
d0m µ¶
P
v2
or v2  c 2k 2  vP2

vP2 ¥ n0 e 2 ´ 2
Thus, v  c 1  c 1  ¦§ 4p 2mc 2d µ¶ l
c 2k 2 0

5.202 From the previous problem


n0 e 2
n2  1 
d0mv2
n0 e 2
1
4p 2d0mn 2

Thus, n0  ( 4p 2n 2md0 e 2 )(1  n2 )  2.36  107 cm3

5.203 For hard x-rays, the electrons in graphite will behave as if nearly free and the formula
of previous problem can be applied. Thus,
n0 e 2
n2  1 
d0mv2
n0 e 2
and n  1
2d0mv2
on taking square root and neglecting higher order terms.
n0 e 2 n e 2 l2
So, n  1   20
2d0mv 2 8p d0me 2
We calculate n0 as follows: There are 6  6.023  1023 electrons in 12 g of graphite
of density 1.6 g/cm3. Thus,
6  6.023  1023
n0  cm3
 .6 )
(121
Using the values of other constants and l  50  1012 m, we get
n  1  5.4  107
244 PART FIVE OPTICS

5.204 (a) The equation of the electron can (under the stated conditions) be written as
mx  gx  kx  eE0 cos vt
5
To solve this equation we shall find it convenient to use complex displacements.
Consider the equation
mz  gz  kz  eE0 ei vt
Its solution is
eE0 ei vt
z
mv2  igv  k
(We ignore transients.)
g k
Now, b and v02 
2m m
( eE0 m )ei vt
We find z
( v02  v2  2i bv)
Now x  Real part of z
eE0 cos ( vt  w )
 –  a cos ( vt  w )
m ( v0  v2 )2  4 b2
2 2 v2

2bv
where tan w  and
v  v02
2

2bv
sin w 
( v  v02 )2  4 b2
2 2 v2

(b) We calculate the power absorbed as

P  < Fx >  < eE0 cos vt (va sin ( vt w )) >

eE0 1 2 bv
 eE0 – – 2 –v
m 2 ( v0  v2 )2  4 b2 v2
2
¥ eE ´ bmv2
¦ 0µ
§ m ¶ ( v02  v2 )2  4 b2 v2

This is clearly maximum when v0  v because P can be written as


2
¥ eE ´ bm
P ¦ 0µ
§ m ¶ [( v02 v)  v ]2  4 b2
5.5 DISPERSION AND ABSORPTION OF LIGHT 245

2
m ¥ eE0 ´
and Pmax  ¦ µ ( for v  v0 )
4b § m ¶
P can also be calculated from
P   g x – x 

¥ gv2 a 2 ´ bmv2 (eE0 m )2


¦ µ  2
§ 2 ¶ ( v0  v2 )2  4 b2 v2

5.205 Let us write the solutions of the wave equation in the form
A  A0 e i ( vt  kx )
where k  2pl and l is the wavelength in the medium. If n a  n  i x, then
2p
k na
l0
(where l0 is the wavelength in vacuum) and the equation becomes
A  A0 e x ax exp[i ( vt1  k ax )]

2p 2p
where xa  x and ka  n
l0 l0
In real form, A  A0 e x ax cos ( vt  k ax )
This represents a plane wave whose amplitude diminishes as it propagates to the
right (provided x < 0). When n  ix, then similarly
A  A0 e x ax cos vt
(on putting n  0 in the above equation).
This represents a standing wave whose amplitude diminishes as one goes to the right
(if x < 0). The wavelength of the wave is infinite (k  0).
Waves of the former type are realized inside metals as well as inside dielectrics when
there is total reflection (penetration of wave).

5.206 In the plasma radio, waves with wavelengths exceeding l0 are not propagated.
We interpret this to mean that the permittivity becomes negative for such waves.
Thus,

n0 e 2 ¥ 2pc ´
0 1 ¦§ if v  l µ¶
d0 m v 2 0
246 PART FIVE OPTICS

n0 e 2 l02
Hence, 1
4p 2d0mc 2
4p 2d0mc 2
or n0   1.984  109 cm3
e 2 l02

5.207 By definition
v
dv
u
dk
dv

d
(vk ) (as v  vk )
l 
dl ll

dk

v k
dv
dk
v'
{ l l
2p 2p
Now, k so, dk   2 d l
l l
dv
Thus, uv l
dl
Its interpretation is the following:
The slope of the v  l curve at l  l is
¥ dv ´
¦§ µ¶
d l l  la
Thus, as is obvious from the figure
¥ dv ´
v a  v ( la )  la ¦ µ
§ d l¶ l  la

is the group velocity for l  l.

5.208 (a) Given that v  a l ( a is a constant )


dv
Then, uv  l
dl
a ¥ 1 ´ 3 a 3
  l ¦ a l32 µ  –  v
l § 2 ¶ 2 l 2
(b) Given that v  bk  k (b is a constant )
So, v  bk 2
5.5 DISPERSION AND ABSORPTION OF LIGHT 247

dv
and u  2bk  2v
dk
c ¥ v´
(c) Given that v ¦§ c is a constant  µ¶
v2 k
So, v3  ck or v  c13 k13
dv 1 1v 1
Thus, u  c 13 k23   v
dk 3 3k 3

5.209 We have
v dv
uv   c2
k dk
Integrating we find
v2  A  c 2 k 2 ( A is a constant )

v2  A
So, k
c
v c
and v
k 1  Av2

Writing this as c d( v) , we get


A
d( v)  1 
v2
(A can be ve or ve.)

5.210 The phase velocity of light in the vicinity of l  534 nm  l0 is obtained as


c 3  108
v ( l0 )    1.829  108 ms
n( l0 ) 1.640
To get the group velocity we need to calculate (dnd l) l  l . We shall use linear inter-
0
polation in the two intervals. Thus,
¥ dn ´ 0.007
¦§ µ¶    28  105 nm1
d l l  521.5 nm 25

¥ dn ´ 0.01
¦§ µ¶    18.2  105 nm1
d l l  561.5 nm 55
248 PART FIVE OPTICS

The (dn/dl) values have been assigned to the mid-points of the two intervals.
Interpolating again, we get

¥ dn ´ ¨ 9. 8 ·
¦§ µ¶  ©28   12.5 105 nm1
d l l  534 nm ª 40 ¹̧

  24.9  105 nm1

c d ¥ c´ c ¨ l ¥ dn ´ ·
Finally u l ¦§ µ¶  ©1  ¦§ µ¶ ¸
n dl n nª n dl ¹

At l  534 nm

3  108 ¨ 534 5 ·
u ©ª1  1.640  24.9  10 ¸¹
1.640

 1.59  108 ms

5.211 We write
v
v  a  bl
k
¥ 2p ´
so, v  k (a  b l)  2pb  ak ¦§ since k  µ

Suppose a wave train at time t  0 has the form

F ( x , 0)  ° f (k )e ikx dk

Then, at time t it will have the form

F ( x , t )  ° f (k )ikxi vt dk

 ° f (k )e ikxi ( 2pbak )t dx

 ° f (k )e ik ( x  at ) ei 2pbt dk

At t  1b  t
F ( x , t )  F ( x  at , 0 )
So at time t  t, the wave train has regained its shape though it has advanced by at.
5.5 DISPERSION AND ABSORPTION OF LIGHT 249

5.212 On passing through the first (polarizer) Nicol, the intensity of light becomes (1/2)I0
because one of the components has been cut off. On passing through the solution,
the plane of polarization of the light will rotate by
w V l H
and its intensity will also decrease by a factor exl. The plane of vibration of the light
wave will then make an angle 90°  w with the principal direction of the analyzer Ni-
col. Thus by Malus law, the intensity of light coming out of the second Nicol will be
1  xl
I 0 e – cos2 (90  w )
2
1  xl
 I 0 e sin 2 w
2

5.213 (a) The multiple reflections are shown in the figure below. Transmission gives a factor
(1  r) while reflections give a factor of r. Thus the transmitted intensity, assum-
ing incoherent light, is
rI0
I0
(1  r )2 I 0  (1  r )2 r 2 I 0  (1  r )2 r 4 I 0  . . .
 (1  r )2 I 0 (1  r 2  r 4  r 6  . . .)
r(1r)I0
1 ¥ (1  r )2 ´ I0(1r) r2(1r)I0
 (1  r )2 I 0   ¦ I0
1 r 2
§ (1  r )2 µ¶
(b) When there is absorption, we pick up a factor
s  exdd in each traversal of the plate. (1r)2I0 r2(1r)2I0
Thus, we get
(1  r )2 sI 0  (1  r )2 s 3 r 2 I 0  (1  r )2 s 5 r 4 I 0  . . .
 (1  r )2 sI 0 (1  s 2 r 2  s 4 r 4  . . .)
s (1  r )2
 I0
1  s 2 r2

5.214 We have t1  exd (1  r )2


1

and t 2  exd (1  r )2
2

where r is the reflectivity (see previous problem) and multiple reflections have been
ignored.
t1
Thus,  e x(d  d ) 2 1

t2
250 PART FIVE OPTICS

ln (t1 t 2 )
or x  0.35 cm1
d2  d1

5.215 On each surface we pick up a factor (1  r) from reflection and a factor exl due to
absorption in each plate.
Thus, t  (1  r )2N exNl
1 (1  r )2 N
and x ln  0.034 cm1
Nl t

5.216 Apart from the factor (1  r) on each end face of the plate, we shall get a factor due
to absorptions. This factor can be calculated by assuming the plate to consist of a
large number of very thin slabs within each of which the absorption coefficient can
be assumed to be constant. Thus, we shall get a product like
. . . ex ( x )dx ex ( x  dx )dx ex ( x  2dx )dx . . .
This product is nothing but
l

°
 x ( x ) dx
e 0

Now x (0)  x1 , x (l )  x 2 and variation with x is linear, so


x
x ( x )  x1  ( x 2  x1 )
l
Thus, the factor becomes
l
¨ x ·
°
 © x1  ( x2  x1 ) dx
ª l ¹̧
e 0  e12 ( x 2  x1 ) l

Finally we collect the factors due to reflection at each surface to get the transmission
coefficient as
l
( x1  x2 )
(1  r )2 e 2

5.217 The spectral density of the incident beam (i.e., intensity of the components whose
wavelength lies in the interval l and l  dl) is
I0
dl ( for l1  l  l2 )
l2  l1
The absorption factor for this component is
¨ l  l1 ·
© x 1  ( x  x 1 )¸ l
l2  l1 2
e ª ¹
5.5 DISPERSION AND ABSORPTION OF LIGHT 251

and the transmission factor due to reflection at the surface is (1  r)2. Thus, the
intensity of the transmitted beam is
l2 ¨ l  l1 ·
I0 l © x1  ( x2  x1 )¸
(1  r )2
l2  l1 ° d le ª l2  l1 ¹

l1

I0 ¥ 1  e( x  x )l ´ 2 1

 (1  r )2 ex l ¦ 1 x ( l2  l1 )
l2  l1 § ( x2  x1 )l µ¶

ex l  ex l
1 2

 (1  r )2 I 0
( x 2  x1 )l

5.218 At the wavelength l0, the absorption coefficient vanishes and loss in transmission
is entirely due to reflection. This factor is the same at all wavelengths and therefore
cancels out in calculating the passband and we need not worry about it.
Now, T0  (transmissivity at l  l0)  (1  r)2
and T  (transmissivity at l)  (1  r)2 ex(l)d
Thus,
T 2
 ead (1  ll ) 0

T0
The edges of the passband are l0  $l2 and at the edge
2
¥ $l ´
T ad ¦ µ
 e § 2l ¶  h 0

T0

$l ln 1h
Thus, 
2 l0 ad

1 ¥ 1´
or $l  2l0 ¦§ ln h µ¶
ad

5.219 We have to derive the law for decrease of intensity in an absorbing medium taking into
account the natural geometrical fall-off (inverse square law) as well as absorption.
Consider a thin spherical shell of thickness dx and internal radius x. Let I (x) and
I (x  dx) be the intensities at the inner and outer surfaces of this shell.
Then, 4px 2I (x )exdx  4p (x  dx )2 I (x  dx )
252 PART FIVE OPTICS

Except for the factor exdx this is the usual equation. We rewrite this as
x 2 I ( x )  I ( x  dx )( x  dx )2 (1  xdx )

¥ dI ´
¦I  dx µ ( x 2  2x dx )(1  xdx )
§ dx ¶

dI
or x2  xx 2 I  2xI  0
dx
d
Hence, ( x 2 I )  x( x 2 I )  0
dx
So, x 2 I  Cexx
where C is a constant of integration.
In our case we apply this equation for a x b. For x  a the usual inverse square
law gives
&
I (a ) 
4p a 2
& xa
Hence, C e
4p
& x ( b  a )
and I (b )  e
4pb 2
This does not take into account reflections. When we do that, we get
&
I (b )  (1  r )2 ex ( b  a )
4pb 2

5.220 The transmission factor is emd and so the intensity will decrease as
e md  e 3.6  11.3  8.1  58.4 times
(We have used m  (m/r)  r and used the known value of density of lead.)

5.221 We require mPb dPb  mAl dAl

¥ mPb ´ ¥ mAl ´
or ¦§ r µ¶ rPbdPb  ¦§ r µ¶ rAl d Al
Pb Al

So, 72.0  11.3  dPb  3.48  2.7  2.6


or dPb  0.3 mm
5.6 OPTICS OF MOVING SOURCES 253

1
5.222 We require  emd
2
ln 2 ln 2
or d   0.80 cm
m ( mr ) r

5.223 We require N plates where


N
¥ 1´ 1
¦§ µ¶ 
2 50
ln 50
So, N  5.6
ln 2

5.6 Optics of Moving Sources


5.224 In the Fizean experiment, light disappears when the wheel rotates to bring a tooth
in the position formerly occupied by a gap in the time taken by light to go from the
wheel to the mirror and back. Thus, distance travelled  2l. Suppose the mth tooth
after the gap has come in place of the latter. Then, time taken is
2(m  1)  1
s ( in the first case)
2zn1

and 2m  1
s (in the second case)
2zn2
2l 1
Then 
c z (n2  n1 )
Hence, c  2lz (n2  n1)  3.024  108 m/s

5.225 When v  c, time dilation effect of relativity can be neglected (i.e., t  t) and we can use
time in the reference frame fixed to the observer. Suppose the source emits short pulses
with intervals T0. Then in the reference frame fixed to the receiver the distance between
two successive pulses is l  cT0  vrT0 when measured along the observation line.
Here vr  v cos u is the projection of the source velocity on the observation line. The
frequency of the pulses received by the observer is
Source
c n0 ¥ vr ´
n  z n0 ¦ 1  µ
l 1  vr c § c¶
(The formula is accurate to first order only.) vT0
n  n0 vr v cos u u
Thus,  
n0 c c Observer
{

vrT0
The frequency increases when the source is cT0
moving towards the observer.
254 PART FIVE OPTICS

5.226 From the previous problem


$n v
 cos u u30
n c
But nl  c gives on differentiation s
$n $l

n l
v2 2T
So, $l  l 2
cos u  l cos u
c mc 2
1
On using T  mv 2 , m  mass of He ion and mc2  4  938 MeV and putting other
2
values, we get
$l  26 nm

5.227 One end of the solar disk is moving towards us while the other end is moving away
from us. The angle u between the direction in which the edges of the disk are mov-
ing and the line of observation is small (cos u  1). Thus,
$l 2vR

l c
where v  2pT is the angular velocity of the Sun. Thus,
c $l
v
2R l
4pR l
So, T
c $l
Putting the values (R  6.95  108 m), we get
T  24.85 days

5.228 Maximum splitting of the spectral lines will occur when both of the stars are moving
along the direction of line of observation as shown in the figure.
We then have the equations
¥ $l ´ 2v
¦§ µ¶ 
l m c
s s
mv 2 gm 2

R 4R2
pR Observer
t
v
5.6 OPTICS OF MOVING SOURCES 255

From these we get


¥ $l ´ c t
d  2R  ¦ µ  2.97  107 km
§ l ¶m p
3
¥ $l ´ c 3 t
and m¦ µ  2.9  1029 kg
§ l ¶ m 2pg

5.229 We define the frame S (the lab frame) by the condition of the problem. In this frame
the mirror is moving with velocity V (along say x-axis) towards left and light of fre-
quency v0 is approaching it from the left. We introduce the frame S whose axes are
parallel to those of S but which is moving with velocity V along x-axis towards left
(so that the mirror is at rest in S). In S, the frequency of the incident light is
1 2
¥ 1  V c ´
v1  v0 ¦
§ 1  V c µ¶
In S, the reflected light still has frequency v1 but it is now moving towards left.
When we transform back to S, this reflected light has the frequency
1 2
¥ 1  V c ´ ¥ 1  V c ´
v  v1 ¦  v0 ¦
§ 1  V c µ¶ § 1  V c µ¶
In the non-relativistic limit
¥ 2V ´
v z v0 ¦ 1  µ
§ c ¶

5.230 From the previous problem, the beat frequency is clearly


2v 2v 2v
$n  n 0  
c (cn0 ) l0
1 103
Hence, n l $n   50 cms  900 kmh
2 2

5.231 From the invariance of phase under Lorentz transformations, we get


vt  kx  vt  kx
y y'
Here v  ck. The primed coordinates refer to the
frame S which is moving to the right with velocity v.
Then, x  g (x  vt) x, x'
¥ vx ´
and ta  g ¦t  2 µ z z'
§ c ¶
256 PART FIVE OPTICS

where g  1 ( 1  v 2 c 2 ) .
Substituting and equating the coefficients of t and x, we get
1  vc
v  gv a  gk av  v a
1  v 2 c 2
vav 1  vc
and k g  gk a  k a
c2 1  v 2 c 2

5.232 From the previous problem using k  2pl, we get

1  vc
la  l
1  vc
1  vc la 2
Thus,  2
1  vc l

v la 2  l2 5642  4342
or  2   0.256
c la  l2 5642  4342

5.233 As in the previous problem


v l2  la 2

c l2  la 2
( l  la )2  1
So, vc  7.1  104 kms
( l  la )2  1

5.234 We go to the frame in which the observer is at rest. In this frame, the velocity of the
source of light is, by relativistic velocity addition formula
3 1
c c
4 2 2c
v 
¨ 3 1 c · 5
1 © c – – 2
ª 4 2 c ¹̧
When this source emits light of proper frequency v0, the frequency recorded by
observer will be

1  vc 3
v  v0  v0
1  vc 7

Note that v < v0 as the source is moving away from the observer (red shift).
5.6 OPTICS OF MOVING SOURCES 257

5.235 In transverse Doppler effect


¥ 1 ´
v  v0 1  b2 z v0 ¦ 1  b2 µ
§ 2 ¶
c c ¥ 1 2´ ¥ 1 2´
So, l  ¦§ 1  b µ¶  l0 ¦§ 1  b µ¶
v v0 2 2
1 2
Hence, $l  bl
2
v2 2T
Using b2   (where T  K.E. of H atoms)
c 2 mc 2
T 1
we get, $l  2
l  656.3 nm  0.70 nm
mc 938

5.236 (a) If light is received by the observer at P at the mo- O' vt O


v
ment when the source is at O, it must have been u
emitted by the source when it was at O and trav-
elled along OP. Then if OP  ct, OO  vt and
cos u  v/c  b. ct
In the frame of the observer, the frequency of the
light is v while its wave vector is
P
v
(cos u, sin u, 0)
c
We can calculate the value of v by relating it to proper frequency v0. The relation is
v
v0  (1  b cos u)
1  b2
To derive the formula in this form it is easiest to note that
v k–v

1  v c
2 2 1  v 2 c 2
is an invariant which takes the value v0 in the rest frame of the source.
v0 1  b2 v0
Thus, v   5  1010 s1
1 b 2
1  b2

(b) For the light to be received at the instant observer sees the source at O, light must
be emitted when the observer is at O at u  90° or cos u  0.
Then, as before
v
v0  or v  v0 1  b2  1.8  1010 s1
1  b2
258 PART FIVE OPTICS

In this case the observer will receive light along OP and he will “see” that the
source is at O even though the source will have moved ahead at the instant the
light is received.

5.237 An electron moving in front of a metal mirror sees an image charge of equal and op-
posite type. The two together constitute a dipole. Let us look at the problem in the
rest frame of the electron. In this frame the grating period is Lorentz contracted to
d a  d 1  v 2 c 2
Because the metal has etchings, the dipole moment of electron–image pair is peri-
odically disturbed with a period d/v. The corresponding frequency is v/d, which is
also the proper frequency of radiation emitted. Due to Doppler effect, the frequency
observed at an angle u is
1  (vc )2 vd
n  n 
1  vc cos u 1  vc cos u
The corresponding wavelength is
c ¥c ´
l  d ¦  cos uµ
n §v ¶
Putting c ≈ v, u  45°, d  2Mm, we get
l  0.586 Mm

5.238 (a) Let vx be the projection of velocity vector of the radiating atom in the observer’s
direction. The number of atoms with projections falling within the interval vx and
vx  dvx is
n(v x )dv x ^ exp(mv x2 2kT )dv x
The frequency of light emitted by the atoms moving with velocity vx is
¥ v ´
v  v0 ¦ 1  x µ
§ c¶
From the expressions the frequency distribution of atoms can be found:
n(v)dv  n(vx)dvx. Now using
v  v0
vx  c
v0
2
¥ mc 2 ¥ v ´ ´ cd v
we get n( v)d v ∼ exp ¦ ¦§ 1  v µ¶ µ v
§ 2kT 0 ¶ 0
Now the spectral radiation density Iv ^ nv.
5.6 OPTICS OF MOVING SOURCES 259

2
¥ v´
a ¦ 1  µ
§ v0 ¶ ¥ mc 2 ´
Hence, I v  I0e ¦§ where a  µ
2kT ¶
(The constant of proportionality is fixed by I0.)
(b) On putting v  v0  1/2 $v and using Iv  I0 /2 in the above relation, we get
2
¥ $v ´
1 a ¦ µ
 e § 2v 0¶

2
2
¥ $v ´
So, a¦ µ  ln 2
§ 2v0 ¶
$v kT
Hence,  (2 ln 2)
2v0 mc 2

$v kT
and  2 (2 ln 2)
v mc 2

5.239 In vacuum, inertial frames are all equivalent; the velocity of light is c in any frame.
This equivalence of inertial frames does not hold in material media and here the
frame in which the medium is at rest is singled out. It is in this frame that the velocity
of light is c/n where n is the refractive index of light for that medium.
The velocity of light in the frame in which the medium is moving is given by the law
of addition of velocities as
cn  V cn  V ¥ c ´¥ V ´
 ¦  V µ ¦1   ... µ
1  cn – V c 2 1  V cn § n ¶§ cn ¶
c V
  V  2  ...
n n
c ¥ 1 ´
z  V ¦1  2 µ
n § n ¶
This is the velocity of light in the medium in a frame in which the medium is moving
with velocity V  c.

5.240 Although speed of light is the same in all inertial frames of refer-
ence according to the principles of relativity, the direction of a light
ray can appear different in different frames. This phenomenon is u
called aberration and to first order in v/c, can be calculated by the
elementary law of addition of velocities applied to light waves. c
The angle of aberration is tan1 vc and in the present case it
equals 1/2 du on either side. Thus, on equating
v 1 1
 tan du y du ( du in radians)
c 2 2 v
260 PART FIVE OPTICS

c 3  108 41 p
or v du   
2 2 3600 180
3  4.1  p
  104 ms  29.8 kms
3.6  3.6

5.241 We consider the invariance of the phase of a wave moving in the x–y plane.
We write
vt  kx x  ky y  vt  kx x  ky y
From Lorentz transformations, L.H.S. is
¥ Vx ´
vag ¦ t  2 µ  kxa ( x  Vt )g  k ya y
§ c ¶
So equating v  g (v  Vkx) V
p
w u  2
¥ V va ´
kx  g ¦ kxa  2 µ
§ c ¶
and k y  k ya

on inverting v  g (v  Vkx)
¥ V v´
kxa  g ¦ kx  2 µ
§ c ¶
ky  ky
So, kx  k cos u, kx  k cos u
and ky  k sin u, ky  k sin u
We get on using ck vand ck  v
cos u  b
cos ua 
1  b cos u
where b  V/c and the primed frame is moving with velocity V in the x-direction
with respect to the unprimed frame. For small b  1, the situation is as given.
We see that cos ua z  b if u  p/2. Then,
¥p ´
ua  ¦  sin1 b µ
§ 2 ¶
This is exactly what we get from elementary non-relativistic law of addition of
velocities.
5.6 OPTICS OF MOVING SOURCES 261

5.242 The statement of the problem is not quite properly worked out and is in fact mislead-
ing. The correct situation is described below. We consider, for simplicity, stars in the
x–z plane. Then the previous formula is applicable, and we have
cos u  b cos u  0.99
cos ua  
1  b cos u 1  0.99 cos u
The distribution of u is given in the figure.
The light that appears to come from P
the forward quadrant in the frame u , u 8.1˚
2
K (u  p to u  p/2) is com- P
pressed into an angle of magnitude u  , u171.9˚
2
8.1° in the forward direction while
u  P → u = P
the remaining stars are spread out. u  0 → u0
171.9˚
The three-dimensional distribution
can also be found out from the three-dimensional generalization of the formula in
the previous problems.

5.243 The field induced by a charged particle moving with ve-


locity V excites the atoms of the medium turning them into sP
sources of light waves. Let us consider two arbitrary points
A and B along the path of the particle. The light waves
emitted from these points when the particle passes them Cs
reach the point P simultaneously and reinforce each other
provided they are in phase, which is the case in general u
s s V
if the time taken by the light wave to propagate from the A B
point A to the point C is equal to that taken by the particle
to fly over the distance AB. Hence, we obtain
v
cos u 
V
where v  c/n is the phase velocity of light. It is evident that the radiation is possible
only if V > v, i.e., when the velocity of the particle exceeds the phase velocity of light
in the medium.

5.244 We must have


c 3 V 1
V    108 ms or 
n 1.6 c 1.6
For electrons this means a K.E. greater than
¨ n · me c 2
Te  me c 2 ©  1¸   me c 2
2 1 
©ª n ¸¹ 1 ( 
11 . 6 ) 2
262 PART FIVE OPTICS

¨ 1 ·
 0.511 ©  1¸ (using mec2  0.511 MeV )
ª 1  (11
 .6 ) 2
¹
 0.144 MeV
For protons with mpc2  938 MeV, we have

¨ 1 ·
T p  938 ©  1¸  264 MeV  0.264 GeV
ª 1  ( 
11 . 6 ) 2
¹
¨ 1 ·
Also, Tmin  29.6 MeV  mc 2 ©  1¸
ª 1  (11
 .6 ) 2
¹
Then mc2  105.3 MeV. This is very nearly the mass of muons.

5.245 From cos u  v/V, we get


V  v sec u
V v sec u sec 30o 2 3 4
So,  sec u    
c c n 1.5 32 3 3
Thus, for electrons

¨ 1 · ¨ 27 ·
Te  0.511 ©  1¸  0.511 ©  1¸  0.289 MeV
ª 1  1627 ¹ ª 11 ¹
¨ 1 ·
Generally T  mc 2 ©  1¸
ª 1  1n cos u
2 2
¹

5.7 Thermal Radiation. Quantum Nature of Light


5.246 (a) The most probable radiation frequency vpr is the frequency for which
d ¥ v ´ v2
uv  3v2 F ¦ µ  F a( vT )  0
dv §T ¶ T
The maximum frequency is the root other than v  0 of this equation. It is
3TF ( vT )
v 
F a( vT )
or vpr  x0 T
where x0 is the solution of the transcendental equation
3F (x0)  x0F (x0)  0
5.7 THERMAL RADIATION. QUANTUM NATURE OF LIGHT 263

(b) The maximum spectral density is the density corresponding to most probable
frequency.
(uv )max  x03 F ( x0 )T 3 ^ T 3
where x0 is defined above.
(c) The radiosity is
@
c ¥ v´ ¨c @ 3 ·
Me 
40° v3 F ¦ µ d v  T 4
§T ¶ © ° x F ( x )dx ¸ ^ T 4
ª4 0 ¹

5.247 For the first black body


b
( lm )1 
T1
b b
Then, ( lm )2   $l 
T1 T2
b bT1
Hence, T2    1.747 kK
bT  $l b  T1$l

5.248 From the radiosity we get the temperature of the black body. It is
14 14
¥M ´ ¥ 3.0  104 ´
T ¦ eµ ¦  852.9 K
§ s ¶ § 5.67  108 µ¶
Hence, the wavelength corresponding to the maximum emissive capacity of the body is
b 0.29
 cm  3.4  104 cm  3.4 Mm
T 852.9
(Note that 3.0 W/cm2  3.0  104 W/m2.)

5.249 The black body temperature of the Sun may be taken as


0.29
T  6042 K
0.48  104
Thus, the radiosity is
M  5.67  108 (6042)4  0.755  108 W/m2
Energy lost by Sun is
4p(6.95)2  1016  0.7555  108  4.5855  1026 W
264 PART FIVE OPTICS

This corresponds to a mass loss of


4.5855  1026
kgs  5.1  109 kgs
9  1016
The Sun loses 1% of its mass in
1.97  1030  102
s z 1.22  1011 years
5.1  109

2.250 For an ideal gas, p  nkT, where n  number density of the particles and k  R/ NA
is Boltzmann’s constant. In a fully ionized hydrogen plasma, both H ions (protons)
and electrons contribute to pressure but since the mass of electrons is quite small
( mp /1836), only protons contribute to mass density. Thus,
2r
n
mH
2rR
and P T
NAmH
(where mH  mp is the proton or hydrogen mass).
Equating this to thermal radiation pressure, we get
2rR u M 4 4sT 4
T  e  
NAmH 3 3 c 3c
3c rR 3c rR
Then T3  
2sNAmH sM
(where M  2 NAmH  molecular weight of hydrogen  2  103 kg).
1 3
Thus, ¥ 3c rR ´
T ¦ z 1.89  107 K
§ sM µ¶

5.251 In time dt after the instant t when the temperature of the ball is T, it loses pd 2sT 4 dt
Joules of energy. As a result its temperature falls by dT and
p
pd 2sT 4 dt  d 3 rCdT
6
(where r  density of copper and C  its specific heat).
C rd dT
Thus, dt 
6s T 4
T0 /h
C rd dT
or t0 
6s
 ° 
T4
T0

C rd
 (h3  1)  2.94 h
18sT03
5.7 THERMAL RADIATION. QUANTUM NATURE OF LIGHT 265

5.252 Taking account of cosine law of emission, we write for the energy radiated per sec-
ond by the hole in cavity 1 as
dI (6)  A cos u d6
where A is a constant and d6 is an element of solid angle around some direction
defined by the symbol 6. Integrating over the whole forward hemisphere, we get
p/2
IA ° cos u 2p sin u d u  pA
0
1 2
l
We find A by equating this to the quantity
pd 2
sT14 –
4
(where s is Stefan-Boltzmann constant and d is the diameter of the hole).
1 2 4
Then A
sd T1
4
Now energy reaching cavity 2 from cavity 1 is
1 2 4
 sd T1 – $6 (where cos u  1)
4
where $6  ( pd 2 /4)l 2 is the solid angle subtended by the hole of cavity 2 at cavity 1.
(We are assuming d  l so $6  area of hole/(distance)2). This must be equal to
sT24 pd 24 which is the energy emitted by cavity 2. Thus equating, we get
1 2 4 pd 2 pd 2
sd T1  sT2
4
4 4l 2 4

d
or T2  T1
2l
Substituting, we get T2  0.380 kK  380 K.

5.253 (a) The total internal energy of the cavity is


4s 4
U T V
c

¥ eU ´ 16s 3
Hence, CV  ¦ µ  T V
§ eT ¶ V c

16  5.67  108
  109  103 JK
3  108
266 PART FIVE OPTICS

1.6  5.67
 nJK  3.024 nJK
3
(b) From the first law
TdS  dU  pdV
U ¥ U´
 VdU  UdV  dV ¦§ as p  µ¶
3 3
4U
 VdU  dV
3
16s 16s 4
 VT 3 dT  T dV
c 3c
16s 16s 3
So, dS  VT 2 dT  T dV
c 3c
¥ 16s ´
d¦ VT 3µ
§ 3c ¶
16s 1
Hence, S VT 3  CV  1.008 nJK
3c 3

5.254 We are given


u( v, T )  Av3 exp(avT )

du ¥ 3 a ´
(a) Then  ¦  µu  0
dv § v T ¶
3T 6000
So, vpr    1012 s1  7.853  1014 s1
a 7.64
(b) We determine the spectral distribution in wavelength
u ( l, T ) d l  u ( v, T ) d v

2pc 2pc C a
But v or l 
l v v
Ca Ca
So, dl dv and dv   dl
v2 l2
(We have put a minus sign before dl to subserve just this fact that dl is ve
where dv is ve.)
5.7 THERMAL RADIATION. QUANTUM NATURE OF LIGHT 267

C a ¥ C a ´ C a4 A ¥ aC a ´
Then, u ( l, T )  u ¦ , T µ  5 exp ¦ 2 µ
l § l
2 ¶ l § lT ¶
This is maximum when
eu ¨5 aC a ·
 6  u ©  2 ¸
el ª l lT ¹
aC a 2pca
or lpr    1.44 Mm
5T 5T

5.255 From Plancks formula


v3 1
uv  
2 3
pc e vkT 1
(a) In a range v  kT (long wavelength or high temperature)
v3 1
uv m
p c vkT
2 3

v2
 kT (using ex  1  x for small x)
p 2c 3
(b) In the range v T (high frequency or low temperature)
v v
1 so, e kT 1
kT
v3
and uv y 2 3 evkT
pc

5.256 We write
uv d v  un dn ( where v  2pn )
2p (2pn )3 1 16p 2 n 3 1
Then un  
p 2c 3 e 2pnkT  1 c3 e 2pnkT  1
¥ 2pc ´
Also u ( l, T )d l  uv d v ¦§ where l  µ
v ¶
2pc
d v  dl
l2
2pc ¥ 2pc ´
u ( l,T )  u¦ ‚Tµ
l2 § l ¶
3
2pc ¥ 2pc ´  1 16p 2c  1
 ¦§ µ¶ 2 3 2p clkT  
l2 l pc e 1 l5 e 2 p c  l kT  1
268 PART FIVE OPTICS

5.257 We write the required power in terms of the radiosity by considering only the energy
radiated in the given range. Then, from the previous problem we have
c
$P  u ( lm , T )$l
4
4p 2c 2  $l
 2pc  k l T
5
lm e m1
But lmT  b, so
4p 2c 2 T 5 1
$P  $l
b5 e 2pc kb  1
Using the given data, we get
2pc  p  3  108  1.05  1034
  4.9643
kb 1.38  1023  2.9  103
1
2pc kb
 7.03  103
e 1
and $P  0.312 W/cm2

5.258 (a) From the curve of the function y (x), we see that y  0.5 when x  1.41.
0.29
Thus, l  1.41  cm  1.105 Mm
3700
(b) At 5000 K,
0.29
l  106 m  0.58 Mm
0.5
So the visible range (0.40 to 0.70) mm corresponds to a range (0.69 to 1.31) of x.
From the curve
y (0.69)  0.07
and y (1.31)  0.44
So the required fraction is 0.37.
(c) The values of x corresponding to 0.76 are
0.76
x1   0.786 at 3000 K
0.29/0.3

0.76
and x2   1.31 at 5000 K
0.29/0.5
5.7 THERMAL RADIATION. QUANTUM NATURE OF LIGHT 269

The requisite fraction is then


4
P2 ¥ T2 ´ 1  y2
 
P1 ¦§ T1 µ¶ 1  y1
l l
ratio of total ratio of the fraction of
power required wavelengths in
the radiated power
4
¥ 5´ 1  0.44
¦ µ   4.91
§ 3¶ 1  0.12

5.259 We use the formula d  v.


Then, the number of photons in the spectral interval (v, v  dv) is

u( v, T )d v v2 1
n ( v )d v   2 3 vkT dv
v pc e 1

Using n( v)d v  n ( l)d l, we get

¥ 2pc ´ 2pc
d ln ( l)  n ¦ dl
§ l µ¶ l2

(2pc )3 1
 2 3 4 2 p c k lT  1
dl
pc l e
8p 1
 4 2 p c k lT  1
l e

5.260 (a) The mean density of the flow of photons at a distance r is


P 4pr 2 Pl
< j>   m2 s1
2pcl 8p 2 cr 2

10  0.589  106
 m2 s1
8p 2  1.054  1034  108  4

10  0.589
  1016 cm2 s1
8p 2  1.054  12

 5.9  1013 cm2 s1


270 PART FIVE OPTICS

(b) If n(r) is the mean concentration (number per unit volume) of photons at a
distance r from the source, then, since all photons are moving outwards with a
velocity c, there is an outward flux of cn which is balanced by the flux from the
source. In steady state, the two are equal, so
< j> Pl
n(r )   n
c 8p c 2r 2
2

1 Pl
So, r
2pc 2n

1 10  0.589  106

6p  108 2  1.054  1034  102  106

102 5.89
  8.87 m
6p 2.108

5.261 The statement made in the question is not always correct. However, it is correct in
certain cases, for example, when light is incident on a perfect reflector or perfect
absorber. Consider the former case. If light is incident at an angle u and reflected at
the angle u, then momentum transferred by each photon is
hn
2 cos u
c
If there are n(n) dn photons in frequency interval (n, n dn), then total momentum
transferred is
@
hn
° 2n(n ) c
cos udn
0

2&e
 cos u
c u u

dp &e
Thus, 
dt c

5.262 Momentum of a photon


h hn E
 p 
l c c
So, the change in momentum per unit time will be
E E E
2r  (1  r )  (1  r ) (1)
ct ct ct
5.7 THERMAL RADIATION. QUANTUM NATURE OF LIGHT 271

The first term in Eq. (1) is the momentum transferred on reflection and the second
on absorption.
The mean pressure < p > is related to the force F exerted by the beam by the expression
pd 2
< p>  F
4
The force F equals momentum transferred per second. This is (assuming that photons
that are not reflected are absorbed)
E E E
2r  (1  r )  (1  r )
ct ct ct
The first term is the momentum transferred on reflection (see Problem 5.261);
the second on absorption. Hence,
4(1  r ) E
< p> 
pd 2 c t

 48.3 atm (on substituting values)

5.263 The momentum transferred to the plate is


E E
(1  r ) [sin u i  cos u j]  r [2 cos u j]
c C
n n
u y
¥ momentum ´ ¥ momentum ´
¦ transferred µ ¦ transferred µ
¦ µ ¦ µ
§ on absorption¶ § on reflection¶ x

E E
 (1  r ) sin u j  (1  r ) cos u j
c c
Its magnitude is

E E
(1  r )2 sin 2 u  (1  r )2 cos2 u  1  r 2  2r cos 2 u
c c
Substitution gives 35 nN s as the answer.
Alternate: p
p0 u u
The Fig. (a) shows incident and reflected beams.
(a)
272 PART FIVE OPTICS

Having drawn the triangle of momenta in Fig. (b), where p


p0, p and p are the momenta of the incident beam, re-
flected beam, and the momentum transferred to the plate, 2u
respectively, and taking into account p  E c, p a  r E c,
we obtain p

E
p1  r 2  2r cos 2 u  35 nN s p0
c
(on substituting values).

5.264 Suppose the mirror has a surface area A. The incident beam
then has a cross-section of A cos u and the incident energy (b)
is IA cos u. Then, from the previous problem, the momen-
tum transferred per second ( force) is
IA cos u IA cos u
 (1  r ) cos u j  (1  r )sin u i u
c c
u
The normal pressure is then
u
I
p  (1  r ) cos2 u
c
(Here j is the unit vector perpendicular to the plane mirror.)
u
Putting in the values, we get u
0.20  104 1
p  1.8   0.6 nNcm 2
3  10 8 2

5.265 We consider a strip defined by the angular range


(u, u  du). From the previous problem, the normal
pressure exerted on this strip is u
2I
cos2 u
c s u
This pressure gives rise to a force whose resultant, by
symmetry, is in the direction of the incident light.
p 
2I I
Thus, F
c ° cos

2 u cos 2pR 2 sin ud u  pR 2
c

Putting in the values, we get


0.70  104
F  p  25  104 N  0.183 MN
3  108
5.7 THERMAL RADIATION. QUANTUM NATURE OF LIGHT 273

5.266 Consider a ring of radius x on the plate. The normal pressure on this ring is, by
Problem 5.264,

2 P
cos2 u
c 4p ( x  h2 R 2 )
2

x
P h2 R 2 u
 S hR
2pc ( x  h2 R 2 )2
2

The total force is then


R
P h2 R 2
° 2pc ( x 2  h 2 R 2 )2
2pxdx
0

R 2 ( 1  h2 )
P h2 R 2 dy

2c ° y2
h2 R 2

P h2 R 2 ¨ 1 1 · P
 © 2 2 2 ¸
2c ªh R R (1  h ) ¹ 2c (1  h2 )
2

5.267 (a) In the reference frame fixed to the mirror, the frequency of the photon is, by the
Doppler shift formula

1 b ¥ 1  b2 ´
vv ¦¦ v µµ (see Eq. 5.6b of the book)
1 b § 1 b ¶
In this frame, momentum imparted to the mirror is
2v 2v 1  b

c c 1 b

(b) In the K frame, the incident particle carries a momentum of vc and returns with
momentum
v 1  b
(see Problem 5.229)
c 1 b
The momentum imparted to the mirror, then, has the magnitude
v ¨ 1  b · 2v 1
©  1¸ 
c ª1  b ¹ c 1 b

Here b  V c.
274 PART FIVE OPTICS

5.268 When light falls on a small mirror and is reflected by it, the mirror recoils. The energy
of recoil is obtained from the incident beam photon and the frequency of reflected
photons is less than the frequency of the incident photons. This shift of frequency
can however be neglected in calculating quantities related to recoil (to the first
approximation).
Thus, the momentum acquired by the mirror as a result of the laser pulse is
2E
p f  pi 
c
On assuming pi  0, we get
2E
pf 
c
Hence the K.E. of the mirror is
p 2f 2E 2

2m mc 2
Suppose the mirror is deflected by an angle u. Then by conservation of energy,
2E 2
Final P.E.  mgl (1  cos u)  Initial K.E. 
mc 2
u 2E 2
or mgl 2 sin 2 
2 mc 2
u ¥ E ´ 1
or sin  ¦ µ
2 § mc ¶ gl

Using the given data,


u 13
sin   4.377  103
2 10  3  108 9.8 s 0.1
5

or u  0.502°

5.269 We shall only consider stars which are not too compact so that the gravitational field
at their surface is weak
gM
1
c2 R
We shall also explain the problem by making clear the meaning of the (slightly
changed) notation.
5.7 THERMAL RADIATION. QUANTUM NATURE OF LIGHT 275

Suppose the photon is emitted by some atom whose total relativistic energies (in-
cluding the rest mass) are E1 and E2 with E1 < E2. These energies are defined in the
absence of gravitational field and we have
E 2  E1
v0 

as the frequency at infinity of the photon that is emitted in 2 → 1 transition. On the
surface of the star, the energies become
E 2 gM ¥ gM ´
E2a  E2  –  E2 ¦ 1  2 µ
c2 R § c R¶

¥ gM ´
E1a  E1 ¦ 1  2 µ
§ c R¶

Thus, from v  E 2a  E1a, we get

¥ gM ´
v  v0 ¦ 1  2 µ
§ c R¶
Here v is the frequency of the photon emitted in the transition 2 → 1 when the atom
is on the surface of the star. It shows that the frequency of spectral lines emitted by
atoms on the surface of some star is less than the frequency of lines emitted by atoms
here on Earth (where the gravitational effect is quite small).
$v gM
Finally, z 2
v0 c R
(The answer given in the book is incorrect in general though it agrees with the above
gM
result for 2  1. )
c R

5.270 The general formula is


2pc
 eV
l
2pc
Thus, l
eV
2pc ¥ 1´
Now, $l  1 µ
eV ¦§ h¶

2pc ¥ h  1´
Hence, V  15.9 kV
e $l ¦§ h µ¶
276 PART FIVE OPTICS

5.271 We have, as in the previous problem


2pc
 eV
l
On the other hand, from Bragg’s law
2d sin a  kl  l
since k  1 when a takes its smallest value.
p c
Thus, V  30.974 kV z 31 kV
ed sin a

5.272 The wavelength of X-rays is the least when all the K.E. of the electrons approaching
the anticathode is converted into the energy of X-rays. But the K.E. of electron is
¨ 1 ·
Tm  mc 2 ©  1¸
©ª 1  v 2 c 2 ¸¹
(mc2  rest mass energy of electrons  0.511 MeV).
2pc
Thus,  Tm
l
1
2pc 2p ¨ 1 · 2p
or l  ©  1¸ 
Tm mc ª© 1  v c
2 2
¸¹ mc (g  1)

1
g  2.70 pm
1  v 2 c 2

5.273 The work function of zinc is


AZn  3.74 eV  3.74  1.602  1019 J
The threshold wavelength for photoelectric effect is given by
2pc
 AZn
l0
2pc
or l0   331.6 nm
A
The maximum velocity of photoelectrons liberated by light of wavelength l is given by
1 ¥1 1´
2
mvmax  2pc ¦  µ
2 § l l0 ¶
5.7 THERMAL RADIATION. QUANTUM NATURE OF LIGHT 277

4pc ¥ 1 1´
So, vmax  ¦  µ  6.55  105 ms
m § l l0 ¶

5.274 From the last equation of the previous problem, we find


(v1 )max 1l1  1l0
h 
(v2 )max 1l2  1l0

¥1 1´ 1 1
Thus, h2 ¦  µ  
§ l2 l0 ¶ l1 l0
1 2 h2 1
or (h  1)  
l0 l2 l1
1 (h2 l2  1l1 )
and 
l0 (h2  1)
2pc 2pc h2  l2 l1
So, A   1.88 eV
l0 l2 h2  1

5.275 When light of sufficiently short wavelength falls on the ball, photoelectrons are
ejected and the copper ball gains positive charge. The charged ball tends to resist
further emission of electrons by attracting them. When the copper ball has enough
charge, even the most energetic electrons are unable to leave it. We can calculate this
final maximum potential of the copper ball. It is obviously equal in magnitude (in
volt) to the maximum K.E. of electrons (in volt) initially emitted.
2pc
Hence, wmax   ACu
le
 8.86  4.47  4.39 V
(Here ACu is the work function of copper.)

5.276 We are given


E  a (1  cos vt) cos v0t
a
 a cos v0 t  [cos ( v0  v)t  cos ( v0  v)t ]
2
It is obvious that light has three frequencies and the maximum frequency is v0  v.
Then, the maximum K.E. of photoelectrons ejected is
 (v  v0)  ALi  0.37 eV
on substituting values and using ALi  2.39 eV.
278 PART FIVE OPTICS

5.277 Suppose N photons fall on the photocell per second. Then the power incident is
2pc
N
l
This will give rise to a photocurrent of
2pc
N –J
l
which means that number of electrons emitted is
2pc
N –J
el
Thus, the number of photoelectrons produced by each photon is
2pcJ
w  0.0198 z 0.02 (on substituting values)
el

5.278 A simple application of Einsteins equation


1 2pc
2
mvmax  hn  hn0   ACs
2 l
gives an incorrect result in this case because the photoelectrons emitted by the ce-
sium electrode are retarded by the small electric field that exists between the cesium
electrode and the copper electrode even in the absence of external emf. This small
electric field is caused by the contact potential difference whose magnitude equals
the difference of work functions
1
(ACu  ACs ) V
e
Its physical origin is explained below.
The maximum velocity of the photoelectrons reaching the copper electrode is then
1 1 2pc
2
mvmax  mv02  ( ACu  ACs )   ACu
2 2 l
Here v0 is the maximum velocity of the photoelectrons immediately after emission.
Putting the values and using ACu  4.47 eV, l  0.22 Mm, we get

vmax  6.41  105 m/s

The origin of contact potential difference is the following. Inside the metals free
electrons can be thought of as Fermi gas which occupy energy levels up to a
maximum called the Fermi energy EF. The work function A measures the depth of
the Fermi level.
5.7 THERMAL RADIATION. QUANTUM NATURE OF LIGHT 279

A1  A2
Outside

A A1 A2

EF

Inside metal 1 2

When metals 1 and 2 are in contact, electrons flow from one to the other till
their Fermi levels are the same. This requires the appearance of contact potential
difference of A1A2 between the two metals externally.

5.279 The maximum K.E. of the photoelectrons emitted by the Zn cathode is


2pc
E max   AZn
l
On calculating this comes out to be 0.993 eV  1.0 eV.
Since an external decelerating voltage of 1.5 V is required to cancel this current,
we infer that a contact potential difference of 1.5  1.0  0.5 V exists in the circuit
whose polarity is opposite to the decelerating voltage.

5.280 The unit of  is Joule-seconds. Since mc2 is the rest energy, mc 2 has the dimension
of time and multiplying by c we get a quantity

c 
mc
whose dimension is length. This quantity is called reduced Compton wavelength.
(The name Compton wavelength is traditionally reserved for 2pmc)

5.281 We consider the collision in the rest frame of the initial electron. Then the reaction is
g  e (rest) → e (moving)
Energy momentum conservation gives
m0c 2
v  m0c 2 
1  b2

v m0c b
or 
c 1  b2
280 PART FIVE OPTICS

(where v is the angular frequency of the photon).


Eliminating v , we get
1 b 1 b
m0c 2  m0c 2  m0c 2
1 b 2 1 b
This gives b  0 which implies v  0.
But a zero energy photon means no photon.

5.282 (a) Compton scattering is the scattering of light by free electrons (free electrons are
the electrons whose binding is much smaller than the typical energy transfer to
the electrons). For this reason the increase in wavelength $l is independent of
the nature of the scattered substance.
(b) This is because the effective number of free electrons increases in both cases.
With increasing angle of scattering, the energy transferred to electrons increases.
With diminishing atomic number of the substance, the binding energy of the
electrons decreases.
(c) The presence of a non-displaced component in the scattered radiation is due to
scattering from strongly bound (inner) electrons as well as nuclei. For scattering by
these, the atom essentially recoils as a whole, there is very little energy transfer.

5.283 Let l0  wavelength of the incident radiation. Then wavelength of the radiation scat-
tered at u1  60° is
¥  ´
l1  l0  2p c (1  cos u1 ) ¦§ where c  µ
mc ¶
Similarly l2  l0  2p c (1  cos u2 )

From the data u1  60°, u2  120° and l2  hl1, we get


(h  1)l0  2p c [1  cos u2  h(1  cos u1 )]

 2p c [1  h  h cos u1  cos u2 ]

¨ h cos u1  cos u2 ·
Hence, l0  2p c ©  1¸
ª h 1 ¹
¨ sin 2 u2 2  h sin 2 u1 2 ·
 4p c © ¸
ª h 1 ¹
 1.21 pm (on substituting values)
Note: The expression l0 given in the book contains misprints.
5.7 THERMAL RADIATION. QUANTUM NATURE OF LIGHT 281

5.284 The wavelength of the photon has increased by a fraction h so its final wavelength is
lf  (2  h) li
and its energy is
v
1h
The K.E. of the Compton electron is the energy lost by the photon and is given by
¥ 1 ´ h
T  v ¦ 1  µ  v
§ 1  h¶ 1h

 0.20 MeV

5.285 (a) From the Compton formula


la  2p c (1  cos 90 )  l
2p c 2p c ¥  ´
Thus, va   ¦§ where 2p c  µ
la l  2p c mc ¶
Substituting the values, we get v  2.24  1020 rad/s.
(b) The K.E. of the scattered electron (in the frame in which the initial electron was
stationary) is simply
T  v  va
2pc 2pc
 
l l  2p c
4p 2 c c 2pcl
   59.5 keV
l( l  2pc ) 1  l2p c

5.286 The wavelength of the incident photon is


2pc
l0 
v
Then, the wavelength of the final photon is
2pc
 2p c (1  cos u )
v
and the energy of the final photon is
2pc v
v a  
2pcv  2p  c (1  cos u) 1  vmc 2 (1  cos u)
282 PART FIVE OPTICS

v
  144.2 keV
1  2 ( vmc 2 )sin 2 (u2)

5.287 We use the equation


h 2p
l
p p
Then, from the Compton formula
2p 2p 
  2p (1  cos u)
pa p mc
1 1 1 u
So,   – 2 sin 2
pa p mc 2
u mc ¥ 1 1´
Hence, sin 2  ¦  µ
2 2 §p a p¶
mc ( p  p a )

2 pp a

u mc ( p  p a )
or sin 
2 2 pp a
Substituting the values, we get

u mc 2 (cp  cp a ) 0.511(1.02  0.255)


sin  
2 2cp – cp a 2  1.02  0.255

or u  120.2°

5.288 From the Compton formula


2p
l  l0  (1  cos u)
mc
From conservation of energy
2pc 2pc
 T
l0 l
2pc
 T
l0  2pmc (1  cos u)

4p 2 u T ¥ 4p 2 u ´
or sin  l0 ¦ l0  sin µ
mc 2 2pc § mc 2¶
5.7 THERMAL RADIATION. QUANTUM NATURE OF LIGHT 283

2 sin 2 u2 T ¥ 1 2 u´
or  ¦  sin 2 (using v0  2pcl0 )
mc 2 v0 § v0 mc 2 2 µ¶
2
¥ 1 ´ 1 sin 2 u2 2 sin 2 u2
Hence, ¦§ v µ¶  2 v mc 2  mc 2T  0
0 0

2 2
¥ 1 sin 2u2 ´ 2 sin 2u2 ¥ sin 2u2 ´
  ¦
¦§ v
0 mc 2 µ¶ mc 2T § mc 2 µ¶

1 sin 2u2 ¨ 2 mc 2 ·
 © 1   1¸
v0 mc ª 2 T sin u2
2
¹

mc 2 sin 2u2
or v0  1
2 mc 2
1
T sin 2u2

T ¨ 2 mc 2 ·
 © 1  1¸
2ª T sin 2u2 ¹
v0  0.677 MeV (on substituting values)

5.289 We see from the previous problem that the electron gains the maximum K.E. when
the photon is scattered backwards, i.e., u  180°, then
mc 2 
v0 
1  2 mc 2 Tmax  1

2pc 2p ¨ 2 mc 2 ·
Hence, l0   © 1  1¸
v0 mc ª Tmax ¹
Substituting the values, we get l0  3.695 pm.

5.290 Refer to the figure. Energy momentum conservation gives


(E,P)
va va
 cos u  p cos w w
c c hv u
v a
sin u  p sin w hv'
c
284 PART FIVE OPTICS

v  mc 2  v a  E
where E2  c2p2  m2c4. We see

v asin u 1lsin u
tan w  
v  v a cos u 1l  1lcos u
l sin u sin u
 
la  l cos u $ll  2 sin 2 u2

u
where $l  la  l  2p lc (1  cos u)  4p lc sin 2
2
2 sin u2 cos u2
Hence, tan w 
$ll  $l2p lc

$l $l $l 4p c
But sin u  2 1  1
4p c 4p c 2p c $l

( 4pmc $l)  1 ( 4pmc $l)  1


Thus, tan w  
1  (2pmc l) 1  (pmc 2 )
or w  31.3 (on substituting values)

5.291 By head-on collision we understand that the electron moves on in the direction of the
incident photon after the collision and the photon is scattered backwards. Then,
v  hmc 2
va  smc 2

(E, p)  (dmc2, mmc) of the electron.


Then by energy momentum conservation (cancelling factors of mc2 and mc) we
have
1hsd
hms
d2  1  m2
On eliminating s and d, we get

1  h h  m  m2  1

or (1  2h  m )  m2 1

Squaring (1  2h)2  2m (1  2h)  1


5.7 THERMAL RADIATION. QUANTUM NATURE OF LIGHT 285

or 4h  4h2  2m (1  2h)
2h(1  h)
or m
1  2h
Thus, the momentum of the Compton electron is
2h(1  h )mc
p  mmc 
1  2h
Now, in a magnetic field
p  Ber
2h(1  h)
Thus, r
(1  2h)mcBe
Substituting values, we get r  3.412 cm.

5.292 This is the inverse of usual Compton scattering. When


we write down the energy–momentum conservation
equations for this process we find that they are the same
s(E,P)
e
for the inverse process as they are for the usual process. 60°
It follows that the formula for Compton shift is appli- 60°
cable except that the energy (frequency) of the photon
is increased on scattering and the wavelength is shifted
downward. With this understanding, we write

$l  2p (1  cos u)
mc
¥  ´ u
 4p ¦ sin 2
§ mc µ¶ 2
 1.21 pm (on substituting values)
ATOMIC AND NUCLEAR
PHYSICS
PART

For this chapter in the book the formulas are given in the CGS units. Since most
6
students are familiar only with MKS units, we shall do the problems in MKS units.
However, wherever needed, we shall also write the formulas in the Gaussian units.

6.1 Scattering of Particles. Rutherford–Bohr Atom

6.1 The Thomson model consists of a uniformly charged nucleus in which the electrons
are at rest at certain equilibrium points (the plum in the pudding model). For the
hydrogen nucleus the charge on the nucleus is e while the charge on the electron is
e. The electron by symmetry must be at the centre of the nuclear charge where the
potential (from Problem 3.38a) is
¥ 1 ´ 3e
w0  ¦
§ 4pd0 µ¶ 2R
where R is the radius of the nuclear charge distribution. The potential energy of the
electron is e w0 and since the electron is at rest, this is also the total energy. To ionize
such an electron will require an energy of E e w0. From this we find
¥ 1 ´ 3e 2
R¦
§ 4pd0 µ¶ 2E

(In Gaussian system the factor 14pd0 is absent.)


On putting the values, we get R 0.16 nm.
Light is emitted when the electron vibrates. If we displace the electron slightly
inside the nucleus by giving it a push r in some radial direction and an energy dE of
oscillation, then since the potential at a distance r in the nucleus is

¥ 1 ´ e ¥3 r2 ´
w(r )  ¦ ¦  µ
§ 4pd0 µ¶ R § 2 2R 2 ¶

the total energy of the nucleus becomes

1 ¥ 1 ´ e2 ¥ 3 r2 ´
mr2  ¦ µ ¦§  2 µ¶   ew0  dE
2 § 4pd0 ¶ R 2 2R
288 PART SIX ATOMIC AND NUCLEAR PHYSICS

1 ¥ 1 ´ e2 2
or dE  mr2  ¦ r
2 § 4pd0 µ¶ 2R 3
This is the energy of a harmonic oscillator whose frequency is
¥ 1 ´ e2
v2  ¦
§ 4pd0 µ¶ mR 3
The vibrating electron emits radiation of frequency v whose wavelength is
2pc 2pc
l  mR 3 ( 4pd0 )12
v e
( In Gaussian units the factor ( 4pd0 )12 is absent.)
On putting the values, we get l 0.237 Mm.

6.2 Equation (6.1a) of the book can be written in MKS units as


u (q q 4pd0 )
tan  1 2
2 2bT
¥ q q ´ cot u2
Thus, b¦ 1 2 µ
§ 4pd0 ¶ 2T
( In Gaussian units the factor 14pd0 is absent.)
For a particle, q12e, for gold, q2 79e.
Substituting, we get b 0.731 pm.

6.3 (a) In the case of Pb, we shall ignore the recoil of the nucleus, both because Pb is quite
heavy (APb  208  52  AHe), and because Pb is not free. Then for a head-on col-
lision, at the distance of closest approach, the K.E. of the a-particle must become
zero (because a-particle will turn back at this point). Then,
2Z e2
T
( 4pd0 )rmin
(In Gaussian units the factor 4pd0 is absent.)
So, rmin  0.591 pm (on substituting values)
(b) Here we have to take into account that part of the energy is spent in the recoil of
Li nucleus. Suppose x1  coordinate of the a-particle from some arbitrary point on
the line joining it to the Li nucleus, x2  coordinate of the Li nucleus with respect
to the same point. Then, we have the energy momentum equations
1 1 2  3e 2
m1 x12  m2 x 22  T
2 2 ( 4pd0 )| x1  x2 |
6.1 SCATTERING OF PARTICLES. RUTHERFORD–BOHR ATOM 289

m1 x1  m2 x 2  2m1 T
Here m1  mass of He nucleus, m2  mass of Li nucleus. Eliminating x 2 , we get
1 1 2 6e 2
T  m1 x12 
2 2m2
 2m1T  m1 x1 
( 4pd0 )( x1  x2 )
We complete the square on the right-hand side and rewrite the above equation as
2
m2 1 ¨ m1 ·
T © m1 (m1  m2 ) x1  2m1T ¸
m1  m2 2m2 ª m1 m2 ¹
6e 2

( 4pd0 )| x1  x2 |
For the least distance of approach, the second term on the right must be greatest
which implies that the first term must vanish.
6e 2 ¥ m1 ´
Thus, |x1  x2|min  ¦§ 1  m µ¶
( 4pd0 )T 2

Using m1 m2  4 7 and other values, we get |x1  x2|min  0.034 pm.
(In Gaussian units the factor 4pd0 is absent.)

6.4 We shall ignore the recoil of Hg nucleus.


(a) Let A be the point of closest approach to the centre C, so AC  rmin. At A the mo-
tion is instantaneously circular because the radial velocity vanishes. Then if v0 is the
speed of the particle at A, the following equations hold
1 Z1 Z2 e 2
'  mv02  (1)
2 ( 4pd0 )rmin
A
mv0rmin  2mT b (2)
mv02 Z1 Z2 e 2
 (3) rmin
rmin ( 4pd0 )rmin
2
b
(This is Newton’s law. Here r is the radius of u
curvature of the path at A and r is minimum at C
A by symmetry). Finally, we have Eq. (6.1a) of the book in the form
Z1 Z2 e 2 u (4)
b cot
( 4pd0 )2T 2
From Eqs. (2) and (3)
2Tb 2 Z Z e2
 1 2
rmin ( 4pd0 )
290 PART SIX ATOMIC AND NUCLEAR PHYSICS

Z1 Z2 e 2 u
or rmin  cot 2
( 4pd0 )2T 2
With z1  2, z2  80, we get rmin  0.231 pm.
(b) From Eqs. (2) and (4), we get
Z1 Z2 e 2cot u2
rmin 
( 4pd0 ) 2mT v0
Substituting in Eq. (1)
1 u
T  mv02  2mT v0 tan
2 2
Solving for v0 we get

2T ¥ u u´
v0  ¦§ sec  tan µ¶
m 2 2
Z1 Z2 e 2 cot u2
Then rmin 
( 4pd0 )2T sec u2  tan u2
Z1 Z2 e 2 u¥ u u´
 cot ¦ sec  tan µ
( 4pd0 )2T 2 § 2 2¶
Z1 Z2 e 2 ¥ u´
 ¦§ 1  cosec µ¶  0.557 pm
( 4pd0 )2T 2

6.5 By momentum conservation


p  pi  p  pf
where p and p are the momenta of proton and pi and pf are the momenta of gold
nucleus, before and after the scattering.
Thus, the momentum transferred to the gold nucleus is clearly
$p  pf  pi  p  p
Although the momentum transferred to the gold
nucleus is not small, the energy associated with this
recoil is quite small and its effect on the motion of
the proton can be neglected to a first approxima- j u
tion. Then, i

$p  2mT (1  cos u)i  2mT sin u j


Here i is the unit vector in the direction of the incident proton and j is normal to it on
the side on which it is scattered. Thus,
6.1 SCATTERING OF PARTICLES. RUTHERFORD–BOHR ATOM 291

u
|$p| z 2 2mT sin
2
u Ze 2
Using tan  (for the proton)
2 ( 4pd0 )2bT
2
¨ ¥ 2bT ( 4pd0 ) ´ ·
we get, |$p| z 2 2mT ©1  ¦ µ¶ ¸
ª § Ze 2 ¹

6.6 The proton moving by the electron first accelerates and then decelerates and it is not
easy to calculate the energy lost by the proton, so energy conservation does not do the
trick. Instead we must directly calculate the momentum acquired by the electron. By
symmetry that momentum is along OA and its magnitude is
pd  ° F> dt

where F is the component of the force on electron along OA. Thus,


@
e2 b 1
pd  ° 4pd0
– 2
b  v t b  v 2t 2
2 2 2
dt
@
@
e 2b dx
 – ° 2 (where x  vt )
4pd0 v @ (b  x 2 )3 2
Evaluate the integral by substituting x  b tan u. vt A

2e 2
Then, pe 
( 4pd0 )vb
pe2 mpe4 b
and Te  
2me ( 4pd0 )2 Tb 2 me
(In Gaussian units there is no factor (4pd0)2.) u
On substituting the values we get Te  3.82 eV.
O

6.7 In the region where potential is non-zero, the K.E. of the particle is, by energy con-
servation, T  U0 and the momentum of the particle has the magnitude 2m T  U 0 . On
the boundary the force is radial, so the tangential component of the momentum does not
change.

So, 2mT sin a  2m (T  U 0 ) sin w

T sin a
or sin w  sin a 
T U0 n
292 PART SIX ATOMIC AND NUCLEAR PHYSICS

U0
where n  1
T
We also have u  2(a  w). Therefore,
u
sin  sin (a  w )  sin a cos w  cos a sin w
2
¥ cos a ´
 sin a ¦ cos w  µ
§ n ¶
n sin u2
or  n2  sin 2 a  cos a
sin a
2
¥ n sin u2 ´
¦§ sin a  cos aµ¶  n  sin a
or 2 2
u
a
w w a
u u u b a
n sin cot 2 a  2n sin cot a  1  n2 cos2
2 2
2 2 2
n cos u2  1
or cot a 
n sin u2
n sin u2
Hence, sin a 
1  n2  2n cos u2

Finally, the impact parameter is


nR sin u2
b  R sin a 
1  n2  2n cos u2

6.8 It is implied that the ball is too heavy to recoil.


(a) The trajectory of the particle is symmetrical about the radius vector through the
point of impact. It is clear from the diagram that
w u
p u
u  p  2w or w  w
2 2
u w
Also, b  ( R  r ) sin w  ( R  r ) cos
2
(b) With b defined above, the fraction of particles scattered
between u and u  du (or the probability of the same) is
| 2pbdb | 1
dP   sin u d u
p ( R  r )2 2
6.1 SCATTERING OF PARTICLES. RUTHERFORD–BOHR ATOM 293

(c) The probability of particle to be deflected after collision is


p 2 0
1 1 1
P ° 2
sin u d u  ° d (cos u) 
2 1 2
0

6.9 From the Eq. (6.1b) of the book


2
dN ¥ Ze 2 ´ d6
n¦ µ
N § ( 4pd0 )2T ¶ sin 4 u2
(We have put q1  2e, q2  Ze here. Also n  number of Pt nuclei in the foil per unit
area.)
NA 1 N rt
  At r – –  A
APt A APt
n n
mass of number of
the foil nuclei per
unit mass
Using the values APt  195, r  21.5  103 kg/m3, NA  6.023  1026/kmol, we get
n  6.641  1022 per m2.
dSn
Also, d6   102 sr
r2
dN
Therefore,  3.36  105 (on substituting values)
N

6.10 A scattered flux density of J (particles per unit area per second) equals J (1r 2 )  r 2 J
particles scattered per unit time per steradian in the given direction. Let n  concentra-
tion of the gold nuclei in the foil. Then, n  NA r  AAu and the number of Au nuclei per
unit area of the foil is nd where d  thickness of the foil. Then from Eq. (6.1b) of the
book (note that n → nd here)
2
¥ Ze 2 ´ u
r 2 J  dN  ndI ¦ cosec 4
§ (4pd0 )2T µ¶ 2
(Here I is the number of a-particles falling on the foil per second.)
4T 2r 2 J u
Hence, d sin 4
nI ( Ze 2  4pd0 )2 2
Using Z  79, AAu  197, r  19.3  103 kg/m3, NA  6.023  1026/kmol and other data
from the problem, we get d  1.47 Mm.
294 PART SIX ATOMIC AND NUCLEAR PHYSICS

6.11 From the Eq. (6.1b) of the book, we find


dNPt n Z2
 Pt – 2Pt  h
dNAg nAg Z Ag
But since the foils have the same mass thickness ( rd), we have
nPt AAg
 (See the Problem 6.9)
nAg APt

h APt
Hence, ZPt  ZAg –
AAg
Substituting ZAg  47, AAg  108, APt  195 and h  1.52, we get ZPt  77.86  78.

6.12 (a) From Eq. (6.1b) of the book, we get


2
r d NA ¥ Ze 2 ´ 2p sin u d u
dN  I 0 t
AAu ¦§ ( 4pd0 )2T µ¶ sin 4 u2
(We have used d6  2p sinu du and N  I0I.)
From the data
2
du  2  radians
57.3
Also, using ZAu  79, AAu  197, we get dN  1.63  106.
(b) This number is
2 p
¥ r d NA ´ ¥ Ze 2 ´ cos (u2)d u
N (u0 )  I 0 t ¦ µ ¦ µ 4p °
A
§ Au ¶ § ( 4p d 0 )2T ¶ u
sin 3 u2
0

The integral is
1 1
dx 1 ¨ 1 · u
2 ° x 2  2 ©ª 2x 2 ¹̧ sin u  2  cot 2 20
sin u  2
 

2
¥ Ze 2 ´ u
Thus, N (u0 )  pnd ¦ µ I 0 t cot 2 0
§ ( 4pd0 )T ¶ 2
where n is the concentration of nuclei in the foil (n  rN1/AAu).
Substitution gives N (u0)  2.02  107.

6.13 The requisite probability can be written easily by analogy with part (b) of the previous
problem. It is
6.1 SCATTERING OF PARTICLES. RUTHERFORD–BOHR ATOM 295

2 p
N (p2) ¥ Ze 2 ´ cos u2 d u
P  nd ¦ µ 4p °
I0 t § ( 4pd0 )2mv ¶
2
p
sin 3 u2
The integral is unity. Thus,
2
¥ Ze 2 ´
P  pnd ¦
§ ( 4pd0 )mv 2 µ¶
On substitution, we get
rAg NA 10.5  103  6.023  1023
n  and P  0.006
AAg 108

6.14 Because of the cosec4 u/2 dependence of the scattering, the number of particles (or
fractions) scattered through u < u0 cannot be calculated directly. But we can write this
fraction as
P (u0)  1  Q (u0)
where Q (u0) is the fraction of particles scattered through u  u0. This fraction has been
calculated before and is (see the solution of Problem 6.12b)
2
¥ Ze 2 ´ u
Q (u0 )  pn ¦ µ cot 2 0
§ ( 4pd0 )T ¶ 2
where n is number of nuclei/cm2. Using the data, we get
Q  0.4
Thus, P (u0)  0.6

6.15 The relevant fraction can be written as (see Problem 6.12 b). (Note that the projectiles
are protons.)
2
$N ¥ e2 ´ u
¦ µ p cot 2 0 – (n1 Z12  n2 Z22 )
N § ( 4pd0 )2T ¶ 2
Here n1(n2) is the number of Zn(Cu) nuclei per cm2 of the foil and Z1(Z2) is the atomic
number of Zn(Cu).
rdNA rdNA
Now n1   0.7 and n2   0.3
M1 M2
(Here M1, M2 are the mass numbers of Zn and Cu.)
Then, substituting the values Z1  30, Z2  29, M1  65.4, M2  63.5, we get
$N
 1.43  103
N
296 PART SIX ATOMIC AND NUCLEAR PHYSICS

6.16 From the Rutherford scattering formula


2
d s ¥ Ze 2 ´ 1
¦ µ
d 6 § ( 4pd0 )2T ¶ sin 4 u2
2
¥ Ze 2 ´ 2p sin u d u
or ds  ¦
§ ( 4pd0 )2T µ¶ sin 4 u2
2
¥ Ze 2 ´ cos u2 d u
¦ µ p
§ ( 4pd0 )T ¶ sin 3 u2
Then integrating from u  u0 to u  p, we get the required cross-section
2 p
¥ Ze 2 ´ cos u2 d u
$s  ¦ µ p°
§ ( 4pd0 )T ¶ u sin 3 u2
0

2
¥ Ze 2 ´ u
¦ µ cot 2 0
§ ( 4pd0 )2T ¶ 2
For uranium nucleus, Z  92 and on putting the values we get, $s  737 b  0.737 kb
(where 1b is 1 barn  1028 m2).

6.17 (a) From the previous problem


2
¥ Ze 2 ´ u
$s  ¦ µ p cot 2 0
§ ( 4pd0 )2T ¶ 2

Ze 2 u p
or T cot 0
4pd0 2 $s
Substituting the values with Z  79, u0  90°, we get T  0.903 MeV.
(b) The differential scattering cross-section is
ds u
 C cosec 4
d6 2
u0
where $s (u  u0 )  4pC cot 2
2
Thus, from the given data
500
C b  39.79 bsr
4p
ds
So, (u  60 )  39.79  16 bsr  0.637 kbsr
d6
6.1 SCATTERING OF PARTICLES. RUTHERFORD–BOHR ATOM 297

6.18 The formula in MKS units is


dE m e2
 0 w2
dt 6pc
For an electron performing (linear) harmonic vibrations, w is in some definite direc-
tion with
wx  v2x (say)
dE m e 2 v4 2
Thus,  0 x
dt 6p c
If the radiation loss is small (i.e., if v is not too large), then the motion of the electron is
always close to simple harmonic with slowly decreasing amplitude. Then, we can write
1
E  m v2 a 2
2
and x  a cos vt
and averaging the above equation, ignoring variation of a in any cycle, we get the
equation (on using < x2 >  1/2 a2)
dE m e 2 v4 1 2 m e 2 v2
 0 – a  0 E
dt 6pc 2 6pmc
since E  1/2 mv2a2 for a harmonic oscillator.
This equation integrates to
E  E0 et /T
where T  6pmce 2 v2 m0 .
It is then seen that energy decreases h times in time t0 given by
6pmc
t0  T ln h  ln h  14.7 ns
e 2 v2 m0

6.19 Moving around the nucleus, the electron radiates and its energy decreases. This means
that the electron gets nearer to the nucleus. By the statement of the problem, we can
assume that the electron is always moving in a circular orbit and the radial acceleration
by Newton’s law is
e2
w (directed inwards)
( 4pd0 )mr 2
dE m e6 1
Thus,  0
dt 6pc ( 4pd0 )2 m 2r 4
On the other hand, in a circular orbit
298 PART SIX ATOMIC AND NUCLEAR PHYSICS

e2
E 
( 4pd0 )2r
e2 dr m0 e 6
So,  
( 4pd0 )2r 2 dt ( 4pd0 )2 6pcm 2r 4
dr m0 e 4
or 
dt ( 4pd0 )3pcm 2r 2
m0 e 4
On integrating r 3  r03  t
4p 2d0cm 2
and so the radius falls to zero in time
4p 2d0 cm 2r03
t0  s  13.1 ps
m0 e 4

6.20 In a circular orbit, we have the following formula


mv 2 Ze 2

r ( 4pd0 )r 2
mvr  n
Ze 2
Then, v
( 4pd0 )n
n2 ( 4pd0 )
and r
Z me 2
The energy is
1 Ze 2
En  mv 2 
2 ( 4pd0 )r
2 2
¥ Ze 2 ´ m ¥ Ze 2 ´ m ¥ Ze 2 ´
¦   m ¦§ 4pd µ¶ 2  2n 2
§ 4pd0 µ¶ 2 2 n2 ¦§ 4pd0 µ¶ n2 0

and the circular frequency of this orbit is


2
v ¥ Ze 2 ´ m
vn  
r ¦§ 4pd0 µ¶ 3n3
On the other hand, the frequency v of the light emitted when the electron makes a
transition n  1 → n is
2
¥ Ze 2 ´ m ¥ 1 1 ´
v¦ 
§ 4pd0 µ¶ 2 2 ¦§ n2 (n  1)2 µ¶
6.1 SCATTERING OF PARTICLES. RUTHERFORD–BOHR ATOM 299

Thus, the inequality vn > v > vn  1 will result if

1 1¥ 1 1 ´ 1
 ¦ 2 
n3 2§n (n  1)2 µ¶ (n  1)3
Multiplying by n2 (n  1)2, we have to prove
(n  1)2 1 n2
 (2n  1) 
n 2 n 1
This can be written as
1 1 1
n  2  n   n 1 2
n 2 n 1
This is obvious because
1 1
1   (since n  1)
n 1 2
3
vn ¥ n  1´ 3
For large n ¦ µ 1
vn  1 § n ¶ n
vn v
So, m 1 and we may say m1
vn  1 vn

6.21 We have the following equations (we ignore reduced mass effects)
mv 2
 kr
r
and mvr  n
So, mv  mk r
n
Therefore r
mk

n  mk
and v
m
The energy levels are
1 1
En  mv 2  kr 2
2 2
1 n  mk 1 n 
  k
2 m 2 mk
k
 n
m
300 PART SIX ATOMIC AND NUCLEAR PHYSICS

6.22 The basic equations have been derived in the Problem 6.20. We rewrite them here and
determine the required values.
2
(a) We have r1 
m( Ze 2  4pd0 )
Thus, r1  52.8 pm for H atom and r1  26.4 pm for He ion, on using Z  1 for
H and Z  2 for He.
Ze 2
Also, v1 
( 4pd0 )
Thus, v1  2.191  106 m/s for H atom and v1  4.382  106 m/s for He ion.
1 m( Ze 2 )2
(b) we have T  mv12 
2 ( 4pd0 )2 2 2
On substituting values, T  13.65 eV for H atom and T  54.6 eV for He ion.
In both the cases, binding energy Eb  T because Eb  E and E  T.
(Recall that for coulomb force V  2T.)
(c) The ionization potential wi is given by
e w i  Eb
So wi  13.65 V for H atom and wi  54.6 V for He ion, on substituting values.
13.65
The energy levels are En   eV (for H atom)
n2
54.6
and En   eV (for He+ ion)
n2
¥ 1´
Thus w1  13.65 ¦ 1  µ  10.23 V (for H atom)
§ 4¶
and w1  4  10.23  40.9 V (for He ion)
The wavelength of the resonance line (n  2 → n  1) is given by
2p c 13.6 13.6
   10.23 eV for H atom
l 4 1
So l  121.2 nm (for H atom)
121.2
and l  30.3 nm (for He ion)
4

6.23 This has been calculated before in Problem 6.20. It is


m( Ze 2 4pd0 )2
v  2.08  1016 rads
3n 3
6.1 SCATTERING OF PARTICLES. RUTHERFORD–BOHR ATOM 301

6.24 An electron moving in a circle with a time period T constitutes a current


e
I
T
and forms a current loop of area pr2. This is equivalent to magnetic moment,
epr 2 evr
m  I pr 2  
T 2
where v  2pr/T . Thus, for the nth orbit
emvr ne 
mn  
2m 2m
(In Gaussian units mn  ne /2mc.)
e
We see that mn  Mn
2m
where M n  n   mvr is the angular momentum.
mn e
Thus, 
M n 2m
e
m1   mB  9.27  1024 A m 2
2m
(In CGS units m1  mB  9.27  1021 erg/G.)

6.25 The revolving electron is equivalent to a circular current.


e e ev
So, I  
T 2pr v 2pr
The magnetic induction
2
m0 I m ev m e2 ¨ me 2 ·
B  0 2  0 –e– –© 2
2r 4p r 4p ( 4pd0 )  ª  ( 4pd0 ) ¸¹
m0m 2e 7

256p 4d30 5
Substitution gives, B  12.56 T at the centre.
( In Gaussian units, B  m 2 e 7 c 5  125.6 kG.)

6.26 From the general formula for the transition n2 → n1


¥ 1 1´
v  E H ¦ 2  2 µ (where EH13.65 eV)
§ n1 n2 ¶
302 PART SIX ATOMIC AND NUCLEAR PHYSICS

(1) For Lyman series: n11; n22, 3, . . . . Thus,


3
v  E H  10.238 eV
4
This corresponds to
2pc
l  0.121 Mm
v
and Lyman lines have l  0.121 Mm with the series limit at 0.0909 Mm.
(2) For Balmer series: n22; n33, 4, . . . . Thus,

¥ 1 1´ 5
v  E H ¦  µ  E  1.876 eV
§ 4 9 ¶ 36 H
This corresponds to l0.65 Mm and so, Balmer series has l  0.65 Mm with the
series limit at l 0.363 Mm.
(3) For Paschen series: n23; n14, 5, . . . . Thus,

¥1 1´ 7
hv  E H ¦  µ  E  0.6635 eV
§ 9 16 ¶ 144 H
This corresponds to l 1.869 Mm with the series limit at l 0.818 Mm.

P
P B

0.1 Mm 1 Mm 10 Mm

6.27 The Balmer line of wavelength 486.1 nm is due to the transition 4 → 2 while the Balmer
line of wavelength 410.2 nm is due to the transition 6 → 2. The line whose wave num-
ber corresponds to the difference in wave numbers of these two lines is due to the tran-
sition 6 → 4. That line belongs to the Brackett series. The wavelength of this line is
1 ll
l  1 2  2.627 Mm
1l2  1l1 l1  l2

6.28 The energies are

¥ 1 1´ 5
EH ¦  µ  E
§ 4 9 ¶ 36 H

¥1 1´ 3
EH ¦  µ  E
§ 4 16 ¶ 16 H
6.1 SCATTERING OF PARTICLES. RUTHERFORD–BOHR ATOM 303

¥1 1´ 21
and EH ¦  µ  E
§ 4 25 ¶ 100 H
They correspond to wavelengths 654.2 nm, 484.6 nm and 433 nm, respectively.
The nth line of the Balmer series has the energy
¥1 1 ´
EH ¦ 
§ 4 (n  2) µ¶
2

For n 19, we get the wavelength 366.7450 nm and for n 20, we get the wavelength
366.4470 nm.
To resolve these lines we require a resolving power of
l 366.6
R   1.23  103
dl 0.298

6.29 For the Balmer series


¥1 2´
 vn   R ¦  2 µ (for n  3)
§4 n ¶
where R  E H  13.65 eV.
2p c ¥1 1´
Thus,  R ¦  2 µ
ln §4 n ¶

2p c 2p c ¥ 1 1 ´
or   R ¦ 2 
ln  1 ln §n (n  1) µ¶
2

¥ 2n  1 ´ 2R
 R ¦ 2  3 (for n 1)
§ n (n  1)2 µ¶ n
2p c 2R 
Thus, dl  3
ln2 n
ln p cn3 pcn3
or  
dl ln R  ln R
On the other hand, for just resolution in a diffraction grating
l l l l l
 kN  k  kl d sin u  sin u
dl d ld ld l
pcn3
Hence, sin u 
lR
u  59.4° (on substituting values)
304 PART SIX ATOMIC AND NUCLEAR PHYSICS

6.30 If all wavelengths are four times shorter but otherwise similar to the hydrogen atom spec-
trum, then the energy levels of the given atom must be four times greater. This means
4 EH
En  
n2
compared to En   E H n2 for hydrogen atom. Therefore, the spectrum is that of He
ion (Z2).

6.31 Because of cascading, all possible transitions are seen. Thus, we look for the number
of ways in which we can select upper and lower levels. The number of ways we can
do this is
1
n(n  1)
2
where the factor 1/2 takes account of the fact that the photon emission always arises
from upper → lower transition.

6.32 These are the Lyman lines


¥1 1´
v  E H ¦  2 µ
§1 n ¶
Here n  2, 3, 4, . . ..
For n 2 we get l 121.1 nm;
for n 3 we get l 102.2 nm;
for n 4 we get l 96.9 nm;
for n 5 we get l 94.64 nm and
for n 6 we get l 93.45 nm.
Thus, at the level of accuracy of our calculation, there are four lines 121.1 nm, 102.2 nm,
96.9 nm and 94.64 nm falling.

6.33 If the wavelengths are l1, l2, then the total energy of the excited state must be
2pc  2pc 
E n  E1  
l1 l2
But E1   4 E H and En   4 E H n2 , where we are ignoring reduced mass effects.
4 E H 2pc  2pc 
Then, 4E H   
n2 l1 l2
Substituting the values we get n223, which we take to mean n 5. The result is
6.1 SCATTERING OF PARTICLES. RUTHERFORD–BOHR ATOM 305

sensitive to the values of the various quantities and small difference gets multiplied
because difference of two large quantities is involved:
EH
n2 
pc  ¥ 1 1´
EH  ¦  µ
2 § l1 l2 ¶

6.34 For the longest wavelength (first) line of the Balmer series, we have on using the
generalized Balmer formula
¥ 1 1 ´
v  Z2R ¦ 2  2 µ
§n m ¶
2pc 8pc
the result l1, Lyman  
Z R (1  14) 3 Z 2 R
2

176pc
Then, $l  l1, Balmer  l1, Lyman 
15Z 2 R
176pc
So, R  2.07  1016 s1
15Z 2 $l

6.35 From the formula of the previous problem


176pc
$l 
15Z 2 R

176pc
or Z
15R $l
Substitution of $l59.3 nm and R from the previous problem gives Z3. This
identifies the ions as Li.

6.36 We start from the generalized Balmer formula

¥ 1 1 ´
v  RZ 2 ¦ 2  2 µ
§n m ¶
Here m n1, n2, . . ., @
The interval between extreme lines of this series (series n) is

¥ 1 1 ´ 2 ¥
1 1 ´ RZ 2
$v  RZ 2 ¦ 2  µ  RZ ¦  µ 
§n ( @ )2 ¶ § n2 (n  1)2 ¶ (n  1)2
306 PART SIX ATOMIC AND NUCLEAR PHYSICS

R
Hence, nZ 1
$v
Then the angular frequency of the first line of this series (series n) is

¥ 1 1 ´ ¨ ¥ n  1´ 2 ·
v1  RZ 2 ¦ 2  µ  $ v © ¦ µ  1¸
§n (n  1)2 ¶ ª§ n ¶ ¹

¨ « Z R$v º2 ·
® ®
 $v © ¬ »  1¸
©ª ­® Z R$v  1 ¼® ¸¹

2Z R$v  1
 $v 2
Z R$v  1
Then the wavelength will be
2

l1 

2pc 2pc Z R /$v  1


v1 $v 2Z R /$v  1
Substitution (with the value of R from Problem 6.34 which is also the correct value
determined directly) gives l10.468 Mm.

6.37 For the third line of Balmer series


¥1 1´ 21
v  RZ 2 ¦ 2  2 µ  RZ 2
§2 5 ¶ 100
2pc 200pc
Hence, l 
v 21RZ 2

200pc
or Z
21R l
Substitution gives Z2. Hence, the binding energy (B.E.) of the electron in the ground
state of this ion is Eb4EH4  13.6554.6 eV. Therefore, ion is He.

6.38 To remove one electron requires 24.6 eV. The ion that is left is He which in its ground
state has a binding energy of 4E H  4  R. The complete binding energy of both elec-
trons is then
E  E 0  4 R
79.1 eV (on substituting values)
6.1 SCATTERING OF PARTICLES. RUTHERFORD–BOHR ATOM 307

6.39 By conservation of energy


1 2p c
mv 2   Eb
2 l
where Eb  4 R is the binding energy of the electron in the ground state of He.
(Recoil of He2 nucleus is negligible). Then,

2 ¥ 2p c ´
v ¦§  Eb µ
m l ¶

2.25  106 m/s (on substituting values)

6.40 Photon can be emitted in H–H collision only if one H is excited to an n 2 state, which
then de-excites to n 1state by emitting a photon. Let v1 and v2 be the velocities of
the two hydrogen atoms after the collision and M their masses. Then, from energy mo-
mentum conservation
Mv1  Mv2  2 MT
(in the frame of the stationary H atom)
1 1 3
and Mv12  Mv22   R  T
2 2 4
3 ¥ 1´
where  R   R ¦ 1  µ is excitation energy of n  2 state from ground state.
4 § 4¶
Eliminating v2, we get
2
1 «® 2 ¥ 2T ´ º® 3
M ¬v1  ¦  v1 µ »   R  T
2 ­® § M ¶ ¼® 4

1 « 2 2T 2T º 3
or M ¬2v1  2 v1  »  R  T
2 ­ M M¼ 4
2
«®¥ 1 2 T ´ º® 1 3
M ¬¦ v1  µ »  T  R  T
­®§ 2 M ¶ ®¼ 2 4
2
«®¥ 1 2T ´ º® 3 1
or M ¬¦ v1 
§ µ¶ »   R  T
­® 2 M ®¼ 4 2
3
For minimum T, the square on the left should vanish. Thus, T   R  20.4 eV.
2
308 PART SIX ATOMIC AND NUCLEAR PHYSICS

6.41 In the rest frame of the original excited nucleus we have the equations
OpgpH
3 p2
and R  c pg  H
4 2M
(where 34 R is the energy available in n 2 → 1 transition corresponding to the first
Lyman line).
3  RM
Then pH2  2McpH  0
2
3
or ( pH  Mc )2  M 2c 2   RM
2
3
pH  Mc  M 2c 2   RM
2
1 2
¥ 3 R ´ 3 R
 Mc  Mc ¦ 1  µ 
§ 2 Mc 2 ¶ 4c
(We could have written this directly by noting that pH2 2 M  cpg ). Then,
3 R
vH   3.3 ms
4 Mc
2
3 3 1 ¥ 3 R ´
6.42 We have d  R and da   R  ¦ µ
4 4 2 M § 4c ¶
d  da 3 R
Then, 
d 8 Mc 2
v
 H  5.5  109  100  0.55  106 %
2c

6.43 We neglect recoil effects. The energy of the first Lyman line photon emitted by He is
¥ 1´
4  R ¦ 1  µ  3 R
§ 4¶
The velocity v of the photoelectron that this photon liberates is given by
1
3 R  mv 2   R
2
(where R on the right is the binding energy of the n 1 electron in H atom).
4 R R
Thus, v 2  3.1  106 ms
m m
Here, m is the mass of the electron.
6.1 SCATTERING OF PARTICLES. RUTHERFORD–BOHR ATOM 309

6.44 Since $l (0.20 nm)  l (121 nm) of the first Lyman line of H atom, we need not
worry about v2/c2 effects. Then,
v ¥ v´
va  ¦§ where b  µ¶
1  b cos u c
v la
Hence, 1  b cos u  
va l
la $l
or b cos u  1  
l l
3 2pc 8pc
But v R so l 
4 v 3R
3 R $l
Hence, v  cb 
8p cos u
Substitution (cos u  1 2 ) gives v7.0  105 m/s.

6.45 (a) If we measure energy from the bottom of the well, then V (x)0 inside the walls.
Then the quantization condition reads

° p dx  2lp  2pn 
pn 
or p
l
p2 p 2n2 
Hence, En  
2m 2ml 2
(Here ° p dx  2lp because we have to consider the integral from l/2 to l/2 and
then back to l/2.)

(b) Here, ° p dx  2prp  2pn 


n
or p
r
n 2 2
Hence, En 
2mr 2
(c) By energy conservation
p2 1
 ax 2  E
2m 2
so, p  2mE  max 2

Then, ° p dx  ° 2mE  max 2 dx


310 PART SIX ATOMIC AND NUCLEAR PHYSICS

2 E a
2E
 2 ma ° a
 x 2 dx
 2 E a

The integral is
a p 2 p 2
a2
° a 2  x 2 dx  a 2 ° cos2 u d u 
2 p°2
(1  cos 2u) d u
a p 2

p
 a2
2
2E m
Thus, ° p dx  p ma –
a
 E – 2p –
a
 2p n 

a
Hence, En  n 
m
(d) It is required to find the energy levels of the circular orbit for the potential
a
U (r )  
r
In a circular orbit, the particle only has tangential velocity and the quantization
condition reads 
° p dx  mv – 2pr  2pn
so, mvr  M  n 
The energy of the particle is
n 2 2 a
E 2

2mr r
Equilibrium requires that the energy as a function of r be minimum. Thus,
n 2 2 a n 2 2
3
 2 or r
mr r ma
ma2
Hence, En  
2n2 2

6.46 The total energy of the H atom in an arbitrary frame is


1 1 e2
E  m V12  M V 22 
2 2 (4pd0 ) r1  r2
Here V1  r1 , V2  r2 , and r1 and r2 are the coordinates of the electron and protons.
m r1  M r2
We define R
M m
6.1 SCATTERING OF PARTICLES. RUTHERFORD–BOHR ATOM 311

rr1  r2
m V1  M V2
Then, V
mM
vV1  V2
M
or V1  V  v
mM
m
and V2  V  v
mM
1 1 mM e2
We get E  (m  M )V 2  v2 
2 2mM 4pd0r
In the frame V0, this reduces to the energy of a particle of mass
mM
m
mM
where m is called the reduced mass.
me 4 me 4
Then, Eb  and R
22 23
m ¥ m´
Since, m  m ¦1  µ
1
m § M¶
M
these values differ by m/M ( 0.54%) from the values obtained without considering
nuclear motion (M1837m).

6.47 The difference between the binding energies is


$EbEb(D)  Eb(H)
m e4 m e4
 
1  m/ M 22 1  m/2 M 22
me 4 ¥ m´
 ¦§ µ
22 2m ¶
Substitution gives $Eb3.7 meV.
For the first line of the Lyman series
2p c ¥ 1 1´ 3
 R ¦  µ  R
l § 1 4¶ 4
8pc 8p c
or l 
3R 3 Eb
312 PART SIX ATOMIC AND NUCLEAR PHYSICS

8p c ¥ 1 1 ´
Hence, lH  lD  
3 § Eb (H) Eb (D) µ¶
¦

1
8pc ¥ me 4 ´ ¥ m m ´
 –¦ 2 µ ¦§ 1  1 µ
3 § 2 ¶ M 2M ¶
8pc m
 –
¥ me 4 ´ 2 M
3¦ 2 µ
§ 2 ¶
m
  l1
2M
(where l1 is the wavelength of the first line of Lyman series without considering
nuclear motion).
Using l1121 nm (see Problem 6.21), we get $l33 pm.

6.48 (a) In the mesonic system, the reduced mass of the system is related to the masses of
the meson (mm ) and proton (mp ) by
mmm p
m  186.04 me
mm  m p

Then, separation between the particles in the ground state is


2 1 2
2

me 186 me 2
0.284 pm

me 4
Eb ( meson )   186  13.65 eV
22
2.54 keV
8p c l (H)
l1 (meson)   1  0.65 nm
3 Eb ( meson ) 186
(On using l1 (H)121 nm.)

(b) In the positronium

me2 m
m  e
2me 2
6.2 WAVE PROPERTIES OF PARTICLES. SCHRODINGER EQUATION 313

Thus, separation between the particles in the ground state is


2
2  105.8 pm
me e 2
me e 4 1
Eb ( positronium )  – 2  Eb (H)  6.8 eV
2 2 2
l1 (positronium)2l1(H)0.243 nm

6.2 Wave Properties of Particles. Schrodinger Equation

6.49 The kinetic energy is non-relativistic in all three cases. Now


2p  2p 
l 
p 2mT
Using T1.602  1017 J, we get le122.6 pm and lp2.86 pm, lU  lp  238 
0.185 pm.
(Here we have used a mass number of 238 for the U nucleus).

2p  2p 
6.50 From l 
p 2mT
4p 2 2 2p 2 2
we find T 
2m l2 m l2
2p 2 2 ¥ 1 1´
Thus, T2  T1  ¦ 2
 2µ
m § l2 l1 ¶
Substitution gives ΔT  451 eV  0.451 keV.

6.51 We shall use Md  2Mn. The centre of mass (C.M.) is moving with velocity
2 M nT 2T
V 
3 Mn 9 Mn
with respect to the laboratory frame. In the C.M. frame, the velocity of neutron is
2T 2T 2T 2
vna  vn  V    
Mn 9 Mn Mn 3
2p  3p 
and lna  
M n vna 2 M nT
314 PART SIX ATOMIC AND NUCLEAR PHYSICS

Substitution gives lna  8.6 pm.


Since the momenta are equal in the C.M. frame, the de Broglie wavelengths will also
be equal. If we do not assume Md  2Mn, we shall get
2p (1  M n  M d )
lna 
2 M nT

6.52 If p1, p2 are the momenta of the two particles then their momenta in the C.M. frame will be
 (p1  p2)/2 as the particles are identical. Hence, their de Broglie wavelength will be
2p  4p 
  ( because p1 > p2 )
1 p12  p22
p1  p2
2
2 2 l1 l2
  .
1 1 l12  l22
 2
l21 l2

6.53 In thermodynamic equilibrium, Maxwell’s velocity distribution law holds, so


2
dN (v )  &(v )dv  Av 2 emv 2kT dv
& (v) is maximum when
¨ 2 mv ·
& a ( v )  &( v ) ©  0
ªv kT ¹̧
This defines the most probable velocity,
2kT
vpr 
m
The de Broglie wavelength of H molecules with the most probable velocity is
2p  2p 
l 
mvpr 2mkT
Substituting the appropriate values m  mH  2mH , T  300 K, we get l  128 pm.
2

6.54 To find the most probable de Broglie wavelength of a gas in thermodynamic


equilibrium, we determine the distribution in l corresponding to Maxwellian
velocity distribution. It is given by
# (l) dl  & (v) dv
(where ve sign takes into account that l decreases as v increases).
6.2 WAVE PROPERTIES OF PARTICLES. SCHRODINGER EQUATION 315

2p  2p 
Now, l or v
mv ml
2p 
dv   dl
m l2
¥ dv ´
2
Thus, #( l)   Av 2 emv 2kT ¦ µ
§ d l¶
2
¥ 2p  ´ ¥ 2p  ´ ¨ m ¥ 2p ´ 2 ·
 A¦ µ ¦ µ exp © –¦ µ ¸
§ m l ¶ § m l2 ¶ ª 2kT § m l ¶ ¹
2
 constant  l4 eal
2p 2 2
where a
mkT
This is maximum when
¨4 2a ·
# a( l)  0  #( l) ©  3¸
ª l l ¹
a p
or lpr  
2 mkT
Using the result of the previous problem, it is
126
lpr   89.1 pm
2

6.55 For a relativistic particle


Total energy  T  mc 2  c 2 p 2  m 2c 4
Squaring, rearranging and taking positive root
T (T  2mc 2 )  cp
2p c
Hence, l
T (T  2mc 2 )
2p

2mT (1  T 2mc 2 )
If we use the non-relativistic formula,
2p 
lNR 
2mT
$l lNR  l T
so,  z
l lNR 4mc 2
316 PART SIX ATOMIC AND NUCLEAR PHYSICS

4mc 2$l
Thus, T (if the error is less than $l)
l
Here we have used that if T 2 mc 2 O 1‚ we can write (1  T 2mc 2 )12 z 1  T 4mc 2 .
For an electron, the error is not more than 1% if
T  4  0.511  0.01 MeV
i.e., T  20.4 keV
For a proton, the error is not more than 1% if
T 4  938  0.01 MeV
i.e., T  37.5 MeV

6.56 The de Broglie wavelength is


2p m0 v 2p 
ldB   1  v 2 c 2
1  v c 2 2 m0 v
and the Compton wavelength is
2p 
lcomp 
m0 c
v 1
The two are equal if b  1  b2 , where b  or b 
c 2
The corresponding K.E. is
m0 c 2
T  m0 c 2   2  1 m0 c 2  0.21 MeV
1  b2
Here m0 is the rest mass of the particle (which here is an electron).

6.57 For relativistic electrons, the formula for the short wavelength limit of X-rays will be
2p c ¥ 1 ´
 m0 c 2  1  c p 2  m 2c 2  mc 2
lsh ¦§ 1  b2 µ¶
2
¥ 2p  ´
¦§ l  mc µ¶  p  m c
or 2 2 2

sh

¥ 2p  ´ ¥ 2p  ´
¦§ l µ¶ ¦§ l  2mc µ¶  p
or 2

sh sh

2p  mc lsh
or p 1
lsh p
6.2 WAVE PROPERTIES OF PARTICLES. SCHRODINGER EQUATION 317

mc lsh
Hence, ldB  lsh 1  3.29 pm
p

6.58 The first minimum in Fraunhofer diffraction is given by b sin u  l (where b is the
width of the slit).
$x  2 $x
Here, sin u  z
l  ( $ x  2)
2 2 2l
b$x 2p 
Thus, l 
2l mn
4pl
so, v  2.02  106 ms
mb$x

6.59 From the Young’s slit formula


ll l 2p 
$x   
d d 2meV
 4.90 Mm (on substituting values)

6.60 From Bragg’s law, for the first case


2p 
2d sin u  n0 l  n0
2meV0
where n0 is an unknown integer. For the next higher voltage
2p 
2d sin u  (n0  1)
2mehV0
n0  1
Thus, n0 
h

¥ 1 ´ 1 1
or n0 ¦ 1  µ  or n0 
§ h¶ h h 1
Going back we get
p 2 2 1
V0  2
 0.150 keV
2med 2 sin 2 u  h 1
Note: In the Bragg’s formula, u is the glancing angle and not the angle of incidence.
We have obtained the correct result by taking u to be the glancing angle. If u is the angle
of incidence, then the glancing angle will be 90   u. Then final answer will be smaller
by a factor tan2 u  1/3.
318 PART SIX ATOMIC AND NUCLEAR PHYSICS

6.61 Path difference is


u
d  d cos u  2d cos2
2
dcos u
Thus, for reflection maximum of the kth order u u
u 2p  d
2d cos2  kl  k
2 2mT
kp  u
Hence, d sec2
2mT 2
Substitution with k  4, gives d  0.232 nm.

6.62 See the analogous Problem 5.156 with X-rays. The glancing angle is obtained from
D
tan 2u 
2l
where D  diameter of the ring, l  distance from the foil to the screen.
Then for the third-order Bragg reflection,
2p 
2d sin u  k l  k ( where k  3)
2mT
p k
Thus, d  0.232 nm
2mT sin u

6.63 Inside the metal, there is a negative potential energy of  eVi. (This potential energy
prevents electrons from leaking out and can be measured in photoelectric effect, etc.)
An electron whose K.E. is eV outside the metal will find its K.E. increased to e (V  Vi)
in the metal. Then,
(a) de Broglie wavelength in the metal
2p 
lm 
2me (V  Vi )
Also, de Broglie wavelength in vacuum
2p 
l0 
2mVe
l0 V
Hence, refractive index n  1 i
lm V
1
Substituting, we get n  1 z 1.05
10
6.2 WAVE PROPERTIES OF PARTICLES. SCHRODINGER EQUATION 319

Vi
(b) We have n 1 1 1 h
V
Vi
then, 1  (1  h)2
V
or Vi  h (2  h) V
V 1
or 
Vi h(2  h)
For h  1%  0.01, we get
V
 50
Vi

6.64 The energy inside the well is entirely kinetic if energy is measured from the value in-
side.
nl p
We require l n
2 2mE
n 2p 2 2
or En  (for n  1, 2, . . .)
2ml 2

6.65 The Bohr condition is


2p 
° p dx  ° l
dx  2pn 

For the case when l is constant (for example in circular orbits), this means
2pr  nl
(Here r is the radius of the circular orbit.)

6.66 From the uncertainty principle (Eq. 6.2b of the book)


$x $p x  

Thus, $p x  m $v x 
$x

or $v x 
m $x
If ∆x  1 Mm 106 m, the uncertainty in velocity is:
For an electron ∆vx  1.16  104 cm/s
320 PART SIX ATOMIC AND NUCLEAR PHYSICS

For a proton ∆vx  6.3 cm/s


For a ball ∆vx  1  1020 cm/s

6.67 As in the previous problem



$v H  1.16 s 106 ms
ml
The actual velocity v1 has been expressed in Problem 6.21. It is v1  2.21  106 m/s.
Thus, ∆v [ v1 (they are of the same order of magnitude).

l 2p  1
6.68 Given that $x   
2p p 2p
 
 
p mv

H 
Then, $v v
m $x
Thus, Δv is of the same order as v.

6.69 We know that initial uncertainty is $v H ml .


With this uncertainty the wave train will spread out to a distance hl in time t0, then
hl hml 2
t0 z z s
ml 
 8.6  1016 s ∼ 1015 s


6.70 Clearly $x  l so, $p x 
l
Now px  ∆px and so,
px2 2
T 
2m 2ml 2
2
Thus, Tmin   0.95 eV
2ml 2

6.71 The momentum of the electron is $px  2mT . Uncertainty in its momentum is
 
$p x  
$x l
6.2 WAVE PROPERTIES OF PARTICLES. SCHRODINGER EQUATION 321

Hence, relative uncertainty

$p x  2 2ml 2 $v
  
px l 2mT T v

Substitution gives $v
 9.75  105 z 104
v

6.72 By uncertainty principle, the uncertainty in momentum



$p 
l
For the ground state, we expect Δp p, so
2
E ∼
2ml 2
The force exerted on the wall can be obtained as
eU 2
F   3
el ml

 
6.73 We write p ∼ $p ∼ ∼
$x x
i.e., all four quantities are of the same order of magnitude. Then,
2
2 1 1 ¥ ´ k
E z 2
 kx 2  ¦§  mk x µ¶  
2mx 2 2m x m
Thus, we get an equilibrium situation (E  minimum) when


x  x0 
mk
k
and then E  E0 ∼   v
m
Quantum mechanics gives
v
E0 
2


6.74 We write r ∼ $r , p ∼ $p ∼
$r
2 e2
Then, E 2

2mr r
322 PART SIX ATOMIC AND NUCLEAR PHYSICS

2
1 ¥  me 2 ´ me 4
 ¦§  µ¶  2
2m r  2
2
Hence, reff   53 pm (for the equilibrium state)
me 2
me 4
and then E   13.6 eV
22

6.75 Suppose the width of the slit (its extension along the y-axis) is d. Then each
electron has an uncertainty $y ∼ d. This translates to an uncertainty $p y ∼ /d. We must
therefore have

py 
d
For the image, broadening has two sources. We write
Δ(d)  d  Δ (d)
where Δ is the width caused by the spreading of electrons due to their transverse
momentum.
l l l
We have $  vy  py 
vx p mv d
l
Thus, $( d)  d 
mv d
For large d, Δ (d) d and quantum effect is unimportant. For small d, quantum effects
are large. But Δ (d) is minimum when
l
d
mv
as we see by completing the square.
Substitution gives d  1.025  105 m  0.01 mm.

6.76 The Schrodinger equation in one dimension for a free particle is


e# 2 e2 #
i 
et 2m ex 2
We write # (x, t)  w (x) X (t)
i dx  2 1 d 2w
Then,   E (say)
x dt 2m w dx 2
6.2 WAVE PROPERTIES OF PARTICLES. SCHRODINGER EQUATION 323

Then, x (t ) ∼ exp ¥¦ iEt ´µ


§ h¶

¥ 2mE ´
w( x ) ∼ exp ¦ i xµ
§  ¶
E must be real and positive if w (x) is to be bound everywhere. Then,
¥i ´
c(x , t )  constant  exp ¦  2mE x  Et µ
§ ¶
This particular solution describes plane waves.

6.77 We look for the solution of Schrodinger equation with


2 d 2c (1)
  Ec (for 0  x  l )
2m dx 2
The boundary condition of impenetrable walls means
c (x)0 for x 0 and x l (as c (x)0 for x < 0 and x > l).
The solution of Eq. (1) is
2mE 2mE
c ( x )  A sin x  B cos x
 
Then c (0)0 ⇒ B0

2mE
c (l)0 ⇒ A sin l 0

2mE
As A  0, so l  np

n 2p 2 2
Hence, En  (for n  1, 2, 3, . . .)
2ml 2
Thus, the ground state wave function is
px
c( x )  A sin
l
We evaluate A by normalization
l p
px l l p
1  A2 ° sin 2 dx  A2 ° sin 2 u d u  A 
0
l p p 2

2
Thus, A
l
324 PART SIX ATOMIC AND NUCLEAR PHYSICS

Finally, the probability P of the particle lying in l/2  x  2l/3 is


2l 3
¥l 2l ´ 2 px
PP¦ x µ 
§3 3¶ l ° sin 2
l
dx
l 3

2p3 2p/3
2 1

p ° sin 2 u d u 
p ° (1  cos 2u) d u
p3 p3

2p3
1¨ 1 · 1 ¥ 2p p 1 4p 1 2p ´
 ©u  sin 2u  ¦   sin  sin µ
pª 2 ¹̧ p3 §
p 3 3 2 3 2 3 ¶

1 ¥p 1 3 1 3´
 ¦   µ
p§3 2 2 2 2 ¶

1 3
   0.609
3 2p

l l
6.78 Here  x
2 2
2mE x 2mE x
So, we have c( x )  B cos  A sin
 
Then the boundary condition
¥ l´
c ¦ µ  0
§ 2¶

2mEl 2mEl
gives B cos  A sin 0
2 2
There are two cases:
(1) A  0,

2mE p
l  np 
2 2
p
or 2mE  (2n  1)
l
p 2 2
and En  (2n  1)2
2ml 2
2 px
cne (x )  cos (2n  1) (for n0, 1, 2, 3, . . . )
l l
6.2 WAVE PROPERTIES OF PARTICLES. SCHRODINGER EQUATION 325

This solution is even under x →x.


(2) B0,
2mE
l  np (for n  1, 2, . . .)
2
2
En  (2np )2
2ml 2
2 2npx
cno  sin ( for n  1, 2, . . .)
l l
This solution is odd.

6.79 The wave function is given in Problem 6.77. We see that


l l
2 npx npx
° cn ( x ) cn ( x )dx  l °0
sin
l
sin
l
dx
0

l
1 ¨ px px ·
 °
l 0ª© cos (n  n)
l
 cos (n  n)
l ¹̧
dx

l
1 ¨ sin (n  n) px l sin (n  n) px l ·
 ©  e
l ª (n  n) pl (n  n) px l ¸¹ 0
If n  n, this expression is zero as n and n are integers.

6.80 We have found that


n 2p 2 2
En 
2ml 2
Let N (E )number of states upto E. This number is n. The number of states up to
EdE is N (EdE ) N (E )dN (E ). Then, dN (E )1and
dN (E ) 1

dE $E
(where $Edifference in energies between the nth and (n1)th levels).
(n  1)2  n2 2 2
Now, $E  p 
2ml 2
2n  1 2 2
 p 
2ml 2
326 PART SIX ATOMIC AND NUCLEAR PHYSICS

p 2 2
2n (neglecting 1  n )
2ml 2
p 2 2 2ml 2
  E 2
2ml 2 p 2 2
p 2
 E
l m
dN (E ) l m
Thus, 
dE p 2E
For the given case this gives
dN (E )
 0.816  107 levels per eV
dE

6.81 (a) Here the Schrodinger equation is


2 ¥ e2 e2 ´
  c  Ec
2m § ex 2 ey 2 µ¶
¦

we take the origin at one of the corners of the rectangle where the particle can lie.
Then the wave function must vanish for
x0 or x l1
or y0 or yl2
We look for a solution in the form
cA sin k1x sin k2y
cosines are not permitted by the boundary conditions.
n1p p
Then, k1  and k2  n2
l1 l2
k12  k22 2 p 2 2 ¥ n12 n22 ´
and E   ¦§ l 2  l 2 µ¶
2m 2m 1 2

Here n1, n2 are non-zero integers.


(b) If l1l2l, then
E n12  n22 2
 p
2 ml 2 2
1st level: n1n21→ p29.87
6.2 WAVE PROPERTIES OF PARTICLES. SCHRODINGER EQUATION 327

n1  1, n2  2º 5
2nd level: » m p 2  24.7
n1  2, n2  1 ¼ 2

3rd level: n1n22→ 4p239.5

n1  1‚ n2  3 º
4th level: » m 5p 2  49.3
n1  3‚ n2  1¼

6.82 The wave function for the ground state is


px py
c11 ( x , y )  A sin sin
a b
We find A by normalization
a b
px py ab
1  A2 ° dx ° dy sin 2 sin 2  A2
0 0
a b 4
2
Thus, A
ab
Then, the requisite probability
a 3 b
4 px py
P ° dx ° dy ab sin 2
a
sin 2
b
0 0

a 3
2 px

a ° dx sin 2
a
(on doing the y integral )
0

a 3
1 ¥ 2px ´ 1 ¥ a sin 2p3 ´
 ° d ¦§ 1  cos µ 
a 0
a ¶ a ¦§ 3 2pa µ¶

1 3
   0.196  19.6%
3 4p

6.83 We proceed exactly as in Problem 6.81. The wave function is chosen in the form
c (x, y, z)A sin k1x sin k2 y sin k3z
(The origin is at one corner of the box and the axes of coordinates are along the edg-
es.) The boundary condition is that c0 for x0, x a, y0, ya, z0, za
n1p n2p n3p
This gives k1  , k2  , k3 
a a a
328 PART SIX ATOMIC AND NUCLEAR PHYSICS

The energy eigenvalues are


p 2 2 2
E (n1 , n2 , n3 )  (n1  n22  n32 )
2ma2
(a) The first level is (1, 1, 1). The second has (1, 1, 2), (1, 2, 1) and (2, 1, 1). The third
level is (1, 2, 2) or (2, 1, 2) or (2, 2, 1) with energy as
9p 2 2
E3 
2ma 2
The fourth energy level is (1, 1, 3) or (1, 3, 1) or (3, 1, 1) with energy
11p 2 2
E4 
2ma 2
2 p 2
(b) Thus, $E  E 4  E 3 
ma 2
(c) The fifth level is (2, 2, 2). The sixth level is (1, 2, 3), (1, 3, 2), (2, 1, 3), (2, 3, 1),
(3, 1, 2), (3, 2, 1) with energy
7 2 p 2
E6 
ma 2
and its degree of degeneracy is 6.

6.84 We can safely assume that the discontinuity occurs at the point x  0. Now the
Schrodinger equation is
2 d 2 c
  U ( x ) c( x )  E c( x )
2m dx 2
We integrate this equation around x 0, i.e., from x d1 to x d2 where d1, d2 are
small positive numbers. Then,
d2 d2
2 d2 c
dx  ° ; E  U ( x )c( x )dx =
2m °d dx 2

1 d 1

d2
¥ dc ´ ¥ dc ´ 2m
or ¦§ µ¶  ¦§ µ¶   2 ° ; E  U ( x )= dx c( x )
dx d 2
dx d  d1 1

Since, the potential and the energy E are finite and c (x) is bound by assumption, the
integral on the right exists and approaches 0 as d1, d2 → 0.
¥ dc ´ ¥ dc ´
Thus, ¦§ µ¶  ¦§ µ (as d1 , d2 m 0)
dx d 2
dx ¶ d 1

So (dc/dx) is continuous at x 0 (the point where U (x) has a finite jump discontinuity).
6.2 WAVE PROPERTIES OF PARTICLES. SCHRODINGER EQUATION 329

6.85 (a) Starting from Schrodinger equation in the regions I and II


d 2c 2mE
 2 c0 ( for x in I ) (1)
dx 2 
d 2c 2mE (U 0  E )
 c  0 ( for x in II ) (2)
dx 2 2
where U0 > E > 0, we easily derive the solutions in I and II, as
cI (x)A sin kxB cos kx (3)
ax ax
cII (x)Ce De (4)
2mE 2 2m(U 0  E ) U0
where k2  ,a  I
2 2 II
The boundary condition is that E x= l
c (0)0 (5) x

¥ dc ´
So c and ¦ are continuous at x l, and c must vanish at x ∞.
§ dx µ¶
Then, cIA sin kx
and cIIDeax
so, A sin klDeal a
b
c
k A cos kla Deal
From this, we get
k
tan kl 
a p
kl
kl 2p
or sin kl 
k 2l 2  a2l 2
kl

2mU 0 l 2 2

2 (6)
 kl
2mU 0 l 2
Plotting the left and right sides of this equation we can find the points at which
the straight lines cross the sine curve. The roots of the equation corresponding to
the eigenvalues of energy Ei and found from the intersection points (kl )i, for which
tan (kl )i < 0 (i.e., second and fourth and other even quadrants). It is seen that
bound states do not always exist. For the first bound state to appear (refer to the
line b above), we need
330 PART SIX ATOMIC AND NUCLEAR PHYSICS

p
(kl )1, min 
2
p 2h2
(b) Substituting, we get (l 2 U 0 )1, min 
8m
as the condition for the appearance of the first bound state. The second bound
state will appear when kl is in the fourth quadrant. The magnitude of the slope of
the straight line must then be less than 1(3p2) corresponding to
3p p
kl2, min   (3)
2 2
p
 (2  2  1)
2
For n bound states, it is easy to convince oneself that the slope of the appropriate
straight line (upper or lower) must be less than 2/(2n  1)p corresponding to
p
(kl )n, min  (2n  1)
2

(2n  1)2 p 2 2
Then, (l 2 U 0 )n, min 
8m
Do not forget to note that for large n bothveandvesigns in the Eq. (6) con-
tribute to solutions.

¥ 3 ´ 2
2
6.86 Given U 0 l 2  ¦ pµ
§4 ¶ m
2
¥ 3 ´ 2
and El  ¦ p µ
2
§ 4 ¶ 2m
3
or kl  p
4
It is easy to check that the condition of the bound state is satisfied. Also,

2m mU 0 2 3
al  (U 0  E )l 2  l  p
 2 2 4
Then, from the previous problem
e 3p4
D  Ae al sin kl  A
2
6.2 WAVE PROPERTIES OF PARTICLES. SCHRODINGER EQUATION 331

By normalization
@ 3 p2
¨l e 
3p x ·
I  A2 © ° sin 2 kx dx  ° e 2 l dx ¸
ª0 l
2 ¹
@
¨l l 1 3p y ·
 A2 © ° (1  cos 2 kx ) dx  l ° e 2 dy ¸
ª2 0 0
2 ¹
¨ 1 ¥ sin 2kl ´ 1 l 3p · ¨1 ¥ 13p ´ 1 (13p ) )
·
 A2 © ¦  µ¶   ¸  A l © 2 ¦§ 1  2 µ¶  2 2 ¸
2

ª 2 § 2k 2 2 ¹ ª ¹
1 2
¨1 2 · l¥ 4 ´ 2¥ 4 ´
Al©  2
¸  A2 ¦ 1  µ or A ¦§ 1  µ
ª 2 3p ¹ 2 § 3p ¶ l 3p ¶
The probability of the particle to be located in the region x > l is
@ 1 @
2¥ 4 ´ e 3p2 32p xl
P  ° c dx  ¦ 1 
2
µ ° 2 e dx
l
l § 3p ¶ l

1 @ 3p
¥ 4 ´  y 2 3p
 ¦1 
§ µ
3p ¶ °e 3 p2 e 2 dy  
3p 3p  4
 14.9 %
l

6.87 The Schrodinger equation is


2m
2c 
2
; E  U (r )= c  0
When c depends on r only,

1 d ¥ 2 dc ´
2c  ¦r µ
r 2 dr § dr ¶
x (r ) d c x x
If we put c ,   2
r dr r r
x
and  2c 
r
d 2 x 2m
we get  2 ; E  U (r )= x  0
dr 2 
Thus, the solution is xA sin kr (for r < r0)
2mE
k2  and x0 (for r > r0)
2
332 PART SIX ATOMIC AND NUCLEAR PHYSICS

(For r < r0 we have rejected a term B cos kr as it does not vanish at r0.) Continuity
of the wave function at rr0 requires
kr0np
n 2p 2 2
Hence, En 
2mr02

6.88 (a) The normalized wave functions are obtained from the normalization of
1  °| c |2 dV  °| c |2 4pr 2 dr
r0 r0
npr
 ° A2 4p x 2 dr  4pA2 ° sin 2 dr
0 0
r0
np
r0 r0 np
 4p A 2
np ° sin 2 r dr  4pA2 
np 2
 r0  2pA2
0

1 1 sin (npr r0 )


Hence, A and c
2pr0 2p  r0 r
(b) The radial probability distribution function is
2 npr
Pn (r )  4pr 2 (c )2  sin 2
r0 r0
For the ground state, n1
2 pr
so, P1 (r )  sin 2
r0 r0
By inspection this is maximum for r  r0 2. Thus, rpr  r0 2.
Thus, the probability for the particle to be found in the region r < rpr is clearly 50%
as one can immediately see from a graph of sin2 x.

x(r )
6.89 If we take c
r
the equation for x (r) has the form
2m
x  [E U (r )] x (r )  0
2
which can be written as x  k 2x0 (for 0  r < r0)

and x  a 2x0 (for r0 < r < ∞)


6.2 WAVE PROPERTIES OF PARTICLES. SCHRODINGER EQUATION 333

2mE 2 m (U 0  E )
where k2  and a2 
2 2
The boundary condition is x(0)0 and x, x are continuous at rr0. These are ex-
actly same as in the one-dimensional problem in Problem 6.85.
We therefore omit further details.

6.90 The Schrodinger equation is


d 2c 2m ¥ 1 ´
 2 ¦ E  kx 2 µ c  0
dx 2  § 2 ¶
2
We are given c  Aeax 2
2
Then, c axAeax 2
2 2
c axAeax 2  a2 x 2 Aeax 2
On substituting, we find the following equation must hold

¨ 2 2 2m ¥ 1 2´ ·
©(a x  a)  2 ¦§ E  2 kx µ¶ ¸ c  0
ª ¹
since c x 0, the bracket must vanish. This means that the coefficient of x 2 as well the
term independent of x must vanish. We get
mk 2mE
a2  and a
2 2
Putting k/mv2 gives us
mv hv
a and E
2h 2

6.91 The Schrodinger equation for the problem in Gaussian units is


2m ¨ e2 ·
 2c  © E  c0
2 ª r ¸¹
In MKS units we should read (e2/4pd0) for e2. So, we put
x(r )
c
r
2m ¨ e2 ·
Then x  E  x0 (1)
2 ©ª r ¸¹
334 PART SIX ATOMIC AND NUCLEAR PHYSICS

We are given that xr cAr (1ar) ear


so, x A (12ar) ear aAr (1ar) ear
and x a2Ar (1ar) ear 2aA (12ar) ear  2aAear
Substitution in Eq. (1) gives the condition
2m
a2 (r  ar 2 )  2a(1  2ar )  2a  ( Er  e 2 )  (1  ar )  0
2
Equating the coefficients of r 2, r, and constant term to zero, we get
2me 2
2a  2a  0 (2)
2
2m
aa2  2 Ea  0 (3)

2m
a2  4aa  2 ( E  e 2a )  0 (4)

2 a2
From Eq. (3), either a0 or E  
2m
me 2 2 2 me 4
In the first case a , E  a  2
2 2m 2
This state is the ground state.
me 2 1 me 2
In the second case a  a , a
2 2 2
me 4 1 me 2
and E  and a 
8 2 2 2
This state has n 2(2s).

6.92 We first find A by normalization


@ @
pA 2 3  x 2
1  ° 4pA2 e2r r r 2 dr 
1 r1 ° e x dx  pA2r13
0
2 0

since the integral has the value 2.


1 1
Thus, A2  or A
pr13 r13 p
(a) The most probable distance rpr is that value of r for which
2 4 2 2r r
P (r )  4pr 2 c(r )  r e 1

r13
6.2 WAVE PROPERTIES OF PARTICLES. SCHRODINGER EQUATION 335

is maximum. This requires


¨ 4 2r 2 · 2r r
P a(r )  ©2r  ¸e 0 1

ªr13 r1 ¹
or rr1rpr

(b) The coulomb force is given bye2/r2, then mean value of its modulus is
@
1 2r r e 2
F   ° 4pr 2 e dr 1

0
pr13 r2
@ @
4e 2 2r r 2e 2 x 2e 2

r12 °0
e dr  1 e dx 
0
r13 r12
In MKS units we should read (e2/4pd0) for e2.
@ @
1 2r /r e 2 e2 e2
(c) U  ° 4pr 2 e dr
3   ° xe x dx  

0
pr13 r r1 0 r1
In MKS units it shall read as (e2/4pd0) for e2.

6.93 We find A by normalization as above. We get


1
A
pr13
Then, the electronic charge density is
2 e2r r 1
r e c e  r( r )
pr13
The potential c (r) due to this charge density is
1 r (r) 3
d ra
4p d 0 ° | r  r a |
w( r ) 

@ @
1 r(r ) e 4r 
So at the origin w(0)  ° 4pr 2 dr   °r e2r r dr 
1

4pd0 0
r 4pd0 0 1
3

@
e e
 °
4pd0r1 0
xex dx 
( 4pd0 )r1

6.94 (a) We start from the Schrodinger equation


d 2c 2m
 2 ; E  U ( x )= c  0
dx 2 
336 PART SIX ATOMIC AND NUCLEAR PHYSICS

which we write as cI  k 2cI  0 ( for x  0)


2mE
k2 
2
and cII  a2cII  0 ( for x  0)
2m
a2  (E  U 0 )  0
2
It is convenient to look for solutions in the form
cIeikxReikx (for x < 0)

and cIIAeiaxBeiax (for x > 0)

In region I (x < 0), the amplitude of eikx is written as unity by convention. In region
II we expect only a transmitted wave to the right, then B0. So,
cIIAeiax (for x > 0)
The boundary conditions follow from the continuity of c and dc/dx at x 0.
So, 1RA
ik (1R)iaA
1 R a ka
Then  or R
1 R k ka
The reflection coefficient is the absolute square of R.
2
2 ka
Thus, R 
ka

(b) In this case, E < U0, a2b2 < 0. Then cI is unchanged in form but
cIIAebxBebx
(we must have B0, since, otherwise c (x) will become unbound as x → ∞).
Finally cIIAebx
Inside the barrier, the particle then has a probability density equal to
|cII|2|A|2 e2bx
This decreases to 1/e of its value in
1 
xeff  
2b 2 2m(U 0  E )
6.2 WAVE PROPERTIES OF PARTICLES. SCHRODINGER EQUATION 337

6.95 The formula is


x2
¨ 2 ·
D z exp ©
©ª 
°
x1
2m(V ( x )  E ) dx ¸
¸¹
Here V (x2)V (x1)E and V (x) > E in the region x2 > x > x1.
(a) For the problem, the integral is trivial
¨ 2l ·
D z exp © 2m(U 0  E ) ¸
ª  ¹
(b) We can without loss of generality take x 0 at the point the potential begins to
climb.
«0 ( for x 0)
®® x
Then, U ( x )  ¬U 0 ( for 0 x l )
® l
®­0 ( for x  l )

¨ 2 l ¥ x ´ ·
Then, D z exp © ° 2m ¦ U 0  E µ dx ¸
©ª  lE U 0
§ l ¶ ¸¹

¨ 2 2mU l ·
0
 exp ©
©ª  l ° x  x0 dx ¸
¸¹
( where x0  lE U 0 )
x0

l
¨ 2 2mU 2 ·
 exp © 0
( x  x0 )32 ¸
©  l 3 ¸
ª x 0 ¹
32
¨ 4 2mU ¥ E ´ ·
0
 exp © ¦l  l µ ¸
©ª 3 l § U0 ¶ ¸
¹
¨ 4l ·
 exp © (U 0  E )3/2 2m ¸ .
ª 3 U 0 ¹

6.96 The potential is


¥ x2 ´
U (x )  U 0 ¦1  2 µ
§ l ¶

E x2 E
The turning points are 1 2 or x  l 1 
U0 l U0
338 PART SIX ATOMIC AND NUCLEAR PHYSICS

¨ 4l 1  ( E U 0 )
« ¥ x2 ´ º ·
Then D z exp © ° 2m ¬U 0 ¦ 1  2 µ  E » dx ¸
©ª  0 ­ § l ¶ ¼ ¸¹

¨ 4l 1  ( E U 0 )
E x2 ·
 exp © ° 2mU 0 1   2 dx ¸
©ª  0
U0 l ¸¹

x0
¨ 4l ·
 exp ©
©ª 
2mV0 °
0
x02  x 2 dx ¸
¸¹
( where x0  1  E V0 )

The integral is
x0 p2
p
°
0
x02  x 2 dx  x02 ° cos u d u  4 x
0
2 2
0

¨ pl ¥ E ´·
Thus, D z exp © 2mU 0 ¦§ 1  U µ¶ ¸
ª  0 ¹

¨ pl 2m ·
 exp © (U 0  E ) ¸
ª  U0 ¹

6.3 Properties of Atoms. Spectra

6.97 From the Rydberg formula, we find


R
En 
(n  al )2
We use R  13.6 eV. Then, for n 2state
13.6
5.39  ( for l  0 (S ) state )
(2 a0 )2
Thus, a0 0.41
For P state
13.6
3.54 
(2  a1 )2
a1 y  0.039
6.3 PROPERTIES OF ATOMS. SPECTRA 339

6.98 The energy of the 3P state must be(E0ew), whereE0 is the energy of the 3S state.
R
Then, E0  ew1 
(3  a1 )2

R
So, a1   3 0.885
E0  ew1

6.99 For the first line of the sharp series (3S → 2P) in a Li atom
2p c R R
 
l1 (3  a0 ) 2 (2  a1 )2
For the short wave, cut-off wavelength of the same series
2p c R

l2 (2  a1 )2
From these two equations, we get on subtraction
2p c ( l1  l2 )
3  a0   R
l1 l2

R l1 l2
 ( where $l  l1  l2 )
2pc$l
Thus, in the ground state, the binding energy of the electron is
R R
Eb   2
 5.32 eV
(2  a0 ) 2
¥ R l1 l2 ´
¦§  1µ
2pc $l ¶

6.100 The energy of the 3S state is


R
E(3S )  2.03 eV
(3  0.41)2
The energy of a 2S state is
R
E (2S )  5.39 eV
(2  0.41)2
The energy of a 2P state is
R
E (2P )  3.55 eV
(2  0.04)2
We see that E (2S) < E (2P) < E (3S). The transitions are 3S → 2P and 2P → 2S. Direct
3S → 2S transition is forbidden by selection rules. The wavelengths are determined by
340 PART SIX ATOMIC AND NUCLEAR PHYSICS

2p c
E 2  E1  $E 
l
Substitution gives l0.816 Mm (for 3S → 2P)
and l0.674 Mm (for 2P → 2S)

6.101 The splitting of the Na lines is due to the fine structure splitting of 3P lines. (The 3S
state is nearly single except for possible hyperfine effects.) The splitting of the 3P
level then equals the energy difference
2p c 2p c 2p c ( l2  l1 ) 2p c$l
$E    
l1 l2 l1 l2 l2
Here, $lwavelength difference and laverage wavelength.
Substitution gives $E2.0 meV.

6.102 The sharp series arise from the transitions nS → mP. The S lines are unsplit so the
splitting is entirely due to the P level. The frequency difference between sequent
lines is $E and is the same for all lines of the sharp series and is given by
1 ¥ 2p c 2p c ´ 2pc$l
 
 ¦§ l1 l2 µ¶ l1 l2
Substitution gives 1.645  1014 rad/s.

6.103 We shall ignore hyperfine interactions. The state with principal quantum number
n 3 has orbital angular momentum quantum number l0, 1,2. The levels with
these terms are 3S, 3P, 3D. The total angular momentum is obtained by combining
spin and angular momenta. For a single electron this leads to
1
J (if L  0)
2
1 1
J L and L  ( if L x 0)
2 2
We then get the final designations 3S1/2, 3P1/2, 3P3/2, 3D3/2, 3D5/2.

6.104 The rule is that if JLS then J takes the values |LS| to LS. Thus,
(a) The values are 1, 2, 3, 4, 5.
(b) The values are 0, 1, 2, 3, 4, 5, 6.
1 3 5 7 9
(c) The values are , , , , .
2 2 2 2 2
6.3 PROPERTIES OF ATOMS. SPECTRA 341

6.105 For the state 4P, L1, S3/2 (since 2S14). For the state 5D, L2, S2.
The possible values of J are
«5 3 1
® , , for 4P
J ¬2 2 2
®­ 4, 3, 2, 1, 0 for 5D

The value of the magnitude of angular momentum is  J ( J  1). Substitution gives


the values
1 3  3 3 5  15
For 4P    ,  
2 2 2 2 2 2

5 7  35
and   
2 2 2
for 5D 0,  2,  6 ,  12,  20

6.106 (a) For the Na atoms, the valence electron has principal quantum number n 4, and
the possible values of orbital angular momentum are l0, 1, 2, 3 so Imax3.
The state is 2F, maximum value of J is 7/2. Thus, the state with maximum angular
momentum will be 2F7/2. For this state

7 9  63
M max    
2 2 2
(b) For the atom with electronic configuration 1s2 2p3d, there are two unequivalent
valence electrons. The total orbital angular moments will be 1, 2, 3 so we pick L
3. The total spin angular momentum will be S0,1so we pick up S1. Finally
J will be 2, 3, 4 so we pick up 4. Thus, maximum angular momentum state is 3F4.
For this state
M max   4  5  2 5

6.107 For the F state L3, for the D state L2. Now if the state has spin S, the possible
angular momenta are |LS| to LS. The number of J angular momentum values
is 2S1 if L  S and 2 L1 if L < S. Since the number of states is 5, we must have
S  L2for D state while S  3 and 2S15 imply S2for F state. Thus for the
F state, total spin angular momentum
MS   2  3   6
while for D state
MS   6
342 PART SIX ATOMIC AND NUCLEAR PHYSICS

6.108 Multiplicity is 2S1 so S1. Total angular momentum is

 J ( J  1)
So, J4. Then, L must equal 3, 4, 5 in order that J4 may be included in |LS|to
LS.

6.109 (a) Here J2, L2. Then S0, 1, 2, 3, 4 and the multiplicities (2S1) are 1, 3,
5, 7, 9.
(b) Here J3/2, L1.Then S  52‚ 32‚ 12 and the multiplicities are 6, 4, 2.
(c) Here J1, L3. Then S2, 3, 4 and the multiplicities are 5, 7, 9.

6.110 The total angular momentum is greatest when L, S are both greatest and add to give
J. Now, for a triplet of S, P, D electrons, we have
Maximum spin S  32 corresponding to

3 5  15
MS    
2 2 2
Maximum orbital angular momentum L  3 corresponding to

M L   3  4   12

Maximum total angular momentum J  92 corresponding to



M 99
2
In vector model
L  J S
or if magnitude is squared
L( L  1)2  J ( J  1)2  S (S  1)2  2J  S
J ( J  1)  S ( S  1)  L( L  1)
Thus, cos( J, S) 
2 J ( J  1) S ( S  1)
Substitution gives  (J, S)  31.1°.

6.111 Total angular momentum  6 means J  2. It is given that S  1. This means that
L  1, 2, or 3. From vector model relation
L( L  1)2  6 2  22  22 6 2 cos 73.2
6.3 PROPERTIES OF ATOMS. SPECTRA 343

 5.998 2 z 6 2
Thus, L  2 and the spectral symbol of the state is 3D2.

6.112 In a system containing a p electron and a d electron, S  0, 1 and L  1, 2, 3.


For S  0, we have the terms 1P1, 1D2, 1F3.
For S  1, we have the terms 3P0, 3P1, 3P2, 3D1, 3D2, 3D3, 3F2, 3F3, 3F4.

6.113 The atom has S1  1/2, L1  1, J1  3/2. The electron has s2  1/2, l2  2, so the total
angular momentum quantum number must be
3 5
j2  or
2 2
In L–S coupling we get S  0, 1; L  1, 2, 3 and the terms that can be formed are the same
as written in the problem above. The possible values of angular momentum are consis-
tent with the addition J1  3/2 to j2  3/2 or 5/2. Using j2  3/2 gives us J  0, 1, 2, 3 and
using j2  5/2 gives J  1, 2, 3, 4. All these values are reached in the problem above.

6.114 Selection rules are $S  0


$L  ±1
$J  0, ±1 (0 → 0 is not allowed)
Thus, 2
D3/2 → 2P1/2 is allowed
3
P1 → 2S1/2 is not allowed
3
F3 → 2S1/2 is not allowed
3
F3 → 3P2 is not allowed (since $L  2)
4
F7/2 → 4D5/2 is allowed

6.115 For a 3D state of an Li atom, S  1/2 because there is only one electron and L  2.
The total degeneracy is
g  (2L  1) (2S  1)  5  2  10
The states are 2D3/2 and 2D5/2 and we find that

¥ 3 ´ ¥ 5 ´
g  ¦ 2   1µ  ¦ 2   1µ  4  6
§ 2 ¶ § 2 ¶
344 PART SIX ATOMIC AND NUCLEAR PHYSICS

6.116 The states with greatest possible total angular momentum are
1 3
For a 2P state J   1  , i.e., 2 P3/2
2 2
Its degeneracy is 2  (3/2)  1  4.
For a 3D state J  1  2  3, i.e., 3D3
Its degeneracy is 2  3  1  7.
3 9
For a 4F state J  3  , i.e., 4 F9 / 2
2 2
Its degeneracy is 2  (9/2)  1  10.

6.117 The degeneracy is 2J  1  7. So we must have J  3. From L  3S, we see that S must
be an integer since L is integer and S can be either integer or half integer. If S  0 then
L  0 but this is consistent with J  3. For S  2, L  6 and so J  3. Thus, the state is
3
F3.

6.118 If the order of filling is K, L, M shells, then electrons occupy 4s2, 3d10 followed by
4p3. Hence, electronic configuration of the element will be 1s22s22p63s23p64s23d104p3.
(There must be three 4p electrons.)
The number of electrons is Z  33 and the element is As. (The 3d subshell must be
filled before 4p gets filled.)

6.119 (a) When the partially filled shell contains three p electrons, the total spin
S must equal 1/2 or 3/2. The state S  3/2 has maximum spin and is totally
symmetric under exchange of spin labels. By Pauli’s exclusion principle this
implies that the angular part of the wave function must be totally antisymmetric.
Since the angular part of the wave function of a p electron is vector r, the total
wave function of three p electrons is the totally antisymmetric combination of r1,
r2, and r3. The only possible combination is

x1 x2 x3
r1  ( r2  r3 )  y1 y2 y3
z1 z2 z3

This combination is scalar and hence, L  0. The spectral term of the ground
state is then

4 S32 (since J  32)


6.3 PROPERTIES OF ATOMS. SPECTRA 345

(b) We can think of four P electrons as consisting of a full p shell with two p holes.
The state of maximum spin S is then S  1. By Pauli’s principle the orbital angular
momentum part must be antisymmetric and can only have the form r1  r2 where
r1, r2 are the coordinates of holes. The result is harder to understand if we do not
use the concept of holes. Four p electrons can have S  0, 1, 2 but the S  2 state
is totally symmetric. The corresponding angular wave function must be totally
antisymmetric. But this is impossible; there is no quantity which is antisymmetric
in four vectors. Thus, the maximum allowed S is S  1. We can construct such a
state by coupling the spins of electrons 1 and 2 to S  1 and of electrons 3 and 4
to S  1 and then coupling the resultant spin states to S  1. Such a state is sym-
metric under the exchange of spins of (1, 2) and (3, 4) but antisymmetric under
the simultaneous exchange of (1, 2) and (3, 4). The conjugate angular wave func-
tion must be antisymmetric under the exchange of (1, 2) and under the exchange
of (3, 4) by Pauli’s principle. It must also be antisymmetric under the simultane-
ous exchange of (1, 2) and (3, 4). (This is because two exchanges of electrons are
involved.) The required angular wave function then has the form
(r1  r2)  (r3  r4)
and is a vector and hence, L  1. Thus, using also the fact that the shell is more
than half-filled, we find the spectral term 3P2 where (J  L  S).

6.120 (a) The maximum spin angular momentum of three electrons can be S  3/2. This
state is totally symmetric and hence, the conjugate angular wave function must be
antisymmetric. By Pauli’s exclusion principle the totally antisymmetric state must
have different magnetic quantum numbers. It is easy to see that for d electrons, the
maximum value of the magnetic quantum number for orbital angular momentum
|MLS|  3 (from 2  1  0). Higher values violate Pauli’s principle. Thus, the state
of highest orbital angular momentum consistent with Pauli’s principle is L  3.
The state of the atom is then 4FJ where J  L S by Hund’s rule. Thus, the state is
4
F3/2
The magnitude of the angular momentum is
3 5 
   15
2 2 2
(b) Seven d electrons mean three holes. Then S  3/2 and L  3 as before. But
J  L  S  9/2 by Hund’s rule for more than half-filled shell. Thus, the state is
4
F9/2
Total angular momentum has the magnitude
9 11 3 
   11
2 2 2
346 PART SIX ATOMIC AND NUCLEAR PHYSICS

6.121 (a) For 3F2: The maximum value of spin is S  1 here. This means there are 2 electrons.
L  3 so s and p electrons are ruled out. Thus, the simplest possibility is of d elec-
trons. This is the correct choice for if we were considering f electrons, the maximum
value of L allowed by Pauli’s principle will be L  5 (maximum value of the magni-
tude of magnetic quantum number will be 3  2  5).
Thus, the atom has two d electrons in the unfilled shell.
(b) For 2P3/2: Here L  1, S  1/2 and J  3/2. Since J  L  S, Hund’s rule implies
that shell is more than half-filled. This means one electron less than a filled shell.
On the basis of the whole picture it is easy to see that we have p electrons. Thus,
the atom has five p electrons.
(c) For 6S5/2: Here S  5/2, L  0. We either have five electrons or five holes. The
angular part is antisymmetric. For five d electrons, the maximum value of the
quantum number consistent with Pauli’s exclusion principle is (2  1  0 1
2)  0 so L  0. For f or g electrons L > 0 whether the shell has five electrons
or five holes. Thus, the atom has five d electrons.

6.122 (a) If S  1 is the maximum spin then there must be two electrons. (If there are two
holes then the shell will be more than half-filled.) This means that there are 6
electrons in the completely filled shell so it is a p shell. By Pauli’s principle the
only antisymmetric combination of two electrons has L  1. Also, J  L S as
the shell is less than half-filled. Thus, the term is 3P0.
(b) S  3/2 means either 3 electrons or 3 holes. As the shell is more than half-filled
the former possibility is ruled out. Thus, we must have seven D electrons. Then
as in Problem 6.120, the ground state is 4F9/2.

6.123 With three electrons S  3/2 and the spin part is totally symmetric. It is given that
the basic term has L  3, so this is the state of highest orbital angular momentum.
This is not possible with p electron so we must have d electrons for which L  3 for
three electrons. For the f, g electrons L > 3. Thus, we have three d electrons. Then as
in Problem 6.120, the ground state is 4F3/2.

6.124 We have five d electrons in the only unfilled shell. Then S  5/2. Maximum value of
L consistent with Pauli’s exclusion principle is L  0. Then J  5/2.
So by Lande’s formula
(52) ( 72)  (52) ( 72)  0
g 1 2
2 (52) (72)

Thus, m  g J ( J  1) mB
6.3 PROPERTIES OF ATOMS. SPECTRA 347

35
2 mB  35 mB
2
The ground state is 6S5/2.

6.125 By Boltzmann’s formula


N2 g
 2 e$E kT
N1 g1
Here $E  energy difference between n  1 and n  2 states.
¥ 1´
$E  13.6 ¦ 1  µ  10.22 eV
§ 4¶
Also, g1  2 and g2  8 (counting 2S and 2P states)
N2
Thus,  4e10.22  1.602  10 19 /1.38  1023  3000  2.7  1017
N1
N2 ¥ 1´
Explicitly h  n2 e$E n kT , $E n   R ¦ 1  2 µ
N1 § n ¶
for the nth excited state because the degeneracy of the state with principal quantum
number n is 2n2.

N g g
6.126 We have  P evkT  P e2p clkT
N0 gS gS
Here gP  degeneracy of the 3P state  6, gS  degeneracy of the 3S state  2,
l  wavelength of the 3P → 3S line and 2p c l  energy difference between 3P and
3S levels.
N
Thus,  1.13  104 (on substituting values)
N0

6.127 Let t  mean lifetime of the excited atoms. Then the number of excited atoms will
decrease with time as et/t. In time t the atom travels a distance vt, so t  l/v. Thus,
the number of excited atoms in a beam that has traversed a distance l has decreased
by el/vt
The intensity of the line is proportional to the number of excited atoms in the beam.
Thus,
1
elv t 
h
348 PART SIX ATOMIC AND NUCLEAR PHYSICS

l
or t  1.29  106 s
v ln h

6.128 As a result of lighting by the mercury lamp, a number of atoms are pumped to the
excited state. In equilibrium the number of such atoms is N. Since the mean lifetime
of the atom is t, the number of atoms decaying per unit time is N/t. Since a photon
of energy 2p c l results from each decay, the total radiated power will be
2p c N
P 
l t
Thus,
Pt P tl
N   6.7  109
2p cl 2p 

6.129 The number of excited atoms per unit volume of the gas in 2P state is
g
N  n P e2p clkT
gS
Here gP  degeneracy of the 2P state  6, gS  degeneracy of the 2S state  2 and
l  wavelength of the resonant line 2P → 2S. The rate of decay of these atoms is
N/t per second per unit volume. Since each such atom emits light of wavelength l,
we must have
1 2p c gP 2p c /lkT
n e P
t l gS
1 2p c gP 2p clkT
Thus, t n e
P l gS
 65.4  109 s  65.4 ns

6.130 (a) We know that


P21sp  A21
P21ind  B21uv
p 2c 3  v3 1 A
 A    v /kT21
v
21 
3 2
pc e3  v / kT 1 e 1
P21ind 1
Thus, sp
 v /kT
P21 e 1
For the transition 2P m 1S ,  v  (34) R.
6.3 PROPERTIES OF ATOMS. SPECTRA 349

Substitution gives 7  1034.


(b) The two rates become equal when ev/kT  2
v
or T  1.71  105 K
k ln 12

6.131 Because of the resonant nature of the process, we can ignore non-resonant process-
es. We also ignore spontaneous emission since it does not contribute to the absorp-
tion coefficient and is a small term if the beam is intense enough.
Suppose I is the intensity of the beam at some point. The decrease in the value of this
intensity on passing through the layer of the substance of thickness dx is equal to

¥I´
dI  xI dx  ( N 1 B12  N 2 B21 ) ¦ µ  v dx
§ c¶

Here, N1  number of atoms in lower level. N2  number of atoms in the upper


level per unit volume. B12, B21 are Einstein coefficients and I/c is energy density in
the beam, c is velocity of light.
A factor v arises because each transition result in a loss or gain of energy v.

v ¥ N B ´
Hence, x N 1 B12 ¦ 1  2 21 µ
c § N 1 B12 ¶

But g1B12  g2B21, so

v ¥ g N ´
x N 1 B12 ¦ 1  1 2 µ
c § g2 N 1 ¶

N2 g
By Boltzmann’s factor  2 e vkT
N1 g1
When v kT, we can put N1  N0 as the total number or atoms per unit volume.

Then, x  x 0 (1  e vkT )

where x 0  (  vc ) N 0 B12 is the absorption coefficient for T → 0.

6.132 A short-lived state of mean lifetime t has an uncertainty in energy of $E t which
is transmitted to the photon it emits as natural broadening. Then,
1 l2
$vnat  so, $lnat 
t 2pct
350 PART SIX ATOMIC AND NUCLEAR PHYSICS

The Doppler broadening on the other hand arises from the thermal motion of radiating
atoms. The effect is non-relativistic and the maximum broadening can be written as
$lDopp 2vpr
 2b 
l c
$lDopp 4pvpr t
Thus, 
$lnat l

2RT
Now, vpr   157 ms
M
$lDopp
Therefore, z 1.2  103
$lnat
Note: Our formula is an order of magnitude estimate.

6.133 From Moseley’s law


3
vK a  R ( Z  1)2
4
or 4 1
lK a 
3 R ( Z  1)2
2 2
lK a (Cu) ¥ Z  1´ ¥ 25 ´
Thus  ¦ Fe µ ¦ µ
lK a ( Fe) § Zcu  1¶ § 28 ¶

Substitution gives lK (Cu)  153.9 pm.


a

6.134 (a) From Moseley’s law


3
vK a  R ( Z  s )2
4
2pc 8pc 1
or lK a  
vK a 3 R ( Z  s )2
We shall take s  1. Then, for aluminium (Z  13)
lK a ( Al )  843.2 pm
and for cobalt (Z  27)
lK (Co)  179.6 pm
a

(b) This difference is nearly equal to the energy of the Ka line which by Moseley’s
law is equal to (Z  23 for vanadium)
6.3 PROPERTIES OF ATOMS. SPECTRA 351

3
$E  vK a   13.62  22  22  4.94 keV
4

6.135 We calculate the Z values corresponding to the given wavelengths using Moseley’s
law. (See Problem 6.134.)
Substitution gives Z  23 corresponding to l  250 pm
and Z  27 corresponding to l  179 pm
There are thus three elements in a row between those whose wavelengths of Ka lines
are equal to 250 pm and 179 pm.

6.136 From Moseley’s law


8p c 1
lK a ( Ni ) 
3 R ( Z  1)2
where Z  28 for Ni. Substitution gives lK a ( Ni )  166.5 pm.
Now, the short wavelength cut-off of the continuous spectrum must be more ener-
getic (smaller wavelength) otherwise Ka lines will not emerge. Then, since
$l  lK a  l0  84 pm
we get l0  82.5 pm
This corresponds to a voltage of
2p c
V
e l0
Substitution gives V  15.0 kV.

6.137 Since the short wavelength cut-off of the continuous spectrum is l  0.50 nm, the
voltage applied must be
2p c
V  2.48 kV
e l0
Since this is greater than the excitation potential of the K series of the characteristic
spectrum (which is only 1.56 kV) the latter will be observed.

6.138 Suppose l0  wavelength of the characteristic X-ray line. Then using the formula for
short wavelength limit of continuous radiation, we get
l0  (2p ceV1 ) 1

l0  (2p ceV2 ) n
352 PART SIX ATOMIC AND NUCLEAR PHYSICS

2p c (n  V1 V2 )
Hence, l0 
eV1 n 1
Using also Moseley’s law, we get

8p c n  1 eV1
Z 1 1 2  29
3R l 3  R n  V1 V2

6.139 The difference in frequencies of the K and L absorption edges is equal, according
to the Bohr picture, to the frequency of the Ka line (see figure). Thus, by Moseley’s
formula Continuum
3 L edge
$v  R ( Z  1)2 n4
4 K n4
edge
4 $v
or Z 1  22 n2
3R Ka line
Therefore, the metal is titanium. n1

6.140 From the figure in Problem 6.139, we see that the binding energy Eb of a K electron
is the sum of the energy of a Ka line and the energy corresponding to the L edge of
absorption spectrum. So,
2p c 3
Eb   R ( Z  1)2
lL 4
For vanadium, Z  23 and the energy of Ka line of vanadium has been calculated in
Problem 6.134b. Using
2p c
 0.51 keV for lL  2.4 nm
lL
we get Eb  5.46 keV

6.141 By Moseley’s law


2p c 3
v  E K  E L   R ( Z  1)2
l 4
where EK is the energy of the K electron and EL is the energy of the L electron. Also the
energy of the line corresponding to the short wavelength cut-off of the K series is
2p c 2p c
EK  
l  $l (2pcv)  $l
6.3 PROPERTIES OF ATOMS. SPECTRA 353

 v
 
(1v)  $l2pc 1  v$l2pc
v v
Hence, EL   v 
1  ( v$l2pc ) (2pcv$l)  1
Substitution gives, for titanium (Z  22),
v  6.85  1018 s1
and hence EL  0.47 keV

6.142 The energy of the Ka radiation of Zn is


3
v   R ( Z  1)2
4
where Z atomic number of zinc  30. The binding energy of the K electrons in
iron is obtained from the wavelength of K absorption edge as E K  2p c lK .
Hence, by Einstein equation,
3 2p c
T   R ( Z  1)2 
4 lK
T  1.463 keV (on substituting values)
This corresponds to a velocity of the photoelectrons of v  2.27  106 m/s.

6.143 From the Lande’s formula


J ( J  1)  S ( S  1)  L( L 1)
g 1
2 J ( J  1)
(a) For S state, L  0. This implies J  S. Then, if S p 0, g  2.
(For singlet states, g is not defined if L  0.)
(b) For singlet states, J  L. Then, for S  0, we have
J ( J  1)  L( L  1)
g 1 1
2 J ( J  1)

6.144 (a) For the 6F1/2 state


5 1
S  , L  3, J 
2 2
34  354  12 38  48 2
So, g 1 1 
2  34 6 3
354 PART SIX ATOMIC AND NUCLEAR PHYSICS

(b) For the 4D1/2 state


3 1
S  , L  2, J 
2 2
34  154  6 18  24
So, g 1 1 0
2  34 6
(c) For the 5F2 state
S  2, L  3, J  2
6  6  12
So, g 1 1
26
(d) For the 5P1 state
S  2, L  1, J  1
262 5
So, g 1 
22 2
(e) For the 3P0 state with J  0, L  S, the g factor is indeterminate.

6.145 (a) For the 1F state


S  0, L  3  J
3434
So, g 1 1
23 4
Hence, m  3  4 mB  12 mB
(b) For the 2D3/2 state
1 3
S  , L  2, J 
2 2
154  34  6 18  24 4
So, g 1 1 
2  154 30 5

4 15 2 3
Hence, m mB  15 mB  2 mB
5 4 5 5
4 J ( J  1)  2  6
(c) We have 1
3 2 J ( J  1)
2
or J ( J  1)  J ( J  1)  4
3
6.3 PROPERTIES OF ATOMS. SPECTRA 355

or J ( J  1)  12 ⇒ J  3
4 8
Hence, m 12 mB  mB
3 3

6.146 The expression for the projection of the magnetic moment is


mZ  gmJ mB
where mJ is the projection of J on the z-axis. Maximum value of the mJ is J.
Thus, gJ  4
Since J  2, we get g  2. Now
J ( J  1)  S (S  1)  L( L  1)
21
2 J ( J  1)
6  S (S  1)  6
1 (as L  2)
26
S (S  1)
1
12
Hence, S (S  1)  12 or S  3

Thus, MS   3  4  2 3 

6.147 The angle between the angular momentum vector and the field direction is the least
when the angular momentum projection is maximum, i.e., J .
Thus, J J ( J  1)  cos 30

J 3
or 
J 1 2
Hence, J3
3  4 1 2 2 3 8 4
Then g 1 1 
23 4 24 3
4 8
and m 3  4 mB  mB
3 3

6.148 For a state with n  3, L  2. Thus, the state with maximum angular momentum is
2
D5/2 .
52  72 12  32  2  3
Then, g 1
2  52  72
356 PART SIX ATOMIC AND NUCLEAR PHYSICS

35  3  24 1 6
1 1 
70 5 5

6 5 7 7
Hence, m  mB  3 mB
5 2 2 5

6.149 To get the greatest possible angular momentum we must have S  Smax  1, L  Lmax
 1  2  3 and J  L  S  4.
4  5 1 2  3  4 10 5
Then g 1 1 
2 45 40 4

5 5 5
and m 4  5 mB  mB
4 2

6.150 Since m  0, we must have either J  0 or g  0. But J  0 is incompatible with L 


2 and S  3/2. Hence, g  0. So, we have
J ( J  1)  32  52  2  3
0 1
2 J ( J  1)
15 9
or 3J ( J  1)   6 
4 4
1
Hence, J
2

1 3  3
Thus, M   
2 2 2

6.151 From the relation

M   J  1  2

we find J  1. From the zero value of the magnetic moment, we find g  0


1  2L( L  1)  2  3
or 1 0
21 2
L( L  1)  8
1 0
4
or 12  L (L  1)
Hence, L  3. The state is 5F1.
6.3 PROPERTIES OF ATOMS. SPECTRA 357

6.152 If M is the total angular momentum vector of the atom, then there is a magnetic
moment
g mB M
Mm 

associated with it. Here, g is the Lande’s factor. In a magnetic field of induction B, the
energy
g mB M – B
H 

is associated with it. This interaction term corresponds with a precession of the an-
gular momentum vector because it leads to an equation of motion of the angular
momentum vector of the form
dM ¥ g mB B ´
6 M ¦§ where 6  µ
dt  ¶
Using Gaussian unit expression of mB, we get mB  0.927  1020 erg/G, B  103 G,
  1.054  1027 erg s and for the 2P3/2 state
32  52  12  32  1  2 1 4
g 1 1 
2  32  52 3 3
and 6  1.17  1010 rad/s
The same formula is valid in MKS units also. But mB  0.927  1023 A m2, B  101 T
and   1.054  1034 J s. The answer is the same.

6.153 The force on an atom with magnetic moment M in a magnetic field of induction B
is given by
F  ( M) B
In the present case, the maximum force arises when M is along the axis or close to it.
eB
Then FZ  ( mZ )max
eZ
Here, ( mZ )max  g mB J . The Lande’s factor g is for the 2P1/2.
12  32  12  32  1  2 12 2
Then, g 1 1 
2  12  32 32 3
1 1
and J so, ( mZ )max  mB
2 3
The magnetic field is given by
m0 2 I pr 2
BZ   2
4p (r  Z 2 )3 2
358 PART SIX ATOMIC AND NUCLEAR PHYSICS

eBZ m Z
or   0 6I pr 2 2
eZ 4p (r  Z 2 )5 2

¥ eBZ ´ m 3I p
Thus, ¦§ µ¶  0
eZ Z  r 4p 8 r 2
Thus, the maximum force is
1 m 3p I
F  mB 0
3 4p 8 r 2
Substitution gives (using data in MKS units)
F  4.1 1027 N

6.154 The magnetic field at a distance r from a long current carrying wire is mostly
tangential and given by
m0 I m 2I
Bw   0
2pr 4p r
The force on a magnetic dipole of moment M due to this magnetic field is also
tangential and has a magnitude (M –  ) Bw .This force is non-vanishing only when the
component of M along r is non-zero. Then
e m 2I
F  mr Bw  mr 0 2
er 4p r
Now, the maximum value of mr  ± mB. Thus, the force is
m0 2I
Fmax  mB  2.97  1026 N
4p r 2

6.155 In the homogenous magnetic field, the atom experiences force


eB
F  gJ mB
eZ
Depending on the sign of J, this can be either upward or downward. Suppose the latter
is true. The atom then traverses first along a parabola inside the field and, once outside,
in a straight line. The total distance between extreme lines on the screen will be

eB « 1 ¥ l1 ´ 2 l1 l2 º
d  2g J mB ¬ ¦ µ   » mV
eZ ­2 § v ¶ v v¼
Here mV is the mass of the vanadium atom. (The first term is the displacement within
the field and the second term is the displacement due to the transverse velocity
acquired in the magnetic field.)
6.3 PROPERTIES OF ATOMS. SPECTRA 359

1
Thus, using mV v 2  T l1 l2
2
eB 2T d 
we get 
eZ g mB Jl1 (l1  2l2 )
For vanadium atom in the ground state 4F3/2 
(3  5)4  (3  5)4  3  4
g 1
2  (3  5)4
30  48 18 2
1 1 
30 30 5
Using J  3/2, and other data, on substituting, we get
eB
 1.45  1013 Gcm
eZ
This value differs from the answer given in the book by almost a factor of 109. For
neutral atoms in Stern-Gerlach experiments, the value T  22 MeV is much too large.
A more appropriate value will be T  2 meV, i.e., 109 times smaller. Then one gets
the right answer.

6.156 (a) The term 3P0 does not split in weak magnetic field as it has zero total angular
momentum.
(b) The term 2F5/2 will split into 2  5/2  1  6 sublevels. The shift in each sublevel
is given by
$E  gmB M Z B
where MJ  J ( J 1), . . ., J and g is the Lande’s factor
(5  7)4  (1  3)4  3  4
So, g 1
2  (5  7)4
38  48 6
1 
70 7
(c) For the 4D1/2 term
(1  3)4  (3  5)4  2  3
g 1
2  (1  3)4
3  15  24
1 11 0
6
Thus, the energy differences vanish and the level does not split.
360 PART SIX ATOMIC AND NUCLEAR PHYSICS

6.157 (a) For the 1D2 term


23023
g 1 1
223
and $E  mB MJ B
where MJ  2, 1, 0, 1, 2.
Thus, the splitting is
$E  4mBB
Substitution gives $E  57.9 MeV.
(b) For the 3F4 term
4  51 2 3  4
g 1
2 45
10 5
1 
40 4
5
and $E   mB BMJ
4
where MJ  4 to 4.
5
Thus, $E  mB B  8  10mB B ( 2g J mB )
4
Substitution gives $E  144.7 MeV.

6.158 (a) The term 1P1 splits into 3 lines with M Z 1, 0 in accordance with the formula
$E  g mB BM Z
1 2 0 1 2
where g 1 1
21 2
The term 1S0 does not split in weak magnetic field. Thus, the transitions between
1
P1 and 1S0 will result in 3 lines, i.e., a normal Zeeman triplet.
(b) The term 2D5/2 will split of into 6 lines in accordance with the formula
$E  g mB BM Z
5 3 1
MZ   ,  , 
2 2 2
5 7 1 3  4  2 3 6
and g 1 
257 5
6.3 PROPERTIES OF ATOMS. SPECTRA 361

The term 2P3/2 will split into 4 lines in accordance with the above formula with
3 1 3  51 3  4 1 2 4
MZ   ,  and g 1 
2 2 235 5
It is seen that the Zeeman splitting is anomalous as g factors are different.
(c) The term 3D1 splits into 3 lines (g  5/2). The term 3P0 does not split. Thus, the
Zeeman spectrum is normal.
(d) For the 5I5 term
562367
g 1
256
36  42 1 9
1 1 
60 10 10
For the 5H4 term
452356 26  30 9
g 1 1 
2 45 40 10
We see that the splitting in the 2 lines given by $E   g mB BMZ is the same
though the number of lines is different (11 and 9). It is then easy to see that
only the lines with these energies occur:  v0 ,  v0  g mB B. The Zeeman pattern
is normal.

6.159 For a singlet term S  0, L  J, g  1. Then the total splitting is $E  2 JmBB. Substi-
tution gives J  3 [ $E/(2 mBB)]. The term is 1F3.

6.160 Since the spectral line is caused by transition between singlet terms, the Zeeman
effect will be normal (since g  1 for both terms). The energy difference between
extreme components of the line will be 2 mBB. This must equal

¥ 2p c ´ 2p c$l
$ ¦ 
§ l µ¶ l2
mB B l2
Thus, $l   35 pm
p c

6.161 From the previous problem, if the components are l, l ± $l, then
l 2p c

$l m B B l
362 PART SIX ATOMIC AND NUCLEAR PHYSICS

For resolution
l l
R (of the instrument)
$l dl
2p c 2p c
Thus, R or B
mB B l m B lR
Hence, the minimum magnetic induction is
2p c
Bmin   4 kG  0.4 T
m B lR

6.162 The 3P0 term does not split. The 3D1 term splits into 3 lines corresponding to the shift.
$E  g mB BM Z
with M Z  1, 0. The interval between neighboring components is then given by
$v  g mB B
 $v
Hence, B
g mB
Now, for the 3D1 term
1 21 2 2 3 46 1
g 1 1 
21 2 4 2
Substitution gives B  3 kG  0.3 T

6.163 (a) For the 2P3/2 term


32  52  12  32  1  2 10 4
g 1 1 
2  32  52 30 3
and the energy of the 2P3/2 sublevels will be
4
E ( M Z )  E0  mB BM Z
3
where M Z  32,  12. Thus, between neighboring sublevels
4
dE ( 2 P3/2 )  mB B
3
For the 2P1/2 term
12  32  12 s 32  1  2
g 1
2  12  32
6.3 PROPERTIES OF ATOMS. SPECTRA 363

68 1 2
1 1 
6 3 3
and the separation between the two sublevels into which the 2P1/2 term will split is
2
$E ( 2 P1/2 )  mB B
3
The ratio of the two splittings is 2 : 1.
(b) The interval between neighboring Zeeman sublevels of the 2P3/2 term is (4/3)mBB.
The energy separation between D1 and D2 lines is
2p c
$l
l2
(this is the natural separation of the 2P term).
4 2p c$l
Thus, mB B 
3 l2h
3p c $l
or B
2mB l2h
B  5.46 kG (on substituting values)

6.164 For the 2P3/2 level, g  4/3 (see Problem 6.163) and the energies of sublevels are
4
E a  E0a  mB BM Za
3
where M Za  32,  12 for the four sublevels.
For the 2S1/2 level, g  2 (since L  0) and
E  E0  2m0 BM Z ( where M Z  12)
Permitted transitions must have $M Z  0,  1 .
Thus, only the following transitions occur
3 1 º
m
2 2 ®®
» $v  mB B  3.96  1010 rads
3 1®
 m
2 2 ®¼
1 1 º
m
2 2 ®® 1
» $v   mB B  1.32  1010 rads
1 1 3
 m ®
2 2¼®
364 PART SIX ATOMIC AND NUCLEAR PHYSICS

1 1 º
m
2 2 ®® 5 mB B
» $v    6.6  1010 rads
1 1 ® 3 
 m
2 2 ®¼
These six lines are shown in the figure.

3/2 2 mBB
1/2 2/3 mBB
1/2
3/2 2/3 mBB

1/2 mBB
1/2 mBB

6.165 The difference arises because of different selection rules in the two cases. In direc-
tion 1 the line is emitted perpendicular to the field. The selection rules are then
$M Z  0,  1
In direction 2, the light is emitted along the direction of the field. Then, the selection
rules are
$M Z  1
$M Z  0 is forbidden.
(a) The transition 2P3/2 → 2S1/2 has been considered in the Problem 6.164. In direction
1 we get all the six lines shown in that problem.
1 1 1 1
In direction 2 the line corresponding to m and m  is forbidden. Then
2 2 2 2
we get four lines.
(b) In the transition 3P2 → 3S1, for the 3P2 level,

2  3 1 2 1 2 3
g 1 
223 2
So the energies of the sublevels are
6.3 PROPERTIES OF ATOMS. SPECTRA 365

3
E a( M Za )  E 0a  mB BM Za (where M Za  2,  1, 0)
2
For the 3S1 lines, g  2 and the energies of the sublevels are
E ( M Z )  E0  2mB BM Z ( where M Z   1, 0)
The lines are
$M Z   1 for  2 m 1,  1 m 0 and 0 m 1
$M Z  0 for  1 m 1, 0 m 0 and 1 m 1
$M Z  1 for 2 m 1, 1 m 0 and 0 m 1
All energy differences are unequal because the two g values are unequal. There are
then nine lines if viewed along direction 1 and six lines if viewed along direction 2.

6.166 For the two levels


E0a  E0  g amBa M Za B
E0  E0a  g mB M Z B
and hence, the shift of the component is the value of
mB B
$v 

; g aM Za  gM Z =
subject to the selection rule $M Z  0,  1.
3  4 1 2 2 3 8 4
For the 3D3 state ga  1  1 
23 4 24 3
2  3 1 2 1 2 3
For the 3P2 state g 1 
223 2
mB B 4 3
Thus, $v  M Za  M Z
 3 2
For the different transitions we have the following table

Transition M Za g a  M Z g Transition M Za g a  M Z g
3→2 m BB 1→1 1/6 mBB
2→2 1/3 mBB 1→0 4/3 mBB
2→1 7/6 mBB 0→1 3/2 mBB
1→2 5/3 mBB 0 → 0 0
366 PART SIX ATOMIC AND NUCLEAR PHYSICS

Transition M Za g a  M Z g Transition M Za g a  M Z g
0 → 1 3/2 mBB 2 → 1 7/6 mBB
1 → 0 4/3 mBB 2 → 2 1/3 mBB
1 → 1 1/6 mBB 3 → 2 mBB
1 → 2 5/3 mBB

There are 15 lines in all.


The lines farthest out are 1 → 2 and 1 → 2. The splitting between them is the
total splitting. It is given by
10 mB B
$v 
3 
Substitution gives $v  7.8  1010 rad/s.

6.4 Molecules and Crystals

6.167 In the first excited rotational level, J  1.


2 1 2
So, EJ 1 2  Iv (classically)
2I 2

Thus, v 2
I
m d2 m d2 d2
Now, I  3miri2   m
2 4 2 4 4
where m is the mass of the molecule and ri is the distance of the atom from the axis.

4 2
Thus, v  1.56  1011 rads
md 2

6.168 The axis of rotation passes through the centre of mass (C.M.) of the HCl molecule. The
distance of the two atoms from the C.M. is
m m
dH  Cl d , dCl  H d
mHCl mHCl
Thus, moment of inertia about the axis
4 mHmCl
I mH dH2  mCl dCl2  d2
2 mH  mCl
6.4 MOLECULES AND CRYSTALS 367

The energy difference between two neighboring levels whose quantum numbers are
J and J 1 is
2 J 2
 2J   7.86 meV
2I I
Hence, J  3 and the levels have quantum numbers 2 and 3.

6.169 The angular momentum is


2IE  M
md 2
Now, I  1.9584  1039 g cm 2
4
(where m is the mass of O2 molecule).
So, M  3.68  1027 erg s  3.49 
(This corresponds to J  3.)

2
6.170 From EJ  J ( J  1)
2I
and the selection rule $J  1 or J → J 1 for a pure rotational spectrum, we get
J
v ( J , J  1) 
I
Thus, transition lines are equispaced in frequency $v  I.
In the case of CH

I  1.93  1040 g cm 2
$v
mcmH
Also, I d2
mc  mH
so, d  1.117  108 cm  111.7 pm

6.171 If the vibrational frequency is v0, the excitation energy of the first vibrational level
will be v0 . Thus, if there are J rotational levels contained in the band between the
ground state and the first vibrational excitation, then
J ( J  1)2
 v0 
2I
368 PART SIX ATOMIC AND NUCLEAR PHYSICS

where, as stated in the problem, we have ignored any coupling between the two.
For HF molecule
mHmF
I d 2  1.336  104 g cm 2
mH  mF
2I v0
Then J ( J  1)   197.4

For J  14, J ( J  1)  210. For J  13, J ( J  1)  182. Thus, 13 levels lie between
the ground state and the first vibrational excitation.

6.172 We proceed as above. On calculating, we get


2I v0
z 1118

Now this must equal
2
¥ 1´
J ( J  1) z ¦ J  µ
§ 2¶
Taking the square root we get, J ≈ 33.

6.173 From the formula


2
J ( J  1) E
2I
2IE
we get J ( J  1)  2

2
¥ 1´ 1 2IE
or ¦§ J  µ¶   2
2 4 
1 1 2IE
Hence, J    2
2 4 

So, 1 1 2I
J 1   (E  $E )
2 4 2

1 2I 2I 1 2IE
we find 1  2 E  2 $E   2
4   4 
1 2
1 2I ¨¥ $E ´ ·
  2 E ©¦ 1  µ  1¸
4  ª§ E   8 I ¶
2
¹
6.4 MOLECULES AND CRYSTALS 369

1 2I $E
  2E
4  2( E  2  8 I )

2I $E

 2 E  2  8 I
2

The quantity dN/dE is equal to 1/$E. For large E it is


dN I

dE 22 E
For an iodine molecule
mI d 2
I  7.57  1038 g cm2
2
Thus, for J  10
dN I 1
 
dE  2
J ( J  1)2
22  J ( J  1)
2I
 1.04  104 levels per eV (on substituting values)

6.174 For the first rotational level


2 2
E rot  2 
2I I
and for the first vibrational level
E vib  v
E vib Iv
Thus, j 
E rot 
Here vfrequency of vibration.
m1m2
Now, I  md 2  d2
m1  m2
(a) For H2 molecule, I4.58  10–41 g cm2 and j 36.
(b) For HI molecule, I 4.247  10–40 g cm2 and j 175.
(c) For I2 molecule, I7.57  10–38 g cm2 and j 2872.

6.175 The energy of the molecule in the first rotational level will be 2 I. The ratio of the
number of molecules at the first excited vibrational level to the number of molecules
at the first excited rotational level is
370 PART SIX ATOMIC AND NUCLEAR PHYSICS

evkT
2 J ( J  1 )

(2 J  1)e 2 IkT

1 1
 e v kT  e IkT  e ( v  2 B ) kT
2

3 3
where B  2I .
For the hydrogen molecule
1
I  mH d 2  4.58  1041 g cm 2
2
Substitution gives ratio as 3.04  10–4.

6.176 By definition
e @ bEv
¤E e Ev  kT ¤e
eb v  0
v
E   @
¤e Ev kT
¤e bEv

v 0

@
e
 ln ¤ eb( v  12) v ( where b1kT )
eb v  0
e 1
 ln e(12) bv
eb 1  ebv
e ¨ 1 ·
 © vb  2
ln(1  ebv )
eb ª 2 ¹̧
1 v
 v  v kT
2 e 1
Thus for one gram mole of diatomic gas
2
¥ v ´
R ¦ µ e v kT
e E § kT ¶
CV N 
vib
eT (e v kT  1)2
where RNk is the gas constant.
In the present case
v
 2.7088
kT
and CV  0.56 R
vib
6.4 MOLECULES AND CRYSTALS 371

6.177 In the rotation vibration band, the main transition is due to change in vibrational
quantum number n → n  1. Along with this the rotational quantum number may
change. The “Zeroth line” 0 → 0 is forbidden in this case so the neighboring lines
arise due to1→ 0 or 0 →1in the rotational quantum number.
2
Now, E  En  J ( J  1)
2I
2
Thus, v  v0  ( 2)
2I
2 2
Hence, $v  
I md 2

2
so, d
m$ v
Substitution gives d0.128 nm.

6.178 If lRwavelength of the red satellite and lVwavelength of the violet satellite,
then
2pc 2pc
  v
lR l0
2pc 2pc
and   v
lV l0
Substitution gives lR424.3 nm and lV386.8 nm.
The two formulas can be combined to give
2pc l0
l 
2pcl0  v 1  l0 v2pc

6.179 As in the previous problem


¥ 1 1´
v  pc ¦  µ
§ lV lR ¶

pc ( lR  lV )
  1.368  1014 rads
lR lV
The force constant x is defined by xmv2, where mreduced mass of the S2 mol-
ecule.
Substitution gives x4.96 N/cm.
372 PART SIX ATOMIC AND NUCLEAR PHYSICS

6.180 The violet satellite arises from the transition1→ 0 in the vibrational state of the scat-
tering molecule while the red satellite arises from the transition 0 → 1. The intensities
of these two transitions are in the ratio of initial populations of the two states, i.e.,
in the ratio evkT .
IV
Thus,  ev/kT  0.067
IR
If the temperature is doubled, the ratio increases to 0.259, an increase of 3.9 times.

6.181 (a) In the case of CO2(OCO): The molecule has 9 degrees of freedom, 3 for each
atom. This means that it can have up to 9 frequencies. Three degrees of freedom
correspond to rigid translation, the frequency associated with this is zero as the
potential energy of the system cannot change under rigid translation. The po-
tential energy will not change under rotations about axes passing through the
C-atom and perpendicular to the OCO line. Thus, there can be atmost four
non-zero frequencies. We must look for modes different from the above.

v1

v2

These are the only collinear modes.


A third mode is doubly degenerate:

v3

(vibration in and perpendicular to the plane of paper).


(b) In case of C2H2 (H  C  C  H): There are 4  3  3  27 different vibrations.
There are three collinear modes.

v1

v2

v3
6.4 MOLECULES AND CRYSTALS 373

Two other doubly degenerate frequencies are

v4

v5

together with their counterparts in the plane perpendicular to the paper.

6.182 Suppose the string is stretched along the x-axis from x0 to xl with the end
points fixed. Suppose y (x, t) is the transverse displacement of the element at x at
time t. Then y (x, t) obeys
e2 y e2 y
 v 2
et 2 ex 2
We look for a stationary wave solution of this equation. It is
v
y ( x , t )  A sin x sin(vt  d)
v
where A and d are constants. In this form y0 at x0. The other condition y0
at xl implies
vl
 Np ( where N  0)
v
l
or N v
pv
N is the number of modes of frequency and is  v.
l
Thus, dN  dv
pv

6.183 Let j (x, y, t) be the displacement of the element at (x, y) at time t. Then it obeys the
equation
e2 j 2 ¥
e2 j e2 j ´
 v 
¦§ ex 2 ey 2 µ¶
et 2
where j 0 at x0, xl, y0 and yl.
We look for a solution in the form
jA sin k1x sin k2 y sin (vtd)
374 PART SIX ATOMIC AND NUCLEAR PHYSICS

Then, v2  v 2 (k12  k22 )


np mp
and k1  , k2 
l l
We write this as
2
¥ lv ´
n2  m 2  ¦ µ
§ pv ¶
Here n, m < 0. Each pair (n, m) determines a mode. The total number of modes
whose frequency is  v is the area of the quadrant of a circle of radius lv/pv, i.e.,
2
p ¥ lv ´
N ¦ µ
4 § pv ¶
l2 S
Then, dN  v dv  v dv
2pv 2 2pv 2
where Sl 2 is the area of the membrane.

6.184 For transverse vibration of a three-dimensional continuum in the form of a cube


(say), we have the equation
e2 J
 v 2 2 J (div J  0)
et 2
Here J J(x, y, z, t). We look for solution in the form
J  A sin k1 x , sin k2 y, sin k3 z , sin ( vt  d)
This requires v2  v 2 (k12  k22  k23 )
From the boundary condition that j 0 for x  0, x  l , y  0, y  l , z  0, z  l ,
we get
np np np
k1  1 , k2  2 , k3  3
l l l
where n1, n2, n3 are non-zero positive integers.
2
¥ lv ´
We then get n12  n22  n32  ¦ µ
§ pv ¶

Each triplet (n1, n2, n3 ) determines a possible mode and the number of such modes,
whose frequency  v, is the volume of the all positive octant of a sphere of radius
lv/pv. Considering also the fact that the subsidiary condition div j 0 implies two
independent values of A for each choice of the wave vector (k1, k2, k3).
We find
6.4 MOLECULES AND CRYSTALS 375

3
1 4p ¥ l v ´ V v3
N ( v)  ¦§ µ¶ 2  2 3
8 3 pv 3p v
V v2
Thus, dN  dv
p 2v 3

6.185 To determine the Debye temperature we cut off the high-frequency modes in such a
way so as to get the total number of modes correctly.
(a) In a linear crystal with n0l atoms, the number of modes of transverse vibrations
in any given plane cannot exceed n0l. Then
v0
l l
n0 l 
pv ° d v  pv v 0
0

The cut-off frequency v0 is related to the Debye temperature 1 by


v0  k1

¥ ´
Thus, 1  ¦ µ pn0 v
§ k¶
(b) In a square lattice, the number of modes of transverse oscillations cannot exceed
n0S.
v0
S S
Thus, n0 S 
2pv 2 ° v d v  4pv 2
v02
0

 ¥ ´
or 1
k
v0  ¦ µ
§ k¶  4pn0 v
(c) In a cubic crystal, the maximum number of transverse waves must be 2n0V (two
for each atom). Thus, we have
v0
V V v03
n0V  2 3
pv °v 2 dv 
3p 2v 3
0

¥ ´
Thus, 1  ¦ µ v ( 6p 2n0 )13
§ k¶

6.186 We proceed as in the previous problem. The total number of modes must be 3n0v
(total transverse and one longitudinal per atom). On the other hand the number of
transverse modes per unit frequency interval is given by
V v2
dN >  dv
p 2v>3
376 PART SIX ATOMIC AND NUCLEAR PHYSICS

while the number of longitudinal nodes per unit frequency interval is given by
V v2
dN   dv
2p 2v3
The total number per unit frequency interval is
V v2 ¥ 2 1´
dN  2 ¦ 3
 3 µ dv
2p § v> v ¶

If the high frequency cut-off is at v0  k1, the total number of modes will be
3
V ¥ 2 1 ´ ¥ k1 ´
3n0V  ¦  3µ ¦ µ
6p § v > v  ¶ §  ¶
2 3

Here n0 is the number of iron atoms per unit volume.


1 3
 ¨ 18p 2n0 ·
Thus, 1
k ©ª 2v>3  1v3 ¸¹
NA rN
For iron n0   A
M r M
(where rdensity, Matomic weight of iron, NAAvogadro number).
Thus, n08.389  1022 cm3
Substituting the data, we get 1469.1 K.

6.187 We apply the same formula but assume v\\ ≈ v. Then

1 v (6p 2n0 )13
k
k1
or v
(6p 2n0 )13
rNA
For aluminum n0   6.023 1022 cm3
M
Thus, v3.39 km/s
The tabulated values are v\\6.3 km/s and v3.1 km/s.

6.188 In the Debye approximation, the number of modes per unit frequency interval is
given by
l ¥ k1 ´
dN  dv ¦§ for 0  v  µ
pv  ¶
6.4 MOLECULES AND CRYSTALS 377

k1
But  pn0v

l
Thus, dN  dv ( for 0  v  pn0 v )
pv
The energy per mode is
1 v
E  v  vkT
2 e 1
Then the total interval energy of the chain is
pn0 v pn0 v
l 1 l v
U ° v d v  ° dv
v  kT
pv 0
2 pv 0
e 1
1 T
l l xdx

4p v
(pn0 v )2 
pv 
(kT )2 ° ex  1
0

1 /T
 1 T2 xdx
 ln0 k
k
(pn0 v )  ln0 k
4 (pn0 v k ) ° ex  1
0

We put ln0 kR for1mole of the chain.


2 1 T
®« 1 ¥ T ´ xdx º®
Then U  R1 ¬  ¦ µ
®­ 4 § 1 ¶
° »
e x  1 ®¼
0

Hence the molar heat capacity is by differentiation


1 T
¥ eU ´ ¨ ¥T ´ xdx 1T ·
CV  ¦
§ eT µ¶ u
 R ©2 ¦ µ
§ ¶ °  u T ¸
ª 1 e  1 e  1¹
x
0

when T 1, CV ≈ R.

6.189 (a) If the chain has N atoms, we can assume atom number 0 and N1 are fixed.
Then the displacement of the nth atom has the form
¥ mp ´
un  A ¦ sin  naµ sin vt
§ L ¶

Putting k  mpL, the allowed frequencies then have the form


ka
v  vmax sin
2
378 PART SIX ATOMIC AND NUCLEAR PHYSICS

In our form onlyve k values are allowed. The number of modes in a wave
number range dk is
Ldk L dk
dN   dv
p p dv
a ka
But dv  vmax cos dk
2 2
dv a
Hence,  2
vmax  v2
dk 2
2L dv
So, dN 
pa v  v2
2
max

(b) The total number of modes is


vmax
2L dv 2L p L
N ° pa v v22
 
pa 2 a
0 max

which is the number of atoms in the chain.

6.190 Molar zero point energy is


9
R1
8
The zero point energy per gram of copper is
9 R1
8 M Cu
where MCu is the atomic weight of copper.
Substitution gives 48.6 J/g.

6.191 (a) By Dulong and Petit’s law, the classical heat capacity is 3R24.94 J/mol-K.
C
Thus,  0.6014
C Cl
From the graph we see that this value of C C Cl corresponds to
T
 0.29
1
65
Hence, 1  224 K
0.29
6.4 MOLECULES AND CRYSTALS 379

(b) The molar heat capacity of aluminum  22.4 J/mol-K corresponds to dimension-
less heat capacity
22.4
 0.898
3  8.314
From the graph this corresponds to T/1  0.65. This gives
250
1 z 385 K
0.65
Then 80 K corresponds to T 1  0.208 .
The corresponding value of C C Cl is 0.42. Hence, C10.5 J/mol-K.
(c) We calculate 1 from the data that C C Cl  0.75 at T  125 K. The x-coordinate
corresponding to 0.75 is 0.40.
125
Hence, 1  3125 K
0.4
Now k1  vmax
So, vmax4.09  1013 rad/s

6.192 We use the Eq. (6.4d) of the book


1T
¨1 ¥ T ´ 4 x 3 dx ·
U  9R 1 ©  ¦ µ
©ª 8 § 1 ¶
° ¸
e x  1 ¸¹
0

¨ 1 ®« @ x 3 dx ®º ¥ T ´ 4 ¥ T ´ 4 @ x 3 dx ·
 9R1 ©  ¬° x » ¦ µ  ¦§ µ¶ ° x ¸
©ª 8 ­® 0 e  1 ¼® § 1 ¶ 1 1/T e  1 ¸¹
In the limit T  1, the third term in the bracket is exponentially small along with its
derivatives. Then we can drop the last term, to get
@
9 R 4 x 3 dx
U  constant  T ° x
13 0
e 1
3 @
Thus, ¥ eU ´ ¥ eU ´ ¥T ´ x 3 dx
CV  ¦ µ ¦ µ
§ eT ¶ V § eT ¶ 1
 36R ¦ µ
§ 1¶ °e x 1
0

Now from the appendix 9 of the book


@
x 3 dx p4
° e x  1 15

0
3
12p 4 ¥ T ´
Thus, CV  ¦ µ
5 § 1¶
380 PART SIX ATOMIC AND NUCLEAR PHYSICS

If we call the 3rd term in the bracket above U3, then


4 @
¥T ´ x3
U3  ¦ µ
§ 1¶ ° 2 sin h( x 2)
 ex /2 dx
1 /T

The maximum value of x (2 sin hx 2) is a finiteve quantity C0 for 0  x < ∞.
3

2
Thus, ¥T ´
U 3  2C 0 ¦ µ e1 T
§ 1¶
We see that U3 is exponentially small as T → 0 and so is dU 3 dT .

6.193 At low temperatures C ^ T 3. This is also a test of the “lowness” of the temperature.
We see that
1/ 3
¥ C1 ´ T1 30
¦§ C µ¶  1.4982 y 1.5  
2 T2 20
3
Thus, T law is obeyed and T1, T2 can be regarded low.

6.194 The total zero point energy of1mole of the solid is (9/8)R1. Dividing this by the
number of modes 3N we get the average zero point energy per mode as (38) k1.

6.195 In the Debye model


dNvAv2 (for 0  v  vm )
vm
Avm3
Then, 3N  ° dN v 
3
(total number of modes is 3N)
0

9N
Thus, A
vm3
vm
9N v2  v
We get U
vm3 ° e v /kT  1
dv (ignoring zero point energy)
0
1
x 3 dx ¥ v´
 9N vm ° ¦§ using x  v µ¶
0
e v x /kT  1
m
m

1
x 3 dx
 9R1° ( using 1  vm k )
0
e x 1 /T  1
1 dU ( x ) x3
Thus,  x 1 /T ( for 0  x  1)
9 R1 dx e 1
6.4 MOLECULES AND CRYSTALS 381

1 x3
For T  , this expression is
2 e 2x  1
1 x3
For T  , the expression is
4 e4x  1
On plotting, we get the figures given in the answer sheet.

9.196 The maximum energy of the photon is vm  k1  28.4 meV (if 1330 K).
To get the corresponding value of the maximum momentum we must know the
dispersion relation vv(k). For small k we know vv|k| where v is velocity
of sound in the crystal. For an order of magnitude estimate, we continue to use this
result for high |k|. Then we estimate v from the values of the modulus of elasticity
and density
E
v^
r
Using E 100 GPa, r8.9  103 kg/m3, we get
v 3  103 m/s
vm
Hence, |k|max ^ ^ 1.5  1019 g cms
v

6.197 (a) From the formula


2 m 3/2 1/2
dn  E dE
p 2 3
The maximum value Emax is determined in terms of n by
E max
2 m 32
n
p 2 3 ° E 12 dE
0

2 m 32 2 32
 E max
p 2 3 3
32
32  ¥
2 ´
or E max ¦§ µ (3p 2n )
2m ¶
2
E max  ( 3p 2 n ) 2  3
2m
(b) Mean kinetic energy
382 PART SIX ATOMIC AND NUCLEAR PHYSICS

E max

° E dn
0
E E max

° dn
0

E max

° E 32 dE
0
 E max

° E 12 dE
0

(2 5) E 52
max 3
 E max
(2 3) E 32
max 5

6.198 The fraction is


E max

° E 12 dE
12 E max
h E max

° E 12 dE
0

 1  23 2  0.646  64.6 %

6.199 We calculate the concentration n of electrons in the Na metal from


2
E max  E F  ( 3p 2 n ) 2  3
2m
Using Fermi level, EF3.07 eV, we get n2.447  1022 cm3.
From this we get the number of electrons per Na atom as
n M

r NA
where rdensity of Na, Mmolar weight in grams of Na, NAAvogadro number.
Substituting values, we get 0.963 electrons per Na atom.

6.200 The mean K.E. of electrons in a Fermi gas is (3/5)EF. This must equal (3/2)kT. Thus,
2E F
T
5k
We calculate EF first.
6.4 MOLECULES AND CRYSTALS 383

NA rN
For Cu n  A  8.442  1022 cm3
M r M
Then EF7.01 eV and T3.25  104 K.

6.201 We write the expression for the number of electrons as


V 2m 32 12
dN  E dE
p 2 3
Hence, if $E is the spacing between neighboring levels near the Fermi level, we must
have
V 2m 3/2 1/2
2 E F $E
p 2 3
(Number 2 is used on the L.H.S. is to take care of both spins of electrons.)

2p 2 3
Thus, $E 
Vm 32 E F12

But E F12  (3p 2n )13
2m12
2p 2 2
So, $E 
mV (3p 2n )13
Substituting the data, we get$E1.79  10–22 eV.

2m 32 12
6.202 (a) From dn( E )  E dE
p 2 3
On using E  (12)mv 2 and mv 2 dn( E )  dn(v ), we get

2 m 32 1 12 m3 2
dn(v )  m v mv dv  v dv
p 2 3 2 p 2 3

This holds for 0 < v < vF , where (12)mv F2  E F and dn (v)0 for v > vF .
(b) Mean velocity is
vF

° v dv3
3
0
v  vF
 vF
4
° v dv2

0
384 PART SIX ATOMIC AND NUCLEAR PHYSICS

v  3
Thus, 
vF 4

6.203 Using the formula of the previous section


m3 2
dn(v )  v dv
p 2 3
2p
We put mv  (where lde Broglie wavelength)
l
2p
Then mdv  dl
l2
Taking account of the fact that l decreases when v increases, we write
(2p )3 d l 8p
dn( l) dn(v )   4 dl
p 3 l4 l

6.204 From the kinetic theory of gases we know


2U
p
3V
Here U is the total internal energy of the gas. This result is applicable to Fermi gas
also. Now at T0, UU0N < E >nV < E >.
2
So, p  n E
3
2 3 2
 n  E F  nE F
3 5 5
2
 (3p 2 )2 3 n5 3
5m
Substituting the values, we get p4.92  104 atm.

6.205 From Richardson’s equation


IaT 2 e –A/kT
where A is the work function in eV. When T increases by $T, I increases to (1h)I.
2 A¥ T ´ 2
¥ T  $T ´ kT ¦§ T  $T  1µ¶ ¥ $T ´ kTA  T $T$T
Then 1h  ¦ µ e  ¦1  µ e
§ T ¶ § T ¶
Expanding and neglecting higher powers of $T T , we get
6.4 MOLECULES AND CRYSTALS 385

$T A
h2  2 $T
T kT
¥ hT ´
Thus, A  kT ¦  2µ
§ $T ¶
Substituting we get, A4.48 eV.

6.206
Outside
a
A
Outside
EF
Inside

U0 b
Inside

The potential energy inside the metal is U0 for the electron and is related to the
work function A by
U0EFA (1)
If T is the K.E. of electrons outside the metal, their K.E. inside the metal will be
(EU0). On entering the metal, electron cannot experience any tangential force so
the tangential component of momentum is unchanged. Then
2mT sin a  2m(T  U 0 ) sin b

sin a U
Hence,  1 0  n (by definition of refractive index) (2)
sin b T
In sodium with one free electron per Na atom, n2.54  1022 cm3, EF3.15 eV
and A2.27 eV (from table).
Using these values, we get
U05.42 eV (using Eq. 1)
and n1.02 (using Eq. 2)

6.207 In a pure (intrinsic) semiconductor, the conductivity is related to the temperature


very closely by the following formula
s  s0 e$d2kT
where $d is the energy gap between the top of valence band and the bottom of con-
duction band; it is also the minimum energy required for the formation of electron–
hole pair. The conductivity increases with temperature and we have
386 PART SIX ATOMIC AND NUCLEAR PHYSICS

$d ¥ 1 1´
 
2k ¦§ T1 T2 µ¶
he
$d T2  T1
or ln h 
2k T1T2
2kT1T2
Hence, $d  ln h
T2  T1
Substitution gives$d0.333 eVEmin.

6.208 The photoelectric threshold determines the band gap $d by


2pc
$d 
lth
On the other hand, the temperature coefficient of resistance is defined by
1 dr d d
a  ln r  ln s
r dT dT dT
where r is the resistivity and s is the conductivity. But
$d
ln s  ln s0 
2kT
$d pc
Then, a  2
 2 0.047 K1
2kT kT lth

6.209 At high temperatures (small values of 1/T ) most of the conductivity is intrinsic, i.e.,
it is due to the transition of electrons from the upper levels of the valence band into
the lower levels of conduction bands.
For this we can apply the formula
¥ Eg ´
s  s0 exp ¦
§ 2kT µ¶
Eg
or ln s  ln s0 
2kT
From this we get the band gap
$ ln s
E g   2k
$(1T )
The slope must be calculated at small 1T . Evaluation gives the slope
6.4 MOLECULES AND CRYSTALS 387

$ ln s
  7000 K
$(1T )
Hence, Eg1.21 eV
At low temperatures (high values of 1/T ) the conductance is mostly due to impuri-
ties. If E0 is the ionization energy of donor levels then we can write the approximate
formula (valid at low temperature)

¥ E ´
s a  s0a exp ¦ 0 µ
§ 2kT ¶

$ ln s a
So, E0  2kT
$(1T )
The slope must be calculated at low temperature. Evaluation gives the slope
$ ln s a 1
   1000 K
$(1T ) 3
Then, E0 ~ 0.057 eV

6.210 We write the conductivity of the sample as ssisg, where siintrinsic conduc-
tivity and sg is the photo conductivity. At t0, assuming saturation, we have
1 1 1 1
  sg 0 or sg 0  
r1 r r1 r
At time t after light source is switched off, we have, because of recombination of
electron and holes in the sample
ssisg 0 e–t/t
where tmean lifetime of electrons and holes.
1 1 ¥1 1´
Thus,   ¦  µ et  t
r2 r § r1 r¶

1 1 ¥1 1´
  ¦  µ et  t
r2 r § r1 r¶

1 1

r1 r r2 ( r  r1 )
or e tt  
1 1 r1 ( r  r2 )

r2 r
388 PART SIX ATOMIC AND NUCLEAR PHYSICS

t
Hence, t
«® r2 ( r  r1 ) º®
ln ¬ »
­® r1 ( r  r2 ) ¼®
Substitution gives t9.87 ms ~ 0.01 s.

6.211 We shall ignore minority carriers.


Drifting holes experience a sideways force in the magnetic field and react by setting
up a Hall electric field Ey to counterbalance it. Thus
VH +V B O
vx B  E y 
h
If the concentration of carriers is n, then
jxnevx
h { Ex

l
j j eV j hB
Hence, n x  x H  x
ev x hB eVH
Ex V
Also, using j x  sE x  
r rl
VhB
we get n
e rlVH
Substituting the data (note that in MKS unit B  5.0 kG  0.5 T, r 2.5  10–2
ohm-m), we get
n4.99  1021 m–3
4.99  1015 cm–3
vx V l V l
Also, the mobility is u0   H   H
Ex B V hBV
Substitution gives u00.05 m2/V-s.

6.212 If an electric field Ex is present in a sample containing equal amounts of both elec-
trons and holes, the two drift in opposite directions.
In the presence of a magnetic field BZ  B, they set up Hall voltages in opposite
directions.
The net Hall electric field is given by
E y  (vx  vx ) B
6.4 MOLECULES AND CRYSTALS 389

 (u0  u0 ) E x B
Ey 1
But  vx
Ex h
Ex
1 vx
Hence, |u0  u0|
hB
Substitution gives |u0  u0| 0.2 m 2 V-s.

6.213 When the sample contains unequal number of carri-


ers of both types whose mobilities are different, static
equilibrium (i.e., no transverse movement of either vx vx
electron or holes) is impossible in a magnetic field. Ey B
The transverse electric field acts differently on elec-
Ex
trons and holes. If the Ey that is set up is as shown in
the figure, the net Lorentz force per unit charge (ef-
fective transverse electric field) on electrons is
E y  vx B
and on holes is
E y  vx B
(We are assuming that B  BZ .) There is then a transverse drift of electrons and holes
and the net transverse current must vanish in equilibrium. Using mobility
u0 Ne e ( E y  u0 E x B )  N h eu0 (E y  u0E x B )  0
Ne u02  Nhu02
or Ey  Ex B
Ne u0  Nhu0
On the other hand
j x  ( N e u0  N hu0 ) eE x
Thus, the Hall coefficient is
Ey 1 N e u02  N hu02
RH  
jx B e ( N e u0  N hu0 )2
We see that RH0 when
2
N e ¥ u0 ´ 1 1
 ¦ µ  2 
N h § u0 ¶ h 4
Thus, h2 4
390 PART SIX ATOMIC AND NUCLEAR PHYSICS

6.5 Radioactivity

6.214 (a) The probability of survival (i.e., not decaying) in time t is elt. Hence, the prob-
ability of decay is 1  elt.
(b) The probability that the particle decays in time dt around time t is the differ-
ence
eltel(t  dt)elt [1eeldt]leltdt
Therefore, the mean lifetime is
@ @
1
° t lelt dt l °0
xex dx
1
0
t @
 @

l
° le  lt dt °e x dx
0 0

6.215 We calculate l first


ln 2
l  9.722  103 per day
T12
Hence, fraction decaying in a month1elt0.253.

1mg
6.216 Here N0   6.023  1023  2.51  1016
24 g
ln 2
Also l  0.04621 h1
T12
So the number of b rays emitted in one hour is
N0 (1el)1.13  1015

6.217 If N0 is the number of radionuclei present initially, then


N 1  N 0 (1  et t ) 1

and hN 1  N 0 (1  et t ) 2

where h2.66 and t23t1. Then,


1  et t 2

h
1  e t t 1

or h  het t  1  et t
1 2

Substituting the values


1.66  2.66 e2/t  e6/t
6.5 RADIOACTIVITY 391

Putting e2/tx, we get


x32.66x1.660
(x21) x1.66(x1)0
or (x1) (x2x1.66)0
Now x p1,so x2x1.660

1  1  4  1.66
or x
2
Negative root has to be rejected because x > 0.
Thus, x0.882
2
This gives t  15.9 s
ln 0.882

6.218 If the half-life is T days, then


1
(2)7T 
2.5
7 ln 2.5
Hence, 
T ln 2
7 ln 2
or T  5.30 days
ln 2.5

6.219 The activity is proportional to the number of parent nuclei (assuming that the daugh-
ter is not radioactive). In half of its half-life period, the number of parent nuclei
decreases by a factor
1
(2)12 
2
So activity decreases to 650 2  460 particles per minute.

6.220 If the decay constant (per hour) is l, then the activity after one hour will decrease
by a factor el. Hence,
0.96el
or l1.11  105 s10.0408 h1
The mean lifetime is 24.5 h.
392 PART SIX ATOMIC AND NUCLEAR PHYSICS

1
6.221 Here N0   6.023  1023
238
2.531  1021
The activity is A1.24  104 dis/s
A
Then l  4.90  1018 s 1
N0
ln 2
Hence, the half-life is T  4.49  109 years
l

6.222 In old wooden atoms, the number of C14 nuclei steadily decreases because of radioac-
tive decay. (In live trees biological processes keep replenishing C14 nuclei, thus main-
taining a balance. This balance starts getting disrupted as soon as the tree is felled.)
If T is the half-life of C 14, then
ln 2
t  3
e T 
5
ln 53
Hence, t T
ln 2
 4105 years z 4.1  103 years

6.223 The problem indicates that in the time since the ore was formed, U 238 nuclei that
have remained undecayed is
h
1 h
ln 2
h t 
Thus, e T
1h
¥ 1  h´
ln ¦
§ h µ¶
or t T
ln 2
Substituting T4.5  10 years, h2.8 we get, t1.98  109 years.
9

6.224 The specific activity of Na24 is


NA N ln 2
l  A  3.22  1017 dis  (g-s)
M MT
Here Mmolar weight of Na24  24 g, NA is Avogadro number and T is the half-life
of Na24. Similarly, the specific activity of U 235 is
6.5 RADIOACTIVITY 393

6.023  1023  ln 2
 0.793  105 dis(g-s)
235  108  365  86400

6.225 Let Vvolume of blood in the body of the human being. Then the total activity of
the blood is AV. Assuming all this activity is due to the injected Na24 and taking ac-
count of the decay of this radionuclide, we get
VAAelt
ln 2 1
Now l h ( where t  5 h )
15
A ln 23 2.0  103 ln 23
Thus, V e  e cm 3  5.95 l
Aa (16 60)

6.226 We see that the specific activity of the sample, i.e., activity of M grams of Co58 in the
sample is
1
M  Ma
Here M and M are the masses of Co58 and Co59 in the sample. Now activity of M
grams of Co58 is
M ln 2
 6.023  1023  dis  s
58 71.3  86400
1.168  1015 M
Thus, from the problem
M
1.168  1015  2.2  1012
M  Ma
M
or  1.88  103  100  0.188%
M  Ma

6.227 Suppose N1, N2 are the initial numbers of component nuclei whose decay constants
are l1, l2 (per hour). Then the activity at any instant is
A  l1 N 1el t  l2 N 2el t
1 2

The activity so defined is in units dis/h. We assume that data ln A given is of its natu-
ral logarithm. The daughter nuclei are assumed non-radioactive.
We see from the data that at large t, the change ln A per hour of elapsed time is
constant and equal to 0.07.
Thus, l20.07 h1
394 PART SIX ATOMIC AND NUCLEAR PHYSICS

We can then see that the best-fit data is obtained by


A(t)51.1 e0.66t10.0 e0.07t
[To get the best-fit we calculate A(t) e 0.07t. We see that it reaches the constant value
10.0 at t7, 10, 14, 20 h very nearly. This fixes the second term. The first term is
then obtained by subtracting out the constant value 10.0 from each value of A(t)e 0.07t
in the data for small t.]
Thus, we get l10.66 h1

T1  1.05 h º
» half-lives
T2  9.9 h ¼
Ratio of radioactive nuclei is given by
N 1 51.1 l2
   0.54
N 2 10.0 l1
The answer given in the book is incorrect.

6.228 Production of the nucleus is governed by the equation


dN
 g  lN (1)
dt

decay
supply
We see that N will approach a constant value g/l. This can also be proved directly.
So, we multiply Eq. (1) by e lt and write
dN lt
e  le lt N  ge lt
dt
d
Then, ( Ne lt )  ge lt
dt
g
or Ne lt  e lt  constant
l
At t0 when the production is started, N0.
g
So, 0   constant
l
g
Hence, N  (1  elt )
l
Now the activity is
6.5 RADIOACTIVITY 395

AlNg (1elt)
1
From the problem  1  e lt
2.7
This gives lt0.463
0.463 0.463  T
so, t   9.5 days
l 0.693
T ¥ A´
Algebraically t  ln 1  µ
ln 2 ¦§ g¶

6.229 (a) Suppose N1 and N2 are the number of two radionuclides A1, A2 at time t. Then
dN 1
l1 N 1 (1)
dt
dN 2
 l1 N 1  l2 N 2 (2)
dt
From Eq. (1) N 1  (N 1)0 e l t 1

where (N1)0 is the initial number of nuclides A1 at time t0.


From Eq. (2)
¥ dN 2 ´
¦§  l2 N 2 µ e l t  l1 ( N 1 )0 e(l
2 1  l2 ) t
dt ¶
l1 ( N 1 )0 (l  l2 ) t
or ( N 2 e l t )  constant 
2 e 1

l1  l2
Since N20 at t0,
l1 ( N 1 )0
constant 
l1  l2
l1 ( N 1 )0 l t
Thus, N2  (e  e l t )
1 2

l1  l2
(b) The activity of radionuclide A2 is l2N2. This is maximum when N2 is maximum.
That happens when dN 2 dt  0.
This requires
l2 el t  l1el t
2 1

ln ( l1 l2 )
or t
l1  l2
396 PART SIX ATOMIC AND NUCLEAR PHYSICS

6.230 (a) This case can be obtained from the previous one on putting
l2l1d
where d is very small and letting d → 0 at the end. Then
l1 ( N 1 )0 dt
N2  (e  1)el t  l1tel t ( N 1 )0
1 1

d
On dropping the subscript1as the two values are equal, we get
N 2  ( N 1 )0 ltelt
(b) This is maximum when
dN 2 1
0 or t
dt l

6.231 Here we have the equations


dN 1
l1 N 1
dt
dN 2
 l1 N 1  l2 N 2
dt
dN 3
and  l2 N 2
dt
From Problem 6.229 N 1  ( N 1 )0 e  l t1

l1 ( N 1 )0 l t
and N2  (e  e l t )
2 1

l1  l2
dN 3 ll
Then  1 2 ( N 1 )0 ( e  l t  e  l t )2 1

dt l1  l2
l1 l2 ¥ el t e l t ´
2 1

or N 3  constant   ( N 1 )0
l1  l2 ¦§ l2 l1 µ¶
Since N30 initially,
l1 l2 ¥1 1´
constant  ( N 1 )0 ¦  µ
l1  l1 § 2
l l1¶

l1 l2 ( N 1 )0 ¨1 1 l t ·
So, N3  © l (1  e )  l (1  e )¸
l t 2 1

l1  l2 ª 2 1 ¹
¨ l el t  l2 el t ·
2 1

 ( N 1 )0 ©1  1 ¸
ª l2  l1 ¹
6.5 RADIOACTIVITY 397

6.232 We have the chain


b a
Bi 210 }}}
l
m Po210 }}}
11 l
m Pb206
2

A1 A2 A3
of the previous problem. Initially
103
( N 1 )0   6.023  1023  2.87  1018
210
A month after preparation
N14.54  1016
N22.52  1018
using the results of the previous problem.
Then Abl1N10.725  1011 dis/s
Aal2N21.46  1011 dis/s

6.233 (a) For Ra226 , Z88 and A226.


After5a and 4b (electron) emissions,
A226  20 206
Z8845282
206
The product is 82Pb .
(b) We require $Z102nm
$A32n  4
Here nnumber of a emissions and mnumber of b emissions.
Thus, n8 and m6.

6.234 The momentum of the a-particle is 2M aT . This is also the recoil momentum of
the daughter nucleus in the opposite direction. The recoil velocity of the daughter
nucleus is
2M aT
Md

2 2T
  3.39  105 ms
196 Mp
The energy of the daughter nucleus is ( M a M d )T and this represents a fraction of
total energy given by
398 PART SIX ATOMIC AND NUCLEAR PHYSICS

M a M d Ma 4 1
    0.02
1 M a M d M a  M d 200 50
(Here Md is the mass of the daughter nucleus.)

6.235 The number of nuclei initially present is


103
 6.023  1023  2.87  1018
210
In the mean lifetime of these nuclei, the number decaying is the fraction
1
1  0.632
e
Thus, the energy released is
2.8710180.6325.31.6021013 J1.54 MJ

6.236 We neglect all recoil effects. Then the figure below, gives the energy of the g-ray
quanta emitted, which is 0.80 MeV.
210
Po
a4
.50
Me
V
a Excited state
5.3
0M
eV
g 0.80 MeV } Pb
206

Ground state

6.237 (a) For an a-particle with initial K.E. 7.0 MeV, the initial velocity is
2T
v0 
Ma

2  7  1.602  106

4  1.672  1024
1.83109 cm/s
Thus, R 6.02 cm
(b) Over the whole path, the number of ion pairs is
7  106
 2.06  105
34
6.5 RADIOACTIVITY 399

Over the first half of the path, we write the formula for the mean path as
R ^ E 3/2 where E is the initial energy. Thus, if the energy of the a-particle after
traversing the first half of the path is E1, then,
1
R0 E13/2  R0 E03/2 or E1  22/3 E0
2
Hence, number of ion pairs formed in the first half of the path length is
E 0  E1
 (1  22/3 )  2.06  105  0.76  105
34 eV

6.238 In b decay
Z XA m Z 1 Y A  e  Q
Q (MX  MY  me) c2
[(MXZme)  (MYZmeme)]c2
(Mp  Md) c2
Since Mp, Md are the masses of the parent and daughter atoms, the binding energy
of the electrons is ignored.
In K capture,
eK  Z X A m Z 1YA  Q
Q(MX  MY) c2mec2
 ( M X c 2  Zme c 2 )  ( MY c 2  ( Z  1)me c 2 )
c2 (Mp  Md)
In b+ decay
ZX
A
→ Z  1Y
A
e+Q
Then Q(MX  MY  me) c2
[MXZme ]c2  [MY(Z  1) me ]c2  2mec2
(Mp  Md  2me )c2

6.239 The reaction is


Be10 → B10e ⎯ne
For maximum K.E. of electrons we can assume the energy of neutrino (⎯ne) to be
zero. The atomic masses are
Be1010.016711 amu
400 PART SIX ATOMIC AND NUCLEAR PHYSICS

B1010.016114 amu
So the K.E. of electrons is (see previous problem)
0.597106 amu × c20.56 MeV
The momentum of electrons with this K.E. is 0.941 MeVc and the recoil energy of
the daughter nucleus is
(0.941)2 (0.941)2
 MeV  47.2 eV
2  M d c 2 2  10  938

6.240 The masses are


Na2424  0.00903 amu and Mg2424  0.01496 amu
The reaction is
Na24 m Mg24  e ⎯ne
The maximum K.E. of electrons is
0.00593931 MeV5.52 MeV
Average K.E. according to the problem is then 5.52/31.84 MeV.
The initial number of Na24 is
103  6.023  1023
 2.51  1019
24
The fraction decaying in a day is
1  (2)24/150.67
Hence the heat produced in a day is
0.672.5110191.841.6021013 J4.95 MJ

6.241 We assume that the parent nucleus is at rest. Then since the daughter nucleus does
not recoil, we have
ppn
i.e., momenta of positron and neutrino (n) are equal and opposite. On the other
hand, total energy released (including rest energy) is

c 2 p 2  me2c 4  cp  Q
(Here we have used the fact that energy of the neutrino is c|pn|cp.)

Now, Q[(Mass of C11 nucleus)  (Mass of B11 nucleus)]c2


6.5 RADIOACTIVITY 401

[Mass of C11 atom  Mass of B11 atom  me ]c2


(0.00213931  0.511) MeV1.47 MeV
Then c2p2(0.511)2(1.47  cp)2
(1.47)2  2.94 cpc2p2
Thus, cp0.646 MeVenergy of neutrino
Also K.E. of electron is
1.47  0.646  0.5110.313 MeV

6.242 The K.E. of the positron is maximum when the energy of neutrino is zero. Since the
recoil energy of the nucleus is quite small, it can be calculated by successive ap-
proximation. The reaction is
N 13 → C 13ene
The maximum energy available to the positron (including its rest energy) is
c2 (Mass of N13 nucleus  Mass of C13 nucleus)
c2 (Mass of N13 atom  Mass of C13 atom  me)
0.00239 c2  mec2
(0.00239931  0.511) MeV
1.71 MeV
The momentum corresponding to this energy is 1.636 MeV/c. The recoil energy of
the nucleus is then
p2 (1.636)2
E   111 eV  0.111 keV
2 M 2  13  931
(on using Mc213931 MeV).

6.243 The process is


eK  Be7 m Li 7  n
The energy available in the process is
Qc2 (Mass of Be7 atom  Mass of Li7 atom)
0.00092931 MeV0.86 MeV
The momentum of a K electron is negligible. So in the rest frame of the Be7 atom,
most of the energy is taken by the neutrino whose momentum is very nearly
402 PART SIX ATOMIC AND NUCLEAR PHYSICS

0.86 MeV/c. The momentum of the recoiling nucleus is equal and opposite. The velocity
of recoil is
0.86 MeVc 0.86
c  3.96  106 cms
M Li 7  931

6.244 In internal conversion, the total energy is used to knock out K electrons. The K.E. of these
electrons is the difference of energy available andbinding energy of K electrons, i.e.,
(87  26)61 keV
The total energy including rest mass of electrons is 0.5110.0610.572 MeV.
The momentum corresponding to this total energy is

(0.572)2  (0.511)2 c  0.257 MeVc


c2 p 0.257
The velocity is then c  0.449 c
E 0.572

6.245 With recoil neglected, the g-particle will have 129 keV energy. To a first approxima-
tion, its momentum will be 129 keV/c and the energy of recoil will be
(0.129)2
MeV  4.18  108 MeV
2  191  931
In the next approximation we therefore write
Eg ≈ 0.129  8.2108 MeV
dEg
So,  3.63  107
Eg

6.246 For maximum (resonant) absorption, the absorbing nucleus must be moving with
enough speed to cancel the momentum of the oncoming photon and have just the
right amount of energy (d129 keV) available for transition to the excited state.

v
Eg dEg
Recoil energy
dEg

Since dEg z d2 2 Mc 2 and momentum of photon is dc, these conditions can be satis-
fied if the velocity of the nucleus is
d d
c  218 ms  0.218 kms
Mc Mc 2
6.6 NUCLEAR REACTIONS 403

6.247 Because of the gravitational shift, the frequency of the g-ray at the location of the
absorber is increased by
dv gh
 2
v c
For this to be compensated by the Doppler shift (assuming that resonant absorption
is possible in the absence of gravitational field), we must have
gh v gh
 or v  0.65 Mms
c2 c c

6.248 The natural lifetime is



'  4.7  1010 eV
t
Thus, the condition dEg  ' implies
gh ' 
 
c 2 d td
c 2
or h  4.64 m
tdg
(Here h is height of the place, not Planck’s constant.)

6.6 Nuclear Reactions

6.249 Initial momentum of the a-particle is 2mTa i (where i is a unit vector in the incident
direction). Final momenta are, respectively, pa and pLi. Conservation of momentum
reads
pa  pLi  2mTa i

Squaring, pa2  pLi2 – 2pa pLi cos 1  2mTa (1)

(where 1 is the angle between pa and pLi ).


Also, by energy conservation, we have
pa2 p2
 Li  Ta
2m 2 M
(Here m and M are, respectively, the masses of a-particle and Li6.)
m 2
So, pa2  pLi  2mTa (2)
M
404 PART SIX ATOMIC AND NUCLEAR PHYSICS

Subtracting Eq. (2) from Eq. (1) we see that


¨¥ m´ ·
pLi ©¦ 1  µ pLi  2 pa cos 1¸  0
ª § M ¶ ¹
Thus, if pLi x 0
1¥ m´
pa   ¦ 1  µ pLi sec 1
2§ M¶
Since pa, pLi are both positive numbers (being magnitudes of vectors), we must
have
1  cos 1 < 0 (if m < M )
This being understood, we write
2
pLi2 ¨ M ¥ m´ ·
©1 ¦§ 1 µ¶ sec2 1¸ Ta
2M ª 4m M ¹
Hence the recoil energy of the Li nucleus is
pLi2 Ta

2 M 1  ( M  m )2 sec2 1
4mM
As we pointed out above, 1 ≠ 60°. If we take 1120°, we get recoil energy of
Li6 MeV.

6.250 (a) In a head-on collision


2mT  pd  pn
pd2 p2
T  n
2 M 2m
(where pd and pn are the momenta of deuteron and neutron after the collision).
Squaring, we get
pd2  pn2  2 pd pn  2mT
m 2
pn2  pd  2mT
M
Since pd p 0 in head-on collisions
1¥ m´
pn   ¦ 1  µ pd
2§ M¶
Going back to energy conservation
6.6 NUCLEAR REACTIONS 405

2
pd2 ¨ M ¥ m´ ·
©1  ¦ 1  µ¶ ¸  T
2M ª 4m § M ¹
pd2 4mM
So,  T
2 M (m  M )2
This is the energy lost by neutron. So the fraction of energy lost is
4mM 8
h 
(m  M ) 2 9
(b) In this case neutron is scattered by 90°. Then we have from the figure
pd  pn j  2mT i
Then, by energy conservation
pn2  2mT p2
 n T
2M 2m j pd
2
p ¥
n m´ ¥ m´
or ¦ 1  µ¶  T ¦§ 1  µ¶
2m § M M i
2mT i
2
p
n M m
or  T pn j
2m M  m
The energy lost by neutron is then
pn2 2m
T  T
2m M  m
or fraction of energy lost is
2m 2
h 
M m 3

6.251 From conservation of momentum


2MT i  pd  p p

or p 2p  2MT  pd2  2 2 MT pd cos u


pp
From energy conservation
pd2 p 2p
T  u
2 M 2m 2MT i
(where Mmass of deuteron, mmass of proton).
pd
m 2
So, p 2p  2mT  pd
M
406 PART SIX ATOMIC AND NUCLEAR PHYSICS

¥ m´
Hence, pd2 ¦ 1  µ  2 2 MT pd cos u  2( M  m )T  0
§ M¶
¥ m´
For real roots 4(2MT )cos2 u  4  2(M  m )T ¦ 1  µ  0
§ M¶
¥ m2 ´
or cos2 u  ¦ 1  2 µ
§ M ¶
m2
Hence, sin 2 u 
M2
m
i.e., u  sin1
M
For deuteron–proton scattering, umax30°.

6.252 This problem has a misprint. Actually the radius R of a nucleus is given by
R  1.3 3 A fm
(where 1 fm1015 m).
Then the number of nucleons per unit volume is
A 3
  (1.3)3  1039 cm3  1.09  1038 cm3
4p3 R 3 4p
The corresponding mass density is
(1.091038mass of a nucleon) cm31.821011 kg/cm3

6.253 (a) The particle x must carry two nucleons and a unit of positive charge. The reac-
tion is
B10 (d, a) Be8
(b) The particle x must contain a proton in addition to the constituents of O17. Thus
the reaction is
O17(d, n) F 18
(c) The particle x must carry nucleon number 4 and two units of ve charge. Thus
the particle must be xa and the reaction is
Na23 (p, a) Ne20
(d) The particle x must carry mass number 37 and have one unit less of positive
charge. Thus xCl37 and the reaction is
Cl37 (p, n) Ar37
6.6 NUCLEAR REACTIONS 407

6.254 From the basic formula


EbZmH(A  Z) mn  M
We define $HmH  1 amu
$nmn  1 amu
$M  A amu
Then clearly EbZ$H(A  Z) $n  $

6.255 The mass number of the given nucleus must be


27
8
(32)3
Thus, the nucleus is Be8. Then the binding energy is
Eb40.0086740.000783  0.00531
0.06069 amu56.5 MeV
(on using 1 amu931 MeV).

6.256 (a) Total binding energy of the O16 nucleus is


Eb80.0086780.007830.00509
0.13709 amu127.6 MeV
So, B.E. per nucleon is 7.98 MeV.
(b) B.E. of neutron in B11 nucleus is
B.E. of B11  B.E. of B10
(Since on removing a neutron from B11 we get B10.)
 $n  $ B  $ B
11 10

 0.00867  0.00930  0.01294


0.01231 amu11.46 MeV
B.E. of an a-particle in B11 is
B.E. of B11  B.E. of Li7  B.E. of a
(Since on removing an a from B11 we get Li7.)
 $ B  $ Li  $ a
11 7

0.009300.016010.00260
408 PART SIX ATOMIC AND NUCLEAR PHYSICS

0.00931 amu8.67 MeV


(c) This energy is
[B.E. of O164 (B.E. of a particles)]
  $ O  4$ a
16

40.002600.00509
0.01549 amu14.42 MeV

6.257 B.E. of a neutron in B11  B.E. of a proton in B11


 ($n  $ B  $ Be )  ($ p  $ B  $ Be )
11 10 11 10

 $n  $ p  $ B  $ Be
10 10

 0.00867  0.00783 0.01294  0.01354


0.00024 amu0.223 MeV
The difference in binding energy is essentially due to the coulomb repulsion between
the proton and the residual nucleus Be10 which together constitute B11.

6.258 Required energy is simply the difference in total binding energies, i.e.,
B.E. of Ne20  2 (B.E. of He4)  B.E. of C12
20dNe  8dHe  12dC
(Here d is binding energy per unit nucleon.)
Substitution gives 11.88 MeV.

6.259 (a) We have for Li 8


41.3 MeV0.044361 amu3$H5$n  $
Hence, $30.0078350.00867  0.094360.022480 amu
Hence, the mass of Li8 is 8.0225 amu.
(b) We have for C10
106.0460.4 MeV 0.06488 amu 6$H4$n  $
Hence, $60.0078340.00867  0.064880.01678 amu
Hence, the mass of C10 is 10.01678 amu.
6.6 NUCLEAR REACTIONS 409

6.260 Suppose M1, M2, M3, M4 are the rest masses of the nuclei A1, A2, A3 and A4 participat-
ing in the reaction, respectively, then
A1A2 → A3A4Q
Here Q is the energy released. Then, by conservation of energy
Qc2 (M1M2  M3  M4)
Now, M1c2c2 (Z1mH(A1  Z) mn)  E1, etc
and Z1Z2Z3Z4 (conservation of charge)
A1A2A3A4 (conservation of heavy particles)
Hence, Q(E3E4)  (E1E2)

6.261 (a) The energy liberated in the fission of 1 kg of U235 is


1000
 6.023  1023  200 MeV  8.21  1010 kJ
235
The mass of coal with equivalent calorific value is
8.21  1010
 2.74  106 kg
30000
(b) The required mass is
30  109  4.1  103 235
  1.49 kg
200  1.602  10  6.023  10
13 23 1000

6.262 The reaction is (in effect)


H2H2 → He4Q
Then Q  2$ H  $ He
2 4

0.02820  0.00260
0.02560 amu23.8 MeV
Hence, the energy released in 1 g of He4 is
6.023  1023
 23.8  16.02  1013 J  5.75  108 kJ
4
This energy can be derived from
5.75  108
 1.9  104 kg of coal
30000
410 PART SIX ATOMIC AND NUCLEAR PHYSICS

6.263 The energy released in the reaction Li6H2 → 2He4 is


$ Li6  $ H2  2$ He4
0.015130.01410  20.00260 amu
0.02403 amu22.37 MeV
(This result for change in B.E. is correct because the contribution of $n and $H can-
cels out by conservation law for protons and neutrons.)
Energy per nucleon is then
22.37
 2.796 MeVnucleon
8
This should be compared with the value 200/2350.85 MeV/nucleon.

6.264 The energy of reaction Li7p → 2He4 is


2B.E. of He4  B.E. of Li7
8da  7dLi
8 × 7.06 75.6017.3 MeV

6.265 The reaction is N14 (a, p) O17.


It is given that (in the laboratory frame where N14 is at rest)
Ta4.0 MeV
j
The momentum of incident a particle is u
i
2maTa i  2hamOTa i
O17
The momentum of outgoing proton is

2m pT p (cos u i  sin u j)  2h p mOTP (cos u i  sin u j)

where h p  m p mO , ha  ma mO , and m0 is the mass of O17.


Then, the momentum of O17 is


2ham0Ta  2h p m0T p cos u i  2mOh pT p sin u j

By energy conservation (conservation of energy including rest mass energy and ki-
netic energy)
M N c 2  M ac 2  Ta
14
6.6 NUCLEAR REACTIONS 411

 M p c 2  Tp  M O c 2 17

2
 ¨©  haTa  h pT p cos u
ª

 h pT p sin 2 u  h pT p sin 2 u ·
¹
Hence, by definition, energy of reaction is
Q  M N c 2  M ac 2  M p c 2  M O c 2
14 17

 T p  haTa  h pT p  2 h p haT pTa  cos u  Ta

 (1  h p )Tp  Ta (1  ha )  2 h p haTa Tp cos u


1.19 MeV (on substituting values)

6.266 (a) The reaction is Li7 (p, n) Be7 and the energy of the reaction is
Q  ( M Be  M Li ) c 2  ( M p  M n ) c 2
7 7

 ($ Li  $ Be ) c 2  $ p  $n
7 7

[0.016010.00783  0.01693  0.00867] amuc2


1.64 MeV
(b) The reaction is Be9 (n, g) Be10. Mass of g is taken zero. The energy of the reac-
tion is
Q  ( M Be  M n  M Be )c 2
9 10

 ( $ Be  $n  $ Be )c 2
9 10

(0.012190.00867  0.01354) amuc2


6.81 MeV
(c) The reaction is Li7 (a, n)B10. The energy of the reaction is
Q  ($ Li7  $ a  $n  $ B10 ) c 2
(0.016010.00260  0.00867  0.01294) amuc2
2.79 MeV
(d) The reaction is O16 (d, a) N14. The energy of the reaction is
Q  ($ O16  $ d  $ a  $ N14 ) c 2

(0.005090.01410  0.00260  0.00307) amuc2


3.11 MeV
412 PART SIX ATOMIC AND NUCLEAR PHYSICS

6.267 The reaction is B10 (n, a) Li7. The energy of the reaction is
Q  ( $ B  $n  $ a  $ Li ) c 2
10 7

(0.012940.00867  0.00260  0.01601) amuc2


2.79 MeV
Since the incident neutron is very slow and B10 is stationary, the final total momen-
tum must also be zero. So the reaction products must emerge in opposite directions.
If their speeds are, respectively, va and vLi, then
4va7vLi
1
and ( 4va2  7vLi2 )  1.672  1024  2.79  1.602  106
2
1 ¥ 4´
So,  4va2 ¦ 1  µ  2.70  1018 cm 2 s2
2 § 7¶
or va9.27106 m/s
Then, vLi5.3106 m/s

6.268 The Q value of the reaction Li7 (p, n) Be7 was calculated in Problem 6.266a. It is
1.64 MeV. We have by conservation of momentum and energy
pppBe
(since initial Li and final neutron are both at rest).
p 2p 2
pBe
  1.64
2m p 2mBe

p 2p ¥ mp ´
Then, ¦ 1  1.64
2m p § mBe µ¶

p 2p 7
Hence, Tp    1.64 MeV  1.91 MeV
2m p 6

6.269 It is understood that Be9 is initially at rest. The moment of the outgoing neutron is
2mnTn j. The momentum of C12 is

2maT i  2mnTn j
Then by energy conservation
6.6 NUCLEAR REACTIONS 413

n 2mnTn j
2maT  2mnTn
T  Q  Tn  2maT i
2mC
(Here mC is the mass of C12.) a Be9
mC (T  Q )  maT (m  ma )T  mC Q C12
Thus, Tn   C
mC  mn mC  mn
Q  (1  ma mC )T
  8.52 MeV
1  mn mC

6.270 The Q value of the reaction Li7 (p, a) He4 is


Q  ($ Li  $ H  2$ He ) c 2
7 4

(0.016010.00783  0.00520) amuc2


0.01864 amuc217.35 MeV
Since the direction of each He4 nuclei is symmetrical, their momenta must also be
equal. Let T be the K.E. of each He4. Then
u
p p  2 2mHeT cos He4
2
(Here pp is the momentum of proton.)
p 2p p u/2
Also  Q  2T  T p  Q
2m p Li7
p 2p sec2 u2
Hence, Tp  Q  2
8mHe He4

mp u
 Tp sec2
2mHe 2

u mp Tp
Hence, cos 
2 2mHe T p  Q
Substitution gives u170.53°
1
Also T  (T p  Q )  9.18 MeV
2

6.271 Energy required is minimum when the reaction products all move in the direction of
the incident particle with the same velocity (so that the combination is at rest in the
C.M. frame). We then have
414 PART SIX ATOMIC AND NUCLEAR PHYSICS

2mTth  (m  M ) v
(Total mass is constant in the non-relativistic limit.)
1 mTth
Tth |Q| (m  M ) v 2 
2 mM
M
or Tth |Q|
mM
¥ m´
Hence, Tth  ¦ 1  µ |Q|
§ M¶

6.272 The result of the previous problem applies and we find that energy required to split
a deuteron is
¥ Mp ´
T  ¦1  Eb  3.3 MeV
§ M d µ¶

6.273 Since the reaction Li7 (p, n) Be7 ( Q1.65 MeV) is initiated, the incident proton
energy must be
¥ Mp ´
T  ¦1   1.65  1.89 MeV
§ M Li µ¶
Since the reaction Be9 (p, n) B9 (Q1.85 MeV) is not initiated,
¥ Mp ´
T  ¦1   1.85  2.06 MeV
§ M Be µ¶
Thus, 1.89 MeV  T p  2.06 MeV.

¥ m ´
6.274 We have 4.0  ¦ 1  n µ |Q|
§ MB ¶11

11
or Q   4  3.67 MeV
12

6.275 The Q value of the reaction Li7 (p, n) Be7 was calculated in Problem 6.266a. It is 1.64
MeV. Hence, the threshold K.E. of protons for initiating this reaction is
¥ mp ´
Tth  ¦ 1  |Q|
§ mLi µ¶
6.6 NUCLEAR REACTIONS 415

8
  1.64  1.87 MeV
7
For the reaction Li7 (p, d) Li6, we find
Q  ( $ Li  $ p  $ d  $ Li ) c 2
7 6

(0.016010.00783  0.01410  0.01513) amuc2


5.02 MeV
The threshold proton energy for initiating this reaction is
¥ mp ´
Tth  ¦ 1  |Q| 5.73 MeV
§ mLi µ¶7

6.276 The Q value of the reaction Li7 (a, n) B10 was calculated in Problem 6.266 c. It is
Q2.79 MeV. Then the threshold energy of a-particle is
¥ m ´
Tth  ¦ 1  a µ |Q|
§ mLi ¶

¥ 4´
 ¦ 1  µ 2.79  4.38 MeV
§ 7¶
The velocity of B10 in this case is simply the velocity of centre of mass
2maTth 1 2Tth
v 
ma  mLi 1  mLi ma ma
This is because both B10 and n are at rest in the C.M. frame at threshold. Substituting
the values of various quantities, we get
v5.27106 m/s

6.277 The momentum of incident neutron 2mnT i that of a particle is 2maTa j and of
Be9 is  2maTa j  2mnT i.
a j
By conservation of energy
maTa  mnT
T  Ta  |Q|
M n
9 i
(Here M is the mass of Be .) C12
¨ ¥ mn ´ · M
Thus, Ta  ©T ¦§ 1  µ |Q|¸ Be9
ª M¶ ¹ M  ma
416 PART SIX ATOMIC AND NUCLEAR PHYSICS

¥ m ´
Using Tth  ¦ 1  n µ |Q|
§ M¶

M ¨¥ mn ´ Tth ·
we get Ta 
M ma ©¦§ 1  M µ¶ T  1  m M  ¸
ª n ¹
(Here M is the mass of C12 nucleus.)
1 ¨ MM a ·
or Ta  ©( M  mn )T  M a  m Tth ¸
M  ma ª n ¹
 2 .21 MeV (on substituting values)

6.278 The formula of Problem 6.271 does not apply here because the photon is always
relativistic. At threshold, the energy of the photon Eg implies a momentum Eg /c. The
velocity of centre of mass with respect to the rest frame of initial H2 is
Eg
(mn  m p )c
Since both n and p are at rest in C.M. frame at threshold, we write
Eg2
Eg   Eb
2(mn  m p )c 2
by conservation of energy. Since the first term is a small correction, we have
Eb2
Eg z Eb 
2(mn  m p )c 2

Thus, dE Eb

Eb 2(mn  m p )c 2

2. 2
  5.9  104  100 0.06%
2  2  938

6.279 The reaction is


pd → He3
Excitation energy of He3 is just the energy available in centre of mass. The velocity
of the centre of mass is
2m pTp 1 2Tp

m p  md 3 mp
6.6 NUCLEAR REACTIONS 417

In the C.M. frame, the K.E. available is (md ≈ 2 mp). So,


2 2
1 ¥ 2 2Tp ´ 1 ¥ 1 2T ´ 2T
mp ¦ µ  2mp ¦3 m µ  3
2 § 3 m p ¶ 2 § p ¶

The total energy available is then


2T
Q
3

Now, Q  c 2 ($n  $ d  $ He )3

c2(0.007830.01410  0.01603) amu


5.49 MeV
Thus, Eexc6.49 MeV

6.280 The reaction is


dC13 → N15* → nN14
Maxima of yields determine the energy levels of N15*. As in the previous problem, the
excitation energy is
EexcQEK
where EKavailable K.E. This is found as in the previous problem. The velocity of
the centre of mass is
2md Ti md 2Ti

md  mC md  mC md
2 2
1 ¥ md ´ 2Ti 1 ¥ md ´ 2Ti mC
So, E K  md ¦§ 1  m  m µ¶ m  2 mC ¦§ m  m µ¶ m  m  m Ti
2 d C d d C d d C

Q is the Q value for the ground state of N15. We have


Q  c 2  ($ d  $C  $ N )
13 15

c2(0.014100.00335  0.00011) amu


16.14 MeV
The excitation energies then are 16.66 MeV, 16.92 MeV, 17.49 MeV and 17.70 MeV.
418 PART SIX ATOMIC AND NUCLEAR PHYSICS

6.281 We have the relation


1
 epsd
h
Here 1h  attenuation factor, nnumber of Cd nuclei per unit volume, seffec-
tive cross-section and dthickness of the plate.
rNA
Now, n
M
(Here rdensity, Mmolar weight of Cd, NAAvogadro number.)
M
Thus, s ln h  2.53 kb
rNA d

1
6.282 Here  en s 2 2  n1s1 d
h
where 1 refers to oxygen and 2 to deutrium nuclei.
Using n22n, n1nconcentration of oxygen nuclei in heavy water, we get
1
 e2s 2  s1 nd
h
Now, using the data for heavy water
1.1  6.023  1023
n  3.313  1022 cm3
20
Thus, substituting the values
I0
h  20.4 
I

6.283 In traversing a distance d, the fraction which is either scattered or absorbed is clearly
1  en ( ss  sa )d
by the usual definition of the attenuation factor. Of this, the fraction scattered is (by
definition of scattering and absorption cross-section)
ss
w  [1  en ( s  s )d ]
S a

s s  sa
r  NA
In iron n  8.39  1022 cm 3
M
Substitution gives w0.352.
6.6 NUCLEAR REACTIONS 419

6.284 (a) Assuming of course, that each reaction produces a radionuclide of the same type,
the decay constant l of the radionuclide is k/w. Hence,
ln 2 w
T  ln 2
l k
(b) Number of bombarding particles is It/e (where echarge on proton).
Then the number of Be7 produced is
It
w
e
If ldecay constant of Be7  ( ln 2)T , then the activity is
It ln 2
A w
e T
eAT
Hence, w  1.98  103
It ln 2

6.285 (a) Suppose N0number of Au197 nuclei in the foil. Then the number of Au197 nuclei
transformed in time t is N0  J  s  t. For this to equal hN0, we must have
th/( J s)
323 years (on substituting values)

(b) Rate of formation of the Au198 nuclei is N0  J  s per second and rate of decay is
ln, where n is the number of Au198 at any instant.
dn
Thus,  N 0  J  s  ln
dt
The maximum number of Au198 nuclei is clearly
N 0  J  s N 0  J  s T
nmax  
l ln 2

because if n is smaller, dndt  0 and n will increase further and if n is large,


dndt 0 and n will decrease. (Actually nmax is approached steadily as t → ∞.)
Substituting N03.0571019 and other values, we get nmax1.011013.

6.286 Rate of formation of the radionuclide is n  J  σ per unit area per second and rate of
decay is λN.
420 PART SIX ATOMIC AND NUCLEAR PHYSICS

dN
Thus,  n  J  s  lN
dt
¥ dN ´
Then ¦§  l N µ e lt  n  J  s – e lt
dt ¶
d
or ( Ne lt )  n  J  s  e lt
dt
n  J  s lt
Hence, Ne lt  constant  e
l
The number of radionuclide at t0 when the process starts is zero. So,
n J s
constant 
l
n J s
Then, N (1  elt )
l

6.287 We apply the formula of the previous problem except that here we have Nnumber
of radionuclide and nnumber of host nuclei originally.
0.2
Here n  6.023  1023  6.115  1020
197
Then, after time t
n  J  s T
N (1  e t ln 2 T )
ln 2
(where Thalf-life of the radionuclide).
After the source of neutrons is cut-off, the activity after time t will be
n  J  s T
A (1  et ln 2T ) et ln 2 T  ln 2 T  n  J  s (1  et ln 2  T ) et ln 2 T
ln 2
Thus, J  Ae t ln 2T ns (1  et ln 2  T )  5.92  109 particlecm 2 -s

6⋅288 Number of nuclei in the first generationNumber of nuclei initiallyN0.


N0 in the second generationN0multiplication factor (k)N0 ⋅ k.
2
N0 in the third generationN0  k  kN0k .
n1
N0 in the nth generationN0k .
Substitution gives 1.25105 neutrons.
6.7 ELEMENTARY PARTICLES 421

6.289 Number of fissions per unit time is clearly P/E. Hence, number of neutrons produced
per unit time is nP E . Substitution gives 7.801018 neutrons/s.

n 1
6.290 (a) This number is k where nnumber of generations in time tt/t
Substitution gives number of neutrons generated per unit time as 388.
¥T ´
n 1 ¦  1µ¶ ln k
(b) We write k  e§ t
T 1
or 1
t ln k
¥ 1 ´
T  t ¦1   10.15 s
§ ln k µ¶

6.7 Elementary Particles

6.291 The formula is


T  c 2 p 2  m02c 4  m0c 2
GeV
Thus, T  5.3 MeV for p  0.10  5.3  103 GeV
c
GeV
T  0.433 GeV for p  1.0
c
GeV
T  9.106 GeV for p  10
c
(Here we have used m0c20.938 GeV.)

6.292 Energy of pions is (1h) m0c2, so


m0 c 2
(1  h)m0 c 2 
1  b2
1
Hence, 1h
1  b2

h(2  h)
or b
1h
Here b  vc of pion. Hence, time dilation factor is 1h and the distance traversed
by the pion in its lifetime will be
422 PART SIX ATOMIC AND NUCLEAR PHYSICS

c bt 0
 ct 0 h(2  h)
1  b2
 15.0 m (on substituting values)

Note: The factor 1 1  b2 can be looked at as a time dilation effect in the laboratory
frame or as length contraction factor brought to the other side in the proper frame of
the pion.

6.293 From the previous problem

l  c t 0 h(h  2)

( where h  T (mp c 2 ) and mp is the rest mass of pions).

l
Substitution gives t0 
c h(2  h)

lmp c

T (T  2mp c 2 )

 26.3 ns
( where we have used h  100139.6  0.716 ).

T
6.294 Here h 1
mc 2
so the lifetime of the pion in the laboratory frame is
h(1h) t02t0
The law of radioactive decay implies that the flux will decrease by the factor
J
 ett  elv t  el /ct 0 h( 2  h )
J0

¥ mcl ´
 exp   0.221
¦§ t T (T  2mc 2 ) µ¶
0

6.295 Energy momentum conservation implies


0pmpn
6.7 ELEMENTARY PARTICLES 423

or mpc2EmEn or mpc2  EnEm


But Enc |pn|c |pm|
Thus, mp2 c 4  2mp c 2  c|p m| c 2 pm2  E m2  c 2 pm2  mm2 c 4
mp2  mm2
Hence, c|p m| c2
2mp
¨ (mp2  mm2 )2 ·
So, Tm  c 2 pm2  mm2 c 4  mm c 2  ©  m 2 c2  m c2
m¸ m
©ª 4mp2 ¸¹
mp2  mm2 (mp  mm2 )
 c 2  mm c 2  c2
2mp 2mp
Substituting mpc2139.6 MeV and mmc2105.7 MeV, we get
Tm4.12 MeV
mp2  mm2
Also, En  c 2  29.8 MeV
2mp

6.296 We have 0pnpp (1)


2
mΣc EnEp
or (m3 c 2  En )2  Ep2
or m32 c 4  2m3c 2 En  Ep2  En2  c 4mp2  c 4mn2
(because Eq. (1) implies Ep2  En2  mp2 c 4  mp2 c 4 )
m32  mn2  mp2 2
Hence, En  c
2m3

¥ m 2  mn2  mp2 ´ (m  mn )2  mp2 2


and Tn  ¦ 3  mn µ c 2  3 c
§ 2m3 ¶ 2m3
Substitution gives Tn19.55 MeV.

6.297 The reaction is


m→ ene⎯nm
The neutrinos are massless. The positron will carry the largest momentum if both
neutrinos ( ne and⎯nm) move in the same direction in the rest frame of the nuon. Then,
the final product is effectively a two-body system and we get from Problem 6.295
424 PART SIX ATOMIC AND NUCLEAR PHYSICS

(mm  me )2
( Te )max  c2
2mm
Substitution gives ( Te )max  52.35 MeV.

6.298 By conservation of energy and momentum


Mc2EpEp
0pppp
Then mp2 c 4  Ep2  pp2 c 2
 ( Mc 2  E p )2  c 2 p2p
 M 2c 4  2Mc 2 E p  m 2p c 4
This is a quadratic equation in M, in the form
Ep
M2  2 M  m 2p  mp2  0
c2
or using Epmpc2T and solving
2
¥ Ep ´ E 2p
¦§ M  2 µ¶  4  m 2p  mp2
c c
Ep E 2p
Hence, M   m 2p  mp2
c2 c4
taking the positive sign. Thus,

T T ¥ T´
M  mp   mp2  2 ¦ 2m p  2 µ
c 2 c § c ¶
Substitution gives M  1115.4 MeVc 2
From the table of masses we identify the particle as a L-particle.

6.299 From the figure, by conservation of energy


mp2 c 4  c 2 pp2  cpn  mm2 c 4  pp2 c 2  c 2 pn2

or ( mp2 c 4  c 2 pp2  cpn )2  mm2 c 4  c 2 pp2  c 2 pn2

or mp2 c 4  2cpn mp2 c 4  c 2 pp2  mm2 c 4


6.7 ELEMENTARY PARTICLES 425

Hence, the energy of the neutrino is


pn pn j
mp2 c 4  mm2 c 4
En  cpn 
2(mp c 2  T )

(on writing mp2 c 4  c 2 pp2  mp c 2  T )


pp ppi
pm pn j ppi
Substitution gives En21.93 MeV.

6.300 By energy conservation

pp pp j

p3 p3i
pn pp j p3i

m32 c 4  c 2 p32  mp2 c 4  c 2 pp2  mn2c 4  c 2 pp2  c 2 p32

or ( m32 c 4  c 2 p32  mp2 c 4  c 2 pp2 )2  mn2 c 4  c 2 pp2  c 2 p32

or m32 c 4  c 2 p32  mp2 c 4  c 2 pp2  2 mp2 c 4  c 2 p32 mp2 c 2  c 2 pp2

 mn2c 2  c 2 pp2  c 2 p32


or using the K.E. of Σ and p
¥ T ´¥ T ´
mn2  m32  mp2  2 ¦ m3  32 µ ¦ mp  p2 µ
§ c ¶§ c ¶

¥ T ´¥ T ´
and mn  m32  mp2  2 ¦ m3  32 µ ¦ mp  p2 µ  0.949 GeVc 2
§ c ¶§ c ¶

6.301 Here, by conservation of momentum


Ep u
pp  2   cos
2c 2
u
or cpp  Ep cos
2
426 PART SIX ATOMIC AND NUCLEAR PHYSICS

u
Thus, Ep2 cos2  En2  mp2 c 4 Ep/2
2
mp c 2
or Ep 
sin u2 u/2
¥ u ´ Ep u/2
and Tp  mp c 2 ¦ cosec  1µ
§ 2 ¶
Ep/2
Substitution gives
Tpmpc2135 MeV (for u60°)
mp c 2  Tp mp c 2 u
Also Eg   cosec
2 2 2
mpc2 (since u60°)

6.302 With particle masses standing for the names of the particles, the reaction is
mM→ m1m2. . .
On R.H.S. let the energy momenta be (E1, cp1), (E2, cp2), etc. On the L.H.S. the en-
ergy momentum of the particle m is (E, cp) and that of the other particle is (Mc2, O),
where, of course, the usual relations, E 2  c2p2m2c4, etc., hold.
From the conservation of energy momentum, we see that
(EMc2)2  c2p2(ΣEi )2  (Σcpi )2
The L.H.S. is
m2c4M2c42Mc2E
We evaluate the R.H.S. in the frame where 3pi0 (C.M. frame of the decay product).
Then, R.H.S.(ΣEi)2  (Σmic2)2 because all energies are ve. Therefore, we have
the result
( 3mi )2  m 2  M 2 2
E c
2M
or since Emc2T, we see that T  Tth, where
( 3mi )2  (m  M )2 2
Tth  c
2M

6.303 By momentum conservation


E  me c 2 u
E 2  me2c 4  2 cos
2 2
6.7 ELEMENTARY PARTICLES 427

u E  me c 2 T E mec2
or cos  
2 E  me c 2 T  2me c 2 2
u/2
Substitution gives u98.8°.
E u/2
E mec2
2

6.304 The formula of Problem 6.302 gives


( 3mi )2  M 2 2
E th  c
2M
when the projectile is a photon.
(a) For ge → e  e  e
9me2  me2 2
E th  c  4me c 2  2.04 MeV
2me
(b) For gp → pp  p
( M p  2mp )2  Mp2
E th  c2
2 Mp

4mp M p  4mp2
 c2
2 Mp

¥ m2 ´
 2 mp  p c 2
¦§ 2 M p µ¶
320.8 MeV

6.305 (a) For pp → ppp⎯p


16m 2p  4m 2p
T  Tth  c 2  6mp c 2  5.63 GeV
2mp
(b) For pp → ppp0
(2m p  mp )2  4m 2p
0
T  Tth  c2
2m p

¥ m2 ´ 0
 2mp  p c 2  0.280 GeV
¦§ 2m p µ¶
428 PART SIX ATOMIC AND NUCLEAR PHYSICS

(mK  m3 )2  (mp  m p )2
6.306 (a) Here Tth  c2
2m p
0.904 GeV (on substituting values)
( mK  m, )2  (mp  m p )2
 0 0
(b) Here Tth  c2
2m p
0.77 GeV (on substituting values)

6.307 From the Gell-Mann Nishijima formula


Y
Q  TZ 
2
1 Y
We get 0  or Y  1
2 2
Also, YBS ⇒ S2
0
Thus, the particle is  (neutral Xi).

6.308 (1) The process n → pe  ne cannot occur as there are two more leptons (e, ne )
on the right compared to on the left.
(2) The process p → me  e is forbidden because this corresponds to a
change of lepton number by 1, (0 on the left, 1 on the right).
(3) The process p → m  nm is forbidden because m, nm being both leptons,
$L2 here.
The following processes (4), (5), (6) are allowed (except that one must distinguish
between muon neutrinos and electron neutrinos). The correct names would be
(4) pe → nne
(5) m m e  ne  nm
(6) K  m m  nm

6.309 (1) p  p m 3  K 
0 0 1 1
So $S0 and process is allowed.
(2) p  p m 3  K 
0 0 1 1
6.7 ELEMENTARY PARTICLES 429

So $S2 and process is forbidden.


(3) p  p m K   K   n
0 0 → 1 1 0
So $S0 and process is allowed.
(4) np → Λ0Σ
0 0 1 1
So $S2 and process is forbidden.
(5) p  n →   KK
0 0 → 2 1 1
So $S2 and process is forbidden.
(6) Kp → Ω  KK 0
1 0 3 1 1
So $S0 and process is allowed.

6.310 (1) Σ → Λ0pis forbidden by energy conservation. The mass difference will be
MeV
M 3  M ,  82
 0 mp
c2
[The process (1) → (2)(3) will be allowed only if m1 > m2m3.]
(2) pp → KKis disallowed by conservation of baryon number.
(3) Kn → ΩKK 0 is forbidden by conservation of charge.
(4) np → Σ  Λ0 is forbidden by strangeness conservation.
(5) p → me  e is forbidden by conservation of muon number (or lepton
number).
(6) m m e  ne  nm is forbidden by the separate conservation of muon number
as well as lepton number.

You might also like